Location via proxy:   [ UP ]  
[Report a bug]   [Manage cookies]                

Provas da IMO

matemática

Problem Books in Mathematics Series Editor Peter Winkler Department of Mathematics Dartmouth College Hanover, NH 03755 USA peter.winkler@dartmouth.edu For other titles published in the series, go to www.springer.com/series/714 ˇ Djukic´ • Vladimir Janković Dusan Ivan Matic´ • Nikola Petrović The IMO Compendium A Collection of Problems Suggested for The International Mathematical Olympiads: 1959-2009 Second Edition Dušan Djukić Department of Mathematics University of Toronto Toronto Ontario, M5S3G3 Canada dusan.djukic@utoronto.ca Vladimir Janković Department of Mathematics University of Belgrade Studentski Trg 16 11000 Belgrade Serbia vjankovic@matf.bg.ac.rs Ivan Matić Department of Mathematics Duke University Durham, North Carolina 27708 USA matic@math.duke.edu Nikola Petrović Science Department Texas A&M University PO Box 23874 Doha Qatar nikola.petrovic@qatar.tamu.edu ISSN 0941-3502 ISBN 978-1-4419-9853-8 e-ISBN 978-1-4419-9854-5 DOI 10.1007/978-1-4419-9854-5 Springer New York Dordrecht Heidelberg London Library of Congress Control Number: 2011926996 © Springer Science+Business Media, LLC 2011 All rights reserved. This work may not be translated or copied in whole or in part without the written permission of the publisher (Springer Science+Business Media, LLC, 233 Spring Street, New York, NY 10013, USA), except for brief excerpts in connection with reviews or scholarly analysis. Use in connection with any form of information storage and retrieval, electronic adaptation, computer software, or by similar or dissimilar methodology now known or hereafter developed is forbidden. The use in this publication of trade names, trademarks, service marks, and similar terms, even if they are not identified as such, is not to be taken as an expression of opinion as to whether or not they are subject to proprietary rights. Printed on acid-free paper Springer is part of Springer Science+Business Media (www.springer.com) Preface The International Mathematical Olympiad (IMO) exists for more than 50 years and has already created a very rich legacy and firmly established itself as the most prestigious mathematical competition in which a high-school student could aspire to participate. Apart from the opportunity to tackle interesting and very challenging mathematical problems, the IMO represents a great opportunity for high-school students to see how they measure up against students from the rest of the world. Perhaps even more importantly, it is an opportunity to make friends and socialize with students who have similar interests, possibly even to become acquainted with their future colleagues on this first leg of their journey into the world of professional and scientific mathematics. Above all, however pleasing or disappointing the final score may be, preparing for an IMO and participating in one is an adventure that will undoubtedly linger in one’s memory for the rest of one’s life. It is to the high-school-aged aspiring mathematician and IMO participant that we devote this entire book. The goal of this book is to include all problems ever shortlisted for the IMOs in a single volume. Up to this point, only scattered manuscripts traded among different teams have been available, and a number of manuscripts were lost for many years or unavailable to many. In this book, all manuscripts have been collected into a single compendium of mathematics problems of the kind that usually appear on the IMOs. Therefore, we believe that this book will be the definitive and authoritative source for high-school students preparing for the IMO, and we suspect that it will be of particular benefit in countries lacking adequate preparation literature. A high-school student could spend an enjoyable year going through the numerous problems and novel ideas presented in the solutions and emerge ready to tackle even the most difficult problems on an IMO. In addition, the skill acquired in the process of successfully attacking difficult mathematics problems will prove to be invaluable in a serious and prosperous career in mathematics. However, we must caution our aspiring IMO participant on the use of this book. Any book of problems, no matter how large, quickly depletes itself if the reader merely glances at a problem and then five minutes later, having determined that the problem seems unsolvable, glances at the solution. VI Preface The authors therefore propose the following plan for working through the book. Each problem is to be attempted at least half an hour before the reader looks at the solution. The reader is strongly encouraged to keep trying to solve the problem without looking at the solution as long as he or she is coming up with fresh ideas and possibilities for solving the problem. Only after all venues seem to have been exhausted is the reader to look at the solution, and then only in order to study it in close detail, carefully noting any previously unseen ideas or methods used. To condense the subject matter of this already very large book, most solutions have been streamlined, omitting obvious derivations and algebraic manipulations. Thus, reading the solutions requires a certain mathematical maturity, and in any case, the solutions, especially in geometry, are intended to be followed through with pencil and paper, the reader filling in all the omitted details. We highly recommend that the reader mark such unsolved problems and return to them in a few months to see whether they can be solved this time without looking at the solutions. We believe this to be the most efficient and systematic way (as with any book of problems) to raise one’s level of skill and mathematical maturity. We now leave our reader with final words of encouragement to persist in this journey even when the difficulties seem insurmountable and a sincere wish to the reader for all mathematical success one can hope to aspire to. Belgrade, November 2010 Dušan Djukić Vladimir Janković Ivan Matić Nikola Petrović Over the previous years we have created the website: www.imomath.com. There you can find the most current information regarding the book, the list of detected errors with corrections, and the results from the previous olympiads. This site also contains problems from other competitions and olympiads, and a collection of training materials for students preparing for competitions. We are aware that this book may still contain errors. If you find any, please notify us at imomath@gmail.com. If you have any questions, comments, or suggestions regarding both our book and our website, please do not hesitate to write to us at the above email address. We would be more than happy to hear from you. Preface VII Acknowledgements The making of this book would have never been possible without the help of numerous individuals, whom we wish to thank. First and foremost, obtaining manuscripts containing suggestions for IMOs was vital in order for us to provide the most complete listing of problems possible. We obtained manuscripts for many of the years from the former and current IMO team leaders of Yugoslavia / Serbia, who carefully preserved these valuable papers throughout the years. Special thanks are due to Prof. Vladimir Mićić, for some of the oldest manuscripts, and to Prof. Zoran Kadelburg. We also thank Prof. Djordje Dugošija and Prof. Pavle Mladenović. In collecting shortlisted and longlisted problems we were also assisted by Prof. Ioan Tomescu from Romania, Hà Duy Hưng from Vietnam, and Zhaoli from China. A lot of work was invested in cleaning up our giant manuscript of errors. Special thanks in this respect go to David Kramer, our copy-editor, and to Prof. Titu Andreescu and his group for checking, in great detail, the validity of the solutions in this manuscript, and for their proposed corrections and alternative solutions to several problems. We also thank Prof. Abderrahim Ouardini from France for sending us the list of countries of origin for the shortlisted problems of 1998, Prof. Dorin Andrica for helping us compile the list of books for reference, and Prof. Ljubomir Čukić for proofreading part of the manuscript and helping us correct several errors. We would also like to express our thanks to all anonymous authors of the IMO problems. Without them, the IMO would obviously not be what it is today. It is a pity that authors’ names are not registered together with their proposed problems. In an attempt to change this, we have tried to trace down the authors of the problems, with partial success. We are thankful to all people who were so kind to help us in our investigation. The names we have found so far are listed in Appendix C. In many cases, the original solutions of the authors were used, and we duly acknowledge this immense contribution to our book, though once again, we regret that we cannot do this individually. In the same vein, we also thank all the students participating in the IMOs, since we have also included some of their original solutions in this book. We thank the following individuals who discussed problems with us and helped us with correcting the mistakes from the previous edition of the book: Xiaomin Chen, Orlando Döhring, Marija Jelić, Rudolfs Kreicbergs, Stefan Mehner, Yasser Ahmady Phoulady, Dominic Shau Chin, Juan Ignacio Restrepo, Arkadii Slinko, Harun Šiljak, Josef Tkadlec, Ilan Vardi, Gerhard Woeginger, and Yufei Zhao. The illustrations of geometry problems were done in WinGCLC, a program created by Prof. Predrag Janičić. This program is specifically designed for creating geometric pictures of unparalleled complexity quickly and efficiently. Even though it is still in its testing phase, its capabilities and utility are already remarkable and worthy of highest compliment. Finally, we would like to thank our families for all their love and support during the making of this book. Contents 1 Introduction . . . . . . . . . . . . . . . . . . . . . . . . . . . . . . . . . . . . . . . . . . . . . . . . . . . 1.1 The International Mathematical Olympiad . . . . . . . . . . . . . . . . . . . . . . 1.2 The IMO Compendium . . . . . . . . . . . . . . . . . . . . . . . . . . . . . . . . . . . . . . 1 1 2 2 Basic Concepts and Facts . . . . . . . . . . . . . . . . . . . . . . . . . . . . . . . . . . . . . . . 2.1 Algebra . . . . . . . . . . . . . . . . . . . . . . . . . . . . . . . . . . . . . . . . . . . . . . . . . . . 2.1.1 Polynomials . . . . . . . . . . . . . . . . . . . . . . . . . . . . . . . . . . . . . . . . 2.1.2 Recurrence Relations . . . . . . . . . . . . . . . . . . . . . . . . . . . . . . . . 2.1.3 Inequalities . . . . . . . . . . . . . . . . . . . . . . . . . . . . . . . . . . . . . . . . . 2.1.4 Groups and Fields . . . . . . . . . . . . . . . . . . . . . . . . . . . . . . . . . . . . 2.2 Analysis . . . . . . . . . . . . . . . . . . . . . . . . . . . . . . . . . . . . . . . . . . . . . . . . . . 2.3 Geometry . . . . . . . . . . . . . . . . . . . . . . . . . . . . . . . . . . . . . . . . . . . . . . . . . 2.3.1 Triangle Geometry . . . . . . . . . . . . . . . . . . . . . . . . . . . . . . . . . . . 2.3.2 Vectors in Geometry . . . . . . . . . . . . . . . . . . . . . . . . . . . . . . . . . . 2.3.3 Barycenters . . . . . . . . . . . . . . . . . . . . . . . . . . . . . . . . . . . . . . . . . 2.3.4 Quadrilaterals . . . . . . . . . . . . . . . . . . . . . . . . . . . . . . . . . . . . . . . 2.3.5 Circle Geometry . . . . . . . . . . . . . . . . . . . . . . . . . . . . . . . . . . . . . 2.3.6 Inversion . . . . . . . . . . . . . . . . . . . . . . . . . . . . . . . . . . . . . . . . . . . 2.3.7 Geometric Inequalities . . . . . . . . . . . . . . . . . . . . . . . . . . . . . . . . 2.3.8 Trigonometry . . . . . . . . . . . . . . . . . . . . . . . . . . . . . . . . . . . . . . . 2.3.9 Formulas in Geometry . . . . . . . . . . . . . . . . . . . . . . . . . . . . . . . . 2.4 Number Theory . . . . . . . . . . . . . . . . . . . . . . . . . . . . . . . . . . . . . . . . . . . . 2.4.1 Divisibility and Congruences . . . . . . . . . . . . . . . . . . . . . . . . . . 2.4.2 Exponential Congruences . . . . . . . . . . . . . . . . . . . . . . . . . . . . . 2.4.3 Quadratic Diophantine Equations . . . . . . . . . . . . . . . . . . . . . . . 2.4.4 Farey Sequences . . . . . . . . . . . . . . . . . . . . . . . . . . . . . . . . . . . . . 2.5 Combinatorics . . . . . . . . . . . . . . . . . . . . . . . . . . . . . . . . . . . . . . . . . . . . . 2.5.1 Counting of Objects . . . . . . . . . . . . . . . . . . . . . . . . . . . . . . . . . . 2.5.2 Graph Theory . . . . . . . . . . . . . . . . . . . . . . . . . . . . . . . . . . . . . . . 5 5 5 6 7 9 10 12 12 13 14 14 15 16 17 17 18 19 19 20 21 22 22 22 23 X 3 Contents Problems . . . . . . . . . . . . . . . . . . . . . . . . . . . . . . . . . . . . . . . . . . . . . . . . . . . . . . 3.1 IMO 1959 . . . . . . . . . . . . . . . . . . . . . . . . . . . . . . . . . . . . . . . . . . . . . . . . . 3.1.1 Contest Problems . . . . . . . . . . . . . . . . . . . . . . . . . . . . . . . . . . . . 3.2 IMO 1960 . . . . . . . . . . . . . . . . . . . . . . . . . . . . . . . . . . . . . . . . . . . . . . . . . 3.2.1 Contest Problems . . . . . . . . . . . . . . . . . . . . . . . . . . . . . . . . . . . . 3.3 IMO 1961 . . . . . . . . . . . . . . . . . . . . . . . . . . . . . . . . . . . . . . . . . . . . . . . . . 3.3.1 Contest Problems . . . . . . . . . . . . . . . . . . . . . . . . . . . . . . . . . . . . 3.4 IMO 1962 . . . . . . . . . . . . . . . . . . . . . . . . . . . . . . . . . . . . . . . . . . . . . . . . . 3.4.1 Contest Problems . . . . . . . . . . . . . . . . . . . . . . . . . . . . . . . . . . . . 3.5 IMO 1963 . . . . . . . . . . . . . . . . . . . . . . . . . . . . . . . . . . . . . . . . . . . . . . . . . 3.5.1 Contest Problems . . . . . . . . . . . . . . . . . . . . . . . . . . . . . . . . . . . . 3.6 IMO 1964 . . . . . . . . . . . . . . . . . . . . . . . . . . . . . . . . . . . . . . . . . . . . . . . . . 3.6.1 Contest Problems . . . . . . . . . . . . . . . . . . . . . . . . . . . . . . . . . . . . 3.7 IMO 1965 . . . . . . . . . . . . . . . . . . . . . . . . . . . . . . . . . . . . . . . . . . . . . . . . . 3.7.1 Contest Problems . . . . . . . . . . . . . . . . . . . . . . . . . . . . . . . . . . . . 3.8 IMO 1966 . . . . . . . . . . . . . . . . . . . . . . . . . . . . . . . . . . . . . . . . . . . . . . . . . 3.8.1 Contest Problems . . . . . . . . . . . . . . . . . . . . . . . . . . . . . . . . . . . . 3.8.2 Some Longlisted Problems 1959–1966 . . . . . . . . . . . . . . . . . . 3.9 IMO 1967 . . . . . . . . . . . . . . . . . . . . . . . . . . . . . . . . . . . . . . . . . . . . . . . . . 3.9.1 Contest Problems . . . . . . . . . . . . . . . . . . . . . . . . . . . . . . . . . . . . 3.9.2 Longlisted Problems . . . . . . . . . . . . . . . . . . . . . . . . . . . . . . . . . 3.10 IMO 1968 . . . . . . . . . . . . . . . . . . . . . . . . . . . . . . . . . . . . . . . . . . . . . . . . . 3.10.1 Contest Problems . . . . . . . . . . . . . . . . . . . . . . . . . . . . . . . . . . . . 3.10.2 Shortlisted Problems . . . . . . . . . . . . . . . . . . . . . . . . . . . . . . . . . 3.11 IMO 1969 . . . . . . . . . . . . . . . . . . . . . . . . . . . . . . . . . . . . . . . . . . . . . . . . . 3.11.1 Contest Problems . . . . . . . . . . . . . . . . . . . . . . . . . . . . . . . . . . . . 3.11.2 Longlisted Problems . . . . . . . . . . . . . . . . . . . . . . . . . . . . . . . . . 3.12 IMO 1970 . . . . . . . . . . . . . . . . . . . . . . . . . . . . . . . . . . . . . . . . . . . . . . . . . 3.12.1 Contest Problems . . . . . . . . . . . . . . . . . . . . . . . . . . . . . . . . . . . . 3.12.2 Longlisted Problems . . . . . . . . . . . . . . . . . . . . . . . . . . . . . . . . . 3.12.3 Shortlisted Problems . . . . . . . . . . . . . . . . . . . . . . . . . . . . . . . . . 3.13 IMO 1971 . . . . . . . . . . . . . . . . . . . . . . . . . . . . . . . . . . . . . . . . . . . . . . . . . 3.13.1 Contest Problems . . . . . . . . . . . . . . . . . . . . . . . . . . . . . . . . . . . . 3.13.2 Longlisted Problems . . . . . . . . . . . . . . . . . . . . . . . . . . . . . . . . . 3.13.3 Shortlisted Problems . . . . . . . . . . . . . . . . . . . . . . . . . . . . . . . . . 3.14 IMO 1972 . . . . . . . . . . . . . . . . . . . . . . . . . . . . . . . . . . . . . . . . . . . . . . . . . 3.14.1 Contest Problems . . . . . . . . . . . . . . . . . . . . . . . . . . . . . . . . . . . . 3.14.2 Longlisted Problems . . . . . . . . . . . . . . . . . . . . . . . . . . . . . . . . . 3.14.3 Shortlisted Problems . . . . . . . . . . . . . . . . . . . . . . . . . . . . . . . . . 3.15 IMO 1973 . . . . . . . . . . . . . . . . . . . . . . . . . . . . . . . . . . . . . . . . . . . . . . . . . 3.15.1 Contest Problems . . . . . . . . . . . . . . . . . . . . . . . . . . . . . . . . . . . . 3.15.2 Shortlisted Problems . . . . . . . . . . . . . . . . . . . . . . . . . . . . . . . . . 3.16 IMO 1974 . . . . . . . . . . . . . . . . . . . . . . . . . . . . . . . . . . . . . . . . . . . . . . . . . 3.16.1 Contest Problems . . . . . . . . . . . . . . . . . . . . . . . . . . . . . . . . . . . . 3.16.2 Longlisted Problems . . . . . . . . . . . . . . . . . . . . . . . . . . . . . . . . . 27 27 27 29 29 30 30 31 31 32 32 33 33 34 34 35 35 35 41 41 41 49 49 49 53 53 53 61 61 62 68 70 70 71 76 79 79 79 83 86 86 87 89 89 90 Contents 3.16.3 Shortlisted Problems . . . . . . . . . . . . . . . . . . . . . . . . . . . . . . . . . 3.17 IMO 1975 . . . . . . . . . . . . . . . . . . . . . . . . . . . . . . . . . . . . . . . . . . . . . . . . . 3.17.1 Contest Problems . . . . . . . . . . . . . . . . . . . . . . . . . . . . . . . . . . . . 3.17.2 Shortlisted Problems . . . . . . . . . . . . . . . . . . . . . . . . . . . . . . . . . 3.18 IMO 1976 . . . . . . . . . . . . . . . . . . . . . . . . . . . . . . . . . . . . . . . . . . . . . . . . . 3.18.1 Contest Problems . . . . . . . . . . . . . . . . . . . . . . . . . . . . . . . . . . . . 3.18.2 Longlisted Problems . . . . . . . . . . . . . . . . . . . . . . . . . . . . . . . . . 3.18.3 Shortlisted Problems . . . . . . . . . . . . . . . . . . . . . . . . . . . . . . . . . 3.19 IMO 1977 . . . . . . . . . . . . . . . . . . . . . . . . . . . . . . . . . . . . . . . . . . . . . . . . . 3.19.1 Contest Problems . . . . . . . . . . . . . . . . . . . . . . . . . . . . . . . . . . . . 3.19.2 Longlisted Problems . . . . . . . . . . . . . . . . . . . . . . . . . . . . . . . . . 3.19.3 Shortlisted Problems . . . . . . . . . . . . . . . . . . . . . . . . . . . . . . . . . 3.20 IMO 1978 . . . . . . . . . . . . . . . . . . . . . . . . . . . . . . . . . . . . . . . . . . . . . . . . . 3.20.1 Contest Problems . . . . . . . . . . . . . . . . . . . . . . . . . . . . . . . . . . . . 3.20.2 Longlisted Problems . . . . . . . . . . . . . . . . . . . . . . . . . . . . . . . . . 3.20.3 Shortlisted Problems . . . . . . . . . . . . . . . . . . . . . . . . . . . . . . . . . 3.21 IMO 1979 . . . . . . . . . . . . . . . . . . . . . . . . . . . . . . . . . . . . . . . . . . . . . . . . . 3.21.1 Contest Problems . . . . . . . . . . . . . . . . . . . . . . . . . . . . . . . . . . . . 3.21.2 Longlisted Problems . . . . . . . . . . . . . . . . . . . . . . . . . . . . . . . . . 3.21.3 Shortlisted Problems . . . . . . . . . . . . . . . . . . . . . . . . . . . . . . . . . 3.22 IMO 1981 . . . . . . . . . . . . . . . . . . . . . . . . . . . . . . . . . . . . . . . . . . . . . . . . . 3.22.1 Contest Problems . . . . . . . . . . . . . . . . . . . . . . . . . . . . . . . . . . . . 3.22.2 Shortlisted Problems . . . . . . . . . . . . . . . . . . . . . . . . . . . . . . . . . 3.23 IMO 1982 . . . . . . . . . . . . . . . . . . . . . . . . . . . . . . . . . . . . . . . . . . . . . . . . . 3.23.1 Contest Problems . . . . . . . . . . . . . . . . . . . . . . . . . . . . . . . . . . . . 3.23.2 Longlisted Problems . . . . . . . . . . . . . . . . . . . . . . . . . . . . . . . . . 3.23.3 Shortlisted Problems . . . . . . . . . . . . . . . . . . . . . . . . . . . . . . . . . 3.24 IMO 1983 . . . . . . . . . . . . . . . . . . . . . . . . . . . . . . . . . . . . . . . . . . . . . . . . . 3.24.1 Contest Problems . . . . . . . . . . . . . . . . . . . . . . . . . . . . . . . . . . . . 3.24.2 Longlisted Problems . . . . . . . . . . . . . . . . . . . . . . . . . . . . . . . . . 3.24.3 Shortlisted Problems . . . . . . . . . . . . . . . . . . . . . . . . . . . . . . . . . 3.25 IMO 1984 . . . . . . . . . . . . . . . . . . . . . . . . . . . . . . . . . . . . . . . . . . . . . . . . . 3.25.1 Contest Problems . . . . . . . . . . . . . . . . . . . . . . . . . . . . . . . . . . . . 3.25.2 Longlisted Problems . . . . . . . . . . . . . . . . . . . . . . . . . . . . . . . . . 3.25.3 Shortlisted Problems . . . . . . . . . . . . . . . . . . . . . . . . . . . . . . . . . 3.26 IMO 1985 . . . . . . . . . . . . . . . . . . . . . . . . . . . . . . . . . . . . . . . . . . . . . . . . . 3.26.1 Contest Problems . . . . . . . . . . . . . . . . . . . . . . . . . . . . . . . . . . . . 3.26.2 Longlisted Problems . . . . . . . . . . . . . . . . . . . . . . . . . . . . . . . . . 3.26.3 Shortlisted Problems . . . . . . . . . . . . . . . . . . . . . . . . . . . . . . . . . 3.27 IMO 1986 . . . . . . . . . . . . . . . . . . . . . . . . . . . . . . . . . . . . . . . . . . . . . . . . . 3.27.1 Contest Problems . . . . . . . . . . . . . . . . . . . . . . . . . . . . . . . . . . . . 3.27.2 Longlisted Problems . . . . . . . . . . . . . . . . . . . . . . . . . . . . . . . . . 3.27.3 Shortlisted Problems . . . . . . . . . . . . . . . . . . . . . . . . . . . . . . . . . 3.28 IMO 1987 . . . . . . . . . . . . . . . . . . . . . . . . . . . . . . . . . . . . . . . . . . . . . . . . . 3.28.1 Contest Problems . . . . . . . . . . . . . . . . . . . . . . . . . . . . . . . . . . . . XI 95 97 97 97 100 100 100 105 107 107 107 113 116 116 116 121 124 124 125 132 135 135 135 138 138 139 144 147 147 147 154 158 158 158 165 168 168 168 177 181 181 182 188 192 192 XII Contents 3.29 3.30 3.31 3.32 3.33 3.34 3.35 3.36 3.37 3.38 3.39 3.40 3.41 3.28.2 Longlisted Problems . . . . . . . . . . . . . . . . . . . . . . . . . . . . . . . . . 3.28.3 Shortlisted Problems . . . . . . . . . . . . . . . . . . . . . . . . . . . . . . . . . IMO 1988 . . . . . . . . . . . . . . . . . . . . . . . . . . . . . . . . . . . . . . . . . . . . . . . . . 3.29.1 Contest Problems . . . . . . . . . . . . . . . . . . . . . . . . . . . . . . . . . . . . 3.29.2 Longlisted Problems . . . . . . . . . . . . . . . . . . . . . . . . . . . . . . . . . 3.29.3 Shortlisted Problems . . . . . . . . . . . . . . . . . . . . . . . . . . . . . . . . . IMO 1989 . . . . . . . . . . . . . . . . . . . . . . . . . . . . . . . . . . . . . . . . . . . . . . . . . 3.30.1 Contest Problems . . . . . . . . . . . . . . . . . . . . . . . . . . . . . . . . . . . . 3.30.2 Longlisted Problems . . . . . . . . . . . . . . . . . . . . . . . . . . . . . . . . . 3.30.3 Shortlisted Problems . . . . . . . . . . . . . . . . . . . . . . . . . . . . . . . . . IMO 1990 . . . . . . . . . . . . . . . . . . . . . . . . . . . . . . . . . . . . . . . . . . . . . . . . . 3.31.1 Contest Problems . . . . . . . . . . . . . . . . . . . . . . . . . . . . . . . . . . . . 3.31.2 Longlisted Problems . . . . . . . . . . . . . . . . . . . . . . . . . . . . . . . . . 3.31.3 Shortlisted Problems . . . . . . . . . . . . . . . . . . . . . . . . . . . . . . . . . IMO 1991 . . . . . . . . . . . . . . . . . . . . . . . . . . . . . . . . . . . . . . . . . . . . . . . . . 3.32.1 Contest Problems . . . . . . . . . . . . . . . . . . . . . . . . . . . . . . . . . . . . 3.32.2 Shortlisted Problems . . . . . . . . . . . . . . . . . . . . . . . . . . . . . . . . . IMO 1992 . . . . . . . . . . . . . . . . . . . . . . . . . . . . . . . . . . . . . . . . . . . . . . . . . 3.33.1 Contest Problems . . . . . . . . . . . . . . . . . . . . . . . . . . . . . . . . . . . . 3.33.2 Longlisted Problems . . . . . . . . . . . . . . . . . . . . . . . . . . . . . . . . . 3.33.3 Shortlisted Problems . . . . . . . . . . . . . . . . . . . . . . . . . . . . . . . . . IMO 1993 . . . . . . . . . . . . . . . . . . . . . . . . . . . . . . . . . . . . . . . . . . . . . . . . . 3.34.1 Contest Problems . . . . . . . . . . . . . . . . . . . . . . . . . . . . . . . . . . . . 3.34.2 Shortlisted Problems . . . . . . . . . . . . . . . . . . . . . . . . . . . . . . . . . IMO 1994 . . . . . . . . . . . . . . . . . . . . . . . . . . . . . . . . . . . . . . . . . . . . . . . . . 3.35.1 Contest Problems . . . . . . . . . . . . . . . . . . . . . . . . . . . . . . . . . . . . 3.35.2 Shortlisted Problems . . . . . . . . . . . . . . . . . . . . . . . . . . . . . . . . . IMO 1995 . . . . . . . . . . . . . . . . . . . . . . . . . . . . . . . . . . . . . . . . . . . . . . . . . 3.36.1 Contest Problems . . . . . . . . . . . . . . . . . . . . . . . . . . . . . . . . . . . . 3.36.2 Shortlisted Problems . . . . . . . . . . . . . . . . . . . . . . . . . . . . . . . . . IMO 1996 . . . . . . . . . . . . . . . . . . . . . . . . . . . . . . . . . . . . . . . . . . . . . . . . . 3.37.1 Contest Problems . . . . . . . . . . . . . . . . . . . . . . . . . . . . . . . . . . . . 3.37.2 Shortlisted Problems . . . . . . . . . . . . . . . . . . . . . . . . . . . . . . . . . IMO 1997 . . . . . . . . . . . . . . . . . . . . . . . . . . . . . . . . . . . . . . . . . . . . . . . . . 3.38.1 Contest Problems . . . . . . . . . . . . . . . . . . . . . . . . . . . . . . . . . . . . 3.38.2 Shortlisted Problems . . . . . . . . . . . . . . . . . . . . . . . . . . . . . . . . . IMO 1998 . . . . . . . . . . . . . . . . . . . . . . . . . . . . . . . . . . . . . . . . . . . . . . . . . 3.39.1 Contest Problems . . . . . . . . . . . . . . . . . . . . . . . . . . . . . . . . . . . . 3.39.2 Shortlisted Problems . . . . . . . . . . . . . . . . . . . . . . . . . . . . . . . . . IMO 1999 . . . . . . . . . . . . . . . . . . . . . . . . . . . . . . . . . . . . . . . . . . . . . . . . . 3.40.1 Contest Problems . . . . . . . . . . . . . . . . . . . . . . . . . . . . . . . . . . . . 3.40.2 Shortlisted Problems . . . . . . . . . . . . . . . . . . . . . . . . . . . . . . . . . IMO 2000 . . . . . . . . . . . . . . . . . . . . . . . . . . . . . . . . . . . . . . . . . . . . . . . . . 3.41.1 Contest Problems . . . . . . . . . . . . . . . . . . . . . . . . . . . . . . . . . . . . 3.41.2 Shortlisted Problems . . . . . . . . . . . . . . . . . . . . . . . . . . . . . . . . . 192 200 203 203 204 212 217 217 218 229 234 234 235 245 249 249 249 253 253 253 262 266 266 267 271 271 271 275 275 275 280 280 281 286 286 287 291 291 291 295 295 295 300 300 300 Contents 4 XIII 3.42 IMO 2001 . . . . . . . . . . . . . . . . . . . . . . . . . . . . . . . . . . . . . . . . . . . . . . . . . 3.42.1 Contest Problems . . . . . . . . . . . . . . . . . . . . . . . . . . . . . . . . . . . . 3.42.2 Shortlisted Problems . . . . . . . . . . . . . . . . . . . . . . . . . . . . . . . . . 3.43 IMO 2002 . . . . . . . . . . . . . . . . . . . . . . . . . . . . . . . . . . . . . . . . . . . . . . . . . 3.43.1 Contest Problems . . . . . . . . . . . . . . . . . . . . . . . . . . . . . . . . . . . . 3.43.2 Shortlisted Problems . . . . . . . . . . . . . . . . . . . . . . . . . . . . . . . . . 3.44 IMO 2003 . . . . . . . . . . . . . . . . . . . . . . . . . . . . . . . . . . . . . . . . . . . . . . . . . 3.44.1 Contest Problems . . . . . . . . . . . . . . . . . . . . . . . . . . . . . . . . . . . . 3.44.2 Shortlisted Problems . . . . . . . . . . . . . . . . . . . . . . . . . . . . . . . . . 3.45 IMO 2004 . . . . . . . . . . . . . . . . . . . . . . . . . . . . . . . . . . . . . . . . . . . . . . . . . 3.45.1 Contest Problems . . . . . . . . . . . . . . . . . . . . . . . . . . . . . . . . . . . . 3.45.2 Shortlisted Problems . . . . . . . . . . . . . . . . . . . . . . . . . . . . . . . . . 3.46 IMO 2005 . . . . . . . . . . . . . . . . . . . . . . . . . . . . . . . . . . . . . . . . . . . . . . . . . 3.46.1 Contest Problems . . . . . . . . . . . . . . . . . . . . . . . . . . . . . . . . . . . . 3.46.2 Shortlisted Problems . . . . . . . . . . . . . . . . . . . . . . . . . . . . . . . . . 3.47 IMO 2006 . . . . . . . . . . . . . . . . . . . . . . . . . . . . . . . . . . . . . . . . . . . . . . . . . 3.47.1 Contest Problems . . . . . . . . . . . . . . . . . . . . . . . . . . . . . . . . . . . . 3.47.2 Shortlisted Problems . . . . . . . . . . . . . . . . . . . . . . . . . . . . . . . . . 3.48 IMO 2007 . . . . . . . . . . . . . . . . . . . . . . . . . . . . . . . . . . . . . . . . . . . . . . . . . 3.48.1 Contest Problems . . . . . . . . . . . . . . . . . . . . . . . . . . . . . . . . . . . . 3.48.2 Shortlisted Problems . . . . . . . . . . . . . . . . . . . . . . . . . . . . . . . . . 3.49 IMO 2008 . . . . . . . . . . . . . . . . . . . . . . . . . . . . . . . . . . . . . . . . . . . . . . . . . 3.49.1 Contest Problems . . . . . . . . . . . . . . . . . . . . . . . . . . . . . . . . . . . . 3.49.2 Shortlisted Problems . . . . . . . . . . . . . . . . . . . . . . . . . . . . . . . . . 3.50 IMO 2009 . . . . . . . . . . . . . . . . . . . . . . . . . . . . . . . . . . . . . . . . . . . . . . . . . 3.50.1 Contest Problems . . . . . . . . . . . . . . . . . . . . . . . . . . . . . . . . . . . . 3.50.2 Shortlisted Problems . . . . . . . . . . . . . . . . . . . . . . . . . . . . . . . . . 304 304 304 308 308 308 312 312 312 317 317 318 322 322 322 326 326 326 331 331 332 336 336 337 341 341 341 Solutions . . . . . . . . . . . . . . . . . . . . . . . . . . . . . . . . . . . . . . . . . . . . . . . . . . . . . . 4.1 Contest Problems 1959 . . . . . . . . . . . . . . . . . . . . . . . . . . . . . . . . . . . . . . 4.2 Contest Problems 1960 . . . . . . . . . . . . . . . . . . . . . . . . . . . . . . . . . . . . . . 4.3 Contest Problems 1961 . . . . . . . . . . . . . . . . . . . . . . . . . . . . . . . . . . . . . . 4.4 Contest Problems 1962 . . . . . . . . . . . . . . . . . . . . . . . . . . . . . . . . . . . . . . 4.5 Contest Problems 1963 . . . . . . . . . . . . . . . . . . . . . . . . . . . . . . . . . . . . . . 4.6 Contest Problems 1964 . . . . . . . . . . . . . . . . . . . . . . . . . . . . . . . . . . . . . . 4.7 Contest Problems 1965 . . . . . . . . . . . . . . . . . . . . . . . . . . . . . . . . . . . . . . 4.8 Contest Problems 1966 . . . . . . . . . . . . . . . . . . . . . . . . . . . . . . . . . . . . . . 4.9 Longlisted Problems 1967 . . . . . . . . . . . . . . . . . . . . . . . . . . . . . . . . . . . 4.10 Shortlisted Problems 1968 . . . . . . . . . . . . . . . . . . . . . . . . . . . . . . . . . . . 4.11 Contest Problems 1969 . . . . . . . . . . . . . . . . . . . . . . . . . . . . . . . . . . . . . . 4.12 Shortlisted Problems 1970 . . . . . . . . . . . . . . . . . . . . . . . . . . . . . . . . . . . 4.13 Shortlisted Problems 1971 . . . . . . . . . . . . . . . . . . . . . . . . . . . . . . . . . . . 4.14 Shortlisted Problems 1972 . . . . . . . . . . . . . . . . . . . . . . . . . . . . . . . . . . . 4.15 Shortlisted Problems 1973 . . . . . . . . . . . . . . . . . . . . . . . . . . . . . . . . . . . 4.16 Shortlisted Problems 1974 . . . . . . . . . . . . . . . . . . . . . . . . . . . . . . . . . . . 347 347 349 351 353 354 355 357 359 361 374 380 383 389 396 401 407 XIV Contents 4.17 4.18 4.19 4.20 4.21 4.22 4.23 4.24 4.25 4.26 4.27 4.28 4.29 4.30 4.31 4.32 4.33 4.34 4.35 4.36 4.37 4.38 4.39 4.40 4.41 4.42 4.43 4.44 4.45 4.46 4.47 4.48 4.49 4.50 Shortlisted Problems 1975 Shortlisted Problems 1976 Longlisted Problems 1977 Shortlisted Problems 1978 Shortlisted Problems 1979 Shortlisted Problems 1981 Shortlisted Problems 1982 Shortlisted Problems 1983 Shortlisted Problems 1984 Shortlisted Problems 1985 Shortlisted Problems 1986 Shortlisted Problems 1987 Shortlisted Problems 1988 Shortlisted Problems 1989 Shortlisted Problems 1990 Shortlisted Problems 1991 Shortlisted Problems 1992 Shortlisted Problems 1993 Shortlisted Problems 1994 Shortlisted Problems 1995 Shortlisted Problems 1996 Shortlisted Problems 1997 Shortlisted Problems 1998 Shortlisted Problems 1999 Shortlisted Problems 2000 Shortlisted Problems 2001 Shortlisted Problems 2002 Shortlisted Problems 2003 Shortlisted Problems 2004 Shortlisted Problems 2005 Shortlisted Problems 2006 Shortlisted Problems 2007 Shortlisted Problems 2008 Shortlisted Problems 2009 ................................... ................................... ................................... ................................... ................................... ................................... ................................... ................................... ................................... ................................... ................................... ................................... ................................... ................................... ................................... ................................... ................................... ................................... ................................... ................................... ................................... ................................... ................................... ................................... ................................... ................................... ................................... ................................... ................................... ................................... ................................... ................................... ................................... ................................... 413 418 422 437 445 453 461 467 476 483 491 498 508 523 537 550 563 573 585 593 606 622 636 650 664 677 691 702 715 730 742 754 765 777 A Notation and Abbreviations . . . . . . . . . . . . . . . . . . . . . . . . . . . . . . . . . . . . . 791 A.1 Notation . . . . . . . . . . . . . . . . . . . . . . . . . . . . . . . . . . . . . . . . . . . . . . . . . . 791 A.2 Abbreviations . . . . . . . . . . . . . . . . . . . . . . . . . . . . . . . . . . . . . . . . . . . . . . 792 B Codes of the Countries of Origin . . . . . . . . . . . . . . . . . . . . . . . . . . . . . . . . . 795 C Authors of Problems . . . . . . . . . . . . . . . . . . . . . . . . . . . . . . . . . . . . . . . . . . . . 797 References . . . . . . . . . . . . . . . . . . . . . . . . . . . . . . . . . . . . . . . . . . . . . . . . . . . . . . . . . 805 1 Introduction 1.1 The International Mathematical Olympiad The International Mathematical Olympiad (IMO) is the most important and prestigious mathematical competition for high-school students. It has played a significant role in generating wide interest in mathematics among high school students, as well as identifying talent. In the beginning, the IMO was a much smaller competition than it is today. In 1959, the following seven countries gathered to compete in the first IMO: Bulgaria, Czechoslovakia, German Democratic Republic, Hungary, Poland, Romania, and the Soviet Union. Since then, the competition has been held annually. Gradually, other Eastern-block countries, countries from Western Europe, and ultimately numerous countries from around the world and every continent joined in. (The only year in which the IMO was not held was 1980, when for financial reasons no one stepped in to host it. Today this is hardly a problem, and hosts are lined up several years in advance.) In the 50th IMO, held in Bremen, no fewer than 104 countries took part. The format of the competition quickly became stable and unchanging. Each country may send up to six contestants and each contestant competes individually (without any help or collaboration). The country also sends a team leader, who participates in problem selection and is thus isolated from the rest of the team until the end of the competition, and a deputy leader, who looks after the contestants. The IMO competition lasts two days. On each day students are given four and a half hours to solve three problems, for a total of six problems. The first problem is usually the easiest on each day and the last problem the hardest, though there have been many notable exceptions. ((IMO96-5) is one of the most difficult problems from all the Olympiads, having been fully solved by only six students out of several hundred!) Each problem is worth 7 points, making 42 points the maximum possible score. The number of points obtained by a contestant on each problem is the result of intense negotiations and, ultimately, agreement among the problem coordinators, assigned by the host country, and the team leader and deputy, who defend the interests of their contestants. This system ensures a relatively objective grade that is seldom off by more than two or three points. D. Djukić et al., The IMO Compendium, Problem Books in Mathematics, DOI 10.1007/978-1-4419-9854-5_1, © Springer Science + Business Media, LLC 2011 1 2 1 Introduction Though countries naturally compare each other’s scores, only individual prizes, namely medals and honorable mentions, are awarded on the IMO. Fewer than one twelfth of participants are awarded the gold medal, fewer than one fourth are awarded the gold or silver medal, and fewer than one half are awarded the gold, silver or bronze medal. Among the students not awarded a medal, those who score 7 points on at least one problem are awarded an honorable mention. This system of determining awards works rather well. It ensures, on the one hand, strict criteria and appropriate recognition for each level of performance, giving every contestant something to strive for. On the other hand, it also ensures a good degree of generosity that does not greatly depend on the variable difficulty of the problems proposed. According to the statistics, the hardest Olympiad was that in 1971, followed by those in 1996, 1993, and 1999. The Olympiad in which the winning team received the lowest score was that in 1977, followed by those in 1960 and 1999. The selection of the problems consists of several steps. Participant countries send their proposals, which are supposed to be novel, to the IMO organizers. The organizing country does not propose problems. From the received proposals (the longlisted problems), the problem committee selects a shorter list (the shortlisted problems), which is presented to the IMO jury, consisting of all the team leaders. From the short-listed problems the jury chooses six problems for the IMO. Apart from its mathematical and competitive side, the IMO is also a very large social event. After their work is done, the students have three days to enjoy events and excursions organized by the host country, as well as to interact and socialize with IMO participants from around the world. All this makes for a truly memorable experience. 1.2 The IMO Compendium Olympiad problems have been published in many books [97]. However, the remaining shortlisted and longlisted problems have not been systematically collected and published, and therefore many of them are unknown to mathematicians interested in this subject. Some partial collections of shortlisted and longlisted problems can be found in the references, though usually only for one year. References [1], [39], [57], [88] contain problems from multiple years. In total, these books cover roughly 50% of the problems found in this book. The goal of this book is to present, in a single volume, our comprehensive collection of problems proposed for the IMO. It consists of all problems selected for the IMO competitions, shortlisted problems from the 10th IMO and from the 12th through 50th IMOs, and longlisted problems from twenty IMOs. We do not have shortlisted problems from the 9th and the 11th IMOs, and we could not discover whether competition problems at those two IMOs were selected from the longlisted problems or whether there existed shortlisted problems that have not been preserved. Since IMO organizers usually do not distribute longlisted problems to the representatives of participant countries, our collection is incomplete. The practice of distribut- 1.2 The IMO Compendium 3 ing these longlists effectively ended in 1989. A selection of problems from the first eight IMOs has been taken from [88]. The book is organized as follows. For each year, the problems that were given on the IMO contest are presented, along with the longlisted and/or shortlisted problems, if applicable. We present solutions to all shortlisted problems. The problems appearing on the IMOs are solved among the other shortlisted problems. The longlisted problems have not been provided with solutions, except for the two IMOs held in Yugoslavia (for patriotic reasons), since that would have made the book unreasonably long. This book has thus the added benefit for professors and team coaches of being a suitable book from which to assign problems. For each problem, we indicate the country that proposed it with a three-letter code. A complete list of country codes and the corresponding countries is given in the appendix. In all shortlists, we also indicate which problems were selected for the contest. We occasionally make references in our solutions to other problems in a straightforward way. After indicating with LL, SL, or IMO whether the problem is from a longlist, shortlist, or contest, we indicate the year of the IMO and then the number of the problem. For example, (SL89-15) refers to the fifteenth problem of the shortlist of 1989. We also present a rough list of all formulas and theorems not obviously derivable that were called upon in our proofs. Since we were largely concerned with only the theorems used in proving the problems of this book, we believe that the list is a good compilation of the most useful theorems for IMO problem solving. The gathering of such a large collection of problems into a book required a massive amount of editing. We reformulated the problems whose original formulations were not precise or clear. We translated the problems that were not in English. Some of the solutions are taken from the author of the problem or other sources, while others are original solutions of the authors of this book. Many of the non-original solutions were significantly edited before being included. We do not make any guarantee that the problems in this book fully correspond to the actual shortlisted or longlisted problems. However, we believe this book to be the closest possible approximation to such a list. 2 Basic Concepts and Facts The following is a list of the most basic concepts and theorems frequently used in this book. We encourage the reader to become familiar with them and perhaps read up on them further in other literature. 2.1 Algebra 2.1.1 Polynomials Theorem 2.1. The quadratic equation ax2 + bx + c = 0 (a, b, c ∈ R, a 6= 0) has solutions √ −b ± b2 − 4ac x1,2 = . 2a The discriminant D of a quadratic equation is defined as D = b2 − 4ac. For D < 0 the solutions are complex and conjugate to each other, for D = 0 the solutions degenerate to one real solution, and for D > 0 the equation has two distinct real solutions.  Definition 2.2. Binomial coefficients nk , n, k ∈ N0 , k ≤ n, are defined as   n n! = . i i!(n − i)!       n  They satisfy ni + i−1 = n+1 for i > 0 and also n0 + n1 + · · · + nn = 2n , n0 − i   m  n+r n n+m j−1 n n = ∑ki=0 ni k−i , n = ∑rj=0 n+n−1 . 1 + · · · + (−1) n = 0, k Theorem 2.3 ((Newton’s) binomial formula). For x, y ∈ C and n ∈ N, n   n n−i i (x + y)n = ∑ x y. i=0 i Theorem 2.4 (Bézout’s theorem). A polynomial P(x) is divisible by the binomial x − a (a ∈ C) if and only if P(a) = 0. D. Djukić et al., The IMO Compendium, Problem Books in Mathematics, DOI 10.1007/978-1-4419-9854-5_2, © Springer Science + Business Media, LLC 2011 5 6 2 Basic Concepts and Facts Theorem 2.5 (The rational root theorem). If x = p/q is a rational zero of a polynomial P(x) = an xn + · · · + a0 with integer coefficients and (p, q) = 1, then p | a0 and q | an . Theorem 2.6 (The fundamental theorem of algebra). Every nonconstant polynomial with coefficients in C has a complex root. Theorem 2.7 (Eisenstein’s criterion (extended)). Let P(x) = an xn + · · · + a1 x + a0 be a polynomial with integer coefficients. If there exist a prime p and an integer k ∈ {0, 1, . . . , n − 1} such that p | a0 , a1 , . . . , ak , p ∤ ak+1 , and p2 ∤ a0 , then there exists an irreducible factor Q(x) of P(x) whose degree is greater than k. In particular, if p can be chosen such that k = n − 1, then P(x) is irreducible. Definition 2.8. Symmetric polynomials in x1 , . . . , xn are polynomials that do not change on permuting the variables x1 , . . . , xn . Elementary symmetric polynomials are σk (x1 , . . . , xn ) = ∑ xi1 · · · xik (the sum is over all k-element subsets {i1 , . . . , ik } of {1, 2, . . . , n}). Theorem 2.9. Every symmetric polynomial in x1 , . . . , xn can be expressed as a polynomial in the elementary symmetric polynomials σ1 , . . . , σn . Theorem 2.10 (Viète’s formulas). Let α1 , . . . , αn and c1 , . . . , cn be complex numbers such that (x − α1 )(x − α2 ) · · · (x − αn ) = xn + c1 xn−1 + c2 xn−2 + · · · + cn . Then ck = (−1)k σk (α1 , . . . , αn ) for k = 1, 2, . . . , n. Theorem 2.11 (Newton’s formulas on symmetric polynomials). Let σk = σk (x1 , . . . , xn ) and let sk = xk1 + xk2 + · · · + xkn , where x1 , . . . , xn are arbitrary complex numbers. Then kσk = s1 σk−1 − s2 σk−2 + · · · + (−1)k sk−1 σ1 + (−1)k−1 sk . 2.1.2 Recurrence Relations Definition 2.12. A recurrence relation is a relation that determines the elements of a sequence xn , n ∈ N0 , as a function of previous elements. A recurrence relation of the form (∀n ≥ k) xn + a1 xn−1 + · · · + ak xn−k = 0 for constants a1 , . . . , ak is called a linear homogeneous recurrence relation of order k. We define the characteristic polynomial of the relation as P(x) = xk + a1xk−1 + · · · + ak . Theorem 2.13. Using the notation introduced in the above definition, let P(x) factorize as P(x) = (x− α1 )k1 (x− α2 )k2 · · · (x− αr )kr , where α1 , . . . , αr are distinct complex 2.1 Algebra 7 numbers and k1 , . . . , kr are positive integers. The general solution of this recurrence relation is in this case given by xn = p1 (n)α1n + p2 (n)α2n + · · · + pr (n)αrn , where pi is a polynomial of degree less than ki . In particular, if P(x) has k distinct roots, then all pi are constant. If x0 , . . . , xk−1 are set, then the coefficients of the polynomials are uniquely determined. 2.1.3 Inequalities Theorem 2.14. The squaring function is always positive; i.e., (∀x ∈ R) x2 ≥ 0. By substituting different expressions for x, many of the inequalities below are obtained. Theorem 2.15 (Bernoulli’s inequalities). 1. If n ≥ 1 is an integer and x > −1 a real number, then (1 + x)n ≥ 1 + nx. 2. If α > 1 or α < 0, then for x > −1, the following inequality holds: (1 + x)α ≥ 1 + α x. 3. If α ∈ (0, 1) then for x > −1 the following inequality holds: (1 + x)α ≤ 1 + α x. Theorem 2.16 (The mean inequalities). For positive real numbers x1 , x2 , . . . , xn it is always the case that QM ≥ AM ≥ GM ≥ HM, where s x21 + · · · + x2n x1 + · · · + xn QM = , AM = , n n √ n GM = n x1 · · · xn , HM = 1 . 1 x1 + · · · + xn Each of these inequalities becomes an equality if and only if x1 = x2 = · · · = xn . The numbers QM, AM, GM, and HM are respectively called the quadratic mean, the arithmetic mean, the geometric mean, and the harmonic mean of x1 , x2 , . . . , xn . Theorem 2.17 (The general mean inequality). Let x1 , . . . , xn be positive real numbers. For each p ∈ R we define the mean of order p of x1 , . . . , xn by Mp =  p p 1/p x1 + · · · + xn n for p 6= 0, and Mq = lim p→q M p for q ∈ {±∞, 0}. Then M p ≤ Mq whenever p ≤ q. Remark. In particular, max xi , QM, AM, GM, HM, and min xi are M∞ , M2 , M1 , M0 , M−1 , and M−∞ respectively. 8 2 Basic Concepts and Facts Theorem 2.18 (Cauchy–Schwarz inequality). Let ai , bi , i = 1, 2, . . . , n, be real numbers. Then ! ! ! 2 n ∑ a i bi i=1 n n ∑ a2i ≤ ∑ b2i i=1 . i=1 Equality occurs if and only if there exists c ∈ R such that bi = cai for i = 1, . . . , n. Theorem 2.19 (Hölder’s inequality). Let ai , bi , i = 1, 2, . . . , n, be nonnegative real numbers, and let p, q be positive real numbers such that 1/p + 1/q = 1. Then n n i=1 i=1 ∑ ai bi ≤ ∑ aip !1/p n ∑ i=1 q bi !1/q . Equality occurs if and only if there exists c ∈ R such that bi = cai for i = 1, . . . , n. The Cauchy–Schwarz inequality is a special case of Hölder’s inequality for p = q = 2. Theorem 2.20 (Minkowski’s inequality). Let ai , bi (i = 1, 2, . . . , n) be nonnegative real numbers and p any real number not smaller than 1. Then n ∑ (ai + bi) i=1 p !1/p n ≤ ∑ i=1 aip !1/p n ∑ + i=1 bip !1/p . For p > 1 equality occurs if and only if there exists c ∈ R such that bi = cai for i = 1, . . . , n. For p = 1 equality occurs in all cases. Theorem 2.21 (Chebyshev’s inequality). Let a1 ≥ a2 ≥ · · · ≥ an and b1 ≥ b2 ≥ · · · ≥ bn be real numbers. Then ! ! n n ∑ ai bi ≥ i=1 n ∑ ai i=1 n ∑ bi i=1 n ≥ n ∑ ai bn+1−i . i=1 The two inequalities become equalities at the same time when a1 = a2 = · · · = an or b1 = b 2 = · · · = bn . Definition 2.22. A real function f defined on an interval I is convex if f (α x + β y) ≤ α f (x) + β f (y) for all x, y ∈ I and all α , β > 0 such that α + β = 1. A function f is said to be concave if the opposite inequality holds, i.e., if − f is convex. Theorem 2.23. If f is continuous on an interval I, then f is convex on that interval if and only if   x+y f (x) + f (y) f ≤ for all x, y ∈ I. 2 2 Theorem 2.24. If f is differentiable, then it is convex if and only if the derivative f ′ is nondecreasing. Similarly, differentiable function f is concave if and only if f ′ is nonincreasing. 2.1 Algebra 9 Theorem 2.25 (Jensen’s inequality). If f : I → R is a convex function, then the inequality f (α1 x1 + · · · + αn xn ) ≤ α1 f (x1 ) + · · · + αn f (xn ) holds for all αi ≥ 0, α1 +· · · + αn = 1, and xi ∈ I. For a concave function the opposite inequality holds. Theorem 2.26 (Muirhead’s inequality). Given x1 , x2 , . . . , xn ∈ R+ and an n-tuple a = (a1 , . . . , an ) of positive real numbers, we define Ta (x1 , . . . , xn ) = ∑ ya11 · · · yann , the sum being taken over all permutations y1 , . . . , yn of x1 , . . . , xn . We say that an ntuple a majorizes an n-tuple b if a1 + · · · + an = b1 + · · · + bn and a1 + · · · + ak ≥ b1 + · · · + bk for each k = 1, . . . , n − 1. If a nonincreasing n-tuple a majorizes a nonincreasing n-tuple b, then the following inequality holds: Ta (x1 , . . . , xn ) ≥ Tb (x1 , . . . , xn ). Equality occurs if and only if x1 = x2 = · · · = xn . Theorem 2.27 (Schur’s inequality). Using the notation introduced for Muirhead’s inequality, Tλ +2 µ ,0,0 (x1 , x2 , x3 ) + Tλ ,µ ,µ (x1 , x2 , x3 ) ≥ 2Tλ +µ ,µ ,0 (x1 , x2 , x3 ), where λ ∈ R, µ > 0. Equality occurs if and only if x1 = x2 = x3 or x1 = x2 , x3 = 0 (and in analogous cases). An equivalent form of the Schur’s inequality is xλ (xµ − yµ )(xµ − zµ ) + yλ (yµ − xµ )(yµ − zµ ) + zλ (zµ − xµ )(zµ − yµ ) ≥ 0. 2.1.4 Groups and Fields Definition 2.28. A group is a nonempty set G equipped with a binary operation ∗ satisfying the following conditions: (i) a ∗ (b ∗ c) = (a ∗ b) ∗ c for all a, b, c ∈ G. (ii) There exists a (unique) identity e ∈ G such that e ∗ a = a ∗ e = a for all a ∈ G. (iii) For each a ∈ G there exists a (unique) inverse a−1 = b ∈ G such that a ∗ b = b ∗ a = e. If n ∈ Z, we define an as a ∗ a ∗ · · ·∗ a (n times) if n ≥ 0, and as (a−1 )−n otherwise. Definition 2.29. A group G = (G, ∗) is commutative or abelian if a ∗ b = b ∗ a for all a, b ∈ G. Definition 2.30. A set A generates a group (G, ∗) if every element of G can be obtained using powers of the elements of A and the operation ∗. In other words, if A is the generator of a group G, then every element g ∈ G can be written as ai11 ∗ · · · ∗ ainn , where a j ∈ A and i j ∈ Z for every j = 1, 2, . . . , n. 10 2 Basic Concepts and Facts Definition 2.31. The order of an element a ∈ G is the smallest n ∈ N, if it exists such that an = e. If no such n exists then the element a is said to be of infinite order. The order of a group is the number of its elements, if it is finite. Each element of a finite group has finite order. Theorem 2.32 (Lagrange’s theorem). In a finite group, the order of an element divides the order of the group. Definition 2.33. A ring is a nonempty set R equipped with two operations + and · such that (R, +) is an abelian group and for any a, b, c ∈ R, (i) (a · b) · c = a · (b · c); (ii) (a + b) · c = a · c + b · c and c · (a + b) = c · a + c · b. A ring is commutative if a · b = b · a for any a, b ∈ R and with identity if there exists a multiplicative identity i ∈ R such that i · a = a · i = a for all a ∈ R. Definition 2.34. A field is a commutative ring with identity in which every element a other than the additive identity has a multiplicative inverse a−1 such that a · a−1 = a−1 · a = i. Theorem 2.35. The following are common examples of groups, rings, and fields: Groups: (Zn , +), (Z p \ {0}, ·), (Q, +), (R, +), (R \ {0}, ·). Rings: (Zn , +, ·), (Z, +, ·), (Z[x], +, ·), (R[x], +, ·). √ Fields: (Z p , +, ·), (Q, +, ·), (Q( 2), +, ·), (R, +, ·), (C, +, ·). 2.2 Analysis Definition 2.36. A sequence {an }∞ n=1 of real numbers has a limit a = limn→∞ an (also denoted by an → a) if (∀ε > 0)(∃nε ∈ N)(∀n ≥ nε ) |an − a| < ε . A function f : (a, b) → R has a limit y = limx→c f (x) if (∀ε > 0)(∃δ > 0)(∀x ∈ (a, b)) 0 < |x − c| < δ ⇒ | f (x) − y| < ε . Definition 2.37. A sequence {xn } converges to x ∈ R if limn→∞ xn = x. A series m ∑∞ n=1 xn converges to s ∈ R if and only if limm→∞ ∑n=1 xn = s. A sequence or series that does not converge is said to diverge. Theorem 2.38. A sequence {an } of real numbers is convergent if it is monotonic and bounded. 2.2 Analysis 11 Definition 2.39. A function f is continuous on [a, b] if the following three relations hold: lim f (x) = f (x0 ), for every x0 ∈ (a, b), x→x0 lim f (x) = f (a), x→a+ and lim f (x) = f (b). x→b− Definition 2.40. A function f : (a, b) → R is differentiable at a point x0 ∈ (a, b) if the following limit exists: f ′ (x0 ) = lim x→x0 f (x) − f (x0 ) . x − x0 A function is differentiable on (a, b) if it is differentiable at every x0 ∈ (a, b). The function f ′ is called the derivative of f . We similarly define the second derivative f ′′ as the derivative of f ′ , and so on. Theorem 2.41. A differentiable function is also continuous. If f and g are differentiable, then f g, α f + β g (α , β ∈ R), f ◦ g, 1/ f (if f 6= 0), f −1 (if well defined) are also differentiable. It holds that (α f + β g)′ = α f ′ + β g′, ( f g)′ = f ′ g + f g′ , ( f ◦ g)′ = ( f ′ ◦ g) · g′, (1/ f )′ = − f ′ / f 2 , ( f /g)′ = ( f ′ g − f g′)/g2 , ( f −1 )′ = 1/( f ′ ◦ f −1 ). Theorem 2.42. The following are derivatives of some elementary functions (a denotes a real constant): (xa )′ = axa−1 , (ln x)′ = 1/x, (ax )′ = ax ln a, (sin x)′ = cos x, (cos x)′ = − sin x. Theorem 2.43 (Fermat’s theorem). Let f : [a, b] → R be a continuous function that is differentiable at every point of (a, b). The function f attains its maximum and minimum in [a, b]. If x0 ∈ (a, b) is a number at which the extremum is attained (i.e., f (x0 ) is the maximum or minimum), then f ′ (x0 ) = 0. Theorem 2.44 (Rolle’s theorem). Let f (x) be a continuous function defined on [a, b], where a, b ∈ R, a < b, and f (a) = f (b). If f is differentiable in (a, b), then there exists c ∈ (a, b) such that f ′ (c) = 0. Definition 2.45. Differentiable functions f1 , f2 , . . . , fk defined on an open subset D of Rn are independent if there is no nonzero differentiable function F : Rk → R such that F( f1 , . . . , fk ) is identically zero on some open subset of D. Theorem 2.46. Functions f1 , . . . , fk : D → R are independent if and only if the k × n matrix [∂ fi /∂ x j ]i, j is of rank k, i.e., when its k rows are linearly independent at some point. 12 2 Basic Concepts and Facts Theorem 2.47 (Lagrange multipliers). Let D be an open subset of Rn and f , f1 , f2 , . . . , fk : D → R independent differentiable functions. Assume that a point a in D is an extremum of the function f within the set of points in D for which f1 = f2 = · · · = fk = 0. Then there exist real numbers λ1 , . . . , λk (so-called Lagrange multipliers) such that a is a stationary point of the function F = f + λ1 f1 + · · · + λk fk , i.e., such that all partial derivatives of F at a are zero. Definition 2.48. Let f be a real function defined on [a, b] and let a = x0 ≤ x1 ≤ · · · ≤ xn = b and ξk ∈ [xk−1 , xk ]. The sum S = ∑nk=1 (xk − xk−1 ) f (ξk ) is called a Darboux sum. If I = limδ →0 S exists (where δ = maxk (xk − xk−1 )), we say that f is integrable and that I is its integral. Every continuous function is integrable on a finite interval. 2.3 Geometry 2.3.1 Triangle Geometry Definition 2.49. The orthocenter of a triangle is the common point of its three altitudes. Definition 2.50. The circumcenter of a triangle is the center of its circumscribed circle (i.e., circumcircle). It is the common point of the perpendicular bisectors of the sides of the triangle. Definition 2.51. The incenter of a triangle is the center of its inscribed circle (i.e., incircle). It is the common point of the internal bisectors of its angles. Definition 2.52. The centroid of a triangle (median point) is the common point of its medians. Theorem 2.53. The orthocenter, circumcenter, incenter, and centroid are well defined (and unique) for every nondegenerate triangle. Theorem 2.54 (Euler’s line). The orthocenter H, centroid G, and circumcenter O −→ −→ of an arbitrary triangle lie on a line and satisfy HG = 2GO. Theorem 2.55 (The nine-point circle). The feet of the altitudes from A, B, C and the midpoints of AB, BC, CA, AH, BH, CH lie on a circle. Theorem 2.56 (Feuerbach’s theorem). The nine-point circle of a triangle is tangent to the incircle and all three excircles of the triangle. Theorem 2.57 (Torricelli’s point). Given a triangle △ABC, let △ABC′ , △AB′C, and △A′ BC be equilateral triangles constructed outward. Then AA′ , BB′ , CC′ intersect in one point. Definition 2.58. Let ABC be a triangle, P a point, and X , Y , Z respectively the feet of the perpendiculars from P to BC, AC, AB. Triangle XY Z is called the pedal triangle of △ABC corresponding to point P. 2.3 Geometry 13 Theorem 2.59 (Simson’s line). The pedal triangle XY Z is degenerate, i.e., X , Y , Z are collinear, if and only if P lies on the circumcircle of ABC. Points X , Y , Z are in this case said to lie on Simson’s line. Theorem 2.60. If M is a point on the circumcircle of △ABC with orthocenter H, then the Simson’s line corresponding to M bisects the segment MH. Theorem 2.61 (Carnot’s theorem). The perpendiculars from X ,Y, Z to BC,CA, AB respectively are concurrent if and only if BX 2 − XC2 + CY 2 − YA2 + AZ 2 − ZB2 = 0. Theorem 2.62 (Desargues’s theorem). Let A1 B1C1 and A2 B2C2 be two triangles. The lines A1 A2 , B1 B2 , C1C2 are concurrent or mutually parallel if and only if the points A = B1C1 ∩ B2C2 , B = C1 A1 ∩C2 A2 , and C = A1 B1 ∩ A2 B2 are collinear. Definition 2.63. Given a point C in the plane and a real number r, a homothety with center C and coefficient r is a mapping of the plane that sends each point A to the −→ − → point A′ such that CA′ = kCA. Theorem 2.64. Let k1 , k2 , and k3 be three circles. Then the three external similitude centers of these three circles are collinear (the external similitude center is the center of the homothety with positive coefficient that maps one circle to the other). Similarly, two internal similitude centers are collinear with the third external similitude center. All variants of the previous theorem can be directly obtained from the Desargues’s theorem applied to the following two triangles: the first triangle is determined by the centers of k1 , k2 , k3 , while the second triangle is determined by the points of tangency of an appropriately chosen circle that is tangent to all three of k1 , k2 , k3 . 2.3.2 Vectors in Geometry → − → Definition 2.65. For any two vectors − a , b in space, we define the scalar product → − → − → − → → → (also known as dot product) of − a and b as − a · b = |− a || b | cos ϕ , and the vector → → − → − → → product (also known as cross product) as − a × b =− p , where ϕ = ∠(− a , b ) and → − → − → → p is the vector with |− p | = |− a || b || sin ϕ | perpendicular to the plane determined by → − →→ − → − → a and b such that the triple of vectors − a , b ,− p is positively oriented (note that → − → − → − → → if − a and b are collinear, then − a × b = 0 ). Both these products are linear with respect to both factors. The scalar product is commutative, while the vector product is → − → → − → anticommutative, i.e., − a × b = − b ×− a . We also define the mixed vector product → − → − → − − → − → − → − → − → of three vectors a , b , c as [ a , b , c ] = (− a × b )·→ c. → − → − − → Remark. The scalar product of vectors → a and b is often denoted by h− a , b i. Theorem 2.66 (Thales’ theorem). Let lines AA′ and BB′ intersect in a point O, A′ 6= − → −→ → − OB a O 6= B′ . Then AB k A′ B′ ⇔ −OA −→′ = −−→′ (Here − → denotes the ratio of two nonzero b OA OB collinear vectors). 14 2 Basic Concepts and Facts Theorem 2.67 (Ceva’s theorem). Let ABC be a triangle and X , Y , Z points on lines BC, CA, AB respectively, distinct from A, B,C. Then the lines AX , BY , CZ are concurrent if and only if → − → −→ − BX CY AZ sin ∡BAX sin ∡CBY sin ∡ACZ · · −→ − → − → = 1, or equivalently, sin ∡X AC sin ∡Y BA sin ∡ZCB = 1 XC YA ZB (the last expression being called the trigonometric form of Ceva’s theorem). Theorem 2.68 (Menelaus’s theorem). Using the notation introduced for Ceva’s theorem, points X,Y, Z are collinear if and only if → − → −→ − BX CY AZ −→ · − →·− → = −1. XC YA ZB Theorem 2.69 (Stewart’s theorem). If D is an arbitrary point on the line BC, then −→ −→ DC 2 BD 2 −→ −→ AD = − → BD + − → CD − BD · DC. BC BC 2 Specifically, if D is the midpoint of BC, then 4AD2 = 2AB2 + 2AC2 − BC2 . 2.3.3 Barycenters Definition 2.70. A mass point (A, m) is a point A that is assigned a mass m > 0. Definition 2.71. The center of mass (barycenter) of the set of mass points (Ai , mi ), −→ − → i = 1, 2, . . . , n, is the point T such that ∑i mi TAi = 0 . Theorem 2.72 (Leibniz’s theorem). Let T be the mass center of the set of mass points {(Ai , mi ) | i = 1, 2, . . . , n} of total mass m = m1 + · · · + mn , and let X be an arbitrary point. Then n n i=1 i=1 ∑ mi X A2i = ∑ mi TA2i + mX T 2 . Specifically, if T is the centroid of △ABC and X an arbitrary point, then AX 2 + BX 2 +CX 2 = AT 2 + BT 2 +CT 2 + 3X T 2 . 2.3.4 Quadrilaterals Theorem 2.73. A quadrilateral ABCD is cyclic (i.e., there exists a circumcircle of ABCD) if and only if ∠ACB = ∠ADB and if and only if ∠ADC + ∠ABC = 180◦ . 2.3 Geometry 15 Theorem 2.74 (Ptolemy’s theorem). A convex quadrilateral ABCD is cyclic if and only if AC · BD = AB ·CD + AD · BC. For an arbitrary quadrilateral ABCD we have Ptolemy’s inequality (see 2.3.7, Geometric Inequalities). Theorem 2.75 (Casey’s theorem). Let k1 , k2 , k3 , and k4 be four circles that all touch a given circle k. Let ti j be the length of a segment determined by an external common tangent of circles ki and k j (i, j ∈ {1, 2, 3, 4}) if both ki and k j touch k internally, or both touch k externally. Otherwise, ti j is set to be the internal common tangent. Then one of the products t12t34 , t13t24 , and t14t23 is the sum of the other two. Some of the circles k1 , k2 , k3 , k4 may be degenerate, i.e., of 0 radius, and thus reduced to being points. In particular, for three points A, B, C on a circle k and a circle k′ touching k at a point on the arc of AC not containing B, we have AC · b = AB · c + a · BC, where a, b, and c are the lengths of the tangent segments from points A, B, and C to k′ . Ptolemy’s theorem is a special case of Casey’s theorem when all four circles are degenerate. Theorem 2.76. A convex quadrilateral ABCD is tangent (i.e., there exists an incircle of ABCD) if and only if AB +CD = BC + DA. Theorem 2.77. For arbitrary points A, B,C, D in space, AC ⊥ BD if and only if AB2 +CD2 = BC2 + DA2. Theorem 2.78 (Newton’s theorem). Let ABCD be a quadrilateral, AD ∩ BC = E, and AB ∩ DC = F (such points A, B,C, D, E, F form a complete quadrilateral). Then the midpoints of AC, BD, and EF are collinear. If ABCD is tangent, then the incenter also lies on this line. Theorem 2.79 (Brocard’s theorem). Let ABCD be a quadrilateral inscribed in a circle with center O, and let P = AB ∩ CD, Q = AD ∩ BC, R = AC ∩ BD. Then O is the orthocenter of △PQR. 2.3.5 Circle Geometry Theorem 2.80 (Pascal’s theorem). If A1 , A2 , A3 , B1 , B2 , B3 are distinct points on a conic γ (e.g., circle), then points X1 = A2 B3 ∩ A3 B2 , X2 = A1 B3 ∩ A3 B1 , and X3 = A1 B2 ∩ A2 B1 are collinear. The special result when γ consists of two lines is called Pappus’s theorem. Theorem 2.81 (Brianchon’s theorem). Let ABCDEF be a convex hexagon. If a conic (e.g., circle) can be inscribed in ABCDEF, then AD, BE, and CF meet in a point. 16 2 Basic Concepts and Facts Theorem 2.82 (The butterfly theorem). Let AB be a chord of a circle k and C its midpoint. Let p and q be two different lines through C that, respectively, intersect k on one side of AB in P and Q and on the other in P′ and Q′ . Let E and F respectively be the intersections of PQ′ and P′ Q with AB. Then it follows that CE = CF. Definition 2.83. The power of a point X with respect to a circle k(O, r) is defined by P(X ) = OX 2 − r2 . For an arbitrary line l through X that intersects k at A and B −→ −→ (A = B when l is a tangent), it follows that P(X) = X A · X B. Definition 2.84. The radical axis of two circles is the locus of points that have equal powers with respect to both circles. The radical axis of circles k1 (O1 , r1 ) and k2 (O2 , r2 ) is a line perpendicular to O1 O2 . The radical axes of three distinct circles are concurrent or mutually parallel. If concurrent, the intersection of the three axes is called the radical center. Definition 2.85. The pole of a line l 6∋ O with respect to a circle k(O, r) is a point A on the other side of l from O such that OA ⊥ l and d(O, l) · OA = r2 . In particular, if l intersects k in two points, its pole will be the intersection of the tangents to k at these two points. Definition 2.86. The polar of the point A from the previous definition is the line l. In particular, if A is a point outside k and AM, AN are tangents to k (M, N ∈ k), then MN is the polar of A. Poles and polars are generally defined in a similar way with respect to arbitrary nondegenerate conics. Theorem 2.87. If A belongs to the polar of B, then B belongs to the polar of A. 2.3.6 Inversion Definition 2.88. An inversion of the plane π about the circle k(O, r) (which belongs to π ) is a transformation of the set π \{O} onto itself such that every point P is transformed into a point P′ on the ray (OP such that OP · OP′ = r2 . In the following statements we implicitly assume exclusion of O. Theorem 2.89. The fixed points of an inversion about a circle k are on the circle k. The inside of k is transformed into the outside and vice versa. Theorem 2.90. If A, B transform into A′ , B′ after an inversion about a circle k, then ∠OAB = ∠OB′ A′ , and also ABB′ A′ is cyclic and perpendicular to k. A circle perpendicular to k transforms into itself. Inversion preserves angles between continuous curves (which includes lines and circles). Theorem 2.91. An inversion transforms lines not containing O into circles containing O, lines containing O into themselves, circles not containing O into circles not containing O, circles containing O into lines not containing O. 2.3 Geometry 17 2.3.7 Geometric Inequalities Theorem 2.92 (The triangle inequality). For any three points A, B, C, AB + BC ≥ AC. Equality occurs when A, B, C are collinear and B is between A and C. In the sequel we will use B(A, B,C) to emphasize that B is between A and C. Theorem 2.93 (Ptolemy’s inequality). For any four points A, B, C, D, AC · BD ≤ AB ·CD + AD · BC. Theorem 2.94 (The parallelogram inequality). For any four points A, B, C, D, AB2 + BC2 + CD2 + DA2 ≥ AC2 + BD2 . Equality occurs if and only if ABCD is a parallelogram. Theorem 2.95. For a given triangle △ABC the point X for which AX + BX + CX is minimal is Toricelli’s point when all angles of △ABC are less than or equal to 120◦ , and is the vertex of the obtuse angle otherwise. The point X2 for which AX22 + BX22 + CX22 is minimal is the centroid (see Leibniz’s theorem). Theorem 2.96 (The Erdős–Mordell inequality). Let P be a point in the interior of △ABC and X,Y, Z projections of P onto BC, AC, AB, respectively. Then PA + PB + PC ≥ 2(PX + PY + PZ). Equality holds if and only if △ABC is equilateral and P is its center. 2.3.8 Trigonometry Definition 2.97. The trigonometric circle is the unit circle centered at the origin O of a coordinate plane. Let A be the point (1, 0) and P(x, y) a point on the trigonometric circle such that ∡AOP = α . We define sin α = y, cos α = x, tan α = y/x, and cot α = x/y. Theorem 2.98. The functions sin and cos are periodic with period 2π . The functions tan and cot are periodic with period π . The following simple identities hold: sin2 x + cos2 x = 1,√sin 0 = sin π = 0, sin(−x) = − sin x, cos(−x) = cos x, sin(π /2) = 1, sin(π /4) = 1/ 2, sin(π /6) = 1/2, cos x = sin(π /2 − x). From these identities other identities can be easily derived. Theorem 2.99. Additive formulas for trigonometric functions: sin(α ± β ) = sin α cos β ± cos α sin β , cos(α ± β ) = cos α cos β ∓ sin α sin β , tan α ±tan β α cot β ∓1 tan(α ± β ) = 1∓tan cot(α ± β ) = cot α tan β , cot α ±cot β . 18 2 Basic Concepts and Facts Theorem 2.100. Formulas for trigonometric functions of 2x and 3x: sin 2x = 2 sin x cos x, cos 2x = 2 cos2 x − 1, 2 tan x tan 2x = 1−tan 2x, Theorem 2.101. For any x ∈ R, sin x = sin 3x = 3 sin x − 4 sin3 x, cos 3x = 4 cos3 x − 3 cosx, x−tan3 x tan 3x = 3 tan . 1−3 tan2 x 2t 1+t 2 and cos x = 1−t 2 , 1+t 2 where t = tan 2x . Theorem 2.102. Transformations from product to sum: 2 cos α cos β = cos(α + β ) + cos(α − β ), 2 sin α cos β = sin(α + β ) + sin(α − β ), 2 sin α sin β = cos(α − β ) − cos(α + β ). Theorem 2.103. The angles α , β , γ of a triangle satisfy cos2 α + cos2 β + cos2 γ + 2 cos α cos β cos γ = 1, tan α + tan β + tan γ = tan α tan β tan γ . Theorem 2.104 (De Moivre’s formula). If i2 = −1, then (cosx + i sin x)n = cos nx + i sin nx. 2.3.9 Formulas in Geometry Theorem 2.105 (Heron’s formula). The area of a triangle ABC with sides a, b, c and semiperimeter s is given by S= p 1p 2 2 s(s − a)(s − b)(s − c) = 2a b + 2a2c2 + 2b2 c2 − a4 − b4 − c4 . 4 Theorem 2.106 (The law of sines). The sides a, b, c and angles α , β , γ of a triangle ABC satisfy a b c = = = 2R, sin α sin β sin γ where R is the circumradius of △ABC. Theorem 2.107 (The law of cosines). The sides and angles of △ABC satisfy c2 = a2 + b2 − 2ab cos γ . Theorem 2.108. The circumradius R and inradius r of a triangle ABC satisfy R = abc 2S 4S and r = a+b+c = R(cos α + cos β + cos γ − 1). If x, y, z denote the distances of the circumcenter in an acute triangle to the sides, then x + y + z = R + r. Theorem 2.109 (Euler’s formula). If O and I are the circumcenter and incenter of △ABC, then OI 2 = R(R − 2r), where R and r are respectively the circumradius and the inradius of △ABC. Consequently, R ≥ 2r. 2.4 Number Theory 19 Theorem 2.110. If a, b, c, d are lengths of the sides of a convex quadrilateral, p its semiperimeter, and α and γ two non-adjacent angles of the quadrilateral, then its area S is given by r α +γ S = (p − a)(p − b)(p − c)(p − d) − abcd cos2 . 2 If the quadrilateral is cyclic, the above formula reduces to p S = (p − a)(p − b)(p − c)(p − d). Theorem 2.111 (Euler’s theorem for pedal triangles). Let X,Y, Z be the feet of the perpendiculars from a point P to the sides of a triangle ABC. Let O denote the circumcenter and R the circumradius of △ABC. Then SXY Z = 1 OP2 1 − 2 SABC . 4 R Moreover, SXY Z = 0 if and only if P lies on the circumcircle of △ABC (see Simson’s line). → − → → Theorem 2.112. If − a = (a1 , a2 , a3 ), b = (b1 , b2 , b3 ), − c = (c1 , c2 , c3 ) are three vectors in coordinate space, then → − − → a · b = a1 b1 + a2 b2 + a3 b3 , → − − → a × b = (a1 b2 − a2 b1 , a2 b3 − a3 b2 , a3 b1 − a1 b3 ),   a 1 a2 a3 →− − → − → [ a , b , c ] = det  b1 b2 b3  . c1 c2 c3 Here detM denotes the determinant of the square matrix M. Theorem 2.113. The area of a triangle ABC and the volume of a tetrahedron ABCD h→ − → − → → −→i 1 − 1 − are equal to 2 |AB × AC| and 6 AB, AC, AD , respectively. Theorem 2.114 (Cavalieri’s principle). If the sections of two solids by the same plane always have equal area, then the volumes of the two solids are equal. 2.4 Number Theory 2.4.1 Divisibility and Congruences Definition 2.115. The greatest common divisor (a, b) = gcd(a, b) of a, b ∈ N is the largest positive integer that divides both a and b. Positive integers a and b are coprime or relatively prime if (a, b) = 1. The least common multiple [a, b] = lcm(a, b) of a, b ∈ N is the smallest positive integer that is divisible by both a and b. It holds that [a, b](a, b) = ab. The above concepts are easily generalized to more than two numbers; i.e., we also define (a1 , a2 , . . . , an ) and [a1 , a2 , . . . , an ]. 20 2 Basic Concepts and Facts Theorem 2.116 (Euclidean algorithm). Since (a, b) = (|a − b|, a) = (|a − b|, b), it follows that starting from positive integers a and b one eventually obtains (a, b) by repeatedly replacing a and b with |a − b| and min{a, b} until the two numbers are equal. The algorithm can be generalized to more than two numbers. Theorem 2.117 (Corollary to Euclidean algorithm). For each a, b ∈ N there exist x, y ∈ Z such that ax + by = (a, b). The number (a, b) is the smallest positive number for which such x and y can be found. Theorem 2.118 (Second corollary to Euclid’s algorithm). For a, m, n ∈ N and a > 1 it follows that (am − 1, an − 1) = a(m,n) − 1. Theorem 2.119 (Fundamental theorem of arithmetic). Every positive integer can be uniquely represented as a product of primes, up to their order. Theorem 2.120. The fundamental theorem also holds in some other √ of arithmetic √ rings, such as Z[i] = {a + bi | a, b ∈ Z}, Z[ 2], Z[ −2], Z[ω ] (where ω is a complex third root of 1). In these cases, the factorization into primes is unique up to the order and divisors of 1. Definition 2.121. Integers a, b are congruent modulo n ∈ N if n | a − b. We then write a ≡ b (mod n). Theorem 2.122 (Chinese remainder theorem). If m1 , m2 , . . . , mk are positive integers pairwise relatively prime and a1 , . . . , ak , c1 , . . . , ck are integers such that (ai , mi ) = 1 (i = 1, . . . , k), then the system of congruences ai x ≡ ci (mod mi ), i = 1, 2, . . . , k , has a unique solution modulo m1 m2 · · · mk . 2.4.2 Exponential Congruences Theorem 2.123 (Wilson’s theorem). If p is a prime, then p | (p − 1)! + 1. Theorem 2.124 (Fermat’s (little) theorem). Let p be a prime number and a an integer with (a, p) = 1. Then a p−1 ≡ 1 (mod p). This theorem is a special case of Euler’s theorem. Definition 2.125. Euler’s function ϕ (n) is defined for n ∈ N as the number of positive integers less than or equal to n and coprime to n. It holds that     1 1 ϕ (n) = n 1 − ··· 1 − , p1 pk α α where n = p1 1 · · · pk k is the factorization of n into primes. Theorem 2.126 (Euler’s theorem). Let n be a natural number and a an integer with (a, n) = 1. Then aϕ (n) ≡ 1 (mod n). 2.4 Number Theory 21 Theorem 2.127 (Existence of primitive roots). Let p be a prime number. There exists g ∈ {1, 2, . . ., p − 1} (called a primitive root modulo p) such that the set {1, g, g2 , . . . , g p−2 } is equal to {1, 2, . . . , p − 1} modulo p. Definition 2.128. Let p be a prime and α a nonnegative integer. We say that pα is the exact power of p that divides an integer a (and α the exact exponent) if pα | a and pα +1 ∤ a. Theorem 2.129. Let a and n be positive integers and p an odd prime. If pα (α ∈ N) is the exact power of p that divides a − 1, then for any integer β ≥ 0, pα +β | an − 1 if and only if pβ | n. (See (SL97-14).) A similar statement holds for p = 2. If 2α (α ∈ N) is the exact power of 2 that divides a2 − 1, then for any integer β ≥ 0, 2α +β | an − 1 if and only if 2β +1 | n. (See (SL89-27).) 2.4.3 Quadratic Diophantine Equations Theorem 2.130. The solutions of a2 +b2 = c2 in integers are given by a = t(m2 −n2 ), b = 2tmn, c = t(m2 + n2 ) (provided that b is even), where t, m, n ∈ Z. The triples (a, b, c) are called Pythagorean (or primitive Pythagorean if gcd(a, b, c) = 1). Definition 2.131. Given D ∈ N that is not a perfect square, a Pell’s equation is an equation of the form x2 − Dy2 = 1, where x, y ∈ Z. Theorem 2.132. If (x0 , y0 ) is the least (nontrivial) solution in N of the Pell’s equation √ x2 − Dy2 = all the nontrivial integer solutions (x, y) are given by x + y D = √1, then ±(x0 + y0 D)n , where n ∈ Z. Definition 2.133. An integer a is a quadratic residue modulo a prime p if there exists x ∈ Z such that x2 ≡ a (mod p). Otherwise, a is a quadratic nonresidue modulo p. Definition 2.134. The Legendre symbol for an integer a and a prime p is defined by     1 if a is a quadratic residue mod p and p ∤ a; a 0 if p | a; =  p −1 otherwise.      2 a Clearly ap = a+p and p = 1 if p ∤ a. The Legendre symbol is multiplicative,     p  b ab i.e., ap p = p . Theorem 2.135 (Euler’s criterion). For each odd prime p and integer a not divisible   p−1 by p, a 2 ≡ ap (mod p). Theorem 2.136. For a prime p > 3,  −1 p      , 2p , and −3 are equal to 1 if and p only if p ≡ 1 (mod 4), p ≡ ±1 (mod 8) and p ≡ 1 (mod 6), respectively. 22 2 Basic Concepts and Facts Theorem 2.137 (Gauss’s reciprocity law). For any two distinct odd primes p and q, we have that    p−1 q−1 p q = (−1) 2 · 2 . q p Definition 2.138. Jacobi symbol for an integer a and an odd positive integer b is defined as  a   a α1  a αk = ··· , b p1 pk α where b = pα1 1 · · · pk k is the factorization of b into primes.  Theorem 2.139. If ab = −1, then a is a quadratic nonresidue modulo b, but the converse is false. All the above identities for Legendre symbols except Euler’s criterion remain true for Jacobi symbols. 2.4.4 Farey Sequences Definition 2.140. For any positive integer n, the Farey sequence Fn is the sequence of rational numbers a/b with 0 ≤ a ≤ b ≤ n and (a, b) = 1 arranged in increasing order. For instance, F3 = { 01 , 13 , 12 , 23 , 11 }. Theorem 2.141. If p1 /q1 , p2 /q2 , and p3 /q3 are three successive terms in a Farey sequence, then p1 + p3 p2 p2 q1 − p1 q2 = 1 and = . q1 + q3 q2 2.5 Combinatorics 2.5.1 Counting of Objects Many combinatorial problems involving the counting of objects satisfying a given set of properties can be properly reduced to an application of one of the following concepts. Definition 2.142. A variation of order n over k is a 1–to–1 mapping of {1, 2, . . . , k} into {1, 2, . . . , n}. For a given n and k, where n ≥ k, the number of different variations n! is Vnk = (n−k)! . Definition 2.143. A variation with repetition of order n over k is an arbitrary mapping of {1, 2, . . .,k} into {1, 2, . . ., n}. For a given n and k the number of different k variations with repetition is V n = kn . Definition 2.144. A permutation of order n is a bijection of {1, 2, . . ., n} into itself (a special case of variation for k = n). For a given n the number of different permutations is Pn = n!. 2.5 Combinatorics 23 Definition 2.145. A combination of order n over k is a k-element subset of  {1, 2, . . . , n}. For a given n and k the number of different combinations is Cnk = nk . Definition 2.146. A permutation with repetition of order n is a bijection of {1, 2, . . . , n} into a multiset of n elements. A multiset is defined to be a set in which certain elements are deemed mutually indistinguishable (for example, as in {1, 1, 2, 3}). If {k1 , k2 , . . . , ks } denotes the set of distinct elements in a multiset and the element ki appears αi times in the multiset, then number of different permutations with repetition is Pn,α1 ,...,αs = α1 !·αn! . A combination is a special case of permutation 2 !···αs ! with repetition for a multiset with two different elements. Theorem 2.147 (The pigeonhole principle). If a set of nk + 1 different elements is partitioned into n mutually disjoint subsets, then at least one subset will contain at least k + 1 elements. Theorem 2.148 (The inclusion–exclusion principle). Let S1 , S2 , . . . , Sn be a family of subsets of the set S. The number of elements of S contained in none of the subsets is given by the formula n |S\(S1 ∪ · · · ∪ Sn )| = |S| − ∑ ∑ (−1)k−1 |Si1 ∩ · · · ∩ Sik | . k=1 1≤i1 <···<ik ≤n 2.5.2 Graph Theory Definition 2.149. A graph G = (V, E) is a set of objects, i.e., vertices, V paired with the multiset E of some pairs of elements of V , i.e., edges. When (x, y) ∈ E, for x, y ∈ V , the vertices x and y are said to be connected by an edge; i.e., the vertices are the endpoints of the edge. A graph for which the multiset E reduces to a proper set (i.e., each pair of vertices are connected by at most one edge) and for which no vertex is connected to itself is called a simple graph. A finite graph is one in which |E| and |V | are finite. Definition 2.150. An oriented graph is one in which the pairs in E are ordered. Definition 2.151. The simple graph Kn consisting of n vertices and in which each pair of vertices is connected is called a complete graph. Definition 2.152. A k-partite graph (bipartite for k = 2) Ki1 ,i2 ,...,ik is a graph whose set of vertices V can be partitioned into k nonempty disjoint subsets of cardinalities i1 , i2 , . . . , ik such that each vertex x in a subset W of V is connected only with the vertices not in W . Definition 2.153. Given a bipartite graph (V, E), let W and M be a partition of its set of vertices (you can think of W as a set of women and M a set of men). Assume that |W | ≤ |M|. A marriage is an injective map f : W → M for which (w, f (w)) ∈ E for every w ∈ W . 24 2 Basic Concepts and Facts Theorem 2.154 (Hall’s marriage theorem). Let W , M be a partition of the set of vertices of a bipartite graph. There exists a marriage f : W → M if and only if for every U ⊆ W the number |U| is not greater than the total number of neighbors of U inside M. Definition 2.155. The degree d(x) of a vertex x is the number of times x is the endpoint of an edge (thus, self-connecting edges are counted twice for corresponding vertices). An isolated vertex is one with degree 0. Theorem 2.156. For a graph G = (V, E) the following identity holds: ∑ d(x) = 2|E|. x∈V As a consequence, the number of vertices of odd degree is even. Definition 2.157. A trajectory (path) of a graph is a finite sequence of vertices, each connected to the previous one. The length of a trajectory is the number of edges through which it passes. A circuit is a path that ends in the starting vertex. A cycle is a circuit in which no vertex appears more than once (except the initial/final vertex). A graph is connected if there exists a trajectory between any two vertices. Definition 2.158. A subgraph G′ = (V ′ , E ′ ) of a graph G = (V, E) is a graph such that V ′ ⊆ V and E ′ contains exactly the edges of E connecting points in V ′ . A connected component of a graph is a connected subgraph such that no vertex of the subgraph is connected with any vertex outside of the subgraph. Definition 2.159. A tree is a connected graph that contains no cycles. Theorem 2.160. A tree with n vertices has exactly n − 1 edges and at least two vertices of degree 1. Definition 2.161. An Euler path is a path in which each edge appears exactly once. Likewise, an Euler circuit is an Euler path that is also a circuit. Theorem 2.162. The following conditions are necessary and sufficient for a finite connected graph G to have an Euler path: • The graph contains an Euler circuit if and only if each vertex has even degree. • The graph contains an Euler path if and only if the number of vertices of odd degree is either 0 or 2 (in the latter case the path starts and ends in the two odd vertices). Definition 2.163. A Hamiltonian circuit is a circuit that contains each vertex of G exactly once (trivially, it is also a cycle). A simple rule to determine whether a graph contains a Hamiltonian circuit has not yet been discovered. 2.5 Combinatorics 25 Theorem 2.164 (Ore’s theorem). Let G be a graph with n vertices. If the sum of the degrees of any two nonadjacent vertices in G is greater than or equal to n, then G has a Hamiltonian circuit. Theorem 2.165 (Ramsey’s theorem). Let r ≥ 1 and q1 , q2 , . . . , qs ≥ r. There exists a minimal positive integer N(q1 , q2 , . . . , qs ; r) such that for n ≥ N, if all subgraphs Kr of Kn are partitioned into s different sets, labeled A1 , A2 . . . ,As , then for some i there exists a complete subgraph Kqi whose subgraphs Kr all belong to Ai . For r = 2 this corresponds to coloring the edges of Kn with s different colors and looking for a monochromatic subgraph Kqi in color i. Theorem 2.166. N(p, q; r) ≤ N(N(p − 1, q; r), N(p, q − 1;r); r − 1) + 1, and in particular, N(p, q; 2) ≤ N(p − 1, q; 2) + N(p, q − 1; 2). The following values of N are known: N(p, q; 1) = p + q − 1, N(2, p; 2) = p, N(3, 3; 2) = 6, N(3, 4; 2) = 9, N(3, 5; 2) = 14, N(3, 6; 2) = 18, N(3, 7; 2) = 23, N(3, 8; 2) = 28, N(3, 9; 2) = 36, N(4, 4; 2) = 18, N(4, 5; 2) = 25. Theorem 2.167 (Turán’s theorem). If a simple graph on n = t(p − 1) + r vertices 2 −r(p−1−r) (0 ≤ r < p − 1) has more than f (n, p) = (p−2)n2(p−1) edges, then it contains K p as a subgraph. The graph containing f (n, p) edges that does not contain Kp is the complete multipartite graph with r parts with t + 1 vertices, and p − 1 − r parts with t vertices. Definition 2.168. A planar graph is one that can be embedded in a plane such that its vertices are represented by points and its edges by lines (not necessarily straight) connecting the vertices such that no two edges intersect each other. Theorem 2.169. A planar graph with n vertices has at most 3n − 6 edges. Theorem 2.170 (Kuratowski’s theorem). Graphs K5 and K3,3 are not planar. Every nonplanar graph contains a subgraph that can be obtained from one of these two graphs by a subdivison of its edges. Theorem 2.171 (Euler’s formula). For a given convex polyhedron let E be the number of its edges, F the number of faces, and V the number of vertices. Then E + 2 = F + V . The same formula holds for a connected planar graph (F is in this case equal to the number of planar regions). 3 Problems 3.1 The First IMO Bucharest–Brasov, Romania, July 23–31, 1959 3.1.1 Contest Problems First Day 1. (POL) For every integer n prove that the fraction further. 21n+4 14n+3 cannot be reduced any 2. (ROU) For which real numbers x do the following equations hold: p p √ √ √ (a) px + 2x − 1 + px − 2x − 1 = 2 , √ √ (b) px + 2x − 1 + px − 2x − 1 = 1 , √ √ (c) x + 2x − 1 + x − 2x − 1 = 2 ? 3. (HUN) Let x be an angle and let the real numbers a, b, c, cosx satisfy the following equation: a cos2 x + b cosx + c = 0 . Write the analogous quadratic equation for a, b, c, cos 2x. Compare the given and the obtained equality for a = 4, b = 2, c = −1. Second Day 4. (HUN) Construct a right-angled triangle whose hypotenuse c is given if it is known that the median from the right angle equals the geometric mean of the remaining two sides of the triangle. 5. (ROU) A segment AB is given and on it a point M. On the same side of AB squares AMKD and BMFE are constructed. The circumcircles of the two squares, whose centers are P and Q, intersect in M and another point N. (a) Prove that lines FA and BC intersect at N. D. Djukić et al., The IMO Compendium, Problem Books in Mathematics, DOI 10.1007/978-1-4419-9854-5_3, © Springer Science + Business Media, LLC 2011 27 28 3 Problems (b) Prove that all such constructed lines MN pass through the same point S, regardless of the selection of M. (c) Find the locus of the midpoints of all segments PQ, as M varies along the segment AB. 6. (CZS) Let α and β be two planes intersecting at a line p. In α a point A is given and in β a point C is given, neither of which lies on p. Construct B in α and D in β such that ABCD is an equilateral trapezoid, AB k CD, in which a circle can be inscribed. 3.2 IMO 1960 29 3.2 The Second IMO Bucharest–Sinaia, Romania, July 18–25, 1960 3.2.1 Contest Problems First Day 1. (BGR) Find all the three-digit numbers for which one obtains, when dividing the number by 11, the sum of the squares of the digits of the initial number. 2. (HUN) For which real numbers x does the following inequality hold: 4x2 √ < 2x + 9 ? (1 − 1 + 2x)2 3. (ROU) A right-angled triangle ABC is given for which the hypotenuse BC has length a and is divided into n equal segments, where n is odd. Let α be the angle with which the point A sees the segment containing the middle of the hypotenuse. Prove that 4nh tan α = 2 , (n − 1)a where h is the height of the triangle. Second Day 4. (HUN) Construct a triangle ABC whose lengths of heights ha and hb (from A and B, respectively) and length of median ma (from A) are given. 5. (CZS) A cube ABCDA′ B′C′ D′ is given. (a) Find the locus of all midpoints of segments XY , where X is any point on segment AC and Y any point on segment B′ D′ . − → −→ (b) Find the locus of all points Z on segments XY such that ZY = 2X Z. 6. (BGR) An isosceles trapezoid with bases a and b and height h is given. (a) On the line of symmetry construct the point P such that both (nonbase) sides are seen from P with an angle of 90◦ . (b) Find the distance of P from one of the bases of the trapezoid. (c) Under what conditions for a, b, and h can the point P be constructed (analyze all possible cases)? 7. (GDR) A sphere is inscribed in a regular cone. Around the sphere a cylinder is circumscribed so that its base is in the same plane as the base of the cone. Let V1 be the volume of the cone and V2 the volume of the cylinder. (a) Prove that V1 = V2 is impossible. (b) Find the smallest k for which V1 = kV2 , and in this case construct the angle at the vertex of the cone. 30 3 Problems 3.3 The Third IMO Budapest–Veszprem, Hungary, July 6–16, 1961 3.3.1 Contest Problems First Day 1. (HUN) Solve the following system of equations: x + y + z = a, 2 x + y2 + z2 = b2 , xy = z2 , where a and b are given real numbers. What conditions must hold on a and b for the solutions to be positive and distinct? 2. (POL) Let a, b, and c be the lengths of a triangle whose area is S. Prove that √ a2 + b2 + c2 ≥ 4S 3 . In what case does equality hold? 3. (BGR) Solve the equation cosn x − sinn x = 1, where n is a given positive integer. Second Day 4. (GDR) In the interior of △P1 P2 P3 a point P is given. Let Q1 , Q2 , and Q3 respectively be the intersections of PP1 , PP2 , and PP3 with the opposing edges of △P1 P2 P3 . Prove that among the ratios PP1 /PQ1 , PP2 /PQ2 , and PP3 /PQ3 there exists at least one not larger than 2 and at least one not smaller than 2. 5. (CZS) Construct a triangle ABC if the following elements are given: AC = b, AB = c, and ∡AMB = ω (ω < 90o ), where M is the midpoint of BC. Prove that the construction has a solution if and only if b tan ω ≤c<b. 2 In what case does equality hold? 6. (ROU) A plane ε is given and on one side of the plane three noncollinear points A, B, and C such that the plane determined by them is not parallel to ε . Three arbitrary points A′ , B′ , and C′ in ε are selected. Let L, M, and N be the midpoints of AA′ , BB′ , and CC′ , and G the centroid of △LMN. Find the locus of all points obtained for G as A′ , B′ , C′ are varied (independently of each other) across ε . 3.4 IMO 1962 31 3.4 The Fourth IMO Prague–Hluboka, Czechoslovakia, July 7–15, 1962 3.4.1 Contest Problems First Day 1. (POL) Find the smallest natural number n with the following properties: (a) In decimal representation it ends with 6. (b) If we move this digit to the front of the number, we get a number 4 times larger. 2. (HUN) Find all real numbers x for which √ √ 1 3−x− x+1 > . 2 3. (CZS) A cube ABCDA′ B′C′ D′ is given. The point X is moving at a constant speed along the square ABCD in the direction from A to B. The point Y is moving with the same constant speed along the square BCC′ B′ in the direction from B′ to C′ . Initially, X and Y start out from A and B′ respectively. Find the locus of all the midpoints of XY . Second Day 4. (ROU) Solve the equation cos2 x + cos2 2x + cos2 3x = 1 . 5. (BGR) On the circle k three points A, B, and C are given. Construct the fourth point on the circle D such that one can inscribe a circle in ABCD. 6. (GDR) Let ABC be an isosceles triangle with circumradius r and inradius ρ . Prove that the distance d between the circumcenter and incenter is given by p d = r(r − 2ρ ) . 7. (USS) Prove that a tetrahedron SABC has five different spheres that touch all six lines determined by its edges if and only if it is regular. 32 3 Problems 3.5 The Fifth IMO Wroclaw, Poland, July 5–13, 1963 3.5.1 Contest Problems First Day 1. (CZS) Determine all real solutions of the equation where p is a real number. p √ x2 − p + 2 x2 − 1 = x, 2. (USS) Find the locus of points in space that are vertices of right angles of which one ray passes through a given point and the other intersects a given segment. 3. (HUN) Prove that if all the angles of a convex n-gon are equal and the lengths of consecutive edges a1 , . . . , an satisfy a1 ≥ a2 ≥ · · · ≥ an , then a1 = a2 = · · · = an . Second Day 4. (USS) Find all solutions x1 , . . . , x5 to the system of equations    x5 + x2 = yx1 ,    x1 + x3 = yx2 , x2 + x4 = yx3 ,   x3 + x5 = yx4 ,    x4 + x1 = yx5 , where y is a real parameter. 5. (GDR) Prove that cos π7 − cos 27π + cos 37π = 12 . 6. (HUN) Five students A, B, C, D, and E have taken part in a certain competition. Before the competition, two persons X and Y tried to guess the rankings. X thought that the ranking would be A, B,C, D, E; and Y thought that the ranking would be D, A, E,C, B. At the end, it was revealed that X didn’t guess correctly any rankings of the participants, and moreover, didn’t guess any of the orderings of pairs of consecutive participants. On the other hand, Y guessed the correct rankings of two participants and the correct ordering of two pairs of consecutive participants. Determine the rankings of the competition. 3.6 IMO 1964 33 3.6 The Sixth IMO Moscow, Soviet Union, June 30–July 10, 1964 3.6.1 Contest Problems First Day 1. (CZS) (a) Find all natural numbers n such that the number 2n − 1 is divisible by 7. (b) Prove that for all natural numbers n the number 2n + 1 is not divisible by 7. 2. (HUN) Denote by a, b, c the lengths of the sides of a triangle. Prove that a2 (b + c − a) + b2(c + a − b) + c2(a + b − c) ≤ 3abc. 3. (YUG) The incircle is inscribed in a triangle ABC with sides a, b, c. Three tangents to the incircle are drawn, each of which is parallel to one side of the triangle ABC. These tangents form three smaller triangles (internal to △ABC) with the sides of △ABC. In each of these triangles an incircle is inscribed. Determine the sum of areas of all four incircles. Second Day 4. (HUN) Each of 17 students talked with every other student. They all talked about three different topics. Each pair of students talked about one topic. Prove that there are three students that talked about the same topic among themselves. 5. (ROU) Five points are given in the plane. Among the lines that connect these five points, no two coincide and no two are parallel or perpendicular. Through each point we construct an altitude to each of the other lines. What is the maximal number of intersection points of these altitudes (excluding the initial five points)? 6. (POL) Given a tetrahedron ABCD, let D1 be the centroid of the triangle ABC and let A1 , B1 ,C1 be the intersection points of the lines parallel to DD1 and passing through the points A, B,C with the opposite faces of the tetrahedron. Prove that the volume of the tetrahedron ABCD is one-third the volume of the tetrahedron A1 B1C1 D1 . Does the result remain true if the point D1 is replaced with any point inside the triangle ABC? 34 3 Problems 3.7 The Seventh IMO Berlin, DR Germany, July 3–13, 1965 3.7.1 Contest Problems First Day 1. (YUG) Find all real numbers x ∈ [0, 2π ] such that √ √ √ 2 cosx ≤ | 1 + sin2x − 1 − sin 2x| ≤ 2. 2. (POL) Consider the system of equations   a11 x1 + a12x2 + a13x3 = 0, a21 x1 + a22x2 + a23x3 = 0,  a31 x1 + a32x2 + a33x3 = 0, whose coefficients satisfy the following conditions: (a) a11 , a22 , a33 are positive real numbers; (b) all other coefficients are negative; (c) in each of the equations the sum of the coefficients is positive. Prove that x1 = x2 = x3 = 0 is the only solution to the system. 3. (CZS) A tetrahedron ABCD is given. The lengths of the edges AB and CD are a and b, respectively, the distance between the lines AB and CD is d, and the angle between them is equal to ω . The tetrahedron is divided into two parts by the plane π parallel to the lines AB and CD. Calculate the ratio of the volumes of the parts if the ratio between the distances of the plane π from AB and CD is equal to k. Second Day 4. (USS) Find all sets of four real numbers x1 , x2 , x3 , x4 such that the sum of any of the numbers and the product of the other three is equal to 2. 5. (ROU) Given a triangle OAB such that ∠AOB = α < 90◦ , let M be an arbitrary point of the triangle different from O. Denote by P and Q the feet of the perpendiculars from M to OA and OB, respectively. Let H be the orthocenter of the triangle OPQ. Find the locus of points H when: (a) M belongs to the segment AB; (b) M belongs to the interior of △OAB. 6. (POL) We are given n ≥ 3 points in the plane. Let d be the maximal distance between two of the given points. Prove that the number of pairs of points whose distance is equal to d is less than or equal to n. 3.8 IMO 1966 35 3.8 The Eighth IMO Sofia, Bulgaria, July 3–13, 1966 3.8.1 Contest Problems First Day 1. (USS) Three problems A, B, and C were given on a mathematics olympiad. All 25 students solved at least one of these problems. The number of students who solved B and not A is twice the number of students who solved C and not A. The number of students who solved only A is greater by 1 than the number of students who along with A solved at least one other problem. Among the students who solved only one problem, half solved A. How many students solved only B? 2. (HUN) If a, b, and c are the sides and α , β , and γ the respective angles of the triangle for which a + b = tan 2γ (a tan α + b tan β ), prove that the triangle is isosceles. 3. (BGR) Prove that the sum of distances from the center of the circumsphere of the regular tetrahedron to its four vertices is less than the sum of distances from any other point to the four vertices. Second Day 4. (YUG) Prove the following equality: 1 1 1 1 + + + ···+ = cot x − cot2n x, sin 2x sin 4x sin 8x sin 2n x where n ∈ N and x ∈ / π Z 2k for every k ∈ N. 5. (CZS) Solve the following system of equations: |a1 − a2 |x2 + |a1 − a3 |x3 + |a1 − a4 |x4 = 1, |a2 − a1 |x1 + |a2 − a3 |x3 + |a2 − a4 |x4 = 1, |a3 − a1 |x1 + |a3 − a2 |x2 + |a3 − a4 |x4 = 1, |a4 − a1 |x1 + |a4 − a2 |x2 + |a4 − a3 |x3 = 1, where a1 , a2 , a3 , and a4 are mutually distinct real numbers. 6. (POL) Let M, K, and L be points on (AB), (BC), and (CA), respectively. Prove that the area of at least one of the three triangles △MAL, △KBM, and △LCK is less than or equal to one-fourth the area of △ABC. 3.8.2 Some Longlisted Problems 1959–1966 1. (CZS) We are given n > 3 points in the plane, no three of which lie on a line. Does there necessarily exist a circle that passes through at least three of the given points and contains none of the other given points in its interior? 36 3 Problems 2. (GDR) Given n positive real numbers a1 , a2 , . . . , an such that a1 a2 · · · an = 1, prove that (1 + a1 )(1 + a2 ) · · · (1 + an ) ≥ 2n . 3. (BGR) A regular triangular prism has height h and a base of side length a. Both bases have small holes in the centers, and the inside of the three vertical walls has a mirror surface. Light enters through the small hole in the top base, strikes each vertical wall once and leaves through the hole in the bottom. Find the angle at which the light enters and the length of its path inside the prism. 4. (POL) Five points in the plane are given, no three of which are collinear. Show that some four of them form a convex quadrilateral. 5. (USS) Prove the inequality tan π sin x π cos x + tan >1 4 sin α 4 cos α for any x, α with 0 ≤ x ≤ π /2 and π /6 < α < π /3. 6. (USS) A convex planar polygon M with perimeter l and area S is given. Let M(R) be the set of all points in space that lie a distance at most R from a point of M . Show that the volume V (R) of this set equals 4 π V (R) = π R3 + lR2 + 2SR. 3 2 7. (USS) For which arrangements of two infinite circular cylinders does their intersection lie in a plane? 8. (USS) We are given a bag of sugar, a two-pan balance, and a weight of 1 gram. How do we obtain 1 kilogram of sugar in the smallest possible number of weighings? 9. (ROU) Find x such that sin 3x cos(60◦ − 4x) + 1 = 0, sin(60◦ − 7x) − cos(30◦ + x) + m where m is a fixed real number. 10. (GDR) How many real solutions are there to the equation x = 1964 sinx − 189? 11. (CZS) Does there exist an integer z that can be written in two different ways as z = x! + y!, where x, y are natural numbers with x ≤ y? 12. (BGR) Find digits x, y, z such that the equality √ xx · · · }x − yy · · · y = zz · · · z | {z | {z } | {z } 2n n n holds for at least two values of n ∈ N, and in that case find all n for which this equality is true. 3.8 IMO 1966 37 13. (YUG) Let a1 , a2 , . . . , an be positive real numbers. Prove the inequality   n 1 ∑ ai a j ≥ 4 2 i< j 1 ∑ ai + a j i< j !2 and find the conditions on the numbers ai for equality to hold. 14. (POL) Compute the largest number of regions into which one can divide a disk by joining n points on its circumference. 15. (POL) Points A, B,C, D lie on a circle such that AB is a diameter and CD is not. If the tangents at C and D meet at P while AC and BD meet at Q, show that PQ is perpendicular to AB. 16. (CZS) We are given a circle K with center S and radius 1 and a square Q with center M and side 2. Let XY be the hypotenuse of an isosceles right triangle XY Z. Describe the locus of points Z as X varies along K and Y varies along the boundary of Q. 17. (ROU) Suppose ABCD and A′ B′C′ D′ are two parallelograms arbitrarily arranged in space, and let points M, N, P, Q divide the segments AA′ , BB′ ,CC′ , DD′ respectively in equal ratios. (a) Show that MNPQ is a parallelogram; (b) Find the locus of MNPQ as M varies along the segment AA′ . 1 18. (HUN) Solve the equation sinx + cos1 x = 1p , where p is a real parameter. Discuss for which values of p the equation has at least one real solution and determine the number of solutions in [0, 2π ) for a given p. 19. (HUN) Construct a triangle given the three exradii. 20. (HUN) We are given three equal rectangles with the same center in three mutually perpendicular planes, with the long sides also mutually perpendicular. Consider the polyhedron with vertices at the vertices of these rectangles. (a) Find the volume of this polyhedron; (b) can this polyhedron be regular, and under what conditions? 21. (BGR) Prove that the volume V and the lateral area S of a right circular cone 2  2S 3 satisfy the inequality 6V ≤ π √3 . When does equality occur? π 22. (BGR) Assume that two parallelograms P, P′ of equal areas have sides a, b and a′ , b′ respectively such that a′ ≤ a ≤ b ≤ b′ and a segment of length b′ can be placed inside P. Prove that P and P′ can be partitioned into four pairwise congruent parts. 23. (BGR) Three faces of a tetrahedron are right triangles, while the fourth is not an obtuse triangle. (a) Prove that a necessary and sufficient condition for the fourth face to be a right triangle is that at some vertex exactly two angles are right. 38 3 Problems (b) Prove that if all the faces are right triangles, then the volume of the tetrahedron equals one -sixth the product of the three smallest edges not belonging to the same face. 24. (POL) There are n ≥ 2 people in a room. Prove that there exist two among them having equal numbers of friends in that room. (Friendship is always mutual.) √ √ √ 25. (GDR) Show that tan 7◦ 30′ = 6 + 2 − 3 − 2. 26. (CZS) (a) Prove that (a1 + a2 + · · · + ak )2 ≤ k(a21 + · · · + a2k ), where k ≥ 1 is a natural number and a1 , . . . , ak are arbitrary real numbers. (b) If real numbers a1 , . . . , an satisfy q a1 + a2 + · · · + an ≥ (n − 1)(a21 + · · · + a2n ), show that they are all nonnegative. 27. (GDR) We are given a circle K and a point P lying on a line g. Construct a circle that passes through P and touches K and g. 28. (CZS) Let there be given a circle with center S and radius 1 in the plane, and let ABC be an arbitrary triangle circumscribed about the circle such that SA ≤ SB ≤ SC. Find the loci of the vertices A, B,C. 29. (ROU) (a) Find the number of ways 500 can be represented as a sum of consecutive integers. (b) Find the number of such representations for N = 2α 3β 5γ , α , β , γ ∈ N. Which of these representations consist only of natural numbers? (c) Determine the number of such representations for an arbitrary natural number N. 30. (ROU) If n is a natural number, prove that 3 (a) log10 (n + 1) > 10n + log10 n;  3n 1 (b) log n! > 10 2 + 13 + · · · + 1n − 1 . 31. (ROU) Solve the equation |x2 − 1| + |x2 − 4| = mx as a function of the parameter m. Which pairs (x, m) of integers satisfy this equation? 32. (BGR) The sides a, b, c of a triangle ABC form an arithmetic progression; the sides of another triangle A1 B1C1 also form an arithmetic progression. Suppose that ∠A = ∠A1 . Prove that the triangles ABC and A1 B1C1 are similar. 33. (BGR) Two circles touch each other from inside, and an equilateral triangle is inscribed in the larger circle. From the vertices of the triangle one draws segments tangent to the smaller circle. Prove that the length of one of these segments equals the sum of the lengths of the other two. 34. (BGR) Determine all pairs of positive integers (x, y) satisfying the equation 2x = 3y + 5. 35. (POL) If a, b, c, d are integers such that ad is odd and bc is even, prove that at least one root of the polynomial ax3 + bx2 + cx + d is irrational. 3.8 IMO 1966 39 36. (POL) Let ABCD be a cyclic quadrilateral. Show that the centroids of the triangles ABC, CDA, BCD, DAB lie on a circle. 37. (POL) Prove that the perpendiculars drawn from the midpoints of the sides of a cyclic quadrilateral to the opposite sides meet at one point. 38. (ROU) Two concentric circles have radii R and r respectively. Determine the greatest possible number of circles that are tangent to both these circles and √ √ 3 √R+√r mutually nonintersecting. Prove that this number lies between 2 · R− r − 1 and 63 20 · R+r R−r . 39. (ROU) In a plane, a circle with center O and radius R and two points A, B are given. (a) Draw a chord CD parallel to AB so that AC and BD intersect at a point P on the circle. (b) Prove that there are two possible positions of point P, say P1 , P2 , and find the distance between them if OA = a, OB = b, AB = d. 40. (CZS) For a positive real number p, find all real solutions to the equation p p x2 + 2px − p2 − x2 − 2px − p2 = 1. 41. (CZS) If A1 A2 . . . An is a regular n-gon (n ≥ 3), how many different obtuse triangles Ai A j Ak exist? 42. (CZS) Let a1 , a2 , . . . , an (n ≥ 2) be a sequence of integers. Show that there is a subsequence ak1 , ak2 , . . . , akm , where 1 ≤ k1 < k2 < · · · < km ≤ n, such that a2k1 + a2k2 + · · · + a2km is divisible by n. 43. (CZS) Five points in a plane are given, no three of which are collinear. Every two of them are joined by a segment, colored either red or gray, so that no three segments form a triangle colored in one color. (a) Prove that (1) every point is a vertex of exactly two red and two gray segments, and (2) the red segments form a closed path that passes through each point. (b) Give an example of such a coloring. 44. (YUG) What is the greatest number of balls of radius 1/2 that can be placed within a rectangular box of size 10 × 10 × 1? 45. (YUG) An alphabet consists of n letters. What is the maximal length of a word, if (i) two neighboring letters in a word are always different, and (ii) no word abab (a 6= b) can be obtained by omitting letters from the given word? 46. (YUG) Let f (a, b, c) = |b − a| b + a 2 |b − a| b + a 2 + − + + + . |ab| ab c |ab| ab c 40 3 Problems Prove that f (a, b, c) = 4 max{1/a, 1/b, 1/c}. 47. (ROU) Find the number of lines dividing a given triangle into two parts of equal area which determine the segment of minimum possible length inside the triangle. Compute this minimum length in terms of the sides a, b, c of the triangle. 48. (USS) Find all positive numbers p for which the equation x2 + px + 3p = 0 has integral roots. 49. (USS) Two mirror walls are placed to form an angle of measure α . There is a candle inside the angle. How many reflections of the candle can an observer see? 50. (USS) Given a quadrangle of sides a, b, c, d and area S, show that S ≤ a+c 2 · b+d 2 . 51. (USS) In a school, n children numbered 1 to n are initially arranged in the order 1, 2, . . . , n. At a command, every child can either exchange its position with any other child or not move. Can they rearrange into the order n, 1, 2, . . . , n − 1 after two commands? 52. (USS) A figure of area 1 is cut out from a sheet of paper and divided into 10 parts, each of which is colored in one of 10 colors. Then the figure is turned to the other side and again divided into 10 parts (not necessarily in the same way). Show that it is possible to color these parts in the 10 colors so that the total area of the portions of the figure both of whose sides are of the same color is at least 0.1. 53. (USS, 1966) Prove that in every convex hexagon of area S one can draw a diagonal that cuts off a triangle of area not exceeding 16 S. 54. (USS, 1966) Find the last two digits of a sum of eighth powers of 100 consecutive integers. 55. (USS, 1966) Given the vertex A and the centroid M of a triangle ABC, find the locus of vertices B such that all the angles of the triangle lie in the interval [40◦ , 70◦ ]. 56. (USS, 1966) Let ABCD be a tetrahedron such that AB ⊥ CD, AC ⊥ BD, and AD ⊥ BC. Prove that the midpoints of the edges of the tetrahedron lie on a sphere. 57. (USS, 1966) Is it possible to choose a set of 100 (or 200) points on the boundary of a cube such that this set is fixed under each isometry of the cube into itself? Justify your answer. 3.9 IMO 1967 41 3.9 The Ninth IMO Cetinje, Yugoslavia, July 2–13, 1967 3.9.1 Contest Problems First Day (July 5) 1. ABCD is a parallelogram; AB = a, AD = 1, α is the size of ∠DAB, and the three angles of the triangle ABD are acute. Prove that the four circles KA , KB , KC , KD , each of radius 1, whose centers√are the vertices A, B, C, D, cover the parallelogram if and only if a ≤ cos α + 3 sin α . 2. Exactly one side of a tetrahedron is of length greater than 1. Show that its volume is less than or equal to 1/8. 3. Let k, m, and n be positive integers such that m + k + 1 is a prime number greater than n + 1. Write cs for s(s + 1). Prove that the product (cm+1 − ck )(cm+2 − ck ) · · · (cm+n − ck ) is divisible by the product c1 c2 · · · cn . Second Day (July 6) 4. The triangles A0 B0C0 and A′ B′C′ have all their angles acute. Describe how to construct one of the triangles ABC, similar to A′ B′C′ and circumscribing A0 B0C0 (so that A, B, C correspond to A′ , B′ , C′ , and AB passes through C0 , BC through A0 , and CA through B0 ). Among these triangles ABC describe, and prove, how to construct the triangle with the maximum area. 5. Consider the sequence (cn ): c1 = a1 + a2 + · · · + a8, c2 = a21 + a22 + · · · + a28, ... ......... cn = an1 + an2 + · · · + an8, ... ......... where a1 , a2 , . . . , a8 are real numbers, not all equal to zero. Given that among the numbers of the sequence (cn ) there are infinitely many equal to zero, determine all the values of n for which cn = 0. 6. In a sports competition lasting n days there are m medals to be won. On the first day, one medal and 1/7 of the remaining m − 1 medals are won. On the second day, 2 medals and 1/7 of the remainder are won. And so on. On the nth day exactly n medals are won. How many days did the competition last and what was the total number of medals? 3.9.2 Longlisted Problems 1. (BGR 1) Prove that all numbers in the sequence 42 3 Problems 107811 110778111 111077781111 , , , ... 3 3 3 are perfect cubes. 2. (BGR 2) Prove that 13 n2 + 12 n + 16 ≥ (n!)2/n (n is a positive integer) and that equality is possible only in the case n = 1. 3. (BGR 3) Prove the trigonometric inequality cosx < 1 − (0, π /2). x2 2 + x4 16 , where x ∈ 4. (BGR 4) Suppose medians ma and mb of a triangle are orthogonal. Prove that: (a) The medians of that triangle correspond to the sides of a right-angled triangle. (b) The inequality 5(a2 + b2 − c2 ) ≥ 8ab is valid, where a, b, and c are side lengths of the given triangle. 5. (BGR 5) Solve the system x2 + x − 1 = y, y2 + y − 1 = z, z2 + z − 1 = x. 6. (BGR 6) Solve the system |x + y| + |1 − x| = 6, |x + y + 1| + |1 − y| = 4. 7. (CZS 1) Find all real solutions of the system of equations x1 + x2 + · · · + xn = a, x21 + x22 + · · · + x2n = a2 , ............... ... xn1 + xn2 + · · · + xnn = an . 8. (CZS 2)IMO1 ABCD is a parallelogram; AB = a, AD = 1, α is the size of ∠DAB, and the three angles of the triangle ABD are acute. Prove that the four circles KA , KB , KC , KD , each of radius 1, whose centers √ are the vertices A, B, C, D, cover the parallelogram if and only if a ≤ cos α + 3 sin α . 9. (CZS 3) The circle k and its diameter AB are given. Find the locus of the centers of circles inscribed in the triangles having one vertex on AB and two other vertices on k. 10. (CZS 4) The square ABCD is to be decomposed into n triangles (nonoverlapping) all of whose angles are acute. Find the smallest integer n for which there exists a solution to this problem and construct at least one decomposition for this n. Answer whether it is possible to ask additionally that (at least) one of these triangles has a perimeter less than an arbitrarily given positive number. 3.9 IMO 1967 43 11. (CZS 5) Let n be a positive integer. Find the maximal number of noncongruent triangles whose side lengths are integers less than or equal to n. 12. (CZS 6) Given a segment AB of the length 1, define the set M of points in the following way: it contains the two points A, B, and also all points obtained from A, B by iterating the following rule: for every pair of points X, Y in M, the set M also contains the point Z of the segment XY for which Y Z = 3X Z. (a) Prove that the set M consists of points X from the segment AB for which the distance from the point A is either AX = 3k 4n or AX = 3k − 2 , 4n where n, k are nonnegative integers. (b) Prove that the point X0 for which AX0 = 1/2 = X0 B does not belong to the set M. 13. (GDR 1) Find whether among all quadrilaterals whose interiors lie inside a semicircle of radius r there exists one (or more) with maximal area. If so, determine their shape and area. 14. (GDR 2) √ Which fraction p/q, where p, q are positive integers less than 100, is closest to 2? Find all digits after the decimal point in the decimal representa√ tion of this fraction that coincide with digits in the decimal representation of 2 (without using any tables). 15. (GDR 3) Suppose tan α = p/q, where p and q are integers and q 6= 0. Prove that the number tan β for which tan 2β = tan 3α is rational only when p2 + q2 is the square of an integer. 16. (GDR 4) Prove the following statement: If r1 and r2 are real numbers whose quotient is irrational, then any real number x can be approximated arbitrarily well by numbers of the form zk1 ,k2 = k1 r1 + k2 r2 , k1 , k2 integers; i.e., for every real number x and every positive real number p two integers k1 and k2 can be found such that |x − (k1 r1 + k2 r2 )| < p. 17. (UNK 1)IMO3 Let k, m, and n be positive integers such that m + k + 1 is a prime number greater than n + 1. Write cs for s(s + 1). Prove that the product (cm+1 − ck )(cm+2 − ck ) · · · (cm+n − ck ) is divisible by the product c1 c2 · · · cn . 18. (UNK 5) If x is a positive rational number, show that x can be uniquely expressed in the form a 2 a3 x = a1 + + + · · · , 2! 3! where a1 , a2 , . . . are integers, 0 ≤ an ≤ n − 1 for n > 1, and the series terminates. Show also that x can be expressed as the sum of reciprocals of different integers, each of which is greater than 106 . 19. (UNK 6) The n points P1 , P2 , . . . , Pn are placed inside or on the boundary of a disk of radius 1 in such a way that the minimum distance dn between any two 44 3 Problems of these points has its largest possible value Dn . Calculate Dn for n = 2 to 7 and justify your answer. 20. (HUN 1) In space, n points (n ≥ 3) are given. Every pair of points determines some distance. Suppose all distances are different. Connect every point with the nearest point. Prove that it is impossible to obtain a polygonal line in such a way. 1 21. (HUN 2) Without using any tables, find the exact value of the product P = cos π 2π 3π 4π 5π 6π 7π cos cos cos cos cos cos . 15 15 15 15 15 15 15 22. (HUN 3) The distance between the centers of the circles k1 and k2 with radii r is equal to r. Points A and B are on the circle k1 , symmetric with respect to the line connecting the centers of the circles. Point P is an arbitrary point on k2 . Prove that PA2 + PB2 ≥ 2r2 . When does equality hold? 23. (HUN 4) Prove that for an arbitrary pair of vectors f and g in the plane, the inequality a f 2 + b f g + cg2 ≥ 0 holds if and only if the following conditions are fulfilled: a ≥ 0, c ≥ 0, 4ac ≥ b2 . 24. (HUN 5)IMO6 Father has left to his children several identical gold coins. According to his will, the oldest child receives one coin and one-seventh of the remaining coins, the next child receives two coins and one-seventh of the remaining coins, the third child receives three coins and one-seventh of the remaining coins, and so on through the youngest child. If every child inherits an integer number of coins, find the number of children and the number of coins. 25. (HUN 6) Three disks of diameter d are touching a sphere at their centers. Moreover, each disk touches the other two disks. How do we choose the radius R of the sphere so that the axis of the whole figure makes an angle of 60◦ with the line connecting the center of the sphere with the point on the disks that is at the largest distance from the axis? (The axis of the figure is the line having the property that rotation of the figure through 120◦ about that line brings the figure to its initial position. The disks are all on one side of the plane, pass through the center of the sphere, and are orthogonal to the axes.) 26. (ITA 1) Let ABCD be a regular tetrahedron. To an arbitrary point M on one edge, say CD, corresponds the point P = P(M), which is the intersection of two lines AH and BK, drawn from A orthogonally to BM and from B orthogonally to AM. What is the locus of P as M varies? 1 The statement so formulated is false. It would be trivially true under the additional assumption that the polygonal line is closed. However, from the offered solution, which is not clear, it does not seem that the proposer had this in mind. 3.9 IMO 1967 45 27. (ITA 2) Which regular polygons can be obtained (and how) by cutting a cube with a plane? 28. (ITA 3) Find values of the parameter u for which the expression y= does not depend on x. tan(x − u) + tanx + tan(x + u) tan(x − u) tanx tan(x + u) 29. (ITA 4)IMO4 The triangles A0 B0C0 and A′ B′C′ have all their angles acute. Describe how to construct one of the triangles ABC, similar to A′ B′C′ and circumscribing A0 B0C0 (so that A, B, C correspond to A′ , B′ , C′ , and AB passes through C0 , BC through A0 , and CA through B0 ). Among these triangles ABC, describe, and prove, how to construct the triangle with the maximum area. 30. (MNG 1) Given m+n numbers ai (i = 1, 2, . . . , m), b j ( j = 1, 2, . . . , n), determine the number of pairs (ai , b j ) for which |i− j| ≥ k, where k is a nonnegative integer. 31. (MNG 2) An urn contains balls of k different colors; there are ni balls of the ith color. Balls are drawn at random from the urn, one by one, without replacement. Find the smallest number of draws necessary for getting m balls of the same color. 32. (MNG 3) Determine the volume of the body obtained by cutting the ball of radius R by the trihedron with vertex in the center of that ball if its dihedral angles are α , β , γ . 33. (MNG 4) In what case does the system x + y + mz = a, x + my + z = b, mx + y + z = c, have a solution? Find the conditions under which the unique solution of the above system is an arithmetic progression. 34. (MNG 5) The faces of a convex polyhedron are six squares and eight equilateral triangles, and each edge is a common side for one triangle and one square. All dihedral angles obtained from the triangle and square with a common edge are equal. Prove that it is possible to circumscribe a sphere around this polyhedron and compute the ratio of the squares of the volumes of the polyhedron and of the ball whose boundary is the circumscribed sphere. 35. (MNG 6) Prove the identity " # n   n x 2k 1 x k 1+2 = sec2n + secn x. ∑ k tan 2 k 2 2 (1 − tan (x/2)) k=0 36. (POL 1) Prove that the center of the sphere circumscribed around a tetrahedron ABCD coincides with the center of a sphere inscribed in that tetrahedron if and only if AB = CD, AC = BD, and AD = BC. 46 3 Problems 37. (POL 2) Prove that for arbitrary positive numbers the following inequality holds: 1 1 1 a 8 + b 8 + c8 + + ≤ . a b c a3 b3 c3 38. (POL 3) Does there exist an integer such that its cube is equal to 3n2 + 3n + 7, where n is integer? 39. (POL 4) Show that the triangle whose angles satisfy the equality sin2 A + sin2 B + sin2 C =2 cos2 A + cos2 B + cos2 C is a right-angled triangle. 40. (POL 5)IMO2 Exactly one side of a tetrahedron is of length greater than 1. Show that its volume is less than or equal to 1/8. 41. (POL 6) A line l is drawn through the intersection point H of the altitudes of an acute-angled triangle. Prove that the symmetric images la , lb , lc of l with respect to sides BC, CA, AB have one point in common, which lies on the circumcircle of ABC. 42. (ROU 1) Decompose into real factors the expression 1 − sin5 x − cos5 x. 43. (ROU 2) The equation x5 + 5λ x4 − x3 + (λ α − 4)x2 − (8λ + 3)x + λ α − 2 = 0 is given. (a) Determine α such that the given equation has exactly one root independent of λ . (b) Determine α such that the given equation has exactly two roots independent of λ . 44. (ROU 3) Suppose p and q are two different positive integers and x is a real number. Form the product (x + p)(x + q). (a) Find the sum S(x, n) = ∑(x + p)(x + q), where p and q take values from 1 to n. (b) Do there exist integer values of x for which S(x, n) = 0? 45. (ROU 4) (a) Solve the equation     2π 4π 3 sin3 x + sin3 + x + sin3 + x + cos 2x = 0. 3 3 4 (b) Suppose the solutions are in the form of arcs AB of the trigonometric circle (where A is the beginning of arcs of the trigonometric circle), and P is a regular n-gon inscribed in the circle with one vertex at A. (1) Find the subset of arcs with the endpoint B at a vertex of the regular dodecagon. 3.9 IMO 1967 47 (2) Prove that the endpoint B cannot be at a vertex of P if 2, 3 ∤ n or n is prime. 46. (ROU 5) If x, y, z are real numbers satisfying the relations x + y + z = 1 and arctanx + arctany + arctanz = π /4, prove that x2n+1 + y2n+1 + z2n+1 = 1 for all positive integers n. 47. (ROU 6) Prove the inequality  x1 x2 · · · xk xn−1 + xn−1 + · · · + xn−1 ≤ xn+k−1 + xn+k−1 + · · · + xn+k−1 , 1 2 1 2 k k where xi > 0 (i = 1, 2, . . . , k), k ∈ N, n ∈ N. √ 48. (SWE 1) Determine all positive roots of the equation xx = 1/ 2. 49. (SWE 2) Let n and k be positive integers such that 1 ≤ n ≤ N + 1, 1 ≤ k ≤ N + 1. Show that 2 min | sin n − sink| < . n6=k N 50. (SWE 3) The function ϕ (x, y, z), defined for all triples (x, y, z) of real numbers, is such that there are two functions f and g defined for all pairs of real numbers such that ϕ (x, y, z) = f (x + y, z) = g(x, y + z) for all real x, y, and z. Show that there is a function h of one real variable such that ϕ (x, y, z) = h(x + y + z) for all real x, y, and z. 51. (SWE 4) A subset S of the set of integers 0, . . . , 99 is said to have property A if it is impossible to fill a crossword puzzle with 2 rows and 2 columns with numbers in S (0 is written as 00, 1 as 01, and so on). Determine the maximal number of elements in sets S with property A. 52. (SWE 5) In the plane a point O and a sequence of points P1 , P2 , P3 , . . . are given. The distances OP1 , OP2 , OP3 , . . . are r1 , r2 , r3 , . . . , where r1 ≤ r2 ≤ r3 ≤ · · · . Let α satisfy 0 < α < 1. Suppose that for every n the distance from the point Pn to any other point of the sequence is greater than or equal to rnα . Determine the exponent β , as large as possible, such that for some C independent of n,2 2 rn ≥ Cnβ , n = 1, 2, . . . . This problem is not elementary. The solution offered by the proposer, which is not quite 1 clear and complete, only shows that if such a β exists, then β ≥ 2(1− α) . 48 3 Problems 53. (SWE 6) In making Euclidean constructions in geometry it is permitted to use a straightedge and compass. In the constructions considered in this question, no compasses are permitted, but the straightedge is assumed to have two parallel edges, which can be used for constructing two parallel lines through two given points whose distance is at least equal to the breadth of the ruler. Then the distance between the parallel lines is equal to the breadth of the straightedge. Carry through the following constructions with such a straightedge. Construct: (a) The bisector of a given angle. (b) The midpoint of a given rectilinear segment. (c) The center of a circle through three given noncollinear points. (d) A line through a given point parallel to a given line. 54. (USS 1) Is it possible to put 100 (or 200) points on a wooden cube such that by all rotations of the cube the points map into themselves? Justify your answer. 55. (USS 2) Find all x for which for all n, √ 3 . 2 56. (USS 3) In a group of interpreters each one speaks one or several foreign languages; 24 of them speak Japanese, 24 Malay, 24 Farsi. Prove that it is possible to select a subgroup in which exactly 12 interpreters speak Japanese, exactly 12 speak Malay, and exactly 12 speak Farsi. sin x + sin 2x + sin 3x + · · · + sin nx ≤ 57. (USS 4)IMO5 Consider the sequence (cn ): c1 = a1 + a2 + · · · + a8, c2 = a21 + a22 + · · · + a28, ... ......... cn = an1 + an2 + · · · + an8, ... ......... where a1 , a2 , . . . , a8 are real numbers, not all equal to zero. Given that among the numbers of the sequence (cn ) there are infinitely many equal to zero, determine all the values of n for which cn = 0. 58. (USS 5) A linear binomial l(z) = Az + B with complex coefficients A and B is given. It is known that the maximal value of |l(z)| on the segment −1 ≤ x ≤ 1 (y = 0) of the real line in the complex plane (z = x + iy) is equal to M. Prove that for every z |l(z)| ≤ M ρ , where ρ is the sum of distances from the point P = z to the points Q1 : z = 1 and Q3 : z = −1. 59. (USS 6) On the circle with center O and radius 1 the point A0 is fixed and points A1 , A2 , . . . , A999 , A1000 are distributed in such a way that ∠A0 OAk = k (in radians). Cut the circle at points A0 , A1 , . . . , A1000 . How many arcs with different lengths are obtained? 3.10 IMO 1968 49 3.10 The Tenth IMO Moscow–Leningrad, Soviet Union, July 5–18, 1968 3.10.1 Contest Problems First Day 1. Prove that there exists a unique triangle whose side lengths are consecutive natural numbers and one of whose angles is twice the measure of one of the others. 2. Find all positive integers x for which p(x) = x2 − 10x − 22, where p(x) denotes the product of the digits of x. 3. Let a, b, c be real numbers. Prove that the system of equations  ax21 + bx1 + c = x2 ,     ax22 + bx2 + c = x3 ,  ············  2  ax + bx + c = xn ,  n−1  n−1  ax2n + bxn + c = x1 , (a) has no real solutions if (b − 1)2 − 4ac < 0; (b) has a unique real solution if (b − 1)2 − 4ac = 0; (c) has more than one real solution if (b − 1)2 − 4ac > 0. Second Day 4. Prove that in any tetrahedron there is a vertex such that the lengths of its sides through that vertex are sides of a triangle. 5. Let a > 0 be a real number and f (x) a real function defined on all of R, satisfying for all x ∈ R, q 1 f (x + a) = + f (x) − f (x)2 . 2 (a) Prove that the function f is periodic; i.e., there exists b > 0 such that for all x, f (x + b) = f (x). (b) Give an example of such a nonconstant function for a = 1. 6. Let [x] denote the integer part of x, i.e., the greatest integer not exceeding x. If n is a positive integer, express as a simple function of n the sum       n+1 n+2 n + 2i + + ···+ + ··· . 2 4 2i+1 3.10.2 Shortlisted Problems 1. (SWE 2) Two ships sail on the sea with constant speeds and fixed directions. It is known that at 9:00 the distance between them was 20 miles; at 9:35, 15 miles; and at 9:55, 13 miles. At what moment were the ships the smallest distance from each other, and what was that distance? 50 3 Problems 2. (ROU 5)IMO1 Prove that there exists a unique triangle whose side lengths are consecutive natural numbers and one of whose angles is twice the measure of one of the others. 3. (POL 4)IMO4 Prove that in any tetrahedron there is a vertex such that the lengths of its sides through that vertex are sides of a triangle. 4. (BGR 2)IMO3 Let a, b, c be real numbers. Prove that the system of equations  ax21 + bx1 + c = x2 ,     ax22 + bx2 + c = x3 ,  ············  2  ax + bx + c = xn ,    n−1 2 n−1 axn + bxn + c = x1 , has a unique real solution if and only if (b − 1)2 − 4ac = 0. Remark. It is assumed that a 6= 0. 5. (BGR 5) Let hn be the apothem (distance from the center to one of the sides) of a regular n-gon (n ≥ 3) inscribed in a circle of radius r. Prove the inequality (n + 1)hn+1 − nhn > r. Also prove that if r on the right side is replaced with a greater number, the inequality will not remain true for all n ≥ 3. 6. (HUN 1) If ai (i = 1, 2, . . . , n) are distinct non-zero real numbers, prove that the equation a1 a2 an + + ···+ =n a1 − x a2 − x an − x has at least n − 1 real roots. 7. (HUN 5) Prove that the product of the radii of three circles exscribed to a given √ triangle does not exceed 3 8 3 times the product of the side lengths of the triangle. When does equality hold? 8. (ROU 2) Given an oriented line ∆ and a fixed point A on it, consider all trapezoids ABCD one of whose bases AB lies on ∆ , in the positive direction. Let E, F be the midpoints of AB and CD respectively. Find the loci of vertices B,C, D of trapezoids that satisfy the following: (i) |AB| ≤ a (a fixed); (ii) |EF| = l (l fixed); (iii) the sum of squares of the nonparallel sides of the trapezoid is constant. Remark. The constants are chosen so that such trapezoids exist. 9. (ROU 3) Let ABC be an arbitrary triangle and M a point inside it. Let da , db , dc be the distances from M to sides BC,CA, AB; a, b, c the lengths of the sides respectively, and S the area of the triangle ABC. Prove the inequality 3.10 IMO 1968 abda db + bcdbdc + cadc da ≤ 51 4S2 . 3 Prove that the left-hand side attains its maximum when M is the centroid of the triangle. 10. (ROU √ 4) Consider two segments of length a, b (a > b) and a segment of length c = ab. (a) For what values of a/b can these segments be sides of a triangle? (b) For what values of a/b is this triangle right-angled, obtuse-angled, or acuteangled? 11. (ROU 6) Find all solutions (x1 , x2 , . . . , xn ) of the equation 1+ 1 x1 + 1 (x1 + 1)(x2 + 1) (x1 + 1) · · · (xn−1 + 1) + + + ···+ = 0. x1 x1 x2 x1 x2 x3 x1 x2 · · · xn 12. (POL 1) If a and b are arbitrary positive real numbers and m an integer, prove that    a m b m 1+ + 1+ ≥ 2m+1 . b a 13. (POL 5) Given two congruent triangles A1 A2 A3 and B1 B2 B3 (Ai Ak = Bi Bk ), prove that there exists a plane such that the orthogonal projections of these triangles onto it are congruent and equally oriented. 14. (BGR 5) A line in the plane of a triangle ABC intersects the sides AB and AC respectively at points X and Y such that BX = CY . Find the locus of the center of the circumcircle of triangle X AY . 15. (UNK 1)IMO6 Let [x] denote the integer part of x, i.e., the greatest integer not exceeding x. If n is a positive integer, express as a simple function of n the sum       n+1 n+2 n + 2i + + ···+ + ··· . 2 4 2i+1 16. (UNK 3) A polynomial p(x) = a0 xk + a1 xk−1 + · · · + ak with integer coefficients is said to be divisible by an integer m if p(x) is divisible by m for all integers x. Prove that if p(x) is divisible by m, then k!a0 is also divisible by m. Also prove that if a0 , k, m are nonnegative integers for which k!a0 is divisible by m, there exists a polynomial p(x) = a0 xk + · · · + ak divisible by m. 17. (UNK 4) Given a point O and lengths x, y, z, prove that there exists an equilateral triangle ABC for which OA = x, OB = y, OC = z, if and only if x+y ≥ z, y+z ≥ x, z + x ≥ y (the points O, A, B,C are coplanar). 18. (ITA 2) If an acute-angled triangle ABC is given, construct an equilateral triangle A′ B′C′ in space such that lines AA′ , BB′ ,CC′ pass through a given point. 19. (ITA 5) We are given a fixed point on the circle of radius 1, and going from this point along the circumference in the positive direction on curved distances 52 3 Problems 0, 1, 2, . . . from it we obtain points with abscissas n = 0, 1, 2, . . . respectively. How many points among them should we take to ensure that some two of them are less than the distance 1/5 apart? 20. (CZS 1) Given n (n ≥ 3) points in space such that every three of them form a triangle with one angle greater than or equal to 120◦ , prove that these points can be denoted by A1 , A2 , . . . , An in such a way that for each i, j, k, 1 ≤ i < j < k ≤ n, angle Ai A j Ak is greater than or equal to 120◦ . 21. (CZS 2) Let a0 , a1 , . . . , ak (k ≥ 1) be positive integers. Find all positive integers y such that a0 | y; (a0 + a1 ) | (y + a1 ); . . . ; (a0 + an ) | (y + an ). 22. (CZS 3)IMO2 Find all positive integers x for which p(x) = x2 − 10x − 22, where p(x) denotes the product of the digits of x. 23. (CZS 4) Find all complex numbers m such that polynomial x3 + y3 + z3 + mxyz can be represented as the product of three linear trinomials. 24. (MNG 1) Find the number of all n-digit numbers for which some fixed digit stands only in the ith (1 < i < n) place and the last j digits are distinct.3 25. (MNG 2) Given k parallel lines and a few points on each of them, find the number of all possible triangles with vertices at these given points.4 26. (GDR)IMO5 Let a > 0 be a real number and f (x) a real function defined on all of R, satisfying for all x ∈ R, q 1 f (x + a) = + f (x) − f (x)2 . 2 (a) Prove that the function f is periodic; i.e., there exists b > 0 such that for all x, f (x + b) = f (x). (b) Give an example of such a nonconstant function for a = 1. 3 4 The problem is unclear. Presumably n, i, j, and the ith digit are fixed. The problem is unclear. The correct formulation could be the following: Given k parallel lines l1 , . . . , lk and ni points on the line li , i = 1, 2, . . . , k, find the maximum possible number of triangles with vertices at these points. 3.11 IMO 1969 53 3.11 The Eleventh IMO Bucharest, Romania, July 5–20, 1969 3.11.1 Contest Problems First Day (July 10) 1. Prove that there exist infinitely many natural numbers a with the following property: the number z = n4 + a is not prime for any natural number n. 2. Let a1 , a2 , . . . , an be real constants and y(x) = cos(a1 + x) + cos(a2 + x) cos(a3 + x) cos(an + x) + + ···+ . 2 22 2n−1 If x1 , x2 are real and y(x1 ) = y(x2 ) = 0, prove that x1 − x2 = mπ for some integer m. 3. Find conditions on the positive real number a such that there exists a tetrahedron k of whose edges (k = 1, 2, 3, 4, 5) have length a, and the other 6 − k edges have length 1. Second Day (July 11) 4. Let AB be a diameter of a circle γ . A point C different from A and B is on the circle γ . Let D be the projection of the point C onto the line AB. Consider three other circles γ1 , γ2 , and γ3 with the common tangent AB: γ1 inscribed in the triangle ABC, and γ2 and γ3 tangent to both (the segment) CD and γ . Prove that γ1 , γ2 , and γ3 have two common tangents. 5. Given n points in the plane such that no three of them are collinear, prove that one can find at least n−3 convex quadrilaterals with their vertices at these points. 2 6. Under the conditions x1 , x2 > 0, x1 y1 > z21 , and x2 y2 > z22 , prove the inequality 8 1 1 ≤ + . 2 2 (x1 + x2 )(y1 + y2 ) − (z1 + z2 ) x1 y1 − z1 x2 y2 − z22 3.11.2 Longlisted Problems 1. (BEL 1) A parabola P1 with equation x2 −2py = 0 and parabola P2 with equation x2 + 2py = 0, p > 0, are given. A line t is tangent to P2 . Find the locus of pole M of the line t with respect to P1 . 2. (BEL 2) (a) Find the equations of regular hyperbolas passing through the points A(α , 0), B(β , 0), and C(0, γ ). (b) Prove that all such hyperbolas pass through the orthocenter H of the triangle ABC. (c) Find the locus of the centers of these hyperbolas. 54 3 Problems (d) Check whether this locus coincides with the nine-point circle of the triangle ABC. 3. (BEL 3) Construct the circle that is tangent to three given circles. 4. (BEL 4) Let O be a point on a nondegenerate conic. A right angle with vertex O intersects the conic at points A and B. Prove that the line AB passes through a fixed point located on the normal to the conic through the point O. 5. (BEL 5) Let G be the centroid of the triangle OAB. (a) Prove that all conics passing through the points O, A, B, G are hyperbolas. (b) Find the locus of the centers of these hyperbolas. 6. (BEL 6) Evaluate (cos(π /4) + i sin(π /4))10 in two different ways and prove that       10 10 1 10 − + = 24 . 1 3 2 5 p 7. (BGR 1) Prove that the equation x3 + y3 + z3 = 1969 has no integral solutions. 8. (BGR 2) Find all functions f defined for all x that satisfy the condition x f (y) + y f (x) = (x + y) f (x) f (y), for all x and y. Prove that exactly two of them are continuous. 9. (BGR 3) One hundred convex polygons are placed on a square with edge of length 38 cm. The area of each of the polygons is smaller than π cm2 , and the perimeter of each of the polygons is smaller than 2π cm. Prove that there exists a disk with radius 1 in the square that does not intersect any of the polygons. 10. (BGR 4) Let M be the point inside the right-angled triangle ABC (∠C = 90◦ ) such that ∠MAB = ∠MBC = ∠MCA = ϕ . Let ψ be the acute angle between the medians of AC and BC. Prove that sin(ϕ +ψ ) = 5. sin(ϕ −ψ ) 11. (BGR 5) Let Z be a set of points in the plane. Suppose that there exists a pair of points that cannot be joined by a polygonal line not passing through any point of Z. Let us call such a pair of points unjoinable. Prove that for each real r > 0 there exists an unjoinable pair of points in plane separated by distance r. 12. (CZS 1) Given a unit cube, find the locus of the centroids of all tetrahedra whose vertices lie on the sides of the cube. 13. (CZS 2) Let p be a prime odd number. Is it possible to find p−1 natural numbers n + 1, n + 2, . . ., n + p − 1 such that the sum of the squares of these numbers is divisible by the sum of these numbers? 14. (CZS 3) Let a and b be two positive real numbers. If x is a real solution of the equation x2 + px + q = 0 with real coefficients p and q such that |p| ≤ a, |q| ≤ b, prove that 3.11 IMO 1969 55  p 1 a + a2 + 4b . (1) 2 Conversely, if x satisfies (1), prove that there exist real numbers p and q with |p| ≤ a, |q| ≤ b such that x is one of the roots of the equation x2 + px + q = 0. |x| ≤ 15. (CZS 4) Let K1 , . . . , Kn be nonnegative integers. Prove that K1 !K2 ! · · · Kn ! ≥ [K/n]!n , where K = K1 + · · · + Kn . 16. (CZS 5) A convex quadrilateral ABCD with sides AB = a, BC = b, CD = c, DA = d and angles α = ∠DAB, β = ∠ABC, γ = ∠BCD, and δ = ∠CDA is given. Let s = (a + b + c + d)/2 and P be the area of the quadrilateral. Prove that P2 = (s − a)(s − b)(s − c)(s − d) − abcd cos2 α +γ . 2 17. (CZS 6) Let d and p be two real numbers. Find the first term of an arithmetic progression a1 , a2 , a3 , . . . with difference d such that a1 a2 a3 a4 = p. Find the number of solutions in terms of d and p. 18. (FRA 1) Let a and b be two nonnegative integers. Denote by H(a, b) the set of numbers n of the form n = pa + qb, where p and q are positive integers. Determine H(a) = H(a, a). Prove that if a 6= b, it is enough to know all the sets H(a, b) for coprime numbers a, b in order to know all the sets H(a, b). Prove that in the case of coprime numbers a and b, H(a, b) contains all numbers greater than or equal to ω = (a − 1)(b − 1) and also ω /2 numbers smaller than ω . 19. (FRA 2) Let n be an integer that is not divisible by any square greater than 1. Denote by xm the last digit of the number xm in the number system with base n. For which integers x is it possible for xm to be 0? Prove that the sequence xm is periodic with period t independent of x. For which x do we have xt = 1. Prove that if m and x are relatively prime, then 0m , 1m , . . . , (n − 1)m are different numbers. Find the minimal period t in terms of n. If n does not meet the given condition, prove that it is possible to have xm = 0 6= x1 and that the sequence is periodic starting only from some number k > 1. 20. (FRA 3) A polygon (not necessarily convex) with vertices in the lattice points of a rectangular grid is given. The area of the polygon is S. If I is the number of lattice points that are strictly in the interior of the polygon and B the number of lattice points on the border of the polygon, find the number T = 2S − B − 2I + 2. 21. (FRA 4) A right-angled triangle OAB has its right angle at the point B. An arbitrary circle with center on the line OB is tangent to the line OA. Let AT be the tangent to the circle different from OA (T is the point of tangency). Prove that the median from B of the triangle OAB intersects AT at a point M such that MB = MT . 22. (FRA 5) Let α (n) be the number of pairs (x, y) of integers such that x + y = n, 0 ≤ y ≤ x, and let β (n) be the number of triples (x, y, z) such that x + y + z = n 56 3 Problems and 0 ≤ z ≤ y ≤ x. Find a simple relation between α (n) and the integer part of the number n+2 2 and the relation among β (n), β (n − 3) and α (n). Then evaluate β (n) as a function of the residue of n modulo 6. What can be said about β (n) 2 (n+3) and 1 + n(n+6) 12 ? And what about 6 ? Find the number of triples (x, y, z) with the property x + y + z ≤ n, 0 ≤ z ≤ y ≤ x as a function of the residue of n modulo 6. What can be said about the relation 2 between this number and the number (n+6)(2n72+9n+12) ? b 23. (FRA 6) Consider the integer d = a c−1 , where a, b, and c are positive integers and c ≤ a. Prove that the set G of integers that are between 1 and d and relatively prime to d (the number of such integers is denoted by ϕ (d)) can be partitioned into n subsets, each of which consists of b elements. What can be said about the ϕ (d) rational number b ? 24. (UNK 1) The polynomial P(x) = a0 xk + a1 xk−1 + · · · + ak , where a0 , . . . , ak are integers, is said to be divisible by an integer m if P(x) is a multiple of m for every integral value of x. Show that if P(x) is divisible by m, then a0 · k! is a multiple of m. Also prove that if a, k, m are positive integers such that ak! is a multiple of m, then a polynomial P(x) with leading term axk can be found that is divisible by m. 25. (UNK 2) Let a, b, x, y be positive integers such that a and b have no common divisor greater than 1. Prove that the largest number not expressible in the form ax + by is ab − a − b. If N(k) is the largest number not expressible in the form ax + by in only k ways, find N(k). 26. (UNK 3) A smooth solid consists of a right circular cylinder of height h and base-radius r, surmounted by a hemisphere of radius r and center O. The solid stands on a horizontal table. One end of a string is attached to a point on the base. The string is stretched (initially being kept in the vertical plane) over the highest point of the solid and held down at the point P on the hemisphere such that OP makes an angle α with the horizontal. Show that if α is small enough, the string will slacken if slightly displaced and no longer remain in a vertical plane. If then pulled tight through P, show that it will cross the common circular section of the hemisphere and cylinder at a point Q such that ∠SOQ = φ , S being where it α initially crossed this section, and sin φ = r tan h . 27. (UNK 4) The segment AB perpendicularly bisects CD at X. Show that, subject to restrictions, there is a right circular cone whose axis passes through X and on whose surface lie the points A, B,C, D. What are the restrictions? 28. (UNK 5) Let us define u0 = 0, u1 = 1 and for n ≥ 0, un+2 = aun+1 + bun , a and b being positive integers. Express un as a polynomial in a and b. Prove the result. Given that b is prime, prove that b divides a(ub − 1). 29. (GDR 1) Find all real numbers λ such that the equation sin4 x − cos4 x = λ (tan4 x − cot4 x) 3.11 IMO 1969 (a) (b) (c) (d) 57 has no solution, has exactly one solution, has exactly two solutions, has more than two solutions (in the interval (0, π /4)). 30. (GDR 2)IMO1 Prove that there exist infinitely many natural numbers a with the following property: The number z = n4 + a is not prime for any natural number n. 31. (GDR 3) Find the number of permutations a1 , . . . , an of the set {1, 2, . . ., n} such that |ai − ai+1 | = 6 1 for all i = 1, 2, . . . , n − 1. Find a recurrence formula and evaluate the number of such permutations for n ≤ 6. 32. (GDR 4) Find the maximal number of regions into which a sphere can be partitioned by n circles. 33. (GDR 5) Given a ring G in the plane bounded by two concentric circles with radii R and R/2, prove that we can cover this region with 8 disks of radius 2R/5. (A region is covered if each of its points is inside or on the border of some disk.) 34. (HUN 1) Let a and b be arbitrary integers. Prove that if k is an integer not divisible by 3, then (a + b)2k + a2k + b2k is divisible by a2 + ab + b2. 35. (HUN 2) Prove that 1 1 1 5 + 3 + ···+ 3 < . 3 2 3 n 4 36. (HUN 3) In the plane 4000 points are given such that each line passes through at most 2 of these points. Prove that there exist 1000 disjoint quadrilaterals in the plane with vertices at these points. 1+ 37. (HUN 4)IMO2 If a1 , a2 , . . . , an are real constants, and if y = cos(a1 + x) + 2 cos(a2 + x) + · · · + n cos(an + x) has two zeros x1 and x2 whose difference is not a multiple of π , prove that y ≡ 0. 38. (HUN 5) Let r and m (r ≤ m) be natural numbers and Ak = 2k−1 2m π . Evaluate 1 m m ∑ ∑ sin(rAk ) sin(rAl ) cos(rAk − rAl ). m2 k=1 l=1 39. (HUN 6) Find the positions of three points A, B,C on the boundary of a unit cube such that min{AB, AC, BC} is the greatest possible. 40. (MNG 1) Find the number of five-digit numbers with the following properties: there are two pairs of digits such that digits from each pair are equal and are next to each other, digits from different pairs are different, and the remaining digit (which does not belong to any of the pairs) is different from the other digits. 41. (MNG 2) Given four real numbers x0 , x1 , α , β , find an expression for the solution of the system xn+2 − α xn+1 − β xn = 0, n = 0, 1, 2, . . . . 58 3 Problems 42. (MNG 3) Let Ak (1 ≤ k ≤ h) be n-element sets such that each two of them have a nonempty intersection. Let A be the union of all the sets Ak , and let B be a subset of A such that for each k (1 ≤ k ≤ h) the intersection of Ak and B consists of exactly two different elements ak and bk . Find all subsets X of the set A with r elements satisfying the condition that for at least one index k, both elements ak and bk belong to X . 43. (MNG 4) Let p and q be two prime numbers greater than 3. Prove that if their difference is 2n , then for any two integers m and n, the number S = p2m+1 + q2m+1 is divisible by 3. 44. (MNG 5) Find the radius of the circle circumscribed about the isosceles triangle whose sides are the solutions of the equation x2 − ax + b = 0. 45. (MNG 6)IMO5 Given n points in the plane such that no three of them are collinear,  prove that one can find at least n−3 convex quadrilaterals with their vertices at 2 these points. 46. (NLD 1) The vertices of an (n + 1)-gon are placed on the edges of a regular n-gon so that the perimeter of the n-gon is divided into equal parts. How does one choose these n + 1 points in order to obtain the (n + 1)gon with (a) maximal area; (b) minimal area? 47. (NLD 2)IMO4 Let A and B be points on the circle γ . A point C, different from A and B, is on the circle γ . Let D be the projection of the point C onto the line AB. Consider three other circles γ1 , γ2 , and γ3 with the common tangent AB: γ1 inscribed in the triangle ABC, and γ2 and γ3 tangent to both (the segment) CD and γ . Prove that γ1 , γ2 , and γ3 have two common tangents. 48. (NLD 3) Let x1 , x2 , x3 , x4 , and x5 be positive integers satisfying x1 x1 x1 −x1 x1 −x1 +x2 −x2 +x2 +x2 −x2 +x2 +x3 +x3 −x3 +x3 +x3 −x3 +x4 −x4 +x4 −x4 +x4 +x4 +x5 +x5 −x5 +x5 −x5 +x5 = 1000, > 0, > 0, > 0, > 0, > 0. (a) Find the maximum of (x1 + x3 )x2 +x4 . (b) In how many different ways can we choose x1 , . . . , x5 to obtain the desired maximum? 49. (NLD 4) A boy has a set of trains and pieces of railroad track. Each piece is a quarter of circle, and by concatenating these pieces, the boy obtained a closed railway. The railway does not intersect itself. In passing through this railway, the train sometimes goes in the clockwise direction, and sometimes in the opposite direction. Prove that the train passes an even number of times through the pieces in the clockwise direction and an even number of times in the counterclockwise direction. Also, prove that the number of pieces is divisible by 4. 3.11 IMO 1969 59 50. (NLD 5) The bisectors of the exterior angles of a pentagon B1 B2 B3 B4 B5 form another pentagon A1 A2 A3 A4 A5 . Construct B1 B2 B3 B4 B5 from the given pentagon A1 A2 A3 A4 A 5 . 51. (NLD 6) A curve determined by p y = x2 − 10x + 52, 0 ≤ x ≤ 100, is constructed in a rectangular grid. Determine the number of squares cut by the curve. 52. (POL 1) Prove that a regular polygon with an odd number of edges cannot be partitioned into four pieces with equal areas by two lines that pass through the center of polygon. 53. (POL 2) Given two segments AB and CD not in the same plane, find the locus of points M such that MA2 + MB2 = MC2 + MD2 . 54. (POL 3) Given a polynomial f (x) with integer coefficients whose value is divisible by 3 for three integers k, k + 1, and k + 2, prove that f (m) is divisible by 3 for all integers m. 55. (POL 4)IMO3 Find the conditions on the positive real number a such that there exists a tetrahedron k of whose edges (k = 1, 2, 3, 4, 5) have length a, and the other 6 − k edges have length 1. 56. (POL 5) Let a and b be two natural numbers that have an equal number n of digits in their decimal expansions. The first m digits (from left to right) of the numbers a and b are equal. Prove that if m > n/2, then 1 a1/n − b1/n < . n 57. (POL 6) On the sides AB and AC of triangle ABC two points K and L are given LC such that KB AK + AL = 1. Prove that KL passes through the centroid of ABC. 58. (SWE 1) Six points P1 , . . . , P6 are given in 3-dimensional space such that no four of them lie in the same plane. Each of the line segments Pj Pk is colored black or white. Prove that there exists one triangle Pj Pk Pl whose edges are of the same color. 59. (SWE 2) For each λ (0 < λ < 1 and λ 6= 1/n for all n = 1, 2, 3, . . . ) construct a continuous function f such that there do not exist x, y with 0 < λ < y = x + λ ≤ 1 for which f (x) = f (y). 60. (SWE 3) Find the natural number n with the following properties: (i) Let S = {p1 , p2 , . . . } be an arbitrary finite set of points in the plane, and r j the distance from Pj to the origin O. We assign to each Pj the closed disk D j with center Pj and radius r j . Then some n of these disks contain all points of S. 60 3 Problems (ii) n is the smallest integer with the above property. 61. (SWE 4) Let a0 , a1 , a2 be determined with a0 = 0, an+1 = 2an + 2n . Prove that if n is power of 2, then so is an . 62. (SWE 5) Which natural numbers can be expressed as the difference of squares of two integers? 63. (SWE 6) Prove that there are infinitely many positive integers that cannot be expressed as the sum of squares of three positive integers. 64. (USS 1) Prove that for a natural number n > 2, (n!)! > n[(n − 1)!]n!. 65. (USS 2) Prove that for a > b2 , s r r q √ 3 1 a − b a + b a − b a + · · · = a − b2 − b. 4 2 66. (USS 3) (a) Prove that if 0 ≤ a0 ≤ a1 ≤ a2 , then   1 1 1 (a0 + a1 x − a2 x2 )2 ≤ (a0 + a1 + a2)2 1 + x + x2 + x3 + x4 . 2 3 2 (b) Formulate and prove the analogous result for polynomials of third degree. 67. (USS 4)IMO6 Under the conditions x1 , x2 > 0, x1 y1 > z21 , and x2 y2 > z22 , prove the inequality 8 1 1 ≤ + . (x1 + x2 )(y1 + y2 ) − (z1 + z2 )2 x1 y1 − z21 x2 y2 − z22 68. (USS 5) Given 5 points in the plane, no three of which are collinear, prove that we can choose 4 points among them that form a convex quadrilateral. 69. (YUG 1) Suppose that positive real numbers x1 , x2 , x3 satisfy x1 x2 x3 > 1, x1 + x2 + x3 < 1 1 1 + + . x1 x2 x3 Prove that: (a) None of x1 , x2 , x3 equals 1. (b) Exactly one of these numbers is less than 1. √ 70. (YUG√2) A park has the shape of a convex pentagon of area 5 3 ha (= 50000 3 m2 ). A man standing at an interior point O of the park notices that he stands at a distance of at most 200 m from each vertex of the pentagon. Prove that he stands at a distance of at least 100 m from each side of the pentagon. 71. (YUG 3) Let four points Ai (i = 1, 2, 3, 4) in the plane determine four triangles. In each of these triangles we choose the smallest angle. The sum of these angles is denoted by S. What is the exact placement of the points Ai if S = 180◦ ? 3.12 IMO 1970 61 3.12 The Twelfth IMO Budapest–Keszthely, Hungary, July 8–22, 1970 3.12.1 Contest Problems First Day (July 13) 1. Given a point M on the side AB of the triangle ABC, let r1 and r2 be the radii of the inscribed circles of the triangles ACM and BCM respectively while ρ1 and ρ2 are the radii of the excircles of the triangles ACM and BCM at the sides AM and BM respectively. Let r and ρ denote the respective radii of the inscribed circle and the excircle at the side AB of the triangle ABC. Prove that r1 r2 r = . ρ1 ρ2 ρ 2. Let a and b be the bases of two number systems and let An = x1 x2 . . . xn (a) , An+1 = x0 x1 x2 . . . xn (a) , Bn = x1 x2 . . . xn (b) , Bn+1 = x0 x1 x2 . . . xn (b) , be numbers in the number systems with respective bases a and b, so that x0 , x1 , x2 , . . . , xn denote digits in the number system with base a as well as in the number system with base b. Suppose that neither x0 nor x1 is zero. Prove that a > b if and only if An Bn < . An+1 Bn+1 3. Let 1 = a0 ≤ a1 ≤ a2 ≤ · · · ≤ an ≤ · · · be a sequence of real numbers. Consider the sequence b1 , b2 , . . . defined by  n  ak−1 1 bn = ∑ 1 − √ . ak ak k=1 Prove that: (a) For all natural numbers n, 0 ≤ bn < 2. (b) Given an arbitrary 0 ≤ b < 2, there is a sequence a0 , a1 , . . . , an , . . . of the above type such that bn > b is true for an infinity of natural numbers n. Second Day (July 14) 4. For what natural numbers n can the product of some of the numbers n, n + 1, n + 2, n + 3, n + 4, n + 5 be equal to the product of the remaining ones? 5. In the tetrahedron ABCD, the edges BD and CD are mutually perpendicular, and the projection of the vertex D to the plane ABC is the intersection of the altitudes of the triangle ABC. Prove that (AB + BC + CA)2 ≤ 6(DA2 + DB2 + DC2) . For which tetrahedra does equality hold? 62 3 Problems 6. Given 100 points in the plane, no three of which are on the same line, consider all triangles that have all their vertices chosen from the 100 given points. Prove that at most 70% of these triangles are acute-angled. 3.12.2 Longlisted Problems 1. (AUT 1) Prove that bc ca ab 1 + + ≤ (a + b + c) b+c c+a a+b 2 9 (a, b, c > 0). 99 2. (AUT 2) Prove that the two last digits of 99 and 99 in decimal representation are equal. 3. (AUT 3) Prove that for a, b ∈ N, a!b! divides (a + b)!. 4. (AUT 4) Solve the system of equations x2 + xy = a2 + ab y2 + xy = a2 − ab, 5. (AUT 5) Prove that q n 1 n+1 a, b real, a 6= 0. 2 n + n+1 + · · · + n+1 ≥ 1 for n ≥ 2. 6. (BEL 1) Prove that the equation in x n bi ∑ x − ai = c, i=1 bi > 0, a1 < a2 < a3 < · · · < an , has n − 1 roots x1 , x2 , x3 , . . . , xn−1 such that a1 < x1 < a2 < x2 < a3 < x3 < · · · < xn−1 < an . 7. (BEL 2) Let ABCD be any quadrilateral. A square is constructed on each side of the quadrilateral, all in the same manner (i.e., outward or inward). Denote the centers of the squares by M1 , M2 , M3 , and M4 . Prove: (a) M1 M3 = M2 M4 ; (b) M1 M3 is perpendicular to M2 M4 . 8. (BEL 3) (SL70-1). 9. (BEL 4) If n is even, prove that   1 1 1 1 1 1 1 1 1 − + − + ···− = 2 + + + ···+ . 2 3 4 n n+2 n+4 n+6 2n 10. (BEL 5) Let A, B,C be angles of a triangle. Prove that 3 1 < cos A + cosB + cosC ≤ . 2 3.12 IMO 1970 63 11. (BEL 6) Let ABCD and A′ B′C′ D′ be two squares in the same plane and oriented in the same direction. Let A′′ , B′′ ,C′′ , and D′′ be the midpoints of AA′ , BB′ ,CC′ , and DD′ . Prove that A′′ B′′C′′ D′′ is also a square. 12. (BGR 1) Let x1 , x2 , x3 , x4 , x5 , x6 be given integers, not divisible by 7. Prove that at least one of the expressions of the form ±x1 ± x2 ± x3 ± x4 ± x5 ± x6 is divisible by 7, where the signs are selected in all possible ways. (Generalize the statement to every prime number!) 13. (BGR 2) A triangle ABC is given. Each side of ABC is divided into equal parts, and through each of the division points are drawn lines parallel to AB, BC, and CA, thus cutting ABC into small triangles. A number 1, 2, or 3 is assign to each of the vertices of these triangles in such a way that the following conditions are satisfied: (i) to A, B,C are assigned 1, 2 and 3 respectively; (ii) points on AB are marked by 1 or 2; (iii) points on BC are marked by 2 or 3; (iv) points on CA are marked by 3 or 1. Prove that there must exist a small triangle whose vertices are marked by 1, 2, and 3. 14. (BGR 3) Let α + β + γ = π . Prove that sin 2α + sin 2β + sin 2γ = 2(sin α + sin β + sin γ )(cos α + cos β + cos γ ) −2(sin α + sin β + sin γ ). 15. (BGR 4) Given a triangle ABC, let R be the radius of its circumcircle, O1 , O2 , O3 the centers √ of its exscribed circles, and q the perimeter of △O1 O2 O3 . Prove that q ≤ 6 3 R. 16. (BGR 5) Show that the equation p p 3 2 − x2 + 3 − x3 = 0 has no real roots. 17. (BGR 6) (SL70-3). Original formulation. In a triangular pyramid SABC one of the angles at S is right and the projection of S onto the base ABC is the orthocenter of ABC. Let r be the radius of the circle inscribed in the base, SA = m, SB = n, SC = p, H the height of the pyramid (through S), and r1 , r2 , r3 the radii of the circles inscribed in the intersections of the pyramid with the planes determined by the altitude of the pyramid and the lines SA, SB, SC respectively. Prove that: (a) m2 + n2 + p2 ≥ 18r2 ; (b) the ratios r1 /H, r2 /H, r3 /H lie in the interval [0.4, 0.5]. 64 3 Problems 18. (CZS 1) (SL70-4). 19. (CZS 2) Let n > 1 be a natural number, a ≥ 1 a real number, and x1 , x2 , . . . , xn x numbers such that x1 = 1, k+1 x = a + αk for k = 1, 2, . . . , n − 1, where αk are real numbers with αk ≤ 1 k(k+1) . k Prove that √ n−1 xn < a + 20. (CZS 3) (SL70-5). 1 . n−1 21. (CZS 4) Find necessary and sufficient conditions on given positive numbers u, v for the following claim to be valid: there exists a right-angled triangle △ABC with CD = u, CE = v, where D, E are points of the segments AB such that AD = DE = EB = 13 AB. 22. (FRA 1) (SL70-6). 23. (FRA 2) Let E be a finite set, PE the family of its subsets, and f a mapping from PE to the set of nonnegative real numbers such that for any two disjoint subsets A, B of E, f (A ∪ B) = f (A) + f (B). Prove that there exists a subset F of E such that if with each A ⊂ E we associate a subset A′ consisting of elements of A that are not in F, then f (A) = f (A′ ), and f (A) is zero if and only if A is a subset of F. 24. (FRA 3) Let n and p be two integers such that 2p ≤ n. Prove the inequality (n − p)! ≤ p!  n+1 2 n−2p . For which values does equality hold? 25. (FRA 4) Suppose that f is a real function defined for 0 ≤ x ≤ 1 having the first derivative f ′ for 0 ≤ x ≤ 1 and the second derivative f ′′ for 0 < x < 1. Prove that if f (0) = f ′ (0) = f ′ (1) = f (1) − 1 = 0, there exists a number 0 < y < 1 such that | f ′′ (y)| ≥ 4. 26. (FRA 5) Consider a finite set of vectors in space {a1 , a2 , . . . , an } and the set E of all vectors of the form x = λ1 a1 + λ2 a2 + · · · + λn an , where λi are nonnegative numbers. Let F be the set consisting of all the vectors in E and vectors parallel to a given plane P. Prove that there exists a set of vectors {b1 , b2 , . . . , b p } such that F is the set of all vectors y of the form y = µ 1 b1 + µ 2 b2 + · · · + µ p b p , where the µ j are nonnegative. 3.12 IMO 1970 65 27. (FRA 6) Find a natural number n such that for all prime numbers p, n is divisible by p if and only if n is divisible by p − 1. 28. (GDR 1) A set G with elements u, v, w, . . . is a group if the following conditions are fulfilled: (i) There is a binary algebraic operation ◦ defined on G such that for all u, v ∈ G there is a w ∈ G with u ◦ v = w. (ii) This operation is associative; i.e., for all u, v, w ∈ G, (u ◦ v) ◦ w = u ◦ (v ◦ w). (iii) For any two elements u, v ∈ G there exists an element x ∈ G such that u ◦ x = v, and an element y ∈ G such that y ◦ u = v. Let K be a set of all real numbers greater than 1. On K is defined an operation by q a ◦ b = ab − (a2 − 1)(b2 − 1). Prove that K is a group. 29. (GDR 2) Prove that the equation 4x + 6x = 9x has no rational solutions. 30. (GDR 3) (SL70-9). 31. (GDR 4) Prove that for any triangle with sides a, b, c and area P the following inequality holds: √ 3 P≤ (abc)2/3 . 4 Find all triangles for which equality holds. 32. (NLD 1) Let there be given an acute angle ∠AOB = 3α , where OA = OB. The point A is the center of a circle with radius OA. A line s parallel to OA passes through B. Inside the given angle a variable line t is drawn through O. It meets the circle in O and C and the given line s in D, where ∠AOC = x. Starting from an arbitrarily chosen position t0 of t, the series t0 ,t1 ,t2 , . . . is determined by defining BDi+1 = OCi for each i (in which Ci and Di denote the positions of C and D, corresponding to ti ). Making use of the graphical representations of BD and OC as functions of x, determine the behavior of ti for i → ∞. 33. (NLD 2) The vertices of a given square are clockwise lettered A, B,C, D. On the side AB is situated a point E such that AE = AB/3. Starting from an arbitrarily chosen point P0 on segment AE and going clockwise around the perimeter of the square, a series of points P0 , P1 , P2 , . . . is marked on the perimeter such that Pi Pi+1 = AB/3 for each i. It will be clear that when P0 is chosen in A or in E, then some Pi will coincide with P0 . Does this possibly also happen if P0 is chosen otherwise? 34. (NLD 3) In connection with a convex pentagon ABCDE we consider the set of ten circles, each of which contains three of the vertices of the pentagon on its circumference. Is it possible that none of these circles contains the pentagon? Prove your answer. 66 3 Problems 35. (NLD 4) Find for every value of n a set of numbers p for which the following statement is true: Any convex n-gon can be divided into p isosceles triangles. Alternative version. The same about division into p polygons with axis of symmetry. 36. (NLD 5) Let x, y, z be nonnegative real numbers satisfying x2 + y2 + z2 = 5 and yz + zx + xy = 2. Which values can the greatest of the numbers x2 − yz, y2 − xz, z2 − xy have? 37. (NLD 6) Solve the set of simultaneous equations v2 + w2 + w2 + 2 2 u +v + u2 + v2 + w2 + u2 + v2 + w2 + u2 + x2 + y2 x2 + y2 x2 + y2 y2 2 x = 6 − 2u, = 6 − 2v, = 6 − 2w, = 6 − 2x, = 6 − 2y. 38. (POL 1) Find the greatest integer A for which in any permutation of the numbers 1, . . . , 100 there exist ten consecutive numbers whose sum is at least A. 39. (POL 2) (SL70-8). 40. (POL 5) Let ABC be a triangle with angles α , β , γ commensurable with π . Starting from a point P interior to the triangle, a ball reflects on the sides of ABC, respecting the law of reflection that the angle of incidence is equal to the angle of reflection. Prove that, supposing that the ball never reaches any of the vertices A, B,C, the set of all directions in which the ball will move through time is finite. In other words, its path from the moment 0 to infinity consists of segments parallel to a finite set of lines. 41. (POL 6) Let a cube of side 1 be given. Prove that there exists a point A on the surface S of the cube such that every point of S can be joined to A by a path on S of length not exceeding 2. Also prove that there is a point of S that cannot be joined with A by a path on S of length less than 2. 42. (ROU 1) (SL70-2). 43. (ROU 2) Prove that the equation x3 − 3 tan π 2 π x − 3x + tan =0 12 12 π has one root x1 = tan 36 , and find the other roots. 44. (ROU 3) If a, b, c are side lengths of a triangle, prove that (a + b)(b + c)(c + a) ≥ 8(a + b − c)(b + c − a)(c + a − b). 3.12 IMO 1970 67 45. (ROU 4) Let M be an interior point of tetrahedron VABC. Denote by A1 , B1 ,C1 the points of intersection of lines MA, MB, MC with the planes V BC,VCA,VAB, and by A2 , B2 ,C2 the points of intersection of lines VA1 ,V B1 ,VC1 with the sides BC,CA, AB. (a) Prove that the volume of the tetrahedron VA2B2C2 does not exceed onefourth of the volume of VABC. (b) Calculate the volume of the tetrahedron V1 A1 B1C1 as a function of the volume of VABC, where V1 is the point of intersection of the line V M with the plane ABC, and M is the barycenter of VABC. 46. (ROU 5) Given a triangle ABC and a plane π having no common points with the triangle, find a point M such that the triangle determined by the points of intersection of the lines MA, MB, MC with π is congruent to the triangle ABC. 47. (ROU 6) Given a polynomial P(x) = ab(a − c)x3 + (a3 − a2c + 2ab2 − b2 c + abc)x2 +(2a2 b + b2c + a2c + b3 − abc)x + ab(b + c), where a, b, c 6= 0, prove that P(x) is divisible by Q(x) = abx2 + (a2 + b2)x + ab and conclude that P(x0 ) is divisible by (a + b)3 for x0 = (a + b + 1)n, n ∈ N. 48. (ROU 7) Let a polynomial p(x) with integer coefficients take the value 5 for five different integer values of x. Prove that p(x) does not take the value 8 for any integer x. 49. (SWE 1) For n ∈ N, let f (n) be the number of positive integers k ≤ n that do not contain the digit 9. Does there exist a positive real number p such that f (n) n ≥ p for all positive integers n? 50. (SWE 2) The area √ of a triangle is S and the sum of the lengths of its sides is L. Prove that 36S ≤ L2 3 and give a necessary and sufficient condition for equality. 51. (SWE 3) Let p be a prime number. A rational number x, with 0 < x < 1, is written in lowest terms. The rational number obtained from x by adding p to both the numerator and the denominator differs from x by 1/p2 . Determine all rational numbers x with this property. 52. (SWE 4) (SL70-10). 53. (SWE 5) A square ABCD is divided into (n − 1)2 congruent squares, with sides parallel to the sides of the given square. Consider the grid of all n2 corners obtained in this manner. Determine all integers n for which it is possible to construct a nondegenerate parabola with its axis parallel to one side of the square and that passes through exactly n points of the grid. 54. (SWE 6) (SL70-11). 68 3 Problems 55. (USS 1) A turtle runs away from an UFO with a speed of 0.2 m/s. The UFO flies 5 meters above the ground, with a speed of 20 m/s. The UFO’s path is a broken line, where after flying in a straight path of length ℓ (in meters) it may ℓ turn through for any acute angle α such that tan α < 1000 . When the UFO’s center approaches within 13 meters of the turtle, it catches the turtle. Prove that for any initial position the UFO can catch the turtle. 56. (USS 2) A square hole of depth h whose base is of length a is given. A dog is tied √ to the center of the square at the bottom of the hole by a rope of length L > 2a2 + h2 , and walks on the ground around the hole. The edges of the hole are smooth, so that the rope can freely slide along it. Find the shape and area of the territory accessible to the dog (whose size is neglected). 57. (USS 3) Let the numbers 1, 2, . . . , n2 be written in the cells of an n × n square board so that the entries in each column are arranged increasingly. What are the smallest and greatest possible sums of the numbers in the kth row? (k a positive integer, 1 ≤ k ≤ n.) 58. (USS 4) (SL70-12). 59. (USS 5) (SL70-7). 3.12.3 Shortlisted Problems 1. (BEL 3) Consider a regular 2n-gon and the n diagonals of it that pass through its center. Let P be a point of the inscribed circle and let a1 , a2 , . . . , an be the angles in which the diagonals mentioned are visible from the point P. Prove that n ∑ tan2 ai = 2n i=1 π cos2 2n π sin4 2n . 2. (ROU 1)IMO2 Let a and b be the bases of two number systems and let An = x1 x2 . . . xn (a) , An+1 = x0 x1 x2 . . . xn (a) , Bn = x1 x2 . . . xn (b) , Bn+1 = x0 x1 x2 . . . xn (b) , be numbers in the number systems with respective bases a and b, so that x0 , x1 , x2 , . . . , xn denote digits in the number system with base a as well as in the number system with base b. Suppose that neither x0 nor x1 is zero. Prove that n n a > b if and only if AAn+1 < BBn+1 . 3. (BGR 6)IMO5 In the tetrahedron SABC the angle BSC is a right angle, and the projection of the vertex S to the plane ABC is the intersection of the altitudes of the triangle ABC. Let z be the radius of the inscribed circle of the triangle ABC. Prove that SA2 + SB2 + SC2 ≥ 18z2 . 4. (CZS 1)IMO4 For what natural numbers n can the product of some of the numbers n, n + 1, n + 2, n + 3, n + 4, n + 5 be equal to the product of the remaining ones? 3.12 IMO 1970 69 5. (CZS 3) Let M be an interior point of the tetrahedron ABCD. Prove that −→ −→ MA vol(MBCD) + MB vol(MACD) −→ −−→ +MC vol(MABD) + MD vol(MABC) = 0 (vol(PQRS) denotes the volume of the tetrahedron PQRS). 6. (FRA 1) In the triangle ABC let B′ and C′ be the midpoints of the sides AC and AB respectively and H the foot of the altitude passing through the vertex A. Prove that the circumcircles of the triangles AB′C′ , BC′ H, and B′CH have a common point I and that the line HI passes through the midpoint of the segment B′C′ . 7. (USS 5) For which digits a do exist integers n ≥ 4 such that each digit of equals a? n(n+1) 2 8. (POL 2)IMO1 Given a point M on the side AB of the triangle ABC, let r1 and r2 be the radii of the inscribed circles of the triangles ACM and BCM respectively and let ρ1 and ρ2 be the radii of the excircles of the triangles ACM and BCM at the sides AM and BM respectively. Let r and ρ denote the radii of the inscribed circle and the excircle at the side AB of the triangle ABC respectively. Prove that r1 r2 r = . ρ1 ρ2 ρ 9. (GDR 3) Let u1 , u2 , . . . , un , v1 , v2 , . . . , vn be real numbers. Prove that ! ! n n n 4 1 + ∑ (ui + vi )2 ≤ 1 + ∑ u2i 1 + ∑ v2i . 3 i=1 i=1 i=1 In what case does equality hold? 10. (SWE 4)IMO3 Let 1 = a0 ≤ a1 ≤ a2 ≤ · · · ≤ an ≤ · · · be a sequence of real numbers. Consider the sequence b1 , b2 , . . . defined by:  n  ak−1 1 bn = ∑ 1 − √ . a ak k k=1 Prove that: (a) For all natural numbers n, 0 ≤ bn < 2. (b) Given an arbitrary 0 ≤ b < 2, there is a sequence a0 , a1 , . . . , an , . . . of the above type such that bn > b is true for infinitely many natural numbers n. 11. (SWE 6) Let P, Q, R be polynomials and let S(x) = P(x3 ) + xQ(x3 ) + x2 R(x3 ) be a polynomial of degree n whose roots x1 , . . . , xn are distinct. Construct with the aid of the polynomials P, Q, R a polynomial T of degree n that has the roots x31 , x32 , . . . , x3n . 12. (USS 4)IMO6 We are given 100 points in the plane, no three of which are on the same line. Consider all triangles that have all vertices chosen from the 100 given points. Prove that at most 70% of these triangles are acute angled. 70 3 Problems 3.13 The Thirteenth IMO Bratislava–Zilina, Czechoslovakia, July 10–21, 1971 3.13.1 Contest Problems First Day (July 13) 1. Prove that the following statement is true for n = 3 and for n = 5, and false for all other n > 2: For any real numbers a1 , a2 , . . . , an , (a1 − a2 )(a1 − a3 ) · · · (a1 − an ) + (a2 − a1 )(a2 − a3) · · · (a2 − an ) + . . . +(an − a1 )(an − a2 ) · · · (an − an−1 ) ≥ 0. 2. Given a convex polyhedron P1 with 9 vertices A1 , . . . , A9 , let us denote by P2 , P3 , . . . , P9 the images of P1 under the translations mapping the vertex A1 to A2 , A3 , . . . , A9 , respectively. Prove that among the polyhedra P1 , . . . , P9 at least two have a common interior point. 3. Prove that the sequence 2n − 3 (n > 1) contains a subsequence of numbers relatively prime in pairs. Second Day (July 14) 4. Given a tetrahedron ABCD all of whose faces are acute-angled triangles, set σ = ∡DAB + ∡BCD − ∡ABC − ∡CDA. Consider all closed broken lines XY ZT X whose vertices X ,Y, Z, T lie in the interior of segments AB, BC,CD, DA respectively. Prove that: (a) if σ 6= 0, then there is no broken line XY ZT of minimal length; (b) if σ = 0, then there are infinitely many such broken lines of minimal length. That length equals 2AC sin(α /2), where α = ∡BAC + ∡CAD + ∡DAB. 5. Prove that for every natural number m ≥ 1 there exists a finite set Sm of points in the plane satisfying the following condition: If A is any point in Sm , then there are exactly m points in Sm whose distance to A equals 1. 6. Consider the n × n array of nonnegative integers   a11 a12 . . . a1n  a21 a22 · · · a2n     .. .. ..  ,  . . .  an1 an2 . . . ann with the following property: If an element ai j is zero, then the sum of the elements of the ith row and the jth column is greater than or equal to n. Prove that the sum of all the elements is greater than or equal to 12 n2 . 3.13 IMO 1971 71 3.13.2 Longlisted Problems 1. (AUT 1) The points S(i, j) with integer Cartesian coordinates 0 < i ≤ n, 0 < j ≤ m, m ≤ n, form a lattice. Find the number of: (a) rectangles with vertices on the lattice and sides parallel to the coordinate axes; (b) squares with vertices on the lattice and sides parallel to the coordinate axes; (c) squares in total, with vertices on the lattice. 2. (AUT 2) Let us denote by s(n) = ∑d|n d the sum of divisors of a natural number n (1 and n included). If n has at most 5 distinct prime divisors, prove that s(n) < 77 76 16 n. Also prove that there exists a natural number n for which s(n) > 16 n holds. 3. (AUT 3) Let a, b, c be positive real numbers, 0 < a ≤ b ≤ c. Prove that for any positive real numbers x, y, z the following inequality holds: (ax + by + cz) n x a + y z (a + c)2 + ≤ (x + y + z)2 . b c 4ac 4. (BGR 1) Let xn = 22 + 1 and let m be the least common multiple of x2 , x3 , . . . , x1971 . Find the last digit of m. 5. (BGR 2) (SL71-1). Original formulation. Consider a sequence of polynomials X0 (x), X1 (x), X2 (x), . . . , Xn (x), . . . , where X0 (x) = 2, X1 (x) = x, and for every n ≥ 1 the following equality holds: 1 Xn (x) = (Xn+1 (x) + Xn−1(x)) . x Prove that (x2 − 4)[Xn2 (x) − 4] is a square of a polynomial for all n ≥ 0. 6. (BGR 3) Let squares be constructed on the sides BC,CA, AB of a triangle ABC, all to the outside of the triangle, and let A1 , B1 ,C1 be their centers. Starting from the triangle A1 B1C1 one analogously obtains a triangle A2 B2C2 . If S, S1 , S2 denote the areas of triangles ABC, A1 B1C1 , A2 B2C2 , respectively, prove that S = 8S1 − 4S2 . 7. (BGR 4) In a triangle ABC, let H be its orthocenter, O its circumcenter, and R its circumradius. pProve that: (a) |OH| = R 1 − 8 cos α cos β cos γ , where α , β , γ are angles of the triangle ABC; (b) O ≡ H if and only if ABC is equilateral. 8. (BGR 5) (SL71-2). Original formulation. Prove that for every natural number n ≥ 1 there exists an infinite sequence M1 , M2 , . . . , Mk , . . . of distinct points in the plane such that for all i, exactly n among these points are at distance 1 from Mi . 9. (BGR 6) The base of an inclined prism is a triangle ABC. The perpendicular projection of B1 , one of the top vertices, is the midpoint of BC. The dihedral 72 3 Problems angle between the lateral faces through BC and AB is α , and the lateral edges of the prism make an angle β with the base. If r1 , r2 , r3 are exradii of a perpendicular section of the prism, assuming that in ABC, cos2 A + cos2 B + cos2 C = 1, ∠A < ∠B < ∠C, and BC = a, calculate r1 r2 + r1 r3 + r2 r3 . 10. (CUB 1) In how many different ways can three knights be placed on a chessboard so that the number of squares attacked would be maximal? 11. (CUB 2) Prove that n! cannot be the square of any natural number. 12. (CUB 3) A system of n numbers x1 , x2 , . . . , xn is given such that x1 = logxn−1 xn , x2 = logxn x1 , ... , xn = logxn−2 xn−1 . Prove that ∏nk=1 xk = 1. 13. (CUB 4) One Martian, one Venusian, and one Human reside on Pluto. One day they make the following conversation: Martian : I have spent 1/12 of my life on Pluto. Human : I also have. Venusian : Me too. Martian : But Venusian and I have spend much more time here than you, Human. Human : That is true. However, Venusian and I are of the same age. Venusian : Yes, I have lived 300 Earth years. Martian : Venusian and I have been on Pluto for the past 13 years. It is known that Human and Martian together have lived 104 Earth years. Find the ages of Martian, Venusian, and Human.5 14. (UNK 1) Note that 83 − 73 = 169 = 132 and 13 = 22 + 32 . Prove that if the difference between two consecutive cubes is a square, then it is the square of the sum of two consecutive squares. 15. (UNK 2) Let ABCD be a convex quadrilateral whose diagonals intersect at O at an angle θ . Let us set OA = a, OB = b, OC = c, and OD = d, c > a > 0, and d > b > 0. Show that if there exists a right circular cone with vertex V , with the properties: (1) its axis passes through O, and (2) its curved surface passes through A, B,C and D, then d 2 b2 (c + a)2 − c2 a2 (d + b)2 . ca(d − b)2 − db(c − a)2 p ca c+a ca c−a ca Show also that if d+b lies between db and db , and d−b = db , then for a suitable choice of θ , a right circular cone exists with properties (1) and (2). OV 2 = 16. (UNK 3) (SL71-4). Original formulation. Two (intersecting) circles are given and a point P through 5 The numbers in the problem are not necessarily in base 10. 3.13 IMO 1971 73 which it is possible to draw a straight line on which the circles intercept two equal chords. Describe a construction by straightedge and compass for the straight line and prove the validity of your construction. 17. (GDR 1) (SL71-3). Original formulation. Find all solutions of the system x + y + z = 3, x + y3 + z3 = 15, 3 x5 + y5 + z5 = 83. 18. (GDR 2) Let a1 , a2 , . . . , an be positive numbers, mg = (a1 a2 · · · an )1/n their geometric mean, and ma = (a1 + a2 + · · · + an )/n their arithmetic mean. Prove that (1 + mg )n ≤ (1 + a1) · · · (1 + an) ≤ (1 + ma)n . 19. (GDR 3) In a triangle P1 P2 P3 let Pi Qi be the altitude from Pi for i = 1, 2, 3 (Qi being the foot of the altitude). The circle with diameter Pi Qi meets the two corresponding sides at two points different from Pi . Denote the length of the segment whose endpoints are these two points by li . Prove that l1 = l2 = l3 . 20. (GDR 4) Let M be the circumcenter of a triangle ABC. The line through M perpendicular to CM meets the lines CA and CB at Q and P respectively. Prove that CP CQ AB = 2. CM CM PQ 21. (HUN 1) (SL71-5). 22. (HUN 2) We are given an n × n board, where n is an odd number. In each cell of the board either +1 or −1 is written. Let ak and bk denote the products of numbers in the kth row and in the kth column respectively. Prove that the sum a1 + a2 + · · · + an + b1 + b2 + · · · + bn cannot be equal to zero. 23. (HUN 3) Find all integer solutions of the equation x2 + y2 = (x − y)3 . 24. (HUN 4) Let A, B, and C denote the angles of a triangle. If sin2 A + sin2 B + sin2 C = 2, prove that the triangle is right-angled. 25. (HUN 5) Let ABC, AA1 A2 , BB1 B2 ,CC1C2 be four equilateral triangles in the plane satisfying only that they are all positively oriented (i.e., in the counterclockwise direction). Denote the midpoints of the segments A2 B1 , B2C1 ,C2 A1 by P, Q, R in this order. Prove that the triangle PQR is equilateral. 26. (HUN 6) An infinite set of rectangles in the Cartesian coordinate plane is given. The vertices of each of these rectangles have coordinates (0, 0), (p, 0), (p, q), (0, q) for some positive integers p, q. Show that there must exist two among them one of which is entirely contained in the other. 74 3 Problems 27. (HUN 7) (SL71-6). 28. (NLD 1) (SL71-7). Original formulation. A tetrahedron ABCD is given. The sum of angles of the tetrahedron at the vertex A (namely ∠BAC, ∠CAD, ∠DAB) is denoted by α , and β , γ , δ are defined analogously. Let P, Q, R, S be variable points on edges of the tetrahedron: P on AD, Q on BD, R on BC, and S on AC, none of them at some vertex of ABCD. Prove that: (a) if α + β 6= 2π , then PQ + QR + RS + SP attains no minimal value; (b) if α + β = 2π , then AB sin α γ = CD sin 2 2 and PQ + QR + RS + SP ≥ 2AB sin α . 2 29. (NLD 2) A rhombus with its incircle is given. At each vertex of the rhombus a circle is constructed that touches the incircle and two edges of the rhombus. These circles have radii r1 , r2 , while the incircle has radius r. Given that r1 and r2 are natural numbers and that r1 r2 = r, find r1 , r2 , and r. 30. (NLD 3) Prove that the system of equations 2yz + x − y − z = a, 2xz − x + y − z = a, 2xy − x − y + z = a, a being a parameter, cannot have five distinct solutions. For what values of a does this system have four distinct integer solutions? 31. (NLD 4) (SL71-8). 32. (NLD 5) Two half-lines a and b, with the common endpoint O, make an acute angle α . Let A on a and B on b be points such that OA = OB, and let b′ be the line through A parallel to b. Let β be the circle with center B and radius BO. We construct a sequence of half-lines c1 , c2 , c3 , . . . , all lying inside the angle α , in the following manner: (i) c1 is given arbitrarily; (ii) for every natural number k, the circle β intercepts on ck a segment that is of the same length as the segment cut on b′ by a and ck+1 . Prove that the angle determined by the lines ck and b has a limit as k tends to infinity and find that limit. 33. (NLD 6) A square 2n × 2n grid is given. Let us consider all possible paths along grid lines, going from the center of the grid to the border, such that (1) no point of the grid is reached more than once, and (2) each of the squares homothetic to the grid having its center at the grid center is passed through only once. (a) Prove that the number of all such paths is equal to 4 ∏ni=2 (16i − 9). (b) Find the number of pairs of such paths that divide the grid into two congruent figures. (c) How many quadruples of such paths are there that divide the grid into four congruent parts? 3.13 IMO 1971 75 34. (POL 1) (SL71-9). 35. (POL 2) (SL71-10). 36. (POL 3) (SL71-11). 37. (POL 4) Let S be a circle, and α = {A1 , . . . , An } a family of open arcs in S. Let N(α ) = n denote the number of elements in α . We say that α is a covering of S S if nk=1 Ak ⊃ S. Let α = {A1 , . . . , An } and β = {B1 , . . . , Bm } be two coverings of S. Show that we can choose from the family of all sets Ai ∩ B j , i = 1, 2, . . . , n, j = 1, 2, . . . , m, a covering γ of S such that N(γ ) ≤ N(α ) + N(β ). 38. (POL 5) Let A, B,C be three points with integer coordinates in the plane and K a circle with radius R passing through A, B,C. Show that AB · BC ·CA ≥ 2R, and if the center of K is in the origin of the coordinates, show that AB · BC ·CA ≥ 4R. 39. (POL 6) (SL71-12). 40. (SWE 1) Prove that       1 1 1 1 1 1− 3 1− 3 1 − 3 ··· 1 − 3 > , 2 3 4 n 2 n = 2, 3, . . . . 41. (SWE 2) Consider the set of grid points (m, n) in the plane, m, n integers. Let σ be a finite subset and define S(σ ) = ∑ (m,n)∈σ (100 − |m| − |n|). Find the maximum of S, taken over the set of all such subsets σ . 42. (SWE 3) Let Li , i = 1, 2, 3, be line segments on the sides of an equilateral triangle, one segment on each side, with lengths li , i = 1, 2, 3. By L∗i we denote the segment of length li with its midpoint on the midpoint of the corresponding side of the triangle. Let M(L) be the set of points in the plane whose orthogonal projections on the sides of the triangle are in L1 , L2 , and L3 , respectively; M(L∗ ) is defined correspondingly. Prove that if l1 ≥ l2 + l3 , we have that the area of M(L) is less than or equal to the area of M(L∗ ). 43. (SWE 4) Show that for nonnegative real numbers a, b and integers n ≥ 2,   a n + bn a+b n ≥ . 2 2 When does equality hold? 44. (SWE 5) (SL71-13). 45. (SWE 6) Let m and n denote integers greater than 1, and let ν (n) be the number n of primes less than or equal to n. Show that if the equation ν (n) = m has a solution n (in n), then so does the equation ν (n) = m − 1. 76 3 Problems 46. (USS 1) (SL71-14). 47. (USS 2) (SL71-15). 48. (USS 3) q A sequence of real numbers x1 , x2 , . . . , xn is given such that xi+1 = 1 xi + 30000 1 − x2i , i = 1, 2, . . . , and x1 = 0. Can n be equal to 50000 if xn < 1? 49. (USS 4) Diagonals of a convex quadrilateral ABCD intersect at a point O. Find all angles of this quadrilateral if ∡OBA = 30◦ , ∡OCB = 45◦ , ∡ODC = 45◦ , and ∡OAD = 30◦ . 50. (USS 5) (SL71-16). 51. (USS 6) Suppose that the sides AB and DC of a convex quadrilateral ABCD are not parallel. On the sides BC and AD, pairs of points (M, N) and (K, L) are chosen such that BM = MN = NC and AK = KL = LD. Prove that the areas of triangles OKM and OLN are different, where O is the intersection point of AB and CD. 52. (YUG 1) (SL71-17). 53. (YUG 2) Denote by xn (p) the multiplicity of the prime p in the canonical rep1 resentation of the number n! as a product of primes. Prove that xn n(p) < p−1 and limn→∞ xn n(p) = 1 p−1 .  54. (YUG 3) A set M is formed of 2n n men, n = 1, 2, . . .. Prove that we can choose a subset P of the set M consisting of n + 1 men such that one of the following conditions is satisfied: (1) every member of the set P knows every other member of the set P; (2) no member of the set P knows any other member of the set P. 55. (YUG 4) Prove that the polynomial x4 + λ x3 + µ x2 + √ν x + 1 has no real roots if λ , µ , ν are real numbers satisfying |λ | + |µ | + |ν | ≤ 2. 3.13.3 Shortlisted Problems 1. (BGR 2) Consider a sequence of polynomials P0 (x), P1 (x), P2 (x), . . . , Pn (x), . . . , where P0 (x) = 2, P1 (x) = x and for every n ≥ 1 the following equality holds: Pn+1 (x) + Pn−1(x) = xPn (x). Prove that there exist three real numbers a, b, c such that for all n ≥ 1, (x2 − 4)[Pn2 (x) − 4] = [aPn+1 (x) + bPn (x) + cPn−1 (x)]2 . (1) 2. (BGR 5)IMO5 Prove that for every natural number m ≥ 1 there exists a finite set Sm of points in the plane satisfying the following condition: If A is any point in Sm , then there are exactly m points in Sm whose distance to A equals 1. 3. (GDR 1) Knowing that the system 3.13 IMO 1971 77 x + y + z = 3, 3 x + y3 + z3 = 15, x4 + y4 + z4 = 35, has a real solution x, y, z for which x2 + y2 + z2 < 10, find the value of x5 + y5 + z5 for that solution. 4. (UNK 3) We are given two mutually tangent circles in the plane, with radii r1 , r2 . A line intersects these circles in four points, determining three segments of equal length. Find this length as a function of r1 and r2 and the condition for the solvability of the problem. 5. (HUN 1)IMO1 Let a, b, c, d, e be real numbers. Prove that the expression (a − b)(a − c)(a − d)(a − e) + (b − a)(b − c)(b − d)(b − e) +(c − a)(c − b)(c − d)(c − e) + (d − a)(d − b)(d − c)(d − e) +(e − a)(e − b)(e − c)(e − d). is nonnegative. 6. (HUN 7) Let n ≥ 2 be a natural number. Find a way to assign natural numbers to the vertices of a regular 2n -gon such that the following conditions are satisfied: (i) only digits 1 and 2 are used; (ii) each number consists of exactly n digits; (iii) different numbers are assigned to different vertices; (iv) the numbers assigned to two neighboring vertices differ at exactly one digit. 7. (NLD 1)IMO4 Given a tetrahedron ABCD all of whose faces are acute-angled triangles, set σ = ∡DAB + ∡BCD − ∡ABC − ∡CDA. Consider all closed broken lines XY ZT X whose vertices X ,Y, Z, T lie in the interior of segments AB, BC,CD, DA respectively. Prove that: (a) if σ 6= 0, then there is no broken line XY ZT of minimal length; (b) if σ = 0, then there are infinitely many such broken lines of minimal length. That length equals 2AC sin(α /2), where α = ∡BAC + ∡CAD + ∡DAB. 8. (NLD 4) Determine whether there exist distinct real numbers a, b, c,t for which: (i) the equation ax2 + btx + c = 0 has two distinct real roots x1 , x2 , (ii) the equation bx2 + ctx + a = 0 has two distinct real roots x2 , x3 , (iii) the equation cx2 + atx + b = 0 has two distinct real roots x3 , x1 . 9. (POL 1) Let Tk = k − 1 for k = 1, 2, 3, 4 and T2k−1 = T2k−2 + 2k−2 , Show that for all k, T2k = T2k−5 + 2k (k ≥ 3). 78 3 Problems  12 n−1 1 + T2n−1 = 2 7  and  17 n−1 1 + T2n = 2 , 7 where [x] denotes the greatest integer not exceeding x.  10. (POL 2)IMO3 Prove that the sequence 2n − 3 (n > 1) contains a subsequence of numbers relatively prime in pairs. 11. (POL 3) The matrix  a11 . . .  ..  . ··· an1 . . .  a1n ..  .  ann n satisfies the inequality ∑ j=1 |a j1 x1 +· · · + a jn xn | ≤ M xi equal to ±1. Show that for each choice of numbers |a11 + a22 + · · · + ann| ≤ M. 12. (POL 6) Two congruent equilateral triangles ABC and A′ B′C′ in the plane are given. Show that the midpoints of the segments AA′ , BB′ ,CC′ either are collinear or form an equilateral triangle. 13. (SWE 5)IMO6 Consider the n × n array of nonnegative integers   a11 a12 . . . a1n  a21 a22 · · · a2n     .. .. ..  ,  . . .  an1 an2 . . . ann with the following property: If an element ai j is zero, then the sum of the elements of the ith row and the jth column is greater than or equal to n. Prove that the sum of all the elements is greater than or equal to 12 n2 . 14. (USS 1) A broken line A1 A2 . . . An is drawn in a 50 × 50 square, so that the distance from any point of the square to the broken line is less than 1. Prove that its total length is greater than 1248. 15. (USS 2) Natural numbers from 1 to 99 (not necessarily distinct) are written on 99 cards. It is given that the sum of the numbers on any subset of cards (including the set of all cards) is not divisible by 100. Show that all the cards contain the same number. 16. (USS 5)IMO2 Given a convex polyhedron P1 with 9 vertices A1 , . . . , A9 , let us denote by P2 , P3 , . . . , P9 the images of P1 under the translations mapping the vertex A1 to A2 , A3 , . . . , A9 respectively. Prove that among the polyhedra P1 , . . . , P9 at least two have a common interior point. 17. (YUG 1) Prove the inequality a1 + a3 a 2 + a 4 a3 + a1 a 4 + a 2 + + + ≥ 4, a1 + a2 a 2 + a 3 a3 + a4 a 4 + a 1 where ai > 0, i = 1, 2, 3, 4. 3.14 IMO 1972 79 3.14 The Fourteenth IMO Warsaw–Toruń, Poland, July 5–17, 1972 3.14.1 Contest Problems First Day (July 10) 1. A set of 10 positive integers is given such that the decimal expansion of each of them has two digits. Prove that there are two disjoint subsets of the set with equal sums of their elements. 2. Prove that for each n ≥ 4 every cyclic quadrilateral can be decomposed into n cyclic quadrilaterals. 3. Let m and n be nonnegative integers. Prove that (2m)!(2n)! m!n!(m+n)! is an integer (0! = 1). Second Day (July 11) 4. Find all solutions in positive real numbers xi (i = 1, 2, 3, 4, 5) of the following system of inequalities: (x21 − x3 x5 )(x22 − x3 x5 ) ≤ 0 (x22 − x4 x1 )(x23 − x4 x1 ) ≤ 0 (x23 − x5 x2 )(x24 − x5 x2 ) ≤ 0 (x24 − x1 x3 )(x25 − x1 x3 ) ≤ 0 (x25 − x2 x4 )(x21 − x2 x4 ) ≤ 0. (i) (ii) (iii) (iv) (v) 5. Let f and ϕ be real functions defined in the interval (−∞, ∞) satisfying the functional equation f (x + y) + f (x − y) = 2ϕ (y) f (x), for arbitrary real x, y (give examples of such functions). Prove that if f (x) is not identically 0 and | f (x)| ≤ 1 for all x, then |ϕ (x)| ≤ 1 for all x. 6. Given four distinct parallel planes, show that a regular tetrahedron exists with a vertex on each plane. 3.14.2 Longlisted Problems 1. (BGR 1) Find all integer solutions of the equation 1 + x + x2 + x3 + x4 = y4 . 2. (BGR 2) Find all real values of the parameter a for which the system of equations x4 = yz − x2 + a, y4 = zx − y2 + a, has at most one real solution. z4 = xy − z2 + a, 80 3 Problems 3. (BGR 3) On a line a set of segments is given of total length less than 1. Prove that every set of n points of the line can be translated in some direction along the line for a distance smaller than n/2 so that none of the points remain on the segments. 4. (BGR 4) Given a triangle, prove that the points of intersection of three pairs of trisectors of the inner angles at the sides lying closest to those sides are vertices of an equilateral triangle. 5. (BGR 5) Given a pyramid whose base is an n-gon inscribable in a circle, let H be the projection of the top vertex of the pyramid to its base. Prove that the projections of H to the lateral edges of the pyramid lie on a circle. 6. (BGR 6) Prove the inequality (n + 1) cos π π − n cos > 1 n+1 n for all natural numbers n ≥ 2. 7. (BGR 7) (SL72-1). 8. (CZS 1) (SL72-2). 9. (CZS 2) Given natural numbers k and n, k ≤ n, n ≥ 3, find the set of all values in the interval (0, π ) that the kth-largest among the interior angles of a convex ngon can take. 10. (CZS 3) Given five points in the plane, no three of which are collinear, prove that there can be found at least two obtuse-angled triangles with vertices at the given points. Construct an example in which there are exactly two such triangles. 11. (CZS 4) (SL72-3). 12. (CZS 5) A circle k = (S, r) is given and a hexagon AA′ BB′CC′ inscribed in it. The lengths of sides of the hexagon satisfy AA′ = A′ B, BB′ = B′C, CC′ = C′ A. Prove that the area P of triangle ABC is not greater than the area P′ of triangle A′ B′C′ . When does P = P′ hold? 13. (CZS 6) Given a sphere K, determine the set of all points A that are vertices of some parallelograms ABCD that satisfy AC ≤ BD and whose entire diagonal BD is contained in K. 14. (UNK 1) (SL72-7). 15. (UNK 2) (SL72-8). 16. (UNK 3) Consider the set S of all the different odd positive integers that are not multiples of 5 and that are less than 30m, m being a positive integer. What is the smallest integer k such that in any subset of k integers from S there must be two integers one of which divides the other? Prove your result. 17. (UNK 4) A solid right circular cylinder with height h and base-radius r has a solid hemisphere of radius r resting upon it. The center of the hemisphere O is on 3.14 IMO 1972 81 the axis of the cylinder. Let P be any point on the surface of the hemisphere and Q the point on the base circle of the cylinder that is furthest from P (measuring along the surface of the combined solid). A string is stretched over the surface from P to Q so as to be as short as possible. Show that if the string is not in a plane, the straight line PO when produced cuts the curved surface of the cylinder. 18. (UNK 5) We have p players participating in a tournament, each player playing against every other player exactly once. A point is scored for each victory, and there are no draws. A sequence of nonnegative integers s1 ≤ s2 ≤ s3 ≤ · · · ≤ s p is given. Show that it is possible for this sequence to be a set of final scores of the players in the tournament if and only if p (i) 1 ∑ si = 2 p(p − 1) k and (ii) for all k < p, i=1 1 ∑ si ≥ 2 k(k − 1). i=1 19. (UNK 6) Let S be a subset of the real numbers with the following properties: (i) If x ∈ S and y ∈ S, then x − y ∈ S; (ii) If x ∈ S and y ∈ S, then xy ∈ S; (iii) S contains an exceptional number x′ such that there is no number y in S satisfying x′ y + x′ + y = 0; (iv) If x ∈ S and x 6= x′ , there is a number y in S such that xy + x + y = 0. Show that (a) S has more than one number in it; (b) x′ 6= −1 leads to a contradiction; (c) x ∈ S and x 6= 0 implies 1/x ∈ S. 20. (GDR 1) (SL72-4). 21. (GDR 2) (SL72-5). 22. (GDR 3) (SL72-6). 23. (MNG 1) Does there exist a 2n-digit number a2n a2n−1 . . . a1 (for an arbitrary n) for which the following equality holds: a2n . . . a1 = (an . . . a1 )2 ? 24. (MNG 2) The diagonals of a convex 18-gon are colored in 5 different colors, each color appearing on an equal number of diagonals. The diagonals of one color are numbered 1, 2, . . .. One randomly chooses one-fifth of all the diagonals. Find the number of possibilities for which among the chosen diagonals there exist exactly n pairs of diagonals of the same color and with fixed indices i, j. 25. (NLD 1) We consider n real variables xi (1 ≤ i ≤ n), where n is an integer and n ≥ 2. The product of these variables will be denoted by p, their sum by s, and the sum of their squares by S. Furthermore, let α be a positive constant. We now study the inequality ps ≤ Sα . Prove that it holds for every n-tuple (xi ) if and only if α = n+1 2 . 26. (NLD 2) (SL72-9). 82 3 Problems 27. (NLD 3) (SL72-10). 28. (NLD 4) The lengths of the sides of a rectangle are given to be odd integers. Prove that there does not exist a point within that rectangle that has integer distances to each of its four vertices. 29. (NLD 5) Let A, B,C be points on the sides B1C1 ,C1 A1 , A1 B1 of a triangle A1 B1C1 such that A1 A, B1 B,C1C are the bisectors of angles of the triangle. We have that AC = BC and A1C1 6= B1C1 . (a) Prove that C1 lies on the circumcircle of the triangle ABC. (b) Suppose that ∡BAC1 = π /6; find the form of triangle ABC. 30. (NLD 6) (SL72-11). 31. (ROU 1) Find values of n ∈ N for which the fraction 3n −2 2n −3 is reducible. 32. (ROU 2) If n1 , n2 , . . . , nk are natural numbers and n1 + n2 + · · · + nk = n, show that max n1 n2 · · · nk = (t + 1)r t k−r , n1 +···+nk =n where t = [n/k] and r is the remainder of n upon division by k; i.e., n = tk + r, 0 ≤ r ≤ k − 1. 33. (ROU 3) A rectangle ABCD is given whose sides have lengths 3 and 2n, where n is a natural number. Denote by U(n) the number of ways in which one can cut the rectangle into rectangles of side lengths 1 and 2. (a) Prove that U(n + 1) +U (n √ − 1) = 4U (n); √ √ √ (b) Prove that U (n) = √1 [( 3 + 1)(2 + 3)n + ( 3 − 1)(2 − 3)n ]. 2 3 34. (ROU 4) If p is a prime number greater than 2 and a, b, c integers not divisible by p, prove that the equation ax2 + by2 = pz + c has an integer solution. 35. (ROU 5) (a) Prove that for a, b, c, d ∈ R, m ∈ [1, +∞) with am+ b = −cm+ d = m, √ p √ 4m2 (i) a2 + b2 + c2 + d 2 + (a − c)2 + (b − d)2 ≥ 1+m 2 , and 2 4m (ii) 2 ≤ 1+m 2 < 4. (b) Express a, b, c, d as functions of m so that there is equality in (1). 36. (ROU 6) A finite number of parallel segments in the plane are given with the property that for any three of the segments there is a line intersecting each of them. Prove that there exists a line that intersects all the given segments. 37. (SWE 1) On a chessboard (8 × 8 squares with sides of length 1) two diagonally opposite corner squares are taken away. Can the board now be covered with nonoverlapping rectangles with sides of lengths 1 and 2? 3.14 IMO 1972 83 38. (SWE 2) Congruent rectangles with sides m (cm) and n (cm) are given (m, n positive integers). Characterize the rectangles that can be constructed from these rectangles (in the fashion of a jigsaw puzzle). (The number of rectangles is unbounded.) 39. (SWE 3) How many tangents to the curve y = x3 − 3x (y = x3 + px) can be drawn from different points in the plane? 40. (SWE 4) Prove the inequalities u sin u π u ≤ ≤ , v sin v 2v for 0 ≤ u < v ≤ π . 2 41. (SWE 5) The ternary expansion x = 0.10101010 . . . is given. Give the binary expansion of x. Alternatively, transform the binary expansion y = 0.110110110 . . . into a ternary expansion. 42. (SWE 6) The decimal number 13101 is given. It is instead written as a ternary number. What are the two last digits of this ternary number? 43. (USS 1) A fixed point A inside a circle is given. Consider all chords XY of the circle such that ∠XAY is a right angle, and for all such chords construct the point M symmetric to A with respect to XY . Find the locus of points M. 44. (USS 2) (SL72-12). 45. (USS 3) Let ABCD be a convex quadrilateral whose diagonals AC and BD intersect at point O. Let a line through O intersect segment AB at M and segment CD at N. Prove that the segment MN is not longer than at least one of the segments AC and BD. 46. (USS 4) Numbers 1, 2, . . . , 16 are written in a 4 × 4 square matrix so that the sum of the numbers in every row, every column, and every diagonal is the same and furthermore that the numbers 1 and 16 lie in opposite corners. Prove that the sum of any two numbers symmetric with respect to the center of the square equals 17. 3.14.3 Shortlisted Problems 1. (BGR 7)IMO5 Let f and ϕ be real functions defined on the set R satisfying the functional equation f (x + y) + f (x − y) = 2ϕ (y) f (x), (1) for arbitrary real x, y (give examples of such functions). Prove that if f (x) is not identically 0 and | f (x)| ≤ 1 for all x, then |ϕ (x)| ≤ 1 for all x. 2. (CZS 1) We are given 3n points A1 , A2 , . . . , A3n in the plane, no three of them collinear. Prove that one can construct n disjoint triangles with vertices at the points Ai . 84 3 Problems 3. (CZS 4) Let x1 , x2 , . . . , xn be real numbers satisfying x1 + x2 + · · · + xn = 0. Let m be the least and M the greatest among them. Prove that x21 + x22 + · · · + x2n ≤ −nmM. 4. (GDR 1) Let n1 , n2 be positive integers. Consider in a plane E two disjoint sets of points M1 and M2 consisting of 2n1 and 2n2 points, respectively, and such that no three points of the union M1 ∪ M2 are collinear. Prove that there exists a straight line g with the following property: Each of the two half-planes determined by g on E (g not being included in either) contains exactly half of the points of M1 and exactly half of the points of M2 . 5. (GDR 2) Prove the following assertion: The four altitudes of a tetrahedron ABCD intersect in a point if and only if AB2 + CD2 = BC2 + AD2 = CA2 + BD2 . 6. (GDR 3) Show that for any n 6≡ 0 (mod 10) there exists a multiple of n not containing the digit 0 in its decimal expansion. 7. (UNK 1)IMO6 (a) A plane π passes through the vertex O of the regular tetrahedron OPQR. We define p, q, r to be the signed distances of P, Q, R from π measured along a directed normal to π . Prove that p2 + q2 + r2 + (q − r)2 + (r − p)2 + (p − q)2 = 2a2 , where a is the length of an edge of a tetrahedron. (b) Given four parallel planes not all of which are coincident, show that a regular tetrahedron exists with a vertex on each plane. 8. (UNK 2)IMO3 Let m and n be nonnegative integers. Prove that m!n!(m + n)! divides (2m)!(2n)!. 9. (NLD 2)IMO4 Find all solutions in positive real numbers xi (i = 1, 2, 3, 4, 5) of the following system of inequalities: (x21 − x3 x5 )(x22 − x3 x5 ) ≤ 0, (x22 − x4 x1 )(x23 − x4 x1 ) ≤ 0, (x23 − x5 x2 )(x24 − x5 x2 ) ≤ 0, (x24 − x1 x3 )(x25 − x1 x3 ) ≤ 0, (x25 − x2 x4 )(x21 − x2 x4 ) ≤ 0. (i) (ii) (iii) (iv) (v) 10. (NLD 3)IMO2 Prove that for each n ≥ 4 every cyclic quadrilateral can be decomposed into n cyclic quadrilaterals. 11. (NLD 6) Consider a sequence of circles K1 , K2 , K3 , K4 , . . . of radii r1 , r2 , r3 , r4 , . . . , respectively, situated inside a triangle ABC. The circle K1 is tangent to AB and AC; K2 is tangent to K1 , BA, and BC; K3 is tangent to K2 , CA, and CB; K4 is tangent to K3 , AB, and AC; etc. 3.14 IMO 1972 85 (a) Prove the relation   √ 1 1 1 1 r1 cot A + 2 r1 r2 + r2 cot B = r cot A + cot B , 2 2 2 2 where r is the radius of the incircle of the triangle ABC. Deduce the existence of a t1 such that 1 1 r1 = r cot B cot C sin2 t1 . 2 2 (b) Prove that the sequence of circles K1 , K2 , . . . is periodic. 12. (USS 2)IMO1 A set of 10 positive integers is given such that the decimal expansion of each of them has two digits. Prove that there are two disjoint subsets of the set with equal sums of their elements. 86 3 Problems 3.15 The Fifteenth IMO Moscow, Soviet Union, July 5–16, 1973 3.15.1 Contest Problems First Day (July 9) −−→ −−→ −−→ 1. Let O be a point on the line l and OP1 , OP2 , . . . , OPn unit vectors such that points P1 , P2 , . . . , Pn and line l lie in the same plane and all points Pi lie in the same half-plane determined by l. Prove that if n is odd, then −−→ −−→ −−→ OP1 + OP2 + · · · + OPn ≥ 1. −−→ −−→ ( OM is the length of vector OM). 2. Does there exist a finite set M of points in space, not all in the same plane, such that for each two points A, B ∈ M there exist two other points C, D ∈ M such that lines AB and CD are parallel but not equal? 3. Determine the minimum of a2 + b2 if a and b are real numbers for which the equation x4 + ax3 + bx2 + ax + 1 = 0 has at least one real solution. Second Day (July 10) 4. A soldier has to investigate whether there are mines in an area that has the form of equilateral triangle. The radius of his detector’s range is equal to one-half the altitude of the triangle. The soldier starts from one vertex of the triangle. Determine the shortest path through which the soldier has to pass in order to check the entire region. 5. Let G be a set of functions f : R → R of the form f (x) = ax + b, where a and b are real numbers and a 6= 0. Suppose that G satisfies the following conditions: (1) If f , g ∈ G, then g ◦ f ∈ G, where (g ◦ f )(x) = g[ f (x)]. (2) If f ∈ G and f (x) = ax + b, then the inverse f −1 of f belongs to G ( f −1 (x) = (x − b)/a). (3) For each f ∈ G there exists a number x f ∈ R such that f (x f ) = x f . Prove that there exists a number k ∈ R such that f (k) = k for all f ∈ G. 6. Let a1 , a2 , . . . , an be positive numbers and q a given real number, 0 < q < 1. Find n real numbers b1 , b2 , . . . , bn that satisfy: (1) ak < bk for all k = 1, 2, . . . , n; b 1 (2) q < k+1 b < q for all k = 1, 2, . . . , n − 1; k (3) b1 + b2 + · · · + bn < 1+q 1−q (a1 + a2 + · · · + an ). 3.15 IMO 1973 87 3.15.2 Shortlisted Problems 1. (BGR 6) Let a tetrahedron ABCD be inscribed in a sphere S. Find the locus of points P inside the sphere S for which the equality AP BP CP DP + + + =4 PA1 PB1 PC1 PD1 holds, where A1 , B1 ,C1 , and D1 are the intersection points of S with the lines AP, BP,CP, and DP, respectively. 2. (CZS 1) Given a circle K, find the locus of vertices A of parallelograms ABCD with diagonals AC ≤ BD, such that BD is inside K. 3. (CZS 6)IMO1 Prove that the sum of an odd number of unit vectors passing through the same point O and lying in the same half-plane whose border passes through O has length greater than or equal to 1. 4. (UNK 1) Let P be a set of 7 different prime numbers and C a set of 28 different composite numbers each of which is a product of two (not necessarily different) numbers from P. The set C is divided into 7 disjoint four-element subsets such that each of the numbers in one set has a common prime divisor with at least two other numbers in that set. How many such partitions of C are there? 5. (FRA 2) A circle of radius 1 is located in a right-angled trihedron and touches all its faces. Find the locus of centers of such circles. 6. (POL 2)IMO2 Does there exist a finite set M of points in space, not all in the same plane, such that for each two points A, B ∈ M there exist two other points C, D ∈ M such that lines AB and CD are parallel? 7. (POL 3) Given a tetrahedron ABCD, let x = AB · CD, y = AC · BD, and z = AD · BC. Prove that there exists a triangle with edges x, y, z. k  8. (ROU 1) Prove that there are exactly [k/2] arrays a1 , a2 , . . . , ak+1 of nonnegative integers such that a1 = 0 and |ai − ai+1| = 1 for i = 1, 2, . . . , k. 9. (ROU 2) Let Ox, Oy, Oz be three rays, and G a point inside the trihedron Oxyz. Consider all planes passing through G and cutting Ox, Oy, Oz at points A, B,C, respectively. How is the plane to be placed in order to yield a tetrahedron OABC with minimal perimeter? 10. (SWE 3)IMO6 Let a1 , a2 , . . . , an be positive numbers and q a given real number, 0 < q < 1. Find n real numbers b1 , b2 , . . . , bn that satisfy: (1) ak < bk for all k = 1, 2, . . . , n; b 1 (2) q < k+1 b < q for all k = 1, 2, . . . , n − 1; k (3) b1 + b2 + · · · + bn < 1+q 1−q (a1 + a2 + · · · + an ). 11. (SWE 4)IMO3 Determine the minimum of a2 + b2 if a and b are real numbers for which the equation x4 + ax3 + bx2 + ax + 1 = 0 88 3 Problems has at least one real solution. 12. (SWE 6) Consider the two square matrices     1 1 1 1 1 1 1 1 1 1  1 1 1 −1 −1   1 1 1 −1 −1         and B= A =  1 −1 −1 1 1   1 1 −1 1 −1   1 −1 −1 1 1   1 −1 −1 −1 1  1 −1 1 −1 1 1 1 −1 1 −1 with entries 1 and −1. The following operations will be called elementary: (1) Changing signs of all numbers in one row; (2) Changing signs of all numbers in one column; (3) Interchanging two rows (two rows exchange their positions); (4) Interchanging two columns. Prove that the matrix B cannot be obtained from the matrix A using these operations. 13. (YUG 4) Find the sphere of maximal radius that can be placed inside every tetrahedron that has all altitudes of length greater than or equal to 1. 14. (YUG 5)IMO4 A soldier has to investigate whether there are mines in an area that has the form of an equilateral triangle. The radius of his detector is equal to one-half of an altitude of the triangle. The soldier starts from one vertex of the triangle. Determine the shortest path that the soldier has to traverse in order to check the whole region. 15. (CUB 1) Prove that for all n ∈ N the following is true: n 2n ∏ sin k=1 √ kπ = 2n + 1. 2n + 1 16. (CUB 2) Given a, θ ∈ R, m ∈ N, and P(x) = x2m − 2|a|mxm cos θ + a2m, factorize P(x) as a product of m real quadratic polynomials. 17. (POL 1)IMO5 Let F be a nonempty set of functions f : R → R of the form f (x) = ax + b, where a and b are real numbers and a 6= 0. Suppose that F satisfies the following conditions: (1) If f , g ∈ F , then g ◦ f ∈ F , where (g ◦ f )(x) = g[ f (x)]. (2) If f ∈ F and f (x) = ax + b, then the inverse f −1 of f belongs to F ( f −1 (x) = (x − b)/a). (3) None of the functions f (x) = x + c, for c 6= 0, belong to F . Prove that there exists x0 ∈ R such that f (x0 ) = x0 for all f ∈ F . 3.16 IMO 1974 89 3.16 The Sixteenth IMO Erfurt–Berlin, DR Germany, July 4–17, 1974 3.16.1 Contest Problems First Day (July 8) 1. Alice, Betty, and Carol took the same series of examinations. There was one grade of A, one grade of B, and one grade of C for each examination, where A, B,C are different positive integers. The final test scores were Alice 20 Betty 10 Carol 9 If Betty placed first in the arithmetic examination, who placed second in the spelling examination? 2. Let △ABC be a triangle. Prove that there exists a point D on the side AB such that CD is the geometric mean of AD and BD if and only if √ C sin A sin B ≤ sin . 2 3. Prove that there does not exist a natural number n for which the number  n  2n + 1 3k 2 ∑ k=0 2k + 1 is divisible by 5. Second Day (July 9) 4. Consider a partition of an 8 × 8 chessboard into p rectangles whose interiors are disjoint such that each rectangle contains an equal number of white and black cells. Assume that a1 < a2 < · · · < a p , where ai denotes the number of white cells in the ith rectangle. Find the maximal p for which such a partition is possible and for that p determine all possible corresponding sequences a1 , a2 , . . . , a p . 5. If a, b, c, d are arbitrary positive real numbers, find all possible values of S= a b c d + + + . a+b+d a+b+c b+c+d a+c+d 6. Let P(x) be a polynomial with integer coefficients. If n(P) is the number of (distinct) integers k such that P2 (k) = 1, prove that n(P) − deg(P) ≤ 2, where deg(P) denotes the degree of the polynomial P. 90 3 Problems 3.16.2 Longlisted Problems 1. (BGR 1) (SL74-11). 2. (BGR 2) Let {un } be the Fibonacci sequence, i.e., u0 = 0, u1 = 1, un = un−1 + un−2 for n > 1. Prove that there exist infinitely many prime numbers p that divide u p−1 . 3. (BGR 3) Let ABCD be an arbitrary quadrilateral. Let squares ABB1 A2 , BCC1 B2 , CDD1C2 , DAA1 D2 be constructed in the exterior of the quadrilateral. Furthermore, let AA1 PA2 and CC1 QC2 be parallelograms. For any arbitrary point P in the interior of ABCD, parallelograms RASC and RPT Q are constructed. Prove that these two parallelograms have two vertices in common. 4. (BGR 4) Let Ka , Kb , Kc with centers Oa , Ob , Oc be the excircles of a triangle ABC, touching the interiors of the sides BC,CA, AB at points Ta , Tb , Tc respectively. Prove that the lines Oa Ta , Ob Tb , Oc Tc are concurrent in a point P for which POa = POb = POc = 2R holds, where R denotes the circumradius of ABC. Also prove that the circumcenter O of ABC is the midpoint of the segment PJ, where J is the incenter of ABC. 5. (BGR 5) A straight cone is given inside a rectangular parallelepiped B, with the apex at one of the vertices, say T , of the parallelepiped, and the base touching the three faces opposite to T . Its axis lies at the long diagonal through T . If V1 and V2 are the volumes of the cone and the parallelepiped respectively, prove √ that 3π V2 V1 ≤ . 27 6. (CUB 1) Prove that the product of two natural numbers with their sum cannot be the third power of a natural number. 7. (CUB 2) Let p be a prime number and n a natural number. Prove that the product   2  1 2n−1 p i N = 2 ∏ ((p − 1)i)! n pi p i=1; 2∤i is a natural number that is not divisible by p. 8. (CUB 3) (SL74-9). 9. (CZS 1) Solve the following system of linear equations with unknown x1 , . . . , xn (n ≥ 2) and parameters c1 , . . . , cn : 2x1 −x2 −x1 +2x2 −x3 −x2 +2x3 −x4 ... ... ... −xn−2 +2xn−1 −xn−1 = c1 ; = c2 ; = c3 ; ... −xn = cn−1 ; +2xn = cn . 3.16 IMO 1974 91 10. (CZS 2) A regular octagon P is given whose incircle k has diameter 1. About k is circumscribed a regular 16-gon, which is also inscribed in P, cutting from P eight isosceles triangles. To the octagon P, three of these triangles are added so that exactly two of them are adjacent and no two of them are opposite to each other. Every 11-gon so obtained is said to be P′ . Prove the following statement: Given a finite set M of points lying in P such that every two points of this set have a distance not exceeding 1, one of the 11-gons P′ contains all of M. 11. (CZS 3) Given a line p and a triangle △ in the plane, construct an equilateral triangle one of whose vertices lies on the line p, while the other two halve the perimeter of △. 12. (CZS 4) A circle K with radius r, a point D on K, and a convex angle with vertex S and rays a and b are given in the plane. Construct a parallelogram ABCD such that A and B lie on a and b respectively, SA + SB = r, and C lies on K. 13. (FIN 1) Prove that 2147 − 1 is divisible by 343. 14. (FIN 2) Let n and k be natural numbers and a1 , a2 , . . . , an positive real numbers satisfying a1 + a2 + · · · + an = 1. Prove that −k −k k+1 a−k . 1 + a2 + · · · + a n ≥ n 15. (FIN 3) (SL74-10). 16. (UNK 1) A pack of 2n cards contains n different pairs of cards. Each pair consists of two identical cards, either of which is called the twin of the other. A game is played between two players A and B. A third person called the dealer shuffles the pack and deals the cards one by one face upward onto the table. One of the players, called the receiver, takes the card dealt, provided he does not have already its twin. If he does already have the twin, his opponent takes the dealt card and becomes the receiver. A is initially the receiver and takes the first card dealt. The player who first obtains a complete set of n different cards wins the game. What fraction of all possible arrangements of the pack lead to A winning? Prove the correctness of your answer. 17. (UNK 2) Show that there exists a set S of 15 distinct circles on the surface of a sphere, all having the same radius and such that 5 touch exactly 5 others, 5 touch exactly 4 others, and 5 touch exactly 3 others. 18. (UNK 3) (SL74-5). 19. (UNK 4) (Alternative to UNK 2) Prove that there exists, for n ≥ 4, a set S of 3n equal circles in space that can be partitioned into three subsets s5 , s4 , and s3 , each containing n circles, such that each circle in sr touches exactly r circles in S. 20. (NLD 1) For which natural numbers n do there exist n natural numbers ai (1 ≤ i ≤ n) such that ∑ni=1 a−2 i = 1? 92 3 Problems 21. (NLD 2) Let M be a √ nonempty subset of Z+ such that for every element x in M, the numbers 4x and [ x] also belong to M. Prove that M = Z+ . 22. (NLD 3) (SL74-8). 23. (POL 1) (SL74-2). 24. (POL 2) (SL74-7). 25. (POL 3) Let f : R → R be of the form f (x) = x + ε sin x, where 0 < |ε | ≤ 1. Define for any x ∈ R, xn = f ◦ · · · ◦ f (x). | {z } n times Show that for every x ∈ R there exists an integer k such that limn→∞ xn = kπ . 26. (POL 4) Let g(k) be the number of partitions of a k-element set M, i.e., the number of families {A1 , A2 , . . . , As } of nonempty subsets of M such that Ai ∩ S A j = 0/ for i 6= j and ni=1 Ai = M. Prove that nn ≤ g(2n) ≤ (2n)2n for every n. 27. (ROU 1) Let C1 and C2 be circles in the same plane, P1 and P2 arbitrary points on C1 and C2 respectively, and Q the midpoint of segment P1 P2 . Find the locus of points Q as P1 and P2 go through all possible positions. Alternative version. Let C1 ,C2 ,C3 be three circles in the same plane. Find the locus of the centroid of triangle P1 P2 P3 as P1 , P2 , and P3 go through all possible positions on C1 , C2 , and C3 respectively. 28. (ROU 2) Let M be a finite set and P = {M1 , M2 , . . . , Mk } a partition of M (i.e., Sk / Mi ∩ M j = 0/ for all i, j ∈ {1, 2, . . . , k}, i 6= j). We define i=1 Mi = M, Mi 6= 0, the following elementary operation on P: Choose i, j ∈ {1, 2, . . . , k}, such that i 6= j and Mi has a elements and M j has b elements such that a ≥ b. Then take b elements from Mi and place them into M j , i.e., M j becomes the union of itself unifies and a b-element subset of Mi , while the same subset is subtracted from Mi (if a = b, Mi is thus removed from the partition). Let a finite set M be given. Prove that the property “for every partition P of M there exists a sequence P = P1 , P2 , . . . , Pr such that Pi+1 is obtained from Pi by an elementary operation and Pr = {M}” is equivalent to “the number of elements of M is a power of 2.” 29. (ROU 3) Let A, B,C, D be points in space. If for every point M on the segment AB the sum area(AMC)+area(CMD)+area(DMB) is constant show that the points A, B,C, D lie in the same plane. 30. (ROU 4) (SL74-6). 3.16 IMO 1974 93 31. (ROU 5) Let yα = ∑ni=1 xαi , where α 6= 0, y > 0, xi > 0 are real numbers, and let λ 6= α be a real number. Prove that yλ > ∑ni=1 xλi if α (λ − α ) > 0, and yλ < ∑ni=1 xλi if α (λ − α ) < 0. 32. (SWE 1) Let a1 , a2 , . . . , an be n real numbers such that 0 < a ≤ ak ≤ b for k = 1, 2, . . . , n. If 1 m1 = (a1 + a2 + · · · + an ) n prove that m2 ≤ ity. (a+b)2 2 4ab m1 1 m2 = (a21 + a22 + · · · + a2n ), n and and find a necessary and sufficient condition for equal- 33. (SWE 2) Let a be a real number such that 0 < a < 1, and let n be a positive integer. Define the sequence a0 , a1 , a2 , . . . , an recursively by a0 = a; 1 ak+1 = ak + a2k n for k = 0, 1, . . . , n − 1. Prove that there exists a real number A, depending on a but independent of n, such that 0 < n(A − an) < A3 . 34. (SWE 3) (SL74-3). 35. (SWE 4) If p and q are distinct prime numbers, then there are integers x0 and y0 such that 1 = px0 + qy0. Determine the maximum value of b − a, where a and b are positive integers with the following property: If a ≤ t ≤ b, and t is an integer, then there are integers x and y with 0 ≤ x ≤ q − 1 and 0 ≤ y ≤ p − 1 such that t = px + qy. 36. (SWE 5) Consider infinite diagrams .. . .. . .. . D = n20 n21 n22 . . . n10 n11 n12 . . . n00 n01 n02 . . . where all but a finite number of the integers ni j , i = 0, 1, 2, . . ., j = 0, 1, 2, . . ., are equal to 0. Three elements of a diagram are called adjacent if there are integers i and j with i ≥ 0 and j ≥ 0 such that the three elements are (i) ni j , ni, j+1 , ni, j+2 , or (ii) ni j , ni+1, j , ni+2, j , or (iii) ni+2, j , ni+1, j+1 , ni, j+2 . An elementary operation on a diagram is an operation by which three adjacent elements ni j are changed into n′i j in such a way that |ni j − n′i j | = 1. Two diagrams are called equivalent if one of them can be changed into the other by a finite sequence of elementary operations. How many inequivalent diagrams exist? 94 3 Problems 37. (USA 1) Let a, b, and c denote the three sides of a billiard table in the shape of an equilateral triangle. A ball is placed at the midpoint of side a and then propelled toward side b with direction defined by the angle θ . For what values of θ will the ball strike the sides b, c, a in that order?  n! 38. (USA 2) Consider the binomial coefficients nk = k!(n−k)! (k = 1, 2, . . . , n −  n  n n 1). Determine all positive integers n for which 1 , 2 , . . . , n−1 are all even numbers. 39. (USA 3) Let n be a positive integer, n ≥ 2, and consider the polynomial equation xn − xn−2 − x + 2 = 0. For each n, determine all complex numbers x that satisfy the equation and have modulus |x| = 1. 40. (USA 4) (SL74-1). 41. (USA 5) Through the circumcenter O of an arbitrary acute-angled triangle, chords A1 A2 , B1 B2 , C1C2 are drawn parallel to the sides BC,CA, AB of the triangle respectively. If R is the radius of the circumcircle, prove that A1 O · OA2 + B1 O · OB2 +C1 O · OC2 = R2 . 42. (USS 1) (SL74-12). 43. (USS 2) An (n2 + n + 1) × (n2 + n + 1) matrix of zeros and ones is given. If no four ones are vertices of a rectangle, prove that the number of ones does not exceed (n + 1)(n2 + n + 1). 44. (USS 3) We are given n mass points of equal mass in space. We define a sequence of points O1 , O2 , O3 , . . . as follows: O1 is an arbitrary point (within the unit distance of at least one of the n points); O2 is the center of gravity of all the n given points that are inside the unit sphere centered at O1 ; O3 is the center of gravity of all of the n given points that are inside the unit sphere centered at O2 ; etc. Prove that starting from some m, all points Om , Om+1 , Om+2 , . . . coincide. 45. (USS 4) (SL74-4). 46. (USS 5) Outside an arbitrary triangle ABC, triangles ADB and BCE are constructed such that ∠ADB = ∠BEC = 90◦ and ∠DAB = ∠EBC = 30◦ . On the segment AC the point F with AF = 3FC is chosen. Prove that ∠DFE = 90◦ and ∠FDE = 30◦ . 47. (VNM 1) Given two points A, B outside of a given plane P, find the positions of points M in the plane P for which the ratio MA MB takes a minimum or maximum. 48. (VNM 2) Let a be a number different from zero. For all integers n define Sn = an + a−n. Prove that if for some integer k both Sk and Sk+1 are integers, then for each integer n the number Sn is an integer. 3.16 IMO 1974 95 49. (VNM 3) Determine an equation of third degree with integral coefficients havπ π ing roots sin 14 , sin 514π , and sin −3 14 . 50. (YUG 1) Let m and n be natural numbers with m > n. Prove that 2(m − n)2(m2 − n2 + 1) ≥ 2m2 − 2mn + 1. 51. (YUG 2) There are n points on a flat piece of paper, any two of them at a distance of at least 2 from each other. An inattentive pupil spills ink on a part of the paper such that the total area of the damaged part equals 3/2. Prove that there exist two vectors of equal length less than 1 and with their sum having a given direction, such that after a translation by either of these two vectors no points of the given set remain in the damaged area. 52. (YUG 3) A fox stands in the center of the field which has the form of an equilateral triangle, and a rabbit stands at one of its vertices. The fox can move through the whole field, while the rabbit can move only along the border of the field. The maximal speeds of the fox and rabbit are equal to u and v, respectively. Prove that: (a) If 2u > v, the fox can catch the rabbit, no matter how the rabbit moves. (b) If 2u ≤ v, the rabbit can always run away from the fox. 3.16.3 Shortlisted Problems 1. I 1 (USA 4)IMO1 Alice, Betty, and Carol took the same series of examinations. There was one grade of A, one grade of B, and one grade of C for each examination, where A, B,C are different positive integers. The final test scores were Alice 20 Betty 10 Carol 9 If Betty placed first in the arithmetic examination, who placed second in the spelling examination? 2. I 2 (POL 1) Prove that the squares with sides 1/1, 1/2, 1/3, . . . may be put into the square with side 3/2 in such a way that no two of them have any interior point in common. 3. I 3 (SWE 3)IMO6 Let P(x) be a polynomial with integer coefficients. If n(P) is the number of (distinct) integers k such that P2 (k) = 1, prove that n(P) − deg(P) ≤ 2, where deg(P) denotes the degree of the polynomial P. 4. I 4 (USS 4) The sum of the squares of five real numbers a1 , a2 , a3 , a4 , a5 equals 1. Prove that the least of the numbers (ai − a j )2 , where i, j = 1, 2, 3, 4, 5 and i 6= j, does not exceed 1/10. 96 3 Problems 5. I 5 (UNK 3) Let Ar , Br ,Cr be points on the circumference of a given circle S. From the triangle Ar BrCr , called △r , the triangle △r+1 is obtained by constructing the points Ar+1 , Br+1 ,Cr+1 on S such that Ar+1 Ar is parallel to BrCr , Br+1 Br is parallel to Cr Ar , and Cr+1Cr is parallel to Ar Br . Each angle of △1 is an integer number of degrees and those integers are not multiples of 45. Prove that at least two of the triangles △1 , △2 , . . . , △15 are congruent. 6. I 6 (ROU 4)IMO3 Does there exist a natural number n for which the number  n  2n + 1 3k 2 ∑ k=0 2k + 1 is divisible by 5? 7. II 1 (POL 2) Let ai , bi be coprime positive integers for i = 1, 2, . . . , k, and m the least common multiple of b1 , . . . , bk . Prove that the greatest common divisor of a1 bm1 , . . . , ak bm equals the greatest common divisor of a1 , . . . , ak . k 8. II 2 (NLD 3)IMO5 If a, b, c, d are arbitrary positive real numbers, find all possible values of a b c d S= + + + . a+b+d a+b+c b+c+d a+c+d 9. II 3 (CUB √ 3) Let x, y, z be real numbers each of whose absolute value is different from 1/ 3 such that x + y + z = xyz. Prove that 3x − x3 3y − y3 3z − z3 3x − x3 3y − y3 3z − z3 + + = · · . 2 2 2 1 − 3x 1 − 3y 1 − 3z 1 − 3x2 1 − 3y2 1 − 3z2 10. II 4 (FIN 3)IMO2 Let △ABC be a triangle. Prove that there exists a point D on the CD is the geometric mean of AD and BD if and only if √ side AB such that sin A sin B ≤ sin C2 . 11. II 5 (BGR 1)IMO4 Consider a partition of an 8 × 8 chessboard into p rectangles whose interiors are disjoint such that each of them has an equal number of white and black cells. Assume that a1 < a2 < · · · < a p , where ai denotes the number of white cells in the ith rectangle. Find the maximal p for which such a partition is possible and for that p determine all possible corresponding sequences a1 , a2 , . . . , a p . 12. II 6 (USS 1) In a certain language words are formed using an alphabet of three letters. Some words of two or more letters are not allowed, and any two such distinct words are of different lengths. Prove that one can form a word of arbitrary length that does not contain any nonallowed word. 3.17 IMO 1975 97 3.17 The Seventeenth IMO Burgas–Sofia, Bulgaria, 1975 3.17.1 Contest Problems First Day (July 7) 1. Let x1 ≥ x2 ≥ · · · ≥ xn and y1 ≥ y2 ≥ · · · ≥ yn be two n-tuples of numbers. Prove that n n i=1 i=1 ∑ (xi − yi)2 ≤ ∑ (xi − zi)2 is true when z1 , z2 , . . . , zn denote y1 , y2 , . . . , yn taken in another order. 2. Let a1 , a2 , a3 , . . . be any infinite increasing sequence of positive integers. (For every integer i > 0, ai+1 > ai .) Prove that there are infinitely many m for which positive integers x, y, h, k can be found such that 0 < h < k < m and am = xah + yak . 3. On the sides of an arbitrary triangle ABC, triangles BPC, CQA, and ARB are externally erected such that ∡PBC = ∡CAQ = 45◦ , ∡BCP = ∡QCA = 30◦ , ∡ABR = ∡BAR = 15◦ . Prove that ∡QRP = 90◦ and QR = RP. Second Day (July 8) 4. Let A be the sum of the digits of the number 44444444 and B the sum of the digits of the number A. Find the sum of the digits of the number B. 5. Is it possible to plot 1975 points on a circle with radius 1 so that the distance between any two of them is a rational number (distances have to be measured by chords)? 6. The function f (x, y) is a homogeneous polynomial of the nth degree in x and y. If f (1, 0) = 1 and for all a, b, c, f (a + b, c) + f (b + c, a) + f (c + a, b) = 0, prove that f (x, y) = (x − 2y)(x + y)n−1. 3.17.2 Shortlisted Problems 1. (FRA) There are six ports on a lake. Is it possible to organize a series of routes satisfying the following conditions: (i) Every route includes exactly three ports; (ii) No two routes contain the same three ports; 98 3 Problems (iii) The series offers exactly two routes to each tourist who desires to visit two different arbitrary ports? 2. (CZS)IMO1 Let x1 ≥ x2 ≥ · · · ≥ xn and y1 ≥ y2 ≥ · · · ≥ yn be two n-tuples of numbers. Prove that n n i=1 i=1 ∑ (xi − yi)2 ≤ ∑ (xi − zi)2 is true when z1 , z2 , . . . , zn denote y1 , y2 , . . . , yn taken in another order. h 9 i −2/3 3. (USA) Find the integer represented by ∑10 . Here [x] denotes the greatn=1 n √ est integer less than or equal to x (e.g. [ 2] = 1). 4. (SWE) Let a1 , a2 , . . . , an , . . . be a sequence of real numbers such that 0 ≤ an ≤ 1 and an − 2an+1 + an+2 ≥ 0 for n = 1, 2, 3, . . . . Prove that 0 ≤ (n + 1)(an − an+1) ≤ 2 for n = 1, 2, 3, . . . . 5. (SWE) Let M be the set of all positive integers that do not contain the digit 9 (base 10). If x1 , . . . , xn are arbitrary but distinct elements in M, prove that n 1 ∑ x j < 80. j=1 6. (USS)IMO4 Let A be the sum of the digits of the number 1616 and B the sum of the digits of the number A. Find the sum of the digits of the number B without calculating 1616 . 7. (GDR) Prove that from x + y = 1 (x, y ∈ R) it follows that   n  m  m+ j j n+i i xm+1 ∑ y + yn+1 ∑ x = 1 (m, n = 0, 1, 2, . . . ). j i j=0 i=0 8. (NLD)IMO3 On the sides of an arbitrary triangle ABC, triangles BPC, CQA, and ARB are externally erected such that ∡PBC = ∡CAQ = 45◦ , ∡BCP = ∡QCA = 30◦ , ∡ABR = ∡BAR = 15◦ . Prove that ∡QRP = 90◦ and QR = RP. 9. (NLD) Let f (x) be a continuous function defined on the closed interval 0 ≤ x ≤ 1. Let G( f ) denote the graph of f (x): G( f ) = {(x, y) ∈ R2 | 0 ≤ x ≤ 1, y = f (x)}. Let Ga ( f ) denote the graph of the translated function f (x − a) (translated over a distance a), defined by Ga ( f ) = {(x, y) ∈ R2 | a ≤ x ≤ a + 1, y = f (x − a)}. Is it possible to find for every a, 0 < a < 1, a continuous function f (x), defined on 0 ≤ x ≤ 1, such that f (0) = f (1) = 0 and G( f ) and Ga ( f ) are disjoint point sets? 3.17 IMO 1975 99 10. (UNK)IMO6 The function f (x, y) is a homogeneous polynomial of the nth degree in x and y. If f (1, 0) = 1 and for all a, b, c, f (a + b, c) + f (b + c, a) + f (c + a, b) = 0, prove that f (x, y) = (x − 2y)(x + y)n−1. 11. (UNK)IMO2 Let a1 , a2 , a3 , . . . be any infinite increasing sequence of positive integers. (For every integer i > 0, ai+1 > ai .) Prove that there are infinitely many m for which positive integers x, y, h, k can be found such that 0 < h < k < m and am = xah + yak . 12. (HEL) Consider on the first quadrant of the trigonometric circle the arcs AM1 = x1 , AM2 = x2 , AM3 = x3 , . . . , AMν = xν , such that x1 < x2 < x3 < · · · < xν . Prove that ν −1 ν −1 π ν −1 ∑ sin 2xi − ∑ sin(xi − xi+1) < 2 + ∑ sin(xi + xi+1). i=0 i=0 i=0 13. (ROU) Let A0 , A1 , . . . , An be points in a plane such that 1 (i) A0 A1 ≤ 12 A1 A2 ≤ · · · ≤ 2n−1 An−1 An and (ii) 0 < ∡A0 A1 A2 < ∡A1 A2 A3 < · · · < ∡An−2 An−1 An < 180◦, where all these angles have the same orientation. Prove that the segments Ak Ak+1 , Am Am+1 do not intersect for each k and m such that 0 ≤ k ≤ m − 2 < n − 2. 14. (YUG) Let x0 = 5 and xn+1 = xn + x1n (n = 0, 1, 2, . . .). Prove that 45 < x1000 < 45.1. 15. (USS)IMO5 Is it possible to plot 1975 points on a circle with radius 1 so that the distance between any two of them is a rational number (distances have to be measured by chords)? 100 3 Problems 3.18 The Eighteenth IMO Vienna–Lienz, Austria, 1976 3.18.1 Contest Problems First Day (July 12) 1. In a convex quadrangle with area 32 cm2 , the sum of the lengths of two nonadjacent edges and of the length of one diagonal is equal to 16 cm. What is the length of the other diagonal? 2. Let P1 (x) = x2 − 2, Pj (x) = P1 (Pj−1 (x)), j = 2, 3, . . . . Show that for arbitrary n, the roots of the equation Pn (x) = x are real and different from one another. 3. A rectangular box can be filled completely with unit cubes. If one places the maximal number of cubes with volume 2 in the box such that their edges are parallel to the edges of the box, one can fill exactly 40% of the box. Determine all possible (interior) sizes of the box. Second Day (July 13) 4. Find the largest number obtainable as the product of positive integers whose sum is 1976. 5. Let a set of p equations be given, a11 x1 + · · · + a1q xq = 0, a21 x1 + · · · + a2q xq = 0, .. . a p1x1 + · · · + a pqxq = 0, with coefficients ai j satisfying ai j = −1, 0, or +1 for all i = 1, . . . , p and j = 1, . . . , q. Prove that if q = 2p, there exists a solution x1 , . . . , xq of this system such that all x j ( j = 1, . . . , q) are integers satisfying |x j | ≤ q and x j 6= 0 for at least one value of j. 6. For all positive integral n, un+1 = un (u2n−1 − 2) − u1, u0 = 2, and u1 = 2 12 . Prove that 3 log2 [un ] = 2n − (−1)n , where [x] is the integral part of x. 3.18.2 Longlisted Problems 1. (BGR 1) (SL76-1). 2. (BGR 2) Let P be a set of n points and S a set of l segments. It is known that: (i) No four points of P are coplanar. (ii) Any segment from S has its endpoints at P. 3.18 IMO 1976 101 (iii) There is a point, say g, in P that is the endpoint of a maximal number of segments from S and that is not a vertex of a tetrahedron having all its edges in S. 2 Prove that l ≤ n3 . 3. (BGR 3) (SL76-2). 4. (BGR 4) Find all pairs of natural numbers (m, n) for which 2m · 3n + 1 is the square of some integer. 5. (BGR 5) Let ABCDS be a pyramid with four faces and with ABCD as a base, and let a plane α through the vertex A meet its edges SB and SD at points M and N, respectively. Prove that if the intersection of the plane α with the pyramid ABCDS is a parallelogram, then SM · SN > BM · DN. 6. (CZS 1) For each point X of a given polytope, denote by f (X ) the sum of the distances of the point X from all the planes of the faces of the polytope. Prove that if f attains its maximum at an interior point of the polytope, then f is constant. 7. (CZS 2) Let P be a fixed point and T a given triangle that contains the point P. Translate the triangle T by a given vector v and denote by T ′ this new triangle. Let r, R, respectively, be the radii of the smallest disks centered at P that contain the triangles T , T ′ , respectively. Prove that r + |v| ≤ 3R and find an example to show that equality can occur. 8. (CZS 3) (SL76-3). 9. (CZS 4) Find all (real) solutions of the system 3x1 − x2 − x3 − x5 = 0, −x1 + 3x2 − x4 − x6 = 0, −x1 + 3x3 − x4 − x7 = 0, −x2 − x3 + 3x4 − x8 = 0, −x1 + 3x5 − x6 − x7 = 0, −x2 − x5 + 3x6 − x8 = 0, −x3 − x5 + 3x7 − x8 = 0, −x4 − x6 − x7 + 3x8 = 0. 10. (FIN 1) Show that the reciprocal of any number of the form 2(m2 + m + 1), where m is a positive integer, can be represented as a sum of consecutive terms in the sequence (a j )∞j=1 , aj = 1 . j( j + 1)( j + 2) 102 3 Problems 11. (FIN 2) (SL76-9). 12. (FIN 3) Five points lie on the surface of a ball of unit radius. Find the maximum of the smallest distance between any two of them. 13. (UNK 1a) (SL76-4). 14. (UNK 1b) A sequence {un} of integers is defined by u1 = 2, u2 = u3 = 7, un+1 = un un−1 − un−2, for n ≥ 3. Prove that for each n ≥ 1, un differs by 2 from an integral square. 15. (UNK 2) Let ABC and A′ B′C′ be any two coplanar triangles. Let L be a point such that ALkBC, A′ LkB′C′ , and M, N similarly defined. The line BC meets B′C′ at P, and similarly defined are Q and R. Prove that PL, QM, RN are concurrent. 16. (UNK 3) Prove that there is a positive integer n such that the decimal representation of 7n contains a block of at least m consecutive zeros, where m is any given positive integer. 17. (UNK 4) Show that there exists a convex polyhedron with all its vertices on the surface of a sphere and with all its faces congruent isosceles triangles whose √ √ ratio of sides are 3 : 3 : 2. 18. (GDR 1) Prove that the number 191976 + 761976: 4 (a) is divisible by the (Fermat) prime number F4 = 22 + 1; (b) is divisible by at least four distinct primes other than F4 . 19. (GDR 2) For a positive integer n, let 6(n) be the natural number whose decimal representation consists of n digits 6. Let us define, for all natural numbers m, k with 1 ≤ k ≤ m,   6(m) · 6(m−1) · · · 6(m−k+1) m = . k 6(1) · 6(2) · · · 6(k)   m Prove that for all m, k, is a natural number whose decimal representation k consists of exactly k(m + k − 1) − 1 digits. 20. (GDR 3) Let (an ), n = 0, 1, . . ., be a sequence of real numbers such that a0 = 0 and 1 a3n+1 = a2n − 1, n = 0, 1, . . . . 2 Prove that there exists a positive number q, q < 1, such that for all n = 1, 2, . . . , |an+1 − an| ≤ q|an − an−1 |, and give one such q explicitly. 21. (GDR 4) Find the largest positive real number p (if it exists) such that the inequality 3.18 IMO 1976 x21 + x22 + · · · + x2n ≥ p(x1 x2 + x2 x3 + · · · + xn−1 xn ) 103 (1) is satisfied for all real numbers xi , and (a) n = 2; (b) n = 5. Find the largest positive real number p (if it exists) such that the inequality (1) holds for all real numbers xi and all natural numbers n, n ≥ 2. 22. (GDR 5) A regular pentagon A1 A2 A3 A4 A5 with side length s is given. At each point Ai a sphere Ki of radius s/2 is constructed. There are two spheres K1 ′ and K2 ′ eah of radius s/2 touching all the five spheres Ki . Decide whether K1 ′ and K2 ′ intersect each other, touch each other, or have no common points. 23. (NLD 1) Prove that in a Euclidean plane there are infinitely many concentric circles C such that all triangles inscribed in C have at least one irrational side. 24. (NLD 2) Let 0 ≤ x1 √ ≤ x2 ≤ · · · ≤ xn ≤ 1. Prove that for all A ≥ 1 there exists an interval I of length 2 n A such that for all x ∈ I, |(x − x1 )(x − x2 ) · · · (x − xn )| ≤ A. 25. (NLD 3) (SL76-5). 26. (NLD 4) (SL76-6). 27. (NLD 5) In a plane three points P, Q, R, not on a line, are given. Let k, l, m be positive numbers. Construct a triangle ABC whose sides pass through P, Q, and R such that P divides the segment AB in the ratio 1 : k, Q divides the segment BC in the ratio 1 : l, and R divides the segment CA in the ratio 1 : m. 28. (POL 1a) Let Q be a unit square in the plane: Q = [0, 1] × [0, 1]. Let T : Q → Q be defined as follows:  (2x, y/2) if 0 ≤ x ≤ 1/2; T (x, y) = (2x − 1, y/2 + 1/2) if 1/2 < x ≤ 1. Show that for every disk D ⊂ Q there exists an integer n > 0 such that T n (D) ∩ D 6= 0. / 29. (POL 1b) (SL76-7). 30. (POL 2) Prove that if P(x) = (x − a)k Q(x), where k is a positive integer, a is a nonzero real number, Q(x) is a nonzero polynomial, then P(x) has at least k + 1 nonzero coefficients. 31. (POL 3) Into every lateral face of a quadrangular pyramid a circle is inscribed. The circles inscribed into adjacent faces are tangent (have one point in common). Prove that the points of contact of the circles with the base of the pyramid lie on a circle. 32. (POL 4) We consider the infinite chessboard covering the whole plane. In every field of the chessboard there is a nonnegative real number. Every number is the 104 3 Problems arithmetic mean of the numbers in the four adjacent fields of the chessboard. Prove that the numbers occurring in the fields of the chessboard are all equal. 33. (SWE 1) A finite set of points P in the plane has the following property: Every line through two points in P contains at least one more point belonging to P. Prove that all points in P lie on a straight line. ∞ 34. (SWE 2) Let {an}∞ 0 and {bn }0 be two sequences determined by the recursion formulas an+1 = an + bn, bn+1 = 3an + bn , n = 0, 1, 2, . . . , and the initial values a0 = b0 = 1. Prove that there exists a uniquely determined constant c such that n|can − bn | < 2 for all nonnegative integers n. 35. (SWE 3) (SL76-8). 36. (USA 1) Three concentric circles with common center O are cut by a common chord in successive points A, B,C. Tangents drawn to the circles at the points A, B,C enclose a triangular region. If the distance from point O to the common chord is equal to p, prove that the area of the region enclosed by the tangents is equal to AB · BC ·CA . 2p 37. (USA 2) From a square board 11 squares long and 11 squares wide, the central square is removed. Prove that the remaining 120 squares cannot be covered by 15 strips each 8 units long and one unit wide. √ √ 38. (USA 3) Let x = a + b, where a and b are natural numbers, x is not an integer, and x < 1976. Prove that the fractional part of x exceeds 10−19.76. 39. (USA 4) In △ABC, the inscribed circle is tangent to side BC at X. Segment AX is drawn. Prove that the line joining the midpoint of segment AX to the midpoint of side BC passes through the center I of the inscribed circle. 40. (USA 5) Let g(x) be a fixed polynomial and define f (x) by f (x) = x2 + xg(x3 ). Show that f (x) is not divisible by x2 − x + 1. 41. (USA 6) (SL76-10). 42. (USS 1) For a point O inside a triangle ABC, denote by A1 , B1 ,C1 the respective intersection points of AO, BO,CO with the corresponding sides. Let n1 = AAO , 1O BO CO n2 = B O , n3 = C O . What possible values of n1 , n2 , n3 can all be positive inte1 1 gers? 43. (USS 2) Prove that if for a polynomial P(x, y) we have P(x − 1, y − 2x + 1) = P(x, y), then there exists a polynomial Φ (x) with P(x, y) = Φ (y − x2 ). √ 44. (USS 3) A circle of radius 1 rolls around a circle of radius 2. Initially, the tangent point is colored red. Afterwards, the red points map from one circle to another by contact. How many red points will be on the bigger circle when the center of the smaller one has made n circuits around the bigger one? 3.18 IMO 1976 105 45. (USS 4) We are given n (n ≥ 5) circles in a plane. Suppose that every three of them have a common point. Prove that all n circles have a common point. 46. (USS 5) For a ≥ 0, b ≥ 0, c ≥ 0, d ≥ 0, prove the inequality a4 + b4 + c4 + d 4 + 2abcd ≥ a2 b2 + a2 c2 + a2 d 2 + b2 c2 + b2 d 2 + c2 d 2 . 47. (VNM 1) (SL76-11). 48. (VNM 2) (SL76-12). 49. (VNM 3) Determine whether there exist 1976 nonsimilar triangles with angles α , β , γ , each of them satisfying the relations sin α + sin β + sin γ 12 = cos α + cos β + cos γ 7 sin α sin β sin γ = and 12 . 25 50. (VNM 4) Find a function f (x) defined for all real values of x such that for all x, f (x + 2) − f (x) = x2 + 2x + 4, and if x ∈ [0, 2), then f (x) = x2 . 51. (YUG 1) Four swallows are catching a fly. At first, the swallows are at the four vertices of a tetrahedron, and the fly is in its interior. Their maximal speeds are equal. Prove that the swallows can catch the fly. 3.18.3 Shortlisted Problems 1. (BGR 1) Let ABC be a triangle with bisectors AA1 , BB1 ,CC1 (A1 ∈ BC, etc.) and M their common point. Consider the triangles MB1 A, MC1 A, MC1 B, MA1 B, MA1C, MB1C, and their inscribed circles. Prove that if four of these six inscribed circles have equal radii, then AB = BC = CA. 2. (BGR 3) Let a0 , a1 , . . . , an , an+1 be a sequence of real numbers satisfying the following conditions: a0 = an+1 = 0, |ak−1 − 2ak + ak+1 | ≤ 1 Prove that |ak | ≤ k(n+1−k) 2 (k = 1, 2, . . . , n). (k = 0, 1, . . . , n + 1). 3. (CZS 3)IMO1 In a convex quadrangle with area 32 cm2 , the sum of the lengths of two nonadjacent edges and of the length of one diagonal is equal to 16 cm. (a) What is the length of the other diagonal? (b) What are the lengths of the edges of the quadrangle if the perimeter is a minimum? (c) Is it possible to choose the edges in such a way that the perimeter is a maximum? 4. (UNK 1a)IMO6 For all positive integral n, un+1 = un (u2n−1 − 2) − u1 , u0 = 2, and u1 = 5/2. Prove that 3 log2 [un ] = 2n − (−1)n , where [x] is the integral part of x. 106 3 Problems 5. (NLD 3)IMO5 Let a set of p equations be given, a11 x1 + · · · + a1q xq = 0, a21 x1 + · · · + a2q xq = 0, .. . a p1x1 + · · · + a pqxq = 0, with coefficients ai j satisfying ai j = −1, 0, or +1 for all i = 1, . . . , p and j = 1, . . . , q. Prove that if q = 2p, there exists a solution x1 , . . . , xq of this system such that all x j ( j = 1, . . . , q) are integers satisfying |x j | ≤ q and x j 6= 0 for at least one value of j. 6. (NLD 4)IMO3 A rectangular box can be filled completely with unit cubes. If one places the maximal number of cubes with volume 2 in the box such that their edges are parallel to the edges of the box, one can fill exactly 40% of the box. Determine all possible (interior) sizes of the box. 7. (POL 1b) Let I = (0, 1] be the unit interval of the real line. For a given number a ∈ (0, 1) we define a map T : I → I by the formula  x + (1 − a) if 0 < x ≤ a, T (x, y) = x−a if a < x ≤ 1. Show that for every interval J ⊂ I there exists an integer n > 0 such that T n (J) ∩ J 6= 0. / 8. (SWE 3) Let P be a polynomial with real coefficients such that P(x) > 0 if x > 0. Prove that there exist polynomials Q and R with nonnegative coefficients such that P(x) = Q(x) R(x) if x > 0. 9. (FIN 2)IMO2 Let P1 (x) = x2 − 2, Pj (x) = P1 (Pj−1 (x)), j = 2, 3, . . . . Show that for arbitrary n the roots of the equation Pn (x) = x are real and different from one another. 10. (USA 6)IMO4 Find the largest number obtainable as the product of positive integers whose sum is 1976. 11. (VNM 1) Prove that there exist infinitely many positive integers n such that the decimal representation of 5n contains a block of 1976 consecutive zeros. 12. (VNM 2) The polynomial 1976(x + x2 + · · · + xn ) is decomposed into a sum of polynomials of the form a1 x + a2x2 + . . . + an xn , where a1 , a2 , · · · , an are distinct positive integers not greater than n. Find all values of n for which such a decomposition is possible. 3.19 IMO 1977 107 3.19 The Nineteenth IMO Belgrade–Arandjelovac, Yugoslavia, July 1–13, 1977 3.19.1 Contest Problems First Day (July 6) 1. Equilateral triangles ABK, BCL, CDM, DAN are constructed inside the square ABCD. Prove that the midpoints of the four segments KL, LM, MN, NK and the midpoints of the eight segments AK, BK, BL, CL, CM, DM, DN, AN are the twelve vertices of a regular dodecagon. 2. In a finite sequence of real numbers the sum of any seven successive terms is negative, and the sum of any eleven successive terms is positive. Determine the maximum number of terms in the sequence. 3. Let n be a given integer greater than 2, and let Vn be the set of integers 1 + kn, where k = 1, 2, . . . . A number m ∈ Vn is called indecomposable in Vn if there do not exist numbers p, q ∈ Vn such that pq = m. Prove that there exists a number r ∈ Vn that can be expressed as the product of elements indecomposable in Vn in more than one way. (Expressions that differ only in order of the elements of Vn will be considered the same.) Second Day (July 7) 4. Let a, b, A, B be given constant real numbers and f (x) = 1 − a cosx − b sinx − A cos2x − B sin2x. Prove that if f (x) ≥ 0 for all real x, then a2 + b2 ≤ 2 and A2 + B2 ≤ 1. 5. Let a and b be natural numbers and let q and r be the quotient and remainder respectively when a2 + b2 is divided by a + b. Determine the numbers a and b if q2 + r = 1977. 6. Let f : N → N be a function that satisfies the inequality f (n + 1) > f ( f (n)) for all n ∈ N. Prove that f (n) = n for all natural numbers n. 3.19.2 Longlisted Problems 1. (BGR 1) A pentagon ABCDE inscribed in a circle for which BC < CD and AB < DE is the base of a pyramid with vertex S. If AS is the longest edge starting from S, prove that BS > CS. 2. (BGR 2) (SL77-1). 108 3 Problems 3. (BGR 3) In a company of n persons, each person has no more than d acquaintances, and in that company there exists a group of k persons, k ≥ d, who are not acquainted with each other. Prove that the number of acquainted pairs is not greater than [n2 /4]. 4. (BGR 4) We are given n points in space. Some pairs of these points are connected by line segments so that the number of segments equals [n2 /4], and a connected triangle exists. Prove that any point from which the maximal number of segments starts is a vertex of a connected triangle. 5. (CZS 1) (SL77-2). 6. (CZS 2) Let x1 , x2 , . . . , xn (n ≥ 1) be real numbers such that 0 ≤ x j ≤ π , j = 1, 2, . . . , n. Prove that if ∑nj=1 (cos x j + 1) is an odd integer, then ∑nj=1 sin x j ≥ 1. 7. (CZS 3) Prove the following assertion: If c1 , c2 , . . . , cn (n ≥ 2) are real numbers such that (n − 1)(c21 + c22 + · · · + c2n ) = (c1 + c2 + · · · + cn )2 , then either all these numbers are nonnegative or all these numbers are nonpositive. 8. (CZS 4) A hexahedron ABCDE is made of two regular congruent tetrahedra ABCD and ABCE. Prove that there exists only one isometry Z that maps points A, B, C, D, E onto B, C, A, E, D, respectively. Find all points X on the surface of hexahedron whose distance from Z (X ) is minimal. 9. (CZS 5) Let ABCD be a regular tetrahedron and Z an isometry mapping A, B, C, D into B, C, D, A, respectively. Find the set M of all points X of the face ABC whose distance from Z (X ) is equal to a given number t. Find necessary and sufficient conditions for the set M to be nonempty. 10. (FRG 1) (SL77-3). 11. (FRG 2) Let n and z be integers greater than 1 and (n, z) = 1. Prove: (a) At least one of the numbers zi = 1 + z + z2 + · · · + zi , i = 0, 1, . . . , n − 1, is divisible by n. (b) If (z − 1, n) = 1, then at least one of the numbers zi , i = 0, 1, . . . , n − 2, is divisible by n. 12. (FRG 3) Let z be an integer > 1 and let M be the set of all numbers of the form zk = 1 + z + · · · + zk , k = 0, 1, . . . . Determine the set T of divisors of at least one of the numbers zk from M. 13. (FRG 4) (SL77-4). 14. (FRG 5) (SL77-5). 15. (GDR 1) Let n be an integer greater than 1. In the Cartesian coordinate system we consider all squares with integer vertices (x, y) such that 1 ≤ x, y ≤ n. Denote by pk (k = 0, 1, 2, . . . ) the number of pairs of points that are vertices of exactly k such squares. Prove that ∑k (k − 1)pk = 0. 3.19 IMO 1977 109 16. (GDR 2) (SL77-6). 17. (GDR 3) A ball K of radius r is touched from the outside by mutually equal balls of radius R. Two of these balls are tangent to each other. Moreover, for two balls K1 and K2 tangent to K and tangent to each other there exist two other balls tangent to K1 , K2 and also to K. How many balls are tangent to K? For a given r determine R. 18. (GDR 4) Given an isosceles triangle ABC with a right angle at C, construct the center M and radius r of a circle cutting on segments AB, BC, CA the segments DE, FG, and HK, respectively, such that ∠DME + ∠FMG + ∠HMK = 180◦ and DE : FG : HK = AB : BC : CA. 19. (UNK 1) Given any integer m > 1 prove that there exist infinitely many positive integers n such that the last m digits of 5n are a sequence am , am−1 , . . . , a1 = 5 (0 ≤ a j < 10) in which each digit except the last is of opposite parity to its successor (i.e., if ai is even, then ai−1 is odd, and if ai is odd, then ai−1 is even). 20. (UNK 2) (SL77-7). 21. (UNK 3) Given that x1 + x2 + x3 = y1 + y2 + y3 = x1 y1 + x2y2 + x3 y3 = 0, prove that x21 y21 2 + = . x21 + x22 + x23 y21 + y22 + y23 3 22. (UNK 4) (SL77-8). 23. (HUN 1) (SL77-9). 24. (HUN 2) Determine all real functions f (x) that are defined and continuous on the interval (−1, 1) and that satisfy the functional equation f (x + y) = f (x) + f (y) 1 − f (x) f (y) (x, y, x + y ∈ (−1, 1)). 25. (HUN 3) Prove the identity   n (z + a) = z + a ∑ (a − kb)k−1 (z + kb)n−k . k=1 k n n n 26. (NLD 1) Let p be a prime number greater than 5. Let V be the collection of all positive integers n that can be written in the form n = kp + 1 or n = kp − 1 (k = 1, 2, . . . ). A number n ∈ V is called indecomposable in V if it is impossible to find k, l ∈ V such that n = kl. Prove that there exists a number N ∈ V that can be factorized into indecomposable factors in V in more than one way. 27. (NLD 2) (SL77-10). 28. (NLD 3) (SL77-11). 29. (NLD 4) (SL77-12). 110 3 Problems 30. (NLD 5) A triangle ABC with ∠A = 30◦ and ∠C = 54◦ is given. On BC a point D is chosen such that ∠CAD = 12◦ . On AB a point E is chosen such that ∠ACE = 6◦ . Let S be the point of intersection of AD and CE. Prove that BS = BC. 31. (POL 1) Let f be a function defined on the set of pairs of nonzero rational numbers whose values are positive real numbers. Suppose that f satisfies the following conditions: (1) f (ab, c) = f (a, c) f (b, c), f (c, ab) = f (c, a) f (c, b); (2) f (a, 1 − a) = 1. Prove that f (a, a) = f (a, −a) = 1, f (a, b) f (b, a) = 1. 32. (POL 2) In a room there are nine men. Among every three of them there are two mutually acquainted. Prove that some four of them are mutually acquainted. 33. (POL 3) A circle K centered at (0, 0) is given. Prove that for every vector (a1 , a2 ) there is a positive integer n such that the circle K translated by the vector n(a1 , a2 ) contains a lattice point (i.e., a point both of whose coordinates are integers). 34. (POL 4) (SL77-13). 35. (ROU 1) Find all numbers N = a1 a2 . . . an for which 9 × a1 a2 . . . an = an . . . a2 a1 such that at most one of the digits a1 , a2 , . . . , an is zero. 36. (ROU 2) Consider a sequence of numbers (a1 , a2 , . . . , a2n ). Define the operation S((a1 , a2 , . . . , a2n )) = (a1 a2 , a2 a3 , . . . , a2n −1 a2n , a2n a1 ). Prove that whatever the sequence (a1 , a2 , . . . , a2n ) is, with ai ∈ {−1, 1} for i = 1, 2, . . . , 2n , after finitely many applications of the operation we get the sequence (1, 1, . . . , 1). 37. (ROU 3) Let A1 , A2 , . . . , An+1 be positive integers such that (Ai , An+1 ) = 1 for every i = 1, 2, . . . , n. Show that the equation A n+1 xA1 1 + xA2 2 + · · · + xAn n = xn+1 has an infinite set of solutions (x1 , x2 , . . . , xn+1 ) in positive integers. 38. (ROU 4) Let m j > 0 for j = 1, 2, . . . , n and a1 ≤ · · · ≤ an < b1 ≤ · · · ≤ bn < c1 ≤ · · · ≤ cn be real numbers. Prove: " #2 !" # n ∑ m j (a j + b j + c j ) j=1 n >3 ∑ mj j=1 n ∑ m j (a j b j + b j c j + c j a j ) . j=1 39. (ROU 5) Consider 37 distinct points in space, all with integer coordinates. Prove that we may find among them three distinct points such that their barycenter has integer coordinates. 40. (SWE 1) The numbers 1, 2, 3, . . . , 64 are placed on a chessboard, one number in each square. Consider all squares on the chessboard of size 2 × 2. Prove that 3.19 IMO 1977 111 there are at least three such squares for which the sum of the 4 numbers contained exceeds 100. 41. (SWE 2) A wheel consists of a fixed circular disk and a mobile circular ring. On the disk the numbers 1, 2, 3, . . ., N are marked, and on the ring N integers a1 , a2 , . . . , aN of sum 1 are marked (see the figure). The ring can be turned · · · a4 · · · 4 a3 into N different positions in which the 3 a2 numbers on the disk and on the ring 2 1 a1 match each other. Multiply every numN aN ber on the ring with the corresponding number on the disk and form the sum of N products. In this way a sum is obtained for every position of the ring. Prove that the N sums are different. 42. (SWE 3) The sequence an,k , k = 1, 2, 3, . . . , 2n , n = 0, 1, 2, . . ., is defined by the following recurrence formula: 1 an,k+2n−1 = a3n−1,k 2 for k = 1, 2, 3, . . ., 2n−1 , n = 0, 1, 2, . . . . a1 = 2, an,k = 2a3n−1,k , Prove that the numbers an,k are all different. 43. (FIN 1) Evaluate n S= ∑ k(k + 1) · · · (k + p), k=1 where n and p are positive integers. 44. (FIN 2) Let E be a finite set of points in space such that E is not contained in a plane and no three points of E are collinear. Show that E contains the vertices of a tetrahedron T = ABCD such that T ∩E = {A, B,C, D} (including interior points of T ) and such that the projection of A onto the plane BCD is inside a triangle that is similar to the triangle BCD and whose sides have midpoints B,C, D. 45. (FIN 2′ ) (SL77-14). 46. (FIN 3) Let f be a strictly increasing function defined on the set of real numbers. For x real and t positive, set g(x,t) = f (x + t) − f (x) . f (x) − f (x − t) Assume that the inequalities 2−1 < g(x,t) < 2 hold for all positive t if x = 0, and for all t ≤ |x| otherwise. 112 3 Problems Show that 14−1 < g(x,t) < 14 for all real x and positive t. 47. (USS 1) A square ABCD is given. A line passing through A intersects CD at Q. Draw a line parallel to AQ that intersects the boundary of the square at points M and N such that the area of the quadrilateral AMNQ is maximal. 48. (USS 2) The intersection of a plane with a regular tetrahedron with edge a is a quadrilateral with perimeter P. Prove that 2a ≤ P ≤ 3a. 49. (USS 3) Find all pairs of integers (p, q) for which all roots of the trinomials x2 + px + q and x2 + qx + p are integers. 50. (USS 4) Determine all positive integers n for which there exists a polynomial Pn (x) of degree n with integer coefficients that is equal to n at n different integer points and that equals zero at zero. 51. (USS 5) Several segments, which we shall call white, are given, and the sum of their lengths is 1. Several other segments, which we shall call black, are given, and the sum of their lengths is 1. Prove that every such system of segments can be distributed on the segment that is 1.51 long in the following way: Segments of the same color are disjoint, and segments of different colors are either disjoint or one is inside the other. Prove that there exists a system that cannot be distributed in that way on the segment that is 1.49 long. 52. (USA 1) Two perpendicular chords are drawn through a given interior point P of a circle with radius R. Determine, with proof, the maximum and the minimum of the sum of the lengths of these two chords if the distance from P to the center of the circle is kR. 53. (USA 2) Find all pairs of integers a and b for which 7a + 14b = 5a2 + 5ab + 5b2. 54. (USA 3) If 0 ≤ a ≤ b ≤ c ≤ d, prove that ab bc cd d a ≥ ba cb d c ad . 55. (USA 4) Through a point O on the diagonal BD of a parallelogram ABCD, segments MN parallel to AB, and PQ parallel to AD, are drawn, with M on AD, and Q on AB. Prove that diagonals AO, BP, DN (extended if necessary) will be concurrent. 56. (USA 5) The four circumcircles of the four faces of a tetrahedron have equal radii. Prove that the four faces of the tetrahedron are congruent triangles. 57. (VNM 1) (SL77-15). 58. (VNM 2) Prove that for every triangle the following inequality holds: ab + bc + ca π ≥ cot , 4S 6 3.19 IMO 1977 113 where a, b, c are lengths of the sides and S is the area of the triangle. 59. (VNM 3) (SL77-16). 60. (VNM 4) Suppose x0 , x1 , . . . , xn are integers and x0 > x1 > · · · > xn . Prove that at least one of the numbers |F(x0 )|, |F(x1 )|, |F(x2 )|, . . . , |F(xn )|, where F(x) = xn + a1 xn−1 + · · · + an , is greater than ai ∈ R, i = 1, . . . , n, n! 2n . 3.19.3 Shortlisted Problems 1. (BGR 2)IMO6 Let f : N → N be a function that satisfies the inequality f (n + 1) > f ( f (n)) for all n ∈ N. Prove that f (n) = n for all natural numbers n. 2. (CZS 1) A lattice point in the plane is a point both of whose coordinates are integers. Each lattice point has four neighboring points: upper, lower, left, and right. Let k be a circle with radius r ≥ 2, that does not pass through any lattice point. An interior boundary point is a lattice point lying inside the circle k that has a neighboring point lying outside k. Similarly, an exterior boundary point is a lattice point lying outside the circle k that has a neighboring point lying inside k. Prove that there are four more exterior boundary points than interior boundary points. 3. (FRG 1)IMO5 Let a and b be natural numbers and let q and r be the quotient and remainder respectively when a2 + b2 is divided by a + b. Determine the numbers a and b if q2 + r = 1977. 4. (FRG 4) Describe all closed bounded figures Φ in the plane any two points of which are connectable by a semicircle lying in Φ . 5. (FRG 5) There are 2n words of length n over the alphabet {0, 1}. Prove that the following algorithm generates the sequence w0 , w1 , . . . , w2n −1 of all these words such that any two consecutive words differ in exactly one digit. (1) w0 = 00 . . . 0 (n zeros). (2) Suppose wm−1 = a1 a2 . . . an , ai ∈ {0, 1}. Let e(m) be the exponent of 2 in the representation of n as a product of primes, and let j = 1 +e(m). Replace the digit a j in the word wm−1 by 1 − a j . The obtained word is wm . 6. (GDR 2) Let n be a positive integer. How many integer solutions (i, j, k, l), 1 ≤ i, j, k, l ≤ n, does the following system of inequalities have: 1 ≤ −j+k+l ≤ n 1 ≤ i−k+l ≤ n 1 ≤ i− j+l ≤ n 1 ≤ i+ j−k ≤ n ? 7. (UNK 2)IMO4 Let a, b, A, B be given constant real numbers and 114 3 Problems f (x) = 1 − a cosx − b sinx − A cos2x − B sin2x. Prove that if f (x) ≥ 0 for all real x, then a2 + b2 ≤ 2 and A2 + B2 ≤ 1. 8. (UNK 4) Let S be a convex quadrilateral ABCD and O a point inside it. The feet of the perpendiculars from O to AB, BC, CD, DA are A1 , B1 , C1 , D1 respectively. The feet of the perpendiculars from O to the sides of Si , the quadrilateral Ai BiCi Di , are Ai+1 Bi+1Ci+1 Di+1 , where i = 1, 2, 3. Prove that S4 is similar to S. 9. (HUN 1) For which positive integers n do there exist two polynomials f and g with integer coefficients of n variables x1 , x2 , . . . , xn such that the following equality is satisfied: ! n ∑ xi i=1 f (x1 , x2 , . . . , xn ) = g(x21 , x22 , . . . , x2n )? 10. (NLD 2)IMO3 Let n be an integer greater than 2. Define V = {1 + kn | k = 1, 2, . . . }. A number p ∈ V is called indecomposable in V if it is not possible to find numbers q1 , q2 ∈ V such that q1 q2 = p. Prove that there exists a number N ∈ V that can be factorized into indecomposable factors in V in more than one way. 11. (NLD 3) Let n be an integer greater than 1. Define     xi + yi n x1 = n, y1 = 1, xi+1 = , yi+1 = 2 xi+1 for i = 1, 2, . . . , where [z] denotes the largest integer less than or equal to z. Prove that √ min{x1 , x2 , . . . xn } = [ n]. 12. (NLD 4)IMO1 On the sides of a square ABCD one constructs inwardly equilateral triangles ABK, BCL, CDM, DAN. Prove that the midpoints of the four segments KL, LM, MN, NK, together with the midpoints of the eight segments AK, BK, BL, CL, CM, DM, DN, AN, are the 12 vertices of a regular dodecagon. 13. (POL 4) Let B be a set of k sequences each having n terms equal to 1 or −1. The product of two such sequences (a1 , a2 , . . . , an ) and (b1 , b2 , . . . , bn ) is defined as (a1 b1 , a2 b2 , . . . , an bn ). Prove that there exists a sequence (c1 , c2 , . . . , cn ) such that the intersection of B and the set containing all sequences from B multiplied by (c1 , c2 , . . . , cn ) contains at most k2 /2n sequences. 14. (FIN 2’) Let E be a finite set of points such that E is not contained in a plane and no three points of E are collinear. Show that at least one of the following alternatives holds: (i) E contains five points that are vertices of a convex pyramid having no other points in common with E; 3.19 IMO 1977 115 (ii) some plane contains exactly three points from E. 15. (VNM 1)IMO2 The length of a finite sequence is defined as the number of terms of this sequence. Determine the maximal possible length of a finite sequence that satisfies the following condition: The sum of each seven successive terms is negative, and the sum of each eleven successive terms is positive. 16. (VNM 3) Let E be a set of n points in the plane (n ≥ 3) whose coordinates are integers such that any three points from E are vertices of a nondegenerate triangle whose centroid doesn’t have both coordinates integers. Determine the maximal n. 116 3 Problems 3.20 The Twentieth IMO Bucharest, Romania, 1978 3.20.1 Contest Problems First Day (July 6) 1. Let n > m ≥ 1 be natural numbers such that the groups of the last three digits in the decimal representation of 1978m, 1978n coincide. Find the ordered pair (m, n) of such m, n for which m + n is minimal. 2. Given any point P in the interior of a sphere with radius R, three mutually perpendicular segments PA, PB, PC are drawn terminating on the sphere and having one common vertex in P. Consider the rectangular parallelepiped of which PA, PB, PC are coterminal edges. Find the locus of the point Q that is diagonally opposite P in the parallelepiped when P and the sphere are fixed. 3. Let { f (n)} be a strictly increasing sequence of positive integers: 0 < f (1) < f (2) < f (3) < . . . . Of the positive integers not belonging to the sequence, the nth in order of magnitude is f ( f (n)) + 1. Determine f (240). Second day (July 7) 4. In a triangle ABC we have AB = AC. A circle is tangent internally to the circumcircle of ABC and also to the sides AB, AC, at P, Q respectively. Prove that the midpoint of PQ is the center of the incircle of ABC. 5. Let ϕ : {1, 2, 3, . . .} → {1, 2, 3, . . . } be injective. Prove that for all n, n n ϕ (k) 1 ≥∑ . 2 k k=1 k=1 k ∑ 6. An international society has its members in 6 different countries. The list of members contains 1978 names, numbered 1, 2, . . . , 1978. Prove that there is at least one member whose number is the sum of the numbers of two, not necessarily distinct, of his compatriots. 3.20.2 Longlisted Problems 1. (BGR 1) (SL78-1). 2. (BGR 2) If f (x) = (x + 2x2 + · · · + nxn )2 = a2 x2 + a3 x3 + · · · + a2n x2n , prove that an+1 + an+2 + · · · + a2n =   n + 1 5n2 + 5n + 2 . 2 12 3.20 IMO 1978 117 3. (BGR 3) Find all numbers α for which the equation x2 − 2x[x] + x − α = 0 has two nonnegative roots. ([x] denotes the largest integer less than or equal to x.) 4. (BGR 4) (SL78-2). 5. (CUB 1) Prove that for any triangle ABC there exists a point P in the plane of the triangle and three points A′ , B′ , and C′ on the lines BC, AC, and AB respectively such that AB · PC′ = AC · PB′ = BC · PA′ = 0.3M 2 , where M = max{AB, AC, BC}. 6. (CUB 2) Prove that for all X > 1 there exists a triangle whose sides have lengths P1 (X ) = X 4 + X 3 + 2X 2 + X + 1, P2 (X) = 2X 3 + X 2 + 2X + 1, and P3 (X ) = X 4 − 1. Prove that all these triangles have the same greatest angle and calculate it. 7. (CUB 3) (SL78-3). 8. (CZS 1) For two given triangles A1 A2 A3 and B1 B2 B3 with areas ∆ A and ∆ B , respectively, Ai Ak ≥ Bi Bk , i, k = 1, 2, 3. Prove that ∆ A ≥ ∆B if the triangle A1 A2 A3 is not obtuse-angled. 9. (CZS 2) (SL78-4). 10. (CZS 3) Show that for any natural number n there exist two prime numbers p and q, p 6= q, such that n divides their difference. 11. (CZS 4) Find all natural numbers n < 1978 with the following property: If m is a natural number, 1 < m < n, and (m, n) = 1 (i.e., m and n are relatively prime), then m is a prime number. 12. (FIN 1) The equation x3 + ax2 + bx + c = 0 has three (not necessarily distinct) real roots t, u, v. For which a, b, c do the numbers t 3 , u3 , v3 satisfy the equation x3 + a3 x2 + b3x + c3 = 0? 13. (FIN 2) The satellites A and B circle the Earth in the equatorial plane at altitude h. They are separated by distance 2r, where r is the radius of the Earth. For which h can they be seen in mutually perpendicular directions from some point on the equator? 14. (FIN 3) Let p(x, y) and q(x, y) be polynomials in two variables such that for x ≥ 0, y ≥ 0 the following conditions hold: (i) p(x, y) and q(x, y) are increasing functions of x for every fixed y. (ii) p(x, y) is an increasing and q(x) is a decreasing function of y for every fixed x. (iii) p(x, 0) = q(x, 0) for every x and p(0, 0) = 0. Show that the simultaneous equations p(x, y) = a, q(x, y) = b have a unique solution in the set x ≥ 0, y ≥ 0 for all a, b satisfying 0 ≤ b ≤ a but lack a solution in the same set if a < b. 118 3 Problems 15. (FRA 1) Prove that for every positive integer n coprime to 10 there exists a multiple of n that does not contain the digit 1 in its decimal representation. 16. (FRA 2) (SL78-6). 17. (FRA 3) (SL78-17). 18. (FRA 4) Given a natural number n, prove that √ the number M(n) of points with integer coordinates inside the circle (O(0, 0), n) satisfies √ √ π n − 5 n + 1 < M(n) < π n + 4 n + 1. 19. (FRA 5) (SL78-7). 20. (UNK 1) Let O be the center of a circle. Let OU, OV be perpendicular radii of the circle. The chord PQ passes through the midpoint M of UV . Let W be a point such that PM = PW , where U,V, M,W are collinear. Let R be a point such that PR = MQ, where R lies on the line PW . Prove that MR = UV . Alternative version: A circle S is given with center O and radius r. Let M be a point whose distance from O is √r2 . Let PMQ be a chord of S. The point N is −→ −−→ defined by PN = MQ. Let R be the √ reflection of N by the line through P that is parallel to OM. Prove that MR = 2r. 21. (UNK 2) A circle touches the sides AB, BC,CD, DA of a square at points K, L, M, N respectively, and BU, KV are parallel lines such that U is on DM and V on DN. Prove that UV touches the circle. 22. (UNK 3) Two nonzero integers x, y (not necessarily positive) are such that x + y 2 +y2 is a divisor of x2 + y2 , and the quotient xx+y is a divisor of 1978. Prove that x = y. 23. (UNK 4) (SL78-8). 24. (UNK 5) (SL78-9). 25. (GDR 1) Consider a polynomial P(x) = ax2 + bx+ c with a > 0 that has two real roots x1 , x2 . Prove that the absolute values of both roots are less than or equal to 1 if and only if a + b + c ≥ 0, a − b + c ≥ 0, and a − c ≥ 0. 26. (GDR 2) (SL78-5). 27. (GDR 3) Determine the sixth number after the decimal point in the number √ √ 20 1978 + 1978 . 28. (GDR 4) Let c, s be real functions defined on R \ {0} that are nonconstant on any interval and satisfy   x c = c(x)c(y) − s(x)s(y) for any x 6= 0, y 6= 0. y Prove that: 3.20 IMO 1978 119 (a) c(1/x) = c(x), s(1/x) = −s(x) for any x 6= 0, and also c(1) = 1, s(1) = s(−1) = 0; (b) c and s are either both even or both odd functions (a function f is even if f (x) = f (−x) for all x, and odd if f (x) = − f (−x) for all x). Find functions c, s that also satisfy c(x) + s(x) = xn for all x, where n is a given positive integer. 29. (GDR 5) (Variant of GDR 4) Given a nonconstant function f : R+ → R such that f (xy) = f (x) f (y) for any x, y > 0, find functions c, s : R+ → R that satisfy c(x/y) = c(x)c(y) − s(x)s(y) for all x, y > 0 and c(x) + s(x) = f (x) for all x > 0. 30. (NLD 1) (SL78-10). 31. (NLD 2) Let the polynomials P(x) = xn + an−1xn−1 + · · · + a1 x + a0, Q(x) = xm + bm−1xm−1 + · · · + b1 x + b0, be given satisfying the identity P(x)2 = (x2 − 1)Q(x)2 + 1. Prove the identity P′ (x) = nQ(x). 32. (NLD 3) Let C be the circumcircle of the square with vertices (0, 0), (0, 1978), (1978, 0), (1978, 1978) in the Cartesian plane. Prove that C contains no other point for which both coordinates are integers. 33. (SWE 1) A sequence (an )∞ 0 of real numbers is called convex if 2an ≤ an−1 +an+1 for all positive integers n. Let (bn )∞ 0 be a sequence of positive numbers and assume that the sequence (α n bn )∞ is convex for any choice of α > 0. Prove that 0 the sequence (log bn )∞ is convex. 0 34. (SWE 2) (SL78-11). 35. (SWE 3) A sequence (an )N0 of real numbers is called concave if 2an ≥ an−1 + an+1 for all integers n, 1 ≤ n ≤ N − 1. (a) Prove that there exists a constant C > 0 such that N ∑ an n=0 !2 N ≥ C(N − 1) ∑ a2n (1) n=0 for all concave positive sequences (an )N0 . (b) Prove that (1) holds with C = 3/4 and that this constant is best possible. 36. (TUR 1) The integers 1 through 1000 are located on the circumference of a circle in natural order. Starting with 1, every fifteenth number (i.e., 1, 16, 31, . . .) is marked. The marking is continued until an already marked number is reached. How many of the numbers will be left unmarked? 37. (TUR 2) Simplify 120 3 Problems 1 1 1 + + , loga (abc) logb (abc) logc (abc) where a, b, c are positive real numbers. 38. (TUR 3) Given a circle, construct a chord that is trisected by two given noncollinear radii. 39. (TUR 4) A is a 2m-digit positive integer each of whose digits is 1. B is an mdigit positive integer each of whose digits is 4. Prove that A + B + 1 is a perfect square. p 40. (TUR 5) If Cn = n! p!(n−p)! (p ≥ 1), prove the identity p−1 p−1 p−1 Cnp = Cn−1 + Cn−2 + · · · + Cpp−1 +C p−1 and then evaluate the sum S = 1 · 2 · 3 + 2 · 3 · 4 + · · ·+ 97 · 98 · 99. 41. (USA 1) (SL78-12). 42. (USA 2) A, B,C, D, E are points on a circle O with radius equal to r. Chords AB and DE are parallel to each other and have length equal to x. Diagonals AC, AD, BE,CE are drawn. If segment XY on O meets AC at X and EC at Y , prove that lines BX and DY meet at Z on the circle. 43. (USA 3) If p is a prime greater than 3, show that at least one of the numbers 3 4 , , . . . , p−2 is expressible in the form 1x + 1y , where x and y are positive intep2 p2 p2 gers. 44. (USA 4) In △ABC with ∠C = 60o , prove that ac + bc ≥ 2. 45. (USA 5) If r > s > 0 and a > b > c, prove that ar bs + br cs + cr as ≥ as br + bscr + cs ar . 46. (USA 6) (SL78-13). 47. (VNM 1) Given the expression n  p n i p 1 h Pn (x) = n x + x2 − 1 + x − x2 − 1 , 2 prove: (a) Pn (x) satisfies the identity Pn (x) − xPn−1(x) + 14 Pn−2 (x) ≡ 0. (b) Pn (x) is a polynomial in x of degree n. 48. (VNM 2) (SL78-14). 49. (VNM 3) Let A, B,C, D be four arbitrary distinct points in space. (a) Prove that using the segments AB +CD, AC + BD and AD + BC it is always possible to construct a triangle T that is nondegenerate and has no obtuse angle. 3.20 IMO 1978 121 (b) What should these four points satisfy in order for the triangle T to be rightangled? 50. (VNM 4) A variable tetrahedron ABCD has the following properties: Its edge lengths can change as well as its vertices, but the opposite edges remain equal (BC = DA, CA = DB, AB = DC); and the vertices A, B,C lie respectively on three fixed spheres with the same center P and radii 3, 4, 12. What is the maximal length of PD? 51. (VNM 5) Find the relations among the angles of the triangle ABC whose altitude AH and median AM satisfy ∠BAH = ∠CAM. 52. (YUG 1) (SL78-15). 53. (YUG 2) (SL78-16). 54. (YUG 3) Let p, q and r be three lines in space such that there is no plane that is parallel to all three of them. Prove that there exist three planes α , β , and γ , containing p, q, and r respectively, that are perpendicular to each other (α ⊥ β , β ⊥ γ , γ ⊥ α ). 3.20.3 Shortlisted Problems 1. (BGR 1) The set M = {1, 2, . . . , 2n} is partitioned into k nonintersecting subsets M1 , M2 , . . . , Mk , where n ≥ k3 + k. Prove that there exist even numbers 2 j1 , 2 j2 , . . . , 2 jk+1 in M that are in one and the same subset Mi (1 ≤ i ≤ k) such that the numbers 2 j1 − 1, 2 j2 − 1, . . . , 2 jk+1 − 1 are also in one and the same subset M j (1 ≤ j ≤ k). 2. (BGR 4) Two identically oriented equilateral triangles, ABC with center S and A′ B′C, are given in the plane. We also have A′ 6= S and B′ 6= S. If M is the midpoint of A′ B and N the midpoint of AB′ , prove that the triangles SB′ M and SA′ N are similar. 3. (CUB 3)IMO1 Let n > m ≥ 1 be natural numbers such that the groups of the last three digits in the decimal representation of 1978m, 1978n coincide. Find the ordered pair (m, n) of such m, n for which m + n is minimal. 4. (CZS 2) Let T1 be a triangle having a, b, c as lengths of its sides and let T2 be another triangle having u, v, w as lengths of its sides. If P, Q are the areas of the two triangles, prove that 16PQ ≤ a2 (−u2 + v2 + w2 ) + b2 (u2 − v2 + w2 ) + c2 (u2 + v2 − w2 ). When does equality hold? 5. (GDR 2) For every integer d ≥ 1, let Md be the set of all positive integers that cannot be written as a sum of an arithmetic progression with difference d, having at least two terms and consisting of positive integers. Let A = M1 , B = M2 r {2}, C = M3 . Prove that every c ∈ C may be written in a unique way as c = ab with a ∈ A, b ∈ B. 122 3 Problems 6. (FRA 2)IMO5 Let ϕ : {1, 2, 3, . . .} → {1, 2, 3, . . . } be injective. Prove that for all n, n n ϕ (k) 1 ∑ k2 ≥ ∑ k . k=1 k=1 7. (FRA 5) We consider three distinct half-lines Ox, Oy, Oz in a plane. Prove the existence and uniqueness of three points A ∈ Ox, B ∈ Oy, C ∈ Oz such that the perimeters of the triangles OAB, OBC, OCA are all equal to a given number 2p > 0. 8. (UNK 4) Let S be the set of all the odd positive integers that are not multiples of 5 and that are less than 30m, m being an arbitrary positive integer. What is the smallest integer k such that in any subset of k integers from S there must be two different integers, one of which divides the other? 9. (UNK 5)IMO3 Let { f (n)} be a strictly increasing sequence of positive integers: 0 < f (1) < f (2) < f (3) < · · · . Of the positive integers not belonging to the sequence, the nth in order of magnitude is f ( f (n)) + 1. Determine f (240). 10. (NLD 1)IMO6 An international society has its members in 6 different countries. The list of members contains 1978 names, numbered 1, 2, . . . , 1978. Prove that there is at least one member whose number is the sum of the numbers of two, not necessarily distinct, of his compatriots. 11. (SWE 2) A function f : I → R, defined on an interval I, is called concave if f (θ x + (1 − θ )y) ≥ θ f (x) + (1 − θ ) f (y) for all x, y ∈ I and 0 ≤ θ ≤ 1. Assume that the functions f1 , . . . , fn , having all nonnegative values, are concave. Prove that the function ( f1 f2 . . . fn )1/n is concave. 12. (USA 1)IMO4 In a triangle ABC we have AB = AC. A circle is tangent internally to the circumcircle of ABC and also to the sides AB, AC, at P, Q respectively. Prove that the midpoint of PQ is the center of the incircle of ABC. 13. (USA 6)IMO2 Given any point P in the interior of a sphere with radius R, three mutually perpendicular segments PA, PB, PC are drawn terminating on the sphere and having one common vertex in P. Consider the rectangular parallelepiped of which PA, PB, PC are coterminal edges. Find the locus of the point Q that is diagonally opposite P in the parallelepiped when P and the sphere are fixed. 14. (VNM 2) Prove that it is possible to place 2n(2n + 1) parallelepipedic (rectangular) pieces of soap of dimensions 1 × 2 × (n + 1) in a cubic box with edge 2n + 1 if and only if n is even or n = 1. Remark. It is assumed that the edges of the pieces of soap are parallel to the edges of the box. 15. (YUG 1) Let p be a prime and A = {a1 , . . . , a p−1} an arbitrary subset of the set of natural numbers such that none of its elements is divisible by p. Let us define a mapping f from P(A) (the set of all subsets of A) to the set P = {0, 1, . . . , p − 1} in the following way: 3.20 IMO 1978 123 (i) if B = {ai1 , . . . , aik } ⊂ A and ∑kj=1 ai j ≡ n (mod p), then f (B) = n, (ii) f (0) / = 0, 0/ being the empty set. Prove that for each n ∈ P there exists B ⊂ A such that f (B) = n. 16. (YUG 2) Determine all the triples (a, b, c) of positive real numbers such that the system a p ax + by − cz = 0, p p 1 − x2 + b 1 − y2 − c 1 − z2 = 0, is compatible in the set of real numbers, and then find all its real solutions. 17. (FRA 3) Prove that for any positive integers x, y, z with xy − z2 = 1 one can find nonnegative integers a, b, c, d such that x = a2 + b2 , y = c2 + d 2 , z = ac + bd. Set z = (2q)! to deduce that for any prime number p = 4q + 1, p can be represented as the sum of squares of two integers. 124 3 Problems 3.21 The Twenty-First IMO London, United Kingdom, 1979 3.21.1 Contest Problems First Day (July 2) 1. Let p and q be natural numbers such that 1− 1 1 1 1 1 p + − + ···− + = . 2 3 4 1318 1319 q Prove that p is divisible by 1979. 2. A pentagonal prism A1 A2 . . . A5 B1 B2 . . . B5 is given. The edges, the diagonals of the lateral walls, and the internal diagonals of the prism are each colored either red or green in such a way that no triangle whose vertices are vertices of the prism has its three edges of the same color. Prove that all edges of the bases are of the same color. 3. Two circles in a plane intersect. Let A be one of the points of intersection. Starting simultaneously from A two points move with constant speeds, each point traveling along its own circle in the same sense. The two points return simultaneously after one revolution. Prove that there is a fixed point P in the plane such that, at any time, the distances from P to the moving points are equal. Second Day (July 3) 4. Given a point P in a given plane π and a point Q not in π , determine all points R in π such that QP+PR is a maximum. QR 5. The nonnegative real numbers x1 , x2 , x3 , x4 , x5 , a satisfy the following relations: 5 ∑ ixi = a, i=1 5 ∑ i3 xi = a2, i=1 5 ∑ i5 xi = a3. i=1 What are the possible values of a? 6. Let S and F be opposite vertices of a regular octagon. A frog starts jumping at vertex S. From any vertex of the octagon except F, it may jump to either of the two adjacent vertices. When it reaches vertex F, the frog stops and stays there. Let an be the number of distinct paths of exactly n jumps ending at F. Prove that for n = 1, 2, 3, . . . , a2n−1 = 0, 1 a2n = √ (xn−1 − yn−1 ), 2 √ √ where x = 2 + 2, y = 2 − 2. 3.21 IMO 1979 125 3.21.2 Longlisted Problems 1. (BEL 1) (SL79-1). 2. (BEL 2) For a finite set E of cardinality n ≥ 3, let f (n) denote the maximum number of 3-element subsets of E, any two of them having exactly one common element. Calculate f (n). 3. (BEL 3) Is it possible to partition 3-dimensional Euclidean space into 1979 mutually isometric subsets? 4. (BEL 4) (SL79-2). 5. (BEL 5) Describe which natural numbers do not belong to the set √ E = {[n + n + 1/2] | n ∈ N}. p 6. (BEL 6) Prove that 12 4 sin2 36◦ − 1 = cos 72◦ . 7. (BRA 1) M = (ai, j ), i, j = 1, 2, 3, 4, is a square matrix of order four. Given that: (i) for each i = 1, 2, 3, 4 and for each k = 5, 6, 7, ai,k = ai,k−4 ; Pi = a1,i + a2,i+1 + a3,i+2 + a4,i+3; Si = a4,i + a3,i+1 + a2,i+2 + a1,i+3; Li = ai,1 + ai,2 + ai,3 + ai,4 ; Ci = a1,i + a2,i + a3,i + a4,i , (ii) for each i, j = 1, 2, 3, 4, Pi = Pj , Si = S j , Li = L j , Ci = C j , and (iii) a1,1 = 0, a1,2 = 7, a2,1 = 11, a2,3 = 2, and a3,3 = 15; find the matrix M. 8. (BRA 2) The sequence (an ) of real numbers is defined as follows: a1 = 1, a2 = 2 and an = 3an−1 − an−2, n ≥ 3.   a2n−1 Prove that for n ≥ 3, an = an−2 + 1, where [x] denotes the integer p such that p ≤ x < p + 1. 9. (BRA 3) The real numbers α1 , α2 , α3 , . . . , αn are positive. Let us denote by h = √ n n α α · · · α the geometric mean, n 1 2 1/α +1/α +···+1/αn the harmonic mean, g = 1 2 αn a = α1 +α2 +···+ the arithmetic mean. Prove that h ≤ g ≤ a, and that each of the n equalities implies the other one. 10. (BGR 1) (SL79-3). 11. (BGR 2) Prove that a pyramid A1 A2 . . . A2k+1 S with equal lateral edges and equal space angles between adjacent lateral walls is regular. Variant. Prove that a pyramid A1 . . . A2k+1 S with equal space angles between adjacent lateral walls is regular if there exists a sphere tangent to all its edges. 126 3 Problems 12. (BGR 3) (SL79-4). 13. (BGR 4) The plane is divided into equal squares by parallel lines; i.e., a square net is given. Let M be an arbitrary set of n squares of this net. Prove that it is possible to choose no fewer than n/4 squares of M in such a way that no two of them have a common point. 14. (CZS 1) Let S be a set of n2 + 1 closed intervals (n is a positive integer). Prove that at least one of the following assertions holds: (i) There exists a subset S′ of n + 1 intervals from S such that the intersection of the intervals in S′ is nonempty. (ii) There exists a subset S′′ of n + 1 intervals from S such that any two of the intervals in S′′ are disjoint. 15. (CZS 2) (SL79-5). 16. (CZS 3) Let Q be a square with side length 6. Find the smallest integer n such that in Q there exists a set S of n points with the property that any square with side 1 completely contained in Q contains in its interior at least one point from S. 17. (CZS 4) (SL79-6). 18. (FIN 1) Show that for no integers a ≥ 1, n ≥ 1 is the sum 1+ 1 1 1 + + ···+ 1 + a 1 + 2a 1 + na an integer. 19. (FIN 2) For k = 1, 2, . . . consider the k-tuples (a1 , a2 , . . . , ak ) of positive integers such that a1 + 2a2 + · · · + kak = 1979. Show that there are as many such k-tuples with odd k as there are with even k. 20. (FIN 3) (SL79-10). 21. (FRA 1) Let E be the set of all bijective mappings from R to R satisfying (∀t ∈ R) f (t) + f −1 (t) = 2t, where f −1 is the mapping inverse to f . Find all elements of E that are monotonic mappings. 22. (FRA 2) Consider two quadrilaterals ABCD and A′ B′C′ D′ in an affine Euclidian plane such that AB = A′ B′ , BC = B′C′ , CD = C′ D′ , and DA = D′ A′ . Prove that the following two statements are true: (a) If the diagonals BD and AC are mutually perpendicular, then the diagonals B′ D′ and A′C′ are also mutually perpendicular. (b) If the perpendicular bisector of BD intersects AC at M, and that of B′ D′ MA M ′ A′ ′ ′ intersects A′C′ at M ′ , then MC =M ′C ′ (if MC = 0 then M C = 0). 3.21 IMO 1979 127 23. (FRA 3) Consider the set E consisting of pairs of integers (a, b), with a ≥ 1 and b ≥ 1, that satisfy in the decimal system the following properties: (i) b is written with three digits, as α2 α1 α0 , α2 6= 0; (ii) a is written as β p . . . β1 β0 for some p; (iii) (a + b)2 is written as β p . . . β1 β0 α2 α1 α0 . Find the elements of E. 24. (FRA 4) Let a and b be coprime integers, greater than or equal to 1. Prove that all integers n greater than or equal to (a − 1)(b − 1) can be written in the form: n = ua + vb, with (u, v) ∈ N × N. 25. (FRG 1) (SL79-7). 26. (FRG 2) Let n be a natural number. If 4n + 2n + 1 is a prime, prove that n is a power of three. 27. (FRG 3) (SL79-8). 28. (FRG 4) (SL79-9). 29. (GDR 1) (SL79-11). 30. (GDR 2) Let M be a set of points in a plane with at least two elements. Prove that if M has two axes of symmetry g1 and g2 intersecting at an angle α = qπ , where q is irrational, then M must be infinite. 31. (GDR 3) (SL79-12). 32. (GDR 4) Let n, k ≥ 1 be natural numbers. Find the number A(n, k) of solutions in integers of the equation |x1 | + |x2 | + · · · + |xk | = n. 33. (HEL 1) (SL79-13). 166 1 34. (HEL 2) Notice that in the fraction 16 64 we can perform a simplification as 664 = 4 obtaining a correct equality. Find all fractions whose numerators and denominators are two-digit positive integers for which such a simplification is correct. 35. (HEL 3) Given a sequence (an ), with a1 = 4 and an+1 = a2n − 2 (∀n ∈ N), prove that there is a triangle with side lengths an − 1, an , an + 1, and that its area is equal to an integer. 36. (HEL 4) A regular tetrahedron A1 B1C1 D1 is inscribed in a regular tetrahedron ABCD, where A1 lies in the plane BCD, B1 in the plane ACD, etc. Prove that A1 B1 ≥ AB/3. 37. (HEL 5) (SL79-14). 38. (HUN 1) Prove the following statement: If a polynomial f (x) with real coefficients takes only nonnegative values, then there exists a positive integer n and polynomials g1 (x), g2 (x), . . . , gn (x) such that 128 3 Problems f (x) = g1 (x)2 + g2 (x)2 + · · · + gn (x)2 . 39. (HUN 2) A desert expedition camps at the border of the desert, and has to provide one liter of drinking water for another member of the expedition, residing on the distance of n days of walking from the camp, under the following conditions: (i) Each member of the expedition can pick up at most 3 liters of water. (ii) Each member must drink one liter of water every day spent in the desert. (iii) All the members must return to the camp. How much water do they need (at least) in order to do that? 40. (HUN 3) A polynomial P(x) has degree at most 2k, where k = 0, 1, 2, . . . . Given that for an integer i, the inequality −k ≤ i ≤ k implies |P(i)| ≤ 1, prove that for all real numbers x, with −k ≤ x ≤ k, the following inequality holds:   2k |P(x)| < (2k + 1) . k 41. (HUN 4) Prove the following statement: There does not exist a pyramid with square base and congruent lateral faces for which the measures of all edges, total area, and volume are integers. 42. (HUN 5) Let a quadratic polynomial g(x) = ax2 + bx + c be given and an integer n ≥ 1. Prove that there exists at most one polynomial f (x) of nth degree such that f (g(x)) = g( f (x)). 43. (ISR 1) Let a, b, c denote the lengths of the sides BC,CA, AB, respectively, of a triangle ABC. If P is any point on the circumference of the circle inscribed in the triangle, show that aPA2 + bPB2 + cPC2 is constant. 44. (ISR 2) (SL79-15). 45. (ISR 3) For any positive integer n we denote by F(n) the number of ways in which n can be expressed as the sum of three different positive integers, without regard to order. Thus, since 10 = 7 + 2 + 1 = 6 + 3 + 1 = 5 + 4 + 1 = 5 + 3 + 2, we have F(10) = 4. Show that F(n) is even if n ≡ 2 or 4 (mod 6), but odd if n is divisible by 6. 46. (ISR 4) (SL79-16). 47. (NLD 1) (SL79-17). 48. (NLD 2) In the plane a circle C of unit radius is given. For any line l a number s(l) is defined in the following way: If l and C intersect in two points, s(l) is their distance; otherwise, s(l) = 0. Let P be a point at distance r from the center of C. One defines M(r) to be the maximum value of the sum s(m) + s(n), where m and n are variable mutually orthogonal lines through P. Determine the values of r for which M(r) > 2. 49. (NLD 3) Let there be given two sequences of integers f i (1), fi (2), . . . (i = 1, 2) satisfying: 3.21 IMO 1979 129 (i) fi (nm) = fi (n) fi (m) if gcd(n, m) = 1; (ii) for every prime P and all k = 2, 3, 4, . . ., fi (Pk ) = fi (P) fi (Pk−1 ) − P2 f (Pk−2 ). Moreover, for every prime P: (iii) f1 (P) = 2P, (iv) f2 (P) < 2P. Prove that | f2 (n)| < f1 (n) for all n. 50. (POL 1) (SL79-18). 51. (POL 2) Let ABC be an arbitrary triangle and let S1 , S2 , . . . , S7 be circles satisfying the following conditions: S1 is tangent to CA and AB, S2 is tangent to S1 , AB, and BC, S3 is tangent to S2 , BC, and CA, ·················· S7 is tangent to S6 , CA and AB. Prove that the circles S1 and S7 coincide. 52. (POL 3) Let a real number λ > 1 be given and a sequence (nk ) of positive n integers such that k+1 nk > λ for k = 1, 2, . . . . Prove that there exists a positive integer c such that no positive integer n can be represented in more than c ways in the form n = nk + n j or n = nr − ns . 53. (POL 4) An infinite increasing sequence of positive integers n j ( j = 1, 2, . . . ) has the property that for a certain c, N1 ∑n j ≤N n j ≤ c, for every N > 0 (i) Prove that there exist finitely many sequences m j (i = 1, 2, . . . , k) such that {n1 , n2 , . . . } = (i) m j+1 > (i) 2m j Sk (i) (i) i=1 {m1 , m2 , . . . } and (1 ≤ i ≤ k, j = 1, 2, . . . ). 54. (ROU 1) (SL79-19). 55. (ROU 2) Let a, b be coprime integers. Show that the equation ax2 + by2 = z3 has an infinite set of solutions (x, y, z) with x, y, z ∈ Z and x, y mutually coprime (in each solution). √ √ 56. (ROU 3) Show that for every natural number n, n 2 − [n 2] > 2n1√2 and that √ √ for every ε > 0 there exists a natural number n with n 2 − [n 2] < 2n1√2 + ε . 57. (ROU 4) Let M be a set, and A, B,C given subsets of M. Find a necessary and sufficient condition for the existence of a set X ⊂ M for which (X ∪A)\(X ∩B) = C. Describe all such sets X . 58. (ROU 5) Prove that there exists a natural number k0 such that for every natural number k > k0 we may find a finite number of lines in the plane, not all parallel to one of them, that divide the plane exactly in k regions. Find k0 . 130 3 Problems 59. (SWE 1) Determine the maximum value of x2 y2 z2 w when x, y, z, w ≥ 0 and 2x + xy + z + yzw = 1. 60. (SWE 2) (SL79-20). 61. (SWE 3) Let a1 ≤ a2 ≤ · · · ≤ an and b1 ≤ b2 ≤ · · · ≤ bn be two sequences such m that ∑m k=1 ak ≥ ∑k=1 bk for all m ≤ n with equality for m = n. Let f be a convex function defined on the real numbers. Prove that n ∑ k=1 n f (ak ) ≤ ∑ f (bk ). k=1 62. (SWE 4) T is a given triangle with vertices P1 , P2 , P3 . Consider an arbitrary subdivision of T into finitely many subtriangles such that no vertex of a subtriangle lies strictly between two vertices of another subtriangle. To each vertex V of the subtriangles there is assigned a number n(V ) according to the following rules: (i) If V = Pi , then n(V ) = i. (ii) If V lies on the side Pi Pj of T , then n(V ) = i or j. (iii) If V lies inside the triangle T , then n(V ) is any of the numbers 1,2,3. Prove that there exists at least one subtriangle whose vertices are numbered 1, 2, and 3. 63. (USA 1) If a1 , a2 , . . . , an denote the lengths of the sides of an arbitrary n-gon, prove that a1 a2 an n 2≥ + + ···+ ≥ , s − a1 s − a2 s − an n − 1 where s = a1 + a2 + · · · + an . 64. (USA 2) From point P on arc BC of the circumcircle about triangle ABC, PX is constructed perpendicular to BC, PY is perpendicular to AC, and PZ perpendicular to AB (all extended if necessary). Prove that BC AC AB = + . PX PY PZ 65. (USA 3) Given f (x) ≤ x for all real x and f (x + y) ≤ f (x) + f (y) prove that f (x) = x for all x. 66. (USA 4) (SL79-23). 67. (USA 5) (SL79-24). 68. (USA 6) (SL79-25). 69. (USS 1) (SL79-21). for all real x, y, 3.21 IMO 1979 131 70. (USS 2) There are 1979 equilateral triangles: T1 , T2 , . . . , T1979 . A side of triangle Tk is equal to 1/k, k = 1, 2, . . . , 1979. At what values of a number a can one place all these triangles into the equilateral triangle with side length a so that they don’t intersect (points of contact are allowed)? 71. (USS 3) (SL79-22). 72. (VNM 1) Let f (x) be a polynomial with integer coefficients. Prove that if f (x) equals 1979 for four different integer values of x, then f (x) cannot be equal to 2 × 1979 for any integral value of x. 73. (VNM 2) In a plane a finite number of equal circles are given. These circles are mutually nonintersecting (they may be externally tangent). Prove that one can use at most four colors for coloring these circles so that two circles tangent to each other are of different colors. What is the smallest number of circles that requires four colors? 74. (VNM 3) Given an equilateral triangle ABC of side a in a plane, let M be a point on the circumcircle of the triangle. Prove that the sum s = MA4 + MB4 + MC4 is independent of the position of the point M on the circle, and determine that constant value as a function of a. 75. (VNM 4) Given an equilateral triangle ABC, let M be an arbitrary point in space. (a) Prove that one can construct a triangle from the segments MA, MB, MC. (b) Suppose that P and Q are two points symmetric with respect to the center O of ABC. Prove that the two triangles constructed from the segments PA, PB, PC and QA, QB, QC are of equal area. 76. (VNM 5) Suppose that a triangle whose sides are of integer lengths is inscribed in a circle of diameter 6.25. Find the sides of the triangle. 77. (YUG 1) By h(n), where n is an integer greater than 1, let us denote the greatest prime divisor of the number n. Are there infinitely many numbers n for which h(n) < h(n + 1) < h(n + 2) holds? 78. (YUG 2) By ω (n), where n is an integer greater than 1, let us denote the number of different prime divisors of the number n. Prove that there exist infinitely many numbers n for which ω (n) < ω (n + 1) < ω (n + 2) holds. 79. (YUG 3) Let S be a unit circle and K a subset of S consisting of several closed arcs. Let K satisfy the following properties: (i) K contains three points A, B,C, that are the vertices of an acute-angled triangle; (ii) for every point A that belongs to K its diametrically opposite point A′ and all points B on an arc of length 1/9 with center A′ do not belong to K. Prove that there are three points E, F, G on S that are vertices of an equilateral triangle and that do not belong to K. 80. (YUG 4) (SL79-26). 132 3 Problems 81. (YUG 5) Let P be the set of rectangular parallelepipeds that have at least one edge of integer length. If a rectangular parallelepiped P0 can be decomposed into parallelepipeds P1 , P2 , . . . , Pn ∈ P, prove that P0 ∈ P. 3.21.3 Shortlisted Problems 1. (BEL 1) Prove that in the Euclidean plane every regular polygon having an even number of sides can be dissected into lozenges. (A lozenge is a quadrilateral whose four sides are all of equal length). 2. (BEL 4) From a bag containing 5 pairs of socks, each pair a different color, a random sample of 4 single socks is drawn. Any complete pairs in the sample are discarded and replaced by a new pair drawn from the bag. The process continues until the bag is empty or there are 4 socks of different colors held outside the bag. What is the probability of the latter alternative? 3. (BGR 1) Find all polynomials f (x) with real coefficients for which f (x) f (2x2 ) = f (2x3 + x). 4. (BGR 3)IMO2 A pentagonal prism A1 A2 . . . A5 B1 B2 . . . B5 is given. The edges, the diagonals of the lateral walls and the internal diagonals of the prism are each colored either red or green in such a way that no triangle whose vertices are vertices of the prism has its three edges of the same color. Prove that all edges of the bases are of the same color. 5. (CZS 2) Let n ≥ 2 be an integer. Find the maximal cardinality of a set M of pairs ( j, k) of integers, 1 ≤ j < k ≤ n, with the following property: If ( j, k) ∈ M, then (k, m) 6∈ M for any m. 6. (CZS 4) Find the real values of p for which the equation p p p 2p + 1 − x2 + 3x + p + 4 = x2 + 9x + 3p + 9 √ in x has exactly two real distinct roots ( t means the positive square root of t). 1 1 7. (FRG 1)IMO1 Given that 1 − 12 + 13 − 14 + · · · − 1318 + 1319 = qp , where p and q are natural numbers having no common factor, prove that p is divisible by 1979. 8. (FRG 3) For all rational x satisfying 0 ≤ x < 1, f is defined by ( f (2x)/4, for 0 ≤ x < 1/2, f (x) = 3/4 + f (2x − 1)/4, for 1/2 ≤ x < 1. Given that x = 0.b1 b2 b3 . . . is the binary representation of x, find f (x). 9. (FRG 4)IMO6 Let S and F be two opposite vertices of a regular octagon. A counter starts at S and each second is moved to one of the two neighboring vertices of the octagon. The direction is determined by the toss of a coin. The 3.21 IMO 1979 133 process ends when the counter reaches F. We define an to be the number of distinct paths of duration n seconds that the counter may take to reach F from S. Prove that for n = 1, 2, 3, . . . , a2n−1 = 0, 1 a2n = √ (xn−1 − yn−1 ), 2 √ √ where x = 2 + 2, y = 2 − 2. 10. (FIN 3) Show that for any vectors a, b in Euclidean space, √ 3 3 2 2 3 |a × b| ≤ |a| |b| |a − b|2. 8 Remark. Here × denotes the vector product. 11. (GDR 1) Given real numbers x1 , x2 , . . . , xn (n ≥ 2), with xi ≥ 1/n (i = 1, 2, . . . , n) and with x21 + x22 + · · · + x2n = 1, find whether the product P = x1 x2 x3 · · · xn has a greatest and/or least value and if so, give these values. 12. (GDR 3) Let R be a set of exactly 6 elements. A set F of subsets of R is called an S-family over R if and only if it satisfies the following three conditions: (i) For no two sets X ,Y in F is X ⊆ Y ; (ii) For any three sets X ,Y, Z in F, X ∪Y ∪ Z 6= R, S (iii) X∈F X = R. We define |F| to be the number of elements of F (i.e., the number of subsets of R belonging to F). Determine, if it exists, h = max|F|, the maximum being taken over all S-families over R. 13. (HEL 1) Show that 20 60 < sin 20◦ < 21 60 . 14. (HEL 5) Find all bases of logarithms in which a real positive number can be equal to its logarithm or prove that none exist. 15. (ISR 2)IMO5 The nonnegative real numbers x1 , x2 , x3 , x4 , x5 , a satisfy the following relations: 5 ∑ ixi = a, i=1 5 ∑ i3 xi = a2, i=1 5 ∑ i5 xi = a3. i=1 What are the possible values of a? 16. (ISR 4) Let K denote the set {a, b, c, d, e}. F is a collection of 16 different subsets of K, and it is known that any three members of F have at least one element in common. Show that all 16 members of F have exactly one element in common. 17. (NLD 1) Inside an equilateral triangle ABC one constructs points P, Q and R such that ∠QAB = ∠PBA = 15◦ , ∠RBC = ∠QCB = 20◦ , ∠PCA = ∠RAC = 25◦ . Determine the angles of triangle PQR. 134 3 Problems 18. (POL 1) Let m positive integers a1 , . . . , am be given. Prove that there exist fewer than 2m positive integers b1 , . . . , bn such that all sums of distinct bk ’s are distinct and all ai (i ≤ m) occur among them. 19. (ROU 1) Consider the sequences (an ), (bn ) defined by a1 = 3, b1 = 100, an+1 = 3an , bn+1 = 100bn . Find the smallest integer m for which bm > a100 . 20. (SWE 2) Given the integer n > 1 and the real number a > 0 determine the maximum of ∑n−1 i=1 xi xi+1 taken over all nonnegative numbers xi with sum a. 21. (USS 1) Let N be the number of integral solutions of the equation x2 − y2 = z3 − t 3 satisfying the condition 0 ≤ x, y, z,t ≤ 106 , and let M be the number of integral solutions of the equation x2 − y2 = z3 − t 3 + 1 satisfying the condition 0 ≤ x, y, z,t ≤ 106 . Prove that N > M. 22. (USS 3)IMO3 There are two circles in the plane. Let a point A be one of the points of intersection of these circles. Two points begin moving simultaneously with constant speeds from the point A, each point along its own circle. The two points return to the point A at the same time. Prove that there is a point P in the plane such that at every moment of time the distances from the point P to the moving points are equal. 23. (USA 4) Find all natural numbers n for which 28 + 211 + 2n is a perfect square. 24. (USA 5) A circle O with center O on base BC of an isosceles triangle ABC is tangent to the equal sides AB, AC. If point P on AB and point Q on AC are selected such that PB ×CQ = (BC/2)2 , prove that line segment PQ is tangent to circle O, and prove the converse. 25. (USA 6)IMO4 Given a point P in a given plane π and also a given point Q not in π , show how to determine a point R in π such that QP+PR is a maximum. QR 26. (YUG 4) Prove that the functional equations and are equivalent. f (x + y) = f (x) + f (y), f (x + y + xy) = f (x) + f (y) + f (xy) (x, y ∈ R) 3.22 IMO 1981 135 3.22 The Twenty-Second IMO Washington DC, United States of America, July 8–20, 1981 3.22.1 Contest Problems First Day (July 13) 1. Find the point P inside the triangle ABC for which BC CA AB + + PD PE PF is minimal, where PD, PE, PF are the perpendiculars from P to BC, CA, AB respectively. 2. Let f (n, r) be the arithmetic mean of the minima of all r-subsets of the set {1, 2, . . . , n}. Prove that f (n, r) = n+1 r+1 . 3. Determine the maximum value of m2 + n2 where m and n are integers satisfying m, n ∈ {1, 2, . . . , 1981} and (n2 − mn − m2)2 = 1. Second Day (July 14) 4. (a) For which values of n > 2 is there a set of n consecutive positive integers such that the largest number in the set in the set is a divisor of the least common multiple of the remaining n − 1 numbers? (b) For which values of n > 2 is there a unique set having the stated property? 5. Three equal circles touch the sides of a triangle and have one common point O. Show that the center of the circle inscribed in and of the circle circumscribed about the triangle ABC and the point O are collinear. 6. Assume that f (x, y) is defined for all positive integers x and y, and that the following equations are satisfied: f (0, y) = y + 1, f (x + 1, 0) = f (x, 1), f (x + 1, y + 1) = f (x, f (x + 1, y)). Determine f (4, 1981). 3.22.2 Shortlisted Problems 1. (BEL)IMO4 (a) For which values of n > 2 is there a set of n consecutive positive integers such that the largest number in the set is a divisor of the least common multiple of the remaining n − 1 numbers? (b) For which values of n > 2 is there a unique set having the stated property? 136 3 Problems 2. (BGR) A sphere S is tangent to the edges AB, BC,CD, DA of a tetrahedron ABCD at the points E, F, G, H respectively. The points E, F, G, H are the vertices of a square. Prove that if the sphere is tangent to the edge AC, then it is also tangent to the edge BD. 3. (CAN) Find the minimum value of max(a + b + c, b + c + d, c + d + e, d + e + f , e + f + g) subject to the constraints (i) a, b, c, d, e, f , g ≥ 0, (ii) a + b + c + d + e + f + g = 1. 4. (CAN) Let { f n } be the Fibonacci sequence {1, 1, 2, 3, 5, . . .}. (a) Find all pairs (a, b) of real numbers such that for each n, a fn + b fn+1 is a member of the sequence. (b) Find all pairs (u, v) of positive real numbers such that for each n, u fn2 + 2 v fn+1 is a member of the sequence. 5. (COL) A cube is assembled with 27 white cubes. The larger cube is then painted black on the outside and disassembled. A blind man reassembles it. What is the probability that the cube is now completely black on the outside? Give an approximation of the size of your answer. 6. (CUB) Let P(z) and Q(z) be complex-variable polynomials, with degree not less than 1. Let Pk = {z ∈ C | P(z) = k}, Qk = {z ∈ C | Q(z) = k}. Let also P0 = Q0 and P1 = Q1 . Prove that P(z) ≡ Q(z). 7. (FIN)IMO6 Assume that f (x, y) is defined for all positive integers x and y, and that the following equations are satisfied: f (0, y) = y + 1, f (x + 1, 0) = f (x, 1), f (x + 1, y + 1) = f (x, f (x + 1, y)). Determine f (2, 2), f (3, 3) and f (4, 4). Alternative version: Determine f (4, 1981). 8. (FRG)IMO2 Let f (n, r) be the arithmetic mean of the minima of all r-subsets of the set {1, 2, . . . , n}. Prove that f (n, r) = n+1 r+1 . 9. (FRG) A sequence (an ) is defined by means of the recursion √ 1 + 4an + 1 + 24an a1 = 1, an+1 = . 16 Find an explicit formula for an . 3.22 IMO 1981 137 10. (FRA) Determine the smallest natural number n having the following property: For every integer p, p ≥ n, it is possible to subdivide (partition) a given square into p squares (not necessarily equal). 11. (NLD) On a semicircle with unit radius four consecutive chords AB, BC, CD, DE with lengths a, b, c, d, respectively, are given. Prove that a2 + b2 + c2 + d 2 + abc + bcd < 4. 12. (NLD)IMO3 Determine the maximum value of m2 +n2 where m and n are integers satisfying m, n ∈ {1, 2, . . .,100} and (n2 − mn − m2)2 = 1. 13. (ROU) Let P be a polynomial of degree n satisfying   n + 1 −1 P(k) = for k = 0, 1, . . . , n. k Determine P(n + 1). 14. (ROU) Prove that a convex pentagon (a five-sided polygon) ABCDE with equal sides and for which the interior angles satisfy the condition ∠A ≥ ∠B ≥ ∠C ≥ ∠D ≥ ∠E is a regular pentagon. 15. (UNK)IMO1 Find the point P inside the triangle ABC for which BC CA AB + + PD PE PF is minimal, where PD, PE, PF are the perpendiculars from P to BC,CA, AB respectively. 16. (UNK) A sequence of real numbers u1 , u2 , u3 , . . . is determined by u1 and the following recurrence relation for n ≥ 1: p 4un+1 = 3 64un + 15. Describe, with proof, the behavior of un as n → ∞. 17. (USS)IMO5 Three equal circles touch the sides of a triangle and have one common point O. Show that the center of the circle inscribed in and of the circle circumscribed about the triangle ABC and the point O are collinear. 18. (USS) Several equal spherical planets are given in outer space. On the surface of each planet there is a set of points that is invisible from any of the remaining planets. Prove that the sum of the areas of all these sets is equal to the area of the surface of one planet. 19. (YUG) A finite set of unit circles is given in a plane such that the area of their union U is S. Prove that there exists a subset of mutually disjoint circles such that the area of their union is greater than 2S 9 . 138 3 Problems 3.23 The Twenty-Third IMO Budapest, Hungary, July 5–14, 1982 3.23.1 Contest Problems First Day (July 9) 1. The function f (n) is defined for all positive integers n and takes on nonnegative integer values. Also, for all m, n, f (m + n) − f (m) − f (n) = 0 or 1; f (2) = 0, f (3) > 0, and f (9999) = 3333. Determine f (1982). 2. A nonisosceles triangle A1 A2 A3 is given with sides a1 , a2 , a3 (ai is the side opposite to Ai ). For all i = 1, 2, 3, Mi is the midpoint of side ai , Ti is the point where the incircle touches side ai , and the reflection of Ti in the interior bisector of Ai yields the point Si . Prove that the lines M1 S1 , M2 S2 , and M3 S3 are concurrent. 3. Consider the infinite sequences {xn } of positive real numbers with the following properties: x0 = 1 and for all i ≥ 0, xi+1 ≤ xi . (a) Prove that for every such sequence there is an n ≥ 1 such that x2 x20 x21 + + · · · + n−1 ≥ 3.999. x1 x2 xn (b) Find such a sequence for which x20 x1 x2 + x12 + · · · + x2n−1 xn < 4 for all n. Second Day (July 10) 4. Prove that if n is a positive integer such that the equation x3 − 3xy2 + y3 = n has a solution in integers (x, y), then it has at least three such solutions. Show that the equation has no solution in integers when n = 2891. 5. The diagonals AC and CE of the regular hexagon ABCDEF are divided by the CN inner points M and N, respectively, so that AM AC = CE = r. Determine r if B, M, and N are collinear. 6. Let S be a square with sides of length 100 and let L be a path within S that does not meet itself and that is composed of linear segments A0 A1 , A1 A2 , . . . , An−1 An with A0 6= An . Suppose that for every point P of the boundary of S there is a point of L at a distance from P not greater than 12 . Prove that there are two points X and Y in L such that the distance between X and Y is not greater than 1 and the length of the part of L that lies between X and Y is not smaller than 198. 3.23 IMO 1982 139 3.23.2 Longlisted Problems  n! 1. (AUS 1) It is well known that the binomial coefficients nk = k!(n−k)! , 0≤ k ≤ n, are positive integers. The factorial n! is defined inductively by 0! = 1, n! = n · (n − 1)! for n ≥ 1. 1 2n (a) Prove that n+1 n is an integer for n ≥ 0. (b) Given a positive integer  k, determine the smallest integer Ck with the propCk 2n erty that n+k+1 n+k is an integer for all n ≥ k. 2. (AUS 2) Given a finite number of angular regions A1 , . . . , Ak in a plane, each Ai being bounded by two half-lines meeting at a vertex and provided with a + or − sign, we assign to each point P of the plane and not on a bounding half-line the number k − l, where k is the number of + regions and l the number of − regions A that contain P. (Note that the boundary A of Ai does not belong to Ai .) For in+ A B C  stance, in the figure we have two + re  R + −A gions QAP and RCQ, and one − re AP Q A gion RBP. Every point inside △ABC receives the number +1, while every point not inside △ABC and not on a boundary halfline the number 0. We say that the interior of △ABC is represented as a sum of the signed angular regions QAP, RBP, and RCQ. (a) Show how to represent the interior of any convex planar polygon as a sum of signed angular regions. (b) Show how to represent the interior of a tetrahedron as a sum of signed solid angular regions, that is, regions bounded by three planes intersecting at a vertex and provided with a + or − sign. 3. (AUS 3) Given n points X1 , X2 , . . . , Xn in the interval 0 ≤ Xi ≤ 1, i = 1, 2, . . . , n, show that there is a point y, 0 ≤ y ≤ 1, such that 1 n 1 |y − Xi| = . ∑ n i=1 2 4. (AUS 4) (SL82-14). Original formulation. Let ABCD be a convex planar quadrilateral and let A1 denote the circumcenter of △BCD. Define B1 ,C1 , D1 in a corresponding way. (a) Prove that either all of A1 , B1 ,C1 , D1 coincide in one point, or they are all distinct. Assuming the latter case, show that A1 ,C1 are on opposite sides of the line B1 D1 , and similarly, B1 , D1 are on opposite sides of the line A1C1 . (This establishes the convexity of the quadrilateral A1 B1C1 D1 .) (b) Denote by A2 the circumcenter of B1C1 D1 , and define B2 ,C2 , D2 in an analogous way. Show that the quadrilateral A2 B2C2 D2 is similar to the quadrilateral ABCD. (c) If the quadrilateral A1 B1C1 D1 was obtained from the quadrilateral ABCD by the above process, what condition must be satisfied by the four points 140 3 Problems A1 , B1 ,C1 , D1 ? Assuming that the four points A1 , B1 ,C1 , D1 satisfying this condition are given, describe a construction by straightedge and compass to obtain the original quadrilateral ABCD. (It is not necessary to actually perform the construction). 5. (BEL 1) Among all triangles with a given perimeter, find the one with the maximal radius of its incircle. 6. (BEL 2) On the three distinct lines a, b, and c three points A, B, and C are given, respectively. Construct three collinear points X ,Y, Z on lines a, b, c, respectively, BY such that AX = 2 and CZ AX = 3. 7. (BEL 3) Find all solutions (x, y) ∈ Z2 of the equation x3 − y3 = 2xy + 8. 8. (BRA 1) (SL82-10). 9. (BRA 2) Let n be a natural number, n ≥ 2, and let φ be Euler’s function; i.e., φ (n) is the number of positive integers not exceeding n and coprime to n. Given any two real numbers α and β , 0 ≤ α < β ≤ 1, prove that there exists a natural number m such that φ (m) α< < β. m 10. (BRA 3) Let r1 , . . . , rn be the radii of n spheres. Call S1 , S2 , . . . , Sn the areas of the set of points of each sphere from which one cannot see any point of any other sphere. Prove that S1 S2 Sn + + · · · + 2 = 4π . rn r12 r22 11. (BRA 4) A rectangular pool table has a hole at each of three of its corners. The lengths of sides of the table are the real numbers a and b. A billiard ball is shot from the fourth corner along its angle bisector. The ball falls in one of the holes. What should the relation between a and b be for this to happen? 12. (BRA 5) Let there be 3399 numbers arbitrarily chosen among the first 6798 integers 1, 2, . . . , 6798 in such a way that none of them divides another. Prove that there are exactly 1982 numbers in {1, 2, . . ., 6798} that must end up being chosen. 13. (BGR 1) A regular n-gonal truncated pyramid is circumscribed around a sphere. Denote the areas of the base and the lateral surfaces of the pyramid by S1 , S2 , and S, respectively. Let σ be the area of the polygon whose vertices are the tangential points of the sphere and the lateral faces of the pyramid. Prove that σ S = 4S1 S2 cos2 14. (BGR 2) (SL82-4). π . n 3.23 IMO 1982 141 15. (CAN 1) Show that the set S of natural numbers n for which 3/n cannot be written as the sum of two reciprocals of natural numbers (S = {n | 3/n 6= 1/p + 1/q for any p, q ∈ N}) is not the union of finitely many arithmetic progressions. 16. (CAN 2) (SL82-7). 17. (CAN 3) (SL82-11). 18. (CAN 4) You are given an algebraic system admitting addition and multiplication for which all the laws of ordinary arithmetic are valid except commutativity of multiplication. Show that (a + ab−1a)−1 + (a + b)−1 = a−1 , where x−1 is the element for which x−1 x = xx−1 = e, where e is the element of the system such that for all a the equality ea = ae = a holds. 19. (CAN 5) (SL82-15). 20. (CZS 1) Consider a cube C and two planes σ , τ , which divide Euclidean space into several regions. Prove that the interior of at least one of these regions meets at least three faces of the cube. 21. (CZS 2) All edges and all diagonals of regular hexagon A1 A2 A3 A4 A5 A6 are colored blue or red such that each triangle A j Ak Am , 1 ≤ j < k < m ≤ 6 has at least one red edge. Let Rk be the number of red segments Ak A j , ( j 6= k). Prove the inequality 6 ∑ (2Rk − 7)2 ≤ 54. k=1 22. (CZS 3) (SL82-19). 23. (FIN 1) Determine the sum of all positive integers whose digits (in base ten) form either a strictly increasing or a strictly decreasing sequence. 24. (FIN 2) Prove that if a person a has infinitely many descendants (children, their children, etc.), then a has an infinite sequence a0 , a1 , . . . of descendants (i.e., a = a0 and for all n ≥ 1, an+1 is always a child of an ). It is assumed that no-one can have infinitely many children. Variant 1. Prove that if a has infinitely many ancestors, then a has an infinite descending sequence of ancestors (i.e., a0 , a1 , . . . where a = a0 and an is always a child of an+1 ). Variant 2. Prove that if someone has infinitely many ancestors, then all people cannot descend from A(dam) and E(ve). 25. (FIN 3) (SL82-12). 26. (FRA 1) Let (an )n≥0 and (bn )n≥0 be two sequences of natural numbers. Determine whether there exists a pair (p, q) of natural numbers that satisfy p<q and a p ≤ aq , b p ≤ bq . 142 3 Problems 27. (FRA 2) (SL82-18). 28. (FRA 3) Let (u1 , . . . , un ) be an ordered ntuple. For each k, 1 ≤ k ≤ n, define √ vk = k u1 u2 · · · uk . Prove that n n k=1 k=1 ∑ vk ≤ e · ∑ uk . (e is the base of the natural logarithm). 29. (FRA 4) Let f : R → R be a continuous function. Suppose that the restriction of f to the set of irrational numbers is injective. What can we say about f ? Answer the analogous question if f is restricted to rationals. 30. (UNK 1) (SL82-9). 31. (UNK 2) (SL82-16). 32. (UNK 3) (SL82-1). 33. (UNK 4) A sequence (un ) of integers is defined for n ≥ 0 by u0 = 0, u1 = 1, and un − 2un−1 + (1 − c)un−2 = 0 (n ≥ 2), where c is a fixed integer independent of n. Find the least value of c for which both of the following statements are true: (i) If p is a prime less than or equal to P, then p divides u p . (ii) If p is a prime greater than P, then p does not divide u p . 34. (GDR 1) Let M be the set of all functions f with the following properties: (i) f is defined for all real numbers and takes only real values. (ii) For all x, y ∈ R the following equality holds: f (x) f (y) = f (x+y)+ f (x−y). (iii) f (0) 6= 0. Determine all functions f ∈ M such that (a) f (1) = 5/2; √ (b) f (1) = 3. 35. (GDR 2) If the inradius of a triangle is half of its circumradius, prove that the triangle is equilateral. 36. (NLD 1) (SL82-13). 37. (NLD 2) (SL82-5). 38. (POL 1) Numbers un,k (1 ≤ k ≤ n) are defined as follows:   n u1,1 = 1, un,k = − ∑ un/d,k/d k d|n, d|k, d>1 (the empty sum is defined to be equal to zero). Prove that n | un,k for every natural number n and for every k (1 ≤ k ≤ n). 39. (POL 2) Let S be the unit circle with center O and let P1 , P2 , . . . , Pn be points of S −→ such that the sum of vectors vi = OPi is the zero vector. Prove that the inequality ∑ni=1 XPi ≥ n holds for every point X . 3.23 IMO 1982 143 40. (POL 3) We consider a game on an infinite chessboard similar to that of solitaire: If two adjacent fields are occupied by pawns and the next field is empty (the three fields lie on a vertical or horizontal line), then we may remove these two pawns and put one of them on the third field. Prove that if in the initial position pawns fill a 3k × n rectangle, then it is impossible to reach a position with only one pawn on the board. 41. (POL 4) (SL82-8). 42. (POL 5) Let F be the family of all k-element subsets of the set {1, 2, . . . , 2k + 1}. Prove that there exists a bijective function f : F → F such that for every A ∈ F , the sets A and f (A) are disjoint. 43. (TUN 1) (a) What is the maximal number of acute angles in a convex polygon? (b) Consider m points in the interior of a convex n-gon. The n-gon is partitioned into triangles whose vertices are among the n + m given points (the vertices of the n-gon and the given points). Each of the m points in the interior is a vertex of at least one triangle. Find the number of triangles obtained. 44. (TUN 2) Let A and B be positions of two ships M and N, respectively, at the moment when N saw M moving with constant speed v following the line Ax. In search of help, N moves with speed kv (k < 1) along the line By in order to meet M as soon as possible. Denote by C the point of meeting of the two ships, and set π AB = d, ∠BAC = α , 0 ≤ α < . 2 Determine the angle ∠ABC = β and time t that N needs in order to meet M. 45. (TUN 3) (SL82-20). 46. (USA 1) Prove that if a diagonal is drawn in a quadrilateral inscribed in a circle, the sum of the radii of the circles inscribed in the two triangles thus formed is the same, no matter which diagonal is drawn. 47. (USA 2) Evaluate sec′′ π4 + sec′′ 34π + sec′′ 54π + sec′′ 74π . (Here sec′′ means the second derivative of sec.) 48. (USA 3) Given a finite sequence of complex numbers c1 , c2 , . . . , cn , show that there exists an integer k (1 ≤ k ≤ n) such that for every finite sequence a1 , a2 , . . . , an of real numbers with 1 ≥ a1 ≥ a2 ≥ · · · ≥ an ≥ 0, the following inequality holds: n ∑ am cm m=1 49. (USA 4) Simplify n k ≤ ∑ cm . m=1 (2n)! ∑ (k!)2 ((n − k)!)2 . k=0 50. (USS 1) Let O be the midpoint of the axis of a right circular cylinder. Let A and B be diametrically opposite points of one base, and C a point of the other 144 3 Problems base circle that does not belong to the plane OAB. Prove that the sum of dihedral angles of the trihedral OABC is equal to 2π . 51. (USS 2) Let n numbers x1 , x2 , . . . , xn be chosen in such a way that 1 ≥ x1 ≥ x2 ≥ · · · ≥ xn ≥ 0. Prove that (1 + x1 + x2 + · · · + xn )α ≤ 1 + xα1 + 2α −1xα2 + · · · + nα −1 xαn if 0 ≤ α ≤ 1. 52. (USS 3) We are given 2n natural numbers 1, 1, 2, 2, 3, 3, . . ., n − 1, n − 1, n, n. Find all n for which these numbers can be arranged in a row such that for each k ≤ n, there are exactly k numbers between the two numbers k. 53. (USS 4) (SL82-3). 54. (USS 5) (SL82-17). 55. (VNM 1) (SL82-6). 56. (VNM 2) Let f (x) = ax2 + bx+ c and g(x) = cx2 +bx +a. If | f (0)| ≤ 1, | f (1)| ≤ 1, | f (−1)| ≤ 1, prove that for |x| ≤ 1, (a) | f (x)| ≤ 5/4, (b) |g(x)| ≤ 2. 57. (YUG 1) (SL82-2). 3.23.3 Shortlisted Problems 1. A1 (UNK 3)IMO1 The function f (n) is defined for all positive integers n and takes on nonnegative integer values. Also, for all m, n, f (m + n) − f (m) − f (n) = 0 or 1; f (2) = 0, f (3) > 0, and f (9999) = 3333. Determine f (1982). 2. A2 (YUG 1) Let K be a convex polygon in the plane and suppose that K is positioned in the coordinate system in such a way that area (K ∩ Qi ) = 1 area K (i = 1, 2, 3, 4, ), 4 where the Qi denote the quadrants of the plane. Prove that if K contains no nonzero lattice point, then the area of K is less than 4. 3. A3 (USS 4)IMO3 Consider the infinite sequences {xn } of positive real numbers with the following properties: x0 = 1 and for all i ≥ 0, xi+1 ≤ xi . 3.23 IMO 1982 145 x20 x1 + x1 + (a) Prove that for every such sequence there is an n ≥ 1 such that ··· + x2n−1 xn ≥ 3.999. (b) Find such a sequence for which x20 x1 x2 + x12 + · · · + x2n−1 xn x2 2 < 4 for all n. 4. A4 (BGR 2) Determine all real values of the parameter a for which the equation 16x4 − ax3 + (2a + 17)x2 − ax + 16 = 0 has exactly four distinct real roots that form a geometric progression. 5. A5 (NLD 2)IMO5 Let A1 A2 A3 A4 A5 A6 be a regular hexagon. Each of its diagonals Ai−1 Ai+1 is divided into the same ratio 1−λ λ , where 0 < λ < 1, by a point Bi in such a way that Ai , Bi , and Bi+2 are collinear (i ≡ 1, . . . , 6 (mod 6)). Compute λ . 6. A6 (VNM 1)IMO6 Let S be a square with sides of length 100 and let L be a path within S that does not meet itself and that is composed of linear segments A0 A1 , A1 A2 , . . . , An−1 An with A0 6= An . Suppose that for every point P of the boundary of S there is a point of L at a distance from P not greater than 12 . Prove that there are two points X and Y in L such that the distance between X and Y is not greater than 1 and the length of that part of L that lies between X and Y is not smaller than 198. 7. B1 (CAN 2) Let p(x) be a cubic polynomial with integer coefficients with leading coefficient 1 and with one of its roots equal to the product of the other two. Show that 2p(−1) is a multiple of p(1) + p(−1) − 2(1 + p(0)). 8. B2 (POL 4) A convex, closed figure lies inside a given circle. The figure is seen from every point of the circumference at a right angle (that is, the two rays drawn from the point and supporting the convex figure are perpendicular). Prove that the center of the circle is a center of symmetry of the figure. 9. B3 (UNK 1) Let ABC be a triangle, and let P be a point inside it such that ∡PAC = ∡PBC. The perpendiculars from P to BC and CA meet these lines at L and M, respectively, and D is the midpoint of AB. Prove that DL = DM. 10. B4 (BRA 1) A box contains p white balls and q black balls. Beside the box there is a pile of black balls. Two balls are taken out of the box. If they have the same color, a black ball from the pile is put into the box. If they have different colors, the white ball is put back into the box. This procedure is repeated until the last two balls are removed from the box and one last ball is put in. What is the probability that this last ball is white? 11. B5 (CAN 3) (a) Find the rearrangement {a1 , . . . , an } of {1, 2, . . . , n} that maximizes a1 a2 + a2a3 + · · · + an a1 = Q. (b) Find the rearrangement that minimizes Q. 12. B6 (FIN 3) Four distinct circles C,C1 ,C2 ,C3 and a line L are given in the plane such that C and L are disjoint and each of the circles C1 ,C2 ,C3 touches the other 146 3 Problems two, as well as C and L. Assuming the radius of C to be 1, determine the distance between its center and L. 13. C1 (NLD 1)IMO2 A scalene triangle A1 A2 A3 is given with sides a1 , a2 , a3 (ai is the side opposite to Ai ). For all i = 1, 2, 3, Mi is the midpoint of side ai , Ti is the point where the incircle touches side ai , and the reflection of Ti in the interior bisector of Ai yields the point Si . Prove that the lines M1 S1 , M2 S2 , and M3 S3 are concurrent. 14. C2 (AUS 4) Let ABCD be a convex plane quadrilateral and let A1 denote the circumcenter of △BCD. Define B1 ,C1 , D1 in a corresponding way. (a) Prove that either all of A1 , B1 ,C1 , D1 coincide in one point, or they are all distinct. Assuming the latter case, show that A1 ,C1 are on opposite sides of the line B1 D1 , and similarly, B1 , D1 are on opposite sides of the line A1C1 . (This establishes the convexity of the quadrilateral A1 B1C1 D1 .) (b) Denote by A2 the circumcenter of B1C1 D1 , and define B2 ,C2 , D2 in an analogous way. Show that the quadrilateral A2 B2C2 D2 is similar to the quadrilateral ABCD. 15. C3 (CAN 5) Show that 1 − sa ≤ (1 + s)a−1 1−s holds for every 1 6= s > 0 real and 0 < a ≤ 1 rational. 16. C4 (UNK 2)IMO4 Prove that if n is a positive integer such that the equation x3 − 3xy2 + y3 = n has a solution in integers (x, y), then it has at least three such solutions. Show that the equation has no solution in integers when n = 2891. 17. C5 (USS 5) The right triangles ABC and AB1C1 are similar and have opposite orientation. The right angles are at C and C1 , and we also have ∠CAB = ∠C1 AB1 . Let M be the point of intersection of the lines BC1 and B1C. Prove that if the lines AM and CC1 exist, they are perpendicular. 18. C6 (FRA 2) Let O be a point of three-dimensional space and let l1 , l2 , l3 be mutually perpendicular straight lines passing through O. Let S denote the sphere with center O and radius R, and for every point M of S, let SM denote the sphere with center M and radius R. We denote by P1 , P2 , P3 the intersection of SM with the straight lines l1 , l2 , l3 , respectively, where we put Pi 6= O if li meets SM at two distinct points and Pi = O otherwise (i = 1, 2, 3). What is the set of centers of gravity of the (possibly degenerate) triangles P1 P2 P3 as M runs through the points of S? 19. C7 (CZS 3) Let M be the set of real numbers of the form √m+n , where m and 2 2 m +n n are positive integers. Prove that for every pair x ∈ M, y ∈ M with x < y, there exists an element z ∈ M such that x < z < y. 20. C8 (TUN 3) Let ABCD be a convex quadrilateral and draw regular triangles ABM, CDP, BCN, ADQ, the first two outward and the other two inward. Prove that MN = AC. What can be said about the quadrilateral MNPQ? 3.24 IMO 1983 147 3.24 The Twenty-Fourth IMO Paris, France, July 1–12, 1983 3.24.1 Contest Problems First Day (July 6) 1. Find all functions f defined on the positive real numbers and taking positive real values that satisfy the following conditions: (i) f (x f (y)) = y f (x) for all positive real x, y; (ii) f (x) → 0 as x → +∞. 2. Let K be one of the two intersection points of the circles W1 and W2 . Let O1 and O2 be the centers of W1 and W2 . The two common tangents to the circles meet W1 and W2 respectively in P1 and P2 , the first tangent, and Q1 and Q2 the second tangent. Let M1 and M2 be the midpoints of P1 Q1 and P2 Q2 , respectively. Prove that ∠O1 KO2 = ∠M1 KM2 . 3. Let a, b, c be positive integers satisfying (a, b) = (b, c) = (c, a) = 1. Show that 2abc − ab − bc − ca is the largest integer not representable as xbc + yca + zab with nonnegative integers x, y, z. Second Day (July 7) 4. Let ABC be an equilateral triangle. Let E be the set of all points from segments AB, BC, and CA (including A, B, and C). Is it true that for any partition of the set E into two disjoint subsets, there exists a right-angled triangle all of whose vertices belong to the same subset in the partition? 5. Prove or disprove the following statement: In the set {1, 2, 3, . . ., 105 } a subset of 1983 elements can be found that does not contain any three consecutive terms of an arithmetic progression. 6. If a, b, and c are sides of a triangle, prove that a2 b(a − b) + b2c(b − c) + c2a(c − a) ≥ 0 and determine when there is equality. 3.24.2 Longlisted Problems 1. (AUS 1) (SL83-1). 2. (AUS 2) Seventeen cities are served by four airlines. It is noted that there is direct service (without stops) between any two cities and that all airline schedules offer round-trip flights. Prove that at least one of the airlines can offer a round trip with an odd number of landings. 148 3 Problems 3. (AUS 3) (a) Given a tetrahedron ABCD and its four altitudes (i.e., lines through each vertex, perpendicular to the opposite face), assume that the altitude dropped from D passes through the orthocenter H4 of ∆ ABC. Prove that this altitude DH4 intersects all the other three altitudes. (b) If we further know that a second altitude, say the one from vertex A to the face BCD, also passes through the orthocenter H1 of ∆ BCD, then prove that all four altitudes are concurrent and each one passes through the orthocenter of the respective triangle. 4. (BEL 1) (SL83-2). 5. (BEL 2) Consider the set Q2 of points in R2 , both of whose coordinates are rational. (a) Prove that the union of segments with vertices from Q2 is the entire set R2 . (b) Is the convex hull of Q2 (i.e., the smallest convex set in R2 that contains Q2 ) equal to R2 ?6 6. (BEL 3) (SL83-3). 7. (BEL 4) Find all numbers x ∈ Z for which the number x4 + x3 + x2 + x + 1 is a perfect square. 8. (BEL 5) (SL83-4). 9. (BRA 1) (SL83-5). 10. (BRA 2) Which of the numbers 1, 2, . . . , 1983 has the largest number of divisors? 11. (BRA 3) A boy at point A wants to get water at a circular lake and carry it to point B. Find the point C on the lake such that the distance walked by the boy is the shortest possible given that the line AB and the lake are exterior to each other. 12. (BRA 4) The number 0 or 1 is to be assigned to each of the n vertices of a regular polygon. In how many different ways can this be done (if we consider two assignments that can be obtained one from the other through rotation in the plane of the polygon to be identical)? 13. (BGR 1) Let p be a prime number and a1 , a2 , . . . , a(p+1)/2 different natural numbers less than or equal to p. Prove that for each natural number r less than or equal to p, there exist two numbers (perhaps equal) ai and a j such that p ≡ ai a j (mod r). 14. (BGR 2) Let l be tangent to the circle k at B. Let A be a point on k and P the foot of perpendicular from A to l. Let M be symmetric to P with respect to AB. Find the set of all such points M. 6 The problem is unclear. In this form, part (a) is false and part (b) is trivial. 3.24 IMO 1983 149 15. (CAN 1) Find all possible finite sequences {n0 , n1 , n2 , . . . , nk } of integers such that for each i, i appears in the sequence ni times (0 ≤ i ≤ k). 16. (CAN 2) (SL83-6). 17. (CAN3) In how many  ways can 1, 2, . . . , 2n be arranged in a 2 × n rectangular a1 a2 · · · an array for which: b1 b2 · · · bn (i) a1 < a2 < · · · < an , (ii) b1 < b2 < · · · < bn , (iii) a1 < b1 , a2 < b2 , . . . , an < bn ? 18. (CAN 4) Let b ≥ 2 be a positive integer. (a) Show that for an integer N, written in base b, to be equal to the sum of the squares of its digits, it is necessary either that N = 1 or that N have only two digits. (b) Give a complete list of all integers not exceeding 50 that, relative to some base b, are equal to the sum of the squares of their digits. (c) Show that for any base b the number of two-digit integers that are equal to the sum of the squares of their digits is even. (d) Show that for any odd base b there is an integer other than 1 that is equal to the sum of the squares of its digits. 19. (CAN 5) (SL83-7). 20. (COL 1) Let f and g be functions from the set A to the same set A. We define f to be a functional nth root of g (n is a positive integer) if f n (x) = g(x), where f n (x) = f n−1 ( f (x)). (a) Prove that the function g : R → R, g(x) = 1/x has an infinite number of nth functional roots for each positive integer n. (b) Prove that there is a bijection from R onto R that has no nth functional root for each positive integer n. 21. (COL 2) Prove that there are infinitely many positive integers n for which it is possible for a knight, starting at one of the squares of an n × n chessboard, to go through each of the squares exactly once. 22. (CUB 1) Does there exist an infinite number of sets C consisting of 1983 consecutive natural numbers such that each of the numbers is divisible by some number of the form a1983, with a ∈ N, a 6= 1? 23. (FIN 1) (SL83-10). 24. (FIN 2) Every x, 0 ≤ x ≤ 1, admits a unique representation x = ∑∞j=0 a j 2− j , where all the a j belong to {0, 1} and infinitely many of them are 0. If b(0) = 1+c 2+c , 1 b(1) = 2+c , c > 0, and ∞ f (x) = a0 + ∑ b(a0 ) · · · b(a j )a j+1 , j=0 150 3 Problems show that 0 < f (x) − x < c for every x, 0 < x < 1. (FIN 2′ ) (SL83-11). 25. (FRG 1) How many permutations a1 , a2 , . . . , an of {1, 2, . . . , n} are sorted into increasing order by at most three repetitions of the following operation: Move from left to right and interchange ai and ai+1 whenever ai > ai+1 for i running from 1 up to n − 1? 26. (FRG 2) Let a, b, c be positive integers satisfying (a, b) = (b, c) = (c, a) = 1. Show that 2abc − ab − bc − ca cannot be represented as bcx + cay + abz with nonnegative integers x, y, z. 27. (FRG 3) (SL83-18). 28. (UNK 1) Show that if the sides a, b, c of a triangle satisfy the equation 2(ab2 + bc2 + ca2) = a2 b + b2 c + c2 a + 3abc, then the triangle is equilateral. Show also that the equation can be satisfied by positive real numbers that are not the sides of a triangle. 29. (UNK 2) Let O be a point outside a given circle. Two lines OAB, OCD through O meet the circle at A, B,C, D, where A,C are the midpoints of OB, OD, respectively. Additionally, the acute angle θ between the lines is equal to the acute angle at which each line cuts the circle. Find cos θ and show that the tangents at A, D to the circle meet on the line BC. 30. (UNK 3) Prove the existence of a unique sequence {un} (n = 0, 1, 2 . . . ) of positive integers such that  n  n+r u2n = ∑ un−r for all n ≥ 0, r r=0 where  m r is the usual binomial coefficient. 31. (UNK 4) (SL83-12). 32. (UNK 5) Let a, b, c be positive real numbers and let [x] denote the greatest integer that does not exceed the real number x. Suppose that f is a function defined on the set of nonnegative integers n and taking real values such that f (0) = 0 and f (n) ≤ an + f ([bn]) + f ([cn]), for all n ≥ 1. Prove that if b + c < 1, there is a real number k such that f (n) ≤ kn for all n, (1) while if b + c = 1, there is a real number K such that f (n) ≤ Kn log2 n for all n ≥ 2. Show that if b + c = 1, there may not be a real number k that satisfies (1). 33. (GDR 1) (SL83-16). 3.24 IMO 1983 151 34. (GDR 2) In a plane are given n points Pi (i = 1, 2, . . . , n) and two angles α and β . Over each of the segments Pi Pi+1 (Pn+1 = P1 ) a point Qi is constructed such that for all i: (i) upon moving from Pi to Pi+1 , Qi is seen on the same side of Pi Pi+1 , (ii) ∠Pi+1 Pi Qi = α , (iii) ∠Pi Pi+1 Qi = β . Furthermore, let g be a line in the same plane with the property that all the points Pi , Qi lie on the same side of g. Prove that n n i=1 i=1 ∑ d(Pi, g) = ∑ d(Qi , g), where d(M, g) denotes the distance from point M to line g. 35. (GDR 3) (SL83-17). 36. (ISR 1) The set X has 1983 members. There exists a family of subsets {S1 , S2 , . . . , Sk } such that: (i) the union of any three of these subsets is the entire set X , while (ii) the union of any two of them contains at most 1979 members. What is the largest possible value of k? 37. (ISR 2) The points A1 , A2 , . . . , A1983 are set on the circumference of a circle and each is given one of the values ±1. Show that if the number of points with the value +1 is greater than 1789, then at least 1207 of the points will have the property that the partial sums that can be formed by taking the numbers from them to any other point, in either direction, are strictly positive. 38. (KWT 1) Let {un } be the sequence defined by its first two terms u0 , u1 and the recursion formula un+2 = un − un+1. (a) Show that un can be written in the form un = α an + β bn , where a, b, α , β are constants independent of n that have to be determined. (b) If Sn = u0 + u1 + · · · + un , prove that Sn + un−1 is a constant independent of n. Determine this constant. 39. (KWT 2) If α is the real root of the equation E(x) = x3 − 5x − 50 = 0 such that xn+1 = (5xn + 50)1/3 and x1 = 5, where n is a positive integer, prove that: (a) x3n+1 − α 3 = 5(xn − α ) (b) α < xn+1 < xn 40. (LUX 1) Four faces of tetrahedron ABCD are congruent triangles whose angles form an arithmetic progression. If the lengths of the sides of the triangles are a < b < c, determine the radius of the sphere circumscribed about the tetrahedron as a function on a, b, and c. What is the ratio c/a if R = a? 152 3 Problems 41. (LUX 2) (SL83-13). 42. (LUX 3) Consider the square ABCD in which a segment is drawn between each vertex and the midpoints of both opposite sides. Find the ratio of the area of the octagon determined by these segments and the area of the square ABCD. 43. (LUX 4) Given a square ABCD, let P, Q, R, and S be four variable points on the sides AB, BC, CD, and DA, respectively. Determine the positions of the points P, Q, R, and S for which the quadrilateral PQRS is a parallelogram, a rectangle, a square, or a trapezoid. 44. (LUX 5) We are given twelve coins, one of which is a fake with a different mass from the other eleven. Determine that coin with three weighings and whether it is heavier or lighter than the others. 45. (LUX 6) Let two glasses, numbered 1 and 2, contain an equal quantity of liquid, milk in glass 1 and coffee in glass 2. One does the following: Take one spoon of mixture from glass 1 and pour it into glass 2, and then take the same spoon of the new mixture from glass 2 and pour it back into the first glass. What happens after this operation is repeated n times, and what as n tends to infinity? 46. (LUX 7) Let f be a real-valued function defined on I = (0, +∞) and having no zeros on I. Suppose that f ′ (x) lim = +∞. x→+∞ f (x) For the sequence un = ln f (n+1) f (n) , prove that un → +∞ (n → +∞). 47. (NLD 1) In a plane, three pairwise intersecting circles C1 ,C2 ,C3 with centers M1 , M2 , M3 are given. For i = 1, 2, 3, let Ai be one of the points of intersection of C j and Ck ({i, j, k} = {1, 2, 3}). Prove that if ∠M3 A1 M2 = ∠M1 A2 M3 = ∠M2 A3 M1 = π /3 (directed angles), then M1 A1 , M2 A2 , and M3 A3 are concurrent. 48. (NLD 2) Prove that in any parallelepiped the sum of the lengths of the edges is less than or equal to twice the sum of the lengths of the four diagonals. 49. (POL 1) Given positive integers k, m, n with km ≤ n and nonnegative real numbers x1 , . . . , xk , prove that ! k n ∏ xmi − 1 i=1 k ≤ m ∑ (xni − 1). i=1 50. (POL 2) (SL83-14). 51. (POL 3) (SL83-15). 52. (ROU 1) (SL83-19). 53. (ROU 2) Let a ∈ R and let z1 , z2 , . . . , zn be complex numbers of modulus 1 satisfying the relation 3.24 IMO 1983 n n k=1 k=1 153 ∑ z3k = 4(a + (a − n)i) − 3 ∑ zk . Prove that a ∈ {0, 1, . . . , n} and zk ∈ {1, i} for all k. 54. (ROU 3) (SL83-20). 55. (ROU 4) For every a ∈ N denote by M(a) the number of elements of the set {b ∈ N | a + b is a divisor of ab}. Find maxa≤1983 M(a). 56. (ROU 5) Consider the expansion (1 + x + x2 + x3 + x4 )496 = a0 + a1 x + · · · + a1984 x1984 . (a) Determine the greatest common divisor of the coefficients a3 , a8 , a13 , . . . , a1983 . (b) Prove that 10340 < a992 < 10347. 57. (ESP 1) In the system of base n2 + 1 find a number N with n different digits such that: (i) N is a multiple of n. Let N = nN ′ . (ii) The number N and N ′ have the same number n of different digits in base n2 + 1, none of them being zero. (iii) If s(C) denotes the number in base n2 + 1 obtained by applying the permutation s to the n digits of the number C, then for each permutation s, s(N) = ns(N ′ ). 58. (ESP 2) (SL83-8). 59. (ESP 3) Solve the equation tan2 (2x) + 2 tan(2x) · tan(3x) − 1 = 0. 60. (SWE 1) (SL83-21). 61. (SWE 2) Let a and b be integers. Is it possible to find integers p and q such that the integers p + na and q + nb have no common prime factor no matter how the integer n is chosen. 62. (SWE 3) A circle γ is drawn and let AB be a diameter. The point C on γ is the midpoint of the line segment BD. The line segments AC and DO, where O is the center of γ , intersect at P. Prove that there is a point E on AB such that P is on the circle with diameter AE. 63. (SWE 4) (SL83-22). 64. (USA 1) The sum of all the face angles about all of the vertices except one of a given polyhedron is 5160. Find the sum of all of the face angles of the polyhedron. 154 3 Problems 65. (USA 2) Let ABCD be a convex quadrilateral whose diagonals AC and BD intersect in a point P. Prove that AP cot ∠BAC + cot∠DAC = . PC cot ∠BCA + cot∠DCA 66. (USA 3) (SL83-9). 67. (USA 4) The altitude from a vertex of a given tetrahedron intersects the opposite face in its orthocenter. Prove that all four altitudes of the tetrahedron are concurrent. 68. (USA 5) Three of the roots of the equation x4 − px3 + qx2 − rx + s = 0 are tan A, tan B, and tanC, where A, B, and C are angles of a triangle. Determine the fourth root as a function only of p, q, r, and s. 69. (USS 1) (SL83-23). 70. (USS 2) (SL83-24). 71. (USS 3) (SL83-25). 72. (USS 4) Prove that for all x1 , x2 , . . . , xn ∈ R the following inequality holds: ∑ n≥i> j≥1 cos2 (xi − x j ) ≥ n(n − 2) . 4 73. (VNM 1) Let ABC be a nonequilateral triangle. Prove that there exist two points P and Q in the plane of the triangle, one in the interior and one in the exterior of the circumcircle of ABC, such that the orthogonal projections of any of these two points on the sides of the triangle are vertices of an equilateral triangle. 74. (VNM 2) In a plane we are given two distinct points A, B and two lines a, b passing through B and A respectively (a ∋ B, b ∋ A) such that the line AB is equally inclined to a and b. Find the locus of points M in the plane such that the product of distances from M to A and a equals the product of distances from M to B and b (i.e., MA · MA′ = MB · MB′ , where A′ and B′ are the feet of the perpendiculars from M to a and b respectively). 75. (VNM 3) Find the sum of the fiftieth powers of all sides and diagonals of a regular 100-gon inscribed in a circle of radius R. 3.24.3 Shortlisted Problems 1. (AUS 1) The localities P1 , P2 , . . . , P1983 are served by ten international airlines A1 , A2 , . . . , A10 . It is noticed that there is direct service (without stops) between any two of these localities and that all airline schedules offer round-trip flights. Prove that at least one of the airlines can offer a round trip with an odd number of landings. 3.24 IMO 1983 155 2. (BEL 1) Let n be a positive integer. Let σ (n) be the sum of the natural divisors d of n (including 1 and n). We say that an integer m ≥ 1 is superabundant (P.Erdös, σ (k) 1944) if ∀k ∈ {1, 2, . . . , m − 1}, σ (m) m > k . Prove that there exists an infinity of superabundant numbers. 3. (BEL 3)IMO4 We say that a set E of points of the Euclidian plane is “Pythagorean” if for any partition of E into two sets A and B, at least one of the sets contains the vertices of a right-angled triangle. Decide whether the following sets are Pythagorean: (a) a circle; (b) an equilateral triangle (that is, the set of three vertices and the points of the three edges). 4. (BEL 5) On the sides of the triangle ABC, three similar isosceles triangles ABP (AP = PB), AQC (AQ = QC), and BRC (BR = RC) are constructed. The first two are constructed externally to the triangle ABC, but the third is placed in the same half-plane determined by the line BC as the triangle ABC. Prove that APRQ is a parallelogram. 5. (BRA 1) Consider the set of all strictly decreasing sequences of n natural numbers having the property that in each sequence no term divides any other term of the sequence. Let A = (a j ) and B = (b j ) be any two such sequences. We say that A precedes B if for some k, ak < bk and ai = bi for i < k. Find the terms of the first sequence of the set under this ordering. 6. (CAN 2) Suppose that {x1 , x2 , . . . , xn } are positive integers for which x1 + x2 + · · · + xn = 2(n + 1). Show that there exists an integer r with 0 ≤ r ≤ n − 1 for which the following n − 1 inequalities hold: xr+1 + · · · + xr+i ≤ 2i + 1 xr+1 + · · · + xn + x1 + · · · + xi ≤ 2(n − r + i) + 1 ∀i, 1 ≤ i ≤ n − r; ∀i, 1 ≤ i ≤ r − 1. Prove that if all the inequalities are strict, then r is unique and that otherwise there are exactly two such r. 7. (CAN 5) Let a be a positive integer and let {an } be defined by a0 = 0 and p an+1 = (an + 1)a + (a + 1)an + 2 a(a + 1)an(an + 1) (n = 1, 2 . . . ). Show that for each positive integer n, an is a positive integer. 8. (ESP 2) In a test, 3n students participate, who are located in three rows of n students in each. The students leave the test room one by one. If N1 (t), N2 (t), N3 (t) denote the numbers of students in the first, second, and third row respectively at time t, find the probability that for each t during the test, |Ni (t) − N j (t)| < 2, i 6= j, i, j = 1, 2, . . . . 9. (USA 3)IMO6 If a, b, and c are sides of a triangle, prove that 156 3 Problems a2 b(a − b) + b2c(b − c) + c2a(c − a) ≥ 0. Determine when there is equality. 10. (FIN 1) Let p and q be integers. Show that there exists an interval I of length 1/q and a polynomial P with integral coefficients such that P(x) − p 1 < 2 q q for all x ∈ I. 11. (FIN 2′ ) Let f : [0, 1] → R be continuous and satisfy: b f (2x) = f (x), f (x) = b + (1 − b) f (2x − 1), where b = 1+c 2+c , 0 ≤ x ≤ 1/2; 1/2 ≤ x ≤ 1, c > 0. Show that 0 < f (x) − x < c for every x, 0 < x < 1. 12. (UNK 4)IMO1 Find all functions f defined on the positive real numbers and taking positive real values that satisfy the following conditions: (i) f (x f (y)) = y f (x) for all positive real x, y. (ii) f (x) → 0 as x → +∞. 13. (LUX 2) Let E be the set of 19833 points of the space R3 all three of whose coordinates are integers between 0 and 1982 (including 0 and 1982). A coloring of E is a map from E to the set {red, blue}. How many colorings of E are there satisfying the following property: The number of red vertices among the 8 vertices of any right-angled parallelepiped is a multiple of 4? 14. (POL 2)IMO5 Prove or disprove: From the interval [1, . . . , 30000] one can select a set of 1000 integers containing no arithmetic triple (three consecutive numbers of an arithmetic progression). 15. (POL 3) Decide whether there exists a set M of natural numbers satisfying the following conditions: (i) For any natural number m > 1 there are a, b ∈ M such that a + b = m. (ii) If a, b, c, d ∈ M, a, b, c, d > 10 and a + b = c + d, then a = c or a = d. 16. (GDR 1) Let F(n) be the set of polynomials P(x) = a0 + a1 x + · · · + an xn , with a0 , a1 , . . . , an ∈ R and 0 ≤ a0 = an ≤ a1 = an−1 ≤ · · · ≤ a[n/2] = a[(n+1)/2]. Prove that if f ∈ F(m) and g ∈ F(n), then f g ∈ F(m + n). 17. (GDR 3) Let P1 , P2 , . . . , Pn be distinct points of the plane, n ≥ 2. Prove that √ 3 √ max Pi Pj > ( n − 1) min Pi Pj . 1≤i< j≤n 1≤i< j≤n 2 18. (FRG 3)IMO3 Let a, b, c be positive integers satisfying (a, b) = (b, c) = (c, a) = 1. Show that 2abc − ab − bc − ca is the largest integer not representable as xbc + yca + zab with nonnegative integers x, y, z. 3.24 IMO 1983 157 19. (ROU 1) Let (Fn )n≥1 be the Fibonacci sequence F1 = F2 = 1, Fn+2 = Fn+1 + Fn (n ≥ 1), and P(x) the polynomial of degree 990 satisfying P(k) = Fk , for k = 992, . . ., 1982. Prove that P(1983) = F1983 − 1. 20. (ROU 3) Solve the system of equations x1 |x1 | = x2 |x2 | + (x1 − a)|x1 − a|, x2 |x2 | = x3 |x3 | + (x2 − a)|x2 − a|, ··· xn |xn | = x1 |x1 | + (xn − a)|xn − a|, in the set of real numbers, where a > 0. 1983 21. (SWE 1) Find the greatest integer less than or equal to ∑2k=1 k1/1983−1 . 22. (SWE 4) Let n be a positive integer having at least two different prime factors. Show that there exists a permutation a1 , a2 , . . . , an of the integers 1, 2, . . . , n such that n 2π ak ∑ k · cos n = 0. k=1 23. (USS 1)IMO2 Let K be one of the two intersection points of the circles W1 and W2 . Let O1 and O2 be the centers of W1 and W2 . The two common tangents to the circles meet W1 and W2 respectively in P1 and P2 , the first tangent, and Q1 and Q2 , the second tangent. Let M1 and M2 be the midpoints of P1 Q1 and P2 Q2 , respectively. Prove that ∠O1 KO2 = ∠M1 KM2 . 24. (USS 2) Let dn be the last nonzero digit of the decimal representation of n!. Prove that dn is aperiodic; that is, there do not exist T and n0 such that for all n ≥ n0 , dn+T = dn . 25. (USS 3) Prove that every partition of 3-dimensional space into three disjoint subsets has the following property: One of these subsets contains all possible distances; i.e., for every a ∈ R+ , there are points M and N inside that subset such that distance between M and N is exactly a. 158 3 Problems 3.25 The Twenty-Fifth IMO Prague, Czechoslovakia, June 29–July 10, 1984 3.25.1 Contest Problems First Day (July 4) 1. Let x, y, z be nonnegative real numbers with x + y + z = 1. Show that 0 ≤ xy + yz + zx − 2xyz ≤ 7 . 27 2. Find two positive integers a, b such that none of the numbers a, b, a + b is divisible by 7 and (a + b)7 − a7 − b7 is divisible by 77 . 3. In a plane two different points O and A are given. For each point X 6= O of the plane denote by α (X) the angle AOX measured in radians (0 ≤ α (X ) < 2π ) and (X) by C(X ) the circle with center O and radius OX + αOX . Suppose each point of the plane is colored by one of a finite number of colors. Show that there exists a point X with α (X ) > 0 such that its color appears somewhere on the circle C(X ). Second Day (July 5) 4. Let ABCD be a convex quadrilateral for which the circle of diameter AB is tangent to the line CD. Show that the circle of diameter CD is tangent to the line AB if and only if the lines BC and AD are parallel. 5. Let d be the sum of the lengths of all diagonals of a convex polygon of n (n > 3) vertices, and let p be its perimeter. Prove that     n−3 d 1 hni n+1 < < −2 . 2 p 2 2 2 6. Let a, b, c, d be odd positive integers such that a < b < c < d, ad = bc, and a + d = 2k , b + c = 2m for some integers k and m. Prove that a = 1. 3.25.2 Longlisted Problems 3 1. (AUS 1) The fraction 10 can be written as the sum of two positive fractions with 3 1 3 1 numerator 1 as follows: 10 = 15 + 10 and also 10 = 14 + 20 . There are the only two ways in which this can be done. 3 In how many ways can 1984 be written as the sum of two positive fractions with numerator 1? Is there a positive integer n, not divisible by 3, such that 3n can be written as the sum of two positive fractions with numerator 1 in exactly 1984 ways? 2. (AUS 2) Given a regular convex 2m-sided polygon P, show that there is a 2msided polygon π with the same vertices as P (but in different order) such that π has exactly one pair of parallel sides. 3.25 IMO 1984 159 3. (AUS 3) The opposite sides of the reentrant hexagon AFBDCE intersect at the points K, L, M (as shown in the figure). It is given that AL = AM = a, BM = BK = b, CK = CL = c, LD = DM = d, ME = EK = e, FK = FL = f . (a) Given length a and the three angles α , β , and γ at the vertices A, B, and C, respectively, satisfying the condition α + β + γ < 180◦ , show that all the angles and sides of the hexagon are thereby uniquely determined. A (b) Prove that D  D 1 1 1 1 + = + .  a e b d DD #eD Easier version of (b). Prove that L # DM # e D #  (a + f )(b + d)(c + e) D Ee P  # F P  P PP e = (a + e)(b + f )(c + d). # K PP C # eB 4. (BEL 1) Given a triangle ABC, three equilateral triangles AEB, BFC, and CGA are constructed in the exterior of ABC. Prove that: (a) CE = AF = BG; (b) CE, AF, and BG have a common point. 5. (BEL 2) For a real number x, let [x] denote the greatest integer not exceeding x. If m ≥ 3, prove that     m(m + 1) m+1 = . 2(2m − 1) 4 6. (BEL 3) Let P, Q, R be the polynomials with real or complex coefficients such that at least one of them is not constant. If Pn + Qn + Rn = 0, prove that n < 3.  7. (BGR 1) Prove that for any natural number n, the number 2n n divides the least common multiple of the numbers 1, 2, . . . , 2n − 1, 2n. 8. (BGR 2) In the plane of a given triangle A1 A2 A3 determine (with proof) a straight line l such that the sum of the distances from A1 , A2 , and A3 to l is the least possible. 9. (BGR 3) The circle inscribed in the triangle A1 A2 A3 is tangent to its sides A1 A2 , A2 A3 , A3 A1 at points T1 , T2 , T3 , respectively. Denote by M1 , M2 , M3 the midpoints of the segments A2 A3 , A3 A1 , A1 A2 , respectively. Prove that the perpendiculars through the points M1 , M2 , M3 to the lines T2 T3 , T3 T1 , T1 T2 meet at one point. 10. (BGR 4) Assume that the bisecting plane of the dihedral angle at edge AB of the tetrahedron ABCD meets the edge CD at point E. Denote by S1 , S2 , S3 , respectively the areas of the triangles ABC, ABE, and ABD. Prove that no tetrahedron exists for which S1 , S2 , S3 (in this order) form an arithmetic or geometric progression. 11. (BGR 5) (SL84-13). 160 3 Problems 12. (CAN 1) (SL84-11). Original formulation. Suppose that a1 , a2 , . . . , a2n are distinct integers such that (x − a1 )(x − a2 ) · · · (x − a2n) + (−1)n−1 (n!)2 = 0 has an integer solution r. Show that r = a1 +a2 +···+a2n . 2n 13. (CAN 2) (SL84-2). Original formulation. Let m, n be nonzero integers. Show that 4mn − m − n can be a square infinitely many times, but that this never happens when either m or n is positive. Alternative formulation. Let m, n be positive integers. Show that 4mn − m − n can be 1 less than a perfect square infinitely often, but can never be a square. 14. (CAN 3) (SL84-6). 15. (CAN 4) Consider all the sums of the form 1985 ∑ ek k5 = ±15 ± 25 ± · · · ± 19855, k=1 where ek = ±1. What is the smallest nonnegative value attained by a sum of this type? 16. (CAN 5) (SL84-19). Original formulation. The triangular array (an,k ) of numbers is given by an,1 = 1/n, for n = 1, 2, . . . , an,k+1 = an−1,k − an,k , for 1 ≤ k ≤ n − 1. Find the harmonic mean of the 1985th row. 17. (FRA 1) (SL84-1). 18. (FRA 2) Let c be the inscribed circle of the triangle ABC, d a line tangent to c which does not pass through the vertices of triangle ABC. Prove the existence of points A1 , B1 ,C1 , respectively, on the lines BC,CA, AB satisfying the following two properties: (i) Lines AA1 , BB1 , and CC1 are parallel. (ii) Lines AA1 , BB1 , and CC1 meet d respectively at points A′ , B′ , and C′ such that A′ A1 B′ B1 C′C1 = ′ = ′ . A′ A BB CC 19. (FRA 3) Let ABC be an isosceles triangle with right angle at point A. Find the minimum of the function F given by √ F(M) = BM + CM − 3AM. 20. (FRG 1) (SL84-5). 21. (FRG 2) (1) Start with a white balls and b black balls. (2) Draw one ball at random. 3.25 IMO 1984 161 (3) If the ball is white, then stop. Otherwise, add two black balls and go to step 2. Let S be the number of draws before the process terminates. For the cases a = b = 1 and a = b = 2 only, find an = P(S = n), bn = P(S ≤ n), limn→∞ bn , and the expectation value of the number of balls drawn: E(S) = ∑n≥1 nan . 22. (FRG 3) (SL84-17). Original formulation. In a permutation (x1 , x2 , . . . , xn ) of the set 1, 2, . . . , n we call a pair (xi , x j ) discordant if i < j and xi > x j . Let d(n, k) be the number of such permutations with exactly k discordant pairs. (a) Find d(n, 2). (b) Show that d(n, k) = d(n, k − 1) + d(n − 1, k) − d(n − 1, k − 1) with d(n, k) = 0 for k < 0 and d(n, 0) = 1 for n ≥ 1. Compute with this recursion a table of d(n, k) for n = 1 to 6. 23. (FRG 4) A 2× 2 ×12 box fixed in space is to be filled with twenty-four 1 ×1 × 2 bricks. In how many ways can this be done? 24. (FRG 5) (SL84-7). Original formulation. Consider several types of 4-cell figures: (a) (b) (c) (d) (e) . Find, with proof, for which of these types of figures it is not possible to number the fields of the 8 × 8 chessboard using the numbers 1, 2, . . . , 64 in such a way that the sum of the four numbers in each of its parts congruent to the given figure is divisible by 4. 25. (UNK 1) (SL84-10). 26. (UNK 2) A cylindrical container has height 6 cm and radius 4 cm. It rests on a circular hoop, also of radius 4 cm, fixed in a horizontal plane with its axis vertical and with each circular rim of the cylinder touching the hoop at two points. The cylinder is now moved so that each of its circular rims still touches the hoop in two points. Find with proof the locus of one of the cylinder’s vertical ends. 27. (UNK 3) The function f (n) is defined on the nonnegative integers n by: f (0) = 0, f (1) = 1,     1 1 f (n) = f n − m(m − 1) − f m(m + 1) − n , 2 2 for 12 m(m − 1) < n ≤ 12 m(m + 1), m ≥ 2. Find the smallest integer n for which f (n) = 5. 28. (UNK 4) A “number triangle” (tnk ) (0 ≤ k ≤ n) is defined by tn,0 = tn,n = 1 (n ≥ 0), 162 3 Problems   √ m √ n−m+1 tn+1,m = 2 − 3 tn,m + 2 + 3 tn,m−1 (1 ≤ m ≤ n). Prove that all tn,m are integers. 29. (GDR 1) Let Sn = {1, . . . , n} and let f be a function that maps every subset of Sn into a positive real number and satisfies the following condition: For all A ⊆ Sn and x, y ∈ Sn , x 6= y, f (A ∪ {x}) f (A ∪ {y}) ≤ f (A ∪ {x, y}) f (A). Prove that for all A, B ⊆ Sn the following inequality holds: f (A) · f (B) ≤ f (A ∪ B) · f (A ∩ B). 30. (GDR 2) Decide whether it is possible to color the 1984 natural numbers 1, 2, 3, . . . , 1984 using 15 colors so that no geometric sequence of length 3 of the same color exists. 31. (LUX 1) Let f 1 (x) = x3 + a1 x2 + b1 x + c1 = 0 be an equation with three positive roots α > β > γ > 0. From the equation f1 (x) = 0 one constructs the equation f2 (x) = x3 + a2 x2 + b2 x + c2 = x(x + b1 )2 − (a1 x + c1 )2 = 0. Continuing this process, we get equations f3 , . . . , fn . Prove that √ n−1 lim 2 −an = α . n→∞ 32. (LUX 2) (SL84-15). 33. (MNG 1) (SL84-4). 34. (MNG 2) One country has n cities and every two of them are linked by a railroad. A railway worker should travel by train exactly once through the entire railroad system (reaching each city exactly once). If it is impossible for worker to travel by train between two cities, he can travel by plane. What is the minimal number of flights that the worker will have to use? 35. (MNG 3) Prove that there exist distinct natural numbers m1 , m2 , . . . , mk satisfying the conditions   1 1 1 π −1984 < 25 − + + ···+ < π −1960 m1 m2 mk where π is the ratio between circle and its diameter. 36. (MNG 4) The set {1, 2, . . . , 49} is divided into three subsets. Prove that at least one of these subsets contains three different numbers a, b, c such that a + b = c. 37. (MAR 1) Denote by [x] the greatest integer not exceeding x. For all real k > 1, define two sequences:   nk an (k) = [nk] and bn (k) = . k−1 If A(k) = {an (k) : n ∈ N} and B(k) = {bn (k) : n ∈ N}, prove that A(k) and B(k) form a partition of N if and only if k is irrational. 3.25 IMO 1984 163 38. (MAR 2) Determine all continuous functions f such that  ∀(x, y) ∈ R2 f (x + y) f (x − y) = ( f (x) f (y))2 . 39. (MAR 3) Let ABC be an isosceles triangle, AB = AC, ∠A = 20◦ . Let D be a point on AB, and E a point on AC such that ∠ACD = 20◦ and ∠ABE = 30◦ . What is the measure of the angle ∠CDE? 40. (NLD 1) (SL84-12). 41. (NLD 2) Determine positive integers p, q, and r such that the diagonal of a block consisting of p × q × r unit cubes passes through exactly 1984 of the unit cubes, while its length is minimal. (The diagonal is said to pass through a unit cube if it has more than one point in common with the unit cube.) 42. (NLD 3) Triangle ABC is given for which BC = AC + 12 AB. The point P divides AB such that RP : PA = 1 : 3. Prove that ∠CAP = 2∠CPA. 43. (POL 1) (SL84-16). 44. (POL 2) (SL84-9). 45. (POL 3) Let X be an arbitrary nonempty set contained in the plane and let sets A1 , A2 , . . . , Am and B1 , B2 , . . . , Bn be its images under parallel translations. Let us suppose that A 1 ∪ A2 ∪ · · · ∪ Am ⊂ B1 ∪ B2 ∪ · · · ∪ Bn and that the sets A1 , A2 , . . . , Am are disjoint. Prove that m ≤ n. 46. (ROU 1) Let (an )n≥1 and (bn )n≥1 be two sequences of natural numbers such that an+1 = nan + 1, bn+1 = nbn − 1 for every n ≥ 1. Show that these two sequences can have only a finite number of terms in common. 47. (ROU 2) (SL84-8). 48. (ROU 3) Let ABC be a triangle with interior angle bisectors AA1 , BB1 , CC1 and incenter I. If σ [IA1 B] + σ [IB1C] + σ [IC1 A] = 12 σ [ABC], where σ [ABC] denotes the area of ABC, show that ABC is isosceles. 49. (ROU 4) Let n > 1 and xi ∈ R for i = 1, . . . , n. Set Sk = xk1 + xk2 + · · · + xkn for k ≥ 1. If S1 = S2 = · · · = Sn+1 , show that xi ∈ {0, 1} for every i = 1, 2, . . . , n. 50. (ROU 5) (SL84-14). 51. (ESP 1) Two cyclists leave simultaneously a point P in a circular runway with constant velocities v1 , v2 (v1 > v2 ) and in the same sense. A pedestrian leaves +v2 P at the same time, moving with velocity v3 = v112 . If the pedestrian and the cyclists move in opposite directions, the pedestrian meets the second cyclist 91 seconds after he meets the first. If the pedestrian moves in the same direction as the cyclists, the first cyclist overtakes him 187 seconds before the second does. Find the point where the first cyclist overtakes the second cyclist the first time. 52. (ESP 2) Construct a scalene triangle such that 164 3 Problems a(tan B − tanC) = b(tan A − tanC). 53. (ESP 3) Find a sequence of natural numbers ai such that ai = ∑i+4 r=1 dr , where dr 6= ds for r 6= s and dr divides ai . 54. (ESP 4) Let P be a convex planar polygon with equal angles. Let l1 , . . . , ln be its sides. Show that a necessary and sufficient condition for P to be regular is that li the sum of the ratios li+1 (i = 1, . . . , n; ln+1 = l1 ) equals the number of sides. 55. (ESP 5) Let a, b, c be natural numbers such that a + b + c = 2pq(p30 − q30 ), p > q being two given positive integers. (a) Prove that k = a3 + b3 + c3 is not a prime number. (b) Prove that if a · b · c is maximum, then 1984 divides k. 56. (SWE 1) Let a, b, c be nonnegative integers such that a ≤ b ≤ c, 2b 6= a + c and a+b+c is an integer. Is it possible to find three nonnegative integers d, e, and f 3 such that d ≤ e ≤ f , f 6= c, and such that a2 + b2 + c2 = d 2 + e2 + f 2 ? 57. (SWE 2) Let a, b, c, d be a permutation of the numbers 1, 9, 8, 4 and let n = (10a + b)10c+d . Find the probability that 1984! is divisible by n. 58. (SWE 3) Let (an )∞ 1 be a sequence such that an ≤ an+m ≤ an + am for all positive integers n and m. Prove that ann has a limit as n approaches infinity. 59. (USA 1) Determine the smallest positive integer m such that 529n + m · 132n is divisible by 262417 for all odd positive integers n. 60. (USA 2) (SL84-20). 61. (USA 3) A fair coin is tossed repeatedly until there is a run of an odd number of heads followed by a tail. Determine the expected number of tosses. 62. (USA 4) From a point P exterior to a circle K, two rays are drawn intersecting K in the respective pairs of points A, A′ and B, B′ . For any other pair of points C,C′ on K, let D be the point of intersection of the circumcircles of triangles PAC and PB′C′ other than point P. Similarly, let D′ be the point of intersection of the circumcircles of triangles PA′C′ and PBC other than point P. Prove that the points P, D, and D′ are collinear. 63. (USA 5) (SL84-18). 64. (USS 1) For a matrix (pi j ) of the format m × n with real entries, set n ai = ∑ pi j j=1 m for i = 1, . . . , m and b j = ∑ pi j for j = 1, . . . , n. (1) i=1 By integering a real number we mean replacing the number with the integer closest to it. Prove that integering the numbers ai , b j , pi j can be done in such a way that (1) still holds. 3.25 IMO 1984 165 65. (USS 2) A tetrahedron is inscribed in a sphere of radius 1 such that the center of the sphere is inside the tetrahedron. Prove that the sum of lengths of all edges of the tetrahedron is greater than 6. 66. (USS 3) (SL84-3). Original formulation. All the divisors of a positive integer n arranged in increasing order are x1 < x2 < · · · < xk . Find all such numbers n for which x25 + x26 − 1 = n. 67. (USS 4) With the medians of an acute-angled triangle another triangle is constructed. If R and Rm are the radii of the circles circumscribed about the first and the second triangle, respectively, prove that 5 Rm > R. 6 68. (USS 5) In the Martian language every finite sequence of letters of the Latin alphabet letters is a word. The publisher “Martian Words” makes a collection of all words in many volumes. In the first volume there are only one-letter words, in the second, two-letter words, etc., and the numeration of the words in each of the volumes continues the numeration of the previous volume. Find the word whose numeration is equal to the sum of numerations of the words Prague, Olympiad, Mathematics. 3.25.3 Shortlisted Problems 1. (FRA 1) Find all solutions of the following system of n equations in n variables: x1 |x1 | − (x1 − a)|x1 − a| = x2 |x2 |, x2 |x2 | − (x2 − a)|x2 − a| = x3 |x3 |, ··· xn |xn | − (xn − a)|xn − a| = x1 |x1 |, where a is a given number. 2. (CAN 2) Prove: (a) There are infinitely many triples of positive integers m, n, p such that 4mn − m − n = p2 − 1. (b) There are no positive integers m, n, p such that 4mn − m − n = p2 . 3. (USS 3) Find all positive integers n such that n = d62 + d72 − 1, where 1 = d1 < d2 < · · · < dk = n are all positive divisors of the number n. 4. (MNG 1)IMO5 Let d be the sum of the lengths of all diagonals of a convex polygon of n (n > 3) vertices and let p be its perimeter. Prove that     n−3 d 1 hni n+1 < < −2 . 2 p 2 2 2 166 3 Problems 5. (FRG 1)IMO1 Let x, y, z be nonnegative real numbers with x + y + z = 1. Show that 7 0 ≤ xy + yz + zx − 2xyz ≤ . 27 6. (CAN 3) Let c be a positive integer. The sequence { fn } is defined as follows: f1 = 1, f2 = c, fn+1 = 2 fn − fn−1 + 2 (n ≥ 2). Show that for each k ∈ N there exists r ∈ N such that fk fk+1 = fr . 7. (FRG 5) (a) Decide whether the fields of the 8 × 8 chessboard can be numbered by the numbers 1, 2, . . . , 64 in such a way that the sum of the four numbers in each of its parts of one of the forms is divisible by four. (b) Solve the analogous problem for 8. (ROU 2)IMO3 In a plane two different points O and A are given. For each point X 6= O of the plane denote by α (X ) the angle AOX measured in radians (0 ≤ (X) α (X ) < 2π ) and by C(X) the circle with center O and radius OX + αOX . Suppose each point of the plane is colored by one of a finite number of colors. Show that there exists a point X with α (X ) > 0 such that its color appears somewhere on the circle C(X ). √ √ √ √ 9. (POL 2) Let a, b, c be positive numbers with a + b + c = 23 . Prove that the system of equations √ √ √y − a + √ z − a = 1, √z − b + √x − b = 1, x − c + y − c = 1, has exactly one solution (x, y, z) in real numbers. 10. (UNK 1) Prove that the product of five consecutive positive integers cannot be the square of an integer. 11. (CAN 1) Let n be a natural number and a1 , a2 , . . . , a2n mutually distinct integers. Find all integers x satisfying (x − a1 ) · (x − a2 ) · · · (x − a2n) = (−1)n (n!)2 . 3.25 IMO 1984 167 12. (NLD 1)IMO2 Find two positive integers a, b such that none of the numbers a, b, a + b is divisible by 7 and (a + b)7 − a7 − b7 is divisible by 77 . 13. (BGR 5) Prove that the volume of a tetrahedron inscribed in a right circular cylinder of volume 1 does not exceed 32π . 14. (ROU 5)IMO4 Let ABCD be a convex quadrilateral for which the circle with diameter AB is tangent to the line CD. Show that the circle with diameter CD is tangent to the line AB if and only if the lines BC and AD are parallel. 15. (LUX 2) Angles of a given triangle ABC are all smaller than 120◦ . Equilateral triangles AFB, BDC and CEA are constructed in the exterior of △ABC. (a) Prove that the lines AD, BE, and CF pass through one point S. (b) Prove that SD + SE + SF = 2(SA + SB + SC). 16. (POL 1)IMO6 Let a, b, c, d be odd positive integers such that a < b < c < d, ad = bc, and a + d = 2k , b + c = 2m for some integers k and m. Prove that a = 1. 17. (FRG 3) In a permutation (x1 , x2 , . . . , xn ) of the set 1, 2, . . . , n we call a pair (xi , x j ) discordant if i < j and xi > x j . Let d(n, k) be the number of such permutations with exactly k discordant pairs. Find d(n, 2) and d(n, 3). 18. (USA 5) Inside triangle ABC there are three circles k1 , k2 , k3 each of which is tangent to two sides of the triangle and to its incircle k. The radii of k1 , k2 , k3 are 1, 4, and 9. Determine the radius of k. 19. (CAN 5) The triangular array (an,k ) of numbers is given by an,1 = 1/n, for n = 1, 2, . . ., an,k+1 = an−1,k − an,k , for 1 ≤ k ≤ n − 1. Prove that the geometric mean of the 1985th row is greater than 2−1984 . 20. (USA 2) Determine all pairs (a, b) of positive real numbers with a 6= 1 such that loga b < loga+1 (b + 1). 168 3 Problems 3.26 The Twenty-Sixth IMO Joutsa, Finland, June 29–July 11, 1985 3.26.1 Contest Problems First Day (July 4) 1. A circle whose center is on the side ED of the cyclic quadrilateral BCDE touches the other three sides. Prove that EB +CD = ED. 2. Each of the numbers in the set N = {1, 2, 3, . . .,n − 1}, where n ≥ 3, is colored with one of two colors, say red or black, so that: (i) i and n − i always receive the same color, and (ii) for some j ∈ N relatively prime to n, i and | j − i| receive the same color for all i ∈ N, i 6= j. Prove that all numbers in N must receive the same color. 3. The weight w(p) of a polynomial p, p(x) = ∑ni=0 ai xi , with integer coefficients ai is defined as the number of its odd coefficients. For i = 0, 1, 2, . . . , let qi (x) = (1 + x)i . Prove that for any finite sequence 0 ≤ i1 < i2 < · · · < in the inequality w(qi1 + · · · + qin ) ≥ w(qi1 ) holds. Second Day (July 5) 4. Given a set M of 1985 positive integers, none of which has a prime divisor larger than 26, prove that M has four distinct elements whose geometric mean is an integer. 5. A circle with center O passes through points A and C and intersects the sides AB and BC of the triangle ABC at points K and N, respectively. The circumscribed circles of the triangles ABC and KBN intersect at two distinct points B and M. Prove that ∡OMB = 90◦ . 6. The sequence f1 , f2 , . . . , fn , . . . of functions is defined for x > 0 recursively by   1 f1 (x) = x, fn+1 (x) = fn (x) fn (x) + . n Prove that there exists one and only one positive number a such that 0 < fn (a) < fn+1 (a) < 1 for all integers n ≥ 1. 3.26.2 Longlisted Problems 1. (AUS 1) (SL85-4). 3.26 IMO 1985 169 2. (AUS 2) We are given a triangle ABC and three rectangles R1 , R2 , R3 with sides parallel to two fixed perpendicular directions and such that their union covers the sides AB, BC, and CA; i.e., each point on the perimeter of ABC is contained in or on at least one of the rectangles. Prove that all points inside the triangle are also covered by the union of R1 , R2 , R3 . 3. (AUS 3) A function f has the following property: If k > 1, j > 1, and (k, j) = m, then f (k j) = f (m) ( f (k/m) + f ( j/m)). What values can f (1984) and f (1985) take? 4. (BEL 1) Let x, y, and z be real numbers satisfying x + y + z = xyz. Prove that x(1 − y2 )(1 − z2 ) + y(1 − z2)(1 − x2 ) + z(1 − x2)(1 − y2) = 4xyz. 5. (BEL 2) (SL85-16). 6. (BEL 3) On a one-way street, an unending sequence of cars of width a, length b passes with velocity v. The cars are separated by the distance c. A pedestrian crosses the street perpendicularly with velocity w, without paying attention to the cars. (a) What is the probability that the pedestrian crosses the street uninjured? (b) Can he improve this probability by crossing the road in a direction other than perpendicular? 7. (BRA 1) A convex quadrilateral is inscribed in a circle of radius 1. Prove that the difference between its perimeter and the sum of the lengths of its diagonals is greater than zero and less than 2. 8. (BRA 2) Let K be a convex set in the xy-plane, symmetric with respect to the origin and having area greater than 4. Prove that there exists a point (m, n) 6= (0, 0) in K such that m and n are integers. 9. (BRA 3) (SL85-2). 10. (BGR 1) (SL85-13). 11. (BGR 2) Let a and b be integers and n a positive integer. Prove that bn−1 a(a + b)(a + 2b) · · ·(a + (n − 1)b) n! is an integer. 12. (CAN 1) Find the maximum value of sin2 θ1 + sin2 θ2 + · · · + sin2 θn subject to the restrictions 0 ≤ θi ≤ π , θ1 + θ2 + · · · + θn = π . 13. (CAN 2) Find the average of the quantity (a1 − a2)2 + (a2 − a3 )2 + · · · + (an−1 − an )2 taken over all permutations (a1 , a2 , . . . , an ) of (1, 2, . . . , n). 170 3 Problems 14. (CAN 3) Let k be a positive integer. Define u0 = 0, u1 = 1, and un = kun−1 − un−2 , n ≥ 2. Show that for each integer n, the number u31 + u32 + · · · + u3n is a multiple of u1 + u2 + · · · + un . 15. (CAN 4) Superchess is played on a 12 × 12 board, and it uses superknights, which move between opposite corner cells of any 3 × 4 subboard. Is it possible for a superknight to visit every other cell of a superchessboard exactly once and return to its starting cell? 16. (CAN 5) (SL85-18). 17. (CUB 1) Set n An = k6 ∑ 2k . k=1 Find limn→∞ An . 18. (CYP 1) The circles (R, r) and (P, ρ ), where r > ρ , touch externally at A. Their direct common tangent touches (R, r) at B and (P, ρ ) at C. The line RP meets the circle (P, ρ ) again at D and the line BC at E. If |BC| = 6|DE|, prove that: (a) the lengths of the sides of the triangle RBE are in an arithmetic progression, and (b) |AB| = 2|AC|. 19. (CYP 2) Solve the system of simultaneous equations √ x − 1/y − 2w + 3z = 1, x + 1/y2 − 4w2 − 9z2 = 3, √ x x − 1/y3 − 8w3 + 27z3 = −5, x2 + 1/y4 − 16w4 − 81z4 = 15. 20. (CZS 1) Let T be the set of all lattice points (i.e., all points with integer coordinates) in three-dimensional space. Two such points (x, y, z) and (u, v, w) are called neighbors if |x − u| + |y − v| + |z − w| = 1. Show that there exists a subset S of T such that for each p ∈ T , there is exactly one point of S among p and its neighbors. 21. (CZS 2) Let A be a set of positive integers such that for any two elements x, y xy of A, |x − y| ≥ 25 . Prove that A contains at most nine elements. Give an example of such a set of nine elements. 22. (CZS 3) (SL85-7). 23. (CZS 4) Let N = {1, 2, 3, . . .}. For real x, y, set S(x, y) = {s | s = [nx +y], n ∈ N}. Prove that if r > 1 is a rational number, there exist real numbers u and v such that S(r, 0) ∩ S(u, v) = 0, / S(r, 0) ∪ S(u, v) = N. 24. (FRA 1) Let d ≥ 1 be an integer that is not the square of an integer. Prove that for every integer n ≥ 1, √ √ (n d + 1)| sin(nπ d)| ≥ 1. 3.26 IMO 1985 171 25. (FRA 2) Find eight positive integers n1 , n2 , . . . , n8 with the following property: For every integer k, −1985 ≤ k ≤ 1985, there are eight integers α1 , α2 , . . . , α8 , each belonging to the set {−1, 0, 1}, such that k = ∑8i=1 αi ni . 26. (FRA 3) (SL85-15). 27. (FRA 4) Let O be a point on the oriented Euclidean plane and (i, j) a directly oriented orthonormal basis. Let C be the circle of radius 1, centered at O. For every real number t and nonnegative integer n let Mn be the point on C for which −−→ −−→ hi, OMn i = cos 2nt (or OMn = cos 2nti + sin 2ntj). Let k ≥ 2 be an integer. Find all real numbers t ∈ [0, 2π ) that satisfy (i) M0 = Mk , and (ii) if one starts from M0 and goes once around C in the positive direction, one meets successively the points M0 , M1 , . . . , Mk−2 , Mk−1 , in this order. 28. (FRG 1) Let M be the set of the lengths of an octahedron whose sides are congruent quadrangles. Prove that M has at most three elements. (FRG 1a) Let an octahedron whose sides are congruent quadrangles be given. Prove that each of these quadrangles has two equal sides meeting at a common vertex. 29. (FRG 2) Call a four-digit number (xyzt)B in the number system with base B stable if (xyzt)B = (dcba)B − (abcd)B , where a ≤ b ≤ c ≤ d are the digits of (xyzt)B in ascending order. Determine all stable numbers in the number system with base B. (FRG 2a) The same problem with B = 1985. (FRG 2b) With assumptions as in FRG 2, determine the number of bases B ≤ 1985 such that there is a stable number with base B. 30. (UNK 1) A plane rectangular grid is given and a “rational point” is defined as a point (x, y) where x and y are both rational numbers. Let A, B, A′ , B′ be four ′ ′ ′ ′ distinct rational points. Let P be a point such that AABB = BBCP = PA PA . In other ′ ′ words, the triangles ABP, A B P are directly or oppositely similar. Prove that P is in general a rational point and find the exceptional positions of A′ and B′ relative to A and B such that there exists a P that is not a rational point. 31. (UNK 2) Let E1 , E2 , and E3 be three mutually intersecting ellipses, all in the same plane. Their foci are respectively F2 , F3 ; F3 , F1 ; and F1 , F2 . The three foci are not on a straight line. Prove that the common chords of each pair of ellipses are concurrent. 32. (UNK 3) A collection of 2n letters contains 2 each of n different letters. The collection is partitioned into n pairs, each pair containing 2 letters, which may be the same or different. Denote the number of distinct partitions by un . (Partitions differing in the order of the pairs in the partition or in the order of the two letters in the pairs are not considered distinct.) Prove that un+1 = (n + 1)un − n(n−1) 2 un−2 . (UNK 3a) A pack of n cards contains n pairs of 2 identical cards. It is shuffled and 2 cards are dealt to each of n different players. Let pn be the probability 172 3 Problems that every one of the n players is dealt two identical cards. Prove that n+1 pn − n(n−1) 2pn−2 . 1 pn+1 = 33. (UNK 4) (SL85-12). 34. (UNK 5) (SL85-20). 35. (GDR 1) We call a coloring f of the elements in the set M = {(x, y) | x = 0, 1, . . . , kn − 1; y = 0, 1, . . . , ln − 1} with n colors allowable if every color appears exactly k and l times in each row and column and there are no rectangles with sides parallel to the coordinate axes such that all the vertices in M have the same color. Prove that every allowable coloring f satisfies kl ≤ n(n + 1). 36. (GDR 2) Determine whether there exist 100 distinct lines in the plane having exactly 1985 distinct points of intersection. 37. (GDR 3) Prove that a triangle with angles α , β , γ , circumradius R, and area A satisfies α β γ 9R2 tan + tan + tan ≤ . 2 2 2 4A 38. (IRL 1) (SL85-21). 39. (IRL 2) Given a triangle ABC and external points X, Y , and Z such that ∡BAZ = ∡CAY , ∡CBX = ∡ABZ, and ∡ACY = ∡BCX, prove that AX, BY , and CZ are concurrent. 40. (IRL 3) Each of the numbers x1 , x2 , . . . , xn equals 1 or −1 and x1 x2 x3 x4 + x2 x3 x4 x5 + · · · + xn−3 xn−2 xn−1 xn +xn−2xn−1 xn x1 + xn−1 xn x1 x2 + xn x1 x2 x3 = 0. Prove that n is divisible by 4. 41. (IRL 4) (SL85-14). 42. (ISR 1) Prove that the product of two sides of a triangle is always greater than the product of the diameters of the inscribed circle and the circumscribed circle. 43. (ISR 2) Suppose that 1985 points are given inside a unit cube. Show that one can always choose 32 of them in such a way that every (possibly degenerate) √ closed polygon with these points as vertices has a total length of less than 8 3. 44. (ISR 3) (SL85-19). 45. (ITA 1) Two persons, X and Y , play with a die. X wins a game if the outcome is 1 or 2; Y wins in the other cases. A player wins a match if he wins two consecutive games. For each player determine the probability of winning a match within 5 games. Determine the probabilities of winning in an unlimited number of games. If X bets 1, how much must Y bet for the game to be fair? 46. (ITA 2) Let C be the curve determined by the equation y = x3 in the rectangular coordinate system. Let t be the tangent to C at a point P of C; t intersects C at 3.26 IMO 1985 173 another point Q. Find the equation of the set L of the midpoints M of PQ as P describes C. Is the correspondence associating P and M a bijection of C on L? Find a similarity that transforms C into L. 47. (ITA 3) Let F be the correspondence associating with every point P = (x, y) the point P′ = (x′ , y′ ) such that x′ = ax + b, y′ = ay + 2b. (1) Show that if a 6= 1, all lines PP′ are concurrent. Find the equation of the set of points corresponding to P = (1, 1) for b = a2 . Show that the composition of two mappings of type (1) is of the same type. 48. (ITA 4) In a given country, all inhabitants are knights or knaves. A knight never lies; a knave always lies. We meet three persons, A, B, and C. Person A says, “If C is a knight, B is a knave.” Person C says, “A and I are different; one is a knight and the other is a knave.” Who are the knights, and who are the knaves? 49. (MNG 1) (SL85-1). 50. (MNG 2) From each of the vertices of a regular n-gon a car starts to move with constant speed along the perimeter of the n-gon in the same direction. Prove that if all the cars end up at a vertex A at the same time, then they never again meet at any other vertex of the n-gon. Can they meet again at A? 51. (MNG 3) Let f1 = (a1 , a2 , . . . , an ), n > 2, be a sequence of integers. From f1 one constructs a sequence fk of sequences as follows: if fk = (c1 , c2 , . . . , cn ), then fk+1 = (ci1 , ci2 , ci3 + 1, ci4 + 1, . . . , cin + 1), where (ci1 , ci2 , . . . , cin ) is a permutation of (c1 , c2 , . . . , cn ). Give a necessary and sufficient condition for f1 under which it is possible for fk to be a constant sequence (b1 , b2 , . . . , bn ), b1 = b2 = · · · = bn , for some k. 52. (MNG 4) In the triangle ABC, let B1 be on AC, E on AB, G on BC, and let EG be parallel to AC. Furthermore, let EG be tangent to the inscribed circle of the triangle ABB1 and intersect BB1 at F. Let r, r1 , and r2 be the inradii of the triangles ABC, ABB1 , and BFG, respectively. Prove that r = r1 + r2 . 53. (MNG 5) For each P inside the triangle ABC, let A(P), B(P), and C(P) be the points of intersection of the lines AP, BP, and CP with the sides opposite to A, B, and C, respectively. Determine P in such a way that the area of the triangle A(P)B(P)C(P) is as large as possible. 54. (MAR 1) Set Sn = ∑np=1 (p5 + p7 ). Determine the greatest common divisor of Sn and S3n . 55. (MAR 2) The points A, B,C are in this order on line D, and AB = 4BC. Let M be a variable point on the perpendicular to D through C. Let MT1 and MT2 be tangents to the circle with center A and radius AB. Determine the locus of the orthocenter of the triangle MT1 T2 . 56. (MAR 3) Let ABCD be a rhombus with angle ∠A = 60◦ . Let E be a point, different from D, on the line AD. The lines CE and AB intersect at F. The lines 174 3 Problems DF and BE intersect at M. Determine the angle ∡BMD as a function of the position of E on AD. 57. (NLD 1) The solid S is defined as the intersection of the six spheres with the six edges of a regular tetrahedron T , with edge length 1, as diameters. Prove that S contains two points at a distance √1 . 6 (NLD 1a) Using the same assumptions, prove that no pair of points in S has a distance larger than √16 . 58. (NLD 2) Prove that there are infinitely many pairs (k, N) of positive integers such that 1 + 2 + · · · + k = (k + 1) + (k + 2) + · · ·+ N. 59. (NLD 3) (SL85-3). 60. (NOR 1) The sequence (sn ), where sn = ∑nk=1 sin k, n = 1, 2, . . . , is bounded. Find an upper and a lower bound. 61. (NOR 2) Consider the set A = {0, 1, 2, . . . , 9} and let (B1 , B2 , . . . , Bk ) be a collection of nonempty subsets of A such that Bi ∩ B j has at most two elements for i 6= j. What is the maximal value of k? 62. (NOR 3) A “large” circular disk is attached to a vertical wall. It rotates clockwise with one revolution per minute. An insect lands on the disk and immediately starts to climb vertically upward with constant speed π3 cm per second (relative to the disk). Describe the path of the insect (a) relative to the disk; (b) relative to the wall. 63. (POL 1) (SL85-6). 64. (POL 2) Let p be a prime. For which k can the set {1, 2, . . ., k} be partitioned into p subsets with equal sums of elements? 65. (POL 3) Define the functions f , F : N → N, by " √ # 3− 5 f (n) = n , F(k) = min{n ∈ N| f k (n) > 0}, 2 where f k = f ◦ · · · ◦ f is f iterated n times. Prove that F(k + 2) = 3F(k + 1) − F(k) for all k ∈ N. 66. (ROU 1) (SL85-5). 67. (ROU 2) Let k ≥ 2 and n1 , n2 , . . . , nk ≥ 1 be natural numbers having the property n2 | 2n1 − 1, n3 | 2n2 − 1, . . . , nk | 2nk−1 − 1, and n1 | 2nk − 1. Show that n1 = n2 = · · · = nk = 1. √ 68. (ROU 3) Show that the sequence {an }n≥1 defined by an = [n 2] contains an infinite number of integer powers of 2. ([x] is the integer part of x.) 69. (ROU 4) Let A and B be two finite disjoint sets of points in the plane such that no three distinct points in A ∪ B are collinear. Assume that at least one of the sets 3.26 IMO 1985 175 A, B contains at least five points. Show that there exists a triangle all of whose vertices are contained in A or in B that does not contain in its interior any point from the other set. 70. (ROU 5) Let C be a class of√functions f : N → N that contains the functions S(x) = x + 1 and E(x) = x − [ x]2 for every x ∈ N. ([x] is the integer part of x.) If C has the property that for every f , g ∈ C, f + g, f g, f ◦ g ∈ C, show that the function max( f (x) − g(x), 0) is in C for all f , g ∈ C. 71. (ROU 6) For every integer r > 1 find the smallest integer h(r) > 1 having the following property: For any partition of the set {1, 2, . . ., h(r)} into r classes, there exist integers a ≥ 0, 1 ≤ x ≤ y such that the numbers a + x, a + y, a + x + y are contained in the same class of the partition. 72. (ESP 1) Construct a triangle ABC given the side AB and the distance OH from the circumcenter O to the orthocenter H, assuming that OH and AB are parallel. 73. (ESP 2) Let A1 A2 , B1 B2 ,C1C2 be three equal segments on the three sides of an equilateral triangle. Prove that in the triangle formed by the lines B2C1 , C2 A1 , A2 B1 , the segments B2C1 , C2 A1 , A2 B1 are proportional to the sides in which they are contained. 74. (ESP 3) Find the triples of positive integers x, y, z satisfying 1 1 1 4 + + = . x y z 5 75. (ESP 4) Let ABCD be a rectangle, AB = a, BC = b. Consider the family of parallel and equidistant straight lines (the distance between two consecutive lines being d) that are at an angle φ , 0 ≤ φ ≤ 90◦ , with respect to AB. Let L be the sum of the lengths of all the segments intersecting the rectangle. Find: (a) how L varies, (b) a necessary and sufficient condition for L to be a constant, and (c) the value of this constant. 76. (SWE 1) Are there integers m and n such that 5m2 − 6mn + 7n2 = 1985? 77. (SWE 2) Two equilateral triangles are inscribed in a circle with radius r. Let A be the area √ of the set consisting of all points interior to both triangles. Prove that 2A ≥ r2 3. 78. (SWE 3) (SL85-17). 79. (SWE 4) Let a, b, and c be real numbers such that Prove that 1 1 1 + + = 0. 2 2 bc − a ca − b ab − c2 a b c + + = 0. 2 2 2 2 (bc − a ) (ca − b ) (ab − c2)2 176 3 Problems 80. (TUR 1) Let E = {1, 2, . . . , 16} and let M be the collection of all 4 × 4 matrices whose entries are distinct members of E. If a matrix A = (ai j )4×4 is chosen randomly from M, compute the probability p(k) of maxi min j ai j = k for k ∈ E. Furthermore, determine l ∈ E such that p(l) = max{p(k) | k ∈ E}. 81. (TUR 2) Given the side a and the corresponding altitude ha of a triangle ABC, find a relation between a and ha such that it is possible to construct, with straightedge and compass, triangle ABC such that the altitudes of ABC form a right triangle admitting ha as hypotenuse. 82. (TUR 3) Find all cubic polynomials x3 + ax2 + bx + c admitting the rational numbers a, b, and c as roots. 83. (TUR 4) Let Γi , i = 0, 1, 2, . . . , be a circle of radius ri inscribed in an angle of measure 2α such that each Γi is externally tangent to Γi+1 and ri+1 < ri . Show that the sum √ of the√areas of the circles Γi is equal to the area of a circle of radius r = 12 r0 ( sin α + csc α ). 84. (TUR 5) (SL85-8). 85. (USA 1) Let CD be a diameter of circle K. Let AB be a chord that is parallel to CD. The line segment AE, with E on K, is parallel to CB; F is the point of intersection of line segments AB and DE. The line segment FG, with G on DC, extended is parallel to CB. Is GA tangent to K at point A? 86. (USA 2) Let l denote the length of the smallest diagonal of all rectangles inscribed in a triangle T . (By inscribed, we mean that all four vertices of the rectl2 angle lie on the boundary of T .) Determine the maximum value of S(T taken ) over all triangles (S(T ) denotes the area of triangle T ). 87. (USA 3) (SL85-9). 88. (USA 4) Determine the range of w(w + x)(w + y)(w + z), where x, y, z, and w are real numbers such that x + y + z + w = x7 + y7 + z7 + w7 = 0. 89. (USA 5) Given that n elements a1 , a2 , . . ., an are organized into n pairs P1 , P2 , . . ., Pn in such a way that two pairs Pi , Pj share exactly one element when (ai , a j ) is one of the pairs, prove that every element is in exactly two of the pairs. 90. (USS 1) Decompose the number 51985 − 1 into a product of three integers, each of which is larger than 5100 . 91. (USS 2) Thirty-four countries participated in a jury session of the IMO, each represented by the leader and the deputy leader of the team. Before the meeting, some participants exchanged handshakes, but no team leader shook hands with his deputy. After the meeting, the leader of the Illyrian team asked every other participant the number of people they had shaken hands with, and all the answers she got were different. How many people did the deputy leader of the Illyrian team greet? 3.26 IMO 1985 177 92. (USS 3) (SL85-11). (USS 3a) Given six numbers, find a method of computing by using not more than 15 additions and 14 multiplications the following five numbers: the sum of the numbers, the sum of products of the numbers taken two at a time, and the sums of the products of the numbers taken three, four, and five at a time. 93. (USS 4) The sphere inscribed in tetrahedron ABCD touches the sides ABD and DBC at points K and M, respectively. Prove that ∡AKB = ∡DMC. 94. (USS 5) (SL85-22). 95. (VNM 1) (SL85-10). (VNM 1a) Prove that for each point M on the edges of a regular tetrahedron there is one and only one point M ′ on the surface of the tetrahedron such that there are at least three curves joining M and M ′ on the surface of the tetrahedron of minimal length among all curves joining M and M ′ on the surface of the tetrahedron. Denote this minimal length by dM . Determine the positions of M for which dM attains an extremum. 96. (VNM 2) Determine all functions f : R → R satisfying the following two conditions: (a) f (x + y) + f (x − y) = 2 f (x) f (y) for all x, y ∈ R, (b) limx→∞ f (x) = 0. 97. (VNM 3) In a plane a circle with radius R and center w and a line Λ are given. The distance between w and Λ is d, d > R. The points M and N are chosen on Λ in such a way that the circle with diameter MN is externally tangent to the given circle. Show that there exists a point A in the plane such that all the segments MN are seen in a constant angle from A. 3.26.3 Shortlisted Problems Proposals of the Problem Selection Committee. 1. (MNG 1)IMO4 Given a set M of 1985 positive integers, none of which has a prime divisor larger than 26, prove that the set has four distinct elements whose geometric mean is an integer. 2. (BRA 3) A polyhedron has 12 faces and is such that: (i) all faces are isosceles triangles, (ii) all edges have length either x or y, (iii) at each vertex either 3 or 6 edges meet, and (iv) all dihedral angles are equal. Find the ratio x/y. 3. (NLD 3)IMO3 The weight w(p) of a polynomial p, p(x) = ∑ni=0 ai xi , with integer coefficients ai is defined as the number of its odd coefficients. For i = 0, 1, 2, . . . , let qi (x) = (1 + x)i . Prove that for any finite sequence 0 ≤ i1 < i2 < · · · < in , the inequality 178 3 Problems w(qi1 + · · · + qin ) ≥ w(qi1 ) holds. 4. (AUS 1)IMO2 Each of the numbers in the set N = {1, 2, 3, . . ., n − 1}, where n ≥ 3, is colored with one of two colors, say red or black, so that: (i) i and n − i always receive the same color, and (ii) for some j ∈ N, relatively prime to n, i and | j − i| receive the same color for all i ∈ N, i 6= j. Prove that all numbers in N must receive the same color. 5. (ROU 1) Let D be the interior of the circle C and let A ∈ C. Show that the |MA| function f : D → R, f (M) = |MM′ | , where M ′ = (AM ∩C, is strictly convex; i.e., f (P) < f (M1 )+2 f (M2 ) , ∀M1 , M2 ∈ D, M1 6= M2 , where P is the midpoint of the segment M1 M2 . q p √ 6. (POL 1) Let xn = 2 2 + 3 3 + . . . + n n. Prove that xn+1 − xn < 1 , n! n = 2, 3, . . . . Alternatives 7. 1a.(CZS 3) The positive integers x1 , . . . , xn , n ≥ 3, satisfy x1 < x2 < · · · < xn < 2x1 . Set P = x1 x2 · · · xn . Prove that if p is a prime number, k a positive integer, and P is divisible by pk , then pPk ≥ n!. 8. 1b.(TUR 5) Find the smallest positive integer n such that (i) n has exactly 144 distinct positive divisors, and (ii) there are ten consecutive integers among the positive divisors of n. 9. 2a.(USA 3) Determine the radius of a sphere S that passes through the centroids of each face of a given tetrahedron T inscribed in a unit sphere with center O. Also, determine the distance from O to the center of S as a function of the edges of T . 10. 2b.(VNM 1) Prove that for every point M on the surface of a regular tetrahedron there exists a point M ′ such that there are at least three different curves on the surface joining M to M ′ with the smallest possible length among all curves on the surface joining M to M ′ . 11. 3a.(USS 3) Find a method by which one can compute the coefficients of P(x) = x6 + a1 x5 + · · · + a6 from the roots of P(x) = 0 by performing not more than 15 additions and 15 multiplications. 12. 3b.(UNK 4) A sequence of polynomials Pm (x, y, z), m = 0, 1, 2, . . . , in x, y, and z is defined by P0 (x, y, z) = 1 and by Pm (x, y, z) = (x + z)(y + z)Pm−1 (x, y, z + 1) − z2Pm−1 (x, y, z) 3.26 IMO 1985 179 for m > 0. Prove that each Pm (x, y, z) is symmetric, in other words, is unaltered by any permutation of x, y, z. 13. 4a.(BGR 1) Let m boxes be given, with some balls in each box. Let n < m be a given integer. The following operation is performed: choose n of the boxes and put 1 ball in each of them. Prove: (a) If m and n are relatively prime, then it is possible, by performing the operation a finite number of times, to arrive at the situation that all the boxes contain an equal number of balls. (b) If m and n are not relatively prime, there exist initial distributions of balls in the boxes such that an equal distribution is not possible to achieve. 14. 4b.(IRL 4) A set of 1985 points is distributed around the circumference of a circle and each of the points is marked with 1 or −1. A point is called “good” if the partial sums that can be formed by starting at that point and proceeding around the circle for any distance in either direction are all strictly positive. Show that if the number of points marked with −1 is less than 662, there must be at least one good point. 15. 5a.(FRA 3) Let K and K ′ be two squares in the same plane, their sides of equal length. Is it possible to decompose K into a finite number of triangles T1 , T2 , . . . , Tp with mutually disjoint interiors and find translations t1 ,t2 , . . . ,t p such that K′ = p [ ti (Ti )? i=1 16. 5b.(BEL 2) If possible, construct an equilateral triangle whose three vertices are on three given circles. 17. 6a.(SWE 3)IMO6 The sequence f1 , f2 , . . . , fn , . . . of functions is defined for x > 0 recursively by   1 f1 (x) = x, fn+1 (x) = fn (x) fn (x) + . n Prove that there exists one and only one positive number a such that 0 < fn (a) < fn+1 (a) < 1 for all integers n ≥ 1. 18. 6b.(CAN 5) Let x1 , x2 , . . . , xn be positive numbers. Prove that x21 2 x1 + x2 x3 + x22 2 x2 + x3 x4 + ···+ x2n−1 2 xn−1 + xn x1 + x2n ≤ n − 1. x2n + x1 x2 Supplementary Problems 19. (ISR 3) For which integers n ≥ 3 does there exist a regular n-gon in the plane such that all its vertices have integer coordinates in a rectangular coordinate system? 180 3 Problems 20. (UNK 5)IMO1 A circle whose center is on the side ED of the cyclic quadrilateral BCDE touches the other three sides. Prove that EB + CD = ED. 21. (IRL 1) The tangents at B and C to the circumcircle of the acute-angled triangle ABC meet at X . Let M be the midpoint of BC. Prove that (a) ∠BAM = ∠CAX, and (b) AM AX = cos ∠BAC. 22. (USS 5)IMO5 A circle with center O passes through points A and C and intersects the sides AB and BC of the triangle ABC at points K and N, respectively. The circumscribed circles of the triangles ABC and KBN intersect at two distinct points B and M. Prove that ∠OMB = 90◦ . 3.27 IMO 1986 181 3.27 The Twenty-Seventh IMO Warsaw, Poland, July 4–15, 1986 3.27.1 Contest Problems First Day (July 9) 1. The set S = {2, 5, 13} has the property that for every a, b ∈ S, a 6= b, the number ab − 1 is a perfect square. Show that for every positive integer d not in S, the set S ∪ {d} does not have the above property. 2. Let A, B,C be fixed points in the plane. A man starts from a certain point P0 and walks directly to A. At A he turns his direction by 60◦ to the left and walks to P1 such that P0 A = AP1 . After he performs the same action 1986 times successively around the points A, B,C, A, B,C, . . . , he returns to the starting point. Prove that ABC is an equilateral triangle, and that the vertices A, B,C are arranged counterclockwise. 3. To each vertex Pi (i = 1, . . . , 5) of a pentagon an integer xi is assigned, the sum s = ∑ xi being positive. The following operation is allowed, provided at least one of the xi ’s is negative: Choose a negative xi , replace it by −xi , and add the former value of xi to the integers assigned to the two neighboring vertices of Pi (the remaining two integers are left unchanged). This operation is to be performed repeatedly until all negative integers disappear. Decide whether this procedure must eventually terminate. Second Day (July 10) 4. Let A, B be adjacent vertices of a regular n-gon in the plane and let O be its center. Now let the triangle ABO glide around the polygon in such a way that the points A and B move along the whole circumference of the polygon. Describe the figure traced by the vertex O. 5. Find, with proof, all functions f defined on the nonnegative real numbers and taking nonnegative real values such that (i) f [x f (y)] f (y) = f (x + y), (ii) f (2) = 0 but f (x) 6= 0 for 0 ≤ x < 2. 6. Prove or disprove: Given a finite set of points with integer coefficients in the plane, it is possible to color some of these points red and the remaining ones white in such a way that for any straight line L parallel to one of the coordinate axes, the number of red colored points and the number of white colored points on L differ by at most 1. 182 3 Problems 3.27.2 Longlisted Problems 1. (AUS 1) Let k be one of the integers 2, 3, 4 and let n = 2k − 1. Prove the inequality 1 + bk + b2k + · · · + bnk ≥ (1 + bn)k for all real b ≥ 0. 2. (AUS 2) Let ABCD be a convex quadrilateral. DA and CB meet at F and AB and DC meet at E. The bisectors of the angles DFC and AED are perpendicular. Prove that these angle bisectors are parallel to the bisectors of the angles between the lines AC and BD. 3. (AUS 3) A line parallel to the side BC of a triangle ABC meets AB in F and AC in E. Prove that the circles on BE and CF as diameters intersect in a point lying on the altitude of the triangle ABC dropped from A to BC. 4. (BEL 1) Find the last eight digits of the binary development of 271986. 5. (BEL 2) Let ABC and DEF be acute-angled triangles. Write d = EF, e = FD, f = DE. Show that there exists a point P in the interior of ABC for which the value of the expression d · AP + e · BP + f ·CP attains a minimum. 6. (BEL 3) In an urn there are one ball marked 1, two balls marked 2, and so on, up to n balls marked n. Two balls are randomly drawn without replacement. Find the probability that the two balls are assigned the same number. 7. (BGR 1) (SL86-11). 8. (BGR 2) (SL86-19). 9. (CAN 1) In a triangle ABC, ∠BAC = 100◦, AB = AC. A point D is chosen on the side AC such that ∠ABD = ∠CBD. Prove that AD + DB = BC. 10. (CAN 2) A set of n standard dice are shaken and randomly placed in a straight line. If n < 2r and r < s, then the probability that there will be a string of at least r, but not more than s, consecutive 1’s can be written as P/6s+2 . Find an explicit expression for P. 11. (CAN 3) (SL86-20). 12. (CHN 1) Let O be an interior point of a tetrahedron A1 A2 A3 A4 . Let S1 , S2 , S3 , S4 be spheres with centers A1 , A2 , A3 , A4 , respectively, and let U,V be spheres with centers at O. Suppose that for i, j = 1, 2, 3, 4, i 6= j, the spheres Si and S j are tangent to each other at a point Bi j lying on Ai A j . Suppose also that U is tangent to all edges Ai A j and V is tangent to the spheres S1 , S2 , S3 , S4 . Prove that A1 A2 A3 A4 is a regular tetrahedron. 13. (CHN 2) Let N = {1, 2, . . . , n}, n ≥ 3. To each pair i, j of elements of N, i 6= j, there is assigned a number fi j ∈ {0, 1} such that fi j + f ji = 1. Let r(i) = 3.27 IMO 1986 183 ∑ j6=i fi j and write M = maxi∈N r(i), m = mini∈N r(i). Prove that for any w ∈ N with r(w) = m there exist u, v ∈ N such that r(u) = M and fuv fvw = 1. 14. (CHN 3) (SL86-17). 15. (CHN 4) Let N = B1 ∪ · · · ∪ Bq be a partition of the set N of all positive integers and let an integer l ∈ N be given. Prove that there exist a set X ⊂ N of cardinality l, an infinite set T ⊂ N, and an integer k with 1 ≤ k ≤ q such that for any t ∈ T and any finite set Y ⊂ X, the sum t + ∑y∈Y y belongs to Bk . 16. (CZS 1) Given a positive integer k, find the least integer nk for which there exist five sets S1 , S2 , S3 , S4 , S5 with the following properties: |S j | = k for j = 1, . . . , 5, 5 [ S j = nk ; j=1 |Si ∩ Si+1 | = 0 = |S5 ∩ S1 |, for i = 1, . . . , 4. 17. (CZS 2) We call a tetrahedron right-faced if each of its faces is a right-angled triangle. (a) Prove that every orthogonal parallelepiped can be partitioned into six rightfaced tetrahedra. (b) Prove that a tetrahedron with vertices A1 , A2 , A3 , A4 is right-faced if and only if there exist four distinct real pnumbers c1 , c2 , c3 , and c4 such that the edges A j Ak have lengths A j Ak = |c j − ck | for 1 ≤ j < k ≤ 4. 18. (CZS 3) (SL86-4). 19. (FIN 1) Let f : [0, 1] → [0, 1] satisfy f (0) = 0, f (1) = 1 and f (x + y) − f (x) = f (x) − f (x − y) for all x, y ≥ 0 with x − y, x + y ∈ [0, 1]. Prove that f (x) = x for all x ∈ [0, 1]. 20. (FIN 2) For any angle α with 0 < α < 180◦ , we call a closed convex planar set an α -set if it is bounded by two circular arcs (or an arc and a line segment) whose angle of intersection is α . Given a (closed) triangle T , find the greatest α such that any two points in T are contained in an α -set S ⊂ T . 21. (FRA 1) Let AB be a segment of unit length and let C, D be variable points of this segment. Find the maximum value of the product of the lengths of the six distinct segments with endpoints in the set {A, B,C, D}. 22. (FRA 2) Let (an )n∈N be the sequence of integers defined recursively by a0 = 0, a1 = 1, an+2 = 4an+1 + an for n ≥ 0. Find the common divisors of a1986 and a6891 . 23. (FRA 3) Let I and J be the centers of the incircle and the excircle in the angle BAC of the triangle ABC. For any point M in the plane of the triangle, not on the line BC, denote by IM and JM the centers of the incircle and the excircle (touching BC) of the triangle BCM. Find the locus of points M for which IIM JJM is a rectangle. 184 3 Problems 24. (FRA 4) Two families of parallel lines are given in the plane, consisting of 15 and 11 lines, respectively. In each family, any two neighboring lines are at a unit distance from one another; the lines of the first family are perpendicular to the lines of the second family. Let V be the set of 165 intersection points of the lines under consideration. Show that there exist not fewer than 1986 distinct squares with vertices in the set V . 25. (FRA 5) (SL86-7). 26. (FRG 1) (SL86-5). 27. (FRG 2) In an urn there are n balls numbered 1, 2, . . . , n. They are drawn at random one by one without replacement and the numbers are recorded. What is the probability that the resulting random permutation has only one local maximum? A term in a sequence is a local maximum if it is greater than all its neighbors. 28. (FRG 3) (SL86-13). 29. (FRG 4) We define a binary operation ⋆ in the plane as follows: Given two points A and B in the plane, C = A⋆ B is the third vertex of the equilateral triangle ABC oriented positively. What is the relative position of three points I, M, O in the plane if I ⋆ (M ⋆ O) = (O ⋆ I) ⋆ M holds? 30. (FRG 5) Prove that a convex polyhedron all of whose faces are equilateral triangles has at most 30 edges. 31. (UNK 1) Let P and Q be distinct points in the plane of a triangle ABC such that AP : AQ = BP : BQ = CP : CQ. Prove that the line PQ passes through the circumcenter of the triangle. 32. (UNK 2) Find, with proof, all solutions of the equation 1x + 2y − 3z = 1 in positive integers x, y, z. 33. (UNK 3) (SL86-1). 34. (UNK 4) For each nonnegative integer n, Fn (x) is a polynomial in x of degree n. Prove that if the identity   n n r Fn (2x) = ∑ (−1)n−r 2 Fr (x) r r=0 holds for each n, then   n r Fn (tx) = ∑ t (1 − t)n−r Fr (x) r r=0 n for each n and all t. 35. (UNK 5) Establish the maximum and minimum values that the sum |a|+|b|+|c| can have if a, b, c are real numbers such that the maximum value of |ax2 + bx+ c| is 1 for −1 ≤ x ≤ 1. 36. (GDR 1) (SL86-9). 3.27 IMO 1986 185 37. (GDR 2) Prove that the set {1, 2, . . . , 1986} can be partitioned into 27 disjoint sets so that none of these sets contains an arithmetic triple (i.e., three distinct numbers in an arithmetic progression). 38. (GDR 3) (SL86-12). 39. (HEL 1) Let S be a k-element set. (a) Find the number of mappings f : S → S such that (i) f (x) 6= x for x ∈ S, (ii) f ( f (x)) = x for x ∈ S. (b) The same with the condition (i) left out. 40. (HEL 2) Find the maximum value that the quantity 2m + 7n can have such that there exist distinct positive integers xi (1 ≤ i ≤ m), y j (1 ≤ j ≤ n) such that the n xi ’s are even, the y j ’s are odd, and ∑m i=1 xi + ∑ j=1 y j = 1986. 41. (HEL 3) Let M, N, P be the midpoints of the sides BC, CA, AB of a triangle ABC. The lines AM, BN, CP intersect the circumcircle of ABC at points A′ , B′ ,C′ , respectively. Show that if A′ B′C′ is an equilateral triangle, then so is ABC. 42. (HUN 1) The integers 1, 2, . . . , n2 are placed on the fields of an n × n chessboard (n > 2) in such a way that any two fields that have a common edge or a vertex are assigned numbers differing by at most n + 1. What is the total number of such placements? 43. (HUN 2) (SL86-10). 44. (IRL 1) (SL86-14). 45. (IRL 2) Given n real numbers a1 ≤ a2 ≤ · · · ≤ an , define M1 = 1 n 2 ∑ ai , M2 = n(n − 1) ∑ ai a j , Q = n i=1 1≤i< j≤n q M12 − M2 . Prove that a1 ≤ M1 − Q ≤ M1 + Q ≤ an and that equality holds if and only if a1 = a2 = · · · = an . 46. (IRL 3) We wish to construct a matrix with 19 rows and 86 columns, with entries xi j ∈ {0, 1, 2} (1 ≤ i ≤ 19, 1 ≤ j ≤ 86), such that: (i) in each column there are exactly k terms equal to 0; (ii) for any distinct j, k ∈ {1, . . . , 86} there is i ∈ {1, . . . , 19} with xi j + xik = 3. For what values of k is this possible? 47. (ISL 1) (SL86-16). 48. (ISL 2) Let P be a convex 1986-gon in the plane. Let A, D be interior points of two distinct sides of P and let B,C be two distinct interior points of the line segment AD. Starting with an arbitrary point Q1 on the boundary of P, define recursively a sequence of points Qn as follows: given Qn extend the directed line segment Qn B to meet the boundary of P in a point Rn and then extend RnC to 186 3 Problems meet the boundary of P again in a point, which is defined to be Qn+1 . Prove that for all n large enough the points Qn are on one of the sides of P containing A or D. 49. (ISL 3) Let C1 ,C2 be circles of radius 1/2 tangent to each other and both tangent internally to a circle C of radius 1. The circles C1 and C2 are the first two terms of an infinite sequence of distinct circles Cn defined as follows: Cn+2 is tangent externally to Cn and Cn+1 and internally to C. Show that the radius of each Cn is the reciprocal of an integer. 50. (LUX 1) Let D be the point on the side BC of the triangle ABC such that AD is the bisector of ∠CAB. Let I be the incenter of △ABC. (a) Construct the points P and Q on the sides AB and AC, respectively, such that PQ is parallel to BC and the perimeter of the triangle APQ is equal to k · BC, where k is a given rational number. (b) Let R be the intersection point of PQ and AD. For what value of k does the equality AR = RI hold? (c) In which case do the equalities AR = RI = ID hold? 51. (MNG 1) Let a, b, c, d be the lengths of the sides of√a quadrilateral circumscribed about a circle and let S be its area. Prove that S ≤ abcd and find conditions for equality. 52. (MNG 2) Solve the system of equations tan x1 + cotx1 tan x2 + cotx2 ··· tan xn + cotxn = 3 tan x2 , = 3 tan x3 , ··· = 3 tan x1 . 53. (MNG 3) For given positive integers r, v, n let S(r, v, n) denote the number of ntuples of nonnegative integers (x1 , . . . , xn ) satisfying the equation x1 + · · · + xn = r and such that xi ≤ v for i = 1, . . . , n. Prove that    m n r − (v + 1)k + n − 1 S(r, v, n) = ∑ (−1)k , k n−1 k=0   r  where m = min n, v+1 . 54. (MNG 4) Find the least integer n with the following property: For any set V of 8 points in the plane, no three lying on a line, and for any set E of n line segments with endpoints in V , one can find a straight line intersecting at least 4 segments in E in interior points. 55. (MNG 5) Given an integer n ≥ 2, determine all n-digit numbers M0 = a1 a2 . . . an (ai 6= 0, i = 1, 2, . . . , n) divisible by the numbers M1 = a2 a3 . . . an a1 , M2 = a3 a4 . . . an a1 a2 , . . . , Mn−1 = an a1 a2 . . . an−1 . 56. (MAR 1) Let A1 A2 A3 A4 A5 A6 be a hexagon inscribed into a circle with center O. Consider the circular arc with endpoints A1 , A6 not containing A2 . For any point 3.27 IMO 1986 187 M of that arc denote by hi the distance from M to the line Ai Ai+1 (1 ≤ i ≤ 5). Construct M such that the sum h1 + · · · + h5 is maximal. 57. (MAR 2) In a triangle ABC, the incircle touches the sides BC, CA, AB in the points A′ , B′ ,C′ , respectively; the excircle in the angle A touches the lines containing these sides in A1 , B1 ,C1 , and similarly, the excircles in the angles B and C touch these lines in A2 , B2 ,C2 and A3 , B3 ,C3 . Prove that the triangle ABC is rightangled if and only if one of the point triples (A′ , B3 ,C′ ), (A3 , B′ ,C3 ), (A′ , B′ ,C2 ), (A2 , B2 ,C′ ), (A2 , B1 ,C2 ), (A3 , B3 ,C1 ), (A1 , B2 ,C1 ), (A1 , B1 ,C3 ) is collinear. 58. (NLD 1) (SL86-6). 59. (NLD 2) (SL86-15). 60. (NLD 3) Prove the inequality (−a + b + c)2(a − b + c)2(a + b − c)2 ≥ (−a2 + b2 + c2 )(a2 − b2 + c2 )(a2 + b2 − c2 ) for all real numbers a, b, c. 61. (ROU 1) Given a positive integer n, find the greatest integer p with the property that for any function f : P(X ) → C, where X and C are sets of cardinality n and p, respectively, there exist two distinct sets A, B ∈ P(X ) such that f (A) = f (B) = f (A ∪ B). (P(X ) is the family of all subsets of X.) 62. (ROU 2) Determine all pairs of positive integers (x, y) satisfying the equation px − y3 = 1, where p is a given prime number. 63. (ROU 3) Let AA′ , BB′ ,CC′ be the bisectors of the angles of a triangle ABC (A′ ∈ BC, B′ ∈ CA, C′ ∈ AB). Prove that each of the lines A′ B′ , B′C′ , C′ A′ intersects the incircle in two points. 64. (ROU 4) Let (an )n∈N be the sequence of integers defined recursively by a1 = a2 = 1, an+2 = 7an+1 − an − 2 for n ≥ 1. Prove that an is a perfect square for every n. 65. (ROU 5) Let A1 A2 A3 A4 be a quadrilateral inscribed in a circle C. Show that there is a point M on C such that MA1 − MA2 + MA3 − MA4 = 0. 66. (SWE 1) One hundred red points and one hundred blue points are chosen in the plane, no three of them lying on a line. Show that these points can be connected pairwise, red ones with blue ones, by disjoint line segments. 67. (SWE 2) (SL86-2). 68. (SWE 3) Consider the equation x4 + ax3 + bx2 + ax + 1 = 0 with real coefficients a, b. Determine the number of distinct real roots and their multiplicities for various values of a and b. Display your result graphically in the (a, b) plane. 69. (TUR 1) (SL86-18). 70. (TUR 2) (SL86-21). 188 3 Problems 71. (TUR 3) Two straight lines perpendicular to each other meet each side of a triangle in points symmetric with respect to the midpoint of that side. Prove that these two lines intersect in a point on the nine-point circle. 72. (TUR 4) A one-person game with two possible outcomes is played as follows: After each play, the player receives either a or b points, where a and b are integers with 0 < b < a < 1986. The game is played as many times as one wishes and the total score of the game is defined as the sum of points received after successive plays. It is observed that every integer x ≥ 1986 can be obtained as the total score whereas 1985 and 663 cannot. Determine a and b. 73. (TUR 5) Let (ai )i∈N be a strictly increasing sequence of positive real numbers such that limi→∞ ai = +∞ and ai+1 /ai ≤ 10 for each i. Prove that for every positive integer k there are infinitely many pairs (i, j) with 10k ≤ ai /a j ≤ 10k+1 . 74. (USA 1) (SL86-8). Alternative formulation. Let A be a set of n points in space. From the family of all segments with endpoints in A, q segments have been selected and colored yellow. Suppose that all yellow segments are of different length. Prove that there exists a polygonal line composed of m yellow segments, where m ≥ 2q n , arranged in order of increasing length. 75. (USA 2) The incenter of a triangle is the midpoint of the line segment of length 4 joining the centroid and the orthocenter of the triangle. Determine the maximum possible area of the triangle. 76. (USA 3) (SL86-3). 77. (USS 1) Find all integers x, y, z that satisfy x3 + y3 + z3 = x + y + z = 8. 78. (USS 2) If T and T1 are two triangles with angles x, y, z and x1 , y1 , z1 , respectively, prove the inequality cos x1 cos y1 cos z1 + + ≤ cot x + coty + cotz. sin x sin y sin z 79. (USS 3) Let AA1 , BB1 ,CC1 be the altitudes in an acute-angled triangle ABC. K and M are points on the line segments A1C1 and B1C1 respectively. Prove that if the angles MAK and CAA1 are equal, then the angle C1 KM is bisected by AK. 80. (USS 4) Let ABCD be a tetrahedron and O its incenter, and let the line OD be perpendicular to AD. Find the angle between the planes DOB and DOC. 3.27.3 Shortlisted Problems 1. (UNK 3)IMO5 Find, with proof, all functions f defined on the nonnegative real numbers and taking nonnegative real values such that (i) f [x f (y)] f (y) = f (x + y), 3.27 IMO 1986 189 (ii) f (2) = 0 but f (x) 6= 0 for 0 ≤ x < 2. 2. (SWE 2) Let f (x) = xn where n is a fixed positive integer and x = 1, 2, . . . . Is the decimal expansion a = 0. f (1) f (2) f (3) . . . rational for any value of n? The decimal expansion of a is defined as follows: If f (x) = d1 (x)d2 (x) . . . . . . dr(x) (x) is the decimal expansion of f (x), then a = 0.1d1 (2)d2 (2) . . . dr(2) (2)d1 (3) . . . dr(3) (3)d1 (4) . . . . 3. (USA 3) Let A, B, and C be three points on the edge of a circular chord such that B is due west of C and ABC is an equilateral triangle whose side is 86 meters long. A boy swam from A directly toward B. After covering a distance of x meters, he turned and swam westward, reaching the shore after covering a distance of y meters. If x and y are both positive integers, determine y. 4. (CZS 3) Let n be a positive integer and let p be a prime number, p > 3. Find at least 3(n + 1) [easier version: 2(n + 1)] sequences of positive integers x, y, z satisfying xyz = pn (x + y + z) that do not differ only by permutation. 5. (FRG 1)IMO1 The set S = {2, 5, 13} has the property that for every a, b ∈ S, a 6= b, the number ab − 1 is a perfect square. Show that for every positive integer d not in S, the set S ∪ {d} does not have the above property. 6. (NLD 1) Find four positive integers each not exceeding 70000 and each having more than 100 divisors. 7. (FRA 5) Let real numbers x1 , x2 , . . . , xn satisfy 0 < x1 < x2 < · · · < xn < 1 and set x0 = 0, xn+1 = 1. Suppose that these numbers satisfy the following system of equations: n+1 1 (1) ∑ xi − x j = 0 where i = 1, 2, . . . , n. j=0, j6=i Prove that xn+1−i = 1 − xi for i = 1, 2, . . . , n. 8. (USA 1) From a collection of n persons q distinct two-member teams are selected and ranked 1, . . . , q (no ties). Let m be the least integer larger than or equal to 2q/n. Show that there are m distinct teams that may be listed so that (i) each pair of consecutive teams on the list have one member in common and (ii) the chain of teams on the list are in rank order. Alternative formulation. Given a graph with n vertices and q edges numbered 1, . . . , q, show that there exists a chain of m edges, m ≥ 2q n , each two consecutive edges having a common vertex, arranged monotonically with respect to the numbering. 9. (GDR 1)IMO6 Prove or disprove: Given a finite set of points with integer coordinates in the plane, it is possible to color some of these points red and the remaining ones white in such a way that for any straight line L parallel to one of 190 3 Problems the coordinate axes, the number of red colored points and the number of white colored points on L differ by at most 1. 10. (HUN 2) Three persons A, B,C, are playing the following game: A k-element subset of the set {1, . . . , 1986} is randomly chosen, with an equal probability of each choice, where k is a fixed positive integer less than or equal to 1986. The winner is A, B or C, respectively, if the sum of the chosen numbers leaves a remainder of 0, 1, or 2 when divided by 3. For what values of k is this game a fair one? (A game is fair if the three outcomes are equally probable.) 11. (BGR 1) Let f (n) be the least number of distinct points in the plane such that for each k = 1, 2, . . . , n there exists a straight line containing exactly k of these points. Find an explicit expression for f (n).   n+2 Simplified version. Show that f (n) = n+1 ([x] denoting the greatest in2 2 teger not exceeding x). 12. (GDR 3)IMO3 To each vertex Pi (i = 1, . . . , 5) of a pentagon an integer xi is assigned, the sum s = ∑ xi being positive. The following operation is allowed, provided at least one of the xi ’s is negative: Choose a negative xi , replace it by −xi , and add the former value of xi to the integers assigned to the two neighboring vertices of Pi (the remaining two integers are left unchanged). This operation is to be performed repeatedly until all negative integers disappear. Decide whether this procedure must eventually terminate. 13. (FRG 3) A particle moves from (0, 0) to (n, n) directed by a fair coin. For each head it moves one step east and for each tail it moves one step north. At (n, y), y < n, it stays there if a head comes up and at (x, n), x < n, it stays there if a tail comes up. Let k be a fixed positive integer. Find the probability that the particle needs exactly 2n + k tosses to reach (n, n). 14. (IRL 1) The circle inscribed in a triangle ABC touches the sides BC,CA, AB in D, E, F, respectively, and X ,Y, Z are the midpoints of EF, FD, DE, respectively. Prove that the centers of the inscribed circle and of the circles around XY Z and ABC are collinear. 15. (NLD 2) Let ABCD be a convex quadrilateral whose vertices do not lie on a circle. Let A′ B′C′ D′ be a quadrangle such that A′ , B′ ,C′ , D′ are the centers of the circumcircles of triangles BCD, ACD, ABD, and ABC. We write T (ABCD) = A′ B′C′ D′ . Let us define A′′ B′′C′′ D′′ = T (A′ B′C′ D′ ) = T (T (ABCD)). (a) Prove that ABCD and A′′ B′′C′′ D′′ are similar. (b) The ratio of similitude depends on the size of the angles of ABCD. Determine this ratio. 16. (ISL 1)IMO4 Let A, B be adjacent vertices of a regular n-gon in the plane and let O be its center. Now let the triangle ABO glide around the polygon in such a way that the points A and B move along the whole circumference of the polygon. Describe the figure traced by the vertex O. 3.27 IMO 1986 191 17. (CHN 3)IMO2 Let A, B,C be fixed points in the plane. A man starts from a certain point P0 and walks directly to A. At A he turns his direction by 60◦ to the left and walks to P1 such that P0 A = AP1 . After he does the same action 1986 times successively around the points A, B,C, A, B,C, . . . , he returns to the starting point. Prove that △ABC is equilateral and that the vertices A, B,C are arranged counterclockwise. 18. (TUR 1) Let AX, BY,CZ be three cevians concurrent at an interior point D of a triangle ABC. Prove that if two of the quadrangles DYAZ, DZBX, DXCY are circumscribable, so is the third. 19. (BGR 2) A tetrahedron ABCD is given such that AD = BC = a; AC = BD = b; AB ·CD = c2 . Let f (P) = AP + BP + CP + DP, where P is an arbitrary point in space. Compute the least value of f (P). 20. (CAN 3) Prove that the sum of the face angles at each vertex of a tetrahedron is a straight angle if and only if the faces are congruent triangles. 21. (TUR 2) Let ABCD be a tetrahedron having each √ sum of opposite sides equal 3 to 1. Prove that rA + rB + rC + rD ≤ , 3 where rA , rB , rC , rD are the inradii of the faces, equality holding only if ABCD is regular. 192 3 Problems 3.28 The Twenty-Eighth IMO Havana, Cuba, July 5–16, 1987 3.28.1 Contest Problems First Day (July 10) 1. Let S be a set of n elements. We denote the number of all permutations of S that have exactly k fixed points by pn (k). Prove that n ∑ kpn (k) = n!. k=0 2. The prolongation of the bisector AL (L ∈ BC) in the acute-angled triangle ABC intersects the circumscribed circle at point N. From point L to the sides AB and AC are drawn the perpendiculars LK and LM respectively. Prove that the area of the triangle ABC is equal to the area of the quadrilateral AKNM. 3. Suppose x1 , x2 , . . . , xn are real numbers with x21 + x22 + · · · + x2n = 1. Prove that for any integer k > 1 there are integers ei not all 0 and with |ei | < k such that √ (k − 1) n |e1 x1 + e2x2 + · · · + en xn | ≤ . kn − 1 Second Day (July 11) 4. Does there exist a function f : N → N, such that f ( f (n)) = n + 1987 for every natural number n? 5. Prove that for every natural number n ≥ 3 it is possible to put n points in the Euclidean plane such that the distance between each pair of points is irrational and each three points determine a nondegenerate triangle with rational area. 6. Letpf (x) = x2 + x + p, for p ∈ N. Prove that if the numbers f (0), f (1), . . ., f ([ p/3 ]) are primes, then all the numbers f (0), f (1), . . ., f (p − 2) are primes. 3.28.2 Longlisted Problems 1. (AUS 1) Let x1 , x2 , . . . , xn be n integers. Let n = p + q, where p and q are positive integers. For i = 1, 2, . . . , n, put Si = xi + xi+1 + · · · + xi+p−1 and Ti = xi+p + xi+p+1 + · · · + xi+n−1 (it is assumed that xi+n = xi for all i). Next, let m(a, b) be the number of indices i for which Si leaves the remainder a and Ti leaves the remainder b on division by 3, where a, b ∈ {0, 1, 2}. Show that m(1, 2) and m(2, 1) leave the same remainder when divided by 3. 3.28 IMO 1987 193 2. (AUS 2) Suppose we have a pack of 2n cards, in the order 1, 2, . . . , 2n. A perfect shuffle of these cards changes the order to n + 1, 1, n + 2, 2, . . .,n − 1, 2n, n; i.e., the cards originally in the first n positions have been moved to the places 2, 4, . . . , 2n, while the remaining n cards, in their original order, fill the odd positions 1, 3, . . . , 2n − 1. Suppose we start with the cards in the above order 1, 2, . . . , 2n and then successively apply perfect shuffles. What conditions on the number n are necessary for the cards eventually to return to their original order? Justify your answer. Remark. This problem is trivial. Alternatively, it may be required to find the least number of shuffles after which the cards will return to the original order. 3. (AUS 3) A town has a road network that consists entirely of one-way streets that are used for bus routes. Along these routes, bus stops have been set up. If the one-way signs permit travel from bus stop X to bus stop Y 6= X , then we shall say Y can be reached from X . We shall use the phrase Y comes after X when we wish to express that every bus stop from which the bus stop X can be reached is a bus stop from which the bus stop Y can be reached, and every bus stop that can be reached from Y can also be reached from X . A visitor to this town discovers that if X and Y are any two different bus stops, then the two sentences “Y can be reached from X ” and “Y comes after X ” have exactly the same meaning in this town. Let A and B be two bus stops. Show that of the following two statements, exactly one is true: (i) B can be reached from A; (ii) A can be reached from B. 4. (AUS 4) Let a1 , a2 , a3 , b1 , b2 , b3 be positive real numbers. Prove that (a1 b2 + a2 b1 + a1 b3 + a3 b1 + a2 b3 + a3 b2 )2 ≥ 4(a1 a2 + a2 a3 + a3a1 )(b1 b2 + b2 b3 + b3 b1 ) and show that the two sides of the inequality are equal if and only if a1 /b1 = a2 /b2 = a3 /b3 . 5. (AUS 5) Let there be given three circles K1 , K2 , K3 with centers O1 , O2 , O3 respectively, which meet at a common point P. Also, let K1 ∩ K2 = {P, A}, K2 ∩ K3 = {P, B}, K3 ∩ K1 = {P,C}. Given an arbitrary point X on K1 , join X to A to meet K2 again in Y , and join X to C to meet K3 again in Z. (a) Show that the points Z, B,Y are collinear. (b) Show that the area of triangle XY Z is less than or equal to 4 times the area of triangle O1 O2 O3 . 6. (AUS 6) (SL87-1). 7. (BEL 1) Let f : (0, +∞) → R be a function having the property that f (x) = f (1/x) for   all x > 0. Prove that there exists a function u : [1, +∞) → R satisfying x+1/x u = f (x) for all x > 0. 2 8. (BEL 2) Determine the least possible value of the natural number n such that n! ends in exactly 1987 zeros. 194 3 Problems 9. (BEL 3) In the set of 20 elements {1, 2, 3, 4, 5, 6, 7, 8, 9, 0, A, B, C, D, J, K, L, U , X , Y , Z} we have made a random sequence of 28 throws. What is the probability that the sequence CUBA JULY 1987 appears in this order in the sequence already thrown? 10. (FIN 1) In a Cartesian coordinate system, the circle C1 has center O1 (−2, 0) and radius 3. Denote the point (1, 0) by A and the origin by O. Prove that there is a constant c > 0 such that for every X that is exterior to C1 , OX − 1 ≥ c min{AX, AX 2 }. Find the largest possible c. 11. (FIN 2) Let S ⊂ [0, 1] be a set of 5 points with {0, 1} ⊂ S. The graph of a real function f : [0, 1] → [0, 1] is continuous and increasing, and it is linear on every subinterval I in [0, 1] such that the endpoints but no interior points of I are in S. We want to compute, using a computer, the extreme values of g(x,t) = f (x+t)− f (x) f (x)− f (x−t) for x − t, x + t ∈ [0, 1]. At how many points (x,t) is it necessary to compute g(x,t) with the computer? 12. (FIN 3) (SL87-3). 13. (FIN 4) Let A be an infinite set of positive integers such that every n ∈ A is the product of at most 1987 prime numbers. Prove that there are an infinite set B ⊂ A and a number p such that the greatest common divisor of any two distinct numbers in B is p. 14. (FRA 1) Given n real numbers 0 < t1 ≤ t2 ≤ · · · ≤ tn < 1, prove that   t22 t1 tnn 2 (1 − tn ) + + ···+ < 1. (1 − t12)2 (1 − t23 )2 (1 − tnn+1 )2 15. (FRA 2) Let a1 , a2 , a3 , b1 , b2 , b3 , c1 , c2 , c3 be nine strictly positive real numbers. We set S1 = a1 b2 c3 , S2 = a2 b3 c1 , S3 = a3 b1 c2 ; T1 = a1 b3 c2 , T2 = a2 b1 c3 , T3 = a3 b2 c1 . Suppose that the set {S1 , S2 , S3 , T1 , T2 , T3 } has at most two elements. Prove that S1 + S2 + S3 = T1 + T2 + T3 . 16. (FRA 3) Let ABC be a triangle. For every point M belonging to segment BC we denote by B′ and c′ the orthogonal projections of M on the straight lines AC and BC. Find points M for which the length of segment B′C′ is a minimum. 17. (FRA 4) Consider the number α obtained by writing one after another the decimal representations of 1, 1987, 19872, . . . to the right the decimal point. Show that α is irrational. 18. (FRA 5) (SL87-4). 3.28 IMO 1987 195 19. (FRG 1) (SL87-14). 20. (FRG 2) (SL87-15). 21. (FRG 3) (SL87-16). 22. (UNK 1) (SL87-5). 23. (UNK 2) A lampshade is part of the surface of a right circular cone whose axis is vertical. Its upper and lower edges are two horizontal circles. Two points are selected on the upper smaller circle and four points on the lower larger circle. Each of these six points has three of the others that are its nearest neighbors at a distance d from it. By distance is meant the shortest distance measured over the curved survace of the lampshade. √ Prove that the area of the lampshade if d 2 (2θ + 3), where cot θ2 = θ3 . 24. (UNK 3) Prove that if the equation x4 + ax3 + bx + c = 0 has all its roots real, then ab ≤ 0. 25. (UNK 4) Numbers d(n, m), with m, n integers, 0 ≤ m ≤ n, are defined by d(n, 0) = d(n, n) = 0 for all n ≥ 0 and md(n, m) = md(n − 1, m) + (2n − m)d(n − 1, m − 1) for all 0 < m < n. Prove that all the d(n, m) are integers. 26. (UNK 5) Prove that if x, y, z are real numbers such that x2 + y2 + z2 = 2, then x + y + z ≤ xyz + 2. 27. (UNK 6) Find, with proof, the smallest real number C with the following property: For every infinite sequence {xi } of positive real numbers such that x1 + x2 + · · · + xn ≤ xn+1 for n = 1, 2, 3, . . . , we have √ √ √ √ x1 + x2 + · · · + xn ≤ C x1 + x2 + · · · + xn for n = 1, 2, 3, . . .. 28. (GDR 1) h 2In i a chess tournament there are n ≥ 5 players, and they have already n played 4 + 2 games (each pair have played each other at most once). (a) Prove that there are five players a, b, c, d, e for which the pairs ab, ac, bc, ad, ae, de have already played. h i 2 (b) Is the statement also valid for the n4 + 1 games played? Make the proof by induction over n. 29. (GDR 2) (SL87-13). 30. (HEL 1) Consider the regular 1987-gon A1 A2 . . . A1987 with center O. Show that the sum of vectors belonging to any proper subset of M = {OA j | j = 1, 2, . . . , 1987} is nonzero. 31. (HEL 2) Construct a triangle ABC given its side a = BC, its circumradius R (2R ≥ a), and the difference 1/k = 1/c − 1/b, where c = AB and b = AC. 196 3 Problems 32. (HEL 3) Solve the equation 28x = 19y + 87z, where x, y, z are integers. 33. (HEL 4) (SL87-6). 34. (HUN 1) (SL87-8). 35. (HUN 2) (SL87-9). 36. (ISL 1) A game consists in pushing a flat stone along a sequence of squares S0 , S1 , S2 , . . . that are arranged in linear order. The stone is initially placed on square S0 . When the stone stops on a square Sk it is pushed again in the same direction and so on until it reaches S1987 or goes beyond it; then the game stops. Each time the stone is pushed, the probability that it will advance exactly n squares is 1/2n. Determine the probability that the stone will stop exactly on square S1987. 37. (ISL 2) Five distinct numbers are drawn successively and at random from the set {1, . . . , n}. Show that the probability of a draw in which the first three numbers as well as all five numbers can be arranged to form an arithmetic progression is 6 greater than (n−2) 3. 38. (ISL 3) (SL87-10). 39. (LUX 1) Let A be a set of polynomials with real coefficients and let them satisfy the following conditions: (i) if f ∈ A and deg f ≤ 1, then f (x) = x − 1; (ii) if f ∈ A and deg f ≥ 2, then either there exists g ∈ A such that f (x) = x2+deg g + xg(x) − 1 or there exist g, h ∈ A such that f (x) = x1+degg g(x) + h(x); (iii) for every f , g ∈ A, both x2+deg f + x f (x) − 1 and x1+deg f f (x) + g(x) belong to A. Let Rn ( f ) be the remainder of the Euclidean division of the polynomial f (x) by xn . Prove that for all f ∈ A and for all natural numbers n ≥ 1 we have Rn ( f )(1) ≤ 0 and Rn ( f )(1) = 0 ⇒ Rn ( f ) ∈ A. 40. (MNG 1) The perpendicular line issued from the center of the circumcircle to the bisector of angle C in a triangle ABC divides the segment of the bisector inside ABC into two segments with ratio of lengths λ . Given b = AC and a = BC, find the length of side c. 41. (MNG 2) Let n points be given arbitrarily in the plane, no three of them collinear. Let us draw segments between pairs of these points. What is the minimum number of segments that can be colored red in such a way that among any four points, three of them are connected by segments that form a red triangle? 42. (MNG 3) Find the integer solutions of the equation h√ i h √ i 2 m = (2 + 2)n . 3.28 IMO 1987 197 43. (MNG 4) Let 2n + 3 points be given in the plane in such a way that no three lie on a line and no four lie on a circle. Prove that the number of circles that pass through three of these points and contain exactly n interior points is not less than 1 2n+3 . 3 2 44. (MAR 1) Let θ1 , θ2 , . . . , θn be real numbers such that sin θ1 + · · · + sin θn = 0. Prove that  2 n | sin θ1 + 2 sin θ2 + · · · + n sin θn | ≤ . 4 45. (MAR 2) Let us consider a variable polygon with 2n sides (n ∈ N) in a fixed circle such that 2n − 1 of its sides pass through 2n − 1 fixed points lying on a straight line ∆ . Prove that the last side also passes through a fixed point lying on ∆. 46. (NLD 1) (SL87-7). 47. (NLD 2) Through a point P within a triangle ABC the lines l, m, and n perpendicular respectively to AP, BP,CP are drawn. Prove that if l intersects the line BC in Q, m intersects AC in R, and n intersects AB in S, then the points Q, R, and S are collinear. 48. (POL 1) (SL87-11). 49. (POL 2) In the coordinate system in the plane we consider a convex polygon W and lines given by equations x = k, y = m, where k and m are integers. The lines determine a tiling of the plane with unit squares. We say that the boundary of W intersects a square if the boundary contains an interior point of the square. Prove that the boundary of W intersects at most 4⌈d⌉ unit squares, where d is the maximal distance of points belonging to W (i.e., the diameter of W ) and ⌈d⌉ is the least integer not less than d. 50. (POL 3) Let P, Q, R be polynomials with real coefficients, satisfying P4 + Q4 = R2 . Prove that there exist real numbers p, q, r and a polynomial S such that P = pS, Q = qS and R = rS2 . Variants: (1) P4 + Q4 = R4 ; (2) gcd(P, Q) = 1; (3) ±P4 + Q4 = R2 or R4 . 51. (POL 4) The function F is a one-to-one transformation of the plane into itself that maps rectangles into rectangles (rectangles are closed; continuity is not assumed). Prove that F maps squares into squares. 52. (POL 5) (SL87-12). 53. (ROU 1) (SL87-17). 54. (ROU 2) Let n be a natural number. Solve in integers the equation xn + yn = (x − y)n+1 . 55. (ROU 3) Two moving bodies M1 , M2 are displaced uniformly on two coplanar straight lines. Describe the union of all straight lines M1 M2 . 56. (ROU 4) (SL87-18). 198 3 Problems 57. (ROU 5) The bisectors of the angles B,C of a triangle ABC intersect the opposite sides in B′ ,C′ respectively. Prove that the straight line B′C′ intersects the inscribed circle in two different points. 58. (ESP 1) Find, with argument, the integer solutions of the equation 3z2 = 2x3 + 385x2 + 256x − 58195. 59. (ESP 2) It is given that a11 , a22 are real numbers, that x1 , x2 , a12 , b1 , b2 are complex numbers, and that a11 a22 = a12 a12 (where a12 is the conjugate of a12 ). We consider the following system in x1 , x2 : x1 (a11 x1 + a12x2 ) = b1 , x2 (a12 x1 + a22x2 ) = b2 . (a) Give one condition to make the system consistent. (b) Give one condition to make argx1 − argx2 = 98◦ . 60. (TUR 1) It is given that x = −2272, y = 103 + 102 c + 10b + a, and z = 1 satisfy the equation ax + by + cz = 1, where a, b, c are positive integers with a < b < c. Find y. 61. (TUR 2) Let PQ be a line segment of constant length λ taken on the side BC of a triangle ABC with the order B, P, Q,C, and let the lines through P and Q parallel to the lateral sides meet AC at P1 and Q1 and AB at P2 and Q2 respectively. Prove that the sum of the areas of the trapezoids PQQ1 P1 and PQQ2 P2 is independent of the position of PQ on BC. 62. (TUR 3) Let l, l ′ be two lines in 3-space and let A, B,C be three points taken on l with B as midpoint of the segment q AC. If a, b, c are the distances of A, B,C from l ′ , respectively, show that b ≤ a2 +c2 2 , equality holding if l, l ′ are parallel. k 2 63. (TUR 4) Compute ∑2n k=0 (−1) ak , where ak are the coefficients in the expansion √ (1 − 2x + x2)n = 2n ∑ a k xk . k=0 64. (USA 1) Let r > 1 be a real number, and let n be the largest integer smaller than n r. Consider an arbitrary real number x with 0 ≤ x ≤ r−1 . By a base-r expansion of x we mean a representation of x in the form x= a 1 a 2 a3 + 2 + 3 + ··· , r r r where the ai are integers with 0 ≤ ai < r. n You may assume without proof that every number x with 0 ≤ x ≤ r−1 has at least one base-r expansion. n Prove that if r is not an integer, then there exists a number p, 0 ≤ p ≤ r−1 , which has infinitely many distinct base-r expansions. 3.28 IMO 1987 199 65. (USA 2) The runs of a decimal number are its increasing or decreasing blocks of digits. Thus 024379 has three runs: 024, 43, and 379. Determine the average number of runs for a decimal number in the set {d1 d2 . . . dn | dk 6= dk+1 , k = 1, 2, . . . , n − 1}, where n ≥ 2. 66. (USA 3) (SL87-2). 67. (USS 1) If a, b, c, d are real numbers such that a2 + b2 + c2 + d 2 ≤ 1, find the maximum of the expression (a + b)4 + (a + c)4 + (a + d)4 + (b + c)4 + (b + d)4 + (c + d)4 . 68. (USS 2) (SL87-19). Original formulation. Let there be given positive real numbers α , β , γ such that α + β + γ < π , α + β > γ , β + γ > α , γ + α > β . Prove that it is possible to draw a triangle with the lengths of the sides sin α , sin β , sin γ . Moreover, prove that its area is less than 1 (sin 2α + sin 2β + sin 2γ ). 8 69. (USS 3) (SL87-20). 70. (USS 4) (SL87-21). 71. (USS 5) To every natural number k, k ≥ 2, there corresponds a sequence an (k) according to the following rule: a0 = k, an = τ (an−1 ) for n ≥ 1, in which τ (a) is the number of different divisors of a. Find all k for which the sequence an (k) does not contain the square of an integer. 72. (VNM 1) Is it possible to cover a rectangle of dimensions m × n with bricks that have the tromino angular shape (an arrangement of three unit squares forming the letter L) if: (a) m × n = 1985 × 1987; (b) m × n = 1987 × 1989? 73. (VNM 2) Let f (x) be a periodic function of period T > 0 defined over R. Its first derivative is continuous on R. Prove that there exist x, y ∈ [0, T ) such that x 6= y and f (x) f ′ (y) = f (y) f ′ (x). 74. (VNM 3) (SL87-22). 75. (VNM 4) Let ak be positive numbers such that a1 ≥ 1 and ak+1 − ak ≥ 1 (k = 1, 2, . . . ). Prove that for every n ∈ N, n 1 ∑ ak+1 1987√ak < 1987. k=1 200 3 Problems 76. (VNM 5) Given two sequences of positive numbers {ak } and {bk } (k ∈ N) such that (i) ak < bk , (ii) cos ak x + cosbk x ≥ − 1k for all k ∈ N and x ∈ R, prove the existence of limk→∞ abkk and find this limit. 77. (YUG 1) Find the least natural number k such that for any a ∈ [0, 1] and any natural number n, 1 ak (1 − a)n < . (n + 1)3 78. (YUG 2) (SL87-23). 3.28.3 Shortlisted Problems 1. (AUS 6) Let f be a function that satisfies the following conditions: (i) If x > y and f (y) − y ≥ v ≥ f (x) − x, then f (z) = v + z, for some number z between x and y. (ii) The equation f (x) = 0 has at least one solution, and among the solutions of this equation, there is one that is not smaller than all the other solutions; (iii) f (0) = 1. (iv) f (1987) ≤ 1988. (v) f (x) f (y) = f (x f (y) + y f (x) − xy). Find f (1987). 2. (USA 3) At a party attended by n married couples, each person talks to everyone else at the party except his or her spouse. The conversations involve sets of persons or cliques C1 ,C2 , . . . ,Ck with the following property: no couple are members of the same clique, but for every other pair of persons there is exactly one clique to which both members belong. Prove that if n ≥ 4, then k ≥ 2n. 3. (FIN 3) Does there exist a second-degree polynomial p(x, y) in two variables such that every nonnegative integer n equals p(k, m) for one and only one ordered pair (k, m) of nonnegative integers? 4. (FRA 5) Let ABCDEFGH be a parallelepiped with AEkBFkCGkDH. Prove the inequality AF + AH + AC ≤ AB + AD + AE + AG. In what cases does equality hold? 5. (UNK 1) Find, with proof, the point P in the interior of an acute-angled triangle ABC for which BL2 + CM 2 + AN 2 is a minimum, where L, M, N are the feet of the perpendiculars from P to BC,CA, AB respectively. 6. (HEL 4) Show that if a, b, c are the lengths of the sides of a triangle and if 2S = a + b + c, then  n−2 an bn cn 2 + + ≥ Sn−1 , n ≥ 1. b+c c+a a+b 3 3.28 IMO 1987 201 7. (NLD 1) Given five real numbers u0 , u1 , u2 , u3 , u4 , prove that it is always possible to find five real numbers v0 , v1 , v2 , v3 , v4 that satisfy the following conditions: (i) ui − vi ∈ N. (ii) ∑0≤i< j≤4 (vi − v j )2 < 4. 8. (HUN 1) (a) Let (m, k) = 1. Prove that there exist integers a1 , a2 , . . . , am and b1 , b2 , . . . , bk such that each product ai b j (i = 1, 2, . . . , m; j = 1, 2, . . . , k) gives a different residue when divided by mk. (b) Let (m, k) > 1. Prove that for any integers a1 , a2 , . . . , am and b1 , b2 , . . . , bk there must be two products ai b j and as bt ((i, j) 6= (s,t)) that give the same residue when divided by mk. 9. (HUN 2) Does there exist a set M in usual Euclidean space such that for every plane λ the intersection M ∩ λ is finite and nonempty? 10. (ISL 3) Let S1 and S2 be two spheres with distinct radii that touch externally. The spheres lie inside a cone C, and each sphere touches the cone in a full circle. Inside the cone there are n additional solid spheres arranged in a ring in such a way that each solid sphere touches the cone C, both of the spheres S1 and S2 externally, as well as the two neighboring solid spheres. What are the possible values of n? 11. (POL 1) Find the number of partitions of the set {1, 2, . . . , n} into three subsets A1 , A2 , A3 , some of which may be empty, such that the following conditions are satisfied: (i) After the elements of every subset have been put in ascending order, every two consecutive elements of any subset have different parity. (ii) If A1 , A2 , A3 are all nonempty, then in exactly one of them the minimal number is even. 12. (POL 5) Given a nonequilateral triangle ABC, the vertices listed counterclockwise, find the locus of the centroids of the equilateral triangles A′ B′C′ (the vertices listed counterclockwise) for which the triples of points A, B′ ,C′ ; A′ , B,C′ ; and A′ , B′ ,C are collinear. 13. (GDR 2)IMO5 Is it possible to put 1987 points in the Euclidean plane such that the distance between each pair of points is irrational and each three points determine a nondegenerate triangle with rational area? 14. (FRG 1) How many words with n digits can be formed from the alphabet {0, 1, 2, 3, 4}, if neighboring digits must differ by exactly one? 15. (FRG 2)IMO3 Suppose x1 , x2 , . . . , xn are real numbers with x21 + x22 + · · · + x2n = 1. Prove that for any integer k > 1 there are integers ei not all 0 and with |ei | < k such that √ (k − 1) n |e1 x1 + e2x2 + · · · + en xn | ≤ . kn − 1 16. (FRG 3)IMO1 Let S be a set of n elements. We denote the number of all permutations of S that have exactly k fixed points by pn (k). Prove: 202 3 Problems (a) ∑nk=0 kpn (k) = n!; (b) ∑nk=0 (k − 1)2 pn (k) = n!. 17. (ROU 1) Prove that there exists a four-coloring of the set M = {1, 2, . . . , 1987} such that any arithmetic progression with 10 terms in the set M is not monochromatic. Alternative formulation. Let M = {1, 2, . . ., 1987}. Prove that there is a function f : M → {1, 2, 3, 4} that is not constant on every set of 10 terms from M that form an arithmetic progression. 18. (ROU 4) For any integer r ≥ 1, determine the smallest integer h(r) ≥ 1 such that for any partition of the set {1, 2, . . . , h(r)} into r classes, there are integers a ≥ 0, 1 ≤ x ≤ y, such that a + x, a + y, a + x + y belong to the same class. 19. (USS 2) Let α , β , γ be positive real numbers such that α + β + γ < π , α + β > γ , β + γ > α , γ + α > β . Prove that with the segments of lengths sin α , sin β , sin γ we can construct a triangle and that its area is not greater than 1 (sin 2α + sin 2β + sin 2γ ). 8 IMO6 20. (USS 3)p Let f (x) = x2 + x + p, p ∈ N. Prove that if the numbers f (0), f (1), . . ., f ([ p/3 ]) are primes, then all the numbers f (0), f (1), . . ., f (p − 2) are primes. 21. (USS 4)IMO2 The prolongation of the bisector AL (L ∈ BC) in the acute-angled triangle ABC intersects the circumscribed circle at point N. From point L to the sides AB and AC are drawn the perpendiculars LK and LM respectively. Prove that the area of the triangle ABC is equal to the area of the quadrilateral AKNM. 22. (VNM 3)IMO4 Does there exist a function f : N → N, such that f ( f (n)) = n + 1987 for every natural number n? 23. (YUG 2) Prove that for every natural number k (k ≥ 2) there exists an irrational number r such that for every natural number m, [rm ] ≡ −1 (mod k). Remark. An easier variant: Find r as a root of a polynomial of second degree with integer coefficients. 3.29 IMO 1988 203 3.29 The Twenty-Ninth IMO Canberra, Australia, July 9–21, 1988 3.29.1 Contest Problems First Day (July 15) 1. Consider two concentric circles of radii R and r (R > r) with center O. Fix P on the small circle and consider the variable chord PA of the small circle. Points B and C lie on the large circle; B, P,C are collinear and BC is perpendicular to AP. (a) For which value(s) of ∠OPA is the sum BC2 + CA2 + AB2 extremal? (b) What are the possible positions of the midpoints U of BA and V of AC as ∡OPA varies? 2. Let n be an even positive integer. Let A1 , A2 , . . . , An+1 be sets having n elements each such that any two of them have exactly one element in common, while every element of their union belongs to at least two of the given sets. For which n can one assign to every element of the union one of the numbers 0 and 1 in such a manner that each of the sets has exactly n/2 zeros? 3. A function f defined on the positive integers (and taking positive integer values) is given by f (1) = 1, f (3) = 3, f (2n) = f (n), f (4n + 1) = 2 f (2n + 1) − f (n), f (4n + 3) = 3 f (2n + 1) − 2 f (n), for all positive integers n. Determine with proof the number of positive integers less than or equal to 1988 for which f (n) = n. Second Day (July 16) 4. Show that the solution set of the inequality 70 k 5 ∑ x−k ≥ 4 k=1 is the union of disjoint half-open intervals with the sum of lengths 1988. 5. In a right-angled triangle ABC let AD be the altitude drawn to the hypotenuse and let the straight line joining the incenters of the triangles ABD, ACD intersect the sides AB, AC at the points K, L respectively. If E and E1 denote the areas of the triangles ABC and AKL respectively, show that EE1 ≥ 2. 6. Let a and b be two positive integers such that ab + 1 divides a2 + b2 . Show that a2 +b2 ab+1 is a perfect square. 204 3 Problems 3.29.2 Longlisted Problems 1. (BGR 1) (SL88-1). hp i 2. (BGR 2) Let an = (n + 1)2 + n2 , n = 1, 2, . . . , where [x] denotes the integer part of x. Prove that (a) there are infinitely many positive integers m such that am+1 − am > 1; (b) there are infinitely many positive integers m such that am+1 − am = 1. 3. (BGR 3) (SL88-2). 4. (CAN 1) (SL88-3). 5. (CUB 1) Let k be a positive integer and Mk the set of all the integers that are between 2k2 + k and 2k2 + 3k, both included. Is it possible to partition Mk into two subsets A and B such that ∑ x2 = ∑ x2 ? x∈A x∈B 6. (CZS 1) (SL88-4). 7. (CZS 2) (SL88-5). 8. (CZS 3) (SL88-6). 9. (FRA 1) If a0 is a positive real number, consider the sequence {an } defined by an+1 = a2n − 1 n+1 for n ≥ 0. Show that there exists a real number a > 0 such that: (i) for all real a0 ≥ a, the sequence {an } → +∞ (n → ∞); (ii) for all real a0 < a, the sequence {an } → 0. 10. (FRA 2) (SL88-7). 11. (FRA 3) (SL88-8). 12. (FRA 4) Show that there do not exist more than 27 half-lines (or rays) emanating from the origin in 3-dimensional space such that the angle between each pair of rays is greater than or equal to π /4. 13. (FRA 5) Let T be a triangle with inscribed circle C. A square with sides of length a is circumscribed about the same circle C. Show that the total length of the parts of the edges of the square interior to the triangle T is at least 2a. 14. (FRG 1) (SL88-9). 15. (FRG 2) Let 1 ≤ k < n. Consider all finite sequences of positive integers with sum n. Find T (n, k), the total number of terms of size k in all of these sequences. 16. (FRG 3) Show that if n runs through all positive integers, then 3.29 IMO 1988  f (n) = n + r n 1 + 3 2 205  runs through all positive integers skipping the terms of the sequence an = 3n2 − 2n. 17. (FRG 4) Show that if n runs through all positive integers, then   √ 1 f (n) = n + 3n + 2 runs all positive integers skipping the terms of the sequence an = h 2 through i n +2n . 3 18. (UNK 1) (SL88-25). 19. (UNK 2) (SL88-26). 20. (UNK 3) It is proposed to partition the set of positive integers into two disjoint subsets A and B subject to the following conditions: (i) 1 is in A; (ii) no two distinct members of A have a sum of the form 2k + 2 (k = 0, 1, 2, . . .); and (iii) no two distinct members of B have a sum of that form. Show that this partitioning can be carried out in a unique manner and determine the subsets to which 1987, 1988, and 1989 belong. 21. (UNK 4) (SL88-27). 22. (UNK 5) (SL88-28). 23. (GDR 1) (SL88-10). 24. (GDR 2) Let Zm,n be the set of all ordered pairs (i, j) with i ∈ {1, . . ., m} and j ∈ {1, . . . , n}. Also let am,n be the number of all those subsets of Zm,n that contain no two ordered pairs (i1 , j1 ), (i2 , j2 ) with |i1 − i2 | + | j1 − j2 | = 1. Show that for all positive integers m and k, a2m,2k ≤ am,2k−1 am,2k+1 . 25. (GDR 3) (SL88-11). 26. (HEL 1) Let AB and CD be two perpendicular chords of a circle with center O and radius r, and let X ,Y, Z,W denote in cyclical order the four parts into which the disk is thus divided. Find the maximum and minimum of the quantity A(Z) A(Y )+A(W ) , where A(U ) denotes the area of U. 27. (HEL 2) (SL88-12). 28. (HEL 3) (SL88-13). 29. (HEL 4) Find positive integers x1 , x2 , . . . , x29 , at least one of which is greater than 1988, such that 206 3 Problems x21 + x22 + · · · + x229 = 29x1 x2 . . . x29 . 30. (HKG 1) Find the total number of different integers that the function   5x f (x) = [x] + [2x] + + [3x] + [4x] 3 takes for 0 ≤ x ≤ 100. 31. (HKG 2) The circle x2 + y2 = r2 meets the coordinate axes at A = (r, 0), B = (−r, 0), C = (0, r), and D = (0, −r). Let P = (u, v) and Q = (−u, v) be two points on the circumference of the circle. Let N be the point of intersection of PQ and the y-axis, and let M be the foot of the perpendicular drawn from P to the x-axis. If r2 is odd, u = pm > qn = v, where p and q are prime numbers, and m and n are natural numbers, show that |AM| = 1, |BM| = 9, |DN| = 8, |PQ| = 8. 32. (HKG 3) Assuming that the roots of x3 + px2 + qx + r = 0 are all real and positive, find a relation between p, q, and r that gives a necessary condition for the roots to be exactly the cosines of three angles of a triangle. 33. (HKG 4) Find a necessary and sufficient condition on the natural number n for the equation xn + (2 + x)n + (2 − x)n = 0 to have a real root. 34. (HKG 5) Express the number 1988 as the sum of some positive integers in such a way that the product of these positive integers is maximal. 35. (HKG 6) In the triangle ABC, let D, E, and F be the midpoints of the three sides, X , Y , and Z the feet of the three altitudes, H the orthocenter, and P, Q, and R the midpoints of the line segments joining H to the three vertices. Show that the nine points D, E, F, P, Q, R, X ,Y, Z lie on a circle. 36. (HUN 1) (SL88-14). 37. (HUN 2) Let n points be given on the surface of a sphere. Show that the surface can be divided into n congruent regions such that each of them contains exactly one of the given points. 38. (HUN 3) In a multiple choice test there were 4 questions and 3 possible answers for each question. A group of students was tested and it turned out that for any 3 of them there was a question that the three students answered differently. What is the maximal possible number of students tested? 39. (ISL 1) (SL88-15). 40. (ISL 2) A sequence of numbers an , n = 1, 2, . . ., is defined as follows: a1 = 1/2, and for each n ≥ 2,   2n − 3 an = an−1 . 2n Prove that ∑nk=1 ak < 1 for all n ≥ 1. 3.29 IMO 1988 207 41. (IDN 1) (a) Let ABC be a triangle with AB = 12 and AC = 16. Suppose M is the midpoint of side BC and points E and F are chosen on sides AC and AB respectively, and suppose that the lines EF and AM intersect at G. If AE = 2AF then find the ratio EG/GF. (b) Let E be a point external to a circle and suppose that two chords EAB and ECD meet at an angle of 40◦ . If AB = BC = CD, find the size of ∠ACD. 42. (IDN 2) (a) Four balls of radius 1 are mutually tangent, three resting on the floor and the fourth resting on the others. A tetrahedron, each of whose edges has length s, is circumscribed around the balls. Find the value of s. (b) Suppose that ABCD and EFGH are opposite faces of a rectangular solid, with ∠DHC = 45◦ and ∠FHB = 60◦ . Find the cosine of ∠BHD. 43. (IDN 3) (a) The polynomial x2k + 1 + (x + 1)2k is not divisible by x2 + x + 1. Find the value of k. (b) If p, q, and r are distinct roots of x3 − x2 + x − 2 = 0, find the value of p3 + q 3 + r 3 . (c) If r is the remainder when each of the numbers 1059, 1417, and 2312 is divided by d, where d is an integer greater than one, find the value of d − r. (d) What is the smallest positive odd integer n such that the product of 21/7 , 23/7 , . . . , 2(2n+1)/7 is greater than 1000? 44. (IDN 4) (a) Let g(x) = x5 + x4 + x3 + x2 + x + 1. What is the remainder when the polynomial g(x12 ) is divided by the polynomial g(x)? (b) If k is a positive integer and f is a function such that√for every positive num 2  12/y √ ber x, f (x2 + 1) x = k, find the value of f 9+y for every positive y2 number y. (c) The function f satisfies the functional equation f (x) + f (y) = f (x + y) − xy − 1 for every pair x, y of real numbers. If f (1) = 1, find the number of integers n for which f (n) = n. 45. (IDN 5) (a) Consider a circle K with diameter AB, a circle L tangent to AB and to K, and a circle M tangent to circle K, circle L, and AB. Calculate the ratio of the area of circle K to the area of circle M. (b) In triangle ABC, AB = AC and ∡CAB = 80◦ . If points D, E, and F lie on sides BC, AC, and AB, respectively, and CE = CD and BF = BD, find the measure of ∡EDF. 46. (IDN 6) (a) Calculate x = √ √ √ √ √ √ (11+6 2) 11−6 2−(11−6 2) 11+6 2 √√ √√ √√ . ( 5+2+ 5−2)−( 5+1) 208 3 Problems (b) For each positive number x, let k = imum value of k. (x+1/x)6 −(x6 +1/x6 )−2 . (x+1/x)3 +(x3 +1/x3 ) Calculate the min- 47. (IRL 1) (SL88-16). 48. (IRL 2) Find all plane triangles whose sides have integer length and whose incircles have unit radius. 49. (IRL 3) Let −1 < x < 1. Show that 6 1 − x2 ∑ 1 − 2x cos(2π k/7) + x2 = k=0 7(1 + x7 ) . 1 − x7 Deduce that π 2π 3π + csc2 + csc2 = 8. 7 7 7 50. (IRL 4) Let g(n) be defined as follows: csc2 g(1) = 0, g(2) = 1, g(n + 2) = g(n) + g(n + 1) + 1 (n ≥ 1). Prove that if n > 5 is a prime, then n divides g(n)(g(n) + 1). 51. (ISR 1) Let A1 , A2 , . . . , A29 be 29 different sequences of positive integers. For 1 ≤ i < j ≤ 29 and any natural number x, we define Ni (x) to be the number of elements of the sequence Ai that are less than or equal to x, and Ni j (x) to be the number of elements of the intersection Ai ∩ A j that are less than or equal to x. It is given that for all 1 ≤ i ≤ 29 and every natural number x, x Ni (x) ≥ , e where e = 2.71828 . . . . Prove that there exists at least one pair i, j (1 ≤ i < j ≤ 29) such that Ni j (1988) > 200. 52. (ISR 2) (SL88-17). 53. (KOR 1) Let x = p, y = q, z = r, w = s be the unique solution of the system of linear equations x + ai y + a2i z + a3i w = a4i , i = 1, 2, 3, 4. Express the solution of the following system in terms of p, q, r, and s: x + a2i y + a4i z + a6i w = a8i , i = 1, 2, 3, 4. Assume the uniqueness of the solution. 54. (KOR 2) (SL88-22). 55. (KOR 3) Find all positive integers x such that the product of all digits of x is given by x2 − 10x − 22. 3.29 IMO 1988 209 56. (KOR 4) The Fibonacci sequence is defined by an+1 = an + an−1 (n ≥ 1), a0 = 0, a1 = a2 = 1. Find the greatest common divisor of the 1960th and 1988th terms of the Fibonacci sequence. 57. (KOR 5) Let C be a cube with edges of length 2. Construct a solid with fourteen faces by cutting off all eight corners of C, keeping the new faces perpendicular to the diagonals of the cube and keeping the newly formed faces identical. If at the conclusion of this process the fourteen faces so formed have the same area, find the area of each face of the new solid. 58. (KOR 6) For each pair of positive integers k and n, let Sk (n) be the base-k digit sum of n. Prove that there are at most two primes p less than 20,000 for which S31 (p) is a composite number. 59. (LUX 1) (SL88-18). 60. (MEX 1) (SL88-19). 61. (MEX 2) Prove that the numbers A, B, and C are equal, where we define A as the number of ways that we can cover a 2 × n rectangle with 2 × 1 rectangles, B as the number of sequences of ones and twos that add up to n, and C as   m m+1 + · · · + 2m if n = 2m, 0 + 2 2m   m+1 m+2 2m+1 + + · · · + 1 3 2m+1 if n = 2m + 1. 62. (MNG 1) The positive integer n has the property that in any set of n integers chosen from the integers 1, 2, . . . , 1988, twenty-nine of them form an arithmetic progression. Prove that n > 1788. 63. (MNG 2) Let ABCD be a quadrilateral. Let A′ BCD′ be the reflection of ABCD in BC, while A′′ B′CD′ is the reflection of A′ BCD′ in CD′ and A′′ B′′C′ D′ is the reflection of A′′ B′CD′ in D′ A′′ . Show that if the lines AA′′ and BB′′ are parallel, then ABCD is a cyclic quadrilateral. 64. (MNG 3) Given n points A1 , A2 , . . . , An , no three collinear, show that the n-gon A1 A2 . . . An can be inscribed in a circle if and only if A1 A2 · A3 An · · · An−1 An + A2 A3 · A4 An · · · An−1 An · A1 An + · · · +An−1 An−2 · A1 An · · · An−3 An = A1 An−1 · A2 An · · · An−2 An . 65. (MNG 4) (SL88-20). 66. (MNG 5) Suppose αi > 0, βi > 0 for 1 ≤ i ≤ n (n > 1) and that ∑ni=1 αi = ∑ni=1 βi = π . Prove that n n cos βi ∑ sin αi ≤ ∑ cot αi . i=1 i=1 210 3 Problems 67. (NLD 1) Given a set of 1988 points in the plane, no three points of the set collinear, the points of a subset with 1788 points are colored blue, and the remaining 200 are colored red. Prove that there exists a line in the plane such that each of the two parts into which the line divides the plane contains 894 blue points and 100 red points. 68. (NLD 2) Let S be the set of all sequences {ai | 1 ≤ i ≤ 7, ai = 0 or 1}. The distance between two elements {ai } and {bi } of S is defined as ∑7i=1 |ai − bi |. Let T be a subset of S in which any two elements have a distance apart greater than or equal to 3. Prove that T contains at most 16 elements. Give an example of such a subset with 16 elements. 69. (POL 1) For a convex polygon P in the plane let P′ denote the convex polygon with vertices at the midpoints of the sides of P. Given an integer n ≥ 3, determine area(P′ ) sharp bounds for the ratio over all convex n-gons P. area(P) 70. (POL 2) In 3-dimensional space a point O is given and a finite set A of segments with the sum of the lengths equal to 1988. Prove that there exists a plane disjoint from A such that the distance from it to O does not exceed 574. 71. (POL 3) Given integers a1 , . . . , a10 , prove that there exists a nonzero sequence (x1 , . . . , x10 ) such that all xi belong to {−1, 0, 1} and the number ∑10 i=1 xi ai is divisible by 1001. 72. (POL 4) (SL88-21). 73. (SGP 1) In a group of n people each one knows exactly three others. They are seated around a table. We say that the seating is perfect if everyone knows the two sitting by their sides. Show that if there is a perfect seating S for the group, then there is always another perfect seating that cannot be obtained from S by rotation or reflection. 74. (SGP 2) (SL88-23). 75. (ESP 1) Let ABC be a triangle with inradius r and circumradius R. Show that A B B C C A 5 r sin + sin sin + sin sin ≤ + . 2 2 2 2 2 2 8 4R 76. (ESP 2) The quadrilateral A1 A2 A3 A4 is cyclic and its sides are a1 = A1 A2 , a2 = A2 A3 , a3 = A3 A4 , and a4 = A4 A1 . The respective circles with centers Ii and radii ρi are tangent externally to each side ai and to the sides ai+1 and ai−1 extended (a0 = a4 ). Show that sin 4 ai ∏ ρi = 4(csc A1 + cscA2 )2 . i=1 77. (ESP 3) Consider h + 1 chessboards. Number the squares of each board from 1 to 64 in such a way that when the perimeters of any two boards of the collection are brought into coincidence in any possible manner, no two squares in the same position have the same number. What is the maximum value of h? 3.29 IMO 1988 211 78. (SWE 1) A two-person game is played with nine boxes arranged in a 3 × 3 square, initially empty, and with white and black stones. At each move a player puts three stones, not necessarily of the same color, in three boxes in either a horizontal or a vertical row. No box can contain stones of different colors: If, for instance, a player puts a white stone in a box containing black stones, the white stone and one of the black stones are removed from the box. The game is over when the center box and the corner boxes each contain one black stone and the other boxes are empty. At one stage of the game x boxes contained one black stone each and the other boxes were empty. Determine all possible values of x. 79. (SWE 2) (SL88-24). 80. (SWE 3) Let S be an infinite set of integers containing zero and such that the distance between successive numbers never exceeds a given fixed number. Consider the following procedure: Given a set X of integers, we construct a new set consisting of all numbers x ± s, where x belongs to X and s belongs to S. Starting from S0 = {0} we successively construct sets S1 , S2 , S3 , . . . using this procedure. Show that after a finite number of steps we do not obtain any new sets; i.e., Sk = Sk0 for k ≥ k0 . 81. (USA 1) There are n ≥ 3 job openings at a factory, ranked 1 to n in order of increasing pay. There are n job applicants, ranked 1 to n in order of increasing ability. Applicant i is qualified for job j if and only if i ≥ j. The applicants arrive one at a time in random order. Each in turn is hired to the highest-ranking job for which he or she is qualified and that is lower in rank than any job already filled. (Under these rules, job 1 is always filled and hiring terminates thereafter.) Show that applicants n and n − 1 have the same probability of being hired. 82. (USA 2) The triangle ABC has a right angle at C. The point P is located on segment AC such that triangles PBA and PBC have congruent inscribed circles. Express the length x = PC in terms of a = BC, b = CA, and c = AB. 83. (USA 3) (SL88-29). 84. (USS 1) (SL88-30). 85. (USS 2) (SL88-31). 86. (USS 3) Let a, b, c be integers different from zero. It is known that the equation ax2 + by2 + cz2 = 0 has a solution (x, y, z) in integers different from the solution x = y = z = 0. Prove that the equation ax2 + by2 + cz2 = 1 has a solution in rational numbers. 87. (USS 4) All the irreducible positive rational numbers such that the product of the numerator and the denominator is less than 1988 are written in increasing order. Prove that any two adjacent fractions a/b and c/d, a/b < c/d, satisfy the equation bc − ad = 1. 88. (USS 5) There are six circles inside a fixed circle, each tangent to the fixed circle and tangent to the two adjacent smaller circles. If the points of contact between 212 3 Problems the six circles and the larger circle are, in order, A1 , A2 , A3 , A4 , A5 , and A6 , prove that A1 A2 · A3 A4 · A5 A6 = A2 A3 · A4 A5 · A6 A1 . 89. (VNM 1) We match sets M of points in the coordinate plane to sets M ∗ according to the rule that (x∗ , y∗ ) belongs to M ∗ if and only if xx∗ + yy∗ ≤ 1 whenever (x, y) ∈ M . Find all triangles Y such that Y ∗ is the reflection of Y at the origin. 90. (VNM 2) Does there exist a number α (0 < α < 1) such that there is an infinite sequence {an } of positive numbers satisfying 1 + an+1 ≤ an + α an , n n = 1, 2, . . .? 91. (VNM 3) A regular 14-gon with side length a is inscribed in a circle of radius one. Prove that r 2−a π > 3 cos . 2a 7 92. (VNM 4) Let p ≥ 2 be a natural number. Prove that there exists an integer n0 such that n0 1 ∑ i √p i + 1 > p. i=1 93. (VNM 5) Given a natural number n, find all polynomials P(x) of degree less than n satisfying the following condition:   n i n P(i)(−1) = 0. ∑ i i=0 94. (VNM 6) Let n + 1 (n ≥ 1) positive integers be given such that for each integer, the set of all prime numbers dividing this integer is a subset of the set of n given prime numbers. Prove that among these n + 1 integers one can find numbers (possibly one number) whose product is a perfect square. 3.29.3 Shortlisted Problems 1. (BGR 1) An integer sequence is defined by an = 2an−1 + an−2 (n > 1), a0 = 0, a1 = 1. Prove that 2k divides an if and only if 2k divides n. 2. (BGR 3) Let n be a positive integer. Find the number of odd coefficients of the polynomial un (x) = (x2 + x + 1)n. 3. (CAN 1) The triangle ABC is inscribed in a circle. The interior bisectors of the angles A, B, and C meet the circle again at A′ , B′ , and C′ respectively. Prove that the area of triangle A′ B′C′ is greater than or equal to the area of triangle ABC. 3.29 IMO 1988 213 4. (CZS 1) An n × n chessboard (n ≥ 2) is numbered by the numbers 1, 2, . . . , n2 (every number occurs once). Prove that there exist two neighboring (which share a common edge) squares such that their numbers differ by at least n. 5. (CZS 2)IMO2 Let n be an even positive integer. Let A1 , A2 , . . . , An+1 be sets having n elements each such that any two of them have exactly one element in common while every element of their union belongs to at least two of the given sets. For which n can one assign to every element of the union one of the numbers 0 and 1 in such a manner that each of the sets has exactly n/2 zeros? 6. (CZS 3) In a given tetrahedron ABCD let K and L be the centers of edges AB and CD respectively. Prove that every plane that contains the line KL divides the tetrahedron into two parts of equal volume. 7. (FRA 2) Let a be the greatest positive root of the equation x3 − 3x2 + 1 = 0. Show that [a1788 ] and [a1988] are both divisible by 17. ([x] denotes the integer part of x.) 8. (FRA 3) Let u1 , u2 , . . . , um be m vectors in the plane, each of length less than or equal to 1, which add up to zero. Show that one can rearrange u1 , u2 , . . . , um as a sequence v1 , v2 , . . . , vm such that each partial sum√v1 , v1 + v2 , v1 + v2 + v3 , . . . , v1 + v2 + · · · + vm has length less than or equal to 5. 9. (FRG 1)IMO6 Let a and b be two positive integers such that ab+1 divides a2 +b2 . 2 +b2 Show that aab+1 is a perfect square. 10. (GDR 1) Let N = {1, 2, . . . , n}, n ≥ 2. A collection F = {A1 , . . . , At } of subsets Ai ⊆ N, i = 1, . . . ,t, is said to be separating if for every pair {x, y} ⊆ N, there is a set Ai ∈ F such that Ai ∩ {x, y} contains just one element. A collection F is said to be covering if every element of N is contained in at least one set Ai ∈ F. What is the smallest value f (n) of t such that there is a set F = {A1 , . . . , At } that is simultaneously separating and covering? 11. (GDR 3) The lock on a safe consists of three wheels, each of which may be set in eight different positions. Due to a defect in the safe mechanism the door will open if any two of the three wheels are in the correct position. What is the smallest number of combinations that must be tried if one is to guarantee being able to open the safe (assuming that the “right combination” is not known)? 12. (HEL 2) In a triangle ABC, choose any points K ∈ BC, L ∈ AC, M ∈ AB, N ∈ LM, R ∈ MK, and F ∈ KL. If E1 , E2 , E3 , E4 , E5 , E6 , and E denote the areas of the triangles AMR, CKR, BKF, ALF, BNM, CLN, and ABC respectively, show that p E ≥ 8 6 E 1 E2 E3 E4 E5 E6 . Remark. Points K, L, M, N, R, F lie on segments BC, AC, AB, LM, MK, KL respectively. 13. (HEL 3)IMO5 In a right-angled triangle ABC, let AD be the altitude drawn to the hypotenuse and let the straight line joining the incenters of the triangles 214 3 Problems ABD, ACD intersect the sides AB, AC at the points K, L respectively. If E and E1 denote the areas of the triangles ABC and AKL respectively, show that EE ≥ 2. 1 14. (HUN 1) For what values of n does there exist an n × n array of entries −1, 0, or 1 such that the 2n sums obtained by summing the elements of the rows and the columns are all different? 15. (ISL 1) Let ABC be an acute-angled triangle. Three lines LA , LB , and LC are constructed through the vertices A, B, and C respectively according to the following prescription: Let H be the foot of the altitude drawn from the vertex A to the side BC; let SA be the circle with diameter AH; let SA meet the sides AB and AC at M and N respectively, where M and N are distinct from A; then LA is the line through A perpendicular to MN. The lines LB and LC are constructed similarly. Prove that LA , LB , and LC are concurrent. 16. (IRL 1)IMO4 Show that the solution set of the inequality 70 k 5 ∑ x−k ≥ 4 k=1 is a union of disjoint intervals the sum of whose lengths is 1988. 17. (ISR 2) In the convex pentagon ABCDE, the sides BC,CD, DE have the same length. Moreover, each diagonal of the pentagon is parallel to a side (AC is parallel to DE, BD is parallel to AE, etc.). Prove that ABCDE is a regular pentagon. 18. (LUX 1)IMO1 Consider two concentric circles of radii R and r (R > r) with center O. Fix P on the small circle and consider the variable chord PA of the small circle. Points B and C lie on the large circle; B, P,C are collinear and BC is perpendicular to AP. (a) For what value(s) of ∠OPA is the sum BC2 + CA2 + AB2 extremal? (b) What are the possible positions of the midpoints U of BA and V of AC as ∠OPA varies? 19. (MEX 1) Let f (n) be a function defined on the set of all positive integers and having its values in the same set. Suppose that f ( f (m) + f (n)) = m + n for all positive integers n, m. Find all possible values for f (1988). 20. (MNG 4) Find the least natural number n such that if the set {1, 2, . . . , n} is arbitrarily divided into two nonintersecting subsets, then one of the subsets contains three distinct numbers such that the product of two of them equals the third. 21. (POL 4) Forty-nine students solve a set of three problems. The score for each problem is a whole number of points from 0 to 7. Prove that there exist two students A and B such that for each problem, A will score at least as many points as B. 22. (KOR 2) Let p be the product of two consecutive integers greater than 2. Show that there are no integers x1 , x2 , . . . , x p satisfying the equation 3.29 IMO 1988 p ∑ i=1 x2i − 4 4p + 1 p ∑ xi i=1 !2 215 = 1. Alternative formulation. Show that there are only two values of p for which there are integers x1 , x2 , . . . , x p satisfying the above inequality. 23. (SGP 2) Let Q be the center of the inscribed circle of a triangle ABC. Prove that for any point P, a(PA)2 + b(PB)2 + c(PC)2 = a(QA)2 + b(QB)2 + c(QC)2 + (a + b + c)(QP)2, where a = BC, b = CA, and c = AB. 24. (SWE 2) Let {ak }∞ 1 be a sequence of nonnegative real numbers such that ak − 2ak+1 + ak+2 ≥ 0 and ∑kj=1 a j ≤ 1 for all k = 1, 2, . . . . Prove that 0 ≤ (ak − ak+1 ) < k22 for all k = 1, 2, . . . . 25. (UNK 1) A positive integer is called a double number if its decimal representation consists of a block of digits, not commencing with 0, followed immediately by an identical block. For instance, 360360 is a double number, but 36036 is not. Show that there are infinitely many double numbers that are perfect squares. 26. (UNK 2)IMO3 A function f defined on the positive integers (and taking positive integer values) is given by f (1) = 1, f (3) = 3, f (2n) = f (n), f (4n + 1) = 2 f (2n + 1) − f (n), f (4n + 3) = 3 f (2n + 1) − 2 f (n), for all positive integers n. Determine with proof the number of positive integers less than or equal to 1988 for which f (n) = n. 27. (UNK 4) The triangle ABC is acute-angled. Let L be any line in the plane of the triangle and let u, v, w be the lengths of the perpendiculars from A, B,C respectively to L. Prove that u2 tan A + v2 tan B + w2 tanC ≥ 2∆ , where ∆ is the area of the triangle, and determine the lines L for which equality holds. 28. (UNK 5) The sequence {an } of integers is defined by a1 = 2, a2 = 7, and 1 a2 1 − < an+1 − n ≤ , 2 an−1 2 Prove that an is odd for all n > 1. for n ≥ 2. 216 3 Problems 29. (USA 3) A number of signal lights are equally spaced along a one-way railroad track, labeled in order 1, 2, . . . , N (N ≥ 2). As a safety rule, a train is not allowed to pass a signal if any other train is in motion on the length of track between it and the following signal. However, there is no limit to the number of trains that can be parked motionless at a signal, one behind the other. (Assume that the trains have zero length.) A series of K freight trains must be driven from Signal 1 to Signal N. Each train travels at a distinct but constant speed (i.e., the speed is fixed and different from that of each of the other trains) at all times when it is not blocked by the safety rule. Show that regardless of the order in which the trains are arranged, the same time will elapse between the first train’s departure from Signal 1 and the last train’s arrival at Signal N. 30. (USS 1) A point M is chosen on the side AC of the triangle ABC in such a way that the radii of the circles inscribed in the triangles ABM and BMC are equal. Prove that B BM 2 = ∆ cot , 2 where ∆ is the area of the triangle ABC. 31. (USS 2) Around a circular table an even number of persons have a discussion. After a break they sit again around the circular table in a different order. Prove that there are at least two people such that the number of participants sitting between them before and after the break is the same. 3.30 IMO 1989 217 3.30 The Thirtieth IMO Braunschweig–Niedersachen, FR Germany, July 13–24, 1989 3.30.1 Contest Problems First Day (July 18) 1. Prove that the set {1, 2, . . . , 1989} can be expressed as the disjoint union of 17 subsets A1 , A2 , . . . , A17 such that: (i) each Ai contains the same number of elements; (ii) the sum of all elements of each Ai is the same for i = 1, 2, . . . , 17. 2. Let ABC be a triangle. The bisector of angle A meets the circumcircle of triangle ABC in A1 . Points B1 and C1 are defined similarly. Let AA1 meet the lines that bisect the two external angles at B and C in point A0 . Define B0 and C0 similarly. If SX1 X2 ...Xn denotes the area of the polygon X1 X2 . . . Xn , prove that SA0 B0C0 = 2SAC1 BA1CB1 ≥ 4SABC . 3. Given a set S in the plane containing n points and satisfying the conditions (i) no three points of S are collinear, (ii) for every point P of S there exist at least k points in S that have the same distance to P, prove that the following inequality holds: k< 1 √ + 2n. 2 Second Day (July 19) 4. The quadrilateral ABCD has the following properties: (i) AB = AD + BC; (ii) there is a point P inside it at a distance x from the side CD such that AP = x + AD and BP = x + BC. Show that 1 1 1 √ ≥√ +√ . x BC AD 5. For which positive integers n does there exist a positive integer N such that none of the integers 1 + N, 2 + N, . . ., n + N is the power of a prime number? 6. We consider permutations (x1 , . . . , x2n ) of the set {1, . . . , 2n} such that |xi − xi+1 | = n for at least one i ∈ {1, . . . , 2n − 1}. For every natural number n, find out whether permutations with this property are more or less numerous than the remaining permutations of {1, . . . , 2n}. 218 3 Problems 3.30.2 Longlisted Problems 1. (AUS 1) In the set Sn = {1, 2, . . . , n} a new multiplication a ∗ b is defined with the following properties: (i) c = a ∗ b is in Sn for any a ∈ Sn , b ∈ Sn . (ii) If the ordinary product a · b is less than or equal to n, then a ∗ b = a · b. (iii) The ordinary rules of multiplication hold for ∗, i.e., (1) a ∗ b = b ∗ a (commutativity) (2) (a ∗ b) ∗ c = a ∗ (b ∗ c) (associativity) (3) If a ∗ b = a ∗ c then b = c (cancellation law). Find a suitable multiplication table for the new product for n = 11 and n = 12. 2. (AUS 2) (SL89-1). 3. (AUS 3) (SL89-2). 4. (AUS 4) (SL89-3). 5. (BGR 1) The sequences a0 , a1 , . . . and b0 , b1 , . . . are defined by the equalities √ √ r q 2 2 a0 = , an+1 = 1 − 1 − a2n, n = 0, 1, 2, . . . 2 2 and b0 = 1, bn+1 = Prove the inequalities p 1 + b2n − 1 , bn 2n+2 an < π < 2n+2 bn , n = 0, 1, 2, . . . . for every n = 0, 1, 2, . . . . 6. (BGR 2) The circles c1 and c2 are tangent at the point A. A straight line l through A intersects c1 and c2 at points C1 and C2 respectively. A circle c, which contains C1 and C2 , meets c1 and c2 at points B1 and B2 respectively. Let κ be the circle circumscribed around triangle AB1 B2 . The circle k tangent to κ at the point A meets c1 and c2 at the points D1 and D2 respectively. Prove that (a) the points C1 ,C2 , D1 , D2 are concyclic or collinear; (b) the points B1 , B2 , D1 , D2 are concyclic if and only if AC1 and AC2 are diameters of c1 and c2 . 7. (BGR 3) (SL89-4). 8. (COL 1) (SL89-5). 9. (COL 2) Let m be a positive integer and define f (m) to be the number of factors of 2 in m! (that is, the greatest positive integer k such that 2k | m!). Prove that there are infinitely many positive integers m such that m − f (m) = 1989. 10. (CUB 1) Given the equation 4x3 + 4x2 y − 15xy2 − 18y3 − 12x2 + 6xy + 36y2 + 5x − 10y = 0, find all positive integer solutions. 3.30 IMO 1989 219 11. (CUB 2) Given the equation y4 + 4y2 x − 11y2 + 4xy − 8y + 8x2 − 40x + 52 = 0, find all real solutions. 12. (CUB 3) Let P(x) be a polynomial such that the following inequalities are satisfied: P(0) > 0; P(1) > P(0); P(2) > 2P(1) − P(0); P(3) > 3P(2) − 3P(1) + P(0); and also for every natural number n, P(n + 4) > 4P(n + 3) − 6P(n + 2) + 4P(n + 1) − P(n). Prove that for every positive natural number n, P(n) is positive. 13. (CUB 4) Let n be a natural number not greater than 44. Prove that for any function f defined over N2 whose images are in the set {1, 2, . . .,n}, there are four ordered pairs (i, j), (i, k), (l, j), and (l, k) such that f (i, j) = f (i, k) = f (l, j) = f (l, k), where i, j, k, l are chosen in such a way that there are natural numbers n, p that satisfy 1989m ≤ i < l < 1989 + 1989m, 1989p ≤ j < k < 1989 + 1989p. 14. (CZS 1) (SL89-6). 15. (CZS 2) A sequence a1 , a2 , a3 , . . . is defined recursively by a1 = 1 and a2k + j = −a j ( j = 1, 2, . . . , 2k ). Prove that this sequence is not periodic. 16. (FIN 1) (SL89-7). 17. (FIN 2) Let a, 0 < a < 1, be a real number and f a continuous function on [0, 1] satisfying f (0) = 0, f (1) = 1, and   x+y f = (1 − a) f (x) + a f (y) 2 for all x, y ∈ [0, 1] with x ≤ y. Determine f (1/7). 18. (FIN 3) There are some boys and girls sitting in an n × n quadratic array. We know the number of girls in every column and row and every line parallel to the diagonals of the array. For which n is this information sufficient to determine the exact positions of the girls in the array? For which seats can we say for sure that a girl sits there or not? 19. (FRA 1) Let a1 , . . . , an be distinct positive integers that do not contain a 9 in their decimal representations. Prove that 1 1 + ···+ ≤ 30. a1 an 20. (FRA 2) (SL89-8). 220 3 Problems 21. (FRA 2b) Same problem as previous, but with a rectangular parallelepiped having at least one integral side. 22. (FRA 3) Let ABC be an equilateral triangle with side length equal to a natural number N. Consider the set S of all points M inside the triangle ABC such that −→ 1 − → − → AM = N (nAB + mAC), where m, n are integers and 0 ≤ m, n, m + n ≤ N. Every point of S is colored in one of the three colors blue, white, red such that no point on AB is colored blue, no point on AC is colored white, and no point on BC is colored red. Prove that there exists an equilateral triangle with vertices in S and side length 1 whose three vertices are colored blue, white, and red. 23. (FRA 3b) Like the previous problem, but with a regular tetrahedron and four different colors used. 24. (FRA 4) (SL89-9). 25. (UNK 1) Let ABC be a triangle. Prove that there is a unique point U in the plane of ABC such that there exist real numbers λ , µ , ν , κ , not all zero, such that λ PL2 + µ PM 2 + ν PN 2 − κ UP2 is constant for all points P of the plane, where L, M, N are the feet of the perpendiculars from P to BC,CA, AB respectively. 26. (UNK 2) Let a, b, c, d be positive integers such that ab = cd and a + b = c − d. Prove that there exists a right-angled triangle the measures of whose sides (in some unit) are integers and whose area measure is ab square units. 27. (UNK 3) Integers cm,n (m ≥ 0, n ≥ 0) are defined by cm,0 = 1 for all m ≥ 0, c0,n = 1 for all n ≥ 0, and cm,n = cm−1,n − ncm−1,n−1 for all m > 0, n > 0. Prove that cm,n = cn,m for all m ≥ 0, n ≥ 0. 28. (UNK 4) Let b1 , b2 , . . . , b1989 be positive real numbers such that the equations xr−1 − 2xr + xr+1 + br xr = 0 (1 ≤ r ≤ 1989) have a solution with x0 = x1990 = 0 but not all of x1 , . . . , x1989 are equal to zero. Prove that 2 b1 + b2 + · · · + b1989 ≥ . 995 29. (HEL 1) Let L denote the set of all lattice points of the plane (points with integral coordinates). Show that for any three points A, B,C of L there is a fourth point D, different from A, B,C, such that the interiors of the segments AD, BD,CD contain no points of L. Is the statement true if one considers four points of L instead of three? 30. (HEL 2) In a triangle ABC for which 6(a + b + c)r2 = abc, we consider a point M on the inscribed circle and the projections D, E, F of M on the sides BC, AC, and AB respectively. Let S, S1 denote the areas of the triangles ABC and DEF respectively. Find the maximum and minimum values of the quotient SS (here r 1 denotes the inradius of ABC and, as usual, a = BC, b = AC, c = AB). 3.30 IMO 1989 221 31. (HEL 3) (SL89-10). 32. (HKG 1) Let ABC be an equilateral triangle. Let D, E, F, M, N, and P be the midpoints of BC, CA, AB, FD, FB, and DC respectively. (a) Show that the line segments AM, EN, and FP are concurrent. (b) Let O be the point of intersection of AM, EN, and FP. Find OM : OF : ON : OE : OP : OA. √ √ √ 33. (HKG 2) Let n be a positive integer. Show that ( 2 + 1)n = m + m − 1 for some positive integer m. 34. (HKG 3) Given an acute triangle find a point inside the triangle such that the sum of the distances from this point to the three vertices is the least. 35. (HKG 4) Find all square numbers S1 and S2 such that S1 − S2 = 1989. 36. (HKG 5) Prove the identity 159 1 2 1 1 2 1 1 2 641 1 + − + + − + ···+ + − =2∑ . 2 3 4 5 6 478 479 480 (161 + k)(480 − k) k=0 37. (HUN 1) (SL89-11). 38. (HUN 2) Connecting the vertices of a regular n-gon we obtain a closed (not necessarily convex) n-gon. Show that if n is even, then there are two parallel segments among the connecting segments and if n is odd then there cannot be exactly two parallel segments. 39. (HUN 3) (SL89-12). 40. (ISL 1) A sequence of real numbers x0 , x1 , x2 , . . . is defined as follows: x0 = 1989 and for each n ≥ 1 1989 n−1 xn = − ∑ xk . n k=0 n Calculate the value of ∑1989 n=0 2 xn . 41. (ISL 2) Alice has two urns. Each urn contains four balls and on each ball a natural number is written. She draws one ball from each urn at random, notes the sum of the numbers written on them, and replaces the balls in the urns from which she took them. This she repeats a large number of times. Bill, on examining the numbers recorded, notices that the frequency with which each sum occurs is the same as if it were the sum of two natural numbers drawn at random from the range 1 to 4. What can he deduce about the numbers on the balls? 42. (ISL 3) (SL89-13). 43. (IDN 1) Let f (x) = a sin2 x + b sin x + c, where a, b, and c are real numbers. Find all values of a, b, and c such that the following three conditions are satisfied simultaneously: (i) f (x) = 381 if sin x = 1/2. (ii) The absolute maximum of f (x) is 444. 222 3 Problems (iii) The absolute minimum of f (x) is 364. 44. (IDN 2) Let A and B be fixed distinct points on the X axis, none of which coincides with the origin O(0, 0), and let C be a point on the Y axis of an orthogonal Cartesian coordinate system. Let g be a line through the origin O(0, 0) and perpendicular to the line AC. Find the locus of the point of intersection of the lines g and BC as C varies along the Y axis. (Give an equation and a description of the locus.) 45. (IDN 3) The expressions a + b + c, ab + ac + bc, and abc are called the elementary symmetric expressions on the three letters a, b, c; symmetric because if we interchange any two letters, say a and c, the expressions remain algebraically the same. The common degree of its terms is called the order of the expression. Let Sk (n) denote the elementary expression on k different letters of order n; for example S4 (3) = abc + abd + acd + bcd. There are four terms in S4 (3). How many terms are there in S9891 (1989)? (Assume that we have 9891 different letters.) 46. (IDN 4) Given two distinct numbers b1 and b2 , their product can be formed in two ways: b1 × b2 and b2 × b1 . Given three distinct numbers, b1 , b2 , b3 , their product can be formed in twelve ways: b1 × (b2 × b3 ); (b1 × b2 ) × b3 ; b1 × (b3 × b2 ); (b1 × b3 ) × b2 ; b2 × (b1 × b3 ); (b2 × b1 ) × b3 ; b2 × (b3 × b1 ); (b2 × b3 ) × b1 ; b3 × (b1 × b2 ); (b3 × b1 ) × b2 ; b3 × (b2 × b1 ); (b3 × b2 ) × b1 . In how many ways can the product of n distinct letters be formed? 47. (IDN 5) Let log22 x − 4 log2 x − m2 − 2m − 13 = 0 be an equation in x. Prove: (a) For any real value of m the equation has has two distinct solutions. (b) The product of the solutions of the equation does not depend on m. (c) One of the solutions of the equation is less than 1, while the other solution is greater than 1. Find the minimum value of the larger solution and the maximum value of the smaller solution. 48. (IDN 6) Let S be the point of intersection of the two lines l1 : 7x − 5y + 8 = 0 and l2 : 3x + 4y − 13 = 0. Let P = (3, 7), Q = (11, 13), and let A and B be points on the line PQ such that P is between A and Q, and B is between P and Q, and such that PA/AQ = PB/BQ = 2/3. Without finding the coordinates of B find the equations of the lines SA and SB. 49. (IND 1) Let A, B denote two distinct fixed points in space. Let X, P denote variable points (in space), while K, N, n denote positive integers. Call (X , K, N, P) admissible if (N − K)PA + K · PB ≥ N · PX. Call (X , K, N) admissible if (X, K, N, P) is admissible for all choices of P. Call (X, N) admissible if (X, K, N) is admissible for some choice of K in the interval 0 < K < N. Finally, call X admissible if (X, N) is admissible for some choice of N (N > 1). Determine: (a) the set of admissible X ; (b) the set of X for which (X , 1989) is admissible but not (X , n), n < 1989. 3.30 IMO 1989 223 50. (IND 2) (SL89-14). 51. (IND 3) Let t(n), for n = 3, 4, 5, . . ., represent the number of distinct, incongruent, integer-sided triangles whose perimeter is n; e.g., t(3) = 1. Prove that hni hn i t(2n − 1) − t(2n) = or +1 . 6 6 52. (IRL 1) (SL89-15). 53. (IRL 2) Let f (x) = (x−a1 )(x−a2 ) · · · (x−an )−2, where n ≥ 3 and a1 , a2 , . . . , an are distinct integers. Suppose that f (x) = g(x)h(x), where g(x), h(x) are both nonconstant polynomials with integer coefficients. Prove that n = 3. 54. (IRL 3) Let f be a function from the real numbers to the real numbers such that f (1) = 1, f (a + b) = f (a) + f (b) for all a, b, and f (x) f (1/x) = 1 for all x 6= 0. Prove that f (x) = x for all real numbers x. 55. (IRL 4) Let [x] denote the greatest integer less than or equal to x. Let α be the positive root of the equation x2 − 1989x − 1 = 0. Prove that there exist infinitely many natural numbers n that satisfy the equation [α n + 1989α [α n]] = 1989n + (19892 + 1)[α n]. 56. (IRL 5) Let n = 2k − 1, where k ≥ 6 is an integer. Let T be the set of all ntuples (x1 , x2 , . . . , xn ) where xi is 0 or 1 (i = 1, 2, . . . , n). For x = (x1 , . . . , xn ) and y = (y1 , . . . , yn ) in T , let d(x, y) denote the number of integers j with 1 ≤ j ≤ n such that x j 6= y j . (In particular d(x, x) = 0.) Suppose that there exists a subset S of T with 2k elements that has the following property: Given any element x in T , there is a unique element y in S with d(x, y) ≤ 3. Prove that n = 23. 57. (ISR 1) (SL89-16). 58. (ISR 2) Let P1 (x), P2 (x), . . . , Pn (x) be polynomials with real coefficients. Show that there exist real polynomials Ar (x), Br (x) (r = 1, 2, 3) such that ∑ns=1 (Ps (x))2 = (A1 (x))2 + (B1 (x))2 = (A2 (x))2 + x(B2 (x))2 = (A3 (x))2 − x(B3 (x))2 . 59. (ISR 3) Let v1 , v2 , . . . , v1989 be a set of coplanar vectors with |vr | ≤ 1 for 1 ≤ r ≤ 1989. Show that it is possible to find εr (1 ≤ r ≤ 1989), each equal to ±1, such that 1989 √ ∑ εr vr ≤ 3. r=1 60. (KOR 1) A real-valued function f on Q satisfies the following conditions for arbitrary α , β ∈ Q: (i) f (0) = 0, (iii) f (αβ ) = f (α ) f (β ), (v) f (m) ≤ 1989 for all m ∈ Z. (ii) f (α ) > 0 if α 6= 0, (iv) f (α + β ) ≤ f (α ) + f (β ), 224 3 Problems Prove that f (α + β ) = max{ f (α ), f (β )} if f (α ) 6= f (β ). Here, Z, Q denote the sets of integers and rational numbers, respectively. 61. (KOR 2) Let A be a set of positive integers such that no positive integer greater than 1 divides all the elements of A. Prove that any sufficiently large positive integer can be written as a sum of elements of A. (Elements may occur several times in the sum.) 62. (KOR 3) (SL89-25). 63. (KOR 4) (SL89-26). 64. (KOR 5) Let a regular (2n + 1)-gon be inscribed in a circle of radius r. We consider all the triangles whose vertices are from those of the regular (2n + 1)gon. (a) How many triangles among them contain the center of the circle in their interior? (b) Find the sum of the areas of all those triangles that contain the center of the circle in their interior. 65. (LUX 1) A regular n-gon A1 A2 A3 . . . Ak . . . An inscribed in a circle of radius R is given. If S is a point on the circle, calculate T = SA21 + SA22 + · · · + SA2n . 66. (MNG 1) (SL89-17). 67. (MNG 2) A family of sets A1 , A2 , . . . , An has the following properties: (i) Each Ai contains 30 elements. (ii) Ai ∩ A j contains exactly one element for all i, j, 1 ≤ i < j ≤ 30. Find the largest possible n if the intersection of all these sets is empty. 68. (MNG 3) If 0 < k ≤ 1 and ai are positive real numbers, i = 1, 2, . . . , n, prove that  a1 a2 + · · · + a n k  an + ···+ a1 + · · · + an−1 k ≥ n . (n − 1)k 69. (MNG 4) (SL89-18). 70. (MNG 5) Three mutually nonparallel lines li (i = 1, 2, 3) are given in a plane. The lines li determine a triangle and reflections fi with axes on lines li . Prove that for every point of the plane, there exists a finite composition of the reflections fi that maps that point to a point interior to the triangle. 71. (MNG 6) (SL89-19). 72. (MAR 1) Let ABCD be a quadrilateral inscribed in a circle with diameter AB √ 3 5−1 such that BC = a, CD = 2a, DA = 2 a. For each point M on the semicircle AB not containing C and D, denote by h1 , h2 , h3 the distances from M to the sides BC, CD, and DA. Find the maximum of h1 + h2 + h3 . 73. (NLD 1) (SL89-20). 74. (NLD 2) (SL89-21). 3.30 IMO 1989 225 75. (PHI 1) (SL89-22). 76. (PHI 2) Let k and s be positive integers. For sets of real numbers {α1 , α2 , . . . , αs } j j and {β1 , β2 , . . . , βs } that satisfy ∑si=1 αi = ∑si=1 βi for each j = 1, 2, . . . , k, we write {α1 , α2 , . . . , αs } =k {β1 , β2 , . . . , βs }. Prove that if {α1 , α2 , . . . , αs } =k {β1 , β2 , . . . , βs } and s ≤ k, then there exists a permutation π of {1, 2, . . . , s} such that βi = απ (i) for i = 1, 2, . . . , s. 77. (POL 1) Given that cos x + cosy + cosz sin x + sin y + sinz = = a, cos(x + y + z) sin(x + y + z) show that cos(y + z) + cos(z + x) + cos(x + y) = a. 78. (POL 2) (SL89-23). Alternative formulation. Two identical packs of n different cards are shuffled together; all arrangements are equiprobable. The cards are then laid face up, one at a time. For every natural number n, find out which is more probable, that at least one pair of identical cards will appear in immediate succession or that there will be no such pair. 79. (POL 3) To each pair (x, y) of distinct elements of a finite set X a number f (x, y) equal to 0 or 1 is assigned in such a way that f (x, y) 6= f (y, x) for all x, y (x 6= y). Prove that exactly one of the following situations occurs: (i) X is the union of two disjoint nonempty subsets U,V such that f (u, v) = 1 for every u ∈ U, v ∈ V . (ii) The elements of X can be labeled x1 , . . . , xn so that f (x1 , x2 ) = f (x2 , x3 ) = · · · = f (xn−1 , xn ) = f (xn , x1 ) = 1. Alternative formulation. In a tournament of n participants, each pair plays one game (no ties). Prove that exactly one of the following situations occurs: (i) The league can be partitioned into two nonempty groups such that each player in one of these groups has won against each player of the other. (ii) All participants can be ranked 1 through n so that ith player wins the game against the (i + 1)st and the nth player wins against the first. 80. (POL 4) We are given a finite collection of segments in the plane, of total length 1. Prove that there exists a line ℓ such that the sum of the lengths of the projections of the given segments to the line ℓ is less than 2/π . 81. (POL 5) (SL89-24). 82. (POR 1) Solve in the set of real numbers the equation 3x3 − [x] = 3, where [x] denotes the integer part of x. 83. (POR 2) Poldavia is a strange kingdom. Its currency unit is the bourbaki and there exist only two types of coins: gold ones and silver ones. Each gold coin 226 3 Problems is worth n bourbakis and each silver coin is worth m bourbakis (n and m are positive integers). Using gold and silver coins, it is possible to obtain sums such as 10000 bourbakis, 1875 bourbakis, 3072 bourbakis, and so on. But Poldavia’s monetary system is not as strange as it seems: (a) Prove that it is possible to buy anything that costs an integral number of bourbakis, as long as one can receive change. (b) Prove that any payment above mn − 2 bourbakis can be made without the need to receive change. 84. (POR 3) Let a, b, c, r, and s be real numbers. Show that if r is a root of ax2 + bx + c = 0 and s is a root of −ax2 + bx + c = 0, then a2 x2 + bx + c = 0 has a root between r and s. 85. (POR 4) Let P(x) be a polynomial with integer coefficients such that P(m1 ) = P(m2 ) = P(m3 ) = P(m4 ) = 7 for given distinct integers m1 , m2 , m3 , and m4 . Show that there is no integer m such that P(m) = 14. 86. (POR 5) Given two natural numbers w and n, the tower of n w’s is the natural number Tn (w) defined by .. Tn (w) = ww .w , with n w’s on the right side. More precisely, T1 (w) = w and Tn+1 (w) = wTn (w) . 2 For example, T3 (2) = 22 = 16, T4 (2) = 216 = 65536, and T2 (3) = 33 = 27. Find the smallest tower of 3’s that exceeds the tower of 1989 2’s. In other words, find the smallest value of n such that Tn (3) > T1989 (2). Justify your answer. 87. (POR 6) A balance has a left pan, a right pan, and a pointer that moves along a graduated ruler. Like many other grocer balances, this one works as follows: An object of weight L is placed in the left pan and another of weight R in the right pan, the pointer stops at the number R − L on the graduated ruler. There are n (≥ 2) bags of coins, each containing n(n−1) + 1 coins. All coins 2 look the same (shape, color, and so on). Of the bags, n − 1 contain genuine coins, all with the same weight. The remaining bag (we don’t know which one it is) contains counterfeit coins. All counterfeit coins have the same weight, and this weight is different from the weight of the genuine coins. A legal weighing consists of placing a certain number of coins in one of the pans, putting a certain number of coins in the other pan, and reading the number given by the pointer in the graduated ruler. With just two legal weighings it is possible to identify the bag containing counterfeit coins. Find a way to do this and explain it. 88. (ROU 1) (SL89-27). 89. (ROU 2) (SL89-28). √ 90. (ROU 3) Prove that the sequence (an )n≥0 , an = [n 2], contains an infinite number of perfect squares. 91. (ROU 4) (SL89-29). 3.30 IMO 1989 227 92. (ROU 5) Find the set of all a ∈ R for which there is no infinite sequence (xn )n≥0 ⊂ R satisfying x0 = a, xn+1 = xn + α , n = 0, 1, . . . , where αβ > 0. β xn + 1 93. (ROU 6) For Φ : N → Z let us define MΦ = { f : N → Z; f (x) > f (Φ (x)), ∀x ∈ N}. (a) Prove that if MΦ1 = MΦ2 6= 0, / then Φ1 = Φ2 . (b) Does this property remain true if MΦ = { f : N → N; f (x) > f (Φ (x)), ∀x ∈ N}? 94. (SWE 1) Prove that a < b implies that a3 −3a ≤ b3 −3b+4. When does equality occur? 95. (SWE 2) (SL89-30). 96. (SWE 3) (SL89-31). 97. (THA 1) Let n be a positive integer, X = {1, 2, . . . , n}, and k a positive integer such that n/2 ≤ k ≤ n. Determine, with proof, the number of all functions f : X → X that satisfy the following conditions: (i) f 2 = f ; (ii) the number of elements in the image of f is k; (iii) for each y in the image of f , the number of all points x in X such that f (x)=y is at most 2. 98. (THA 2) Let f : N → N be such that (i) f is strictly increasing; (ii) f (mn) = f (m) f (n) ∀m, n ∈ N; and (iii) if m 6= n and mn = nm , then f (m) = n or f (n) = m. Determine f (30). 99. (THA 3) An arithmetic function is a real-valued function whose domain is the set of positive integers. Define the convolution product of two arithmetic functions f and g to be the arithmetic function f ⋆ g, where ( f ⋆ g)(n) = ∑ i j=n f (i)g( j), and f ⋆k = f ⋆ f ⋆ · · · ⋆ f (k times). We say that two arithmetic functions f and g are dependent if there exists a nontrivial polynomial of two variables P(x, y) = ∑i, j ai j xi y j with real coefficients such that P( f , g) = ∑ ai j f ⋆i ⋆ g⋆ j = 0, i, j and say that they are independent if they are not dependent. Let p and q be two distinct primes and set 228 3 Problems f1 (n) =  1 if n = p, 0 otherwise; f2 (n) =  1 if n = q, 0 otherwise. Prove that f1 and f2 are independent. 100. (THA 4) Let A be an n × n matrix whose elements are nonnegative real numbers. Assume that A is a nonsingular matrix and all elements of A−1 are nonnegative real numbers. Prove that every row and every column of A has exactly one nonzero element. 101. (TUR 1) Let ABC be an equilateral triangle and Γ the semicircle drawn exteriorly to the triangle, having BC as diameter. Show that if a line passing through A trisects BC, it also trisects the arc Γ . 102. (TUR 2) If in a convex quadrilateral ABCD, E and F are the midpoints of the sides BC and DA respectively. Show that the sum of the areas of the triangles EDA and FBC is equal to the area of the quadrangle. 103. (USA 1) An accurate 12-hour analog clock has an hour hand, a minute hand, and a second hand that are aligned at 12:00 o’clock and make one revolution in 12 hours, 1 hour, and 1 minute, respectively. It is well known, and not difficult to prove, that there is no time when the three hands are equally spaced around the clock, with each separating angle 2π /3. Let f (t), g(t), h(t) be the respective absolute deviations of the separating angles from 2π /3 at t hours after 12:00 o’clock. What is the minimum value of max{ f (t), g(t), h(t)}? 104. (USA 2) For each nonzero complex number z, let argz be the unique real number t such that −π < t ≤ π and z = |z|(cost + ı sint). Given a real number c > 0 and a complex number z 6= 0 with arg z 6= π , define B(c, z) = {b ∈ R | |w − z| < b ⇒ | argw − argz| < c}. Determine necessary and sufficient conditions, in terms of c and z, such that B(c, z) has a maximum element, and determine what this maximum element is in this case. 105. (USA 3) (SL89-32). 106. (USA 4) Let n > 1 be a fixed integer. Define functions f0 (x) = 0, f1 (x) = 1 − cos x, and for k > 0, fk+1 (x) = 2 fk (x) cos x − fk−1 (x). If F(x) = f1 (x) + f2 (x) + · · · + fn (x), prove that π (a) 0 < F(x) < 1 for 0 < x < n+1 , and π π (b) F(x) > 1 for n+1 < x < n . 107. (VNM 1) Let E be the set of all triangles whose only points with integer coordinates (in the Cartesian coordinate system in space), in its interior or on its sides, are its three vertices, and let f be the function of area of a triangle. Determine the set of values f (E) of f . 3.30 IMO 1989 229 108. (VNM 2) For every sequence (x1 , x2 , . . . , xn ) of the numbers {1, 2, . . . , n} arranged in any order, denote by f (s) the sum of absolute values of the differences between two consecutive members of s. Find the maximum value of f (s) (where s runs through the set of all such sequences). 109. (VNM 3) Let Ax, By be two noncoplanar rays with AB as a common perpendicular, and let M, N be two mobile points on Ax and By respectively such that AM + BN = MN. First version. Prove that there exist infinitely many lines coplanar with each of the lines MN. Second version. Prove that there exist infinitely many rotations around a fixed axis ∆ mapping the line Ax onto a line coplanar with each of the lines MN. 110. (VNM 4) Do there exist two sequences of real numbers {ai }, {bi }, i ∈ N = {1, 2, 3, . . . }, satisfying the following conditions: 3π ≤ ai ≤ bi , 2 cos ai x + cosbi x ≥ − 1 i for all i ∈ N and all x, 0 < x < 1? 111. (VNM number c such that for all natural numbers n, √ 5) Find the greatest √ √ √ {n 2} ≥ nc (where {n 2} = n 2 − [n 2]; [x] is the integer part of x). For this √ number c, find all natural numbers n for which {n 2} = nc . 3.30.3 Shortlisted Problems 1. (AUS 2)IMO2 Let ABC be a triangle. The bisector of angle A meets the circumcircle of triangle ABC in A1 . Points B1 and C1 are defined similarly. Let AA1 meet the lines that bisect the two external angles at B and C in point A0 . Define B0 and C0 similarly. If SX1 X2 ...Xn denotes the area of the polygon X1 X2 . . . Xn , prove that SA0 B0C0 = 2SAC1 BA1CB1 ≥ 4SABC . 2. (AUS 3) Ali Barber, the carpet merchant, has a rectangular piece of carpet whose dimensions are unknown. Unfortunately, his tape measure is broken and he has no other measuring instruments. However, he finds that if he lays it flat on the floor of either of his storerooms, then each corner of the carpet touches a different wall of that room. If the two rooms have dimensions of 38 feet by 55 feet and 50 feet by 55 feet, what are the carpet dimensions? 3. (AUS 4) Ali Barber, the carpet merchant, has a rectangular piece of carpet whose dimensions are unknown. Unfortunately, his tape measure is broken and he has no other measuring instruments. However, he finds that if he lays it flat on the floor of either of his storerooms, then each corner of the carpet touches a different wall of that room. He knows that the sides of the carpet are integral numbers of feet and that his two storerooms have the same (unknown) length, but widths of 38 feet and 50 feet respectively. What are the carpet dimensions? 230 3 Problems 4. (BGR 3) Prove that for every integer n > 1 the equation xn xn−1 x2 x + + ···+ + + 1 = 0 n! (n − 1)! 2! 1! has no rational roots. 5. (COL 1) Consider the polynomial p(x) = xn + nxn−1 + a2 xn−2 + · · · + an having all real roots. If r116 + r216 + · · · + rn16 = n, where the r j are the roots of p(x), find all such roots. 6. (CZS 1) For a triangle ABC, let k be its circumcircle with radius r. The bisectors of the inner angles A, B, and C of the triangle intersect respectively the circle k again at points A′ , B′ , and C′ . Prove the inequality 16Q3 ≥ 27r4 P, where Q and P are the areas of the triangles A′ B′C′ and ABC respectively. 7. (FIN 1) Show that any two points lying inside a regular n-gon E can be joined  by two circular arcs lying inside E and meeting at an angle of at least 1 − 2n π . 8. (FRA 2) Let R be a rectangle that is the union of a finite number of rectangles Ri , 1 ≤ i ≤ n, satisfying the following conditions: (i) The sides of every rectangle Ri are parallel to the sides of R. (ii) The interiors of any two different Ri are disjoint. (iii) Every Ri has at least one side of integral length. Prove that R has at least one side of integral length. 9. (FRA 4) For all integers n, n ≥ 0, there exist uniquely determined integers an , bn , cn such that  n √ √ √ √ 3 3 3 3 1 + 4 2 − 4 4 = an + bn 2 + cn 4. Prove that cn = 0 implies n = 0. 10. (HEL 3) Let g : C → C, w ∈ C, a ∈ C, w3 = 1 (w 6= 1). Show that there is one and only one function f : C → C such that f (z) + f (wz + a) = g(z), z ∈ C. Find the function f . 11. (HUN 1) Define sequence an by ∑d|n ad = 2n . Show that n|an . 12. (HUN 3) At n distinct points of a circular race course there are n cars ready to start. Each car moves at a constant speed and covers the circle in an hour. On hearing the initial signal, each of them selects a direction and starts moving immediately. If two cars meet, both of them change directions and go on without loss of speed. Show that at a certain moment each car will be at its starting point. 3.30 IMO 1989 231 13. (ISL 3)IMO4 The quadrilateral ABCD has the following properties: (i) AB = AD + BC; (ii) there is a point P inside it at a distance x from the side CD such that AP = x + AD and BP = x + BC. Show that 1 1 1 √ ≥√ +√ . x BC AD 14. (IND 2) A bicentric quadrilateral is one that is both inscribable in and circumscribable about a circle. Show that for such a quadrilateral, the centers of the two associated circles are collinear with the point of intersection of the diagonals. 15. (IRL 1) Let a, b, c, d, m, n be positive integers such that a2 + b2 + c2 + d 2 = 1989, a + b + c + d = m2 , and the largest of a, b, c, d is n2 . Determine, with proof, the values of m and n. 16. (ISR 1) The set {a0 , a1 , . . . , an } of real numbers satisfies the following conditions: (i) a0 = an = 0; (ii) for 1 ≤ k ≤ n − 1, n−1 ak = c + ∑ ai−k (ai + ai+1 ). i=k Prove that c ≤ 1 4n . 17. (MNG 1) Given seven points in the plane, some of them are connected by segments so that: (i) among any three of the given points, two are connected by a segment; (ii) the number of segments is minimal. How many segments does a figure satisfying (i) and (ii) contain? Give an example of such a figure. 18. (MNG 4) Given a convex polygon A1 A2 . . . An with area S and a point M in the same plane, determine the area of polygon M1 M2 . . . Mn , where Mi is the image of M under rotation RAαi around Ai by α , i = 1, 2, . . . , n. 19. (MNG 6) A positive integer is written in each square of an m × n board. The allowed move is to add an integer k to each of two adjacent numbers in such a way that no negative numbers are obtained. (Two squares are adjacent if they have a common side.) Find a necessary and sufficient condition for it to be possible for all the numbers to be zero by a finite sequence of moves. 20. (NLD 1)IMO3 Given a set S in the plane containing n points and satisfying the conditions: (i) no three points of S are collinear, (ii) for every point P of S there exist at least k points in S that have the same distance to P, 232 3 Problems prove that the following inequality holds: k< 1 √ + 2n. 2 21. (NLD 2) Prove that the intersection of a plane and a regular tetrahedron can be an obtuse-angled triangle and that the obtuse angle in any such triangle is always smaller than 120◦. 22. (PHI 1)IMO1 Prove that the set {1, 2, . . ., 1989} can be expressed as the disjoint union of 17 subsets A1 , A2 , . . . , A17 such that: (i) each Ai contains the same number of elements; (ii) the sum of all elements of each Ai is the same for i = 1, 2, . . . , 17. 23. (POL 2)IMO6 We consider permutations (x1 , . . . , x2n ) of the set {1, . . . , 2n} such that |xi − xi+1 | = n for at least one i ∈ {1, . . ., 2n − 1}. For every natural number n, find out whether permutations with this property are more or less numerous than the remaining permutations of {1, . . . , 2n}. 24. (POL 5) For points A1 , . . . , A5 on the sphere of radius 1, what is the maximum value that min1≤i, j≤5 Ai A j can take? Determine all configurations for which this maximum is attained. (Or: determine the diameter of any set {A1 , . . . , A5 } for which this maximum is attained.) 25. (KOR 3) Let a, b be integers that are not perfect squares. Prove that if x2 − ay2 − bz2 + abw2 = 0 has a nontrivial solution in integers, then so does x2 − ay2 − bz2 = 0. 26. (KOR 4) Let n be a positive integer and let a, b be given real numbers. Determine the range of x0 for which n ∑ xi = a i=0 n and ∑ x2i = b, i=0 where x0 , x1 , . . . , xn are real variables. 27. (ROU 1) Let m be a positive odd integer, m ≥ 2. Find the smallest positive integer n such that 21989 divides mn − 1. 28. (ROU 2) Consider in a plane Π the points O, A1 , A2 , A3 , A4 such that σ (OAi A j ) ≥ 1 for all i, j = 1, 2, 3, 4, i 6= j.√Prove that there is at least one pair i0 , j0 ∈ {1, 2, 3, 4} such that σ (OAi0 A j0 ) ≥ 2. (We have denoted by σ (OAi A j ) the area of triangle OAi A j .) 29. (ROU 4) A flock of 155 birds sit down on a circle C. Two birds Pi , Pj are mutually visible if m(Pi Pj ) ≤ 10◦ . Find the smallest number of mutually visible pairs of birds. (One assumes that a position (point) on C can be occupied simultaneously by several birds.) 3.30 IMO 1989 233 30. (SWE 2)IMO5 For which positive integers n does there exist a positive integer N such that none of the integers 1 + N, 2 + N, . . . , n + N is the power of a prime number? 31. (SWE 3) Let a1 ≥ a2 ≥ a3 be given positive integers and let N(a1 , a2 , a3 ) be the number of solutions (x1 , x2 , x3 ) of the equation a1 a 2 a 3 + + = 1, x1 x2 x3 where x1 , x2 , and x3 are positive integers. Show that N(a1 , a2 , a3 ) ≤ 6a1 a2 (3 + ln(2a1 )). 32. (USA 3) The vertex A of the acute triangle ABC is equidistant from the circumcenter O and the orthocenter H. Determine all possible values for the measure of angle A. 234 3 Problems 3.31 The Thirty-First IMO Beijing, China, July 8–19, 1990 3.31.1 Contest Problems First Day (July 12) 1. Given a circle with two chords AB,CD that meet at E, let M be a point of chord AB other than E. Draw the circle through D, E, and M. The tangent line to the circle DEM at E meets the lines BC, AC at F, G, respectively. Given AM AB = λ , GE find EF . 2. On a circle, 2n − 1 (n ≥ 3) different points are given. Find the minimal natural number N with the property that whenever N of the given points are colored black, there exist two black points such that the interior of one of the corresponding arcs contains exactly n of the given 2n − 1 points. 3. Find all positive integers n having the property that 2n +1 n2 is an integer. Second Day (July 13) 4. Let Q+ be the set of positive rational numbers. Construct a function f : Q+ → Q+ such that f (x) f (x f (y)) = , for all x, y in Q+ . y 5. Two players A and B play a game in which they choose numbers alternately according to the following rule: At the beginning, an initial natural number n0 > 1 is given. Knowing n2k , player A may choose any n2k+1 ∈ N such that n2k ≤ n2k+1 ≤ n22k . Then player B chooses a number n2k+2 ∈ N such that n2k+1 = pr , n2k+2 where p is a prime number and r ∈ N. It is stipulated that player A wins the game if he (she) succeeds in choosing the number 1990, and player B wins if he (she) succeeds in choosing 1. For which natural numbers n0 can player A manage to win the game, for which n0 can player B manage to win, and for which n0 can players A and B each force a tie? 6. Is there a 1990-gon with the following properties (i) and (ii)? (i) All angles are equal; (ii) The lengths of the 1990 sides form a permutation of the numbers 12 , 22 , . . ., 19892 , 19902. 3.31 IMO 1990 235 3.31.2 Longlisted Problems 1. (AUS 1) In triangle ABC, point O is the circumcenter, H is the orthocenter. Denote by A1 , B1 , and C1 the circumcenters of the triangles CHB, CHA, and AHB respectively. Prove that the triangles ABC and A1 B1C1 are congruent and that their nine-point circles coincide. 2. (AUS 2) Prove that        1 1990 1 1989 1 1988 1990 · − + 1990 0 1989 1 1988 2     m (−1) 1990 − m 1 995 −···+ + ···− + 1 = 0. 1990 − m m 995 995 3. (AUS 3) (SL90-1) 4. (CAN 1) (SL90-2) 5. (COL 1) Let b be a positive integer. Assume that there exist exactly 1990 triangles ABC with integral side-lengths satisfying the following conditions: (i) ∠ABC = 12 ∠BAC; (ii) AC = b. Find the minimal value for b. 6. (COL 2) Assume that the function f : (Z+ )3 → N satisfy the following conditions: (i) f (0, 0, 0) = 1; (ii) f (x, y, z) = f (x − 1, y, z) + f (x, y − 1, z) + f (x, y, z − 1); (iii) When applying the above relation iteratively, if any of x′ , y′ , z′ is negative, then f (x′ , y′ , z′ ) = 0. Prove that if x, y, z are the side lengths of a triangle, then integers for any integers k, m > 1. ( f (x,y,z))k f (mx,my,mz) is not an 7. (CUB 1) Let A and B be two points in the plane α , and let r be the line passing through A and B. There are n distinct points P1 , P2 , . √ . . , Pn in one of the halfplanes divided by the line r. Prove that there are at least n distinct values among the distances AP1 , AP2 , . . . , APn , BP1 , BP2 , . . . , BPn . 8. (CZS 1) (SL90-3) 9. (CZS 2) (SL90-4) 10. (CZS 3) Let p, k, and x be positive integers such that p > k and x < h p(p−k+1) 2(k−1) i , where [q] denotes the largest integer not greater than q. Prove that when x balls are put into p boxes arbitrarily, there exist k boxes with the same number of balls. 11. (CZS 4) In a group of mathematicians, every mathematician has some friends (friendship is symmetrical relation). Prove that there exists a mathematician, such that the average of the numbers of friends of all his friends is not less than the average of the number of friends of all the mathematicians. 236 3 Problems 12. (CZS 5) For any permutation p of the set {1, 2, . . .,n} define d(p) = |p(1) − 1| + |p(2) − 2| + · · · + |p(n) − n|. Denote by i(p) the number of integer pairs (i, j) in the permutation p such that 1 ≤ i < j ≤ n and p(i) > p( j). Find all real numbers c such that the inequality i(p) ≤ cd(p) holds for any positive integer n and any permutation p. 13. (FIN 1) Six cities A, B, C, D, E, and F are located at the vertices of a regular hexagon in that order. Let G be the center of the hexagon. The sides of the hexagon are the roads connecting these cities. Furthermore, there are roads connecting the cities B, C, E, F, and G. Because of raining, one or more of the roads may be destroyed. The probability of each road remaining undestroyed is equal to p. Determine the probability that it is possible to travel between the cities A and D. 14. (FIN 2) We call a set S ⊆ R superinvariant, if for any stretching A of the set S by the transformation taking x to A(x) = x0 + a(x−x0) (here a > 0 is a real number), there exists a transformation B, B(x) = x + b such that the images of S under A and B agree: i.e. for any x ∈ S, there exists y ∈ S such that A(x) = B(y) and for any t ∈ S there is u ∈ S such that B(t) = A(u). Determine all superinvariant sets. 15. (FRA 1) (SL90-5) 16. (FRA 2) We say that an integer k ≥ 1 has property P, if there exists at least one integer m ≥ 1 which cannot be expressed in the form m = ε1 z1 + ε2 zk2 + ε2k zk2k , where zi is nonnegative integer and εi = ±1, i = 1, 2, . . . , 2k. Prove that there are infinitely many integers k having the property P. 17. (FRA 3) 1990 mathematicians attend a meeting. Every mathematician has at least 1327 friends (friendship is symmetric relation). Prove that it is possible to find four mathematicians such that any two of them are friends. 18. (FRG 1) Find, with proof, the least positive integer n having the following property: All the binary representations of 1, 2, . . . , 1990 appear after the decimal point in the binary expression of 1/n. 19. (FRG 2) (SL90-6) 20. (FRG 3) Is it possible to express the three-dimensional space as a union of disjoint circles? 21. (HEL 1) Point O is in the interior of △ABC. Three lines through O parallel to BC, CA, and AB intersect the sides AB and AC at D and E; the sides BC and BA at F and G; and the sides CA and CB at H and I, respectively. 22. (HEL 2) (SL90-7) 23. (HUN 1) (SL90-8) 24. (HUN 2) Find the real number t such that the following system of equations has a unique real number solution (x, y, z, v): 3.31 IMO 1990 237 x+y+z+v = 0 (xy + yz + zv) + t(xz + xv + yv) = 0. 25. (HUN 3) (SL90-9) 26. (ISL 1) Prove that there exist infinitely many positive integers n such that 12 +22 +···+n2 is a perfect square. Obviously, 1 is the least integer having this propn erty. Find the next two least integers with this property. 27. (ISL 2) (SL90-10) 28. (IND 1) Let ABC be an acute-angled triangle. Assume that the circle Γ satisfies the following two conditions: (i) Γ intersects all three sides of △ABC. (ii) These points form a hexagon whose three pairs of opposite sides are parallel. (The hexagon may be degenerate if two or more vertices coincide. In this case opposite sides being parallel is defined through limit behavior.) Construct the locus of the centers of such circles Γ . 29. (IND 2) Function f (n), n ∈ N is defined as follows: Let A(n) and B(n) be coprime positive integers such that A(n) (2n)! = . B(n) n!(n + 1000)! If B(n) = 1 then f (n) = 1; if B(n) 6= 1 then f (n) is the largest prime factor of B(n). Prove that the set of values of f (n) is finite and find the maximum value for f (n). 30. (IND 3) (SL90-11) 31. (IND 4) Let S = {1, 2, . . . , 1990}. A 31-element subset of S is called good if the sum of its elements is divisible by 5. Find the number of good subsets of S. 32. (IRN 1) Using the following five figures is it possible to construct a parallelepiped whose side lengths are all integers greater than 1 and whose volume is 1990? In the following figure, every square represents a unit cube. 33. (IRN 2) Let S be a set with 1990 elements. Let P be a set of ordered sequences of 100 elements from S. If x = (. . . , a, . . . , b, . . .) ∈ P, for a, b ∈ S, then we say that the ordered pair (a, b) appears in x. Assume that any ordered pair from S appears in at most one element of P. Prove that P has at most 800 elements. 34. (IRN 3) There are n non-coplanar points in the space. Prove that there exists a circle that passes through exactly three of those points. 238 3 Problems 35. (IRN 4) Prove that if |x| < 1, then x x2 x3 x 2x2 3x3 + + + · · · = + + + ··· (1 − x)2 (1 + x2)2 (1 − x3 )2 1 − x 1 + x2 1 − x3 36. (IRL 1) (SL90-12) 37. (IRL 2) (SL90-13) 38. (IRL 3) Let α be a positive solution of the quadratic equation x2 = 1990x + 1. For every m, n ∈ N define the operation m ⋆ n = mn + [α m][α n], where [x] denotes the largest integer not exceeding x. Prove that (p ⋆ q) ⋆ r = p ⋆ (q ⋆ r) holds for all p, q, r ∈ N. 39. (IRL 4) Let a, b, c be integers. Prove that there are integers p1 , q1 , r1 , p2 , q2 , and r2 satisfying a = q1 r2 − q2 r1 , b = r1 p2 − r2 p1 , and c = p1 q2 − p2 q1 . 40. (ISR 1) Given three letters X , Y , Z, we can construct letter sequences such as XZ, ZZY XYY , XXY ZXX , etc. For any given sequence, one can perform the following operations: T1 If the right-most letter is Y , we add Y Z after it, for example: T1 (XY ZZXY ) = XY ZZXYY Z; T2 If the sequence contains YYY , this can be replaced by Z as in the following example: T2 (XXYY ZYYY X ) = X XYY ZZX; T3 X p can be replaced by X pX where p is any subsequence of letters: Example: T3 (XXY Z) = XXY ZX ; T4 In a sequence that contains one or more letters Z, we can replace the first Z by XY . Example: T4 (X XYY ZZX ) = X XYY XY ZX ; T5 We can replace any of X X , YY , ZZ by X , for example: T5 (ZZY XYY ) = XY XX , or T5 (ZZY XYY ) = XY XYY , or T5 (ZZY XYY ) = ZZY X X. Using the above operations is it possible to obtain XY ZZ starting from XY Z?  41. (ISR 2) Given a positive integer n, calculate Sn = ∑nr=0 2r−2n · 2n−r n . 42. (ITA 1) Find n points P1 , . . . , Pn on the circumference of a unit circle such that ∑1≤i< j≤n Pi Pj is maximal. 43. (ITA 2) Let V be a finite set of points in the three-dimensional space. Let S1 , S2 , S3 be the sets consisting of the orthogonal projections of the points of V onto the planes Oyz, Ozx, and Oxy respectively. Prove that |V |2 ≤ |S1 | · |S2 | · |S3 |, where |A| denotes the number of elements in the set A. 44. (ITA 3) Prove that for any positive integer n, the number of odd integers among  the binomial coefficients nk (0 ≤ k ≤ n) is a power of 2. 45. (ITA 4) A tourist is looking for a treasure on an island. The treasure is hidden behind the series of doors each of which is colored with one of n possible colors. The tourist has n keys, all of different colors. Each key can open any door, however, a key gets destroyed when it opens the door of the same color as the key itself (if it opens a door of some other color, it remains intact). Once the tourist 3.31 IMO 1990 239 starts using a particular key, she must continue using only that key until it gets destroyed. Find the least number of doors to ensure that no tourist can get the treasure, no matter how he chooses the order of keys. 46. (JPN 1) Let P be an interior point of triangle ABC. Let Q, R, S be the intersections of AP, BP, CP with sides BC, CA, AB respectively. Prove that SQRS ≤ 14 SABC . 47. (JPN 2) (SL90-14) 48. (JPN 3) Prove that √ √ √ 2 + 5 + 1990 is irrational. 49. (LUX 1) Let AB and AC be two chords of a circle with center O. The diameter perpendicular to BC intersects AB and AC at F and G respectively (F is inside the circle). Let T be the point on tangency of the circle and the tangent from G. Prove that F is the projection of T on OG. 50. (MEX 1) During the duration of the class, n children sit in a circle and play the following game: The teacher goes around the children in the clockwise direction and hands out candies according to the following rules: The teacher selects a child, gives him/her a candy as well to the child child next to him; then the teacher skips one child and gives a candy to the next one; then the teacher skips two children, gives a candy to the next; then skips over three children, ... Find the value of n such that the teacher ends up giving at least one candy to each of the children after finitely many steps. 51. (MEX 2) (SL90-15) 52. (MNG 1) Let a > 0 be a real number. Assume that real numbers a1 , . . . , an satisfy 0 < ai ≤ a for i = 1, 2, . . . , n. Prove that: (a) If n = 4, then 1 4 a1 a2 + a2 a 3 + a 3 a 4 + a 4 a 1 ≤ 2. ∑ ai − a i=1 a2 (b) If n = 6, then 1 6 a 1 a2 + a2 a3 + · · · + a 5 a6 + a6 a1 ai − ≤ 3. ∑ a i=1 a2 53. (MNG 2) Find all real solutions to the system of equations: x3 + y3 = 1, x5 + y5 = 1. 54. (MNG 3) Given a set M = {1, 2, . . . , n}, let φ : M → M be a bijection. (a) Prove that there are bijections φ1 , φ2 : M → M such that φ2 ◦ φ1 = φ and φ12 = φ22 = id, where id is the identity mapping. 240 3 Problems (b) Prove that the conclusion in (a) still holds if M is the set of all positive integers. 55. (MNG 4) Given points A, M, M1 , and a rational number λ 6= −1, construct a triangle ABC such that: M ∈ BC, M1 ∈ B1C1 , where B1 and C1 are the projections of B, C to AC and AB respectively, and BM B1 M1 = = λ. MC M1C1 56. (MAR 1) For positive integers n, p, n ≥ p, define real number Kn,p as follows: 1 Kn,0 = n+1 , Kn,p = Kn−1,p−1 − Kn,p−1 for 1 ≤ p ≤ n. (a) If Sn = ∑np=0 Kn,p , n = 0, 1, 2, . . ., find limn→∞ Sn . (b) Find Tn = ∑np=0 (−1) p Kn,p , n = 0, 1, 2, . . .. 57. (MAR 2) The sequence {un } is defined by u1 = 1, u2 = 1, un = un−1 + 2un−2, for n ≥ 3. (a) Prove that for any positive integers n, p (p > 1), un+p = un+1 u p + 2unu p−1 . (b) Find the greatest common divisor of un and un+3 . 58. (NLD 1) (SL90-16) 59. (NLD 2) Given eight real numbers a1 ≤ a2 ≤ · · · ≤ a7 ≤ a8 , let x = y= a21 +···+a28 . 8 a1 +···+a8 , 8 Prove that 2 60. (NLD 3) (SL90-17) p y − x2 ≤ a8 − a1 ≤ 4 p y − x2 . 61. (NLD 4) Prove that we can fill in the three dimensional space with regular tetrahedrons and regular octahedrons all of which have the same edge-lengths. Find the ratio of the number of regular tetrahedrons used to the number of regular octahedrons. 62. (NOR 1) (SL90-18) 63. (POL 1) (SL90-19) 64. (POL 2) Given an m-element set M and its k-element subset K ⊆ M, we say that a function f : K → M has a path, if there exists an element x0 ∈ K such that f (x0 ) = x0 , or there exists a chain x0 , x1 , . . . , x j = x0 ∈ K such that xi = f (xi−1 ), for i = 1, 2, . . . , j. Find the number of functions f : K → M that have paths. 65. (POL 3) (SL90-20) 66. (POL 4) Find all continuous bounded functions f : R → R such that ( f (x))2 − ( f (y))2 = f (x + y) f (x − y), for all x, y, ∈ R. 67. (PRK 1) Let a + bi and c + di be two roots of the equation xn = 1990 (n ≥ 3 is an integer). Assume that f (2, 1) = (1, 2) where f is the linear transformation: 3.31 IMO 1990 f=  241  ac . bd Denote by r the distance between the image of (2, 2) and the origin. Find the range for the values of r. 68. (PRK 2) A mobile point M starts from the origin O(0, 0) and moves along the line l with slope k, where k is an irrational number. (a) Prove that the point O(0, 0) is the only rational point (i.e. with both rational coordinates) on the line l. (b) Prove that for any number ε > 0 there are integers m and n such that the distance between l and the point (m, n) is less than ε . 69. (PRK 3) Consider the set of cuboids: three edges a, b, c from a common vertex 2 satisfy the condition: ab = ac5 . (a) Prove that there are 100 pairs of cuboids in this set with equal volumes in each pair. (b) For each pair of the above cuboids, find the ratio of the sum of their edges. 70. (PRK 4) Let M be a point on the side BC of a triangle ABC. (a) Prove that if M is the midpoint of BC, then AB2 + AC2 = 2(AM 2 + BM 2 ). (b) If there exists a point N ∈ BC different than M satisfying AB2 + AC2 = 2(AN 2 + BN 2 ), find the region that the point A might occupy. 71. (PRK 5) Given a point P = (p1 , . . . , pn ), find the point X = (x1 , . . . , xn ) satisfying x1 ≤ x2 ≤ · · · ≤ xn such that X minimizes the expression q (x1 − p1 )2 + · · · + (xn − pn )2 . 72. (KOR 1) Let n ≥ 5 be a positive integer. Let a1 , b1 , a2 , b2 , . . . , an , bn be integers such that the ordered (ai , bi ) are distinct for i = 1, 2, . . . , n and |a1 b2 − a2 b1 | = |a2 b3 − a3 b2 | = · · · = |an−1 bn − an bn−1 | = 1. Prove that there exists a pair of indices i and j that satisfy 2 ≤ |i − j| ≤ n − 2 and |ai b j − a j bi | = 1. Alternative formulation. Let n ≥ 5 be a positive integers and let P1 , . . . , Pn be the points with integral coordinates in the coordinate system with the origin O. The areas of the triangles OP1 P2 , OP2 P3 , . . . , OPn−1 Pn are equal to 12 . Prove that there exists a pair of integers i, j, such that 2 ≤ |i − j| ≤ n − 2 for which the area of △OPi Pj is equal to 12 . 73. (KOR 2) A function f : Q → R satisfies the following conditions: (i) f (0) = 0 and for every nonzero a ∈ Q, f (a) > 0; (ii) f (α + β ) = f (α ) f (β ); (iii) f (α + β ) ≤ max{ f (α ), f (β )}. Let x be an integer for which f (x) 6= 1. Prove that f (1 + x + · · · + xn ) = 1 for every positive integer n. 74. (KOR 3) Let L be a subset of the plane defined by L = {(41x + 2y, 59x + 15y) : x, y ∈ Z}. Prove that every parallelogram with center at the origin and area of 1990 contains at least two points of L. 242 3 Problems 75. (ROU 1) (SL90-21) 76. (ROU 2) Prove that there are at least two non-congruent cyclic quadrilaterals with equal areas and perimeters. 77. (ROU 3) Let a, b, c ∈ R. Prove that (a2 + ab + b2)(b2 + bc + c2)(c2 + ca + a2) ≥ (ab + bc + ca)3. When does the equality hold? 78. (ROU 4) (SL90-22) 79. (ROU 5) (SL90-23) 80. (ESP 1) Function f : N × N → Q satisfies the following conditions: (i) f (1, 1) = 1; (ii) f (p + 1, q) + f (p, q + 1) = f (p, q) for all p, q ∈ N; (iii) q f (p + 1, q) = p f (p, q + 1) for all p, q ∈ N. Find f (1990, 31). 81. (ESP 2) Circle k(K, ρ ) tangents the sides AB and BC of △ABC and intersects the side BC at points D and E. Let p be the distance from K to the side BC. (a) Prove that a(p − ρ ) = 2s(r − ρ ), where r is the inradius, s the semiperimeter of △ABC and a the length of the side BC. (b) Prove that p rr1 (ρ − r)(r1 − ρ ) DE = , r1 − r where r1 is the radius of the excircle of △ABC opposite to A. 82. (ESP 3) Define the symmedian Sa of triangle ABC as the line symmetric to the median from A with respect to the bisector of ∠CAB. Assume that the median ma intersects BC at A′ and the circumcircle at A1 . Assume that the symmedian Sa intersects BC at M and the circumcircle at A2 . Denote by O the circumcenter of △ABC. If A1 , O, and A2 are collinear, prove that: ′ b2 +c2 (a) AA AM = 2bc ; (b) (b2 + c2)2 + 4b2 c2 = a2 (b2 + c2 ). 83. (SWE 1) Point D lies on the hypothenuse BC of the right triangle ABC. The inradii of the triangles ADB and ADC are equal. Prove that SABC = AD2 . 84. (SWE 2) Let a1 , a2 , . . . , an ∈ (0, 2n) be n distinct integers (n ≥ 4). Prove that there exists a subset of the set {a1 , . . . , an } whose sum of elements is divisible by 2n. 85. (SWE 3) Let A1 , A2 , . . . , An (n ≥ 4) be n convex sets in plane. Given that every three of these sets have a common point, prove that there exists a point belonging to all the sets. 86. (SWE 4) Given a function f (x) = sin x + sin(π x) and a positive number d, prove that for every n ∈ N there exists a real number p such that p > n and | f (x + p) − f (x)| < d holds for all real numbers x. 3.31 IMO 1990 243 87. (THA 1) Let m be a positive odd integer not divisible by 3. Prove that h  √ m i 112 | 4m + 2 + 2 . 88. (THA 2) (SL90-24) 89. (THA 3) Let n be a positive integer. Let S1 , . . . , Sn be pairwise non-intersecting sets such that Sk has exactly k elements (k = 1, 2, . . . , n). Denote S = S1 ∪ S2 ∪ · · · ∪ Sn . A function f : S → S maps all elements of Sk to a fixed element of Sk for k = 1, 2, . . . , n. Find the number of functions g : S → S satisfying f ◦ g ◦ f = f (i.e. f (g( f (x))) = f (x) for all x). 90. (TUR 1) Let P be a variable point on the circumference of a quarter-circle with radii OA, OB (∠AOB = 90◦ ). Let H be the projection of P on OA. Find the locus of the incenters of △HPO. 91. (TUR 2) Quadrilateral ABCD is circumscribed around the circle with center O. If AB = CD and M and K are the midpoints of BC and AD respectively prove that OM = OK. 92. (TUR 3) Let n be a positive integer and m = (n+1)(n+2) . There are n distinct lines 2 L1 , L2 , . . . , Ln in coordinate plane and m distinct points A1 , A2 , . . . , Am satisfying the following two conditions: (i) Any two of the lines are non-parallel. (ii) Any three lines are non-concurrent. (iii) Only A1 does not line on any line Lk and there are exactly k + 1 among the points A1 , . . . , Am that lie on line Lk (k = 1, 2, . . . , n). Prove that there exists a unique polynomial p(x, y) of degree n satisfying p(A1 ) = 1 and p(A j ) = 0 for j = 2, 3, . . . , m. 93. (TUR 4) (SL90-25) 94. (USA 1) Given an integer n > 1 and a real number t ≥ 1 let P be a parallelogram with vertices (0, 0), (0,t), (tF2n+1,tF2n ), (tF2n+1 ,tF2n + t), where Fn is the n-th term of the Fibonacci sequence defined by F0 = 1, F1 = 1, and Fm+1 = Fm + Fm−1 for m ≥ 1. Let L be the number of integral points (i.e. points whose all coordinates are integers) in the interior of P, and let M be the area of P. (a) Prove that for any integral point (a, b) there exists a unique pair of integers ( j, k) such that √ j(F √n+1 , Fn )√+ k(Fn , Fn−1 ) = (a, b). (b) Prove that L − M ≤ 2. 95. (USA 2) (SL90-26) 96. (USA 3) Given a triangle ABC, points X , Y , Z are on the sides BC, CA, AB, respectively, such that △XY Z ∼ △ABC with ∠X = ∠A, ∠Y = ∠B, and ∠Z = ∠C. Prove that the orthocenter of triangle XY Z is the circumcenter of the triangle ABC. 97. (USA 4) Given a convex hexagon ABCDEF, assume that ∠BCA = ∠DEC = ∠AFB = ∠CBD = ∠EDG. Prove that AB = CD = EF. 244 3 Problems 98. (USS 1) (SL90-27) 99. (USS 2) Given a 10 × 10 chessboard colored in a standard way, prove that for any 46 unit squares without common edges one can choose 30 squares of the same color. 100. (USS 3) (SL90-28) 101. (USS 4) The side lengths of two equilateral triangles ABC and KLM are 1 and 1 4 respectively. The triangle KLM is located in the interior of the triangle ABC. Denote by Σ the sum of the distances from A to the lines KL, LM, and MK. Find the position of KLM that maximizes Σ . 102. (USS 5) We call a point (x, y) a lattice point of the coordinate plane if both x and y are integers. Knowing that the vertices of triangle ABC are all lattice points, and that there exists exactly one lattice point in the interior of △ABC (excluding the sides), prove that SABC ≤ 92 . 103. (VNM 1) Find the minimal value of the function q √ √ f (x) = 15 − 12 cosx + 4 − 2 3 sin x q q √ √ + 7 − 4 3 sin x + 10 − 4 3 sin x − 6 cosx. 104. (VNM 2) Let x, y, z ∈ R such that x ≥ y ≥ z > 0. Prove that x2 y y2 z z2 x + + ≥ x2 + y2 + z2 . z x y 105. (YUG 1) Let S and T respectively be the circumcenter and the centroid of the triangle ABC. If M is a point in the plane of △ABC such that 90◦ ≤ ∠SMT < 180◦ , denote by A1 , B1 , and C1 the intersections of AM, BM, CM with the circumcircle of the triangle ABC respectively. Prove that MA1 + MB1 + MC1 ≥ MA + MB + MC. 106. (YUG 2) Let S be the incenter of the triangle ABC. Let A1 , B1 , C1 be the intersections of AS, BS, and CS respectively with the circumcircle of the triangle ABC. Prove that SA1 + SB1 + SC1 ≥ SA + SB + SC. 107. (YUG 3) Let a, b, c, and P be the side lengths and the area of a triangle, respectively. Prove that  √   a2 + b2 + c2 − 4 3P · a2 + b2 + c2   ≥ 2 a2 (b − c)2 + b2 (c − a)2 + c2 (a − b)2 . 108. (YUG 4) Let (a1 , a2 , . . . an ) be a permutation of the set {1, 2, . . ., n}. Prove that 1 2 n − 1 a 1 a2 an−1 + + ···+ ≤ + + ···+ . 2 3 n a 2 a3 an 3.31 IMO 1990 245 3.31.3 Shortlisted Problems 1. (AUS 3) The integer 9 can be written as a sum of two consecutive integers: 9 = 4 + 5. Moreover, it can be written as a sum of (more than one) consecutive positive integers in exactly two ways: 9 = 4 + 5 = 2 + 3 + 4. Is there an integer that can be written as a sum of 1990 consecutive integers and that can be written as a sum of (more than one) consecutive positive integers in exactly 1990 ways? 2. (CAN 1) Given n countries with three representatives each, m committees A(1), A(2), . . . A(m) are called a cycle if (i) each committee has n members, one from each country; (ii) no two committees have the same membership; (iii) for i = 1, 2, . . . , m, committee A(i) and committee A(i + 1) have no member in common, where A(m + 1) denotes A(1); (iv) if 1 < |i − j| < m − 1, then committees A(i) and A( j) have at least one member in common. Is it possible to have a cycle of 1990 committees with 11 countries? 3. (CZS 1)IMO2 On a circle, 2n − 1 (n ≥ 3) different points are given. Find the minimal natural number N with the property that whenever N of the given points are colored black, there exist two black points such that the interior of one of the corresponding arcs contains exactly n of the given 2n − 1 points. 4. (CZS 2) Assume that the set of all positive integers is decomposed into r (disjoint) subsets A1 ∪A2 ∪ · · · Ar = N. Prove that one of them, say Ai , has the following property: There exists a positive m such that for any k one can find numbers a1 , a2 , . . . , ak in Ai with 0 < a j+1 − a j ≤ m (1 ≤ j ≤ k − 1). 5. (FRA 1) Given △ABC with no side equal to another side, let G, K, and H be its centroid, incenter, and orthocenter, respectively. Prove that ∠GKH > 90◦ . 6. (FRG 2)IMO5 Two players A and B play a game in which they choose numbers alternately according to the following rule: At the beginning, an initial natural number n0 > 1 is given. Knowing n2k , player A may choose any n2k+1 ∈ N such that n2k ≤ n2k+1 ≤ n22k . Then player B chooses a number n2k+2 ∈ N such that n2k+1 = pr , n2k+2 where p is a prime number and r ∈ N. It is stipulated that player A wins the game if he (she) succeeds in choosing the number 1990, and player B wins if he (she) succeeds in choosing 1. For which natural numbers n0 can player A manage to win the game, for which n0 can player B manage to win, and for which n0 can players A and B each force a tie? 7. (HEL 2) Let f (0) = f (1) = 0 and 246 3 Problems 2 f (n + 2) = 4n+2 f (n + 1) − 16n+1 f (n) + n · 2n , n = 0, 1, 2, 3, . . .. Show that the numbers f (1989), f (1990), f (1991) are divisible by 13. 8. (HUN 1) For a given positive integer k denote the square of the sum of its digits by f1 (k) and let fn+1 (k) = f1 ( fn (k)). Determine the value of f1991 (21990). 9. (HUN 3) The incenter of the triangle ABC is K. The midpoint of AB is C1 and that of AC is B1 . The lines C1 K and AC meet at B2 , the lines B1 K and AB at C2 . If the areas of the triangles AB2C2 and ABC are equal, what is the measure of angle ∠CAB? 10. (ISL 2) A plane cuts a right circular cone into two parts. The plane is tangent to the circumference of the base of the cone and passes through the midpoint of the altitude. Find the ratio of the volume of the smaller part to the volume of the whole cone. 11. (IND 3′ )IMO1 Given a circle with two chords AB,CD that meet at E, let M be a point of chord AB other than E. Draw the circle through D, E, and M. The tangent line to the circle DEM at E meets the lines BC, AC at F, G, respectively. GE Given AM AB = λ , find EF . 12. (IRL 1) Let ABC be a triangle and L the line through C parallel to the side AB. Let the internal bisector of the angle at A meet the side BC at D and the line L at E and let the internal bisector of the angle at B meet the side AC at F and the line L at G. If GF = DE, prove that AC = BC. 13. (IRL 2) An eccentric mathematician has a ladder with n rungs that he always ascends and descends in the following way: When he ascends, each step he takes covers a rungs of the ladder, and when he descends, each step he takes covers b rungs of the ladder, where a and b are fixed positive integers. By a sequence of ascending and descending steps he can climb from ground level to the top rung of the ladder and come back down to ground level again. Find, with proof, the minimum value of n, expressed in terms of a and b. 14. (JPN 2) In the coordinate plane a rectangle with vertices (0, 0), (m, 0), (0, n), (m, n) is given where both m and n are odd integers. The rectangle is partitioned into triangles in such a way that (i) each triangle in the partition has at least one side (to be called a “good” side) that lies on a line of the form x = j or y = k, where j and k are integers, and the altitude on this side has length 1; (ii) each “bad” side (i.e., a side of any triangle in the partition that is not a “good” one) is a common side of two triangles in the partition. Prove that there exist at least two triangles in the partition each of which has two good sides. 15. (MEX 2) Determine for which positive integers k the set X = {1990, 1990 + 1, 1990 + 2, . . ., 1990 + k} 3.31 IMO 1990 247 can be partitioned into two disjoint subsets A and B such that the sum of the elements of A is equal to the sum of the elements of B. 16. (NLD 1)IMO6 Is there a 1990-gon with the following properties (i) and (ii)? (i) All angles are equal; (ii) The lengths of the 1990 sides form a permutation of the numbers 12 , 22 , . . ., 19892 , 19902. 17. (NLD 3) Unit cubes are made into beads by drilling a hole through them along a diagonal. The beads are put on a string in such a way that they can move freely in space under the restriction that the vertices of two neighboring cubes are touching. Let A be the beginning vertex and B be the end vertex. Let there be p × q × r cubes on the string (p, q, r ≥ 1). (a) Determine for which values of p, q, and r it is possible to build a block with dimensions p, q, and r. Give reasons for your answers. (b) The same question as (a) with the extra condition that A = B.   18. (NOR) Let a, b be natural numbers with 1 ≤ a ≤ b, and M = a+b 2 . Define the function f : Z → Z by  n + a, if n < M, f (n) = n − b, if n ≥ M. Let f 1 (n) = f (n), f i+1 (n) = f ( f i (n)), i = 1, 2, . . . . Find the smallest natural number k such that f k (0) = 0. 19. (POL 1) Let P be a point inside a regular tetrahedron T of unit volume. The four planes passing through P and parallel to the faces of T partition T into 14 pieces. Let f (P) be the joint volume of those pieces that are neither a tetrahedron nor a parallelepiped (i.e., pieces adjacent to an edge but not to a vertex). Find the exact bounds for f (P) as P varies over T . 20. (POL 3) Prove that every integer k greater than 1 has a multiple that is less than k4 and can be written in the decimal system with at most four different digits. 21. (ROU 1′ ) Let n be a composite natural number and p a proper divisor of n. Find the binary representation of the smallest natural number N such that (1+2 p +2n−p )N−1 is an integer. 2n 22. (ROU 4) Ten localities are served by two international airlines such that there exists a direct service (without stops) between any two of these localities and all airline schedules offer round-trip service between the cities they serve. Prove that at least one of the airlines can offer two disjoint round trips each containing an odd number of landings. 23. (ROU 5)IMO3 Find all positive integers n having the property that integer. 2n +1 n2 is an 24. (THA 2) Let a, b, c, d be nonnegative real numbers such that ab+ bc+ cd +da = 1. Show that 248 3 Problems a3 b3 c3 d3 1 + + + ≥ . b+c+d a+c+d a+b+d a+b+c 3 25. (TUR 4)IMO4 Let Q+ be the set of positive rational numbers. Construct a function f : Q+ → Q+ such that f (x f (y)) = f (x) , y for all x, y in Q+ . 26. (USA 2) Let P be a cubic polynomial with rational coefficients, and let q1 , q2 , q3 , . . . be a sequence of rational numbers such that qn = P(qn+1 ) for all n ≥ 1. Prove that there exists k ≥ 1 such that for all n ≥ 1, qn+k = qn . 27. (USS 1) Find all natural numbers n for which every natural number whose decimal representation has n − 1 digits 1 and one digit 7 is prime. 28. (USS 3) Prove that in the coordinate plane it is impossible to draw a closed broken line satisfying the following conditions: (i) the coordinates of each vertex are rational; (ii) the length of each of its edges is 1; (iii) the line has an odd number of vertices. 3.32 IMO 1991 249 3.32 The Thirty-Second IMO Sigtuna, Sweden, July 12–23, 1991 3.32.1 Contest Problems First Day (July 17) 1. Prove for each triangle ABC the inequality 1 IA · IB · IC 8 < ≤ , 4 lA lB lC 27 where I is the incenter and lA , lB , lC are the lengths of the angle bisectors of ABC. 2. Let n > 6 and let a1 < a2 < . . . < ak be all natural numbers that are less than n and relatively prime to n. Show that if a1 , a2 , . . . , ak is an arithmetic progression, then n is a prime number or a natural power of two. 3. Let S = {1, 2, 3, . . ., 280}. Find the minimal natural number n such that in any n-element subset of S there are five numbers that are pairwise relatively prime. Second Day (July 18) 4. Suppose G is a connected graph with n edges. Prove that it is possible to label the edges of G from 1 to n in such a way that in every vertex v of G with two or more incident edges, the set of numbers labeling those edges has no common divisor greater than 1. 5. Let ABC be a triangle and M an interior point in ABC. Show that at least one of the angles ∡MAB, ∡MBC, and ∡MCA is less than or equal to 30◦ . 6. Given a real number a > 1, construct an infinite and bounded sequence x0 , x1 , x2 , . . . such that for all natural numbers i and j, i 6= j, the following inequality holds: |xi − x j ||i − j|a ≥ 1. 3.32.2 Shortlisted Problems 1. (PHI 3) Let ABC be any triangle and P any point in its interior. Let P1 , P2 be the feet of the perpendiculars from P to the two sides AC and BC. Draw AP and BP, and from C drop perpendiculars to AP and BP. Let Q1 and Q2 be the feet of these perpendiculars. Prove that the lines Q1 P2 , Q2 P1 , and AB are concurrent. 2. (JPN 5) For an acute triangle ABC, M is the midpoint of the segment BC, P is a point on the segment AM such that PM = BM, H is the foot of the perpendicular line from P to BC, Q is the point of intersection of segment AB and the line passing through H that is perpendicular to PB, and finally, R is the point of intersection of the segment AC and the line passing through H that is perpendicular to PC. Show that the circumcircle of △QHR is tangent to the side BC at point H. 250 3 Problems 3. (PRK 1) Let S be any point on the circumscribed circle of △PQR. Then the feet of the perpendiculars from S to the three sides of the triangle lie on the same straight line. Denote this line by l(S, PQR). Suppose that the hexagon ABCDEF is inscribed in a circle. Show that the four lines l(A, BDF), l(B, ACE), l(D, ABF), and l(E, ABC) intersect at one point if and only if CDEF is a rectangle. 4. (FRA 2)IMO5 Let ABC be a triangle and M an interior point in ABC. Show that at least one of the angles ∡MAB, ∡MBC, and ∡MCA is less than or equal to 30◦ . 5. (ESP 4) In the triangle ABC, with ∡A = 60◦ , a parallel IF to AC is drawn through the incenter I of the triangle, where F lies on the side AB. The point P on the side BC is such that 3BP = BC. Show that ∡BFP = ∡B/2. 6. (USS 4)IMO1 Prove for each triangle ABC the inequality 1 IA · IB · IC 8 < ≤ , 4 lA lB lC 27 where I is the incenter and lA , lB , lC are the lengths of the angle bisectors of ABC. 7. (CHN 2) Let O be the center of the circumsphere of a tetrahedron ABCD. Let L, M, N be the midpoints of BC,CA, AB respectively, and assume that AB + BC = AD +CD, BC +CA = BD + AD, and CA + AB = CD + BD. Prove that ∠LOM = ∠MNG = ∠NOL. 8. (NLD 1) Let S be a set of n points in the plane. No three points of S are collinear. Prove that there exists a set P containing 2n − 5 points satisfying the following condition: In the interior of every triangle whose three vertices are elements of S lies a point that is an element of P. 9. (FRA 3) In the plane we are given a set E of 1991 points, and certain pairs of these points are joined with a path. We suppose that for every point of E, there exist at least 1593 other points of E to which it is joined by a path. Show that there exist six points of E every pair of which are joined by a path. Alternative version. Is it possible to find a set E of 1991 points in the plane and paths joining certain pairs of the points in E such that every point of E is joined with a path to at least 1592 other points of E, and in every subset of six points of E there exist at least two points that are not joined? 10. (USA 5)IMO4 Suppose G is a connected graph with n edges. Prove that it is possible to label the edges of G from 1 to n in such a way that in every vertex v of G with two or more incident edges, the set of numbers labeling those edges has no common divisor greater than 1. 11. (AUS 4) Prove that   (−1)m 1991 − m 1 = . ∑ 1991 − m m 1991 m=0 995 3.32 IMO 1991 251 12. (CHN 3)IMO3 Let S = {1, 2, 3, . . ., 280}. Find the minimal natural number n such that in any n-element subset of S there are five numbers that are pairwise relatively prime. 13. (POL 4) Given any integer n ≥ 2, assume that the integers a1 , a2 , . . . , an are not divisible by n and, moreover, that n does not divide a1 + a2 + · · · + an . Prove that there exist at least n different sequences (e1 , e2 , · · · , en ) consisting of zeros or ones such that e1 a1 + e2 a2 + · · · + en an is divisible by n. 14. (POL 3) Let a, b, c be integers and p an odd prime number. Prove that if f (x) = ax2 + bx + c is a perfect square for 2p − 1 consecutive integer values of x, then p divides b2 − 4ac. 15. (USS 2) Let an be the last nonzero digit in the decimal representation of the number n!. Does the sequence a1 , a2 , . . . , an , . . . become periodic after a finite number of terms? 16. (ROU 1)IMO2 Let n > 6 and a1 < a2 < · · · < ak be all natural numbers that are less than n and relatively prime to n. Show that if a1 , a2 , . . . , ak is an arithmetic progression, then n is a prime number or a natural power of two. 17. (HKG 4) Find all positive integer solutions x, y, z of the equation 3x + 4y = 5z . 18. (BGR 1) Find the highest degree k of 1991 for which 1991k divides the number 19901991 1992 + 19921991 1990 . 19. (IRL 5) Let a be a rational number with 0 < a < 1 and suppose that cos 3π a + 2 cos2π a = 0. (Angle measurements are in radians.) Prove that a = 2/3. 20. (IRL 3) Let α be the positive root of the equation x2 = 1991x + 1. For natural numbers m, n define m ∗ n = mn + [α m][α n], where [x] is the greatest integer not exceeding x. Prove that for all natural numbers p, q, r, (p ∗ q) ∗ r = p ∗ (q ∗ r). 21. (HKG 6) Let f (x) be a monic polynomial of degree 1991 with integer coefficients. Define g(x) = f 2 (x) − 9. Show that the number of distinct integer solutions of g(x) = 0 cannot exceed 1995. 22. (USA 4) Real constants a, b, c are such that there is exactly one square all of whose vertices lie on the cubic curve y = x3 + ax2 + bx + c. Prove that the square √ 4 has sides of length 72. 23. (IND 2) Let f and g be two integer-valued functions defined on the set of all integers such that (i) f (m + f ( f (n))) = − f ( f (m + 1) − n for all integers m and n; 252 3 Problems (ii) g is a polynomial function with integer coefficients and g(n) = g( f (n)) for all integers n. Determine f (1991) and the most general form of g. 24. (IND 1) An odd integer n ≥ 3 is said to be “nice” if there is at least one permutation a1 , a2 , . . . , an of 1, 2, . . . , n such that the n sums a1 − a2 + a3 − · · · − an−1 + an , a2 − a 3 + a4 − · · · − a n + a1 , a3 − a 4 + a5 − · · · − a 1 + a2 , . . . , a n − a 1 + a2 − · · · − an−2 + an−1 are all positive. Determine the set of all “nice” integers. 25. (USA 1) Suppose that n ≥ 2 and x1 , x2 , . . . , xn are real numbers between 0 and 1 (inclusive). Prove that for some index i between 1 and n − 1 the inequality 1 xi (1 − xi+1 ) ≥ x1 (1 − xn ) 4 holds. 26. (CZS 1) Let n ≥ 2 be a natural number and let the real numbers p, a1 , a2 , . . ., an , b1 , b2 , . . ., bn satisfy 1/2 ≤ p ≤ 1, 0 ≤ ai , 0 ≤ bi ≤ p, i = 1, . . . , n, and ∑ni=1 ai = ∑ni=1 bi = 1. Prove the inequality n n i=1 j=1 j6=i ∑ bi ∏ a j ≤ p . (n − 1)n−1 27. (POL 2) Determine the maximum value of the sum ∑i< j xi x j (xi + x j ) over all n-tuples (x1 , . . . , xn ), satisfying xi ≥ 0 and ∑ni=1 xi = 1. 28. (NLD 2)IMO6 Given a real number a > 1, construct an infinite and bounded sequence x0 , x1 , x2 , . . . such that for all natural numbers i and j, i 6= j, the following inequality holds: |xi − x j ||i − j|a ≥ 1. 29. (FIN 2) We call a set S on the real line R superinvariant if for any stretching A of the set by the transformation taking x to A(x) = x0 + a(x − x0 ) there exists a translation B, B(x) = x + b, such that the images of S under A and B agree; i.e., for any x ∈ S there is a y ∈ S such that A(x) = B(y) and for any t ∈ S there is a u ∈ S such that B(t) = A(u). Determine all superinvariant sets. Remark. It is assumed that a > 0. 30. (BGR 3) Two students A and B are playing the following game: Each of them writes down on a sheet of paper a positive integer and gives the sheet to the referee. The referee writes down on a blackboard two integers, one of which is the sum of the integers written by the players. After that, the referee asks student A: “Can you tell the integer written by the other student?” If A answers “no,” the referee puts the same question to student B. If B answers “no,” the referee puts the question back to A, and so on. Assume that both students are intelligent and truthful. Prove that after a finite number of questions, one of the students will answer “yes.” 3.33 IMO 1992 253 3.33 The Thirty-Third IMO Moscow, Russia, July 10–21, 1992 3.33.1 Contest Problems First Day (July 15) 1. Find all integer triples (p, q, r) such that 1 < p < q < r and (p − 1)(q − 1)(r − 1) is a divisor of (pqr − 1). 2. Find all functions f : R → R such that f (x2 + f (y)) = y + f (x)2 for all x, y in R. 3. Given nine points in space, no four of which are coplanar, find the minimal natural number n such that for any coloring with red or blue of n edges drawn between these nine points there always exists a triangle having all edges of the same color. Second Day (July 16) 4. In the plane, let there be given a circle C, a line l tangent to C, and a point M on l. Find the locus of points P that has the following property: There exist two points Q and R on l such that M is the midpoint of QR and C is the incircle of PQR. 5. Let V be a finite subset of Euclidean space consisting of points (x, y, z) with integer coordinates. Let S1 , S2 , S3 be the projections of V onto the yz, xz, xy planes, respectively. Prove that |V |2 ≤ |S1 ||S2 ||S3 | (|X | denotes the number of elements of X ). 6. For each positive integer n, denote by s(n) the greatest integer such that for all positive integer k ≤ s(n), n2 can be expressed as a sum of squares of k positive integers. (a) Prove that s(n) ≤ n2 − 14 for all n ≥ 4. (b) Find a number n such that s(n) = n2 − 14. (c) Prove that there exist infinitely many positive integers n such that s(n) = n2 − 14. 3.33.2 Longlisted Problems 1. (AUS 1) Points D and E are chosen on the sides AB and AC of the triangle ABC in such a way that if F is the intersection point of BE and CD, then AE + EF = AD + DF. Prove that AC + CF = AB + BF. 254 3 Problems 2. (AUS 2) (SL92-1). Original formulation. Let m be a positive integer and x0 , y0 integers such that x0 , y0 are relatively prime, y0 divides x20 + m, and x0 divides y20 + m. Prove that there exist positive integers x and y such that x and y are relatively prime, y divides x2 + m, x divides y2 + m, and x + y ≤ m + 1. 3. (AUS 3) Let ABC be a triangle, O its circumcenter, S its centroid, and H its orthocenter. Denote by A1 , B1 , and C1 the centers of the circles circumscribed about the triangles CHB, CHA, and AHB, respectively. Prove that the triangle ABC is congruent to the triangle A1 B1C1 and that the nine-point circle of △ABC is also the nine-point circle of △A1 B1C1 . 4. (CAN 1) Let p, q, and r be the angles of a triangle, and let a = sin 2p, b = sin 2q, and c = sin 2r. If s = (a + b + c)/2, show that s(s − a)(s − b)(s − c) ≥ 0. When does equality hold? 5. (CAN 2) Let I, H, O be the incenter, centroid, and circumcenter of the nonisosceles triangle ABC. Prove that AIkHO if and only if ∡BAC = 120◦. 6. (CAN 3) Suppose that n numbers x1 , x2 , . . . , xn are chosen randomly from the set {1, 2, 3, 4, 5}. Prove that the probability that x21 + x22 + · · · + x2n ≡ 0 (mod 5) is at least 1/5. 7. (CAN 4) Let X be a bounded, nonempty set of points in the Cartesian plane. Let f (X) be the set of all points that are at a distance of at most 1 from some point in X . Let f n (X) = f ( f (. . . ( f (X )). . . )) (n times). Show that f n (X ) becomes “more circular” as n gets larger. In other words, if rn = sup{radii of circles contained in f n (X )} and Rn = inf{radii of circles containing f n (X )}, then show that Rn /rn gets arbitrarily close to 1 as n becomes arbitrarily large. 8. (CHN 1) (SL92-2). 9. (CHN 2) (SL92-3). 10. (CHN 3) (SL92-4). 11. (COL 1) Let φ (n, m), m 6= 1, be the number of positive integers less than or equal to n that are coprime with m. Clearly, φ (m, m) = φ (m), where φ (m) is Euler’s phi function. Find all integers m that satisfy the following inequality: φ (n, m) φ (m) ≥ n m for every positive integer n. 12. (COL 2) Given a triangle ABC such that the circumcenter is in the interior of the incircle, prove that the triangle ABC is acute-angled. 13. (COL 3) (SL92-5). 3.33 IMO 1992 255 14. (FIN 1) Integers a1 , a2 , . . . , an satisfy |ak | = 1 and n ∑ ak ak+1ak+2 ak+3 = 2, k=1 where an+ j = a j . Prove that n 6= 1992. 15. (FIN 2) Prove that there exist 78 lines in the plane such that they have exactly 1992 points of intersection. 16. (FIN 3) Find all triples (x, y, z) of integers such that 1 2 3 2 + 2+ 2= . 2 x y z 3 17. (FRA 1) (SL92-20). 18. (FRG 1) Fibonacci numbers are defined as follows: F1 = F2 = 1, Fn+2 = Fn+1 + Fn , n ≥ 1. Let an be the number of words that consist of n letters 0 or 1 and contain no two letters 1 at distance two from each other. Express an in terms of Fibonacci numbers. 19. (FRG 2) Denote by an the greatest number that is not divisible by 3 and that divides n. Consider the sequence s0 = 0, sn = a1 + a2 + · · · + an , n ∈ N. Denote by A(n) the number of all sums sk (0 ≤ k ≤ 3n , k ∈ N0 ) that are divisible by 3. Prove the formula A(n) = 3n−1 + 2 · 3(n/2)−1 cos(nπ /6), n ∈ N0 . 20. (FRG 3) Let X and Y be two sets of points in the plane and M be a set of segments connecting points from X and Y . Let k be a natural number. Prove that the segments from M can be painted using k colors in such a way that for any point x ∈ X ∪Y and two colors α and β (α 6= β ), the difference between the number of α -colored segments and the number of β -colored segments originating in X is less than or equal to 1. 21. (UNK 1) Prove that if x, y, z > 1 and 1x + 1y + 1z = 2, then √ x+y+z ≥ 22. (UNK 2) (SL92-21). √ x−1+ p √ y − 1 + z − 1. 23. (HKG 1) An Egyptian number is a positive integer that can be expressed as a sum of positive integers, not necessarily distinct, such that the sum of their reciprocals is 1. For example, 32 = 2 + 3 + 9 + 18 is Egyptian because 12 + 13 + 1 1 9 + 18 = 1. Prove that all integers greater than 23 are Egyptian. 24. (ISL 1) Let Q+ denote the set of nonnegative rational numbers. Show that there exists exactly one function f : Q+ →Q+ satisfying the following conditions:  q 1 (i) if 0 < q < 2 , then f (q) = 1 + f 1−2q ; 256 3 Problems (ii) if 1 < q ≤ 2, then f (q) = 1 + f (q + 1); (iii) f (q) f (1/q) = 1 for all q ∈ Q+ . Find the smallest rational number q ∈ Q+ such that f (q) = 19/92. 25. (IND 1) (a) Show that the set N of all natural numbers can be partitioned into three disjoint subsets A, B, and C satisfying the following conditions: A2 = A, AB = B, B2 = C, AC = C, C2 = B, BC = A, where HK stands for {hk | h ∈ H, k ∈ K} for any two subsets H, K of N, and H 2 denotes HH. (b) Show that for every such partition of N, min{n ∈ N | n ∈ A and n + 1 ∈ A} is less than or equal to 77. 26. (IND 2) (SL92-6). 27. (IND 3) Let ABC be an arbitrary scalene triangle. Define Σ to be the set of all circles y that have the following properties: (i) y meets each side of △ABC in two (possibly coincident) points; (ii) if the points of intersection of y with the sides of the triangle are labeled by P, Q, R, S, T , U, with the points occurring on the sides in orders B(B, P, Q,C), B(C, R, S, A), B(A, T,U, B), then the following relations of parallelism hold: T SkBC; PUkCA; RQkAB. (In the limiting cases, some of the conditions of parallelism will hold vacuously; e.g., if A lies on the circle y, then T , S both coincide with A and the relation T SkBC holds vacuously.) (a) Under what circumstances is Σ nonempty? (b) Assuming that Σ is nonempty, show how to construct the locus of centers of the circles in the set Σ . (c) Given that the set Σ has just one element, deduce the size of the largest angle of △ABC. (d) Show how to construct the circles in Σ that have, respectively, the largest and the smallest radii. 28. (IND 4) (SL92-7). Alternative formulation. Two circles G1 and G2 are inscribed in a segment of a circle G and touch each other externally at a point W . Let A be a point of intersection of a common internal tangent to G1 and G2 with the arc of the segment, and let B and C be the endpoints of the chord. Prove that W is the incenter of the triangle ABC. 29. (IND 5) (SL92-8). 30. (IND 6) Let Pn = (19 + 92)(192 + 922) · · · (19n + 92n) for each positive integer n. Determine, with proof, the least positive integer m, if it exists, for which Pm is divisible by 3333. 31. (IRL 1) (SL92-19). 3.33 IMO 1992 257 32. (IRL 2) Let Sn = {1, 2, . . . , n} and fn : Sn → Sn be defined inductively as follows: f1 (1) = 1, fn (2 j) = j ( j = 1, 2, . . . , [n/2]) and (i) if n = 2k (k ≥ 1), then fn (2 j − 1) = fk ( j) + k ( j = 1, 2, . . . , k); (ii) if n = 2k + 1 (k ≥ 1), then fn (2k + 1) = k + fk+1 (1), fn (2 j − 1) = k + fk+1 ( j + 1) ( j = 1, 2, . . . , k). Prove that fn (x) = x if and only if x is an integer of the form for some positive integer d. (2n + 1)(2d − 1) 2d+1 − 1 33. (IRL 3) Let a, b, c be positive real numbers and p, q, r complex numbers. Let S be the set of all solutions (x, y, z) in C of the system of simultaneous equations ax + by + cz = p, ax2 + by2 + cz2 = q, ax3 + bx3 + cx3 = r. Prove that S has at most six elements. 34. (IRL 4) Let a, b, c be integers. Prove that there are integers p1 , q1 , r1 , p2 , q2 , r2 such that a = q1 r2 − q2r1 , b = r1 p2 − r2 p1 , c = p1 q2 − p2 q1 . 35. (IRN 1) (SL92-9). 36. (IRN 2) Find all rational solutions of a2 + c2 + 17(b2 + d 2 ) = 21, ab + cd = 2. 37. (IRN 3) Let the circles C1 , C2 , and C3 be orthogonal to the circle C and intersect each other inside C forming acute angles of measures A, B, and C. Show that A + B +C < π. 38. (ITA 1) (SL92-10). 39. (ITA 2) Let n ≥ 2 be an integer. Find the minimum k for which there exists a partition of {1, 2, . . . , k} into n subsets X1 , X2 , . . . , Xn such that the following condition holds: for any i, j, 1 ≤ i < j ≤ n, there exist x1 ∈ X1 , x2 ∈ X2 such that |xi − x j | = 1. 40. (ITA 3) The colonizers of a spherical planet have decided to build N towns, each having area 1/1000 of the total area of the planet. They also decided that any two points belonging to different towns will have different latitude and different longitude. What is the maximal value of N? 41. (JPN 1) Let S be a set of positive integers n1 , n2 , . . . , n6 and let n( f ) denote the number n1 n f (1) + n2 n f (2) + · · · + n6 n f (6) , where f is a permutation of {1, 2, . . . , 6}. Let 258 3 Problems Ω = {n( f ) | f is a permutation of {1, 2, . . . , 6}}. Give an example of positive integers n1 , . . . , n6 such that Ω contains as many elements as possible and determine the number of elements of Ω . 42. (JPN 2) (SL92-11). 43. (KOR 1) Find the number of positive integers n satisfying φ (n) | n such that  ∞  n n−1 − = 1992. ∑ m m=1 m What is the largest number among them? As usual, φ (n) is the number of positive integers less than or equal to n and relatively prime to n.7 44. (KOR 2) (SL92-16). 45. (KOR 3) Let n be a positive integer. Prove that the number of ways to express n as a sum of distinct positive integers (up to order) and the number of ways to express n as a sum of odd positive integers (up to order) are the same. 46. (KOR 4) Prove that the sequence 5, 12, 19, 26, 33, . . . contains no term of the form 2n − 1. 3n+2 47. (KOR 5) Find the largest integer not exceeding ∏1992 n=1 3n+1 . 48. (MNG 1) Find all the functions f : R+ → R satisfying the identity x y f (x) f (y) = yα · f + xβ · f , x, y ∈ R+ , 2 2 where α , β are given real numbers. 49. (MNG 2) Given real numbers xi (i = 1, 2, . . . , 4x + 2) such that 4x+2 ∑ (−1)i+1xi xi+1 = 4m (x1 = x4k+3 ), i=1 prove that it is possible to choose numbers xk1 , . . . , xk6 such that 6 ∑ (−1)i xki xki+1 > m i=1 (xk1 = xk7 ). 50. (MNG 3) Let N be a point inside the triangle ABC. Through the midpoints of the segments AN, BN, and CN the lines parallel to the opposite sides of △ABC are constructed. Let AN , BN , and CN be the intersection points of these lines. If N is the orthocenter of the triangle ABC, prove that the nine-point circles of △ABC and △AN BN CN coincide. Remark. The statement of the original problem was that the nine-point circles of the triangles AN BN CN and AM BMCM coincide, where N and M are the orthocenter and the centroid of △ABC. This statement is false. 7 The problem in this  n formulation  n−1  is senseless. The correct formulation could be, “Find . . . such that ∑∞ = 1992 . . . .” m=1 m − m 3.33 IMO 1992 259 51. (NLD 1) (SL92-12). 52. (NLD 2) Let n be an integer > 1. In a circular arrangement of n lamps L0 , . . . , Ln−1 , each one of which can be either ON or OFF, we start with the situation that all lamps are ON, and then carry out a sequence of steps, Step0, Step1, . . . . If L j−1 ( j is taken mod n) is ON, then Step j changes the status of L j (it goes from ON to OFF or from OFF to ON) but does not change the status of any of the other lamps. If L j−1 is OFF, then Step j does not change anything at all. Show that: (a) There is a positive integer M(n) such that after M(n) steps all lamps are ON again. (b) If n has the form 2k , then all lamps are ON after n2 − 1 steps. (c) If n has the form 2k + 1, then all lamps are ON after n2 − n + 1 steps. 53. (NZL 1) (SL92-13). 54. (POL 1) Suppose that n > m ≥ 1 are integers such that the string of digits 143 occurs somewhere in the decimal representation of the fraction m/n. Prove that n > 125 55. (POL 2) (SL92-14). 56. (POL 3) A directed graph (any two distinct vertices joined by at most one directed line) has the following property: If x, u, and v are three distinct vertices such that x → u and x → v, then u → w and v → w for some vertex w. Suppose that x → u → y → · · · → z is a path of length n, that cannot be extended to the right (no arrow goes away from z). Prove that every path beginning at x arrives after n steps at z. 57. (POL 4) For positive numbers a, b, c define A = (a + b + c)/3, G = (abc)1/3 , H = 3/(a−1 + b−1 + c−1 ). Prove that  3 A 1 3 A ≥ + · , G 4 4 H for every a, b, c > 0. 58. (POR 1) Let ABC be a triangle. Denote by a, b, and c the lengths of the sides opposite to the angles A, B, and C, respectively. Prove that8 bc sin A + sin B + sinC = . a + b + c cos(A/2) sin(B/2) sin(C/2) 59. (PRK 1) Let a regular 7-gon A0 A1 A2 A3 A4 A5 A6 be inscribed in a circle. Prove that for any two points P, Q on the arc A0 A6 the following equality holds: 6 6 i=0 i=0 ∑ (−1)i PAi = ∑ (−1)iQAi . 8 The statement of the problem is obviously wrong, and the authors couldn’t determine a suitable alteration of the formulation which would make the problem correct. We put it here only for completeness of the problem set. 260 3 Problems 60. (PRK 2) (SL92-15). 61. (PRK 3) There are a board with 2n · 2n (= 4n2 ) squares and 4n2 − 1 cards numbered with different natural numbers. These cards are put one by one on each of the squares. One square is empty. We can move a card to an empty square from one of the adjacent squares (two squares are adjacent if they have a common edge). Is it possible to exchange two cards on two adjacent squares of a column (or a row) in a finite number of movements? 62. (ROU 1) Let c1 , . . . , cn (n ≥ 2) be real numbers such that 0 ≤ ∑ ci ≤ n. Prove that there exist integers k1 , . . . , kn such that ∑ ki = 0 and 1 − n ≤ ci + nki ≤ n for every i = 1, . . . , n. 63. (ROU 2) Let a and b be integers. Prove that 2a2 −1 b2 +2 is not an integer. 64. (ROU 3) For any positive integer n consider all representations n = a1 +· · ·+ ak , where a1 > a2 > · · · > ak > 0 are integers such that for all i ∈ {1, 2, . . . , k − 1}, the number ai is divisible by ai+1 . Find the longest such representation of the number 1992. 65. (SAF 1) If A, B, C, and D are four distinct points in space, prove that there is a plane P on which the orthogonal projections of A, B, C, and D form a parallelogram (possibly degenerate). 66. (ESP 1) A circle of radius ρ is tangent to the sides AB and AC of the triangle ABC, and its center K is at a distance p from BC. (a) Prove that a(p− ρ ) = 2s(r − ρ ), where r is the inradius and 2s the perimeter of ABC. (b) Prove that if the circle intersect BC at D and E, then p 4 rr1 (ρ − r)(r1 − ρ ) DE = , (r1 − r) where r1 is the exradius corresponding to the vertex A. 67. (ESP 2) In a triangle, a symmedian is a line through a vertex that is symmetric to the median with respect to the internal bisector (all relative to the same vertex). In the triangle ABC, the median ma meets BC at A′ and the circumcircle again at A1 . The symmedian sa meets BC at M and the circumcircle again at A2 . Given that the line A1 A2 contains the circumcenter O of the triangle, prove that: AA′ b2 + c2 (a) = ; AM 2bc 2 2 2 2 (b) 1 + 4b c = a (b + c2 ). 68. (ESP 3) Show that the numbers tan(rπ /15), where r is a positive integer less than 15 and relatively prime to 15, satisfy x8 − 92x6 + 134x4 − 28x2 + 1 = 0. 69. (SWE 1) (SL92-17). 3.33 IMO 1992 261 70. (THA 1) Let two circles A and B with unequal radii r and R, respectively, be tangent internally at the point A0 . If there exists a sequence of distinct circles (Cn ) such that each circle is tangent to both A and B, and each circle Cn+1 touches circle Cn at the point An , prove that ∞ 4π Rr ∑ |An+1An | < R + r . n=1 71. (THA 2) Let P1 (x, y) and P2 (x, y) be two relatively prime polynomials with complex coefficients. Let Q(x, y) and R(x, y) be polynomials with complex coefficients and each of degree not exceeding d. Prove that there exist two integers A1 , A2 not simultaneously zero with |Ai | ≤ d + 1 (i = 1, 2) and such that the polynomial A1 P1 (x, y) + A2 P2 (x, y) is coprime to Q(x, y) and R(x, y). 72. (TUR 1) In a school six different courses are taught: mathematics, physics, biology, music, history, geography. The students were required to rank these courses according to their preferences, where equal preferences were allowed. It turned out that: (i) mathematics was ranked among the most preferred courses by all students; (ii) no student ranked music among the least preferred ones; (iii) all students preferred history to geography and physics to biology; and (iv) no two rankings were the same. Find the greatest possible value for the number of students in this school. 73. (TUR 2) Let {An | n = 1, 2, . . .} be a set of points in the plane such that for each n, the disk with center An and radius 2n contains no other point A j . For any given positive real numbers a < b and R, show that there is a subset G of the plane satisfying: (i) the area of G is greater than or equal to R; 1 (ii) for each point P in G, a < ∑∞ n=1 |An P| < b. n n o π 74. (TUR 3) Let S = 1992 m | n, m ∈ Z . Show that every real number x ≥ 0 is an accumulation point of S. 75. (TWN 1) A sequence {an } of positive integers is defined by   √ 1 an = n + n + , n ∈ N. 2 Determine the positive integers that occur in the sequence. 76. (TWN 2) Given any triangle ABC and any positive integer n, we say that n is a decomposable number for triangle ABC if there exists a decomposition of the triangle ABC into n subtriangles with each subtriangle similar to △ABC. Determine the positive integers that are decomposable numbers for every triangle. 77. (TWN 3) Show that if 994 integers are chosen from 1, 2, . . . , 1992 and one of the chosen integers is less than 64, then there exist two among the chosen integers such that one of them is a factor of the other. 262 3 Problems 78. (USA 1) Let Fn be the nth Fibonacci number, defined by F1 = F2 = 1 and Fn = Fn−1 + Fn−2 for n > 2. Let A0 , A1 , A2 , . . . be a sequence of points on a circle of radius 1 such that the minor arc from Ak−1 to Ak runs clockwise and such that µ (Ak−1 Ak ) = 4F2k+1 2 F2k+1 +1 for k ≥ 1, where µ (XY ) denotes the radian measure of the arc XY in the clockwise direction. What is the limit of the radian measure of arc A0 An as n approaches infinity? 79. (USA 2) (SL92-18). 80. (USA 3) Given a graph with n vertices and a positive integer m that is less than n, prove that the graph contains a set of m + 1 vertices in which the difference between the largest degree of any vertex in the set and the smallest degree of any vertex in the set is at most m − 1. 81. (USA 4) Suppose that points X ,Y, Z are located on sides BC, CA, and AB, respectively, of △ABC in such a way that △XY Z is similar to △ABC. Prove that the orthocenter of △XY Z is the circumcenter of △ABC. 82. (VNM 1) Let f (x) = xm + a1 xm−1 + · · · + am−1 x + am and g(x) = xn + b1 xn−1 + · · · + bn−1 + bn be two polynomials with real coefficients such that for each real number x, f (x) is the square of an integer if and only if so is g(x). Prove that if n + m > 0, then there exists a polynomial h(x) with real coefficients such that f (x) · g(x) = (h(x))2 . 3.33.3 Shortlisted Problems 1. (AUS 2) Prove that for any positive integer m there exists an infinite number of pairs of integers (x, y) such that (i) x and y are relatively prime; (ii) y divides x2 + m; (iii) x divides y2 + m. 2. (CHN 1) Let R+ be the set of all nonnegative real numbers. Given two positive real numbers a and b, suppose that a mapping f : R+ → R+ satisfies the functional equation f ( f (x)) + a f (x) = b(a + b)x. Prove that there exists a unique solution of this equation. 3. (CHN 2) The diagonals of a quadrilateral ABCD are perpendicular: AC⊥BD. Four squares, ABEF, BCGH,CDIJ, DAKL, are erected externally on its sides. The intersection points of the pairs of straight lines CL, DF; DF, AH; AH, BJ; BJ,CL are denoted by P1 , Q1 , R1 , S1 , respectively, and the intersection points of the pairs of straight lines AI, BK; BK,CE; CE, DG; DG, AI are denoted by P2 , Q2 , R2 , S2 , respectively. Prove that P1 Q1 R1 S1 ∼ = P2 Q2 R2 S2 . 4. (CHN 3)IMO3 Given nine points in space, no four of which are coplanar, find the minimal natural number n such that for any coloring with red or blue of n edges 3.33 IMO 1992 263 drawn between these nine points there always exists a triangle having all edges of the same color. 5. (COL 3) Let ABCD be a convex quadrilateral such that AC = BD. Equilateral triangles are constructed on the sides of the quadrilateral. Let O1 , O2 , O3 , O4 be the centers of the triangles constructed on AB, BC,CD, DA respectively. Show that O1 O3 is perpendicular to O2 O4 . 6. (IND 2)IMO2 Find all functions f : R → R such that f (x2 + f (y)) = y + f (x)2 for all x, y in R. 7. (IND 4) Circles G, G1 , G2 are three circles related to each other as follows: Circles G1 and G2 are externally tangent to one another at a point W and both these circles are internally tangent to the circle G. Points A, B,C are located on the circle G as follows: Line BC is a direct common tangent to the pair of circles G1 and G2 , and line WA is the transverse common tangent at W to G1 and G2 , with W and A lying on the same side of the line BC. Prove that W is the incenter of the triangle ABC. 8. (IND 5) Show that in the plane there exists a convex polygon of 1992 sides satisfying the following conditions: (i) its side lengths are 1, 2, 3, . . . , 1992 in some order; (ii) the polygon is circumscribable about a circle. Alternative formulation. Does there exist a 1992-gon with side lengths 1, 2, 3, . . ., 1992 circumscribed about a circle? Answer the same question for a 1990gon. 9. (IRN 1) Let f (x) be a polynomial with rational coefficients and α be a real number such that α 3 − α = f (α )3 − f (α ) = 331992. Prove that for each n ≥ 1, ( f (n) (α ))3 − f (n) (α ) = 331992, where f (n) (x) = f ( f (. . . f (x))), and n is a positive integer. 10. (ITA 1)IMO5 Let V be a finite subset of Euclidean space consisting of points (x, y, z) with integer coordinates. Let S1 , S2 , S3 be the projections of V onto the yz, xz, xy planes, respectively. Prove that |V |2 ≤ |S1 ||S2 ||S3 | (|X | denotes the number of elements of X ). 11. (JPN 2) In a triangle ABC, let D and E be the intersections of the bisectors of ∠ABC and ∠ACB with the sides AC, AB, respectively. Determine the angles ∠A, ∠B, ∠C if ∡BDE = 24◦ , ∡CED = 18◦ . 12. (NLD 1) Let f , g, and a be polynomials with real coefficients, f and g in one variable and a in two variables. Suppose 264 3 Problems f (x) − f (y) = a(x, y)(g(x) − g(y)) for all x, y ∈ R. Prove that there exists a polynomial h with f (x) = h(g(x)) for all x ∈ R. 13. (NZL 1)IMO1 Find all integer triples (p, q, r) such that 1 < p < q < r and (p − 1)(q − 1)(r − 1) is a divisor of (pqr − 1). 14. (POL 2) For any positive integer x define g(x) = greatest odd divisor of x,  x/2 + x/g(x), if x is even; f (x) = 2(x+1)/2 , if x is odd. Construct the sequence x1 = 1, xn+1 = f (xn ). Show that the number 1992 appears in this sequence, determine the least n such that xn = 1992, and determine whether n is unique. 15. (PRK 2) Does there exist a set M with the following properties? (i) The set M consists of 1992 natural numbers. (ii) Every element in M and the sum of any number of elements have the form mk (m, k ∈ N, k ≥ 2). 16. (KOR 2) Prove that N = 5125 −1 525 −1 is a composite number. 17. (SWE 1) Let α (n) be the number of digits equal to one in the binary representation of a positive integer n. Prove that: (a) the inequality α (n2 ) ≤ 12 α (n)(α (n) + 1) holds; (b) the above inequality is an equality for infinitely many positive integers; 2 (c) there exists a sequence (ni )∞ 1 such that α (ni )/α (ni ) → 0 as i → ∞. 2 Alternative parts: Prove that there exists a sequence (ni )∞ 1 such that α (ni )/α (ni ) tends to (d) ∞; (e) an arbitrary real number γ ∈ (0, 1); (f) an arbitrary real number γ ≥ 0. 18. (USA 2) Let [x] denote the greatest integer less than or equal to x. Pick any x1 in [0, 1) and define the sequence x1 , x2 , x3 , . . . by xn+1 = 0 if xn = 0 and xn+1 = 1/xn − [1/xn ] otherwise. Prove that x1 + x2 + · · · + xn < F1 F2 Fn + + ···+ , F2 F3 Fn+1 where F1 = F2 = 1 and Fn+2 = Fn+1 + Fn for n ≥ 1. 19. (IRL 1) Let f (x) = x8 + 4x6 + 2x4 + 28x2 + 1. Let p > 3 be a prime and suppose there exists an integer z such that p divides f (z). Prove that there exist integers z1 , z2 , . . . , z8 such that if g(x) = (x − z1 )(x − z2 ) · · · (x − z8 ), then all coefficients of f (x) − g(x) are divisible by p. 3.33 IMO 1992 265 20. (FRA 1)IMO4 In the plane, let there be given a circle C, a line l tangent to C, and a point M on l. Find the locus of points P that have the following property: There exist two points Q and R on l such that M is the midpoint of QR and C is the incircle of PQR. 21. (UNK 2)IMO6 For each positive integer n, denote by s(n) the greatest integer such that for all positive integers k ≤ s(n), n2 can be expressed as a sum of squares of k positive integers. (a) Prove that s(n) ≤ n2 − 14 for all n ≥ 4. (b) Find a number n such that s(n) = n2 − 14. (c) Prove that there exist infinitely many positive integers n such that s(n) = n2 − 14. 266 3 Problems 3.34 The Thirty-Fourth IMO Istanbul, Turkey, July 13–24, 1993 3.34.1 Contest Problems First Day (July 18) 1. Let n > 1 be an integer and let f (x) = xn + 5xn−1 + 3. Prove that there do not exist polynomials g(x), h(x), each having integer coefficients and degree at least one, such that f (x) = g(x)h(x). 2. A, B,C, D are four points in the plane, with C, D on the same side of the line AB, such that AC · BD = AD · BC and ∡ADB = 90◦ + ∡ACB. Find the ratio AB ·CD , AC · BD and prove that circles ACD, BCD are orthogonal. (Intersecting circles are said to be orthogonal if at either common point their tangents are perpendicular.) 3. On an infinite chessboard, a solitaire game is played as follows: At the start, we have n2 pieces occupying n2 squares that form a square of side n. The only allowed move is a jump horizontally or vertically over an occupied square to an unoccupied one, and the piece that has been jumped over is removed. For what positive integers n can the game end with only one piece remaining on the board? Second Day (July 19) 4. For three points A, B,C in the plane we define m(ABC) to be the smallest length of the three altitudes of the triangle ABC, where in the case of A, B,C collinear, m(ABC) = 0. Let A, B,C be given points in the plane. Prove that for any point X in the plane, m(ABC) ≤ m(ABX) + m(AXC) + m(X BC). 5. Let N = {1, 2, 3, . . .}. Determine whether there exists a strictly increasing function f : N → N with the following properties: f (1) = 2; f ( f (n)) = f (n) + n (1) (n ∈ N). (2) 6. Let n be an integer greater than 1. In a circular arrangement of n lamps L0 , . . ., Ln−1 , each one of that can be either on or off, we start with the situation where all lamps are on, and then carry out a sequence of steps, Step0 , Step1 , . . . . If L j−1 (indices are taken modulo n) is on, then Step j changes the status of L j (it goes from on to off or from off to on) but does not change the status of any of the other lamps. If L j−1 is off, then Step j does not change anything at all. Show that: 3.34 IMO 1993 267 (a) There is a positive integer M(n) such that after M(n) steps all lamps are on again. (b) If n has the form 2k , then all lamps are on after n2 − 1 steps. (c) If n has the form 2k + 1, then all lamps are on after n2 − n + 1 steps. 3.34.2 Shortlisted Problems 1. (BRA 1) Show that there exists a finite set A ⊂ R2 such that for every X ∈ A there are points Y1 ,Y2 , . . . ,Y1993 in A such that the distance between X and Yi is equal to 1, for every i. 2. (CAN 2) Let triangle ABC be such that its circumradius R is equal to 1. Let r be the inradius of ABC and let p be the inradius of the orthic triangle A′ B′C′ of triangle ABC. Prove that p ≤ 1 − 13 (1 + r)2 . Remark. The orthic triangle is the triangle whose vertices are the feet of the altitudes of ABC. 3. (ESP 1) Consider the triangle ABC, its circumcircle k with center O and radius R, and its incircle with center I and radius r. Another circle kc is tangent to the sides CA,CB at D, E, respectively, and it is internally tangent to k. Show that the incenter I is the midpoint of DE. 4. (ESP 2) In the triangle ABC, let D, E be points on the side BC such that ∠BAD = ∠CAE. If M, N are, respectively, the points of tangency with BC of the incircles of the triangles ABD and ACE, show that 1 1 1 1 + = + . MB MD NC NE 5. (FIN 3)IMO3 On an infinite chessboard, a solitaire game is played as follows: At the start, we have n2 pieces occupying n2 squares that form a square of side n. The only allowed move is a jump horizontally or vertically over an occupied square to an unoccupied one, and the piece that has been jumped over is removed. For what positive integers n can the game end with only one piece remaining on the board? 6. (GER 1)IMO5 Let N = {1, 2, 3, . . . }. Determine whether there exists a strictly increasing function f : N → N with the following properties: f (1) = 2; f ( f (n)) = f (n) + n (1) (n ∈ N). ac − b2 (2) 7. (GEO 3) Let a, b, c be given integers a > 0, = P = P1 · · · Pm where P1 , . . . , Pm are (distinct) prime numbers. Let M(n) denote the number of pairs of integers (x, y) for which ax2 + 2bxy + cy2 = n. Prove that M(n) is finite and M(n) = M(Pk · n) for every integer k ≥ 0. 268 3 Problems 8. (IND 1) Define a sequence h f (n)i∞ n=1 of positive integers by f (1) = 1 and  f (n − 1) − n, if f (n − 1) > n; f (n) = f (n − 1) + n, if f (n − 1) ≤ n, for n ≥ 2. Let S = {n ∈ N | f (n) = 1993}. (a) Prove that S is an infinite set. (b) Find the least positive integer in S. (c) If all the elements of S are written in ascending order as n1 < n2 < n3 < · · · , show that ni+1 lim = 3. i→∞ ni 9. (IND 4) (a) Show that the set Q+ of all positive rational numbers can be partitioned into three disjoint subsets A, B,C satisfying the following conditions: BA = B, B2 = C, BC = A, where HK stands for the set {hk | h ∈ H, k ∈ K} for any two subsets H, K of Q+ and H 2 stands for HH. (b) Show that all positive rational cubes are in A for such a partition of Q+ . (c) Find such a partition Q+ = A ∪ B ∪C with the property that for no positive integer n ≤ 34 are both n and n + 1 in A; that is, min{n ∈ N | n ∈ A, n + 1 ∈ A} > 34. 10. (IND 5) A natural number n is said to have the property P if whenever n divides an − 1 for some integer a, n2 also necessarily divides an − 1. (a) Show that every prime number has property P. (b) Show that there are infinitely many composite numbers n that possess property P. 11. (IRL 1)IMO1 Let n > 1 be an integer and let f (x) = xn + 5xn−1 + 3. Prove that there do not exist polynomials g(x), h(x), each having integer coefficients and degree at least one, such that f (x) = g(x)h(x). 12. (IRL 2) Let n, k be positive integers with k ≤ n and let S be a set containing n distinct real numbers. Let T be the set of all real numbers of the form x1 + x2 + · · · + xk , where x1 , x2 , . . . , xk are distinct elements of S. Prove that T contains at least k(n − k) + 1 distinct elements. 13. (IRL 3) Let S be the set of all pairs (m, n) of relatively prime positive integers m, n with n even and m < n. For s = (m, n) ∈ S write n = 2k n0 , where k, n0 are positive integers with n0 odd and define f (s) = (n0 , m + n − n0). Prove that f is a function from S to S and that for each s = (m, n) ∈ S, there exists a positive integer t ≤ m+n+1 such that f t (s) = s, where 4 f t (s) = ( f ◦ f ◦ · · · ◦ f )(s). | {z } t times 3.34 IMO 1993 269 If m + n is a prime number that does not divide 2k − 1 for k = 1, 2, . . . , m + n − 2,  prove that the smallest value of t that satisfies the above conditions is m+n+1 , 4 where [x] denotes the greatest integer less than or equal to x. 14. (ISR 1) The vertices D, E, F of an equilateral triangle lie on the sides BC,CA, AB respectively of a triangle ABC. If a, b, c are the respective lengths of these sides, and S the area of ABC, prove that √ 2 2S DE ≥ p √ . a2 + b2 + c2 + 4 3S 15. (MKD 1)IMO4 For three points A, B,C in the plane we define m(ABC) to be the smallest length of the three altitudes of the triangle ABC, where in the case of A, B,C collinear, m(ABC) = 0. Let A, B,C be given points in the plane. Prove that for any point X in the plane, m(ABC) ≤ m(ABX) + m(AXC) + m(X BC). 16. (MKD 3) Let n ∈ N, n ≥ 2, and A0 = (a01 , a02 , . . . , a0n ) be any n-tuple of natural numbers such that 0 ≤ a0i ≤ i − 1, for i = 1, . . . , n. The n-tuples A1 = (a11 , a12 , . . . , a1n ), A2 = (a21 , a22 , . . . , a2n ), . . . are defined by ai+1, j = Card{ai,l | 1 ≤ l ≤ j − 1, ai,l ≥ ai, j }, for i ∈ N and j = 1, . . . ,n. Prove that there exists k ∈ N, such that Ak+2 = Ak . 17. (NLD 2)IMO6 Let n be an integer greater than 1. In a circular arrangement of n lamps L0 , . . ., Ln−1 , each one of that can be either on or off, we start with the situation where all lamps are on, and then carry out a sequence of steps, Step0 , Step1 , . . . . If L j−1 (indices are taken modulo n) is on, then Step j changes the status of L j (it goes from on to off or from off to on) but does not change the status of any of the other lamps. If L j−1 is off, then Step j does not change anything at all. Show that: (a) There is a positive integer M(n) such that after M(n) steps all lamps are on again. (b) If n has the form 2k , then all lamps are on after n2 − 1 steps. (c) If n has the form 2k + 1, then all lamps are on after n2 − n + 1 steps. 18. (POL 1) Let Sn be the number of sequences (a1 , a2 , . . . , an ), where ai ∈ {0, 1}, in which no six consecutive blocks are equal. Prove that Sn → ∞ as n → ∞. 19. (ROU 2) Let a, b, n be positive integers, b > 1 and bn − 1 | a. Show that the representation of the number a in the base b contains at least n digits different from zero. 20. (ROU 3) Let c1 , . . . , cn ∈ R (n ≥ 2) such that 0 ≤ ∑ni=1 ci ≤ n. Show that we can find integers k1 , . . . , kn such that ∑ni=1 ki = 0 and 1 − n ≤ ci + nki ≤ n for every i = 1, . . . , n. 270 3 Problems 21. (UNK 1) A circle S is said to cut a circle Σ diametrally if their common chord is a diameter of Σ . Let SA , SB , SC be three circles with distinct centers A, B,C respectively. Prove that A, B,C are collinear if and only if there is no unique circle S that cuts each of SA , SB , SC diametrally. Prove further that if there exists more than one circle S that cuts each of SA , SB , SC diametrally, then all such circles pass through two fixed points. Locate these points in relation to the circles SA , SB , SC . 22. (UNK 2)IMO2 A, B,C, D are four points in the plane, with C, D on the same side of the line AB, such that AC · BD = AD · BC and ∡ADB = 90◦ + ∡ACB. Find the ratio AB ·CD , AC · BD and prove that circles ACD, BCD are orthogonal. (Intersecting circles are said to be orthogonal if at either common point their tangents are perpendicular.) 23. (UNK 3) A finite set of (distinct) positive integers is called a “DS-set” if each of the integers divides the sum of them all. Prove that every finite set of positive integers is a subset of some DS-set. 24. (USA 3) Prove that a b c d 2 + + + ≥ b + 2c + 3d c + 2d + 3a d + 2a + 3b a + 2b + 3c 3 for all positive real numbers a, b, c, d. 25. (VNM 1) Solve the following system of equations, in which a is a given number satisfying |a| > 1: x21 = ax2 + 1, x22 = ax3 + 1, ··· ··· x2999 = ax1000 + 1, x21000 = ax1 + 1. 26. (VNM 2) Let a, b, c, d be four nonnegative numbers satisfying a + b + c + d = 1. Prove the inequality abc + bcd + cda + dab ≤ 1 176 + abcd. 27 27 3.35 IMO 1994 271 3.35 The Thirty-Fifth IMO Hong Kong, July 9–22, 1994 3.35.1 Contest Problems First Day (July 13) 1. Let m and n be positive integers. The set A = {a1 , a2 , . . . , am } is a subset of 1, 2, . . . , n. Whenever ai + a j ≤ n, 1 ≤ i ≤ j ≤ m, ai + a j also belongs to A. Prove that a1 + a2 + · · · + a m n+1 ≥ . m 2 2. N is an arbitrary point on the bisector of ∠BAC. P and O are points on the lines AB and AN, respectively, such that ∡ANP = 90◦ = ∡APO. Q is an arbitrary point on NP, and an arbitrary line through Q meets the lines AB and AC at E and F respectively. Prove that ∡OQE = 90◦ if and only if QE = QF. 3. For any positive integer k, Ak is the subset of {k + 1, k + 2, . . .,2k} consisting of all elements whose digits in base 2 contain exactly three 1’s. Let f (k) denote the number of elements in Ak . (a) Prove that for any positive integer m, f (k) = m has at least one solution. (b) Determine all positive integers m for which f (k) = m has a unique solution. Second Day (July 14) 4. Determine all pairs (m, n) of positive integers such that n3 +1 mn−1 is an integer. 5. Let S be the set of real numbers greater than −1. Find all functions f : S → S such that f (x + f (y) + x f (y)) = y + f (x) + y f (x) for all x and y in S, and f (x)/x is strictly increasing for −1 < x < 0 and for 0 < x. 6. Find a set A of positive integers such that for any infinite set P of prime numbers, there exist positive integers m ∈ A and n 6∈ A, both the product of the same number (at least two) of distinct elements of P. 3.35.2 Shortlisted Problems a2 n 1. A1 (USA) Let a0 = 1994 and an+1 = an +1 for each nonnegative integer n. Prove that 1994 − n is the greatest integer less than or equal to an , 0 ≤ n ≤ 998. 2. A2 (FRA)IMO1 Let m and n be positive integers. The set A = {a1 , a2 , . . . , am } is a subset of {1, 2, . . ., n}. Whenever ai + a j ≤ n, 1 ≤ i ≤ j ≤ m, ai + a j also belongs to A. Prove that a1 + a2 + · · · + a m n+1 ≥ . m 2 272 3 Problems 3. A3 (UNK)IMO5 Let S be the set of real numbers greater than −1. Find all functions f : S → S such that f (x + f (y) + x f (y)) = y + f (x) + y f (x) for all x and y in S, and f (x)/x is strictly increasing for −1 < x < 0 and for 0 < x. 4. A4 (MNG) Let R denote the set of all real numbers and R+ the subset of all positive ones. Let α and β be given elements in R, not necessarily distinct. Find all functions f : R+ → R such that x y f (x) f (y) = yα f + xβ f for all x and y in R+ . 2 2 2 5. A5 (POL) Let f (x) = x 2x+1 for x 6= 0. Define f (0) (x) = x and f (n) (x) = f ( f (n−1) (x)) for all positive integers n and x 6= 0. Prove that for all nonnegative integers n and x 6= −1, 0, or 1, f (n) (x) 1 = 1+   2n  . (n+1) f (x) f x+1 x−1 6. C1 (UKR) On a 5 × 5 board, two players alternately mark numbers on empty cells. The first player always marks 1’s, the second 0’s. One number is marked per turn, until the board is filled. For each of the nine 3 × 3 squares the sum of the nine numbers on its cells is computed. Denote by A the maximum of these sums. How large can the first player make A, regardless of the responses of the second player? 7. C2 (COL) In a certain city, age is reckoned in terms of real numbers rather than integers. Every two citizens x and x′ either know each other or do not know each other. Moreover, if they do not, then there exists a chain of citizens x = x0 , x1 , . . . , xn = x′ for some integer n ≥ 2 such that xi−1 and xi know each other. In a census, all male citizens declare their ages, and there is at least one male citizen. Each female citizen provides only the information that her age is the average of the ages of all the citizens she knows. Prove that this is enough to determine uniquely the ages of all the female citizens. 8. C3 (MKD) Peter has three accounts in a bank, each with an integral number of dollars. He is only allowed to transfer money from one account to another so that the amount of money in the latter is doubled. (a) Prove that Peter can always transfer all his money into two accounts. (b) Can Peter always transfer all his money into one account? 9. C4 (EST) There are n + 1 fixed positions in a row, labeled 0 to n in increasing order from right to left. Cards numbered 0 to n are shuffled and dealt, one in each position. The object of the game is to have card i in the ith position for 0 ≤ i ≤ n. If this has not been achieved, the following move is executed. Determine the smallest k such that the kth position is occupied by a card l > k. Remove this card, slide all cards from the (k + 1)st to the lth position one place to the right, and replace the card l in the lth position. 3.35 IMO 1994 273 (a) Prove that the game lasts at most 2n − 1 moves. (b) Prove that there exists a unique initial configuration for which the game lasts exactly 2n − 1 moves. 10. C5 (SWE) At a round table are 1994 girls, playing a game with a deck of n cards. Initially, one girl holds all the cards. In each turn, if at least one girl holds at least two cards, one of these girls must pass a card to each of her two neighbors. The game ends when and only when each girl is holding at most one card. (a) Prove that if n ≥ 1994, then the game cannot end. (b) Prove that if n < 1994, then the game must end. 11. C6 (FIN) On an infinite square grid, two players alternately mark symbols on empty cells. The first player always marks X ’s, the second O’s. One symbol is marked per turn. The first player wins if there are 11 consecutive X ’s in a row, column, or diagonal. Prove that the second player can prevent the first from winning. 12. C7 (BRA) Prove that for any integer n ≥ 2, there exists a set of 2n−1 points in the plane such that no 3 lie on a line and no 2n are the vertices of a convex 2n-gon. 13. G1 (FRA) A semicircle Γ is drawn on one side of a straight line l. C and D are points on Γ . The tangents to Γ at C and D meet l at B and A respectively, with the center of the semicircle between them. Let E be the point of intersection of AC and BD, and F the point on l such that EF is perpendicular to l. Prove that EF bisects ∠CFD. 14. G2 (UKR) ABCD is a quadrilateral with BC parallel to AD. M is the midpoint of CD, P that of MA and Q that of MB. The lines DP and CQ meet at N. Prove that N is not outside triangle ABM.9 15. G3 (RUS) A circle ω is tangent to two parallel lines l1 and l2 . A second circle ω1 is tangent to l1 at A and to ω externally at C. A third circle ω2 is tangent to l2 at B, to ω externally at D, and to ω1 externally at E. AD intersects BC at Q. Prove that Q is the circumcenter of triangle CDE. 16. G4 (AUS-ARM)IMO2 N is an arbitrary point on the bisector of ∠BAC. P and O are points on the lines AB and AN, respectively, such that ∡ANP = 90◦ = ∡APO. Q is an arbitrary point on NP, and an arbitrary line through Q meets the lines AB and AC at E and F respectively. Prove that ∡OQE = 90◦ if and only if QE = QF. 17. G5 (CYP) A line l does not meet a circle ω with center O. E is the point on l such that OE is perpendicular to l. M is any point on l other than E. The tangents from M to ω touch it at A and B. C is the point on MA such that EC is perpendicular to MA. D is the point on MB such that ED is perpendicular to MB. 9 This problem is false. However, it is true if “not outside ABM” is replaced by “not outside ABCD”. 274 3 Problems The line CD cuts OE at F. Prove that the location of F is independent of that of M. 18. N1 (BGR) M is a subset of {1, 2, 3, . . ., 15} such that the product of any three distinct elements of M is not a square. Determine the maximum number of elements in M. 19. N2 (AUS)IMO4 Determine all pairs (m, n) of positive integers such that an integer. n3 +1 mn−1 is 20. N3 (FIN)IMO6 Find a set A of positive integers such that for any infinite set P of prime numbers, there exist positive integers m ∈ A and n 6∈ A, both the product of the same number of distinct elements of P. 21. N4 (FRA) For any positive integer x0 , three sequences {xn }, {yn }, and {zn } are defined as follows: (i) y0 = 4 and z0 = 1; (ii) if xn is even for n ≥ 0, xn+1 = x2n , yn+1 = 2yn , and zn+1 = zn ; (iii) if xn is odd for n ≥ 0, xn+1 = xn − y2n − zn , yn+1 = yn , and zn+1 = yn + zn . The integer x0 is said to be good if xn = 0 for some n ≥ 1. Find the number of good integers less than or equal to 1994. 22. N5 (ROU)IMO3 For any positive integer k, Ak is the subset of {k+1, k+2, . . ., 2k} consisting of all elements whose digits in base 2 contain exactly three 1’s. Let f (k) denote the number of elements in Ak . (a) Prove that for any positive integer m, f (k) = m has at least one solution. (b) Determine all positive integers m for which f (k) = m has a unique solution. 23. N6 (LVA) Let x1 and x2 be relatively prime positive integers. For n ≥ 2, define xn+1 = xn xn−1 + 1. (a) Prove that for every i > 1, there exists j > i such that xii divides x jj . (b) Is it true that x1 must divide x jj for some j > 1? 24. N7 (UNK) A wobbly number is a positive integer whose digits in base 10 are alternately nonzero and zero, the units digit being nonzero. Determine all positive integers that do not divide any wobbly number. 3.36 IMO 1995 275 3.36 The Thirty-Sixth IMO Toronto, Canada, July 13–25, 1995 3.36.1 Contest Problems First Day (July 19) 1. Let A, B,C, and D be distinct points on a line, in that order. The circles with diameters AC and BD intersect at X and Y . O is an arbitrary point on the line XY but not on AD. CO intersects the circle with diameter AC again at M, and BO intersects the other circle again at N. Prove that the lines AM, DN, and XY are concurrent. 2. Let a, b, and c be positive real numbers such that abc = 1. Prove that 1 1 1 3 + + ≥ . a3 (b + c) b3 (a + c) c3 (a + b) 2 3. Determine all integers n > 3 such that there are n points A1 , A2 , . . . , An in the plane that satisfy the following two conditions simultaneously: (a) No three lie on the same line. (b) There exist real numbers p1 , p2 , . . . , pn such that the area of △Ai A j Ak is equal to pi + p j + pk , for 1 ≤ i < j < k ≤ n. Second Day (July 20) 4. The positive real numbers x0 , x1 , . . . , x1995 satisfy x0 = x1995 and xi−1 + 2 xi−1 = 2xi + 1 xi for i = 1, 2, . . . , 1995. Find the maximum value that x0 can have. 5. Let ABCDEF be a convex hexagon with AB = BC = CD, DE = EF = FA, and ∡BCD = ∡EFA = π /3 (that is, 60◦ ). Let G and H be two points interior to the hexagon, such that angles AGB and DHE are both 2π /3 (that is, 120◦). Prove that AG + GB + GH + DH + HE ≥ CF. 6. Let p be an odd prime. Find the number of p-element subsets A of {1, 2, . . . , 2p} such that the sum of all elements of A is divisible by p. 3.36.2 Shortlisted Problems 1. A1 (RUS)IMO2 Let a, b, and c be positive real numbers such that abc = 1. Prove that 1 1 1 3 + + ≥ . a3 (b + c) b3 (a + c) c3 (a + b) 2 276 3 Problems 2. A2 (SWE) Let a and b be nonnegative integers such that ab ≥ c2 , where c is an integer. Prove that there is a number n and integers x1 , x2 , . . . , xn , y1 , y2 , . . . , yn such that n ∑ x2i = a, i=1 n ∑ y2i = b, n and i=1 ∑ xi yi = c. i=1 3. A3 (UKR) Let n be an integer, n ≥ 3. Let a1 , a2 , . . . , an be real numbers such that 2 ≤ ai ≤ 3 for i = 1, 2, . . . , n. If s = a1 + a2 + · · · + an , prove that a21 + a22 − a23 a22 + a23 − a24 a2 + a21 − a22 + + ···+ n ≤ 2s − 2n. a1 + a 2 − a 3 a2 + a3 − a4 a n + a 1 − a2 4. A4 (USA) Let a, b, and c be given positive real numbers. Determine all positive real numbers x, y, and z such that x+y+z = a+b+c and 4xyz − (a2x + b2 y + c2 z) = abc. 5. A5 (UKR) Let R be the set of real numbers. Does there exist a function f : R → R that simultaneously satisfies the following three conditions? (a) There is a positive number M such that −M ≤ f (x) ≤ M for all x. (b) f (1) = 1. (c) If x 6= 0, then     2 1 1 f x + 2 = f (x) + f . x x 6. A6 (JPN) Let n be an integer, n ≥ 3. Let x1 , x2 , . . . , xn be real numbers such that xi < xi+1 for 1 ≤ i ≤ n − 1. Prove that ! ! n−1 n n(n − 1) ∑ xi x j > ∑ (n − i)xi ∑ ( j − 1)x j . 2 i< j i=1 j=2 7. G1 (BGR)IMO1 Let A, B,C, and D be distinct points on a line, in that order. The circles with diameters AC and BD intersect at X and Y . O is an arbitrary point on the line XY but not on AD. CO intersects the circle with diameter AC again at M, and BO intersects the other circle again at N. Prove that the lines AM, DN, and XY are concurrent. 8. G2 (GER) Let A, B, and C be noncollinear points. Prove that there is a unique point X in the plane of ABC such that X A2 + X B2 + AB2 = X B2 + XC2 + BC2 = XC2 + XA2 +CA2 . 9. G3 (TUR) The incircle of ABC touches BC, CA, and AB at D, E, and F respectively. X is a point inside ABC such that the incircle of X BC touches BC at D also, and touches CX and X B at Y and Z, respectively. Prove that EFZY is a cyclic quadrilateral. 3.36 IMO 1995 277 10. G4 (UKR) An acute triangle ABC is given. Points A1 and A2 are taken on the side BC (with A2 between A1 and C), B1 and B2 on the side AC (with B2 between B1 and A), and C1 and C2 on the side AB (with C2 between C1 and B) such that ∠AA1 A2 = ∠AA2 A1 = ∠BB1 B2 = ∠BB2 B1 = ∠CC1C2 = ∠CC2C1 . The lines AA1 , BB1 , and CC1 form a triangle, and the lines AA2 , BB2 , and CC2 form a second triangle. Prove that all six vertices of these two triangles lie on a single circle. 11. G5 (NZL)IMO5 Let ABCDEF be a convex hexagon with AB = BC = CD, DE = EF = FA, and ∡BCD = ∡EFA = π /3 (that is, 60◦ ). Let G and H be two points interior to the hexagon such that angles AGB and DHE are both 2π /3 (that is, 120◦ ). Prove that AG + GB + GH + DH + HE ≥ CF. 12. G6 (USA) Let A1 A2 A3 A4 be a tetrahedron, G its centroid, and A′1 , A′2 , A′3 , and A′4 the points where the circumsphere of A1 A2 A3 A4 intersects GA1 , GA2 , GA3 , and GA4 , respectively. Prove that GA1 · GA2 · GA3 · GA4 ≤ GA′1 · GA′2 · GA′3 · GA′4 and 1 1 1 1 1 1 1 1 + + + ≤ + + + . GA′1 GA′2 GA′3 GA′4 GA1 GA2 GA3 GA4 13. G7 (LVA) O is a point inside a convex quadrilateral ABCD of area S. K, L, M, and N are interior points of the sides AB, BC,√CD, and respectively. If OKBL √ DA√ and OMDN are parallelograms, prove that S ≥ S1 + S2 , where S1 and S2 are the areas of ONAK and OLCM respectively. 14. G8 (COL) Let ABC be a triangle. A circle passing through B and C intersects the sides AB and AC again at C′ and B′ , respectively. Prove that BB′ ,CC′ , and HH ′ are concurrent, where H and H ′ are the orthocenters of triangles ABC and AB′C′ respectively. 15. N1 (ROU) Let k be a positive integer. Prove that there are infinitely many perfect squares of the form n2k − 7, where n is a positive integer. 16. N2 (RUS) Let Z denote the set of all integers. Prove that for any integers A and B, one can find an integer C for which M1 = {x2 + Ax + B : x ∈ Z} and M2 = {2x2 + 2x + C : x ∈ Z} do not intersect. 17. N3 (CZE)IMO3 Determine all integers n > 3 such that there are n points A1 , A2 , . . . , An in the plane that satisfy the following two conditions simultaneously: (a) No three lie on the same line. (b) There exist real numbers p1 , p2 , . . . , pn such that the area of △Ai A j Ak is equal to pi + p j + pk , for 1 ≤ i < j < k ≤ n. 18. N4 (BGR) Find all positive integers x and y such that x + y2 + z3 = xyz, where z is the greatest common divisor of x and y. 278 3 Problems 19. N5 (IRL) At a meeting of 12k people, each person exchanges greetings with exactly 3k + 6 others. For any two people, the number who exchange greetings with both is the same. How many people are at the meeting? 20. N6 (POL)IMO6 Let p be an odd prime. Find the number of p-element subsets A of {1, 2, . . . , 2p} such that the sum of all elements of A is divisible by p. 21. N7 (BLR) Does there exist an integer n > 1 that satisfies the following condition? The set of positive integers can be partitioned into n nonempty subsets such that an arbitrary sum of n − 1 integers, one taken from each of any n − 1 of the subsets, lies in the remaining subset. 22. N8 (GER)√Let p √ be an odd prime. Determine positive integers x and y for which √ x ≤ y and 2p − x − y is nonnegative and as small as possible. 23. S1 (UKR) Does there exist a sequence F(1), F(2), F(3), . . . of nonnegative integers that simultaneously satisfies the following three conditions? (a) Each of the integers 0, 1, 2, . . . occurs in the sequence. (b) Each positive integer occurs in the sequence infinitely often. (c) For any n ≥ 2,    F F n163 = F(F(n)) + F(F(361)). 24. S2 (POL)IMO4 The positive real numbers x0 , x1 , . . . , x1995 satisfy x0 = x1995 and xi−1 + 2 xi−1 = 2xi + 1 xi for i = 1, 2, . . . , 1995. Find the maximum value that x0 can have. 25. S3 (POL) For an integer x ≥ 1, let p(x) be the least prime that does not divide x, and define q(x) to be the product of all primes less than p(x). In particular, p(1) = 2. For x such that p(x) = 2, define q(x) = 1. Consider the sequence x0 , x1 , x2 , . . . defined by x0 = 1 and xn+1 = xn p(xn ) q(xn ) for n ≥ 0. Find all n such that xn = 1995. 26. S4 (NZL) Suppose that x1 , x2 , x3 , . . . are positive real numbers for which xnn = n−1 ∑ xnj j=0 for n = 1, 2, 3, . . . . Prove that for all n, 2− 1 1 ≤ xn < 2 − n . 2n−1 2 3.36 IMO 1995 279 27. S5 (FIN) For positive integers n, the numbers f (n) are defined inductively as follows: f (1) = 1, and for every positive integer n, f (n + 1) is the greatest integer m such that there is an arithmetic progression of positive integers a1 < a2 < · · · < am = n for which f (a1 ) = f (a2 ) = · · · = f (am ). Prove that there are positive integers a and b such that f (an + b) = n + 2 for every positive integer n. 28. S6 (IND) Let N denote the set of all positive integers. Prove that there exists a unique function f : N → N satisfying f (m + f (n)) = n + f (m + 95) for all m and n in N. What is the value of ∑19 k=1 f (k)? 280 3 Problems 3.37 The Third-Seventh IMO Mumbai, India, July 5–17, 1996 3.37.1 Contest Problems First Day (July 10) 1. We are given a positive integer r and a rectangular board ABCD with dimensions |AB| = 20, |BC| = 12. The rectangle is divided into a grid of 20 × 12 unit squares. The following moves are permitted on the board: One can move from one√square to another only if the distance between the centers of the two squares is r. The task is to find a sequence of moves leading from the square corresponding to vertex A to the square corresponding to vertex B. (a) Show that the task cannot be done if r is divisible by 2 or 3. (b) Prove that the task is possible when r = 73. (c) Is there a solution when r = 97? 2. Let P be a point inside △ABC such that ∠APB − ∠C = ∠APC − ∠B. Let D, E be the incenters of △APB, △APC respectively. Show that AP, BD, and CE meet in a point. 3. Let N0 denote the set of nonnegative integers. Find all functions f from N0 into itself such that f (m + f (n)) = f ( f (m)) + f (n), ∀m, n ∈ N0 . Second Day (July 11) 4. The positive integers a and b are such that the numbers 15a + 16b and 16a − 15b are both squares of positive integers. What is the least possible value that can be taken on by the smaller of these two squares? 5. Let ABCDEF be a convex hexagon such that AB is parallel to DE, BC is parallel to EF, and CD is parallel to AF. Let RA , RC , RE be the circumradii of triangles FAB, BCD, DEF respectively, and let P denote the perimeter of the hexagon. Prove that P RA + RC + RE ≥ . 2 6. Let p, q, n be three positive integers with p + q < n. Let (x0 , x1 , . . . , xn ) be an (n + 1)-tuple of integers satisfying the following conditions: (i) x0 = xn = 0. (ii) For each i with 1 ≤ i ≤ n, either xi − xi−1 = p or xi − xi−1 = −q. Show that there exists a pair (i, j) of distinct indices with (i, j) 6= (0, n) such that xi = x j . 3.37 IMO 1996 281 3.37.2 Shortlisted Problems 1. A1 (SVN) Let a, b, and c be positive real numbers such that abc = 1. Prove that ab bc ca + + ≤ 1. a5 + b5 + ab b5 + c5 + bc c5 + a5 + ca When does equality hold? 2. A2 (IRL) Let a1 ≥ a2 ≥ · · · ≥ an be real numbers such that ak1 + ak2 + · · · + akn ≥ 0 for all integers k > 0. Let p = max{|a1 |, . . . , |an |}. Prove that p = a1 and that (x − a1)(x − a2) · · · (x − an ) ≤ xn − an1 for all x > a1 . 3. A3 (HEL) Let a > 2 be given, and define recursively a0 = 1, a1 = a, an+1 = ! a2n − 2 an . a2n−1 Show that for all k ∈ N, we have  p 1 1 1 1 1 + + + ···+ < 2 + a − a2 − 4 . a0 a 1 a 2 ak 2 4. A4 (KOR) Let a1 , a2 , . . . , an be nonnegative real numbers, not all zero. (a) Prove that xn − a1 xn−1 − · · ·− an−1x − an = 0 has precisely one positive real root. n (b) Let A = ∑nj=1 a j , B = ∑ ja j , and let R be the positive real root of the j=1 equation in part (a). Prove that AA ≤ RB . 5. A5 (ROU) Let P(x) be the real polynomial function P(x) = ax3 + bx2 + cx + d. Prove that if |P(x)| ≤ 1 for all x such that |x| ≤ 1, then |a| + |b| + |c| + |d| ≤ 7. 6. A6 (IRL) Let n be an even positive integer. Prove that there exists a positive integer k such that k = f (x)(x + 1)n + g(x)(xn + 1) for some polynomials f (x), g(x) having integer coefficients. If k0 denotes the least such k, determine k0 as a function of n. A6′ Let n be an even positive integer. Prove that there exists a positive integer k such that 282 3 Problems k = f (x)(x + 1)n + g(x)(xn + 1) for some polynomials f (x), g(x) having integer coefficients. If k0 denotes the least such k, show that k0 = 2q , where q is the odd integer determined by n = q2r , r ∈ N. A6′′ Prove that for each positive integer n, there exist polynomials f (x), g(x) having integer coefficients such that n n f (x)(x + 1)2 + g(x)(x2 + 1) = 2. 7. A7 (ARM) Let f be a function from the set of real numbers R into itself such that for all x ∈ R, we have | f (x)| ≤ 1 and       13 1 1 f x+ + f (x) = f x + + f x+ . 42 6 7 Prove that f is a periodic function (that is, there exists a nonzero real number c such that f (x + c) = f (x) for all x ∈ R). 8. A8 (ROU)IMO3 Let N0 denote the set of nonnegative integers. Find all functions f from N0 into itself such that f (m + f (n)) = f ( f (m)) + f (n), ∀m, n ∈ N0 . 9. A9 (POL) Let the sequence a(n), n = 1, 2, 3, . . ., be generated as follows: a(1) = 0, and for n > 1, a(n) = a([n/2]) + (−1) n(n+1) 2 . (Here [t] = the greatest integer ≤ t.) (a) Determine the maximum and minimum value of a(n) over n ≤ 1996 and find all n ≤ 1996 for which these extreme values are attained. (b) How many terms a(n), n ≤ 1996, are equal to 0? 10. G1 (UNK) Let triangle ABC have orthocenter H, and let P be a point on its circumcircle, distinct from A, B,C. Let E be the foot of the altitude BH, let PAQB and PARC be parallelograms, and let AQ meet HR in X. Prove that EX is parallel to AP. 11. G2 (CAN)IMO2 Let P be a point inside △ABC such that ∠APB − ∠C = ∠APC − ∠B. Let D, E be the incenters of △APB, △APC respectively. Show that AP, BD and CE meet in a point. 12. G3 (UNK) Let ABC be an acute-angled triangle with BC > CA. Let O be the circumcenter, H its orthocenter, and F the foot of its altitude CH. Let the perpendicular to OF at F meet the side CA at P. Prove that ∠FHP = ∠BAC. Possible second part: What happens if |BC| ≤ |CA| (the triangle still being acuteangled)? 3.37 IMO 1996 283 13. G4 (USA) Let △ABC be an equilateral triangle and let P be a point in its interior. Let the lines AP, BP,CP meet the sides BC,CA, AB in the points A1 , B1 ,C1 respectively. Prove that A1 B1 · B1C1 · C1 A1 ≥ A1 B · B1C ·C1 A. 14. G5 (ARM)IMO5 Let ABCDEF be a convex hexagon such that AB is parallel to DE, BC is parallel to EF, and CD is parallel to AF. Let RA , RC , RE be the circumradii of triangles FAB, BCD, DEF respectively, and let P denote the perimeter of the hexagon. Prove that P RA + RC + RE ≥ . 2 15. G6 (ARM) Let the sides of two rectangles be {a, b} and {c, d} with a < c ≤ d < b and ab < cd. Prove that the first rectangle can be placed within the second one if and only if (b2 − a2 )2 ≤ (bd − ac)2 + (bc − ad)2. 16. G7 (UNK) Let ABC be an acute-angled triangle with circumcenter O and circumradius R. Let AO meet the circle BOC again in A′ , let BO meet the circle COA again in B′ , and let CO meet the circle AOB again in C′ . Prove that OA′ · OB′ · OC′ ≥ 8R3 . When does equality hold? 17. G8 (RUS) Let ABCD be a convex quadrilateral, and let RA , RB , RC , and RD denote the circumradii of the triangles DAB, ABC, BCD, and CDA respectively. Prove that RA + RC > RB + RD if and only if ∠A + ∠C > ∠B + ∠D. 18. G9 (UKR) In the plane are given a point O and a polygon F (not necessarily convex). Let P denote the perimeter of F , D the sum of the distances from O to the vertices of F , and H the sum of the distances from O to the lines containing the sides of F . Prove that P2 D2 − H 2 ≥ . 4 19. N1 (UKR) Four integers are marked on a circle. At each step we simultaneously replace each number by the difference between this number and the next number on the circle, in a given direction (that is, the numbers a, b, c, d are replaced by a − b, b − c, c − d, d − a). Is it possible after 1996 such steps to have numbers a, b, c, d such that the numbers |bc − ad|, |ac − bd|, |ab − cd| are primes? 20. N2 (RUS)IMO4 The positive integers a and b are such that the numbers 15a + 16b and 16a − 15b are both squares of positive integers. What is the least possible value that can be taken on by the smaller of these two squares? 284 3 Problems 21. N3 (BGR) A finite sequence of integers a0 , a1 , . . . , an is called quadratic if for each i ∈ {1, 2, . . . , n} we have the equality |ai − ai−1| = i2 . (a) Prove that for any two integers b and c, there exist a natural number n and a quadratic sequence with a0 = b and an = c. (b) Find the smallest natural number n for which there exists a quadratic sequence with a0 = 0 and an = 1996. 22. N4 (BGR) Find all positive integers a and b for which  2  2  2  a b a + b2 + = + ab b a ab where as usual, [t] refers to greatest integer that is less than or equal to t. 23. N5 (ROU) Let N0 denote the set of nonnegative integers. Find a bijective function f from N0 into N0 such that for all m, n ∈ N0 , f (3mn + m + n) = 4 f (m) f (n) + f (m) + f (n). 24. C1 (FIN)IMO1 We are given a positive integer r and a rectangular board ABCD with dimensions |AB| = 20, |BC| = 12. The rectangle is divided into a grid of 20 × 12 unit squares. The following moves are permitted on the board: One can move from one √ square to another only if the distance between the centers of the two squares is r. The task is to find a sequence of moves leading from the square corresponding to vertex A to the square corresponding to vertex B. (a) Show that the task cannot be done if r is divisible by 2 or 3. (b) Prove that the task is possible when r = 73. (c) Is there a solution when r = 97? 25. C2 (UKR) An (n − 1) × (n − 1) square is divided into (n − 1)2 unit squares in the usual manner. Each of the n2 vertices of these squares is to be colored red or blue. Find the number of different colorings such that each unit square has exactly two red vertices. (Two coloring schemes are regarded as different if at least one vertex is colored differently in the two schemes.) 26. C3 (USA) Let k, m, n be integers such that 1 < n ≤ m − 1 ≤ k. Determine the maximum size of a subset S of the set {1, 2, 3, . . .,k} such that no n distinct elements of S add up to m. 27. C4 (FIN) Determine whether or not there exist two disjoint infinite sets A and B of points in the plane satisfying the following conditions: (i) No three points in A ∪ B are collinear, and the distance between any two points in A ∪ B is at least 1. (ii) There is a point of A in any triangle whose vertices are in B, and there is a point of B in any triangle whose vertices are in A . 28. C5 (FRA)IMO6 Let p, q, n be three positive integers with p + q < n. Let (x0 , x1 , . . . , xn ) be an (n + 1)-tuple of integers satisfying the following conditions: (i) x0 = xn = 0. 3.37 IMO 1996 285 (ii) For each i with 1 ≤ i ≤ n, either xi − xi−1 = p or xi − xi−1 = −q. Show that there exists a pair (i, j) of distinct indices with (i, j) 6= (0, n) such that xi = x j . 29. C6 (CAN) A finite number of beans are placed on an infinite row of squares. A sequence of moves is performed as follows: At each stage a square containing more than one bean is chosen. Two beans are taken from this square; one of them is placed on the square immediately to the left, and the other is placed on the square immediately to the right of the chosen square. The sequence terminates if at some point there is at most one bean on each square. Given some initial configuration, show that any legal sequence of moves will terminate after the same number of steps and with the same final configuration. 30. C7 (IRL) Let U be a finite set and let f , g be bijective functions from U onto itself. Let S = {w ∈ U : f ( f (w)) = g(g(w))}, T = {w ∈ U : f (g(w)) = g( f (w))}, and suppose that U = S ∪ T . Prove that for every w ∈ U , f (w) ∈ S if and only if g(w) ∈ S. 286 3 Problems 3.38 The Thirty-Eighth IMO Mar del Plata, Argentina, July 18–31, 1997 3.38.1 Contest Problems First Day (July 24) 1. An infinite square grid is colored in the chessboard pattern. For any pair of positive integers m, n consider a right-angled triangle whose vertices are grid points and whose legs, of lengths m and n, run along the lines of the grid. Let Sb be the total area of the black part of the triangle and Sw the total area of its white part. Define the function f (m, n) = |Sb − Sw|. (a) Calculate f (m, n) for all m, n that have the same parity. (b) Prove that f (m, n) ≤ 12 max(m, n). (c) Show that f (m, n) is not bounded from above. 2. In triangle ABC the angle at A is the smallest. A line through A meets the circumcircle again at the point U lying on the arc BC opposite to A. The perpendicular bisectors of CA and AB meet AU at V and W , respectively, and the lines CV, BW meet at T . Show that AU = T B + TC. 3. Let x1 , x2 , . . . , xn be real numbers satisfying the conditions |x1 + x2 + · · · + xn | = 1 and |xi | ≤ n+1 for i = 1, 2, . . . , n. 2 Show that there exists a permutation y1 , . . . , yn of the sequence x1 , . . . , xn such that n+1 |y1 + 2y2 + · · · + nyn | ≤ . 2 Second Day (July 25) 4. An n × n matrix with entries from {1, 2, . . . , 2n − 1} is called a silver matrix if for each i the union of the ith row and the ith column contains 2n − 1 distinct entries. Show that: (a) There exist no silver matrices for n = 1997. (b) Silver matrices exist for infinitely many values of n. 2 5. Find all pairs of integers x, y ≥ 1 satisfying the equation xy = yx . 6. For a positive integer n, let f (n) denote the number of ways to represent n as the sum of powers of 2 with nonnegative integer exponents. Representations that differ only in the ordering in their summands are not considered to be distinct. (For instance, f (4) = 4 because the number 4 can be represented in the following four ways: 4; 2+2; 2+1+1; 1+1+1+1.) Prove that the inequality 2n holds for any integer n ≥ 3. 2 /4 < f (2n ) < 2n 2 /2 3.38 IMO 1997 287 3.38.2 Shortlisted Problems 1. (BLR)IMO1 An infinite square grid is colored in the chessboard pattern. For any pair of positive integers m, n consider a right-angled triangle whose vertices are grid points and whose legs, of lengths m and n, run along the lines of the grid. Let Sb be the total area of the black part of the triangle and Sw the total area of its white part. Define the function f (m, n) = |Sb − Sw|. (a) Calculate f (m, n) for all m, n that have the same parity. (b) Prove that f (m, n) ≤ 12 max(m, n). (c) Show that f (m, n) is not bounded from above. 2. (CAN) Let R1 , R2 , . . . be the family of finite sequences of positive integers defined by the following rules: R1 = (1), and if Rn−1 = (x1 , . . . , xs ), then Rn = (1, 2, . . . , x1 , 1, 2, . . . , x2 , . . . , 1, 2, . . . , xs , n). For example, R2 = (1, 2), R3 = (1, 1, 2, 3), R4 = (1, 1, 1, 2, 1, 2, 3, 4). Prove that if n > 1, then the kth term from the left in Rn is equal to 1 if and only if the kth term from the right in Rn is different from 1. 3. (GER) For each finite set U of nonzero vectors in the plane we define l(U) to be the length of the vector that is the sum of all vectors in U . Given a finite set V of nonzero vectors in the plane, a subset B of V is said to be maximal if l(B) is greater than or equal to l(A) for each nonempty subset A of V . (a) Construct sets of 4 and 5 vectors that have 8 and 10 maximal subsets respectively. (b) Show that for any set V consisting of n ≥ 1 vectors, the number of maximal subsets is less than or equal to 2n. 4. (IRN)IMO4 An n × n matrix with entries from {1, 2, . . .,2n − 1} is called a coveralls matrix if for each i the union of the ith row and the ith column contains 2n − 1 distinct entries. Show that: (a) There exist no coveralls matrices for n = 1997. (b) Coveralls matrices exist for infinitely many values of n. 5. (ROU) Let ABCD be a regular tetrahedron and M, N distinct points in the planes ABC and ADC respectively. Show that the segments MN, BN, MD are the sides of a triangle. 6. (IRL) (a) Let n be a positive integer. Prove that there exist distinct positive integers x, y, z such that xn−1 + yn = zn+1 . (b) Let a, b, c be positive integers such that a and b are relatively prime and c is relatively prime either to a or to b. Prove that there exist infinitely many triples (x, y, z) of distinct positive integers x, y, z such that xa + yb = zc . 288 3 Problems Original formulation: Let a, b, c, n be positive integers such that n is odd and ac is relatively prime to 2b. Prove that there exist distinct positive integers x, y, z such that (i) xa + yb = zc , and (ii) xyz is relatively prime to n. 7. (RUS) Let ABCDEF be a convex hexagon such that AB = BC, CD = DE, EF = FA. Prove that BC DE FA 3 + + ≥ . BE DA FC 2 When does equality occur? 8. (UNK)IMO2 In triangle ABC the angle at A is the smallest. A line through A meets the circumcircle again at the point U lying on the arc BC opposite to A. The perpendicular bisectors of CA and AB meet AU at V and W , respectively, and the lines CV, BW meet at T . Show that AU = T B + TC. Original formulation. Four different points A, B,C, D are chosen on a circle Γ such that the triangle BCD is not right-angled. Prove that: (a) The perpendicular bisectors of AB and AC meet the line AD at certain points W and V , respectively, and that the lines CV and BW meet at a certain point T. (b) The length of one of the line segments AD, BT , and CT is the sum of the lengths of the other two. 9. (USA) Let A1 A2 A3 be a nonisosceles triangle with incenter I. Let Ci , i = 1, 2, 3, be the smaller circle through I tangent to Ai Ai+1 and Ai Ai+2 (the addition of indices being mod 3). Let Bi , i = 1, 2, 3, be the second point of intersection of Ci+1 and Ci+2 . Prove that the circumcenters of the triangles A1 B1 I, A2 B2 I, A3 B3 I are collinear. 10. (CZE) Find all positive integers k for which the following statement is true: If F(x) is a polynomial with integer coefficients satisfying the condition 0 ≤ F(c) ≤ k for each c ∈ {0, 1, . . . , k + 1}, then F(0) = F(1) = · · · = F(k + 1). 11. (NLD) Let P(x) be a polynomial with real coefficients such that P(x) > 0 for all x ≥ 0. Prove that there exists a positive integer n such that (1 + x)n P(x) is a polynomial with nonnegative coefficients. 12. (ITA) Let p be a prime number and let f (x) be a polynomial of degree d with integer coefficients such that: (i) f (0) = 0, f (1) = 1; (ii) for every positive integer n, the remainder of the division of f (n) by p is either 0 or 1. Prove that d ≥ p − 1. 13. (IND) In town A, there are n girls and n boys, and each girl knows each boy. In town B, there are n girls g1 , g2 , . . . , gn and 2n − 1 boys b1 , b2 , . . ., b2n−1 . The 3.38 IMO 1997 289 girl gi , i = 1, 2, . . . , n, knows the boys b1 , b2 , . . . , b2i−1 , and no others. For all r = 1, 2, . . . , n, denote by A(r), B(r) the number of different ways in which r girls from town A, respectively town B, can dance with r boys from their own town, forming r pairs, each girl with a boy she knows. Prove that A(r) = B(r) for each r = 1, 2, . . . , n. 14. (IND) Let b, m, n be positive integers such that b > 1 and m 6= n. Prove that if bm − 1 and bn − 1 have the same prime divisors, then b + 1 is a power of 2. 15. (RUS) An infinite arithmetic progression whose terms are positive integers contains the square of an integer and the cube of an integer. Show that it contains the sixth power of an integer. 16. (BLR) In an acute-angled triangle ABC, let AD, BE be altitudes and AP, BQ internal bisectors. Denote by I and O the incenter and the circumcenter of the triangle, respectively. Prove that the points D, E, and I are collinear if and only if the points P, Q, and O are collinear. 2 17. (CZE)IMO5 Find all pairs of integers x, y ≥ 1 satisfying the equation xy = yx . 18. (UNK) The altitudes through the vertices A, B,C of an acute-angled triangle ABC meet the opposite sides at D, E, F, respectively. The line through D parallel to EF meets the lines AC and AB at Q and R, respectively. The line EF meets BC at P. Prove that the circumcircle of the triangle PQR passes through the midpoint of BC. 19. (IRL) Let a1 ≥ · · · ≥ an ≥ an+1 = 0 be a sequence of real numbers. Prove that s n n √ √ √ a ≤ ∑ k ∑ k( ak − ak+1). k=1 k=1 20. (IRL) Let D be an internal point on the side BC of a triangle ABC. The line AD meets the circumcircle of ABC again at X . Let P and Q be the feet of the perpendiculars from X to AB and AC, respectively, and let γ be the circle with diameter XD. Prove that the line PQ is tangent to γ if and only if AB = AC. 21. (RUS)IMO3 Let x1 , x2 , . . . , xn be real numbers satisfying the conditions n+1 for i = 1, 2, . . . , n. 2 Show that there exists a permutation y1 , . . . , yn of the sequence x1 , . . . , xn such that n+1 |y1 + 2y2 + · · · + nyn | ≤ . 2 22. (UKR) (a) Do there exist functions f : R → R and g : R → R such that |x1 + x2 + · · · + xn | = 1 f (g(x)) = x2 and and |xi | ≤ g( f (x)) = x3 for all x ∈ R? (b) Do there exist functions f : R → R and g : R → R such that f (g(x)) = x2 and g( f (x)) = x4 for all x ∈ R? 290 3 Problems 23. (UNK) Let ABCD be a convex quadrilateral and O the intersection of its diagonals AC and BD. If OA sin ∠A + OC sin ∠C = OB sin ∠B + OD sin ∠D, prove that ABCD is cyclic. 24. (LTU)IMO6 For a positive integer n, let f (n) denote the number of ways to represent n as the sum of powers of 2 with nonnegative integer exponents. Representations that differ only in the ordering in their summands are not considered to be distinct. (For instance, f (4) = 4 because the number 4 can be represented in the following four ways: 4; 2+2; 2+1+1; 1+1+1+1.) Prove that the inequality 2n 2 /4 < f (2n ) < 2n 2 /2 holds for any integer n ≥ 3. 25. (POL) The bisectors of angles A, B,C of a triangle ABC meet its circumcircle again at the points K, L, M, respectively. Let R be an internal point on the side AB. The points P and Q are defined by the following conditions: RP is parallel to AK, and BP is perpendicular to BL; RQ is parallel to BL, and AQ is perpendicular to AK. Show that the lines KP, LQ, MR have a point in common. 26. (ITA) For every integer n ≥ 2 determine the minimum value that the sum a0 + a1 + · · · + an can take for nonnegative numbers a0 , a1 , . . . , an satisfying the condition a0 = 1, ai ≤ ai+1 + ai+2 for i = 0, . . . , n − 2. 3.39 IMO 1998 291 3.39 The Thirty-Ninth IMO Taipei, Taiwan, July 10–21, 1998 3.39.1 Contest Problems First Day (July 15) 1. A convex quadrilateral ABCD has perpendicular diagonals. The perpendicular bisectors of AB and CD meet at a unique point P inside ABCD. Prove that ABCD is cyclic if and only if triangles ABP and CDP have equal areas. 2. In a contest, there are m candidates and n judges, where n ≥ 3 is an odd integer. Each candidate is evaluated by each judge as either pass or fail. Suppose that each pair of judges agrees on at most k candidates. Prove that k n−1 ≥ . m 2n 3. For any positive integer n, let τ (n) denote the number of its positive divisors (including 1 and itself). Determine all positive integers m for which there exists 2 a positive integer n such that ττ(n(n)) = m. Second Day (July 16) 4. Determine all pairs (x, y) of positive integers such that x2 y + x + y is divisible by xy2 + y + 7. 5. Let I be the incenter of triangle ABC. Let K, L, and M be the points of tangency of the incircle of ABC with AB, BC, and CA, respectively. The line t passes through B and is parallel to KL. The lines MK and ML intersect t at the points R and S. Prove that ∠RIS is acute. 6. Determine the least possible value of f (1998), where f is a function from the set N of positive integers into itself such that for all m, n ∈ N, f (n2 f (m)) = m[ f (n)]2 . 3.39.2 Shortlisted Problems 1. (LUX)IMO1 A convex quadrilateral ABCD has perpendicular diagonals. The perpendicular bisectors of AB and CD meet at a unique point P inside ABCD. Prove that ABCD is cyclic if and only if triangles ABP and CDP have equal areas. 2. (POL) Let ABCD be a cyclic quadrilateral. Let E and F be variable points on the sides AB and CD, respectively, such that AE : EB = CF : FD. Let P be the point on the segment EF such that PE : PF = AB : CD. Prove that the ratio between the areas of triangles APD and BPC does not depend on the choice of E and F. 292 3 Problems 3. (UKR)IMO5 Let I be the incenter of triangle ABC. Let K, L, and M be the points of tangency of the incircle of ABC with AB, BC, and CA, respectively. The line t passes through B and is parallel to KL. The lines MK and ML intersect t at the points R and S. Prove that ∠RIS is acute. 4. (ARM) Let M and N be points inside triangle ABC such that ∠MAB = ∠NAC and ∠MBA = ∠NBC. Prove that AM · AN BM · BN CM ·CN + + = 1. AB · AC BA · BC CA · CB 5. (FRA) Let ABC be a triangle, H its orthocenter, O its circumcenter, and R its circumradius. Let D be the reflection of A across BC, E that of B across CA, and F that of C across AB. Prove that D, E, and F are collinear if and only if OH = 2R. 6. (POL) Let ABCDEF be a convex hexagon such that ∠B + ∠D + ∠F = 360◦ and AB CD EF · · = 1. BC DE FA Prove that BC AE FD · · = 1. CA EF DB 7. (UNK) Let ABC be a triangle such that ∠ACB = 2∠ABC. Let D be the point on the side BC such that CD = 2BD. The segment AD is extended to E so that AD = DE. Prove that ∠ECB + 180◦ = 2∠EBC. 8. (IND) Let ABC be a triangle such that ∠A = 90◦ and ∠B < ∠C. The tangent at A to its circumcircle ω meets the line BC at D. Let E be the reflection of A across BC, X the foot of the perpendicular from A to BE, and Y the midpoint of AX. Let the line BY meet ω again at Z. Prove that the line BD is tangent to the circumcircle of triangle ADZ. 9. (MNG) Let a1 , a2 , . . . , an be positive real numbers such that a1 + a2 + · · · + an < 1. Prove that a1 a2 · · · an [1 − (a1 + a2 + · · · + an )] 1 ≤ . (a1 + a2 + · · · + an )(1 − a1)(1 − a2) · · · (1 − an) nn+1 10. (AUS) Let r1 , r2 , . . . , rn be real numbers greater than or equal to 1. Prove that 1 1 1 n + + ···+ ≥√ . n r1 + 1 r2 + 1 rn + 1 r1 r2 · · · rn + 1 11. (RUS) Let x, y, and z be positive real numbers such that xyz = 1. Prove that x3 y3 z3 3 + + ≥ . (1 + y)(1 + z) (1 + z)(1 + x) (1 + x)(1 + y) 4 3.39 IMO 1998 293 12. (POL) Let n ≥ k ≥ 0 be integers. The numbers c(n, k) are defined as follows: c(n, 0) = c(n, n) = 1 c(n + 1, k) = 2k c(n, k) + c(n, k − 1) for all n ≥ 0; for n ≥ k ≥ 1. Prove that c(n, k) = c(n, n − k) for all n ≥ k ≥ 0. 13. (BGR)IMO6 Determine the least possible value of f (1998), where f is a function from the set N of positive integers into itself such that for all m, n ∈ N, f (n2 f (m)) = m[ f (n)]2 . 14. (UNK)IMO4 Determine all pairs (x, y) of positive integers such that x2 y + x + y is divisible by xy2 + y + 7. 15. (AUS) Determine all pairs (a, b) of real numbers such that a⌊bn⌋ = b⌊an⌋ for all positive integers n. (Note that ⌊x⌋ denotes the greatest integer less than or equal to x.) 16. (UKR) Determine the smallest integer n ≥ 4 for which one can choose four different numbers a, b, c, and d from any n distinct integers such that a+b−c−d is divisible by 20. 17. (UNK) A sequence of integers a1 , a2 , a3 , . . . is defined as follows: a1 = 1, and for n ≥ 1, an+1 is the smallest integer greater than an such that ai + a j 6= 3ak for any i, j, k in {1, 2, . . . , n + 1}, not necessarily distinct. Determine a1998 . 18. (BGR) Determine all positive integers n for which there exists an integer m such that 2n − 1 is a divisor of m2 + 9. 19. (BLR)IMO3 For any positive integer n, let τ (n) denote the number of its positive divisors (including 1 and itself). Determine all positive integers m for which 2 there exists a positive integer n such that ττ(n(n)) = m. 20. (ARG) Prove that for each positive integer n, there exists a positive integer with the following properties: (i) It has exactly n digits. (ii) None of the digits is 0. (iii) It is divisible by the sum of its digits. 21. (CAN) Let a0 , a1 , a2 , . . . be an increasing sequence of nonnegative integers such that every nonnegative integer can be expressed uniquely in the form ai + 2a j + 4ak , where i, j, k are not necessarily distinct. Determine a1998 . 22. (UKR) A rectangular array of numbers is given. In each row and each column, the sum of all numbers is an integer. Prove that each nonintegral number x in the array can be changed into either ⌈x⌉ or ⌊x⌋ so that the row sums and column sums remain unchanged. (Note that ⌈x⌉ is the least integer greater than or equal to x, while ⌊x⌋ is the greatest integer less than or equal to x.) 294 3 Problems 23. (BLR) Let n be an integer greater than 2. A positive integer is said to be attainable if it is 1 or can be obtained from 1 by a sequence of operations with the following properties: (i) The first operation is either addition or multiplication. (ii) Thereafter, additions and multiplications are used alternately. (iii) In each addition one can choose independently whether to add 2 or n. (iv) In each multiplication, one can choose independently whether to multiply by 2 or by n. A positive integer that cannot be so obtained is said to be unattainable. (a) Prove that if n ≥ 9, there are infinitely many unattainable positive integers. (b) Prove that if n = 3, all positive integers except 7 are attainable. 24. (SWE) Cards numbered 1 to 9 are arranged at random in a row. In a move, one may choose any block of consecutive cards whose numbers are in ascending or descending order, and switch the block around. For example, 916532748 may be changed to 913562748. Prove that in at most 12 moves, one can arrange the 9 cards so that their numbers are in ascending or descending order. 25. (NZL) Let U = {1, 2, . . . , n}, where n ≥ 3. A subset S of U is said to be split by an arrangement of the elements of U if an element not in S occurs in the arrangement somewhere between two elements of S. For example, 13542 splits {1, 2, 3} but not {3, 4, 5}. Prove that for any n − 2 subsets of U, each containing at least 2 and at most n − 1 elements, there is an arrangement of the elements of U that splits all of them. 26. (IND)IMO2 In a contest, there are m candidates and n judges, where n ≥ 3 is an odd integer. Each candidate is evaluated by each judge as either pass or fail. Suppose that each pair of judges agrees on at most k candidates. Prove that mk ≥ n−1 2n . 27. (BLR) Ten points such that no three of them lie on a line are marked in the plane. Each pair of points is connected with a segment. Each of these segments is painted with one of k colors in such a way that for any k of the ten points, there are k segments each joining two of them with no two being painted the same color. Determine all integers k, 1 ≤ k ≤ 10, for which this is possible. 28. (IRN) A solitaire game is played on an m × n rectangular board, using mn markers that are white on one side and black on the other. Initially, each square of the board contains a marker with its white side up, except for one corner square, which contains a marker with its black side up. In each move, one can take away one marker with its black side up, but must then turn over all markers that are in squares having an edge in common with the square of the removed marker. Determine all pairs (m, n) of positive integers such that all markers can be removed from the board. 3.40 IMO 1999 295 3.40 The Fortieth IMO Bucharest, Romania, July 10–22, 1999 3.40.1 Contest Problems First Day (July 16) 1. A set S of points in the plane will be called completely symmetric if it has at least three elements and satisfies the following condition: For every two distinct points A, B from S the perpendicular bisector of the segment AB is an axis of symmetry for S. Prove that if a completely symmetric set is finite, then it consists of the vertices of a regular polygon. 2. Let n ≥ 2 be a fixed integer. Find the least constant C such that the inequality !4 ∑ xi x j (x2i + x2j ) ≤ C ∑ xi i< j i holds for every x1 , . . . , xn ≥ 0 (the sum on the left consists of For this constant C, characterize the instances of equality.  n 2 summands). 3. Let n be an even positive integer. We say that two different cells of an n × n board are neighboring if they have a common side. Find the minimal number of cells on the n × n board that must be marked so that every cell (marked or not marked) has a marked neighboring cell. Second Day (July 17) 4. Find all pairs of positive integers (x, p) such that p is a prime, x ≤ 2p , and x p−1 is a divisor of (p − 1)x + 1. 5. Two circles Ω1 and Ω 2 touch internally the circle Ω in M and N, and the center of Ω 2 is on Ω1 . The common chord of the circles Ω1 and Ω2 intersects Ω in A and B. MA and MB intersect Ω1 in C and D. Prove that Ω2 is tangent to CD. 6. Find all the functions f : R → R that satisfy f (x − f (y)) = f ( f (y)) + x f (y) + f (x) − 1 for all x, y ∈ R. 3.40.2 Shortlisted Problems 1. N1 (TWN)IMO4 Find all pairs of positive integers (x, p) such that p is a prime, x ≤ 2p , and x p−1 is a divisor of (p − 1)x + 1. 2. N2 (ARM) Prove that every positive rational number can be represented in the a3 + b3 form 3 , where a, b, c, d are positive integers. c + d3 296 3 Problems 3. N3 (RUS) Prove that there exist two strictly increasing sequences (an ) and (bn ) such that an (an + 1) divides b2n + 1 for every natural number n. 4. N4 (FRA) Denote by S the set of all primes p such that the decimal representation of 1p has its fundamental period divisible by 3. For every p ∈ S such that 1 1 p has its fundamental period 3r one may write p = 0.a1 a2 . . . a3r a1 a2 . . . a3r . . . , where r = r(p); for every p ∈ S and every integer k ≥ 1 define f (k, p) by f (k, p) = ak + ak+r(p) + ak+2r(p) . (a) Prove that S is infinite. (b) Find the highest value of f (k, p) for k ≥ 1 and p ∈ S. 5. N5 (ARM) Let n, k be positive integers such that n is not divisible by 3 and k ≥ n. Prove that there exists a positive integer m that is divisible by n and the sum of whose digits in decimal representation is k. 6. N6 (BLR) Prove that for every real number M there exists an infinite arithmetic progression such that: (i) each term is a positive integer and the common difference is not divisible by 10; (ii) the sum of the digits of each term (in decimal representation) exceeds M. 7. G1 (ARM) Let ABC be a triangle and M an interior point. Prove that min{MA, MB, MC} + MA + MB + MC < AB + AC + BC. 8. G2 (JPN) A circle is called a separator for a set of five points in a plane if it passes through three of these points, it contains a fourth point in its interior, and the fifth point is outside the circle. Prove that every set of five points such that no three are collinear and no four are concyclic has exactly four separators. 9. G3 (EST)IMO1 A set S of points in space will be called completely symmetric if it has at least three elements and satisfies the following condition: For every two distinct points A, B from S the perpendicular bisector of the segment AB is an axis of symmetry for S. Prove that if a completely symmetric set is finite, then it consists of the vertices of either a regular polygon, a regular tetrahedron, or a regular octahedron. 10. G4 (UNK) For a triangle T = ABC we take the point X on the side (AB) such that AX/XB = 4/5, the point Y on the segment (CX) such that CY = 2Y X, and, if possible, the point Z on the ray (CA such that ∡CX Z = 180◦ − ∡ABC. We denote by Σ the set of all triangles T for which ∡XY Z = 45◦ . Prove that all the triangles from Σ are similar and find the measure of their smallest angle. 11. G5 (FRA) Let ABC be a triangle, Ω its incircle and Ωa , Ωb , Ωc three circles orthogonal to Ω passing through B and C, A and C, and A and B respectively. 3.40 IMO 1999 297 The circles Ωa , Ωb meet again in C′ ; in the same way we obtain the points B′ and A′ . Prove that the radius of the circumcircle of A′ B′C′ is half the radius of Ω . 12. G6 (RUS)IMO5 Two circles Ω1 and Ω2 touch internally the circle Ω in M and N, and the center of Ω2 is on Ω1 . The common chord of the circles Ω1 and Ω 2 intersects Ω in A and B. MA and MB intersect Ω 1 in C and D. Prove that Ω2 is tangent to CD. 13. G7 (ARM) The point M inside the convex quadrilateral ABCD is such that MA = MC, ∠AMB = ∠MAD + ∠MKD, ∠CMD = ∠MCB + ∠MAB. Prove that AB ·CM = BC · MD and BM · AD = MA ·CD. 14. G8 (RUS) Points A, B,C divide the circumcircle Ω of the triangle ABC into three arcs. Let X be a variable point on the arc AB, and let O1 , O2 be the incenters of the triangles CAX and CBX . Prove that the circumcircle of the triangle X O1 O2 intersects Ω in a fixed point. 15. A1 (POL)IMO2 Let n ≥ 2 be a fixed integer. Find the least constant C such that the inequality !4 ∑ xi x j (x2i + x2j ) ≤ C ∑ xi i< j i holds for every x1 , . . . , xn ≥ 0 (the sum on the left consists of For this constant C, characterize the instances of equality. n 2 summands). 16. A2 (RUS) The numbers from 1 to n2 are randomly arranged in the cells of an n × n square (n ≥ 2). For any pair of numbers situated in the same row or in the same column, the ratio of the greater number to the smaller one is calculated. Let us call the characteristic of the arrangement the smallest of these n2 (n − 1) fractions. What is the highest possible value of the characteristic? 17. A3 (FIN) A game is played by n girls (n ≥ 2), everybody having a ball. Each of the n2 pairs of players, in an arbitrary order, exchange the balls they have at that moment. The game is called nice if at the end nobody has her own ball, and it is called tiresome if at the end everybody has her initial ball. Determine the values of n for which there exists a nice game and those for which there exists a tiresome game. 18. A4 (BLR) Prove that the set of positive integers cannot be partitioned into three nonempty subsets such that for any two integers x, y taken from two different subsets, the number x2 − xy + y2 belongs to the third subset. 19. A5 (JPN)IMO6 Find all the functions f : R → R that satisfy f (x − f (y)) = f ( f (y)) + x f (y) + f (x) − 1 for all x, y ∈ R. 20. A6 (SWE) For n ≥ 3 and a1 ≤ a2 ≤ · · · ≤ an given real numbers we have the following instructions: 298 3 Problems (1) place the numbers in some order in a circle; (2) delete one of the numbers from the circle; (3) if just two numbers are remaining in the circle, let S be the sum of these two numbers. Otherwise, if there are more than two numbers in the circle, replace (x1 , x2 , x3 , . . . , x p−1 , x p ) with (x1 + x2 , x2 + x3 , . . . , x p−1 + x p , x p + x1 ). Afterwards, start again with step (2). Show that the largest sum S that can result in this way is given by the formula  n  n−2 Smax = ∑  k  ak . 2 −1 k=2 21. C1 (IND) Let n ≥ 1 be an integer. A path from (0, 0) to (n, n) in the xy plane is a chain of consecutive unit moves either to the right (move denoted by E) or upwards (move denoted by N), all the moves being made inside the half-plane x ≥ y. A step in a path is the occurrence of two consecutive moves of the form EN. Show that the number of paths from (0, 0) to (n, n) that contain exactly s steps (n ≥ s ≥ 1) is    1 n−1 n . s s−1 s−1 22. C2 (CAN) (a) If a 5 × n rectangle can be tiled using n pieces like those shown in the diagram, prove that n is even. (b) Show that there are more than 2 · 3k−1 ways to tile a fixed 5 × 2k rectangle (k ≥ 3) with 2k pieces. (Symmetric constructions are considered to be different.) 23. C3 (UNK) A biologist watches a chameleon. The chameleon catches flies and rests after each catch. The biologist notices that: (i) the first fly is caught after a resting period of one minute; (ii) the resting period before catching the 2mth fly is the same as the resting period before catching the mth fly and one minute shorter than the resting period before catching the (2m + 1)th fly; (iii) when the chameleon stops resting, he catches a fly instantly. (a) How many flies were caught by the chameleon before his first resting period of 9 minutes? (b) After how many minutes will the chameleon catch his 98th fly? (c) How many flies were caught by the chameleon after 1999 minutes passed? 24. C4 (UNK) Let A be a set of N residues (mod N 2 ). Prove that there exists a set B of N residues (mod N 2 ) such that the set A + B = {a + b | a ∈ A, b ∈ B} contains at least half of all residues (mod N 2 ). 3.40 IMO 1999 299 25. C5 (BLR)IMO3 Let n be an even positive integer. We say that two different cells of an n × n board are neighboring if they have a common side. Find the minimal number of cells on the n × n board that must be marked so that every cell (marked or not marked) has a marked neighboring cell. 26. C6 (UNK) Suppose that every integer has been given one of the colors red, blue, green, yellow. Let x and y be odd integers such that |x| = 6 |y|. Show that there are two integers of the same color whose difference has one of the following values: x, y, x + y, x − y. 27. C7 (IRL) Let p > 3 be a prime number. For each nonempty subset T of {0, 1, 2, 3, . . ., p − 1} let E(T ) be the set of all (p − 1)-tuples (x1 , . . . , x p−1 ), where each xi ∈ T and x1 + 2x2 + · · · + (p − 1)x p−1 is divisible by p and let |E(T )| denote the number of elements in E(T ). Prove that |E({0, 1, 3})| ≥ |E({0, 1, 2})|, with equality if and only if p = 5. 300 3 Problems 3.41 The Forty-First IMO Taejon, South Korea, July 13–25, 2000 3.41.1 Contest Problems First day (July 18) 1. Two circles G1 and G2 intersect at M and N. Let AB be the line tangent to these circles at A and B, respectively, such that M lies closer to AB than N. Let CD be the line parallel to AB and passing through M, with C on G1 and D on G2 . Lines AC and BD meet at E; lines AN and CD meet at P; lines BN and CD meet at Q. Show that EP = EQ. 2. Let a, b, c be positive real numbers with product 1. Prove that     1 1 1 a−1+ b−1+ c−1+ ≤ 1. b c a 3. Let n ≥ 2 be a positive integer and λ a positive real number. Initially there are n fleas on a horizontal line, not all at the same point. We define a move of choosing two fleas at some points A and B, with A to the left of B, and letting the flea from A jump over the flea from B to the point C such that BC/AB = λ . Determine all values of λ such that for any point M on the line and for any initial position of the n fleas, there exists a sequence of moves that will take them all to a position right of M. Second Day (July 19) 4. A magician has one hundred cards numbered 1 to 100. He puts them into three boxes, a red one, a white one, and a blue one, so that each box contains at least one card. A member of the audience draws two cards from two different boxes and announces the sum of numbers on those cards. Given this information, the magician locates the box from which no card has been drawn. How many ways are there to put the cards in the three boxes so that the trick works? 5. Does there exist a positive integer n such that n has exactly 2000 prime divisors and 2n + 1 is divisible by n? 6. A1 A2 A3 is an acute-angled triangle. The foot of the altitude from Ai is Ki , and the incircle touches the side opposite Ai at Li . The line K1 K2 is reflected in the line L1 L2 . Similarly, the line K2 K3 is reflected in L2 L3 and K3 K1 is reflected in L3 L1 . Show that the three new lines form a triangle with vertices on the incircle. 3.41.2 Shortlisted Problems 1. C1 (HUN)IMO4 A magician has one hundred cards numbered 1 to 100. He puts them into three boxes, a red one, a white one, and a blue one, so that each box 3.41 IMO 2000 301 contains at least one card. A member of the audience draws two cards from two different boxes and announces the sum of numbers on those cards. Given this information, the magician locates the box from which no card has been drawn. How many ways are there to put the cards in the three boxes so that the trick works? 2. C2 (ITA) A brick staircase with three steps of width 2 is made of twelve unit cubes. Determine all integers n for which it is possible to build a cube of side n using such bricks. 3. C3 (COL) Let n ≥ 4 be a fixed positive integer. Given a set S = {P1, P2 , . . . , Pn } of points in the plane such that no three are collinear and no four concyclic, let at , 1 ≤ t ≤ n, be the number of circles Pi Pj Pk that contain Pt in their interior, and let m(S) = a1 + a2 + · · · + an . Prove that there exists a positive integer f (n), depending only on n, such that the points of S are the vertices of a convex polygon if and only if m(S) = f (n). 4. C4 (CZE) Let n and k be positive integers such that n/2 < k ≤ 2n/3. Find the least number m for which it is possible to place m pawns on m squares of an n ×n chessboard so that no column or row contains a block of k adjacent unoccupied squares. 5. C5 (RUS) In the plane we have n rectangles with parallel sides. The sides of distinct rectangles lie on distinct lines. The boundaries of the rectangles cut the plane into connected regions. A region is nice if it has at least one of the vertices of the n rectangles on its boundary. Prove that the sum of the numbers of the vertices of all nice regions is less than 40n. (There can be nonconvex regions as well as regions with more than one boundary curve.) 6. C6 (FRA) Let p and q be relatively prime positive integers. A subset S of {0, 1, 2, . . . } is called ideal if 0 ∈ S and for each element n ∈ S, the integers n + p and n + q belong to S. Determine the number of ideal subsets of {0, 1, 2 . . .}. 7. A1 (USA)IMO2 Let a, b, c be positive real numbers with product 1. Prove that     1 1 1 a−1+ b−1+ c−1+ ≤ 1. b c a 8. A2 (UNK) Let a, b, c be positive integers satisfying the conditions b > 2a and c > 2b. Show that there exists a real number t with the property that all the three numbers ta,tb,tc have their fractional parts lying in the interval (1/3, 2/3]. 9. A3 (BLR) Find all pairs of functions f : R → R, g : R → R such that f (x + g(y)) = x f (y) − y f (x) + g(x) for all x, y ∈ R. 302 3 Problems 10. A4 (UNK) The function F is defined on the set of nonnegative integers and takes nonnegative integer values satisfying the following conditions: For every n ≥ 0, (i) F(4n) = F(2n) + F(n); (ii) F(4n + 2) = F(4n) + 1; (iii) F(2n + 1) = F(2n) + 1. Prove that for each positive integer m, the number of integers n with 0 ≤ n < 2m and F(4n) = F(3n) is F(2m+1 ). 11. A5 (BLR)IMO3 Let n ≥ 2 be a positive integer and λ a positive real number. Initially there are n fleas on a horizontal line, not all at the same point. We define a move of choosing two fleas at some points A and B, with A to the left of B, and letting the flea from A jump over the flea from B to the point C such that BC/AB = λ . Determine all values of λ such that for any point M on the line and for any initial position of the n fleas, there exists a sequence of moves that will take them all to a position right of M. 12. A6 (IRL) A nonempty set A of real numbers is called a B3 -set if the conditions a1 , a2 , a3 , a4 , a5 , a6 ∈ A and a1 + a2 + a3 = a4 + a5 + a6 imply that the sequences (a1 , a2 , a3 ) and (a4 , a5 , a6 ) are identical up to a permutation. Let A = {a0 = 0 < a1 < a2 < · · · }, B = {b0 = 0 < b1 < b2 < · · · } be infinite sequences of real numbers with D(A) = D(B), where, for a set X of real numbers, D(X ) denotes the difference set {|x − y| | x, y ∈ X }. Prove that if A is a B3 -set, then A = B. 13. A7 (RUS) For a polynomial P of degree 2000 with distinct real coefficients let M(P) be the set of all polynomials that can be produced from P by permutation of its coefficients. A polynomial P will be called n-independent if P(n) = 0 and we can get from any Q in M(P) a polynomial Q1 such that Q1 (n) = 0 by interchanging at most one pair of coefficients of Q. Find all integers n for which n-independent polynomials exist. 14. N1 (JPN) Determine all positive integers n ≥ 2 that satisfy the following condition: For all integers a, b relatively prime to n, a ≡ b (mod n) if and only if ab ≡ 1 (mod n). 15. N2 (FRA) For a positive integer n, let d(n) be the number of all positive divisors of n. Find all positive integers n such that d(n)3 = 4n. 16. N3 (RUS)IMO5 Does there exist a positive integer n such that n has exactly 2000 prime divisors and 2n + 1 is divisible by n? 17. N4 (BRA) Determine all triples of positive integers (a, m, n) such that am + 1 divides (a + 1)n . 18. N5 (BGR) Prove that there exist infinitely many positive integers n such that p = nr, where p and r are respectively the semiperimeter and the inradius of a triangle with integer side lengths. 3.41 IMO 2000 303 19. N6 (ROU) Show that the set of positive integers that cannot be represented as a sum of distinct perfect squares is finite. 20. G1 (NLD) In the plane we are given two circles intersecting at X and Y . Prove that there exist four points A, B,C, D with the following property: For every circle touching the two given circles at A and B, and meeting the line XY at C and D, each of the lines AC, AD, BC, BD passes through one of these points. 21. G2 (RUS)IMO1 Two circles G1 and G2 intersect at M and N. Let AB be the line tangent to these circles at A and B, respectively, such that M lies closer to AB than N. Let CD be the line parallel to AB and passing through M, with C on G1 and D on G2 . Lines AC and BD meet at E; lines AN and CD meet at P; lines BN and CD meet at Q. Show that EP = EQ. 22. G3 (IND) Let O be the circumcenter and H the orthocenter of an acute triangle ABC. Show that there exist points D, E, and F on sides BC, CA, and AB respectively such that OD + DH = OE + EH = OF + FH and the lines AD, BE, and CF are concurrent. 23. G4 (RUS) Let A1 A2 . . . An be a convex polygon, n ≥ 4. Prove that A1 A2 . . . An is cyclic if and only if to each vertex A j one can assign a pair (b j , c j ) of real numbers, j = 1, 2, . . . n, such that Ai A j = b j ci − bi c j for all i, j with 1 ≤ i ≤ j ≤ n. 24. G5 (UNK) The tangents at B and A to the circumcircle of an acute-angled triangle ABC meet the tangent at C at T and U respectively. AT meets BC at P, and Q is the midpoint of AP; BU meets CA at R, and S is the midpoint of BR. Prove that ∠ABQ = ∠BAS. Determine, in terms of ratios of side lengths, the triangles for which this angle is a maximum. 25. G6 (ARG) Let ABCD be a convex quadrilateral with AB not parallel to CD, let X be a point inside ABCD such that ∡ADX = ∡BCX < 90◦ and ∡DAX = ∡CBX < 90◦ . If Y is the point of intersection of the perpendicular bisectors of AB and CD, prove that ∡AY B = 2∡ADX. 26. G7 (IRN) Ten gangsters are standing on a flat surface, and the distances between them are all distinct. At twelve o’clock, when the church bells start chiming, each of them fatally shoots the one among the other nine gangsters who is the nearest. At least how many gangsters will be killed? 27. G8 (RUS)IMO6 A1 A2 A3 is an acute-angled triangle. The foot of the altitude from Ai is Ki , and the incircle touches the side opposite Ai at Li . The line K1 K2 is reflected in the line L1 L2 . Similarly, the line K2 K3 is reflected in L2 L3 , and K3 K1 is reflected in L3 L1 . Show that the three new lines form a triangle with vertices on the incircle. 304 3 Problems 3.42 The Forty-Second IMO Washington DC, United States of America, July 1–14, 2001 3.42.1 Contest Problems First Day (July 8) 1. In acute triangle ABC with circumcenter O and altitude AP, ∡C ≥ ∡B + 30◦ . Prove that ∡A + ∡COP < 90◦ . 2. Prove that for all positive real numbers a, b, c, √ a a2 + 8bc +√ b b2 + 8ca +√ c c2 + 8ab ≥ 1. 3. Twenty-one girls and twenty-one boys took part in a mathematical competition. It turned out that (i) each contestant solved at most six problems, and (ii) for each pair of a girl and a boy, there was at least one problem that was solved by both the girl and the boy. Show that there is a problem that was solved by at least three girls and at least three boys. Second Day (July 9) 4. Let n be an odd integer greater than 1 and let c1 , c2 , . . . , cn be integers. For each permutation a = (a1 , a2 , . . . , an ) of {1, 2, . . . , n}, define S(a) = ∑ni=1 ci ai . Prove that there exist permutations a 6= b of {1, 2, . . ., n} such that n! is a divisor of S(a) − S(b). 5. Let ABC be a triangle with ∡BAC = 60◦ . Let AP bisect ∠BAC and let BQ bisect ∠ABC, with P on BC and Q on AC. If AB + BP = AQ + QB, what are the angles of the triangle? 6. Let a > b > c > d be positive integers and suppose ac + bd = (b + d + a − c)(b + d − a + c). Prove that ab + cd is not prime. 3.42.2 Shortlisted Problems 1. A1 (IND) Let T denote the set of all ordered triples (p, q, r) of nonnegative integers. Find all functions f : T → R such that  0 if pqr = 0,    1  1 + ( f (p + 1, q − 1, r) + f (p − 1, q + 1, r) f (p, q, r) = 6   + f (p − 1, q, r + 1) + f (p + 1, q, r − 1)   + f (p, q + 1, r − 1) + f (p, q − 1, r + 1)) otherwise. 3.42 IMO 2001 305 2. A2 (POL) Let a0 , a1 , a2 , . . . be an arbitrary √infinite sequence of positive numbers. Show that the inequality 1+ an > an−1 n 2 holds for infinitely many positive integers n. 3. A3 (ROU) Let x1 , x2 , . . . , xn be arbitrary real numbers. Prove the inequality √ x1 x2 xn + + ···+ < n. 1 + x21 1 + x21 + x22 1 + x21 + · · · + x2n 4. A4 (LTU) Find all functions f : R → R satisfying f (xy)( f (x) − f (y)) = (x − y) f (x) f (y) for all x, y. 5. A5 (BGR) Find all positive integers a1 , a2 , . . . , an such that 99 a 0 a1 an−1 = + + ···+ , 100 a1 a2 an where a0 = 1 and (ak+1 − 1)ak−1 ≥ a2k (ak − 1) for k = 1, 2, . . . , n − 1. 6. A6 (KOR)IMO2 Prove that for all positive real numbers a, b, c, √ a a2 + 8bc +√ b b2 + 8ca +√ c c2 + 8ab ≥ 1. 7. C1 (COL) Let A = (a1 , a2 , . . . , a2001 ) be a sequence of positive integers. Let m be the number of 3-element subsequences (ai , a j , ak ) with 1 ≤ i < j < k ≤ 2001 such that a j = ai + 1 and ak = a j + 1. Considering all such sequences A, find the greatest value of m. 8. C2 (CAN)IMO4 Let n be an odd integer greater than 1 and let c1 , c2 , . . . , cn be integers. For each permutation a = (a1 , a2 , . . . , an ) of {1, 2, . . . , n}, define S(a) = ∑ni=1 ci ai . Prove that there exist permutations a 6= b of {1, 2, . . ., n} such that n! is a divisor of S(a) − S(b). 9. C3 (RUS) Define a k-clique to be a set of k people such that every pair of them are acquainted with each other. At a certain party, every pair of 3-cliques has at least one person in common, and there are no 5-cliques. Prove that there are two or fewer people at the party whose departure leaves no 3-clique remaining. 10. C4 (NZL) A set of three nonnegative integers {x, y, z} with x < y < z is called historic if {z − y, y − x} = {1776, 2001}. Show that the set of all nonnegative integers can be written as the union of disjoint historic sets. 11. C5 (FIN) Find all finite sequences (x0 , x1 , . . . , xn ) such that for every j, 0 ≤ j ≤ n, x j equals the number of times j appears in the sequence. 12. C6 (CAN) For a positive integer n define a sequence of zeros and ones to be balanced if it contains n zeros and n ones. Two balanced sequences a and b are neighbors if you can move one of the 2n symbols of a to another position to form 306 3 Problems b. For instance, when n = 4, the balanced sequences 01101001 and 00110101 are neighbors because the third (or fourth) zero in the first sequence can be moved to the first or second position to form the second sequence. Prove that there is a  1 2n set S of at most n+1 balanced sequences such that every balanced sequence n is equal to or is a neighbor of at least one sequence in S. 13. C7 (FRA) A pile of n pebbles is placed in a vertical column. This configuration is modified according to the following rules. A pebble can be moved if it is at the top of a column that contains at least two more pebbles than the column immediately to its right. (If there are no pebbles to the right, think of this as a column with 0 pebbles.) At each stage, choose a pebble from among those that can be moved (if there are any) and place it at the top of the column to its right. If no pebbles can be moved, the configuration is called a final configuration. For each n, show that no matter what choices are made at each stage, the final configuration is unique. Describe that configuration in terms of n. 14. C8 (GER)IMO3 Twenty-one girls and twenty-one boys took part in a mathematical competition. It turned out that (i) each contestant solved at most six problems, and (ii) for each pair of a girl and a boy, there was at least one problem that was solved by both the girl and the boy. Show that there is a problem that was solved by at least three girls and at least three boys. 15. G1 (UKR) Let A1 be the center of the square inscribed in acute triangle ABC with two vertices of the square on side BC. Thus one of the two remaining vertices of the square is on side AB and the other is on AC. Points B1 ,C1 are defined in a similar way for inscribed squares with two vertices on sides AC and AB, respectively. Prove that lines AA1 , BB1 ,CC1 are concurrent. 16. G2 (KOR)IMO1 In acute triangle ABC with circumcenter O and altitude AP, ∡C ≥ ∡B + 30◦. Prove that ∡A + ∡COP < 90◦ . 17. G3 (UNK) Let ABC be a triangle with centroid G. Determine, with proof, the position of the point P in the plane of ABC such that AP · AG + BP· BG + CP ·CG is a minimum, and express this minimum value in terms of the side lengths of ABC. 18. G4 (FRA) Let M be a point in the interior of triangle ABC. Let A′ lie on BC with MA′ perpendicular to BC. Define B′ on CA and C′ on AB similarly. Define p(M) = MA′ · MB′ · MC′ . MA · MB · MC Determine, with proof, the location of M such that p(M) is maximal. Let µ (ABC) denote the maximum value. For which triangles ABC is the value of µ (ABC) maximal? 3.42 IMO 2001 307 19. G5 (HEL) Let ABC be an acute triangle. Let DAC, EAB, and FBC be isosceles triangles exterior to ABC, with DA = DC, EA = EB, and FB = FC such that ∠ADC = 2∠BAC, ∠BEA = 2∠ABC, ∠CFB = 2∠ACB. D′ Let be the intersection of lines DB and EF, let E ′ be the intersection of EC and DF, and let F ′ be the intersection of FA and DE. Find, with proof, the value of the sum DB EC FA + + . DD′ EE ′ FF ′ 20. G6 (IND) Let ABC be a triangle and P an exterior point in the plane of the triangle. Suppose AP, BP,CP meet the sides BC,CA, AB (or extensions thereof) in D, E, F, respectively. Suppose further that the areas of triangles PBD, PCE, PAF are all equal. Prove that each of these areas is equal to the area of triangle ABC itself. 21. G7 (BGR) Let O be an interior point of acute triangle ABC. Let A1 lie on BC with OA1 perpendicular to BC. Define B1 on CA and C1 on AB similarly. Prove that O is the circumcenter of ABC if and only if the perimeter of A1 B1C1 is not less than any one of the perimeters of AB1C1 , BC1 A1 , and CA1 B1 . 22. G8 (ISR)IMO5 Let ABC be a triangle with ∡BAC = 60◦ . Let AP bisect ∠BAC and let BQ bisect ∠ABC, with P on BC and Q on AC. If AB + BP = AQ + QB, what are the angles of the triangle? 23. N1 (AUS) Prove that there is no positive integer n such that for k = 1, 2, . . . , 9, the leftmost digit (in decimal notation) of (n + k)! equals k. 24. N2 (COL) Consider the system x + y = z + u, 2xy = zu. Find the greatest value of the real constant m such that m ≤ x/y for every positive integer solution x, y, z, u of the system with x ≥ y. 25. N3 (UNK) Let a1 = 1111, a2 = 1212, a3 = 1313, and an = |an−1 − an−2 | + |an−2 − an−3 |, Determine a1414 . n ≥ 4. 26. N4 (VNM) Let p ≥ 5 be a prime number. Prove that there exists an integer a with 1 ≤ a ≤ p − 2 such that neither a p−1 − 1 nor (a + 1) p−1 − 1 is divisible by p2 . 27. N5 (BGR)IMO6 Let a > b > c > d be positive integers and suppose ac + bd = (b + d + a − c)(b + d − a + c). Prove that ab + cd is not prime. 28. N6 (RUS) Is it possible to find 100 positive integers not exceeding 25,000 such that all pairwise sums of them are different? 308 3 Problems 3.43 The Forty-Third IMO Glasgow, United Kingdom, July 19–30, 2002 3.43.1 Contest Problems First Day (July 24) 1. Let n be a positive integer. Each point (x, y) in the plane, where x and y are nonnegative integers with x + y = n, is colored red or blue, subject to the following condition: If a point (x, y) is red, then so are all points (x′ , y′ ) with x′ ≤ x and y′ ≤ y. Let A be the number of ways to choose n blue points with distinct x-coordinates, and let B be the number of ways to choose n blue points with distinct y-coordinates. Prove that A = B. 2. The circle S has center O, and BC is a diameter of S. Let A be a point of S such that ∡AOB < 120◦ . Let D be the midpoint of the arc AB that does not contain C. The line through O parallel to DA meets the line AC at I. The perpendicular bisector of OA meets S at E and at F. Prove that I is the incenter of the triangle CEF. 3. Find all pairs of positive integers m, n ≥ 3 for which there exist infinitely many positive integers a such that am + a − 1 an + a2 − 1 is itself an integer. Second Day (July 25) 4. Let n ≥ 2 be a positive integer, with divisors 1 = d1 < d2 < · · · < dk = n. Prove that d1 d2 + d2 d3 + · · · + dk−1 dk is always less than n2 , and determine when it is a divisor of n2 . 5. Find all functions f from the reals to the reals such that ( f (x) + f (z))( f (y) + f (t)) = f (xy − zt) + f (xt + yz) for all real x, y, z,t. 6. Let n ≥ 3 be a positive integer. Let C1 ,C2 ,C3 , . . . ,Cn be unit circles in the plane, with centers O1 , O2 , O3 , . . . , On respectively. If no line meets more than two of the circles, prove that 1 (n − 1)π ∑ Oi O j ≤ 4 . 1≤i< j≤n 3.43.2 Shortlisted Problems 1. N1 (UZB) What is the smallest positive integer t such that there exist integers x1 , x2 , . . . , xt with x31 + x32 + · · · + xt3 = 20022002? 3.43 IMO 2002 309 2. N2 (ROU)IMO4 Let n ≥ 2 be a positive integer, with divisors 1 = d1 < d2 < · · · < dk = n. Prove that d1 d2 + d2 d3 + · · · + dk−1 dk is always less than n2 , and determine when it is a divisor of n2 . 3. N3 (MNG) Let p1 , p2 , . . . , pn be distinct primes greater than 3. Show that 2 p1 p2 ···pn + 1 has at least 4n divisors. 4. N4 (GER) Is there a positive integer m such that the equation 1 1 1 1 m + + + = a b c abc a + b + c has infinitely many solutions in positive integers a, b, c? 5. N5 (IRN) Let m, n ≥ 2 be positive integers, and let a1 , a2 , . . . , an be integers, none of which is a multiple of mn−1 . Show that there exist integers e1 , e2 , . . . , en , not all zero, with |ei | < m for all i, such that e1 a1 + e2 a2 + · · · + en an is a multiple of mn . 6. N6 (ROU)IMO3 Find all pairs of positive integers m, n ≥ 3 for which there exist infinitely many positive integers a such that is itself an integer. am + a − 1 an + a2 − 1 7. G1 (FRA) Let B be a point on a circle S1 , and let A be a point distinct from B on the tangent at B to S1 . Let C be a point not on S1 such that the line segment AC meets S1 at two distinct points. Let S2 be the circle touching AC at C and touching S1 at a point D on the opposite side of AC from B. Prove that the circumcenter of triangle BCD lies on the circumcircle of triangle ABC. 8. G2 (KOR) Let ABC be a triangle for which there exists an interior point F such that ∠AFB = ∠BFC = ∠CFA. Let the lines BF and CF meet the sides AC and AB at D and E respectively. Prove that AB + AC ≥ 4DE. 9. G3 (KOR)IMO2 The circle S has center O, and BC is a diameter of S. Let A be a point of S such that ∡AOB < 120◦ . Let D be the midpoint of the arc AB that does not contain C. The line through O parallel to DA meets the line AC at I. The perpendicular bisector of OA meets S at E and at F. Prove that I is the incenter of the triangle CEF. 10. G4 (RUS) Circles S1 and S2 intersect at points P and Q. Distinct points A1 and B1 (not at P or Q) are selected on S1 . The lines A1 P and B1 P meet S2 again at A2 and B2 respectively, and the lines A1 B1 and A2 B2 meet at C. Prove that as A1 and B1 vary, the circumcenters of triangles A1 A2C all lie on one fixed circle. 11. G5 (AUS) For any set S of five points in the plane, no three of which are collinear, let M(S) and m(S) denote the greatest and smallest areas, respectively, 310 3 Problems of triangles determined by three points from S. What is the minimum possible value of M(S)/m(S)? 12. G6 (UKR)IMO6 Let n ≥ 3 be a positive integer. Let C1 ,C2 ,C3 , . . . ,Cn be unit circles in the plane, with centers O1 , O2 , O3 , . . . , On respectively. If no line meets more than two of the circles, prove that 1 (n − 1)π ≤ . O O 4 i j 1≤i< j≤n ∑ 13. G7 (BGR) The incircle Ω of the acute-angled triangle ABC is tangent to BC at K. Let AD be an altitude of triangle ABC and let M be the midpoint of AD. If N is the other common point of Ω and KM, prove that Ω and the circumcircle of triangle BCN are tangent at N. 14. G8 (ARM) Let S1 and S2 be circles meeting at the points A and B. A line through A meets S1 at C and S2 at D. Points M, N, K lie on the line segments CD, BC, BD respectively, with MN parallel to BD and MK parallel to BC. Let E and F be points on those arcs BC of S1 and BD of S2 respectively that do not contain A. Given that EN is perpendicular to BC and FK is perpendicular to BD, prove that ∡EMF = 90◦ . 15. A1 (CZE) Find all functions f from the reals to the reals such that f ( f (x) + y) = 2x + f ( f (y) − x) for all real x, y. 16. A2 (YUG) Let a1 , a2 , . . . be an infinite sequence of real numbers for which there exists a real number c with 0 ≤ ai ≤ c for all i such that |ai − a j | ≥ 1 i+ j for all i, j with i 6= j. Prove that c ≥ 1. 17. A3 (POL) Let P be a cubic polynomial given by P(x) = ax3 + bx2 + cx + d, where a, b, c, d are integers and a 6= 0. Suppose that xP(x) = yP(y) for infinitely many pairs x, y of integers with x 6= y. Prove that the equation P(x) = 0 has an integer root. 18. A4 (IND)IMO5 Find all functions f from the reals to the reals such that ( f (x) + f (z))( f (y) + f (t)) = f (xy − zt) + f (xt + yz) for all real x, y, z,t. 19. A5 (IND) Let n be a positive integer that is not a perfect cube. Define real numbers a, b, c by a= √ 3 n, b= 1 , a − [a] c= 1 , b − [b] 3.43 IMO 2002 311 where [x] denotes the integer part of x. Prove that there are infinitely many such integers n with the property that there exist integers r, s,t, not all zero, such that ra + sb + tc = 0. 20. A6 (IRN) Let A be a nonempty set of positive integers. Suppose that there are positive integers b1 , . . . , bn and c1 , . . . , cn such that (i) for each i the set bi A + ci = {bi a + ci | a ∈ A} is a subset of A, and (ii) the sets bi A + ci and b j A + c j are disjoint whenever i 6= j. Prove that 1 1 + ···+ ≤ 1. b1 bn 21. C1 (COL)IMO1 Let n be a positive integer. Each point (x, y) in the plane, where x and y are nonnegative integers with x + y ≤ n, is colored red or blue, subject to the following condition: If a point (x, y) is red, then so are all points (x′ , y′ ) with x′ ≤ x and y′ ≤ y. Let A be the number of ways to choose n blue points with distinct x-coordinates, and let B be the number of ways to choose n blue points with distinct y-coordinates. Prove that A = B. 22. C2 (ARM) For n an odd positive integer, the unit squares of an n × n chessboard are colored alternately black and white, with the four corners colored black. A tromino is an L-shape formed by three connected unit squares. For which values of n is it possible to cover all the black squares with nonoverlapping trominos? When it is possible, what is the minimum number of trominos needed? 23. C3 (COL) Let n be a positive integer. A sequence of n positive integers (not necessarily distinct) is called full if it satisfies the following condition: For each positive integer k ≥ 2, if the number k appears in the sequence, then so does the number k − 1, and moreover, the first occurrence of k − 1 comes before the last occurrence of k. For each n, how many full sequences are there? 24. C4 (BGR) Let T be the set of ordered triples (x, y, z), where x, y, z are integers with 0 ≤ x, y, z ≤ 9. Players A and B play the following guessing game: Player A chooses a triple (x, y, z) in T , and Player B has to discover A’s triple in as few moves as possible. A move consists of the following: B gives A a triple (a, b, c) in T , and A replies by giving B the number |x + y − a − b| + |y + z − b − c| + |z + x − c − a|. Find the minimum number of moves that B needs to be sure of determining A’s triple. 25. C5 (BRA) Let r ≥ 2 be a fixed positive integer, and let F be an infinite family of sets, each of size r, no two of which are disjoint. Prove that there exists a set of size r − 1 that meets each set in F . 26. C6 (POL) Let n be an even positive integer. Show that there is a permutation x1 , x2 , . . . , xn of 1, 2, . . . , n such that for every 1 ≤ i ≤ n the number xi+1 is one of 2xi , 2xi − 1, 2xi − n, 2xi − n − 1 (where we take xn+1 = x1 ). 27. C7 (NZL) Among a group of 120 people, some pairs are friends. A weak quartet is a set of four people containing exactly one pair of friends. What is the maximum possible number of weak quartets? 312 3 Problems 3.44 The Forty-Fourth IMO Tokyo, Japan, July 7–19, 2003 3.44.1 Contest Problems First Day (July 13) 1. Let A be a 101-element subset of the set S = {1, 2, . . ., 1000000}. Prove that there exist numbers t1 ,t2 , . . . ,t100 in S such that the sets A j = {x + t j |x ∈ A}, j = 1, 2, . . . , 100, are pairwise disjoint. 2. Determine all pairs (a, b) of positive integers such that a2 is a positive integer. 2ab2 − b3 + 1 3. Each pair of opposite sides of a convex hexagon √ has the following property: The distance between their midpoints is equal to 3/2 times the sum of their lengths. Prove that all the angles of the hexagon are equal. Second Day (July 14) 4. Let ABCD be a cyclic quadrilateral. Let P, Q, R be the feet of the perpendiculars from D to the lines BC,CA, AB, respectively. Show that PQ = QR if and only if the bisectors of ∠ABC and ∠ADC are concurrent with AC. 5. Let n be a positive integer and let x1 ≤ x2 ≤ · · · ≤ xn be real numbers. (a) Prove that !2 n 2(n2 − 1) n ∑ |xi − x j | ≤ 3 ∑ (xi − x j )2 . i, j=1 i, j=1 (b) Show that equality holds if and only if x1 , . . . , xn is an arithmetic progression. 6. Let p be a prime number. Prove that there exists a prime number q such that for every integer n, the number n p − p is not divisible by q. 3.44.2 Shortlisted Problems 1. A1 (USA) Let ai j , i = 1, 2, 3, j = 1, 2, 3, be real numbers such that ai j is positive for i = j and negative for i 6= j. Prove that there exist positive real numbers c1 , c2 , c3 such that the numbers a11 c1 + a12c2 + a13 c3 , a21 c1 + a22c2 + a23c3 , are all negative, all positive, or all zero. a31 c1 + a32c2 + a33c3 3.44 IMO 2003 313 2. A2 (AUS) Find all nondecreasing functions f : R → R such that (i) f (0) = 0, f (1) = 1; (ii) f (a) + f (b) = f (a) f (b) + f (a + b − ab) for all real numbers a, b such that a < 1 < b. 3. A3 (GEO) Consider pairs of sequences of positive real numbers a1 ≥ a2 ≥ a3 ≥ · · · , b1 ≥ b2 ≥ b3 ≥ · · · and the sums An = a1 + · · · + an , Bn = b1 + · · · + bn , n = 1, 2, . . . . For any pair define ci = min{ai , bi } and Cn = c1 + · · · + cn , n = 1, 2, . . . . (a) Does there exist a pair (ai )i≥1 , (bi )i≥1 such that the sequences (An )n≥1 and (Bn )n≥1 are unbounded while the sequence (Cn )n≥1 is bounded? (b) Does the answer to question (1) change by assuming additionally that bi = 1/i, i = 1, 2, . . .? Justify your answer. 4. A4 (IRL)IMO5 Let n be a positive integer and let x1 ≤ x2 ≤ · · · ≤ xn be real numbers. (a) Prove that !2 n 2(n2 − 1) n ∑ |xi − x j | ≤ 3 ∑ (xi − x j )2 . i, j=1 i, j=1 (b) Show that equality holds if and only if x1 , . . . , xn is an arithmetic progression. 5. A5 (KOR) Let R+ be the set of all positive real numbers. Find all functions f : R+ → R+ that satisfy the following conditions: √ √ √ (i) f (xyz) + f (x) + f (y) + f (z) = f ( xy) f ( yz) f ( zx) for all x, y, z ∈ R+ . (ii) f (x) < f (y) for all 1 ≤ x < y. 6. A6 (USA) Let n be a positive integer and let (x1 , . . . , xn ), (y1 , . . . , yn ) be two sequences of positive real numbers. Suppose (z2 , z3 , . . . , z2n ) is a sequence of positive real numbers such that z2i+ j ≥ xi y j for all 1 ≤ i, j ≤ n. Let M = max{z2 , . . . , z2n }. Prove that      M + z2 + · · · + z2n 2 x1 + · · · + xn y1 + · · · + yn ≥ . 2n n n 7. C1 (BRA)IMO1 Let A be a 101-element subset of the set S = {1, 2, . . ., 1000000}. Prove that there exist numbers t1 ,t2 , . . . ,t100 in S such that the sets A j = {x + t j | x ∈ A}, j = 1, 2, . . . , 100, are pairwise disjoint. 8. C2 (GEO) Let D1 , . . . , Dn be closed disks in the plane. (A closed disk is a region bounded by a circle, taken jointly with this circle.) Suppose that every point in the plane is contained in at most 2003 disks Di . Prove that there exists a disk Dk that intersects at most 7 · 2003 − 1 other disks Di . 314 3 Problems 9. C3 (LTU) Let n ≥ 5 be a given integer. Determine the largest integer k for which there exists a polygon with n vertices (convex or not, with non-self-intersecting boundary) having k internal right angles. 10. C4 (IRN) Let x1 , . . . , xn and y1 , . . . , yn be real numbers. Let A = (ai j )1≤i, j≤n be the matrix with entries  1, if xi + y j ≥ 0; ai j = 0, if xi + y j < 0. Suppose that B is an n × n matrix whose entries are 0, 1 such that the sum of the elements in each row and each column of B is equal to the corresponding sum for the matrix A. Prove that A = B. 11. C5 (ROU) Every point with integer coordinates in the plane is the center of a disk with radius 1/1000. (a) Prove that there exists an equilateral triangle whose vertices lie in different disks. (b) Prove that every equilateral triangle with vertices in different disks has side length greater than 96. 12. C6 (SAF) Let f (k) be the number of integers n that satisfy the following conditions: (i) 0 ≤ n < 10k , so n has exactly k digits (in decimal notation), with leading zeros allowed; (ii) the digits of n can be permuted in such a way that they yield an integer divisible by 11. Prove that f (2m) = 10 f (2m − 1) for every positive integer m. 13. G1 (FIN)IMO4 Let ABCD be a cyclic quadrilateral. Let P, Q, R be the feet of the perpendiculars from D to the lines BC,CA, AB, respectively. Show that PQ = QR if and only if the bisectors of ∠ABC and ∠ADC are concurrent with AC. 14. G2 (HEL) Three distinct points A, B,C are fixed on a line in this order. Let Γ be a circle passing through A and C whose center does not lie on the line AC. Denote by P the intersection of the tangents to Γ at A and C. Suppose Γ meets the segment PB at Q. Prove that the intersection of the bisector of ∠AQC and the line AC does not depend on the choice of Γ . 15. G3 (IND) Let ABC be a triangle and let P be a point in its interior. Denote by D, E, F the feet of the perpendiculars from P to the lines BC, CA, and AB, respectively. Suppose that AP2 + PD2 = BP2 + PE 2 = CP2 + PF 2 . Denote by IA , IB , IC the excenters of the triangle ABC. Prove that P is the circumcenter of the triangle IA IB IC . 16. G4 (ARM) Let Γ1 , Γ2 , Γ3 , Γ4 be distinct circles such that Γ1 , Γ3 are externally tangent at P, and Γ2 , Γ4 are externally tangent at the same point P. Suppose that 3.44 IMO 2003 315 Γ1 and Γ2 ; Γ2 and Γ3 ; Γ3 and Γ4 ; Γ4 and Γ1 meet at A, B,C, D, respectively, and that all these points are different from P. Prove that AB · BC PB2 = . AD · DC PD2 17. G5 (KOR) Let ABC be an isosceles triangle with AC = BC, whose incenter is I. Let P be a point on the circumcircle of the triangle AIB lying inside the triangle ABC. The lines through P parallel to CA and CB meet AB at D and E, respectively. The line through P parallel to AB meets CA and CB at F and G, respectively. Prove that the lines DF and EG intersect on the circumcircle of the triangle ABC. 18. G6 (POL)IMO3 Each pair of opposite sides of a convex hexagon√has the following property: The distance between their midpoints is equal to 3/2 times the sum of their lengths. Prove that all the angles of the hexagon are equal. 19. G7 (SAF) Let ABC be a triangle with semiperimeter s and inradius r. The semicircles with diameters BC,CA, AB are drawn outside of the triangle ABC. The circle tangent to all three semicircles has radius t. Prove that √ ! s s 3 <t ≤ + 1− r. 2 2 2 20. N1 (POL) Let m be a fixed integer greater than 1. The sequence x0 , x1 , x2 , . . . is defined as follows:  i 2, if 0 ≤ i ≤ m − 1; xi = ∑mj=1 xi− j , if i ≥ m. Find the greatest k for which the sequence contains k consecutive terms divisible by m. 21. N2 (USA) Each positive integer a undergoes the following procedure in order to obtain the number d = d(a): (1) move the last digit of a to the first position to obtain the number b; (2) square b to obtain the number c; (3) move the first digit of c to the end to obtain the number d. (All the numbers in the problem are considered to be represented in base 10.) For example, for a = 2003, we have b = 3200, c = 10240000, and d = 02400001 = 2400001 = d(2003). Find all numbers a for which d(a) = a2 . 22. N3 (BGR)IMO2 Determine all pairs (a, b) of positive integers such that is a positive integer. a2 2ab2 − b3 + 1 316 3 Problems 23. N4 (ROU) Let b be an integer greater than 5. For each positive integer n, consider the number xn = 11 . . . 1} 22 . . . 2} 5, | {z | {z n−1 n written in base b. Prove that the following condition holds if and only if b = 10: There exists a positive integer M such that for every integer n greater than M, the number xn is a perfect square. 24. N5 (KOR) An integer n is said to be good if |n| is not the square of an integer. Determine all integers m with the following property: m can be represented in infinitely many ways as a sum of three distinct good integers whose product is the square of an odd integer. 25. N6 (FRA)IMO6 Let p be a prime number. Prove that there exists a prime number q such that for every integer n, the number n p − p is not divisible by q. 26. N7 (BRA) The sequence a0 , a1 , a2 , . . . is defined as follows: a0 = 2, ak+1 = 2a2k − 1 for k ≥ 0. Prove that if an odd prime p divides an , then 2n+3 divides p2 − 1. 27. N8 (IRN) Let p be a prime number and let A be a set of positive integers that satisfies the following conditions: (i) the set of prime divisors of the elements in A consists of p − 1 elements; (ii) for any nonempty subset of A, the product of its elements is not a perfect pth power. What is the largest possible number of elements in A? 3.45 IMO 2004 317 3.45 The Forty-Fifth IMO Athens, Greece, July 7–19, 2004 3.45.1 Contest Problems First Day (July 12) 1. Let ABC be an acute-angled triangle with AB 6= AC. The circle with diameter BC intersects the sides AB and AC at M and N, respectively. Denote by O the midpoint of BC. The bisectors of the angles BAC and MNG intersect at R. Prove that the circumcircles of the triangles BMR and CNR have a common point lying on the line segment BC. 2. Find all polynomials P(x) with real coefficients that satisfy the equality P(a − b) + P(b − c) + P(c − a) = 2P(a + b + c) for all triples a, b, c of real numbers such that ab + bc + ca = 0. 3. Determine all m × n rectangles that can be covered with hooks made up of 6 unit squares, as in the figure: Rotations and reflections of hooks are allowed. The rectangle must be covered without gaps and overlaps. No part of a hook may cover area outside the rectangle. Second Day (July 13) 4. Let n ≥ 3 be an integer and t1 ,t2 , . . . ,tn positive real numbers such that   1 1 1 2 n + 1 > (t1 + t2 + · · · + tn ) + + ···+ . t1 t2 tn Show that ti , t j , tk are the side lengths of a triangle for all i, j, k that satisfy 1 ≤ i < j < k ≤ n. 5. In a convex quadrilateral ABCD the diagonal BD does not bisect the angles ABC and CDA. The point P lies inside ABCD and satisfies ∠PBC = ∠DBA and ∠PDC = ∠BDA. Prove that ABCD is a cyclic quadrilateral if and only if AP = CP. 6. We call a positive integer alternate if its decimal digits are alternately odd and even. Find all positive integers n such that n has an alternate multiple. 318 3 Problems 3.45.2 Shortlisted Problems 1. A1 (KOR)IMO4 Let n ≥ 3 be an integer and t1 ,t2 , . . . ,tn positive real numbers such that   1 1 1 2 n + 1 > (t1 + t2 + · · · + tn ) + + ···+ . t1 t2 tn Show that ti , t j , tk are the side lengths of a triangle for all i, j, k that satisfy 1 ≤ i < j < k ≤ n. 2. A2 (ROU) An infinite sequence a0 , a1 , a2 , . . . of real numbers satisfies the condition an = |an+1 − an+2| for every n ≥ 0 with a0 and a1 positive and distinct. Can this sequence be bounded? 3. A3 (CAN) Does there exist a function s : Q → {−1, 1} such that if x and y are distinct rational numbers satisfying xy = 1 or x + y ∈ {0, 1}, then s(x)s(y) = −1? Justify your answer. 4. A4 (KOR)IMO2 Find all polynomials P(x) with real coefficients that satisfy the equality P(a − b) + P(b − c) + P(c − a) = 2P(a + b + c) for all triples a, b, c of real numbers such that ab + bc + ca = 0. 5. A5 (THA) Let a, b, c > 0 and ab + bc + ca = 1. Prove the inequality r r r 1 3 1 3 1 3 1 + 6b + + 6c + + 6a ≤ . a b c abc 6. A6 (RUS) Find all functions f : R → R satisfying the equation  f x2 + y2 + 2 f (xy) = ( f (x + y))2 for all x, y ∈ R. 7. A7 (IRL) Let a1 , a2 , . . . , an be positive real numbers, n > 1. Denote by gn their geometric mean, and by A1 , A2 , . . . , An the sequence of arithmetic means dek fined by Ak = a1 +a2 +···+a , k = 1, 2, . . . , n. Let Gn be the geometric mean of k A1 , A2 , . . . , An . Prove the inequality r Gn g n nn + ≤ n+1 A n Gn and establish the cases of equality. 8. C1 (PRI) There are 10001 students at a university. Some students join together to form several clubs (a student may belong to different clubs). Some clubs join together to form several societies (a club may belong to different societies). There are a total of k societies. Suppose that the following conditions hold: (i) Each pair of students are in exactly one club. (ii) For each student and each society, the student is in exactly one club of the society. 3.45 IMO 2004 319 (iii) Each club has an odd number of students. In addition, a club with 2m + 1 students (m is a positive integer) is in exactly m societies. Find all possible values of k. 9. C2 (GER) Let n and k be positive integers. There are given n circles in the plane. Every two of them intersect at two distinct points, and all points of intersection they determine are distinct. Each intersection point must be colored with one of n distinct colors so that each color is used at least once, and exactly k distinct colors occur on each circle. Find all values of n ≥ 2 and k for which such a coloring is possible. 10. C3 (AUS) The following operation is allowed on a finite graph: Choose an arbitrary cycle of length 4 (if there is any), choose an arbitrary edge in that cycle, and delete it from the graph. For a fixed integer n ≥ 4, find the least number of edges of a graph that can be obtained by repeated applications of this operation from the complete graph on n vertices (where each pair of vertices are joined by an edge). 11. C4 (POL) Consider a matrix of size n × n whose entries are real numbers of absolute value not exceeding 1, and the sum of all entries is 0. Let n be an even positive integer. Determine the least number C such that every such matrix necessarily has a row or a column with the sum of its entries not exceeding C in absolute value. 12. C5 (NZL) Let N be a positive integer. Two players A and B, taking turns, write numbers from the set {1, . . . ,N} on a blackboard. A begins the game by writing 1 on his first move. Then, if a player has written n on a certain move, his adversary is allowed to write n + 1 or 2n (provided the number he writes does not exceed N). The player who writes N wins. We say that N is of type A or of type B according as A or B has a winning strategy. (a) Determine whether N = 2004 is of type A or of type B. (b) Find the least N > 2004 whose type is different from that of 2004. 13. C6 (IRN) For an n × n matrix A, let Xi be the set of entries in row i, and Y j the set of entries in column j, 1 ≤ i, j ≤ n. We say that A is golden if X1 , . . . , Xn ,Y1 , . . . ,Yn are distinct sets. Find the least integer n such that there exists a 2004 × 2004 golden matrix with entries in the set {1, 2, . . . , n}. 14. C7 (EST)IMO3 Determine all m × n rectangles that can be covered with hooks made up of 6 unit squares, as in the figure: Rotations and reflections of hooks are allowed. The rectangle must be covered without gaps and overlaps. No part of a hook may cover area outside the rectangle. 320 3 Problems 15. C8 (POL) For a finite graph G, let f (G) be the number of triangles and g(G) the number of tetrahedra formed by edges of G. Find the least constant c such that g(G)3 ≤ c · f (G)4 for every graph G. 16. G1 (ROU)IMO1 Let ABC be an acute-angled triangle with AB 6= AC. The circle with diameter BC intersects the sides AB and AC at M and N, respectively. Denote by O the midpoint of BC. The bisectors of the angles BAC and MNG intersect at R. Prove that the circumcircles of the triangles BMR and CNR have a common point lying on the line segment BC. 17. G2 (KAZ) The circle Γ and the line ℓ do not intersect. Let AB be the diameter of Γ perpendicular to ℓ, with B closer to ℓ than A. An arbitrary point C 6= A, B is chosen on Γ . The line AC intersects ℓ at D. The line DE is tangent to Γ at E, with B and E on the same side of AC. Let BE intersect ℓ at F, and let AF intersect Γ at G 6= A. Prove that the reflection of G in AB lies on the line CF. 18. G3 (KOR) Let O be the circumcenter of an acute-angled triangle ABC with ∠B < ∠C. The line AO meets the side BC at D. The circumcenters of the triangles ABD and ACD are E and F, respectively. Extend the sides BA and CA beyond A, and choose on the respective extension points G and H such that AG = AC and AH = AB. Prove that the quadrilateral EFGH is a rectangle if and only if ∠ACB − ∠ABC = 60◦. 19. G4 (POL)IMO5 In a convex quadrilateral ABCD the diagonal BD does not bisect the angles ABC and CDA. The point P lies inside ABCD and satisfies ∠PBC = ∠DBA and ∠PDC = ∠BDA. Prove that ABCD is a cyclic quadrilateral if and only if AP = CP. 20. G5 (SCG) Let A1 A2 . . . An be a regular n-gon. The points B1 , . . . , Bn−1 are defined as follows: (i) If i = 1 or i = n − 1, then Bi is the midpoint of the side Ai Ai+1 . (ii) If i 6= 1, i 6= n − 1, and S is the intersection point of A1 Ai+1 and An Ai , then Bi is the intersection point of the bisector of the angle Ai SAi+1 with Ai Ai+1 . Prove the equality ∠A1 B1 An + ∠A1 B2 An + · · · + ∠A1Bn−1 An = 180◦ . 21. G6 (UNK) Let P be a convex polygon. Prove that there is a convex hexagon that is contained in P and that occupies at least 75 percent of the area of P. 22. G7 (RUS) For a given triangle ABC, let X be a variable point on the line BC such that C lies between B and X and the incircles of the triangles ABX and ACX intersect at two distinct points P and Q. Prove that the line PQ passes through a point independent of X . 23. G8 (SCG) A cyclic quadrilateral ABCD is given. The lines AD and BC intersect at E, with C between B and E; the diagonals AC and BD intersect at F. Let M 3.45 IMO 2004 321 be the midpoint of the side CD, and let N 6= M be a point on the circumcircle of the triangle ABM such that AN/BN = AM/BM. Prove that the points E, F, and N are collinear. 24. N1 (BLR) Let τ (n) denote the number of positive divisors of the positive integer n. Prove that there exist infinitely many positive integers a such that the equation τ (an) = n does not have a positive integer solution n. 25. N2 (RUS) The function ψ from the set N of positive integers into itself is defined by the equality ψ (n) = n ∑ (k, n), k=1 n ∈ N, where (k, n) denotes the greatest common divisor of k and n. (a) Prove that ψ (mn) = ψ (m)ψ (n) for every two relatively prime m, n ∈ N. (b) Prove that for each a ∈ N the equation ψ (x) = ax has a solution. (c) Find all a ∈ N such that the equation ψ (x) = ax has a unique solution. 26. N3 (IRN) A function f from the set of positive integers N into itself is such that for all m, n ∈ N the number (m2 + n)2 is divisible by f 2 (m) + f (n). Prove that f (n) = n for each n ∈ N. 27. N4 (POL) Let k be a fixed integer greater than 1, and let m = 4k2 − 5. Show that there exist positive integers a and b such that the sequence (xn ) defined by x0 = a, x1 = b, xn+2 = xn+1 + xn for n = 0, 1, 2, . . . has all of its terms relatively prime to m. 28. N5 (IRN)IMO6 We call a positive integer alternate if its decimal digits are alternately odd and even. Find all positive integers n such that n has an alternate multiple. 29. N6 (IRL) Given an integer n > 1, denote by Pn the product of all positive integers x less than n and such that n divides x2 − 1. For each n > 1, find the remainder of Pn on division by n. 30. N7 (BGR) Let p be an odd prime and n a positive integer. In the coordinate plane, eight distinct points with integer coordinates lie on a circle with diameter of length pn . Prove that there exists a triangle with vertices at three of the given points such that the squares of its side lengths are integers divisible by pn+1 . 322 3 Problems 3.46 The Forty-Sixth IMO Mérida, Mexico, July 8–19, 2005 3.46.1 Contest Problems First Day (July 13) 1. Six points are chosen on the sides of an equilateral triangle ABC: A1 , A2 on BC; B1 , B2 on CA; C1 ,C2 on AB. These points are vertices of a convex hexagon A1 A2 B1 B2C1C2 with equal side lengths. Prove that the lines A1 B2 , B1C2 and C1 A2 are concurrent. 2. Let a1 , a2 , . . . be a sequence of integers with infinitely many positive terms and infinitely many negative terms. Suppose that for each positive integer n, the numbers a1 , a2 , . . . , an leave n different remainders on division by n. Prove that each integer occurs exactly once in the sequence. 3. Let x, y, and z be positive real numbers such that xyz ≥ 1. Prove that x5 − x2 x5 + y2 + z2 + y5 − y2 y5 + z2 + x2 + z5 − z2 z5 + x 2 + y 2 ≥ 0. Second Day (July 14) 4. Consider the sequence a1 , a2 , . . . defined by an = 2n + 3n + 6n − 1 (n = 1, 2, . . .). Determine all positive integers that are relatively prime to every term of the sequence. 5. Let ABCD be a given convex quadrilateral with sides BC and AD equal in length and not parallel. Let E and F be interior points of the sides BC and AD respectively such that BE = DF. The lines AC and BD meet at P; the lines BD and EF meet at Q; the lines EF and AC meet at R. Consider all the triangles PQR as E and F vary. Show that the circumcircles of these triangles have a common point other than P. 6. In a mathematical competition, six problems were posed to the contestants. Each pair of problems was solved by more than 2/5 of the contestants. Nobody solved all six problems. Show that there were at least two contestants who each solved exactly five problems. 3.46.2 Shortlisted Problems 1. A1 (ROU) Find all monic polynomials p(x) with integer coefficients of degree two for which there exists a polynomial q(x) with integer coefficients such that p(x)q(x) is a polynomial having all coefficients ±1. 3.46 IMO 2005 323 2. A2 (BGR) Let R+ denote the set of positive real numbers. Determine all functions f : R+ → R+ such that f (x) f (y) = 2 f (x + y f (x)) for all positive real numbers x and y. 3. A3 (CZE) Four real numbers p, q, r, s satisfy p + q + r + s = 9 and p2 + q2 + r2 + s2 = 21. Prove that ab − cd ≥ 2 holds for some permutation (a, b, c, d) of (p, q, r, s). 4. A4 (IND) Find all functions f : R → R satisfying the equation f (x + y) + f (x) f (y) = f (xy) + 2xy + 1 for all real x and y. 5. A5 (KOR)IMO3 Let x, y and z be positive real numbers such that xyz ≥ 1. Prove that x5 − x2 y5 − y2 z5 − z2 + 5 + 5 ≥ 0. 5 2 2 2 2 x +y +z y +z +x z + x2 + y2 6. C1 (AUS) A house has an even number of lamps distributed among its rooms in such a way that there are at least three lamps in every room. Each lamp shares a switch with exactly one other lamp, not necessarily from the same room. Each change in the switch shared by two lamps changes their states simultaneously. Prove that for every initial state of the lamps there exists a sequence of changes in some of the switches at the end of which each room contains lamps that are on as well as lamps that are off. 7. C2 (IRN) Let k be a fixed positive integer. A company has a special method to sell sombreros. Each customer can convince two persons to buy a sombrero after he/she buys one; convincing someone already convinced does not count. Each of these new customers can convince two others and so on. If each of the two customers convinced by someone makes at least k persons buy sombreros (directly or indirectly), then that someone wins a free instructional video. Prove that if n persons bought sombreros, then at most n/(k + 2) of them got videos. 8. C3 (IRN) In an m × n rectangular board of mn unit squares, adjacent squares are ones with a common edge, and a path is a sequence of squares in which any two consecutive squares are adjacent. Each square of the board can be colored black or white. Let N denote the number of colorings of the board such that there exists at least one black path from the left edge of the board to its right edge, and let M denote the number of colorings in which there exist at least two nonintersecting black paths from the left edge to the right edge. Prove that N 2 ≥ 2mn M. 9. C4 (COL) Let n ≥ 3 be a given positive integer. We wish to label each side and each diagonal of a regular n-gon P1 . . . Pn with a positive integer less than or equal to r so that: 324 3 Problems (i) every integer between 1 and r occurs as a label; (ii) in each triangle Pi Pj Pk two of the labels are equal and greater than the third. Given these conditions: (a) Determine the largest positive integer r for which this can be done. (b) For that value of r, how many such labelings are there? 10. C5 (SCG) There are n markers, each with one side white and the other side black, aligned in a row so that their white sides are up. In each step, if possible, we choose a marker with the white side up (but not one of the outermost markers), remove it, and reverse the closest marker to the left and the closest marker to the right of it. Prove that one can achieve the state with only two markers remaining if and only if n − 1 is not divisible by 3. 11. C6 (ROU)IMO6 In a mathematical competition, six problems were posed to the contestants. Each pair of problems was solved by more than 2/5 of the contestants. Nobody solved all six problems. Show that there were at least two contestants who each solved exactly five problems. 12. C7 (USA) Let n ≥ 1 be a given integer, and let a1 , . . . , an be a sequence of integers such that n divides the sum a1 + · · · + an . Show that there exist permutations σ and τ of 1, 2, . . . , n such that σ (i) + τ (i) ≡ ai (mod n) for all i = 1, . . . , n. 13. C8 (BGR) Let M be a convex n-gon, n ≥ 4. Some n − 3 of its diagonals are colored green and some other n − 3 diagonals are colored red, so that no two diagonals of the same color meet inside M. Find the maximum possible number of intersection points of green and red diagonals inside M. 14. G1 (HEL) In a triangle ABC satisfying AB + BC = 3AC the incircle has center I and touches the sides AB and BC at D and E, respectively. Let K and L be the symmetric points of D and E with respect to I. Prove that the quadrilateral ACKL is cyclic. 15. G2 (ROU)IMO1 Six points are chosen on the sides of an equilateral triangle ABC: A1 , A2 on BC; B1 , B2 on CA; C1 ,C2 on AB. These points are vertices of a convex hexagon A1 A2 B1 B2C1C2 with equal side lengths. Prove that the lines A1 B2 , B1C2 and C1 A2 are concurrent. 16. G3 (UKR) Let ABCD be a parallelogram. A variable line l passing through the point A intersects the rays BC and DC at points X and Y , respectively. Let K and L be the centers of the excircles of triangles ABX and ADY , touching the sides BX and DY , respectively. Prove that the size of angle KCL does not depend on the choice of the line l. 17. G4 (POL)IMO5 Let ABCD be a given convex quadrilateral with sides BC and AD equal in length and not parallel. Let E and F be interior points of the sides BC and AD respectively such that BE = DF. The lines AC and BD meet at P; the lines BD and EF meet at Q; the lines EF and AC meet at R. Consider all the triangles PQR as E and F vary. Show that the circumcircles of these triangles have a common point other than P. 3.46 IMO 2005 325 18. G5 (ROU) Let ABC be an acute-angled triangle with AB 6= AC; let H be its orthocenter and M the midpoint of BC. Points D on AB and E on AC are such that AE = AD and D, H, E are collinear. Prove that HM is orthogonal to the common chord of the circumcircles of triangles ABC and ADE. 19. G6 (RUS) The median AM of a triangle ABC intersects its incircle ω at K and L. The lines through K and L parallel to BC intersect ω again at X and Y . The lines AX and AY intersect BC at P and Q. Prove that BP = CQ. 20. G7 (KOR) In an acute triangle ABC, let D, E, F, P, Q, R be the feet of perpendiculars from A, B, C, A, B, C to BC, CA, AB, EF, FD, DE, respectively. Prove that p(ABC)p(PQR) ≥ p(DEF)2 , where p(T ) denotes the perimeter of triangle T. 21. N1 (POL)IMO4 Consider the sequence a1 , a2 , . . . defined by an = 2n + 3n + 6n − 1 (n = 1, 2, . . .). Determine all positive integers that are relatively prime to every term of the sequence. 22. N2 (NLD)IMO2 Let a1 , a2 , . . . be a sequence of integers with infinitely many positive terms and infinitely many negative terms. Suppose that for each positive integer n, the numbers a1 , a2 , . . . , an leave n different remainders on division by n. Prove that each integer occurs exactly once in the sequence. 23. N3 (MNG) Let a, b, c, d, e, and f be positive integers. Suppose that the sum S = a+ b + c+d + e+ f divides both abc+ de f and ab+ bc+ ca− de−e f − f d. Prove that S is composite. 24. N4 (COL) Find all positive integers n > 1 for which there exists a unique integer a with 0 < a ≤ n! such that an + 1 is divisible by n!. 25. N5 (NLD) Denote by d(n) the number of divisors of the positive integer n. A positive integer n is called highly divisible if d(n) > d(m) for all positive integers m < n. Two highly divisible integers m and n with m < n are called consecutive if there exists no highly divisible integer s satisfying m < s < n. (a) Show that there are only finitely many pairs of consecutive highly divisible integers of the form (a, b) with a | b. (b) Show that for every prime number p there exist infinitely many positive highly divisible integers r such that pr is also highly divisible. 26. N6 (IRN) Let a and b be positive integers such that an + n divides bn + n for every positive integer n. Show that a = b. 27. N7 (RUS) Let P(x) = an xn + an−1 xn−1 + · · · + a0 , where a0 , . . . , an are integers, an > 0, n ≥ 2. Prove that there exists a positive integer m such that P(m!) is a composite number. 326 3 Problems 3.47 The Forty-Seventh IMO Ljubljana, Slovenia, July 6–18, 2006 3.47.1 Contest Problems First Day (July 12) 1. Let ABC be a triangle with incenter I. A point P in the interior of the triangle satisfies ∠PBA + ∠PCA = ∠PBC + ∠PCB. Show that AP ≥ AI, and that equality holds if and only if P = I. 2. Let P be a regular 2006-gon. A diagonal of P is called good if its endpoints divide the boundary of P into two parts, each composed of an odd number of sides of P. The sides of P are also called good. Suppose P has been dissected into triangles by 2003 diagonals, no two of which have a common point in the interior of P. Find the maximum number of isosceles triangles having two good sides that could appear in such a configuration. 3. Determine the least real number M such that the inequality ab(a2 − b2 ) + bc(b2 − c2 ) + ca(c2 − a2 ) ≤ M(a2 + b2 + c2 )2 holds for all real numbers a, b, and c. Second Day (July 13) 4. Determine all pairs (x, y) of integers such that 1 + 2x + 22x+1 = y2 . 5. Let P(x) be a polynomial of degree n > 1 with integer coefficients and let k be a positive integer. Consider the polynomial Q(x) = P(P(. . . P(P(x)). . . )), where P occurs k times. Prove that there are at most n integers t that satisfy the equality Q(t) = t. 6. Assign to each side b of a convex polygon P the maximum area of a triangle that has b as a side and is contained in P. Show that the sum of the areas assigned to the sides of P is at least twice the area of P. 3.47.2 Shortlisted Problems 1. A1 (EST) A sequence of real numbers a0 , a1 , a2 , . . . is defined by the formula ai+1 = [ai ] · {ai }, for i ≥ 0; here a0 is an arbitrary number, [ai ] denotes the greatest integer not exceeding ai , and {ai } = ai − [ai ]. Prove that ai = ai+2 for i sufficiently large. 3.47 IMO 2006 327 2. A2 (POL) The sequence of real numbers a0 , a1 , a2 , . . . is defined recursively by a0 = −1 and n an−k ∑ k + 1 = 0, for n ≥ 1. k=0 Show that an > 0 for n ≥ 1. 3. A3 (RUS) The sequence c0 , c1 , . . . , cn , . . . is defined by c0 = 1, c1 = 0, and cn+2 = cn+1 + cn for n ≥ 0. Consider the set S of ordered pairs (x, y) for which there is a finite set J of positive integers such that x = ∑ j∈J c j , y = ∑ j∈J c j−1 . Prove that there exist real numbers α , β , and M with the following property: an ordered pair of nonnegative integers (x, y) satisfies the inequality m < α x + β y < M if and only if (x, y) ∈ S. Remark: A sum over the elements of the empty set is assumed to be 0. 4. A4 (SRB) Prove the inequality ai a j n ∑ ai + a j ≤ 2(a1 + a2 + · · · + an) ∑ ai a j i< j i< j for positive real numbers a1 , a2 , . . . , an . 5. A5 (KOR) Let a, b, c be the sides of a triangle. Prove that √ √ √ b+c−a c+a−b a+b−c √ √ +√ √ √ √ +√ √ √ ≤ 3. b+ c− a c+ a− b a+ b− c 6. A6 (IRL)IMO3 Determine the smallest number M such that the inequality |ab(a2 − b2 ) + bc(b2 − c2 ) + ca(c2 − a2 )| ≤ M(a2 + b2 + c2 )2 holds for all real numbers a, b, c 7. C1 (FRA) We have n ≥ 2 lamps L1 , . . . , Ln in a row, each of them being either on or off. Every second we simultaneously modify the state of each lamp as follows: if the lamp Li and its neighbors (only one neighbor for i = 1 or i = n, two neighbors for other i) are in the same state, then Li is switched off; otherwise, Li is switched on. Initially all the lamps are off except the leftmost one which is on. (a) Prove that there are infinitely many integers n for which all the lamps will eventually be off. (b) Prove that there are infinitely many integers n for which the lamps will never be all off. 8. C2 (SRB)IMO2 A diagonal of a regular 2006-gon is called odd if its endpoints divide the boundary into two parts, each composed of an odd number of sides. Sides are also regarded as odd diagonals. Suppose the 2006-gon has been dissected into triangles by 2003 nonintersecting diagonals. Find the maximum possible number of isosceles triangles with two odd sides. 328 3 Problems 9. C3 (COL) Let S be a finite set of points in the plane such that no three of them are on a line. For each convex polygon P whose vertices are in S, let a(P) be the number of vertices of P, and let b(P) be the number of points of S that are outside P. Prove that for every real number x ∑ xa(P)(1 − x)b(P) = 1, P where the sum is taken over all convex polygons with vertices in S. Remark. A line segment, a point, and the empty set are considered convex polygons of 2, 1, and 0 vertices respectively. 10. C4 (TWN) A cake has the form of an n × n square composed of n2 unit squares. Strawberries lie on some of the unit squares so that each row and each column contains exactly one strawberry; call this arrangement A . Let B be another such arrangement. Suppose that every grid rectangle with one vertex at the top left corner of the cake contains no fewer strawberries of arrangement B than of arrangement A . Prove that arrangement B can be obtained from A by performing a number of switches, defined as follows: A switch consists in selecting a grid rectangle with only two strawberries, situated at its top right corner and bottom left corner, and moving these two strawberries to the other two corners of that rectangle. 11. C5 (ARG) An (n, k)-tournament is a contest with n players held in k rounds such that: (i) Each player plays in each round, and every two players meet at most once. (ii) If player A meets player B in round i, player C meets player D in round i, and player A meets player C in round j, then player B meets player D in round j. Determine all pairs (n, k) for which there exists an (n, k)-tournament. 12. C6 (COL) A holey triangle is an upward equilateral triangle of side length n with n upward unit triangular holes cut out. A diamond is a 60◦ –120◦ unit rhombus. Prove that a holey triangle T can be tiled with diamonds if and only if the following condition holds: every upward equilateral triangle of side length k in T contains at most k holes, for 1 ≤ k ≤ n. 13. C7 (JPN) Consider a convex polyhedron without parallel edges and without an edge parallel to any face other than the two faces adjacent to it. Call a pair of points of the polyhedron antipodal if there exist two parallel planes passing through these points and such that the polyhedron is contained between these planes. Let A be the number of antipodal pairs of vertices, and let B be the number of antipodal pairs of midpoint edges. Determine the difference A − B in terms of the numbers of vertices, edges, and faces. 14. G1 (KOR)IMO1 Let ABC be a triangle with incenter I. A point P in the interior of the triangle satisfies ∠PBA + ∠PCA = ∠PBC + ∠PCB. Show that AP ≥ AI and that equality holds if and only if P coincides with I. 3.47 IMO 2006 329 15. G2 (UKR) Let ABC be a trapezoid with parallel sides AB > CD. Points K and L lie on the line segments AB and CD, respectively, so that AK/KB = DL/LC. Suppose that there are points P and Q on the line segment KL satisfying ∠APB = ∠BCD and ∠CQD = ∠ABC. Prove that the points P, Q, B, and C are concyclic. 16. G3 (USA) Let ABCDE be a convex pentagon such that ∠BAC = ∠CAD = ∠DAE and ∠ABC = ∠ACD = ∠ADE. The diagonals BD and CE meet at P. Prove that the line AP bisects the side CD. 17. G4 (RUS) A point D is chosen on the side AC of a triangle ABC with ∠C < ∠A < 90◦ in such a way that BD = BA. The incircle of ABC is tangent to AB and AC at points K and L, respectively. Let J be the incenter of triangle BCD. Prove that the line KL intersects the line segment AJ at its midpoint. 18. G5 (HEL) In triangle ABC, let J be the center of the excircle tangent to side BC at A1 and to the extensions of sides AC and AB at B1 and C1 , respectively. Suppose that the lines A1 B1 and AB are perpendicular and intersect at D. Let E be the foot of the perpendicular from C1 to line DJ. Determine the angles ∠BEA1 and ∠AEB1 . 19. G6 (BRA) Circles ω1 and ω2 with centers O1 and O2 are externally tangent at point D and internally tangent to a circle ω at points E and F, respectively. Line t is the common tangent of ω1 and ω2 at D. Let AB be the diameter of ω perpendicular to t, so that A, E, and O1 are on the same side of t. Prove that the lines AO1 , BO2 , EF, and t are concurrent. 20. G7 (SVK) In a triangle ABC, let Ma , Mb , Mc , be respectively the midpoints of the sides BC, CA, AB, and let Ta , Tb , Tc be the midpoints of the arcs BC, CA, AB of the circumcircle of ABC, not counting the opposite vertices. For i ∈ {a, b, c} let ωi be the circle with Mi Ti as diameter. Let pi be the common external tangent to ω j , ωk ({i, j, k} = {a, b, c}) such that ωi lies on the opposite side of pi from ω j , ωk . Prove that the lines pa , pb , pc form a triangle similar to ABC and find the ratio of similitude. 21. G8 (POL) Let ABCD be a convex quadrilateral. A circle passing through the points A and D and a circle passing through the points B and C are externally tangent at a point P inside the quadrilateral. Suppose that ∠PAB + ∠PDC ≤ 90◦ and ∠PBA + ∠PCD ≤ 90◦ . Prove that AB +CD ≥ BC + AD. 22. G9 (RUS) Points A1 , B1 , C1 are chosen on the sides BC, CA, AB of a triangle ABC respectively. The circumcircles of triangles AB1C1 , BC1 A1 , CA1 B1 intersect the circumcircle of triangle ABC again at points A2 , B2 , C2 respectively (A2 6= A, B2 6= B, C2 6= C). Points A3 , B3 , C3 are symmetric to A1 , B1 , C1 with respect to the midpoints of the sides BC, CA, AB, respectively. Prove that the triangles A2 B2C2 and A3 B3C3 are similar. 23. G10 (SRB)IMO6 Assign to each side b of a convex polygon P the maximum area of a triangle that has b as a side and is contained in P. Show that the sum of the areas assigned to the sides of P is at least twice the area of P. 330 3 Problems 24. N1 (USA)IMO4 Determine all pairs (x, y) of integers satisfying the equation 1 + 2x + 22x+1 = y2 . 25. N2 (CAN) For x ∈ (0, 1) let y ∈ (0, 1) be the number whose nth digit after the decimal point is the 2n th digit after the decimal point of x. Show that if x is rational then so is y. 26. N3 (SAF) The sequence f (1), f (2), f (3), . . . is defined by f (n) = h n i 1 h n i h n i + + ···+ , n 1 2 n where [x] denotes the integral part of x. (a) Prove that f (n + 1) > f (n) infinitely often. (b) Prove that f (n + 1) < f (n) infinitely often. 27. N4 (ROU)IMO5 Let P(x) be a polynomial of degree n > 1 with integer coefficients and let k be a positive integer. Consider the polynomial Q(x) = P(P(. . . P(P(x)). . . )), where P occurs k times. Prove that there are at most n integers t such that Q(t) = t. 28. N5 (RUS) Find all integer solutions of the equation x7 − 1 = y5 − 1. x−1 29. N6 (USA) Let a > b > 1 be relatively prime positive integers. Define the weight of an integer c, denoted by w(c), to be the minimal possible value of |x| + |y| taken over all pairs of integers x and y such that ax + by = c. An integer c is called a local champion if w(c) ≥ w(c ± a) and w(c) ≥ w(c ± b). Find all local champions and determine their number. 30. N7 (EST) Prove that for every positive integer n there exists an integer m such that 2m + m is divisible by n. 3.48 IMO 2007 331 3.48 The Forty-Eighth IMO Hanoi, Vietnam, July 19–31, 2007 3.48.1 Contest Problems First Day (July 25) 1. Real numbers a1 , a2 , . . . , an are given. For each i (1 ≤ i ≤ n) define di = max{a j | 1 ≤ j ≤ i} − min{a j | i ≤ j ≤ n} and let d = max{di | 1 ≤ i ≤ n}. (a) Prove that for any real numbers x1 ≤ x2 ≤ · · · ≤ xn , max{|xi − ai | | 1 ≤ i ≤ n} ≥ d . 2 (1) (b) Show that there are real numbers x1 ≤ x2 ≤ · · · ≤ xn such that equality holds in (1). 2. Consider five points A, B, C, D, and E such that ABCD is a parallelogram and BCED is a cyclic quadrilateral. Let ℓ be a line passing through A. Suppose that ℓ intersects the interior of the segment DC at F and intersects line BC at G. Suppose also that EF = EG = EC. Prove that ℓ is the bisector of angle DAB. 3. In a mathematical competition some competitors are friends. Friendship is always mutual. Call a group of competitors a clique if each two of them are friends. (In particular, any group of fewer than two competitors is a clique.) The number of members of a clique is called its size. Given that in this competition, the largest size of a clique is even, prove that the competitors can be arranged in two rooms such that the largest size of a clique contained in one room is the same as the largest size of a clique contained in the other room. Second Day (July 26) 4. In triangle ABC the bisector of angle BCA intersects the circumcircle again at R, the perpendicular bisector of BC at P, and the perpendicular bisector of AC at Q. The midpoint of BC is K and the midpoint of AC is L. Prove that the triangles RPK and RQL have the same area. 5. Let a and b be positive integers. Show that if 4ab − 1 divides (4a2 − 1)2 , then a = b. 6. Let n be a positive integer. Consider  S = (x, y, z) | x, y, z ∈ {0, 1, . . . , n}, x + y + z > 0 as a set of (n + 1)3 − 1 points in three-dimensional space. Determine the smallest possible number of planes, the union of which contains S but does not include (0, 0, 0). 332 3 Problems 3.48.2 Shortlisted Problems 1. A1 (NZL) IMO1 Given a sequence a1 , a2 , . . . , an of real numbers, for each i (1 ≤ i ≤ n) define di = max{a j : 1 ≤ j ≤ i} − min{a j : i ≤ j ≤ n} and let d = max{di : 1 ≤ i ≤ n}. (a) Prove that for arbitrary real numbers x1 ≤ x2 ≤ · · · ≤ xn , max{|xi − ai | : 1 ≤ i ≤ n} ≥ d . 2 (1) (b) Show that there exists a sequence x1 ≤ x2 ≤ · · · ≤ xn of real numbers such that we have equality in (1). 2. A2 (BGR) Consider those functions f : N → N that satisfy the condition f (m + n) ≥ f (m) + f ( f (n)) − 1, for all m, n ∈ N. Find all possible values of f (2007). 3. A3 (EST) Let n be a positive integer, and let x and y be positive real numbers such that xn + yn = 1. Prove that ! ! n n 1 + x2k 1 + y2k 1 ∑ 1 + x4k ∑ 1 + y4k < (1 − x)(1 − y) . k=1 k=1 4. A4 (THA) Find all functions f : R+ → R+ such that f (x + f (y)) = f (x + y) + f (y) for all x, y ∈ R+ . 5. A5 (HRV) Let c > 2, and let a(1), a(2), . . . be a sequence of nonnegative real numbers such that a(m + n) ≤ 2a(m) + 2a(n) for all m, n ≥ 1, and 1 a(2k ) ≤ for all k ≥ 0. (k + 1)c Prove that the sequence a(n) is bounded. 6. A6 (POL) Let a1 , a2 , . . . , a100 be nonnegative real numbers such that a21 + a22 + · · · + a2100 = 1. Prove that a21 a2 + a22a3 + · · · + a2100a1 < 12 . 25 3.48 IMO 2007 333 7. A7 (NLD)IMO6 Let n > 1 be an integer. Consider the following subset of space: S = {(x, y, z)|x, y, z ∈ {0, 1, . . . , n}, x + y + z > 0} . Find the smallest number of planes that jointly contain all (n + 1)3 − 1 points of S but none of them passes through the origin. 8. C1 (SRB) Let n be an integer. Find all sequences a1 , a2 , . . . , an2 +n satisfying the following conditions: (i) ai ∈ {0, 1} for all 1 ≤ i ≤ n2 + n; (ii) ai+1 +ai+2 +· · ·+ai+n < ai+n+1 +ai+n+2 +· · ·+ai+2n for all 0 ≤ i ≤ n2 −n. 9. C2 (JPN) A unit square is dissected into n > 1 rectangles such that their sides are parallel to the sides of the square. Any line parallel to a side of the square and intersecting its interior also intersects the interior of some rectangle. Prove that in this dissection, there exists a rectangle having no point on the boundary of the square. 10. C3 (NLD) Find all positive integers n for which the numbers in the set S = {1, 2, . . . ,n} can be colored red and blue, with the following condition being satisfied: the set S × S × S contains exactly 2007 ordered triples (x, y, z) such that (i) x, y, z are of the same color, and (ii) x + y + z is divisible by n. 11. C4 (IRN) Let A0 = {a1 , . . . , an } be a finite sequence of real numbers. For each k ≥ 0, from the sequence Ak = (x1 , . . . , xn ) we construct a new sequence Ak+1 in the following way: (i) We choose a partition {1, . . . , n} = I ∪ J, where I and J are two disjoint sets, such that the expression ∑ xi − ∑ x j i∈I j∈J attains the smallest possible value. (We allow the set I or J to be empty; in this case the corresponding sum is 0.) If there are several such partitions, one is chosen arbitrarily. (ii) We set Ak+1 = (y1 , . . . , yn ), where yi = xi + 1 if i ∈ I, and yi = xi − 1 if i ∈ J. Prove that for some k, the sequence Ak contains an element x such that |x| ≥ n/2. 12. C5 (ROU) In the Cartesian coordinate plane define the strip Sn = {(x, y) : n ≤ x < n + 1} for every integer n. Assume that each strip Sn is colored either red or blue, and let a and b be two distinct positive integers. Prove that there exists a rectangle with side lengths a and b such that its vertices have the same color. 13. C6 (RUS)IMO3 In a mathematical competition some competitors are friends; friendship is always mutual. Call a group of competitors a clique if each two of them are friends. The number of members in a clique is called its size. 334 3 Problems It is known that the largest size of a clique is even. Prove that the competitors can be arranged in two rooms such that the largest size of a clique in one room is the same as the largest size of a clique in the other room. √ 14. C7 (AUT) Let α < 3−2 5 be a positive real number. Prove that there exist positive integers n and p such that p > α · 2n and for which one can select 2p distinct subsets S1 , . . . , S p , T1 , . . . , Tp of the set {1, 2, . . . , n} such that Si ∩ T j 6= 0/ for all 1 ≤ i, j ≤ p. 15. C8 (UKR) Given a convex n-gon P in the plane, for every three vertices of P, consider the triangle determined by them. Call such a triangle good if all its sides are of unit length. Prove that there are not more than 2n/3 good triangles. 16. G1 (CZE) IMO4 In a triangle ABC the bisector of angle BCA intersects the circumcircle again at R, the perpendicular bisector of BC at P, and the perpendicular bisector of AC at Q. The midpoint of BC is K and the midpoint of AC is L. Prove that the triangles RPK and RQL have the same area. 17. G2 (CAN) Given an isosceles triangle ABC, assume that AB = AC. The midpoint of the side BC is denoted by M. Let X be a variable point on the shorter arc MA of the circumcircle of triangle ABM. Let T be the point in the angle domain BMA for which ∠T MX = 90◦ and T X = BX. Prove that ∠MT B − ∠CT M does not depend on X . 18. G3 (UKR) The diagonals of a trapezoid ABCD intersect at point P. Point Q lies between the parallel lines BC and AD such that ∠AQD = ∠CQB, and the line CD separates the points P and Q. Prove that ∠BQP = ∠DAQ. 19. G4 (LUX) IMO2 Consider five points A, B, C, D, and E such that ABCD is a parallelogram and BCED is a cyclic quadrilateral. Let ℓ be a line passing through A. Suppose that ℓ intersects the interior of the segment DC at F and intersects line BC at G. Suppose also that EF = EG = EC. Prove that ℓ is the bisector of angle DAB. 20. G5 (UNK) Let ABC be a fixed triangle, and let A1 , B1 , C1 be the midpoints of sides BC, CA, AB respectively. Let P be a variable point on the circumcircle. Let lines PA1 , PB1 , PC1 meet the circumcircle again at A′ , B′ , C′ respectively. Assume that the points A, B, C, A′ , B′ , C′ are distinct, and lines AA′ , BB′ , CC′ form a triangle. Prove that the area of this triangle does not depend on P. 21. G6 (USA) Let ABCD be a convex quadrilateral, and let points A1 , B1 , C1 , and D1 lie on sides AB, BC, CD, and DA respectively. Consider the areas of triangles AA1 D1 , BB1 A1 , CC1 B1 , and DD1C1 ; let S be the sum of the two smallest ones, and let S1 be the area of the quadrilateral A1 B1C1 D1 . Find the smallest positive real number k such that kS1 ≥ S holds for every convex quadrilateral ABCD. 22. G7 (IRN) Given an acute triangle ABC with angles α , β , and γ at vertices A, B, and C respectively such that β > γ , let I be its incenter, and R the circumradius. Point D is the foot of the altitude from vertex A. Point K lies on line AD such 3.48 IMO 2007 335 that AK = 2R, and D separates A and K. Finally, lines DI and KI meet sides AC and BC at E and F respectively. Prove that if IE = IF, then β ≤ 3γ . 23. G8 (POL) A point P lies on the side AB of a convex quadrilateral ABCD. Let ω be the incircle of the triangle CPD, and let I be its incenter. Suppose that ω is tangent to the incircles of triangles APD and BPC at points K and L, respectively. Let the lines AC and BD meet at E, and let the lines AK and BL meet at F. Prove that the points E, I, and F are collinear. 24. N1 (AUT) Find all pairs (k, n) of positive integers for which 7k − 3n divides k4 + n 2 . 25. N2 (CAN) Let b, n > 1 be integers. Suppose that for each k > 1 there exists an integer ak such that b − ank is divisible by k. Prove that b = An for some integer A. 26. N3 (NLD) Let X be a set of 10000 integers, none of which is divisible by 47. Prove that there exists a 2007-element subset Y of X such that a − b + c − d + e is not divisible by 47 for any a, b, c, d, e ∈ Y . 27. N4 (POL) For every integer k ≥ 2, prove that 23k divides the number  k+1   k  2 2 − k−1 2k 2 but 23k+1 does not. 28. N5 (IRN) Find all surjective functions f : N → N such that for every m, n ∈ N and every prime p, the number f (m + n) is divisible by p if and only if f (m) + f (n) is divisible by p. 29. N6 (UNK) IMO5 Let k be a positive integer. Prove that the number (4k2 − 1)2 has a positive divisor of the form 8kn − 1 if and only if k is even. 30. N7 (IND) For a prime p and a positive integer n, denote by ν p (n) the exponent of p in the prime factorization of n!. Given a positive integer d and a finite set {p1 , . . . , pk } of primes, show that there are infinitely many positive integers n such that d | ν pi (n) for all 1 ≤ i ≤ k. 336 3 Problems 3.49 The Forty-Ninth IMO Madrid, Spain, July 10–22, 2008 3.49.1 Contest Problems First Day (July 16) 1. An acute-angled triangle ABC has orthocenter H. The circle passing through H with center the midpoint of BC intersects the line BC at A1 and A2 . Similarly, the circle passing through H with center the midpoint of CA intersects the line CA at B1 and B2 , and the circle passing through H with center the midpoint of AB intersects the line AB at C1 and C2 . Show that A1 , A2 , B1 , B2 , C1 , C2 lie on a circle. 2. (a) Prove that x2 y2 z2 + + ≥1 2 2 (x − 1) (y − 1) (z − 1)2 for all real numbers x, y, z each different from 1 and satisfying xyz = 1. (b) Prove that equality holds above for infinitely many triples of rational numbers x, y, z each different from 1 and satisfying xyz = 1. 3. Prove that there exist infinitely many √ positive integers n such that n2 + 1 has a prime divisor that is greater than 2n + 2n. Second Day (July 17) 4. Find all functions f : (0, +∞) → (0, +∞) (so f is a function from the positive real numbers to the positive real numbers) such that ( f (w))2 + ( f (x))2 w2 + x2 = f (y2 ) + f (z2 ) y 2 + z2 for all positive real numbers w, x, y, z satisfying wx = yz. 5. Let n and k be positive integers with k ≥ n and k − n an even number. Let 2n lamps labeled 1, 2, . . . , 2n be given, each of which can be either on or off. Initially all the lamps are off. We consider sequence of steps: at each step one of the lamps is switched (from on to off or from off to on). Let N be the number of such sequences consisting of k steps and resulting in the state in which lamps 1 through n are all on, and lamps n + 1 through 2n are all off. Let M be the number of such sequences consisting of k steps and resulting in the state in which lamps 1 through n are all on, and lamps n + 1 through 2n are all off, but where none of the lamps n + 1 through 2n is ever switched on. Determine the ratio N/M. 3.49 IMO 2008 337 6. Let ABCD be a convex quadrilateral with |BA| 6= |BC|. Denote the incircles of triangles ABC and ADC by ω1 and ω2 respectively. Suppose that there exists a circle ω tangent to the ray BA beyond A and to the ray BC beyond C that is also tangent to the lines AD and CD. Prove that the common external tangents of ω1 and ω2 intersect on ω . 3.49.2 Shortlisted Problems 1. A1 (KOR) IMO4 Find all functions f : (0, +∞) → (0, +∞) (so f is a function from the positive real numbers to the positive real numbers) such that ( f (w))2 + ( f (x))2 w2 + x2 = f (y2 ) + f (z2 ) y 2 + z2 for all positive real numbers w, x, y, z, satisfying wx = yz. 2. A2 (AUT) IMO2 (a) Prove that x2 y2 z2 + + ≥1 (x − 1)2 (y − 1)2 (z − 1)2 for all real numbers x, y, z each different from 1 and satisfying xyz = 1. (b) Prove that equality holds above for infinitely many triples of rational numbers x, y, z each different from 1 and satisfying xyz = 1. 3. A3 (NLD) Let S ⊆ R be a set of real numbers. We say that a pair ( f , g) of functions from S to S is a Spanish couple on S if they satisfy the following conditions: (i) Both functions are strictly increasing, i.e., f (x) < f (y) and g(x) < g(y) for all x, y ∈ S with x < y; (ii) The inequality f (g(g(x))) < g( f (x)) holds for all x ∈ S. Decide whether there exists a Spanish couple (a) on the set S = N of positive integers; (b) on the set S = {a − 1/b : a, b ∈ N}. 4. A4 (AUT) For an integer m, denote by t(m) the unique number in {1, 2, 3} such that m + t(m) is a multiple of 3. A function f : Z → Z satisfies f (−1) = 0, f (0) = 1, f (1) = −1, and f (2n + m) = f (2n − t(m)) − f (m) for all integers m, n ≥ 0 with 2n > m. Prove that f (3p) ≥ 0 holds for all integers p ≥ 0. 5. A5 (SVK) Let a, b, c, d be positive real numbers such that abcd = 1 and a + b + c + d > Prove that a+b+c+d < a b c d + + + . b c d a b c d a + + + . a b c d 338 3 Problems 6. A6 (LTU) Let f : R → N be a function that satisfies     1 1 f x+ = f y+ , for all x, y ∈ R. f (y) f (x) Prove that there is a positive integer that is not a value of f . 7. A7 (GER) Prove that for any four positive real numbers a, b, c, d, the inequality (a − b)(a − c) (b − c)(b − d) (c − d)(c − a) (d − a)(d − b) + + + ≥0 a+b+c b+c+d c+d+a d +a+b holds. Determine all cases of equality. 8. C1 (NLD) A box is a rectangle in the plane whose sides are parallel to the coordinate axes and have positive lengths. Two boxes intersect if they have a common point in their interior or on their boundary. Find the largest n for which there exist n boxes B1 , . . . , Bn such that Bi and B j intersect if and only if i 6≡ j ± 1 (mod n). 9. C2 (SRB) For every positive integer n determine the number of permutations (a1 , . . . , an ) of the set {1, 2, . . . , n} with the following property: 2(a1 + a2 + · · · + ak ) is divisible by k for k = 1, 2, . . . , n. 10. C3 (PER) Consider the set S of all points with integer coordinates in the coordinate plane. For a positive integer k, two distinct points A, B ∈ S will be called k-friends if there is a point C ∈ S such that the area of the triangle ABC is equal to k. A set T ⊆ S will be called a k-clique if every two points in T are k-friends. Find the least positive integer k for which there exists a k-clique with more than 200 elements. 11. C4 (FRA) IMO5 Let n and k be positive integers with k ≥ n and k − n an even number. Let 2n lamps labeled 1, 2, . . . , 2n be given, each of which can be either on or off. Initially all the lamps are off. We consider sequence of steps: at each step one of the lamps is switched (from on to off or from off to on). Let N be the number of such sequences consisting of k steps and resulting in the state in which lamps 1 through n are all on, and lamps n + 1 through 2n are all off. Let M be the number of such sequences consisting of k steps and resulting in the state in which lamps 1 through n are all on, and lamps n + 1 through 2n are all off, but where none of the lamps n + 1 through 2n is ever switched on. Determine the ratio N/M. 12. C5 (RUS) Let S = {x1 , x2 , . . . , xk+l } be a (k + l)-element set of real numbers contained in the interval [0, 1]; k and l are positive integers. A k-element subset A ⊆ S is called nice if 1 1 xi − ∑ k xi ∈A lx ∑ j ∈S\A xj ≤ k+l . 2kl 3.49 IMO 2008 339  2 Prove that the number of nice subsets is at least k+l · k+l k . 13. C6 (NLD) For n ≥ 2, let S1 , S2 , . . . , S2n be 2n subsets of A = {1, 2, 3, . . ., 2n+1 } that satisfy the following property: There do not exist indices a and b with a < b and elements x, y, z ∈ A with x < y < z such that y, z ∈ Sa and x, z ∈ Sb . Prove that at least one of the sets S1 , S2 , . . . , S2n contains no more than 4n elements. 14. G1 (RUS) IMO1 An acute-angled triangle ABC has orthocenter H. The circle passing through H with center the midpoint of BC intersects the line BC at A1 and A2 . Similarly, the circle passing through H with center the midpoint of CA intersects the line CA at B1 and B2 , and the circle passing through H with center the midpoint of AB intersects the line AB at C1 and C2 . Show that A1 , A2 , B1 , B2 , C1 , C2 lie on a circle. 15. G2 (LUX) Given a trapezoid ABCD with parallel sides AB and CD, assume that there exist points E on line BC outside the segment BC, and F inside the segment AD, such that ∠DAE = ∠CBF. Denote by I the intersection point of CD and EF, and by J the intersection point of AB and EF. Let K be the midpoint of the segment EF. Assume that K does not lie on the lines AB and CD. Prove that I belongs to the circumcircle of △ABK if and only if K belongs to the circumcircle of △CDJ. 16. G3 (PER) Let ABCD be a convex quadrilateral and let P and Q be the points such that PQDA and QPBC are cyclic quadrilaterals. Suppose that there exists a point E on the line segment PQ such that ∠PAE = ∠QDE and ∠PBE = ∠QCE. Show that the quadrilateral ABCD is cyclic. 17. G4 (IRN) Let BE and CF be altitudes in an acute triangle ABC. Two circles passing through the points A and F are tangent to the line BC at the points P and Q so that B lies between C and Q. Prove that the lines PE and QF intersect on the circumcircle of △AEF. 18. G5 (NLD) Let k and n be integers with 0 ≤ k ≤ n − 2. Consider a set L of n lines in the plane such that no two of them are parallel and no three have a common point. Denote by I the set of intersection points of lines in L. Let O be a point in the plane not lying on any line of L. A point X ∈ I is colored red if the open line segment (OX ) intersects at most k lines from L. Prove that I contains at least 12 (k + 1)(k + 2) red points. 19. G6 (SRB) Let ABCD be a convex quadrilateral. Prove that there exists a point P inside the quadrilateral such that ∠PAB + ∠PDC = ∠PBC + ∠PAD = ∠PCD + ∠PBA = ∠PDA + ∠PCB = 90◦ if and only if the diagonals AC and BD are perpendicular. 20. G7 (RUS) IMO6 Let ABCD be a convex quadrilateral with |BA| = 6 |BC|. Denote the incircles of triangles ABC and ADC by ω1 and ω2 respectively. Suppose that there exists a circle ω tangent to the ray BA beyond A and to the ray BC beyond 340 3 Problems C that is also tangent to the lines AD and CD. Prove that the common external tangents of ω1 and ω2 intersect on ω . 21. N1 (AUS) Let n be a positive integer and let p be a prime number. Prove that if a, b, c are integers (not necessarily positive) satisfying the equations an + pb = bn + pc = cn + pa, then a = b = c. 22. N2 (IRN) Let a1 , a2 , . . . , an be distinct positive integers, n ≥ 3. Prove that there exist distinct indices i and j such that ai + a j does not divide any of the numbers 3a1 , 3a2, . . . , 3an . 23. N3 (IRN) Let a0 , a1 , a2 be a sequence of positive integers such that the greatest common divisor of any two consecutive terms is greater than the preceding term, i.e. (ai , ai+1 ) > ai−1 for all i ≥ 1. Prove that an ≥ 2n for all n ≥ 0. 24. N4 (SRB) Let n be a positive integer. Show that the numbers  n   n   n   n  2 −1 2 −1 2 −1 2 −1 , , , . . . , n−1 0 1 2 2 −1 are congruent modulo 2n to 1, 3, 5, . . . , 2n − 1 in some order. 25. N5 (FRA) For every n ∈ N let d(n) denote the number of (positive) divisors of n. Find all functions f : N → N with the following properties: (i) d( f (x)) = x for all x ∈ N; (ii) f (xy) divides (x − 1)yxy−1 f (x) for all x, y ∈ N. 26. N6 (LTU) IMO3 Prove that there exist infinitely many √ positive integers n such that n2 + 1 has a prime divisor that is greater than 2n + 2n. 3.50 IMO 2009 341 3.50 The Fiftieth IMO Bremen, Germany, July 10–22, 2009 3.50.1 Contest Problems First Day (July 15) 1. Let n be a positive integer and let a1 , . . . , ak (k ≥ 2) be distinct integers in the set {1, . . . , n} such that n divides ai (ai+1 − 1) for i = 1, . . . , k − 1. Prove that n does not divide ak (a1 − 1). 2. Let ABC be a triangle with circumcenter O. The points P and Q are interior points of the sides CA and AB, respectively. Let K, L and M be the midpoints of the segments BP, CQ, and PQ, respectively, and let Γ be the circle passing through K, L, and M. Suppose that the line PQ is tangent to the circle Γ . Prove that OP = OQ. 3. Suppose that s1 , s2 , s3 , . . . is a strictly increasing sequence of positive integers such that the subsequences ss1 , ss2 , ss3 , . . . and ss1 +1 , ss2 +1 , ss3 +1 , . . . are both arithmetic progressions. Prove that the sequence s1 , s2 , s3 , . . . is itself an arithmetic progression. Second Day (July 16) 4. Let ABC be a triangle with AB = AC. The angle bisectors of ∠CAB and ∠ABC meet the sides BC and CA at D and E, respectively. Let K be the incenter of triangle ADC. Suppose that ∠BEK = 45◦ . Find all possible values of ∠CAB. 5. Determine all functions f from the set of positive integers to the set of positive integers such that, for all positive integers a and b, there exists a nondegenerate triangle with sides of lengths a, f (b), and f (b + f (a) − 1). (A triangle is nondegenerate if its vertices are not collinear.) 6. Let a1 , a2 , . . . , an be distinct positive integers and let M be a set of n − 1 positive integers not containing s = a1 + a2 + · · · + an . A grasshopper is to jump along the real axis, starting at the point 0 and making n jumps to the right with lengths a1 , a2 , . . . , an in some order. Prove that the order can be chosen in such a way that the grasshopper never lands on any point in M. 3.50.2 Shortlisted Problems 1. A1 (CZE) Find the largest possible integer k such that the following statement is true: 342 3 Problems Let 2009 arbitrary nondegenerate triangles be given. In every triangle the three sides are colored, such that one is blue, one is red, and one is white. Now, for every color separately, let us sort the lengths of the sides. We obtain b1 ≤ b2 ≤ · · · ≤ b2009 the lengths of the blue sides, r1 ≤ r2 ≤ · · · ≤ r2009 the lengths of the red sides, and w1 ≤ w2 ≤ · · · ≤ w2009 the lengths of the white sides. Then there exist k indices j such that we can form a nondegenerate triangle with side lengths b j , r j , w j . 2. A2 (EST) Let a, b, c be positive real numbers such that prove that 1 a + 1b + 1c = a + b + c. 1 1 1 3 + + ≤ . 2 2 2 (2a + b + c) (2b + c + a) (2c + a + b) 16 3. A3 (FRA) IMO5 Determine all functions f from the set of positive integers to the set of positive integers such that for all positive integers a and b, there exists a nondegenerate triangle with sides of lengths a, f (b), and f (b + f (a) − 1). (A triangle is nondegenerate if its vertices are not collinear.) 4. A4 (BLR) Let a, b, c be positive real numbers such that ab + bc + ca ≤ 3abc. Prove that s s s  √ √ √ √ a2 + b2 b 2 + c2 c2 + a 2 + + +3 ≤ 2 a + b+ b +c+ c + a . a+b b+c c+a 5. A5 (BLR) Let f be any function that maps the set of real numbers into the set of real numbers. Prove that there exist real numbers x and y such that f (x − f (y)) > y f (x) + x. 6. A6 (USA) IMO3 Suppose that s1 , s2 , s3 , . . . is a strictly increasing sequence of positive integers such that the subsequences ss1 , ss2 , ss3 , . . . and ss1 +1 , ss2 +1 , ss3 +1 , . . . are both arithmetic progressions. Prove that the sequence s1 , s2 , s3 , . . . is itself an arithmetic progression. 7. A7 (JPN) Find all functions f from the set of real numbers into the set of real numbers that satisfy for all real x, y the identity f (x f (x + y)) = f (y f (x)) + x2 . 3.50 IMO 2009 343 8. C1 (NZL) Consider 2009 cards, each having one gold side and one black side, lying in parallel on a long table. Initially all cards show their gold sides. Two players, standing by the same long side of the table, play a game with alternating moves. Each move consists in choosing a block of 50 consecutive cards, the leftmost of which is showing gold, and turning them all over, so those that showed gold now show black and vice versa. The last player who can make a legal move wins. (a) Does the game necessarily end? (b) Does there exist a winning strategy for the starting player? 9. C2 (ROU) For any integer n ≥ 2, let N(n) be the maximal number of triples (ai , bi , ci ), i = 1, . . . , N(n), consisting of nonnegative integers ai , bi , and ci such that the following two conditions are satisfied: (i) ai + bi + ci = n for all i = 1, . . . , N(n), (ii) If i 6= j, then ai 6= a j , bi 6= b j , and ci 6= c j . Determine N(n) for all n ≥ 2. 10. C3 (RUS) Let n be a positive integer. Given a sequence ε1 , . . . , εn−1 with εi = 0 or εi = 1 for each i = 1, . . . , n − 1, the sequences a0 , . . . , an and b0 , . . . , bn are constructed by the following rules: a0 =b0 = 1, a1 = b1 = 7,  2ai−1 + 3ai, if εi = 0, ai+1 = for i = 1, . . . , n − 1, 3ai−1 + ai , if εi = 1,  2bi−1 + 3bi, if εn−i = 0, bi+1 = for i = 1, . . . , n − 1. 3bi−1 + bi , if εn−i = 1, Prove that an = bn . 11. C4 (NLD) For an integer m ≥ 1 we consider partitions of a 2m × 2m chessboard into rectangles consisting of cells of the chessboard in which each of the 2m cells along one diagonal forms a separate rectangle of side length 1. Determine the smallest possible sum of rectangle perimeters in such a partition. 12. C5 (NLD) Five identical empty buckets of 2-liter capacity stand at the vertices of a regular pentagon. Cinderella and her wicked stepmother go through a sequence of rounds: At the beginning of every round the stepmother takes one liter of water from the nearby river and distributes it arbitrarily among the five buckets. Then Cinderella chooses a pair of neighboring buckets, empties them into the river, and puts them back. Then the next round begins. The stepmother’s goal is to make one of the buckets overflow. Cinderella’s goal is to prevent this. Can the wicked stepmother enforce a bucket overflow? 13. C6 (BGR) On a 999 × 999 board a limp rook can move in the following way: From any square it can move to any of its adjacent squares, i.e., a square having a common side with it, and every move must be a turn: i.e., the directions of any two consecutive moves must be perpendicular. A nonintersecting route of the limp rook consists of a sequence of distinct squares that the limp rook can visit 344 3 Problems in that order by an admissible sequence of moves. Such a nonintersecting route is called cyclic if the limp rook can, after reaching the last square of the route, move directly to the first square of the route and start over. How many squares does the longest possible cyclic, nonintersecting route of a limp rook visit? 14. C7 (RUS) IMO6 Let a1 , a2 , . . . , an be distinct positive integers and let M be a set of n − 1 positive integers not containing s = a1 + a2 + · · · + an . A grasshopper is to jump along the real axis, starting at the point 0 and making n jumps to the right with lengths a1 , a2 , . . . , an in some order. Prove that the order can be chosen in such a way that the grasshopper never lands on any point in M. 15. C8 (AUT) For any integer n ≥ 2 we compute the integer h(n) by applying the following procedure to its decimal representation. Denote by r the rightmost digit of n. 1◦ If r = 0, then the decimal representation of h(n) results from the decimal representation of n by removing this rightmost digit 0. 2◦ If 1 ≤ r ≤ 9 we split the decimal representation of n into a maximal right part R that consists solely of digits not less than r and into the left part L that either is empty or ends with a digit strictly smaller than r. Then the decimal representation of h(n) consists of the decimal representation of L, followed by two copies of the decimal representation of R− 1. For instance, for the number n = 17,151,345,543 we will have L = 17,151, R = 345,543, and h(n) = 17,151,345,542,345,542. Prove that, starting with an arbitrary integer n ≥ 2, iterated application of h produces the integer 1 after finitely many steps. 16. G1 (BEL) IMO4 Let ABC be a triangle with AB = AC. The angle bisectors of ∠CAB and ∠ABC meet the sides BC and CA at D and E, respectively. Let K be the incenter of triangle ADC. Suppose that ∠BEK = 45◦ . Find all possible values of ∠CAB. 17. G2 (RUS) IMO2 Let ABC be a triangle with circumcenter O. The points P and Q are interior points of the sides CA and AB, respectively. Let K, L, and M be the midpoints of the segments BP, CQ, and PQ, respectively, and let Γ be the circle passing through K, L, and M. Suppose that the line PQ is tangent to the circle Γ . Prove that OP = OQ. 18. G3 (IRN) Let ABC be a triangle. The incircle of ABC touches the sides AB and AC at the points Z and Y , respectively. Let G be the point where the lines BY and CZ meet, and let R and S be the points such that the two quadrilaterals BCY R and BCSZ are parallelograms. Prove that GR = GS. 19. G4 (UNK) Given a cyclic quadrilateral ABCD, let the diagonals AC and BD meet at E and the lines AD and BC meet at F. The midpoints of AB and CD are G and H, respectively. Show that EF is tangent at E to the circle through the points E, G, and H. 3.50 IMO 2009 345 20. G5 (POL) Let P be a polygon that is convex and symmetric with respect to some point O. Prove that for some parallelogram R satisfying P ⊆ R we have |SR | √ ≤ 2. |SP | 21. G6 (UKR) Let the sides AD and BC of the quadrilateral ABCD (such that AB is not parallel to CD) intersect at point P. Points O1 and O2 are the circumcenters and points H1 and H2 are the orthocenters of the triangles ABP and DCP, respectively. Denote the midpoints of segments O1 H1 and O2 H2 by E1 and E2 , respectively. Prove that the perpendicular from E1 on CD, the perpendicular from E2 on AB, and the line H1 H2 are concurrent. 22. G7 (IRN) Let ABC be a triangle with incenter I and let X, Y , and Z be the incenters of the triangles BIC, CIA, and AIB respectively. Let the triangle XY Z be equilateral. Prove that ABC is equilateral too. 23. G8 (BGR) Let ABCD be a circumscribed quadrilateral. Let g be a line through A that meets the segment BC in M and the line CD in N. Denote by I1 , I2 , and I3 the incenters of △ABM, △MNC, and △NDA, respectively. Show that the orthocenter of △I1 I2 I3 lies on g. 24. N1 (AUS) IMO1 Let n be a positive integer and let a1 , . . . , ak (k ≥ 2) be distinct integers in the set {1, . . . , n} such that n divides ai (ai+1 − 1) for i = 1, . . . , k − 1. Prove that n does not divide ak (a1 − 1). Original formulation: A social club has n members. They have the membership numbers 1, 2, . . . , n, respectively. From time to time members send presents to other members including items they have already received as presents from other members. In order to avoid the embarrassing situation that a member might receive a present that he or she has sent to other members, the club adds the following rule to its statutes at one of its annual general meetings: “A member with membership number a is permitted to send a present to a member with membership number b if and only if a(b − 1) is a multiple of n.” Prove that if each member follows this rule, none will receive a present from another member that he or she has already sent to other members. Alternative formulation: Let G be a directed graph with n vertices v1 , v2 , . . . , vn such that there is an edge going from va to vb if and only if a and b are distinct and a(b − 1) is a multiple of n. Prove that this graph does not contain a directed cycle. 25. N2 (PER) A positive integer N is called balanced if N = 1 or if N can be written as a product of an even number of not necessarily distinct primes. Given positive integers a and b, consider the polynomial P defined by P(x) = (x + a)(x + b). (a) Prove that there exist distinct positive integers a and b such that all the numbers P(1), P(2), . . . , P(50) are balanced. (b) Prove that if P(n) is balanced for all positive integers n, then a = b. 26. N3 (EST) Let f be a nonconstant function from the set of positive integers into the set of positive integers, such that a − b divides f (a) − f (b) for all distinct 346 3 Problems positive integers a and b. Prove that there exist infinitely many primes p such that p divides f (c) for some positive integer c. 27. N4 (PRK) Find all positive integers n such that there exists a sequence of positive integers a1 , a2 , . . . , an satisfying ak+1 = a2k + 1 −1 ak−1 + 1 for every k with 2 ≤ k ≤ n − 1. 28. N5 (HUN) Let P(x) be a nonconstant polynomial with integer coefficients. Prove that there is no function T from the set of integers into the set of integers such that the number of integers x with T n (x) = x is equal to P(n) for every n ≥ 1, where T n denotes the n-fold application of T . 29. N6 (TUR) Let k be a positive integer. Show that if there exists a sequence a0 , a1 , . . . of integers satisfying the condition an = then k − 2 is divisible by 3. an−1 + nk for all n ≥ 1, n 30. N7 (MNG) Let a and b be distinct integers greater than 1. Prove that there exists a positive integer n such that (an − 1)(bn − 1) is not a perfect square. 4 Solutions 4.1 Solutions to the Contest Problems of IMO 1959 1. The desired result (14n + 3, 21n + 4) = 1 follows from 3(14n + 3) − 2(21n + 4) = 1. 2. For roots to be real we must have 2x − 1 ≥ 0 ⇒ x ≥ 1/2 and x ≥ √ the square 2 ≥ 2x − 1 ⇒ (x − 1)2 ≥ 0, which always holds. Then we have 2x − 1 ⇒ x p p √ √ x + 2x − 1 + x − 2x − 1 = c ⇐⇒ q  √ 2 2, 1/2 ≤ x ≤ 1, 2 2 c = 2x + 2 x − 2x − 1 = 2x + 2|x − 1| = 4x − 2, x ≥ 1. (a) c2 = 2. The equation holds for 1/2 ≤ x ≤ 1. (b) c2 = 1. The equation has no solution. (c) c2 = 4. The equation holds for 4x − 2 = 4 ⇒ x = 3/2. 3. Multiplying the equality by 4(a cos2 x − b cosx + c), we obtain 4a2 cos4 x + 2(4ac − 2b2 ) cos2 x + 4c2 = 0. Plugging in 2 cos2 x = 1 + cos 2x we obtain (after quite a bit of manipulation): a2 cos2 2x + (2a2 + 4ac − 2b2) cos 2x + (a2 + 4ac − 2b2 + 4c2 ) = 0. For a = 4, b = 2, and c = −1 we get 4 cos2 x + 2 cosx − 1 = 0 and 16 cos2 2x + 8 cos 2x − 4 = 0 ⇒ 4 cos2 2x + 2 cos2x − 1 = 0. 4. Analysis. Let a and b be the other two sides of the √ triangle. From the conditions 2 = a2 + b2 and c/2 = ab ⇔ 3/2c2 = a2 + b2 + 2ab = of the problem we have c p (a + b)2 ⇔ 3/2c = a + b. Given a desiredp △ABC let D be a point on (AC such that CD = CB. In that case, AD = a + b = 3/2c, and also, since BC = CD, it follows that ∠ADB = 45◦ . Construction. From p a segment of length c we elementarily construct a segment AD of length 3/2 c. We then construct a ray (DX such that ∠ADX = 45◦ D. Djukić et al., The IMO Compendium, Problem Books in Mathematics, DOI 10.1007/978-1-4419-9854-5_4, © Springer Science + Business Media, LLC 2011 347 348 5. 4 Solutions and a circle k(A, c) that intersects the ray at point B. Finally, we construct the perpendicular from B to AD; point C is the foot of that perpendicular. Proof. It holds thatp AB = c, and, since CB = CD, it √ also holds that AC + CB = AC + CD = AD = 3/2 c. From this it follows that AC · CB = c/2. Since BC is perpendicular to AD, it follows that ∡BCA = 90◦ . Thus ABC is the desired triangle. p √ √ Discussion. Since AB 2 = 2c > 3/2 c = AD > AB, the circle k intersects the ray DX in exactly two points, which correspond to two symmetric solutions. (a) It suffices to prove that AF ⊥ BC, since then for the intersection point X we have ∠AXC = ∠BXF = 90◦, implying that X belongs to the circumcircles of both squares and thus that X = N. The relation AF ⊥ BC holds because from MA = MC, MF = MB, and ∠AMC = ∠FMB it follows that △AMF is obtained by rotating △BMC by 90◦ around M. (b) Since N is on the circumcircle of BMFE, it follows that ∠ANM = ∠MNB = 45◦ . Hence MN is the bisector of ∠ANB. It follows that MN passes through c of the circle with diameter AB (i.e., the circumthe midpoint of the arc AB circle of △ABN) not containing N. (c) Let us introduce a coordinate system such that A = (0, 0), B = (b, 0), and M = (m, 0). Setting in general W = (xW , yW ) for an arbitrary point W and denoting by R the midpoint of PQ, we have yR = (yP + yQ )/2 = (m + b − m)/4 = b/4 and xR = (xP + xQ )/2 = (m + m + b)/4 = (2m + b)/4, the parameter m varying from 0 to b. Thus the locus of all points R is the closed segment R1 R2 where R1 = (b/4, b/4) and R2 = (b/4, 3b/4). 6. Analysis. For AB k CD to hold evidently neither must intersect p and hence constructing lines r in α through A and s in β through C, both being parallel to p, we get that B ∈ r and D ∈ s. Hence the problem reduces to a planar problem in γ , determined by r and s. Denote by A′ the foot of the perpendicular from A to s. Since ABCD is isosceles and has an incircle, it follows AD = BC = (AB + CD)/2 = A′C. The remaining parts of the problem are now obvious. 4.2 Contest Problems 1960 349 4.2 Solutions to the Contest Problems of IMO 1960 1. Given the number acb, since 11 | acb, it follows that c = a + b or c = a + b − 11. In the first case, a2 + b2 + (a + b)2 = 10a + b, and in the second case, a2 + b2 + (a + b − 11)2 = 10(a − 1) + b. In the first case the LHS is even, and hence b ∈ {0, 2, 4, 6, 8}, while in the second case it is odd, and hence b ∈ {1, 3, 5, 7, 9}. Analyzing the 10 quadratic equations for a we obtain that the only valid solutions are 550 and 803. 2. The for x ≥ −1/2 and x 6= 0. Furthermore, 4x2 /(1 − √ LHS2term is well-defined √ 2 1 + 2x) = (1 + 1 + 2x) . Since  2 √ √ f (x) = 1 + 1 + 2x − 2x − 9 = 2 1 + 2x − 7 is increasing and since f (45/8) = 0, it follows that the inequality holds precisely for −1/2 ≤ x < 45/8 and x 6= 0. 3. Let B′C′ be the middle of the n = 2k + 1 segments and let D be the foot of the perpendicular from A to the hypotenuse. Let us assume B(C, D,C′ , B′ , B). Then from CD < BD, CD√ + BD = a, and CD · BD = h2 we have CD2 − a ·CD + h2 = 0 =⇒ CD = (a − a2 − 4h2 )/2 . Let us define ∡DAC′ = γ and ∡DAB′ = β ; ′ −CD = (k + 1)a/(2k + then tan β √ = DB′ /h and tan γ = DC′ /h. Since DB′ = CB√ ′ 2 2 1) − (c − c − 4h )/2 and DC = ka/(2k + 1) − (c − c2 − 4h2)/2, we have a tan α = tan(β − γ ) = = tan β − tan γ (2k+1)h = 2 −4h2 a2 1 + tan β · tan γ 1 + a 4h − 4h2 (2k+1) 2 2 4h(2k + 1) 4nh = . 4ak(k + 1) (n2 − 1)a The case B(C,C′ , D, B′ , B) is similar. 4. Analysis. Let A′ and B′ be the feet of the perpendiculars from A and B, respectively, to the opposite sides, A1 the midpoint of BC, and let D′ be the foot of the perpendicular from A1 to AC. We then have AA1 = ma , AA′ = ha , ∠AA′ A1 = 90◦ , A1 D′ = hb /2, and ∠AD′ A1 = 90◦ . Construction. We construct the quadrilateral AD′ A1 A′ (starting from the circle with diameter AA1 ). Then C is the intersection of A′ A1 and AD′ , and B is on the line A1C such that CA1 = A1 B and B(B, A1 ,C). Discussion. We must have ma ≥ ha and ma ≥ hb /2. The number of solutions is 0 if ma = ha = hb /2, 1 if two of ma , ha , hb /2 are equal, and 2 otherwise. 5. (a) The locus of the points is the square EFGH where these four points are the centers of the faces ABB′ A′ , BCC′ B′ , CDD′C′ and DAA′ D′ . (b) The locus of the points is the rectangle IJKL where these points are on AB′ , CB′ , CD′ , and AD′ at a distance of AA′ /3 with respect to the plane ABCD. 350 4 Solutions 6. Let E, F respectively be the midpoints of the bases AB,CD of the isosceles trapezoid ABCD. (a) The point P is on the intersection of EF and the circle with diameter BC. (b) Let x√= EP. Since △BEP ∼ △PFC, we have x(h − x) = ab/4 ⇒ x1,2 = (h ± h2 − ab)/2 . (c) If h2 > ab there are two solutions, if h2 = ab there is only one solution, and if h2 < ab there are no solutions. 7. Let A be the vertex of the cone, O the center of the sphere, S the center of the base of the cone, B a point on the base circle, and r the radius of the sphere. Let ∠SAB = α . We easily obtain AS = r(1 + sin α )/ sin α and SB = r(1 + sin α ) tan α / sin α and hence V1 = π SB2 · SA/3 = π r3 (1 + sin α )2 /[3 sin α (1 − sin α )] . We also have V2 = 2π r3 and hence k= (1 + sin α )2 ⇒ (1 + 6k) sin2 α + 2(1 − 3k) sin α + 1 = 0 . 6 sin α (1 − sin α ) The discriminant of this quadratic must be nonnegative: (1 − 3k)2 − (1 + 6k) ≥ 0 ⇒ k ≥ 4/3. Hence we cannot have k = 1. For k = 4/3 we have sin α = 1/3, whose construction is elementary. 4.3 Contest Problems 1961 351 4.3 Solutions to the Contest Problems of IMO 1961 1. This is a problem solvable using elementary manipulations, so we shall state only the final solutions. For a = 0 we get (x, y, z) = (0, 0, 0). For a 6= 0 we get (x, y, z) ∈ {(t1 ,t2 , z0 ), (t2 ,t1 , z0 )}, where p a2 − b2 a2 + b2 ± (3a2 − b2 )(3b2 − a2) z0 = and t1,2 = . 2a 4a For the solutions to be positive and distinct the following conditions are necessary and sufficient: 3b2 > a2 > b2 and a > 0. √ 2. Using S = bc sin α /2 , a2 = b2 + c2 − 2bc cos α and ( 3 sin α + cos α )/2 = cos(α − 60◦) we have √ √ a2 + b2 + c2 ≥ 4S 3 ⇔ b2 + c2 ≥ bc( 3 sin α + cos α ) ⇔ ⇔ (b − c)2 + 2bc(1 − cos(α − 60◦ )) ≥ 0, where equality holds if and only if b = c and α = 60◦, i.e., if the triangle is equilateral. 3. For n ≥ 2 we have 1 = cosn x − sinn x ≤ | cosn x − sinn x| ≤ | cosn x| + | sinn x| ≤ cos2 x + sin2 x = 1. Hence sin2 x = | sinn x| and cos2 x = | cosn x|, from which it follows that sin x, cos x ∈ {1, 0, −1} ⇒ x ∈ π Z/2. By inspection one obtains the set of solutions {mπ | m ∈ Z} for even n and {2mπ , 2mπ − π /2 | m ∈ Z} for odd n. √ For n = 1 we have 1 = cos x − sin x = − 2 sin(x − π /4), which yields the set of solutions {2mπ , 2mπ − π /2 | m ∈ Z}. 4. Let xi = PPi /PQi for i = 1, 2, 3. For all i we have SPPj Pk 1 PQi = = , xi + 1 Pi Qi SP1 P2 P3 where the indices j and k are distinct and different from i. Hence we have 1 1 1 + + x1 + 1 x2 + 1 x3 + 1 S(PP2 P3 ) + S(PP1P3 ) + S(PP2P3 ) = = 1. S(P1 P2 P3 ) f (x1 , x2 , x3 ) = It follows that 1/(xi + 1) ≥ 1/3 for some i and 1/(x j + 1) ≤ 1/3 for some j. Consequently, xi ≤ 2 and x j ≥ 2. 352 4 Solutions 5. Analysis. Let C1 be the midpoint of AB. In △AMB we have MC1 = b/2, AB = c, and ∠AMB = ω . Thus, given AB = c, the point M is at the intersection of the circle k(C′ , b/2) and the set of points e that view AB at an angle of ω . The construction of ABC is now obvious. Discussion. It suffices to establish the conditions for which k and e intersect. Let E be the midpoint of one of the arcs that make up e. A necessary and sufficient condition for k to intersect e is c b c ω ω = C′ A ≤ ≤ C′ E = cot ⇔ b tan ≤ c < b. 2 2 2 2 2 6. Let h(X ) denote the distance of a point X from ε , X restricted to being on the same side of ε as A, B, and C. Let G1 be the (fixed) centroid of △ABC and G′1 the centroid of △A′ B′C′ . It is trivial to prove that G is the midpoint of G1 G′1 . Hence varying G′1 across ε , we get that the locus of G is the plane α parallel to ε such that X ∈ α ⇔ h(X ) = h(G1 ) h(A) + h(B) + h(C) = . 2 6 4.4 Contest Problems 1962 353 4.4 Solutions to the Contest Problems of IMO 1962 1. From the conditions of the problem we have n = 10x + 6 and 4n = 6 · 10m + x for some integer x. Eliminating x from these two equations, we get 40n = 6 · 10m+1 + n − 6 ⇒ n = 2(10m+1 − 1)/13. Hence we must find the smallest m such that this fraction is an integer. By inspection, this happens for m = 6, and for this m we obtain n = 153846, which indeed satisfies the conditions of the problem. √ √ 2. We note that f (x) = 3 − x − x + 1 is well-defined only for −1 ≤ x ≤ 3 and is decreasing (and obviously continuous) on this interval. We also note that f (−1) = 2 > 1/2 and v v ! √ ! u √ !2 u u u 1 √31 2 1 31 1 31 f 1− =t + −t − = . 8 4 4 4 4 2 √ Hence the inequality is satisfied for −1 ≤ x < 1 − 31/8. 3. By inspecting the four different stages of this periodic motion we easily obtain that the locus of the midpoints of XY is the edges of MNCQ, where M, N, and Q are the centers of ABB′ A′ , BCC′ B′ , and ABCD, respectively.     4. Since cos 2x = 1 + cos2 x and cos α + cos β = 2 cos α +2 β cos α −2 β , we have cos2 x + cos2 2x + cos2 3x = 1 ⇔ cos 2x + cos 4x + 2 cos2 3x = 2 cos3x(cos x + cos 3x) = 0 ⇔ 4 cos 3x cos2x cos x = 0. Hence the solutions are x ∈ {π /2 + mπ , π /4 + mπ /2, π /6 + mπ /3 | m ∈ Z}. 5. Analysis. Let ABCD be the desired quadrilateral. Let us assume w.l.o.g. that AB > BC (for AB = BC the construction is trivial). For a tangent quadrilateral we have AD − DC = AB − BC. Let X be a point on AD such that DX = DC. We then have AX = AB − BC and ∡AXC = ∡ADC + ∡CDX = 180◦ − ∠ABC/2. Constructing X and hence D is now obvious. 6. This problem is a special case, when the triangle is isosceles, of Euler’s formula, which holds for all triangles. 7. The spheres are arranged in a similar manner as in the planar case where we have one incircle and three excircles. Here we have one “insphere” and four “exspheres” corresponding to each of the four sides. Each vertex of the tetrahedron effectively has three tangent lines drawn from it to each of the five spheres. Repeatedly using the equality of the three tangent segments from a vertex (in the same vein as for tangent planar quadrilaterals) we obtain SA + BC = SB + CA = SC + AB from the insphere. From the exsphere opposite of S we obtain SA − BC = SB −CA = SC − AB, hence SA = SB = SC and AB = BC = CA. By symmetry, we also have AB = AC = AS. Hence indeed, all the edges of the tetrahedron are equal in length and thus we have shown that the tetrahedron is regular. 354 4 Solutions 4.5 Solutions to the Contest Problems of IMO 1963 1. Obviously, x ≥ 0 and p ≥ 0; p hence squaring the given equation p yields an equivalent equation 5x2 − p − 4 + 4 (x2 − 1)(x2 − p) = x2 , i.e., 4 (x2 − 1)(x2 − p) = (p + 4) − 4x2 . If 4x2 ≤ (p + 4), we may square the equation once again to get −16(p + 1)x2 + 16p = −8(p + 4)x2 + (p + 4)2 , which is equivalent to x2 = (4 − p)2 /[4(4 − 2p)]. We immediately get p < 2 hence 4− p x= √ . 2 4 − 2p Placing this x in the original equation gives |3p − 4| = 4 − 3p hence 0 ≤ p ≤ 43 . Thus for 0 ≤ p ≤ 43 we have x = (4 − p)2 /[4(4 − 2p)], otherwise the solution doesn’t exist. 2. Let A be the given point, BC the given segment, and B1 , B2 the closed balls with the diameters AB and AC respectively. Consider one right angle ∠AOK with K ∈ [BC]. If B′ ,C′ are the feet of the perpendiculars from B,C to AO respectively, then O lies on the segment B′C′ , which implies that it lies on exactly one of the segments AB′ , AC′ . Hence O belongs to exactly one of the balls B1 , B2 ; i.e., O ∈ B1 ∆ B2 . This is obviously the required locus. −−→ −−→ −−→ 3. Let OA1 , OA2 , . . . , OAn be the vectors corresponding respectively to the edges a1 , a2 , . . . , an of the polygon. By the conditions of the problem, these vectors −−→ −−→ − → satisfy OA1 + · · · + OAn = 0 , ∠A1 OA2 = ∠A2 OA3 = · · · = ∠An OA1 = 2π /n and OA1 ≥ OA2 ≥ · · · ≥ OAn . Our task is to prove that OA1 = · · · = OAn . Let l be the line through O perpendicular to OAn , and B1 , . . . , Bn−1 the projections of A1 , . . . , An−1 onto l respectively. By the assumptions, the sum of the −−→ → − OBi ’s is 0 . On the other hand, since OBi ≤ OBn−i for all i ≤ n/2, all the sums −−→ −−−→ OBi + OBn−i lie on the same side of the point O. Hence all these sums must → − be equal to 0 . Consequently, OAi = OAn−i , from which the result immediately follows. 4. Summing up all the equations yields 2(x1 + x2 + x3 + x4 + x5 ) = y(x1 + x2 + x3 + x4 +x5 ). If y = 2, then the given equations imply x1 −x2 = x2 −x3 = · · · = x5 −x1 ; hence x1 = x2 = · · · = x5 , which is clearly a solution. If y 6= 2, then x1 + · · ·+x5 = 0, and summing the first three equalities gives x2 = y(x1 + x2 + x3 ). Using that x1 + x3 = yx2 we obtain x2 = (y2 + y)x2 , i.e., (y2 + y − 1)x2 = 0. If y2 + y − 1 6= 0, then x2 = 0, and similarly x1 = · · · = x5 = 0. If y2 + y − 1 = 0, it is easy to prove that the last two equations are the consequence of the first three. Thus choosing any values for x1 and x5 will give exactly one solution for x2 , x3 , x4 . 5. The LHS of the desired identity equals S = cos(π /7) + cos(3π /7) + cos(5π /7). Now sin 27π sin 47π − sin 27π sin 67π − sin 47π sin 67π π 1 = + + = ⇒ S= . 7 2 2 2 2 2 6. The result is EDACB. S sin 4.6 Contest Problems 1964 355 4.6 Solutions to the Contest Problems of IMO 1964 1. Let n = 3k + r, where 0 ≤ r < 2. Then 2n = 23k+r = 8k · 2r ≡ 2r (mod 7). Thus the remainder of 2n modulo 7 is 1, 2, 4 if n ≡ 0, 1, 2 (mod 3). Hence 2n − 1 is divisible by 7 if and only if 3 | n, while 2n + 1 is never divisible by 7. 2. By substituting a = x + y, b = y + z, and c = z + x (x, y, z > 0) the given inequality becomes 6xyz ≤ x2 y + xy2 + y2 z + yz2 + z2 x + zx2 , which follows immediately by the AM–GM inequality applied to x2 y, xy2 , x2 z, xz2 , y2 z, yz2 . 3. Let r be the radius of the incircle of △ABC, ra , rb , rc the radii of the smaller circles corresponding to A, B,C, and ha , hb , hc the altitudes from A, B,C respectively. The coefficient of similarity between the smaller triangle at A and the triangle ABC is 1 − 2r/ha , from which we easily obtain ra = (ha − 2r)r/ha = (s − a)r/s. Similarly, rb = (s − b)r/s and rc = (s − c)r/s. Now a straightforward computation gives that the sum of areas of the four circles is given by Σ= (b + c − a)(c + a − b)(a + b − c)(a2 + b2 + c2 )π . (a + b + c)3 4. Let us call the topics T1 , T2 , T3 . Consider an arbitrary student A. By the pigeonhole principle there is a topic, say T3 , he discussed with at least 6 other students. If two of these 6 students discussed T3 , then we are done. Suppose now that the 6 students discussed only T1 and T2 and choose one of them, say B. By the pigeonhole principle he discussed one of the topics, say T2 , with three of these students. If two of these three students also discussed T2 , then we are done. Otherwise, all the three students discussed only T1 , which completes the task. 5. Let us first compute the number of intersection points of the perpendiculars passing through two distinct points B and C. The perpendiculars from B to the lines through C other than BC meet all perpendiculars from C, which counts to 3 · 6 = 18 intersection points. Each perpendicular from B to the 3 lines not containing C can intersect at most 5 of the perpendiculars passing through C, which counts to another 3 · 5 = 15 intersection points. Thus there are 18 + 15 = 33 intersection points corresponding to B,C. It follows that the required total number is at most 10 · 33 = 330. But some of these points, namely the orthocenters of the triangles with vertices at the given points, are counted thrice. There are 10 such points. Hence the maximal number of intersection points is 330 − 2 · 10 = 310. Remark. The jury considered only the combinatorial part of the problem and didn’t require an example in which 310 points appear. However, it is “easily” verified that, for instance, the set of points A(1, 1), B(e, π ), C(e2 , π 2 ), D(e3 , π 3 ), E(e4 , π 4 ) works. 356 4 Solutions 6. We shall prove that the statement is valid in the general case, for an arbitrary point D1 inside △ABC. Since D1 belongs to the plane ABC, there are real num−→ −→ −−→ −→ bers a, b, c such that (a + b + c)DD1 = aDA + bDB + cDC. Since AA1 k DD1 , −−→ −−→ −−→ it holds that AA1 = kDD1 for some k ∈ R. Now it is easy to get DA1 = −→ −→ −−→ −→ −→ −→ −→ −−→ −(bDB+ cDC)/a, DB1 = −(aDA+cDC)/b, and DC1 = −(aDA+ bDB)/c. This implies −→ −→ − → a2 DA + b(a + 2b + c)DB+ c(a + b + 2c)DC −−−→ D1 A 1 = − , a(a + b + c) − → −→ −→ a(2a + b + c)DA+ b2 DB + c(a + b + 2c)DC −−−→ D1 B 1 = − , and b(a + b + c) − → −→ − → −−−→ a(2a + b + c)DA+ b(a + 2b + c)DB+ c2 DC D1C1 = − . c(a + b + c) By using 6VD1 A1 B1C1 = we get VD1 A1 B1C1 h−−−→ −−−→ −−−→i h−→ −→ −→i D1 A1 , D1 B1 , D1C1 and 6VDABC = DA, DB, DC a2 b(a + 2b + c) c(a + b + 2c) a(2a + b + c) b2 c(a + b + 2c) a(2a + b + c) b(a + 2b + c) c2 = 3 6abc(a + b + c) h−→ −→ −→i · DA, DB, DC = 3VDABC . 4.7 Contest Problems 1965 357 4.7 Solutions to the Contest Problems of IMO 1965 p √ √ 1. Let us set S = 1 + sin2x − 1 − sin2x . Notice that S2 = 2 − 2 1 − sin2 2x = √ 2 − 2| cos2x| ≤ 2, implying S ≤ 2. Thus the righthand inequality holds for all x. It remains to investigate the left-hand inequality. If π /2 ≤ x ≤ 3π /2, then cos x ≤ 0 and the inequality trivially holds. Assume now that cos x > 0. Then the inequality is equivalent to 2 + 2 cos2x = 4 cos2 x ≤ S2 = 2 − 2| cos2x|, which is equivalent to cos 2x ≤ 0, i.e., to x ∈ [π /4, π /2] ∪ [3π /2, 7π /4]. Hence the solution set is π /4 ≤ x ≤ 7π /4. 2. Suppose that (x1 , x2 , x3 ) is a solution. We may assume w.l.o.g. that |x1 | ≥ |x2 | ≥ |x3 |. Suppose that |x1 | > 0. From the first equation we obtain that 0 = |x1 | · a11 + a12 x2 x3 + a13 ≥ |x1 | · (a11 − |a12| − |a13|) > 0, x1 x1 which is a contradiction. Hence |x1 | = 0 and consequently x1 = x2 = x3 = 0. 3. Let d denote the distance between the lines AB and CD. Being parallel to AB and CD, the plane π intersects the faces of the tetrahedron in a parallelogram EFGH. Let X ∈ AB be a point such that HX k DB. Clearly VAEHBFG = VAXEH +VXEHBFG . D Let MN be the common perpendicN ular to lines AB and CD (M ∈ AB, N ∈ CD) and let MN, BN meet the H plane π at Q and R respectively. Then G it holds that BR/RN = MQ/QN = k Q C and consequently AX/XB = AE/EC = R A E AH/HD = BF/FC = BG/GD = k. F X Now we have VAXEH /VABCD = k3 /(k + M 1)3 . Furthermore, if h = 3VABCD/SABC B is the height of ABCD from D, then VXEHBFG = 1 k SXBFE h and 2 k+1 SXBFE = SABC − SAXE − SEFC = (k + 1)2 − 1 − k2 2k = . 2 (k + 1) (1 + k)2 These relations give us VXEHBFG /VABCD = 3k2 /(1 + k)3. Finally, VAEHBFG k3 + 3k2 = . VABCD (k + 1)3 Similarly, VCEFDHG /VABCD = (3k + 1)/(k + 1)3 , and hence the required ratio is (k3 + 3k2 )/(3k + 1). 358 4 Solutions 4. It is easy to see that all xi are nonzero. Let x1 x2 x3 x4 = p. The given system of equations can be rewritten as xi + p/xi = 2, i = 1, 2, 3, 4. The equation x+ p/x = 2 has at most two real solutions, say y and z. Then each xi is equal either to y or to z. There are three cases: (i) x1 = x2 = x3 = x4 = y. Then y + y3 = 2 and hence y = 1. (ii) x1 = x2 = x3 = y, x4 = z. Then z + y3 = y + y2 z = 2. It is easy to obtain that the only possibilities for (y, z) are (−1, 3) and (1, 1). (iii) x1 = x2 = y, x3 = x4 . In this case the only possibility is y = z = 1. Hence the solutions for (x1 , x2 , x3 , x4 ) are (1, 1, 1, 1), (−1, −1, −1, 3), and the cyclic permutations. 5. (a) Let A′ and B′ denote the feet of the perpendiculars from A and B to OB and OA respectively. We claim that H ∈ A′ B′ . Indeed, since MPHQ is a parallelogram, we have B′ P/B′ A = BM/BA = MQ/AA′ = PH/AA′ , which implies by Thales’s theorem that H ∈ A′ B′ . It is easy to see that the locus of H is the whole segment A′ B′ . (b) In this case the locus of points H is obviously the interior of the triangle OA′ B′ . 6. We recall the simple statement that every two diameters of a set must have a common point. Consider any point B that is an endpoint of k ≥ 2 diameters BC1 , BC2 , . . . , BCk . We may assume w.l.o.g. that all the points C1 , . . . ,Ck lie on the arc C1Ck , whose center is B and measure does not exceed 60◦ . We observe that for 1 < i < k any diameter with the endpoint Ci has to intersect both the diameters C1 B and Ck B. Hence Ci B is the only diameter with an endpoint at Ci if i = 2, . . . , k − 1. In other words, with each point that is an endpoint of k ≥ 2 we can associate k − 2 points that are endpoints of exactly one diameter. We now assume that each Ai is an endpoint of exactly ki ≥ 0 diameters, and that k1 , . . . , ks ≥ 2, while ks+1 , . . . , kn ≤ 1. The total number D of diameters satisfies the inequality 2D ≤ k1 + k2 + · · · + ks + (n − s). On the other hand, by the above consideration we have (k1 − 2) + · · · + (ks − 2) ≤ n − s, i.e., k1 + · · · + ks ≤ n + s. Hence 2D ≤ (n + s) + (n − s) = 2n, which proves the result. 4.8 Contest Problems 1966 359 4.8 Solutions to the Contest Problems of IMO 1966 1. Let Na , Nb , Nc , Nab , Nac , Nbc , Nabc denote the number of students who solved exactly the problems whose letters are stated in the index of the variable. From the conditions of the problem we have Na + Nb + Nc + Nab + Nbc + Nac + Nabc = 25, Nb + Nbc = 2(Nc + Nbc ), Na − 1 = Nac + Nabc + Nab , Na = Nb + Nc . From the first and third equations we get 2Na + Nb + Nc + Nbc = 26, and from the second and fourth we get 4Nb + Nc = 26 and thus Nb ≤ 6. On the other hand, we have from the second equation Nb = 2Nc + Nbc ⇒ Nc ≤ Nb /2 ⇒ 26 ≤ 9Nb /2 ⇒ Nb ≥ 6; hence Nb = 6. 2. Angles α and β are less than 90◦ , otherwise if w.l.o.g. α ≥ 90◦ we have tan(γ /2) · (a tan α + b tan β ) < b tan(γ /2) tan β ≤ b tan(γ /2) cot(γ /2) = b < a + b . Since a ≥ b ⇔ tan a ≥ tan b, Chebyshev’s inequality gives a tan α + b tan β ≥ (a + b)(tan α + tan β )/2. Due to the convexity of the tan function we also have (tan α + tan β )/2 ≥ tan[(α + β )/2] = cot(γ /2). Hence we have γ 1 γ tan (a tan α + b tan β ) ≥ tan (a + b)(tan α + tan β ) 2 2 2 γ γ ≥ tan (a + b) cot = a + b. 2 2 The equalities can hold only if a = b. Thus the triangle is isosceles. 3. Consider a coordinate system in which the points of the regular tetrahedron are placed at A(−a, −a, −a), B(−a, a, a), C(a, −a, a) and D(a, a, −a). Then the center of the tetrahedron is at O(0, 0, 0). For a point X (x, y, z) we see that the √ sum XA + XB + XC + X D by the QM–AM inequality does not exceed 2 XA2 + XB2 + XC2 + XD2 . Now, since X A2 = (x + a)2 + (y + a)2 + (z + a)2 etc., we easily obtain XA2 + XB2 + XC2 + X D2 = 4(x2 + y2 + z2 ) + 12a2 ≥ 12a2 = OA2 + OB2 + OC2 + OD2 . √ Hence XA + XB + XC + XD ≥ 2 OA2 + OB2 + OC2 + OD2 = OA + OB + OC + OD. 4. It suffices to prove 1/sin 2k x = cot 2k−1 x − cot2k x for any integer k and real x, i.e., 1/sin 2x = cot x − cot2x for all real x. We indeed have cot x − cot2x = cot x − cot2 x − 1 = 2 cotx  cos x 2 +1 sin x 2 cosx sinx = 1 1 = . 2 sin x cos x sin 2x 5. We define L1 = |a1 − a2 |x2 + |a1 − a3 |x3 + |a1 − a4 |x4 and analogously L2 , L3 , and L4 . Let us assume w.l.o.g. that a1 < a2 < a3 < a4 . In that case, 360 4 Solutions 2|a1 − a2 ||a2 − a3 |x2 = |a3 − a2 |L1 − |a1 − a3 |L2 + |a1 − a2 |L3 = |a3 − a2 | − |a1 − a3| + |a1 − a2| = 0, 2|a2 − a3 ||a3 − a4 |x3 = |a4 − a3 |L2 − |a2 − a4 |L3 + |a2 − a3 |L4 = |a4 − a3 | − |a2 − a4| + |a2 − a3| = 0. Hence it follows that x2 = x3 = 0 and consequently x1 = x4 = 1/|a1 − a4 |. This solution set indeed satisfies the starting equations. It is easy to generalize this result to any ordering of a1 , a2 , a3 , a4 . 6. Let S denote the area of △ABC. Let A1 , B1 ,C1 be the midpoints of BC, AC, AB respectively. We note that SA1 B1C = SA1 BC1 = SAB1C1 = SA1 B1C1 = S/4. Let us assume w.l.o.g. that M ∈ [AC1 ]. We then must have K ∈ [BA1 ] and L ∈ [CB1 ]. However, we then have S(KLM) > S(KLC1 ) > S(KB1C1 ) = S(A1 B1C1 ) = S/4. Hence, by the pigeonhole principle one of the remaining three triangles △MAL, △KBM, and △LCK must have an area less than or equal to S/4. This completes the proof. 4.9 Longlisted Problems 1967 361 4.9 Solutions to the Longlisted Problems of IMO 1967 1. Let us denote the nth term of the given sequence by an . Then   1 103n+3 − 102n+3 102n+2 − 10n+1 10n+2 − 1 an = +7 + 3 9 9 9  n+1 3 1 10 −1 = (103n+3 − 3 · 102n+2 + 3 · 10n+1 − 1) = . 27 3 2. (n!)2/n = ((1 · 2 · · · n)1/n )2 ≤  1+2+···+n 2 n =  n+1 2 2 ≤ 13 n2 + 12 n + 16 . 3. Consider the function f : [0, π /2] → R defined by f (x) = 1 − x2 /2 + x4 /16 − cos x. It is easy to calculate that f ′ (0) = f ′′ (0) = f ′′′ (0) = 0 and f ′′′′ (x) = 3/2 − cosx. Since f ′′′′ (x) > 0, f ′′′ (x) is increasing. Together with f ′′′ (0) = 0, this gives f ′′′ (x) > 0 for x > 0; hence f ′′ (x) is increasing, etc. Continuing in the same way we easily conclude that f (x) > 0. 4. (a) Let ABCD be a parallelogram, and K, L the midpoints of segments BC and CD respectively. The sides of △AKL are equal and parallel to the medians of △ABC. (b) Using the formulas 4m2a = 2b2 + 2c2 − a2 etc., it is easy to obtain that m2a + m2b = m2c is equivalent to a2 + b2 = 5c2 . Then 5(a2 + b2 − c2 ) = 4(a2 + b2 ) ≥ 8ab. 5. If one of x, y, z is equal to 1 or −1, then we obtain solutions (−1, −1, −1) and (1, 1, 1). We claim that these are the only solutions to the system. Let f (t) = t 2 + t − 1. If among x, y, z one is greater than 1, say x > 1, we have x < f (x) = y < f (y) = z < f (z) = x, which is impossible. It follows that x, y, z ≤ 1. Suppose now that one of x, y, z, say x, is less than −1. Since mint f (t) = −5/4, we have x = f (z) ∈ [−5/4, −1). Also, since f ([−5/4, −1)) = (−1, −11/16) ⊆ (−1, 0) and f ((−1, 0)) = [−5/4, −1), it follows that y = f (x) ∈ (−1, 0), z = f (y) ∈ [−5/4, −1), and x = f (z) ∈ (−1, 0), which is a contradiction. Therefore −1 ≤ x, y, z ≤ 1. If −1 < x, y, z < 1, then x > f (x) = y > f (y) = z > f (z) = x, a contradiction. This proves our claim. 6. The given system has two solutions: (−2, −1) and (−14/3, 13/3). 7. Let Sk = xk1 + xk2 + · · · + xkn and let σk , k = 1, 2, . . . , n denote the kth elementary symmetric polynomial in x1 , . . . , xn . The given system can be written as Sk = ak , k = 1, . . . , n. Using Newton’s formulas kσk = S1 σk−1 − S2 σk−2 + · · · + (−1)k Sk−1 σ1 + (−1)k−1 Sk , k = 1, 2, . . . , n, 362 4 Solutions the system easily leads to σ1 = a and σk = 0 for k = 2, . . . , n. By Vieta’s formulas, x1 , x2 , . . . , xn are the roots of the polynomial xn − axn−1 , i.e., a, 0, 0, . . ., 0 in some order. Remark. This solution does not use the assumption that the x j ’s are real. 8. The circles KA , KB , KC , KD cover the parallelogram if and only if for every point X inside the parallelogram, the length of one of the segments X A, X B, XC, X D does not exceed 1. Let O and r be the center and radius of the circumcircle of △ABD. For every point X inside △ABD, it holds that X A ≤ r or X B ≤ r or X D ≤ r. Similarly, for X inside △BCD, XB ≤ r or XC ≤ r or X D ≤ r. Hence KA , KB , KC , KD cover the parallelogram if and only if r ≤ 1, which is equivalent to ∠ABD ≥ 30◦ . However, √ this last is exactly equivalent to a = AB = 2r sin ∠ADB ≤ 2 sin(α + 30◦ ) = 3 sin α + cos α . 9. The incenter of any such triangle lies inside the circle k. We shall show that every point S interior to the circle S is the incenter of one such triangle. If S lies on the segment AB, then it is obviously the incenter of an isosceles triangle inscribed in k that has AB as an axis of symmetry. Let us now suppose S does not lie on AB. Let X and Y be the intersection points of lines AS and BS with k, and let Z be the foot of the perpendicular from S to AB. Since the quadrilateral BZSX is cyclic, we have ∠ZXS = ∠ABS = ∠SXY and analogously ∠ZY S = ∠SY X , which implies that S is the incenter of △XY Z. 10. Let n be the number of triangles and let b and i be the numbers of vertices on the boundary and in the interior of the square, respectively. Since all the triangles are acute, each of the vertices of the square belongs to at least two triangles. Additionally, every vertex on the boundary belongs to at least three, and every vertex in the interior belongs to at least five triangles. Therefore 3n ≥ 8 + 3b + 5i. Moreover, the sum of angles at any vertex that lies in the interior, on the boundary, or at a vertex of the square is equal to 2π , π , π /2 respectively. The sum of all angles of the triangles equals nπ , which gives us nπ = 4 · π /2 + bπ + 2iπ , i.e., n = 2 + b + 2i. This relation together with (1) easily yields that i ≥ 2. Since each of the vertices inside the square belongs to at least five triangles, and at most two contain both, it follows that n ≥ 8. (1) C D K A L B It is shown in the figure that the square can be decomposed into eight acute triangles. Obviously one of them can have an arbitrarily small perimeter. 4.9 Longlisted Problems 1967 363 11. We have to find the number pn of triples of positive integers (a, b, c) satisfying a ≤ b ≤ c ≤ n and a + b > c. Let us denote by pn (k) the number of such triples with c = k, k = 1, 2, . . . , n. For k even, pn (k) = k + (k − 2) + (k − 4) + · · · + 2 = 2 + 2k + 1)/4. Hence (k2 + 2k)/4, and for k odd, pn (k) = (k( n(n + 2)(2n + 5)/24, for 2 | n, pn = pn (1) + pn (2) + · · · + pn (n) = (n + 1)(n + 3)(2n + 1)/24, for 2 ∤ n. 12. Let us denote by Mn the set of points of the segment AB obtained from A and B by not more than n iterations of (∗). It can be proved by induction that   3k 3k − 2 Mn = X ∈ AB | AX = n or for some k ∈ N . 4 4n S Thus (a) immediately follows from M = Mn . It also follows that if a, b ∈ N and a/b ∈ M, then 3 | a(b − a). Therefore 1/2 6∈ M. √ 13. The maximum area is 3 3r2 /4 (where r is the radius of the semicircle) and is attained in the case of a trapezoid with two vertices at the endpoints of the diameter of the semicircle and the other two vertices dividing the semicircle into three equal arcs. 14. We have that √ p √ |p − q 2| |p2 − 2q2| 1 √ ≥ √ , − 2 = = q q q(p + q 2) q(p + q 2) (1) because |p2 − 2q2 | ≥ 1. The greatest p, q ≤ 100 satisfying the equation |p2 − 2q2 | = 1√are (p, q) = (99, 70). It is easy to verify using (1) that 99 among the 70 best approximates 2 √ fractions p/q with p, q ≤ 100. The numbers 99 = 1.41428 . . . and 2 coincide 70 √ p −5 up to the fourth decimal digit: indeed, (1) gives 7 · 10 < q − 2 < 8 · 10−5 . Second solution. By using some basic facts about Farey sequences, one can find √ p 99 41 99 that 41 < 2 < 29 70 and that 29 < q < 70 implies p ≥ 41 + 99 > 100 because √ 99 99 · 29 − 41 · 70 = 1. Of the two fractions 41 29 and 70 , the latter is closer to 2. 15. Given that tan α ∈ Q, we have that tan β is rational if and only if tan γ is rational, 2t where γ = β − α and 2γ = α . Putting t = tan γ we obtain qp = tan 2γ = 1−t 2, 2 which leads to the quadratic equation pt + 2qt − p = 0. This equation has rational solutions if and only if its discriminant 4(p2 + q2 ) is a perfect square, and the result follows. 16. First let us notice that all the numbers zm1 ,m2 = m1 r1 + m2 r2 (m1 , m2 ∈ Z) are distinct, since r1 /r2 is irrational. Thus for any n ∈ N the interval [−n(|r1 | + |r2 |), n(|r1 | + |r2 |)] contains (2n + 1)2 numbers zm1 ,m2 , where |m1 |, |m2 | ≤ n. Therefore some two of these (2n + 1)2 numbers, say zm1 ,m2 , zn1 ,n2 , differ by 1 |+|r2 |) 1 |+|r2 |) at most 2n(|r = (|r2(n+1) . By taking n large enough we can achieve that (2n+1)2−1 zq1 ,q2 = |zm1 ,m2 − zn1 ,n2 | ≤ p. If now k is the integer such that kzq1 ,q2 ≤ x < (k + 1)zq1 ,q2 , then zkq1 ,kq2 = kzq1 ,q2 differs from x by at most p, as desired. 364 4 Solutions 17. Using cr − cs = (r − s)(r + s + 1) we can easily get (cm+1 − ck ) · · · (cm+n − ck ) (m − k + n)! (m + k + n + 1)! = · . c1 c2 · · · cn (m − k)!n! (m + k + 1)!(n + 1)!  m−k+n The first factor (m−k+n)! is clearly an integer. The second factor is n (m−k)!n! = also an integer because by the assumption, m + k + 1 and (m + k)!(n + 1)! are coprime, and (m + k + n + 1)! is divisible by both; hence it is also divisible by their product. 18. In the first part, it is sufficient to show that each rational number of the form m/n!, m, n ∈ N, can be written uniquely in the required form. We prove this by induction on n. The statement is trivial for n = 1. Let us assume it holds for n −1, and let there be given a rational number m/n!. Let us take an ∈ {0, . . . , n − 1} such that m − an = nm1 for some m1 ∈ N. By the inductive hypothesis, there are unique a1 ∈ N0 , ai ∈ {0, . . . , i − 1} (i = 1, . . . , n − 1) such that m1 /(n − 1)! = ∑n−1 i=1 ai /i!, and then n m m1 an ai = + =∑ , n! (n − 1)! n! i=1 i! as desired. On the other hand, if m/n! = ∑ni=1 ai /i!, multiplying by n! we see that m − an must be a multiple of n, so the choice of an was unique and therefore the representation itself. This completes the induction. In particular, since ai | i! and i!/ai > (i − 1)! ≥ (i − 1)!/ai−1 , we conclude that each rational q, 0 < q < 1, can be written as the sum of different reciprocals. Now we prove the second part. Let x > 0 be a rational number. For any integer 1 m > 106 , let n > m be the greatest integer such that y = x− m1 − m+1 − · · ·− 1n > 0. Then y can be written as the sum of reciprocals of different positive integers, which all must be greater than n. The result follows immediately. 19. Suppose n ≤ 6. Let us decompose the disk by its radii into n congruent regions, so that one of the points Pj lies on the boundaries of two of these regions. Then one of these regions contains two of the n given points. Since the diameter of each of these regions is 2 sin πn , we have dn ≤ 2 sin πn . This value is attained if Pi are the vertices of a regular n-gon inscribed in the boundary circle. Hence Dn = 2 sin πn . For n = 7 we have D7 ≤ D6 = 1. This value is attained if six of the seven points form a regular hexagon inscribed in the boundary circle and the seventh is at the center. Hence D7 = 1. 20. The statement so formulated is false. It would be true under the additional assumption that the polygonal line is closed. However, from the offered solution, which is not clear, it does not seem that the proposer had this in mind. 21. Using the formula cos x cos 2x cos 4x · · · cos 2n−1 x = simple induction, we obtain cos sin 2n x 2n sin x , which is shown by π 2π 4π 7π π 2π 4π 8π 1 cos cos cos = − cos cos cos cos = , 15 15 15 15 15 15 15 15 16 4.9 Longlisted Problems 1967 365 3π 6π 1 5π 1 cos = , cos = . 15 15 4 15 2 Multiplying these equalities, we get that the required product P equals 1/128. cos 22. Let O1 and O2 be the centers of circles k1 and k2 and let C be the midpoint of the segment AB. Using the well-known relation for elements of a triangle, we obtain PA2 + PB2 = 2PC2 + 2CA2 ≥ 2O1C2 + 2CA2 = 2O1 A2 = 2r2 . Equality holds if P coincides with O1 or if A and B coincide with O2 . 23. Suppose that a ≥ 0, c ≥ 0, 4ac ≥ b2 . If a = 0, then b = 0, and the inequality reduces to the obvious cg2 ≥ 0. Also, if a > 0, then  b a f + b f g + cg = a f + g 2a 2 2 2 + 4ac − b2 2 g ≥ 0. 4a Suppose now that a f 2 + b f g + cg2 ≥ 0 holds for an arbitrary pair of vectors f , g. Substituting f by tg (t ∈ R) we get that (at 2 + bt + c)g2 ≥ 0 holds for any real number t. Therefore a ≥ 0, c ≥ 0, 4ac ≥ b2 . 24. Let m be the total number of coins and suppose that the kth child receive xk coins. By the condition of the problem, the number of coins that remain after him was 6(xk − k). This gives us a recurrence relation xk+1 = k + 1 + 6(xk − k) − k − 1 6 6 = xk + , 7 7 7 which, together with the condition x1 = 1 + (m − 1)/7, yields xk = 6k−1 (m − 36) + 6 for 1 ≤ k ≤ n. 7k Since we are given xn = n, we obtain 6n−1 (m − 36) = 7n (n − 6). It follows that 6n−1 | n − 6, which is possible only for n = 6. Hence, n = 6 and m = 36. √ 25. The answer is R = (4 + 3)d/6. 26. Let L be the midpoint of the edge AB. Since P is the orthocenter of △ABM and ML is its altitude, P lies on ML and therefore belongs to the triangular area LCD. Moreover, from the similarity of triangles ALP and MLB we have LP · LM = LA · LB = a2 /4, where a is the side length of tetrahedron ABCD. It easily follows that the locus of P is the image of the segment CD under the inversion of the plane LCD with center L and radius a/2. This locus is the arc of a circle with center L and endpoints at the orthocenters of triangles ABC and ABD. 366 4 Solutions 27. Regular polygons with 3, 4, and 6 sides can be obtained by cutting a cube with a plane, as shown in the figure. A polygon with more than 6 sides cannot be obtained in such a way, for a cube has 6 faces. Also, if a pentagon is obtained by cutting a cube with a plane, then its sides lying on opposite faces are parallel; hence it cannot be regular. 28. The given expression can be transformed into y= 4 cos 2u + 2 − 3. cos 2u − cos2x It does not depend on x if and only if cos 2u = −1/2, i.e., u = ±π /3 + kπ for some k ∈ Z. 29. Let arc la be the locus of points A lying on the opposite side from A0 with respect to the line B0C0 such that ∠B0 AC0 = ∠A′ . Let ka be the circle containing la , and let Sa be the center of ka . We similarly define lb , lc , kb , kc , Sb , Sc . It is easy to show that circles ka , kb , kc have a common point S inside △ABC. Let A1 , B1 , C1 be the points on the arcs la , lb , lc diametrically opposite to S with respect to Sa , Sb , Sc respectively. Then A0 ∈ B1C1 because ∠B1 A0 S = ∠C1 A0 S = 90◦ ; similarly, B0 ∈ A1C1 and C0 ∈ A1 B1 . Hence the triangle A1 B1C1 is circumscribed about △A0 B0C0 and similar to △A′ B′C′ . Moreover, we claim that △A1 B1C1 is the triangle ABC with the desired properties having the maximum side BC and hence the maximum area. Indeed, if ABC is any other such triangle and Sb′ , Sc′ are the projections of Sb and Sc onto the line BC, it holds that BC = 2Sb′ Sc′ ≤ 2Sb Sc = B1C1 , which proves the maximality of B1C1 . 30. We assume without loss of generality that m ≤ n. Let r and s be the numbers of pairs for which i − j ≥ k and of those for which j − i ≥ k. The desired number is r + s. We easily find that  (m − k)(m − k + 1)/2, k < m, r= 0, k ≥ m,   m(2n − 2k − m + 1)/2, k < n − m, s = (n − k)(n − k + 1)/2, n − m ≤ k < n,  0, k ≥ n. 31. Suppose that n1 ≤ n2 ≤ · · · ≤ nk . If nk < m, there is no solution. Otherwise, the solution is 1 + (m − 1)(k − s + 1) + ∑ ni , i<s where s is the smallest i for which m ≤ ni holds. 4.9 Longlisted Problems 1967 367 32. Let us denote by V the volume of the given body, and by Va , Vb , Vc the volumes of the parts of the given ball that lie inside the dihedra of the given C ′ B′ trihedron. It holds that Va = 2R3 α /3, 3 3 Vb = 2R β /3, Vc = 2R γ /3. It is easy O A A′ to see that 2(Va + Vb + Vc ) = 4V + 4π R3 /3, from which it follows that B C 1 V = R3 (α + β + γ − π ). 3 33. If m 6∈ {−2, 1}, the system has the unique solution x= b + a − (1 + m)c a + c − (1 + m)b b + c − (1 + m)a , y= , z= . (2 + m)(1 − m) (2 + m)(1 − m) (2 + m)(1 − m) The numbers x, y, z form an arithmetic progression if and only if a, b, c do so. For m = 1 the system has a solution if and only if a = b = c, while for m = −2 it has a solution if and only if a + b + c = 0. In both these cases it has infinitely many solutions. 34. Each vertex of the polyhedron is a vertex of exactly two squares and triangles (more than two is not possible; otherwise, the sum of angles at a vertex exceeds 360◦ ). By using the condition that the trihedral angles are equal it is easy to see that such a polyhedron is uniquely determined by its side length. The polyhedron obtained from a cube by “cutting” its vertices, as shown in the figure, satisfies the conditions. Now it is easy to calculate that the ratio of the squares of volumes of that polyhedron and of the ball whose boundary is the circumscribed sphere is equal to 25/(8π 2 ). 35. The given sum can be rewritten as   k n   n n 2 x ∑ k tan 2 + ∑ k k=0 k=0 n 2 tan2 2x 1 − tan2 2x !k . 2 2 tan (x/2) 1−cos x Since 1−tan 2 (x/2) = cos x , the above sum is transformed using the binomial formula into n  n  1 − cosx x 2 x 1 + tan + 1+ = sec2n + secn x. 2 cos x 2 36. Suppose that the skew edges of the tetrahedron ABCD are equal. Let K, L, M, P, Q, R be the midpoints of edges AB, AC, AD, CD, DB, BC respectively. Segments KP, LQ, MR have the common midpoint T . 368 4 Solutions A We claim that the lines KP, LQ and MR are axes of symmetry of the tetraK M hedron ABCD. From LM k CD k RQ and similarly LR k MQ and LM = L T CD/2 = AB/2 = LR it follows that B D LMQR is a rhombus and therefore Q LQ ⊥ MR. We similarly show that KP R P is perpendicular to LQ and MR, and thus it is perpendicular to the plane C LMQR. Since the lines AB and CD are parallel to the plane LMQR, they are perpendicular to KP. Hence the points A and C are symmetric to B and D with respect to the line KP, which means that KP is an axis of symmetry of the tetrahedron ABCD. Similarly, so are the lines LQ and MR. The centers of circumscribed and inscribed spheres of tetrahedron ABCD must lie on every axis of symmetry of the tetrahedron, and hence both must coincide with T . Conversely, suppose that the centers of circumscribed and inscribed spheres of the tetrahedron ABCD coincide with some point T . Then the orthogonal projections of T onto the faces ABC and ABD are the circumcenters O1 and O2 of these two triangles, and moreover, T O1 = T O2 . Pythagoras’s theorem gives AO1 = AO2 , which by the law of sines implies ∠ACB = ∠ADB. Now it easily follows that the sum of the angles at one vertex of the tetrahedron is equal to 180◦ . Let D′ , D′′ , and D′′′ be the points in the plane ABC lying outside △ABC such that △D′ BC ∼ = △DBC, △D′′CA ∼ = △DCA, and △D′′′ AB ∼ = △DAB. The ′′ ′′′ angle D AD is then straight, and hence A, B,C are midpoints of the segments D′′ D′′′ , D′′′ D′ , D′ D′′ respectively. Hence AD = D′′ D′′′ /2 = BC, and analogously AB = CD and AC = BD. 37. Using the A–G mean inequality we obtain 8a2 b3 c3 ≤ 2a8 + 3b8 + 3c8 , 8a3 b2 c3 ≤ 3a8 + 2b8 + 3c8 , 8a3 b3 c2 ≤ 3a8 + 3b8 + 2c8 . By adding these inequalities and dividing by 3a3 b3 c3 we obtain the desired one. 38. Suppose that there exist integers n and m such that m3 = 3n2 + 3n + 7. Then from m3 ≡ 1 (mod 3) it follows that m = 3k + 1 for some k ∈ Z. Substituting into the initial equation we obtain 3k(3k2 + 3k + 1) = n2 + n + 2. It is easy to check that n2 + n + 2 cannot be divisible by 3, and so this equality cannot be true. Therefore our equation has no solutions in integers. 39. Since sin2 A + sin2 B + sin2 C + cos2 A + cos2 B + cos2 C = 3, the given equality is equivalent to cos2 A + cos2 B + cos2 C = 1, which by multiplying by 2 is transformed into 0 = cos 2A + cos2B + 2 cos2 C = 2 cos(A + B) cos(A − B) + 2 cos2 C = 2 cosC(cos(A − B) − cosC). 4.9 Longlisted Problems 1967 369 It follows that either cosC = 0 or cos(A − B) = cosC. In both cases the triangle is right-angled. 40. Suppose CD is the longest edge of the tetrahedron ABCD, AB = a, CK and DL are the altitudes of the triangles ABC and ABD respectively, and DM is the altitude of the tetrahedron ABCD. Then CK 2 ≤ 1 − a2 /4, since CK is a leg of the right triangle whose other leg has length not less than a/2 and whose hypotenuse has length not greater than 1 (AKC or BKC). In the similar way we can show that DL2 ≤ 1 − a2/4. Since DM ≤ DL, then DM 2 ≤ 1 − a2/4. It follows that   1 a  1 a2 1 V = CK DM ≤ a 1 − = a(2 − a)(2 + a) 3 2 6 4 24 1 1 1 = [1 − (a − 1)2](2 + a) ≤ ·1·3 = . 24 24 8 41. It is well known that the points K, L, M, symmetric to H with respect to BC,CA, AB respectively, lie on the circumcircle k of the triangle ABC. For K, this follows from an elementary calculation of angles of triangles HBC and noting that ∡KBC = ∡HBC = ∡KAC. For other points the proof is analogous. Since the lines la , lb pass through K and la A L L and lb is obtained from la by rotaM P tion about C for an angle 2γ = ∠LCK, lc H it follows that the intersection point P lb of la and lb is at the circumcircle of KLC, that is, k. Similarly, lb and lc B C meet at a point on k; hence they must pass through the same point P. K l 42. E = (1 − sin x)(1 − cos x)[3 + 2(sin x + cos x) + 2 sin x cos x + sin x cos x(sin x + cos x)]. 43. We can write the given equation in the form x5 − x3 − 4x2 − 3x − 2 + λ (5x4 + α x2 − 8x + α ) = 0. A root of this equation is independent of λ if and only if it is a common root of the equations x5 − x3 − 4x2 − 3x − 2 = 0 and 5x4 + α x2 − 8x + α = 0. 2 2 The first of these two equations is equivalent √to (x − 2)(x + x + 1) = 0 and has three different roots: x1 = 2, x2,3 = (−1 ± i 3)/2. (a) For α = −64/5, x1 = 2 is the unique root independent of λ . (b) For α = −3 there are two roots independent of λ : x1 = ω and x2 = ω 2 .   44. (a) S(x, n) = n(n − 1) x2 + (n + 1)x + (n + 1)(3n + 2)/12 . 370 4 Solutions (b) It is easy to see that the equation S(x, n) = 0 has two roots   p x1,2 = −(n + 1) ± (n + 1)/3 /2. They are integers if and only if n = 3k2 − 1 for some k ∈ N. 45. (a) Using the formula 4 sin3 x = 3 sin x − sin 3x one can easily reduce the given equation to sin 3x = cos 2x. Its solutions are given by x = (4k + 1)π /10, k ∈ Z. (b) (1) The point B corresponding to the solution x = (4k + 1)π /10 is a vertex of the regular dodecagon if and only if (4k + 1)π /10 = 2mπ /12, i.e., 3(4k +1) = 5m for some m ∈ Z. This is possible if and only if 5 | 4k +1, i.e., k ≡ 1 (mod 5). (2) Similarly, if the point B corresponding to x = (4k + 1)π /10 is a vertex of a polygon P, then (4k + 1)n = 20m for some m ∈ N, which implies that 4 | n. 46. Let us set arctanx = a, arctan y = b, arctan z = c. Then tan(a + b) = x+y+z−xyz tan(a + b + c) = 1−yz−zx−xy = 1, which implies that x+y 1−xy and (x − 1)(y − 1)(z − 1) = xyz − xy − yz − zx + x + y + z − 1 = 0. One of x, y, z is equal to 1, say z = 1, and consequently x + y = 0. Therefore x2n+1 + y2n+1 + z2n+1 = x2n+1 + (−x)2n+1 + 12n+1 = 1. 47. Using the A–G mean inequality we get (n + k − 1)xn1x2 · · · xk ≤ nxn+k−1 + xn+k−1 + · · · + xn+k−1 , 1 2 k n+k−1 n+k−1 n (n + k − 1)x1x2 · · · xk ≤ x1 + nx2 + · · · + xn+k−1 , k ...... ......... (n + k − 1)x1x2 · · · xnk ≤ xn+k−1 + xn+k−1 + · · · + nxn+k−1 . 1 2 k By adding these inequalities and dividing by n + k − 1 we obtain the desired one. Remark. This is also an immediate consequence of Muirhead’s inequality. 48. Put f (x) = x ln x. The given equation is equivalent to f (x) = f (1/2), which has the solutions x1 = 1/2 and x2 = 1/4. Since the function f is decreasing on (0, 1/e), and increasing on (1/e, +∞), this equation has no other solutions. 49. Since sin 1, sin 2, . . . , sin(N + 1) ∈ (−1, 1), two of these N + 1 numbers have distance less than 2/N. Therefore | sin n − sin k| < 2/N for some integers 1 ≤ k, n ≤ N + 1, n 6= k. 50. Since ϕ (x, y, z) = f (x + y, z) = ϕ (0, x + y, z) = g(0, x + y + z), it is enough to put h(t) = g(0,t). 51. If  there  exist two numbers ab, bc ∈ S, then one can fill a crossword puzzle as ab . The converse is obvious. Hence the set S has property A if and only if bc 4.9 Longlisted Problems 1967 371 the set of first digits and the set of second digits of numbers in S are disjoint. Thus the maximum size of S is 25. 52. This problem is not elementary. The solution offered by the proposer was not quite clear and complete (the existence was not proved). 53. (a) We can construct two lines parallel to the rays of the angle, at equal distances from the rays. The intersection of these two lines lies on the bisector of the angle. (b) If the length of a segment AB exceeds the breadth of the ruler, we can construct parallel lines through A and B in two different ways. The diagonal in the resulting rhombus is the perpendicular bisector of the segment AB. If the segment AB is too short, we can construct a line l parallel to AB and centrally project AB onto l from a point C chosen sufficiently close to the segment, thus obtaining an arbitrarily long segment A′ B′ k AB. Then we construct the midpoint D′ of A′ B′ as above. The line D′C intersects the segment AB at its midpoint D. By means of lines parallel to DC the segment AB can be prolonged symmetrically, and then the perpendicular bisector can be found as above. (c) follows immediately from part (b). (d) Let there be given a point P and a line l. We draw an arbitrary line through P that intersects l at A, and two lines l1 and l2 parallel to AP, at equal distances from AP and on either side of AP. Line l1 intersects l at B. We can construct the midpoint C of AP. If BC intersects l2 at D, then PD is parallel to l. 54. Let S be the given set of points on the cube. Let x, y, z denote the numbers of points from S lying at a vertex, at the midpoint of an edge, at the midpoint of a face of the cube, respectively, and let u be the number of all other points from S. Either there are no points from S at the vertices of the cube, or there is a point from S at each vertex. Hence x is either 0 or 8. Similarly, y is either 0 or 12, and z is either 0 or 6. Any other point of S has 24 possible images under rotations of the cube. Hence u is divisible by 24. Since n = x + y + z + u and 6 | y, z, u, it follows that either 6 | n or 6 | n − 8, i.e., n ≡ 0 or n ≡ 2 (mod 6). Thus n = 200 is possible, while n = 100 is not, because n ≡ 4 (mod 6). 55. It is enough to find all x from (0, 2π ] such that the given inequality holds for all integers n. Suppose 0 < x < 2π /3. If n is the maximum integer √ for which nx ≤ 2π /3, we have π /3 < nx√≤ 2π /3, and consequently sin nx ≥ 3/2. Thus sin x + sin 2x + · · · + sin nx > 3/2. Suppose now that 2π /3 ≤ x < 2π . We have √ cos 2x − cos 2n+1 cos 2x + 1 cot 4x 3 2 x sin x + · · · + sin nx = ≤ ≤ . x x = 2 sin 2 2 sin 2 2 2 For x = 2π the given inequality clearly holds for all n. Hence, the inequality holds for all n if and only if 2π /3 + 2kπ ≤ x ≤ 2π + 2kπ for some integer k. 372 4 Solutions 56. We shall prove by induction on n the following statement: If in some group of interpreters exactly n persons, n ≥ 2, speak each of the three languages, then it is possible to select a subgroup in which each language is spoken by exactly two persons. The statement of the problem easily follows from this: it suffices to select six such groups. The case n = 2 is trivial. Let us assume n ≥ 2, and let N j , Nm , N f , N jm , N j f , Nm f , N jm f be the sets of those interpreters who speak only Japanese, only Malay, only Farsi, only Japanese and Malay, only Japanese and Farsi, only Malay and Farsi, and all the three languages, respectively, and n j , nm , n f , n jm , n j f , nm f , n jm f the cardinalities of these sets, respectively. By the condition of the problem, n j + n jm + n j f + n jm f = nm + n jm + nm f + n jm f = n f + n j f + nm f + n jm f = 24, and consequently n j − nm f = nm − n j f = n f − n jm = c. Now if c < 0, then n jm , n j f , nm f > 0, and it is enough to select one interpreter from each of the sets N jm , N j f , Nm f . If c > 0, then n j , nm , n f > 0, and it is enough to select one interpreter from each of the sets N j , Nm , N f and then use the inductive assumption. Also, if c = 0, then w.l.o.g. n j = nm f > 0, and it is enough to select one interpreter from each of the sets N j , Nm f and then use the inductive hypothesis. This completes the induction. 57. Obviously cn > 0 for all even n. Thus cn = 0 is possible only for an odd n. Let us assume a1 ≤ a2 ≤ · · · ≤ a8 : in particular, a1 ≤ 0 ≤ a8 . If |a1 | < |a8 |, then there exists n0 such that for every odd n > n0 , 7|a1 |n < an8 ⇒ an1 + · · · + an7 + an8 > 7an1 + an8 > 0, contradicting the condition that cn = 0 for infinitely many n. Similarly |a1 | > |a8 | is impossible, and we conclude that a1 = −a8 . Continuing in the same manner we can show that a2 = −a7 , a3 = −a6 and a4 = −a5 . Hence cn = 0 for every odd n. 58. The following sequence of equalities and inequalities gives an even stronger estimate than needed. 1 |l(z)| = |Az + B| = |(z + 1)(A + B) + (z − 1)(A − B)| 2 1 = |(z + 1) f (1) + (z − 1) f (−1)| 2 1 ≤ (|z + 1| · | f (1)| + |z − 1| · | f (−1)|) 2 1 1 ≤ (|z + 1| + |z − 1|)M = ρ M. 2 2 59. By the arc AB we shall always mean the positive arc AB. We denote by |AB| the length of arc AB. Let a basic arc be one of the n + 1 arcs into which the circle is partitioned by the points A0 , A1 , . . . , An , where n ∈ N. 4.9 Longlisted Problems 1967 373 Suppose that A p A0 and A0 Aq are the basic arcs with an endpoint at A0 , and that xn , yn are their lengths, respectively. We show by induction on n that for each n the length of a basic arc is equal to xn , yn , or xn + yn . The statement is trivial for n = 1. Assume that it holds for n, and let Ai An+1 , An+1 A j be basic arcs. We shall prove that these two arcs have lengths xn , yn , or xn + yn . If i, j are both strictly positive, then |Ai An+1 | = |Ai−1 An | and |An+1 A j | = |An A j−1 | are equal to xn , yn , or xn + yn by the inductive hypothesis. Let us assume now that i = 0, i.e., that A p An+1 and An+1 A0 are basic arcs. Then |A p An+1 | = |A0 An+1−p| ≥ |A0 Aq | = yn and similarly |An+1 Aq | ≥ xn , but |A p Aq | = xn + yn , from which it follows that |A p An+1 | = |A0 Aq | = yn and consequently n + 1 = p + q. Also, xn+1 = |An+1 A0 | = yn − xn and yn+1 = yn . Now, all basic arcs have lengths yn − xn , xn , yn , xn + yn . A presence of a basic arc of length xn + yn would spoil our inductive step. However, if any basic arc Ak Al has length xn + yn , then we must have l − q = k − p because 2π is irrational, and therefore the arc Ak Al contains either the point Ak−p (if k ≥ p) or the point Ak+q (if k < p), which is impossible; hence, the proof is complete for i = 0. The proof for j = 0 is analogous. This completes the induction. It can be also seen from the above considerations that the basic arcs take only two distinct lengths if and only if n = p + q − 1. If we denote by nk the sequence of n’s for which this holds, and by pk , qk the sequences of the corresponding p, q, we have p1 = q1 = 1 and ( (pk + qk , qk ), if {pk /(2π )} + {qk/(2π )} > 1, (pk+1 , qk+1 ) = (pk , pk + qk ), if {pk /(2π )} + {qk/(2π )} < 1. It is now “easy” to calculate that p19 = p20 = 333, q19 = 377, q20 = 710, and thus n19 = 709 < 1000 < 1042 = n20 . It follows that the lengths of the basic arcs for n = 1000 take exactly three different values. 374 4 Solutions 4.10 Solutions to the Shortlisted Problems of IMO 1968 1. Since the ships are sailing with constant speeds and directions, the second ship is sailing at a constant speed and direction in reference to the first ship. Let A be the constant position of the first ship in this frame. Let B1 , B2 , B3 , and B on line b defining the trajectory of the ship be positions of the second ship with respect to the first ship at 9:00, 9:35, 9:55, and at the moment the two ships were closest. Then we have the following equations for distances (in miles): AB1 = 20, AB2 = 15, AB3 = 13, B1 B2 : B2 B3 = 7 : 4, AB2i = AB2 + BB2i . Since BB1 > BB2 > BB3 , it follows that B(B3 , B, B2 , B1 ) or B(B, B3 , B2 , B1 ). We get a system of three quadratic equations with three unknowns: AB, BB3 and B3 B2 (BB3 being negative if B(B3 , B, B1 , B2 ), positive otherwise). This can be solved by eliminating AB and then BB3 . The unique solution ends up being AB = 12, BB3 = 5, B3 B2 = 4, and consequently, the two ships are closest at 10:20 when they are at a distance of 12 miles. 2. The sides a, b, c of a triangle ABC with ∠ABC = 2∠BAC satisfy b2 = a(a + c) (this statement is the lemma in (SL98-7)). Taking into account the remaining condition that a, b, c are consecutive integers with a < b, we obtain three cases: (i) a = n, b = n+ 1, c = n + 2. We get the equation (n + 1)2 = n(2n +2), giving us (a, b, c) = (1, 2, 3), which is not a valid triangle. (ii) a = n, b = n + 2, c = n + 1. We get (n +2)2 = n(2n + 1) ⇒ (n −4)(n +1) = 0, giving us the triangle (a, b, c) = (4, 6, 5). (iii) a = n + 1, b = n + 2, c = n. We get (n + 2)2 = (n + 1)(2n + 1) ⇒ n2 − n − 3 = 0, which has no positive integer solutions for n. Hence, the only solution is the triangle with sides of lengths 4, 5, and 6. 3. A triangle cannot be formed out of three lengths if and only if one of them is larger than the sum of the other two. Let us assume this is the case for all triplets of edges out of each vertex in a tetrahedron ABCD. Let w.l.o.g. AB be the largest edge of the tetrahedron. Then AB ≥ AC + AD and AB ≥ BC + BD, from which it follows that 2AB ≥ AC + AD + BC + BD. This implies that either AB ≥ AC + BC or AB ≥ AD + BD, contradicting the triangle inequality. Hence the three edges coming out of at least one of the vertices A and B form a triangle. Remark. The proof can be generalized to prove that in a polyhedron with only triangular surfaces there is a vertex such that the edges coming out of this vertex form a triangle. 4. We will prove the equivalence in the two directions separately: (⇒) Suppose {x1 , . . . , xn } is the unique solution of the equation. Since {xn , x1 , x2 , . . . , xn−1 } is also a solution, it follows that x1 = x2 = · · · = xn = x and 4.10 Shortlisted Problems 1968 375 the system of equations reduces to a single equation ax2 + (b − 1)x + c = 0. For the solution for x to be unique the discriminant (b − 1)2 − 4ac of this quadratic equation must be 0. (⇐) Assume (b − 1)2 − 4ac = 0. Adding up the equations, we get n ∑ f (xi ) = 0, i=1 where f (x) = ax2 + (b − 1)x + c. 2 But by the assumed condition, f (x) = a x + b−1 . Hence we must have 2a b−1 f (xi ) = 0 for all i, and xi = − 2a , which is indeed a solution. 5. We have hk = r cos(π /k) for all k ∈ N. Using cos x = 1 − 2 sin2 (x/2) and cosx = 2/(1 + tan2 (x/2))−1 and tan x > x > sin x for all 0 < x < π /2, it suffices to prove     π2 2 (n + 1) 1 − 2 −n −1 > 1 4(n + 1)2 1 + π 2/(4n2 )   1 π2 ⇔ 1 + 2n 1 − − >1 1 + π 2/(4n2 ) 2(n + 1)   π2 1 1 ⇔ 1+ − > 1, 2 2 n + π /(4n) n + 1 where the last inequality holds because π 2 < 4n. It is also apparent that as n tends to infinity the term in parentheses tends to 0, and hence it is not possible to strengthen the bound. This completes the proof. 1 2 n 6. We define f (x) = a1a−x + a2a−x + · · · + ana−x . Let us assume w.l.o.g. a1 < a2 < · · · < an . We note that for all 1 ≤ i < n the function f is continuous in the interval (ai , ai+1 ) and satisfies limx→ai f (x) = −∞ and limx→ai+1 f (x) = ∞. Hence the equation f (x) = n will have a real solution in each of the n−1 intervals (ai , ai+1 ). Remark. In fact, this equation has exactly n solutions, and hence they are all real. Moreover, the solutions are distinct if all ai are of the same sign, since x = 0 is an evident solution. 7. Let ra , rb , rc denote the radii of the exscribed circles corresponding to the sides of lengths a, b, c respectively, and R, p and S denote the circumradius, semiperimeter, and area of the that ra (p − a) = rb (p − b) = pgiven triangle. It is well-known abc rc (p − c) = S = p(p − a)(p − b)(p − c) = . Hence, the desired inequality 4R √ √ 3 3 8 abc reduces to 3 3 2 R, which is by the law of sines equivalent to √ 3 3 sin α + sin β + sin γ ≤ . 2 This inequality immediately follows from Jensen’s inequality, since the sine is concave on [0, π ]. Equality holds if and only if the triangle is equilateral. ra rb rc ≤ p≤ 8. Let G be the point such that BCDG is a parallelogram and let H be the midpoint of AG. Obviously HEFD is also a parallelogram, and thus DH = EF = l. If AD2 + BC2 = m2 is fixed, then from the Stewart theorem we have 376 4 Solutions DH 2 = 2DA2 + 2DG2 − AG2 2m2 − AG2 = , 4 4 which is fixed. Thus G and H are fixed points, and from here the locus of D is a circle with center H and radius l. The locus of B is the segment (GI], where I ∈ ∆ is a point in the positive direction such that AI = a. Finally, the locus of C is a region of the plane consisting of a rectangle sandwiched between two semicircles of radius l −→ −→ centered at points H and H ′ , where H ′ is a point such that IH ′ = GH. 9. We note that Sa = ada /2, Sb = bdb /2, and Sc = cdc /2 are the areas of the triangles MBC, MCA, and MAB respectively. The desired inequality now follows from 1 S2 Sa Sb + Sb Sc + Sc Sa ≤ (Sa + Sb + Sc )2 = . 3 3 Equality holds if and only if Sa = Sb = Sc , which is equivalent to M being the centroid of the triangle. √ 10. (a) Let us set k = a/b > 1. Then a = kb and c = kb, √ and a > c > b. The segments a, b, c form a√triangle if and only if k < k + 1, which holds if and only if 1 < k < 3+2 5 . (b) The√ triangle is right-angled if and only if a2 = b2 + c2 ⇔ k2 = k + 1 ⇔ k√= 1+ 5 1+ 5 2 2 . Also, it is acute-angled if and only if k < k + 1 ⇔ 1 < k < 2 and obtuse-angled if √ 1+ 5 2 <k< √ 3+ 5 2 . 11. Introducing yi = x1i , we transform our equation to 0 = 1 + y1 + (1 + y1)y2 + · · · + (1 + y1) · · · (1 + yn−1 )yn = (1 + y1)(1 + y2) · · · (1 + yn ). The solutions are n-tuples (y1 , . . . , yn ) with yi 6= 0 for all i and y j = −1 for at least one index j. Returning to xi , we conclude that the solutions are all the n-tuples (x1 , . . . , xn ) with xi 6= 0 for all i, and x j = −1 for at least one index j. 12. The given inequality is equivalent to (a + b)m/bm + (a + b)m/am ≥ 2m+1 , which    m can be rewritten as 1 1 1 2 + ≥ . 2 am bm a+b m Since f (x) = 1/x is a convex function for every m ∈ Z, the last inequality immediately follows from Jensen’s inequality ( f (a) + f (b))/2 ≥ f ((a + b)/2). 13. Translating one of the triangles if necessary, we may assume w.l.o.g. that B1 ≡ A1 . We also assume that B2 6≡ A2 and B3 6≡ A3 , since the result is obvious otherwise. There exists a plane π through A1 that is parallel to both A2 B2 and A3 B3 . Let A′2 , A′3 , B′2 , B′3 denote the orthogonal projections of A2 , A3 , B2 , B3 onto π , and let h2 , h3 denote the distances of A2 , B2 and of A3 , B3 from π . By the Pythagorean theorem, A′2 A′3 2 = A2 A23 − (h2 +h3 )2 = B2 B23 − (h2 + h3 )2 = B′2 B′3 2 , and similarly 4.10 Shortlisted Problems 1968 377 A1 A′2 = A1 B′2 and A1 A′3 = A1 B′3 ; hence △A1 A′2 A′3 and △A1 B′2 B′3 are congruent. If these two triangles are equally oriented, then we have finished. Otherwise, they are symmetric with respect to some line a passing through A1 , and consequently the projections of the triangles A1 A2 A3 and A1 B2 B3 onto the plane through a perpendicular to π coincide. 14. Let O, D, E be the circumcenter of △ABC and the midpoints of AB and AC, and given arbitrary X ∈ AB and Y ∈ AC such that BX = CY , let O1 , D1 , E1 be the circumcenter of △AXY and the midpoints of AX and AY , respectively. Since AD = AB/2 and AD1 = AX/2, it follows that DD1 = BX/2 and similarly EE1 = CY /2. Hence O1 is at the same distance BX/2 = CY /2 from the lines OD and OE and lies on the half-line bisector l of ∠DOE. If we let X ,Y vary along the segments AB and AC, we obtain that the locus of O1 is the segment OP, where P ∈ l is a point at distance min(AB, AC)/2 from OD and OE. 15. Set       n+1 n+2 n + 2i f (n) = + + ···+ + . . .. 2 4 2i+1 We prove by induction that f (n) = n. This obviously holds for n = 1. Let us assume that f (n − 1) = n − 1. Define     n + 2i n − 1 + 2i g(i, n) = − . 2i+1 2i+1 We have that f (n) − f (n + 1) = ∑∞ i=0 g(i, n). We also note that g(i, n) = 1 if and only if 2i+1 | n + 2i; otherwise, g(i, n) = 0. The divisibility 2i+1 | n + 2i is equivalent to 2i | n and 2i+1 ∤ n, which for a given n holds for exactly one i ∈ N0 . Thus it follows that f (n) − f (n − 1) = 1 ⇒ f (n) = n. The proof by induction is now complete. Second solution. It is easy to show that [x + 1/2] = [2x] − [x] for x ∈ R. Now f (x) = ([x] − [x/2]) + ([x/2] − [x/4])+ · · · = [x]. Hence, f (n) = n for all n ∈ N. 16. We shall prove the result by induction on k. It trivially holds for k = 0. Assume that the statement is true for some k − 1, and let p(x) be a polynomial of degree k. Let us set p1 (x) = p(x + 1) − p(x). Then p1 (x) is a polynomial of degree k − 1 with leading coefficient ka0 . Also, m | p1 (x) for all x ∈ Z and hence by the inductive assumption m | (k − 1)! · ka0 = k!a0 , which completes  the induction. On the other hand, for any a0 , k and m | k!a0 , p(x) = k!a0 kx is a polynomial with leading coefficient a0 that is divisible by m. 17. Let there be given an equilateral triangle ABC and a point O such that OA = x, OB = y, OC = z. Let X be the point in the plane such that △CX B and △COA are congruent and equally oriented. Then BX = x and the triangle X OC is equilateral, which implies OX = z. Thus we have a triangle OBX with BX = x, BO = y, and OX = z. Conversely, given a triangle OBX with BX = x, BO = y and OX = z it is easy to construct the triangle ABC. 378 4 Solutions 18. The required construction is not feasible. In fact, let us consider the special case ∠BOC = 135◦ , ∠AOC = 120◦ , ∠AOB = 90◦ , where AA′ ∩ BB′ ∩ CC′ = {O}. Denoting OA′ , OB′ , OC′ by a, b, c respectively we obtain the system of equations √ a2 + b2 = a2 + c2 + ac = b2 + c2 + 2bc. Assuming w.l.o.g. c = 1 we easily obtain a3 − a2 − a − 1 = 0, which is an irreducible equation of third degree. By a known theorem, its solution a is not constructible by ruler and compass. 19. We shall denote by dn the shortest curved distance from the initial point to the nth point in the positive direction. The sequence dn goes as follows: 0, 1, 2, 3, 4, 5, 6, 0.72, 1.72, . . . , 5.72, 0.43, 1.43, . . . , 5.43, 0.15 = d19 . Hence the required number of points is 20. 20. Let us denote the points A1 , A2 , . . . , An in such a manner that A1 An is a diameter of the set of given points, and A1 A2 ≤ A1 A3 ≤ · · · ≤ A1 An . Since for each 1 < i < n it holds that A1 Ai < A1 An , we have ∠Ai A1 An < 120◦ and hence ∠Ai A1 An < 60◦ (otherwise, all angles in △A1 Ai An are less than 120◦ ). It follows that for all 1 < i < j ≤ n, ∠Ai A1 A j < 120◦ . Consequently, the angle in the triangle A1 Ai A j that is at least 120◦ must be ∠A1 Ai A j . Moreover, for any 1 < i < j < k ≤ n it holds that ∠Ai A j Ak ≥ ∠A1 A j Ak − ∠A1A j Ai > 120◦ − 60◦ = 60◦ (because ∠A1 A j Ai < 60◦ ); hence ∠Ai A j Ak ≥ 120◦ . This proves that the denotation is correct. Remark. It is easy to show that the diameter is unique. Hence the denotation is also unique. 21. The given conditions are equivalent to y − a0 being divisible by a0 , a0 + a1 , a0 + a2 , . . . , a0 + an, i.e., to y = k[a0 , a0 + a1 , . . . , a0 + an ] + a0 , k ∈ N0 . 22. It can be shown by induction on the number of digits of x that p(x) ≤ x for all x ∈ N. It follows that x2 − 10x − 22 ≤ x, which implies x ≤ 12. Since 0 < x2 − 10x − 22 = (x − 12)(x + 2) + 2, one easily obtains x ≥ 12. Now one can directly check that x = 12 is indeed a solution, and thus the only one. 23. We may assume w.l.o.g. that in all the factors the coefficient of x is 1. Suppose that x + ay + bz is one of the linear factors of p(x, y, z) = x3 + y3 + z3 + mxyz. Then p(x) is 0 at every point (x, y, z) with z = −ax − by. Hence x3 + y3 + (−ax − by)3 + mxy(−ax − by) = (1 − a3 )x3 − (3ab + m)(ax + by)xy + (1 − b3 )y3 ≡ 0. This is obviously equivalent to a3 = b3 = 1 and m = −3ab, from which it follows √ −1+i 3 2 that m ∈ {−3, −3ω , −3ω }, where ω = . Conversely, for each of the 2 three possible values for m there are exactly three possibilities (a, b). Hence −3, −3ω , −3ω 2 are the desired values. 24. If the ith digit is 0, then the result is ( 9! 9k− j (10− j)! , if i > k − j, . k− j−1 9! 9 (9− j)! , otherwise 4.10 Shortlisted Problems 1968 379 If the ith digit is not 0, then the above results are multiplied by 8. 25. The answer is ∑ n p nq n r + 1≤p<q<r≤k      nq np n + n . p q ∑ 2 2 1≤p<q≤k 26. (a) We shall show that the period of f is 2a. From ( f (x + a) − 1/2)2 = f (x) − f (x)2 we obtain   1 f (x) − f (x)2 + f (x + a) − f (x + a)2 = . 4 Subtracting the above relation for x + a in place of x we get f (x) − f (x)2 = f (x+2a)− f (x+ 2a)2 , which implies ( f (x) − 1/2)2 = ( f (x + 2a) − 1/2)2 . Since f (x) ≥ 1/2 holds for all x by the condition of the problem, we conclude that f (x + 2a) = f (x). (b) The following function, as is directly verified, satisfies the conditions:  1/2 if 2n ≤ x < 2n + 1, f (x) = for n = 0, 1, 2, . . . . 1 if 2n + 1 ≤ x < 2n + 2, 380 4 Solutions 4.11 Solutions to the Contest Problems of IMO 1969 1. Set a = 4m4 , where m ∈ N and m > 1. We then have z = n4 + 4m4 = (n2 + 2m2 )2 − (2mn)2 = (n2 + 2m2 + 2mn)(n2 + 2m2 − 2mn). Since n2 + 2m2 − 2mn = (n − m)2 +m2 ≥ m2 > 1, it follows that z must be composite. Thus we have found infinitely many a that satisfy the condition of the problem. 2. Using cos(a + x) = cos a cos x − sin a sin x, we obtain y(x) = A sin x + B cosx where A = − sin a1 − sin a2 /2 − · · · − sin an /2n−1 and B = cos a1 + cos a2 /2 + · · · + cos an /2n−1 . Numbers A and B cannot both be equal to 0, for otherwise y would be identically equal to 0, while on the other hand, we have y(−a1 ) = cos(a1 − a1 ) + cos(a2 − a1 )/2 + · · · + cos(an − a1 )/2n−1 ≥ 1 − 1/2 − · · · − 1/2n−1 = 1/2n−1 > 0. Setting A = C cos φ and B = C sin φ , where C 6= 0 (such C and φ always exist), we get y(x) = C sin(x + φ ). It follows that the zeros of y are of the form x0 ∈ −φ + π Z, from which y(x1 ) = y(x2 ) ⇒ x1 − x2 = mπ immediately follows. 3. We have several cases: 1◦ k = 1. W.l.o.g. let AB = a and the remaining √ segments have length 1. Let M be the midpoint of CD. Then AM = BM√= 3/2 (△CDA√and △CDB are equilateral) and 0 < AB < AM + BM = 3, i.e., 0 < a < 3. It is evident that all values of a within this interval are realizable. 2◦ k = 2. We have two subcases. First, let p AC = AD = a. Let M √ be the midpoint of CD. √We have CD = 1, AM = a2 − 1/4, and BM = 3/2. Then we have 1 − 3/2 = p AB−BM < p √ √ √ AM < AB + BM = 1 + 3/2, which gives us 2 − 3 < a < 2 + 3. Second,√ let AB = CD = a. Let M be the midpoint of CD. From △MAB we get a < 2. p p √ √ √ Thus, from 2 − 3 < 2 <p 2 + 3 it follows that the required con√ dition in this case is 0 < a < 2 + 3. All values for a in this range are realizable. 3◦ k = 3. We show that such a tetrahedron exists for all a. Assume a > 1. Assume AB = AC = AD = a. Varying A along the line perpendicular to the plane the center of △BCD we achieve all values of √ BCD and through √ a > 1/ 3. For a ≤ 1/ 3 we can observe a similar tetrahedron with three edges of length 1/a and three of length 1 and proceed as before. 4◦ k = 4. By observing the similar tetrahedron we this case to k = 2 p reduce √ with length 1/a instead of a. Thus we get√ a > 2 − 3. 5◦ k = 5. We reduce to k = 1 and get a > 1/ 3. 4. Let O be the midpoint of AB, i.e., the center of γ . Let O1 , O2 , and O3 respectively be the centers of γ1 , γ2 , and γ3 and let r1 , r2 , r3 respectively be the radii of γ1 , γ2 and γ3 . Let C1 , C2 , and C3 respectively be the points of tangency of γ1 , γ2 and γ3 with AB. Let D2 and D3 respectively be the points of tangency of γ2 and γ3 with CD. Finally, let G2 and G3 respectively be the points of tangency of γ2 and γ3 with γ . We have B(G2 , O2 , O), G2 O2 = O2 D2 , and 4.11 Contest Problems 1969 381 G2 O = OB. Hence, G2 , D2 , B are collinear. Similarly, G3 , D3 , A are collinear. It follows that AG2 D2 D and BG3 D3 D are cyclic, since ∠AG2 D2 = ∠D2 DA = ∠D3 DB = ∠BG3 D3 = 90◦ . Hence BC22 = BD2 · BG2 = BD · BA = BC2 ⇒ BC2 = BC and hence AC2 = AB − BC. Similarly, AC3 = AC. We thus have AC1 = (AC + AB − BC)/2 = (AC3 + AC2 )/2. Hence, C1 is the midpoint of C2C3 . We also have r2 + r3 = C2C3 = AC + BC − AB = 2r1 , from which it follows that O1 , O2 , O3 are collinear. Second solution. We shall prove the statement for arbitrary points A, B,C on γ . Let us apply the inversion ψ with respect to the circle γ1 . We denote by Xb the image of an object X under ψ . Also, ψ maps lines BC,CA, AB onto circles ab, b b, cb, b respectively. Circles ab, b, cb pass through the center O1 of γ1 and have radii equal to the radius of γb. Let P, Q, R be the centers of ab, b b, cb respectively. The line CD maps onto a circle k through Cb and O1 that is perpendicular to cb. Therefore its center K lies in the intersection of the tangent t to cb and the line b 1 ). Let O be a point such that RO1 KO is a parallelogram PQ (which bisects CO ′ ′ and γ2 , γ3 the circles centered at O tangent to k. It is easy to see that γ2′ and γ3′ are also tangent to cb, since OR and OK have lengths equal to the radii of k and b cb. Hence γ2′ and γ3′ are the images of γ2 and γ3 under ψ . Moreover, since QAOK b B, b b O are and PBOK are parallelograms and Q, P, K are collinear, it follows that A, also collinear. Hence the centers of γ1 , γ2 , γ3 are collinear, lying on the line O1 O, and the statement follows. Third solution. Moreover, the statement holds for an arbitrary point D ∈ BC. Let E, F, G, H be the points of tangency of γ2 with AB,CD and of γ3 with AB,CD, respectively. Let Oi be the center of γi , i = 1, 2, 3. As is shown in the third solution of (SL93-3), EF and GH meet at O1 . Hence the problem of proving the collinearity of O1 , O2 , O3 reduces to the following simple problem: Let D, E, F, G, H be points such that D ∈ EG, F ∈ DH and DE = DF, DG = DH. Let O1 , O2 , O3 be points such that ∠O2 ED = ∠O2 FD = 90◦ , ∠O3 GD = ∠O3 HD = 90◦ , and O1 = EF ∩ GH. Then the points O1 , O2 , and O3 are collinear. Let K2 = DO2 ∩ EF and K3 = DO3 ∩ GH. Then O2 K2 /O2 D = DK3 /DO3 = K2 O1 /DO3 and hence by Thales’ theorem O1 ∈ O2 O3 . 5. We first prove the following lemma. Lemma. If of five points in a plane no three belong to a single line, then there exist four that are the vertices of a convex quadrilateral. Proof. If the convex hull of the five points A, B,C, D, E is a pentagon or a quadrilateral, the statement automatically holds. If the convex hull is a triangle, then w.l.o.g. let △ABC be that triangle and D, E points in its interior. Let the line DE w.l.o.g. intersect [AB] and [AC]. Then B,C, D, E form the desired quadrilateral.  We now observe each quintuplet of points within the set. There are n5 such quintuplets, and for each of them there is at least one quadruplet of points forming a convex quadrilateral. Each quadruplet, however, will be counted 382 4 Solutions  1 n up to n − 4 times. Hence we have found at least n−4 quadruplets. Since 5  n−3 1 n ⇔ (n − 5)(n − 6)(n + 8) ≥ 0, which always holds, it follows n−4 5 ≥ 2  that we have found at least n−3 desired quadruplets of points. 2 √ √ √ √ 6. Define u1 = x1 y1 + z1 , u2 = x2 y2 + z2 , v1 = x1 y1 − z1 , and v2 = x2 y2 − z2 . By expanding both sides of the equation we can easily verify (x1 + x2 )(y1 +y2 )− √ √ (z1 + z2 )2 = (u1 + u2 )(v1 + v2 ) + ( x1 y2 − x2 y1 )2 ≥ (u1 + u2 )(v1 + v2 ). Since xi yi − z2i = ui vi for i = 1, 2, it suffices to prove 8 1 1 ≤ + (u1 + u2)(v1 + v2) u1 v1 u2 v2 ⇔ 8u1u2 v1 v2 ≤ (u1 + u2)(v1 + v2 )(u1 v1 + u2 v2 ). √ √ This follows from the AM–GM inequalities 2 u1 u2 ≤ u1 + u2 , 2 v1 v2 ≤ v1 +v2 √ and 2 u1 v1 u2 v2 ≤ u1 v1 + u2 v2 . Equality holds if and only if x1 y2 = x2 y1 , u1 = u2 and v1 = v2 , i.e. if and only if x1 = x2 , y1 = y2 and z1 = z2 . Second solution. Let us define f (x, y, z) = 1/(xy − z2 ). The problem actually states that   x1 + x2 y1 + y2 z1 + z2 2f , , ≤ f (x1 , y1 , z1 ) + f (x2 , y2 , z2 ), 2 2 2 i.e., that the function f is convex on the set D = {(x, y, z) ∈ R2 | xy − z2 > 0}. It is known that a twice continuously differentiable function f (t1 ,t2 , . . . ,tn ) is convex if its Hessian [ fi′′j ]ni, j=1 is positive definite, or equivalently (by Sylvester’s criterion), if its principal minors Dk = det[ fi′′j ]ki, j=1 , k = 1, 2, . . . , n, are positive. In the case of our f this is directly verified: D1 = 2y2 /(xy − z2 )3 , D2 = 3xy + z2 /(xy − z2 )5 , D3 = 6/(xy − z2)6 are obviously positive. 4.12 Shortlisted Problems 1970 383 4.12 Solutions to the Shortlisted Problems of IMO 1970 1. Denote respectively by R and r the radii of the circumcircle and incircle, by A1 , . . . , An , B1 , . . . , Bn ,the vertices of the 2n-gon and by O its center. Let P′ be the point symmetric to P with respect to O. Then Ai P′ Bi P is a parallelogram, and applying cosine theorem on triangles Ai Bi P and PP′ Bi yields 4R2 = PA2i + PB2i − 2PAi · PBi cos ai 4r2 = PB2i + P′ B2i − 2PBi · P′ Bi cos ∠PBi P′ . Since Ai P′ Bi P is a parallelogram, we have that P′ Bi = PAi and ∠PBi P′ = π − ai . Subtracting the expression for 4r2 from the one for 4R2 yields 4(R2 − r2 ) = −4PAi · PBi cos ai = −8S△AiBi P cot ai , hence we conclude that tan2 ai = 2 4S△A i Bi P (R2 − r2 )2 . (1) Denote by Mi the foot of the perpendicular from P to Ai Bi and let mi = PMi . Then S△Ai Bi P = Rmi . Substituting this into (1) and adding up these relations for i = 1, 2, . . . , n, we obtain ! n n 4R2 2 2 ∑ tan ai = (R2 − r2)2 ∑ mi . i=1 i=1 Note that all the points Mi lie on a circle with diameter OP and form a regular −−→ −−→ −→ n-gon. Denote its center by F. We have that m2i = kPMi k2 = kFMi − FPk2 = −−→ − → −−→ −→ −−→ −→ kFMi 2 k + kFP2 k − 2hFMi , FPi = r2 /2 − 2hFMi , FPi. From this it follows that n n i=1 i=1 n −−→ −→ −−→ −→ ∑ m2i = 2n(r/2)2 − 2 ∑ hFMi , FPi = 2n(r/2)2 − 2h ∑ FMi , FPi = 2n(r/2)2, i=1 → −−→ − because ∑ni=1 FMi = 0 . Thus n 4R2 ∑ tan2 ai = (R2 − r2)2 2n i=1  r 2 2 = 2n (r/R)2 2 (1 − (r/R)2) = 2n π cos2 2n π sin4 2n . Remark. For n = 1 there is no regular 2-gon. However, if we think of a 2-gon as a line segment, the statement will remain true. 2. Suppose that a > b. Consider the polynomial P(X ) = x1 X n−1 + x2 X n−2 + · · · + xn−1 X + xn . We have An = P(a), Bn = P(b), An+1 = x0 an + P(a), and Bn+1 = x0 bn + P(b). The inequality An /An+1 < Bn /Bn+1 becomes P(a)/(x0 an + P(a)) < P(b)/(x0 bn + P(b)), i.e., bn P(a) < an P(b). Since a > b, we have that ai > bi and hence xi an bn−i ≥ xi bn an−i (also, for i ≥ 1 the inequality is strict). Summing up all these inequalities for i = 1, . . . , n we get an P(b) > bn P(a), which completes the proof for a > b. 384 4 Solutions On the other hand, for a < b we analogously obtain the opposite inequality An /An+1 > Bn /Bn+1 , while for a = b we have equality. Thus An /An+1 < Bn /Bn+1 ⇔ a > b. 3. We shall use the following lemma Lemma. If an altitude of a tetrahedron passes through the orthocenter of the opposite side, then each of the other altitudes possesses the same property. Proof. Denote the tetrahedron by SABC and let a = BC, b = CA, c = AB, m = SA, n = SB, p = SC. It is enough to prove that an altitude passes through the orthocenter of the opposite side if and only if a2 + m2 = b2 + n2 = c2 + p2 . Suppose that the foot S′ of the altitude from S is the orthocenter of ABC. Then SS′ ⊥ ABC ⇒ SB2 − SC2 = S′ B2 − S′C2 . But from AS′ ⊥ BC it follows that AB2 − AC2 = S′ B2 − S′C2 . From these two equalities it can be concluded that n2 − p2 = c2 − b2 , or equivalently, n2 + b2 = c2 + p2 . Analogously, a2 + m2 = n2 + b2 , so we have proved the first part of the equivalence. Now suppose that a2 +m2 = b2 +n2 = c2 + p2 . Defining S′ as before, we get n2 − p2 = S′ B2 − S′C2 . From the condition n2 − p2 = c2 − b2 (⇔ b2 + n2 = c2 + p2 ) we conclude that AS′ ⊥ BC. In the same way CS′ ⊥ AB, which proves that S′ is the orthocenter of △ABC. The lemma is thus proven. Now using the lemma it is easy to see that if one of the angles at S is right, than so are the others. Indeed, suppose that ∠ASB = π /2. From the lemma we have that the altitude from C passes through the orthocenter of △ASB, which is S, so CS ⊥ ASB and ∠CSA = ∠CSB = π /2. Therefore m2 + n2 = c2 , n2 + p2 = a2 , and p2 + m2 = b2 , so it follows that m2 + n2 + p2 = (a2 + b2 + c2 )/2. By the inequality between the arithmetic and quadric means, we have that (a2 + b2 + c2 )/2 ≥ 2s2 /3, where s denotes the semiperimeter of △ABC. It remains to be shown that 2s2 /3 ≥ 18r2 . Since 2 S△ABC = sr, this is equivalent to 2s4 /3 ≥ 18SABC = 18s(s − a)(s − b)(s − c) by Heron’s formula. This reduces to s3 ≥ 27(s − a)(s − b)(s − c), which is an obvious consequence of the AM–GM mean inequality. Remark. In the place of the lemma one could prove that the opposite edges of the tetrahedron are mutually perpendicular and proceed in the same way. 4. Suppose that n is such a natural number. If a prime number p divides any of the numbers n, n + 1, . . .,n + 5, then it must divide another one of them, so the only possibilities are p = 2, 3, 5. Moreover, n + 1, n + 2, n + 3, n + 4 have no prime divisors other than 2 and 3 (if some prime number greater than 3 divides one of them, then none of the remaining numbers can have that divisor). Since two of these numbers are odd, they must be powers of 3 (greater than 1). However, there are no two powers of 3 whose difference is 2. Therefore there is no such natural number n. Second solution. Obviously, none of n, n + 1, . . ., n + 5 is divisible by 7; hence they form a reduced system of residues. We deduce that n(n + 1) · · · (n + 5) ≡ 1 · 2 · · ·6 ≡ −1 (mod 7). If {n, . . . , n + 5} can be partitioned into two subsets with 4.12 Shortlisted Problems 1970 385 the same products, both congruent to, say, p modulo 7, then p2 ≡ −1 (mod 7), which is impossible. Remark. Erdős has proved that a set n, n + 1, . . .,n + m of consecutive natural numbers can never be partitioned into two subsets with equal products of elements. 5. Denote respectively by A1 , B1 ,C1 and D1 the points of intersection of the lines AM, BM, CM, and DM with the opposite sides of the tetrahedron. Since −−→ −−→ vol(MBCD) = vol(ABCD)MA1 /AA1 , the relation we have to prove is equivalent to −−→ −−→ −−→ −−→ −→ MA1 −→ MB1 −→ MC1 −−→ MD1 MA · −−→ + MB · −−→ + MC · −−→ + MD · −−→ = 0. (1) BB1 DD1 AA1 CC1 There exist unique real numbers α , β , γ , and δ such that α + β + γ + δ = 1 and for every point O in space −−→ − → −→ −→ −→ OM = α OA + β OB + γ OC + δ OD. (2) −−→ − → −→ − → − → − → −→ − → (This follows easily from OM = OA + AM = OA + kAB + l AC + mAD = AB + −→ − → −→ − → −→ − → k(OB − OA) + l(OC − OA) + m(OD − OA) for some k, l, m ∈ R.) Further, from the condition that A1 belongs to the plane BCD we obtain for every O in space the following equality for some β ′ , γ ′ , δ ′ : −−→ −→ −→ −→ OA1 = β ′ OB + γ ′ OC + δ ′ OD. (3) −−→ −−→ −−→ − → −−→ However, for λ = MA1 /AA1 , OM = λ OA + (1 − λ )OA1 ; hence substituting − → (2) and (3) in this expression and equating coefficients for OA we obtain −−→ −−→ −−→ −−→ −−→ −−→ λ = MA1 /AA1 = α . Analogously, β = MB1 /BB1 , γ = MC1 /CC1 , and δ = −−→ −−→ MD1 /DD1 ; hence (1) follows immediately for O = M. Remark. The statement of the problem actually follows from the fact that M is the center of mass of the system with masses vol(MBCD), vol(MACD), vol(MABD), vol(MABC) at A, B,C, D respectively. Our proof is actually a formal verification of this fact. 6. Let F be the midpoint of B′C′ , A′ the midpoint of BC, and I the intersection point of the line HF and the circle circumscribed about △BHC′ . Denote by M the intersection point of the line AA′ with the circumscribed circle about the triangle ABC. Triangles HB′C′ and ABC are similar. Since ∠C′ IF = ∠ABC = ∠A′ MC, ∠C′ FI = ∠AA′ B = ∠MA′C, 2C′ F = C′ B′ , and 2A′C = CB, it follows that △C′ IB′ ∼ △CMB, hence ∠FIB′ = ∠A′ MB = ∠ACB. Now one concludes that I belongs to the circumscribed circles of △AB′C′ (since ∠C′ IB′ = 180◦ − ∠C′ AB′ ) and △HCB′ . Second Solution. We denote the angles of △ABC by α , β , γ . Evidently △ABC ∼ △HC′ B′ . Within △HC′ B′ there exists a unique point I such that ∠HIB′ = 180◦ − γ , ∠HIC′ = 180◦ − β , and ∠C′ IB′ = 180◦ − α , and all three circles must contain 386 4 Solutions this point. Let HI and B′C′ intersect in F. It remains to show that FB′ = FC′ . From ∠HIB′ + ∠HB′ F = 180◦ we obtain ∠IHB′ = ∠IB′ F. Similarly, ∠IHC′ = ∠IC′ F. Thus circles around △IHC′ and △IHB′ are both tangent to B′C′ , giving us FB′ 2 = FI · FH = FC′ 2 . 7. For a = 5 one can take n = 10, while for a = 6 one takes n = 11. Now assume a 6∈ {5, 6}. If there exists an integer n such that each digit of n(n + 1)/2 is equal to a, then there is an integer k such that n(n + 1)/2 = (10k − 1)a/9. After multiplying both sides of the equation by 72, one obtains 36n2 + 36n = 8a · 10k − 8a, which is equivalent to 9(2n + 1)2 = 8a · 10k − 8a + 9. (1) So 8a · 10k − 8a + 9 is the square of some odd integer. This means that its last digit is 1, 5, or 9. Therefore a ∈ {1, 3, 5, 6, 8}. If a = 3 or a = 8, the number on the RHS of (1) is divisible by 5, but not by 25 (for k ≥ 2), and thus cannot be a square. It remains to check the case a = 1. In that case, (1) becomes 9(2n + 1)2 = 8 ·10k + 1, or equivalently [3(2n +1)−1][3(2n + 1) + 1] = 8 · 10k ⇒ (3n + 1)(3n + 2) = 2 · 10k . Since the factors 3n + 1, 3n + 2 are relatively prime, this implies that one of them is 2k+1 and the other one is 5k . It is directly checked that their difference really equals 1 only for k = 1 and n = 1, which is excluded. Hence, the desired n exists only for a ∈ {5, 6}. 8. Let AC = b, BC = a, AM = x, BM = y,CM = l. Denote by I1 the incenter and by S1 the center of the excircle of ∆ AMC. Suppose that P1 and Q1 are feet of perpendiculars from I1 and S1 , respectively, to the line AC. Then △I1CP1 ∼ △S1CQ1 , hence r1 /ρ1 = CP1 /CQ1 . We have CP1 = (AC + MC − AM)/2 = (b + l − x)/2 and CQ1 = (AC + MC + AM)/2 = (b + l + x)/2. Hence r1 b+l−x = . ρ1 b + l + x We similarly obtain r2 b+l−y r a+b−x−y = and = . ρ2 b + l + y ρ a+b+x+y What we have to prove is now equivalent to (b + l − x)(a + l − y) a + b − x − y = . (b + l + x)(a + l + y) a + b + x + y (1) Multiplying both sides of (1) by (a + l + y)(b + l + x)(a + b + x + y) we obtain an expression that reduces to l 2 x + l 2 y + x2 y + xy2 = b2 y + a2 x. Dividing both sides by c = x + y, we get that (1) is equivalent to l 2 = b2 y/(x + y)+ a2 x/(x + y) − xy, which is exactly Stewart’s theorem for l. This finally proves the desired result. q q 9. Let us set a = ∑ni=1 u2i and b = ∑ni=1 v2i . By Minkowski’s inequality (for p = 2) we have ∑ni=1 (ui + vi )2 ≤ (a + b)2 . Hence the LHS of the desired inequality 4.12 Shortlisted Problems 1970 387 is not greater than 1 + (a + b)2 , while the RHS is equal to 4(1 + a2 )(1 + b2 )/3. Now it is sufficient to prove that 3 + 3(a + b)2 ≤ 4(1 + a2)(1 + b2). The last inequality can be reduced to the trivial 0 ≤ (a − b)2 + (2ab − 1)2. The equality in √ the initial inequality holds if and only if ui /vi = c for some c ∈ R and a = b = 1/ 2. 10. (a) Since an−1 < an , we have   ak−1 1 ak − ak−1 1− √ = 3/2 ak ak ak   √ √ √ 2( ak − ak−1 ) ak 1 1 ≤ = 2 − . √ √ √ ak ak−1 ak−1 ak Summing up all these inequalities for k = 1, 2, . . . , n we obtain   1 1 bn ≤ 2 √ − √ < 2. a0 an (b) Choose a real number q > 1, and let ak = qk , k = 1, 2, . . . . Then we deduce √ (1 − ak−1/ak ) / ak = (1 − 1/q)/qk/2 , and consequently     √ q+1 1 n 1 1 bn = 1 − = 1 − . ∑ qk/2 q k=1 q qn/2 √ Since ( q + 1)/q can be arbitrarily close to 2, one can set q such that √ ( q + 1)/q > b. Then bn ≥ b for all sufficiently large n. Second solution. (a) Note that   n ak−1 1 1 1 − = (ak − ak−1 ) · 3/2 ; √ ∑ ∑ ak ak k=1 a k=1 n bn = k hence bn represents exactly the lower Darboux sum for the R R function f (x) = x−3/2 on the interval [a0 , an ]. Then bn ≤ aa0n x−3/2 dx < 1+∞ x−3/2 dx = 2. R (b) For each b < 2 there exists a number α > 1 such that 1α x−3/2 dx > b + (2 − b)/2. Now, by Darboux’s theorem, there exists a sequence 1 = a0 ≤ a1 ≤ · · · ≤ an = α such that the corresponding Darboux sums are arbitrarily close to the value of the integral. In particular, there is a sequence a0 , . . . , an with bn > b. 11. Let S(x) = (x−x1 )(x−x2 ) · · · (x−xn ). We have x3 −x3i = (x−xi )(ω x−xi )(ω 2 x− xi ), where ω is a primitive third root of 1. Multiplying these equalities for i = 1, . . . , n we obtain 388 4 Solutions T (x3 ) = (x3 − x31 )(x3 − x32 ) · · · (x3 − x3n ) = S(x)S(ω x)S(ω 2 x). Since S(ω x) = P(x3 )+ ω xQ(x3 )+ ω 2 x2 R(x3 ) and S(ω 2 x) = P(x3 )+ ω 2 xQ(x3 )+ ω x2 R(x3 ), the above expression reduces to T (x3 ) = P3 (x3 ) + x3 Q3 (x3 ) + x6 R3 (x3 ) − 3P(x3 )Q(x3 )R(x3 ). Therefore the zeros of the polynomial T (x) = P3 (x) + xQ3 (x) + x2 R3 (x) − 3P(x)Q(x)R(x) are exactly x31 , . . . , x3n . It is easily verified that deg T = deg S = n, and hence T is the desired polynomial. 12. Lemma. Five points are given in the plane such that no three of them are collinear. Then there are at least three triangles with vertices at these points that are not acute-angled. Proof. We consider three cases, according to whether the convex hull of these points is a triangle, quadrilateral, or pentagon. (i) Let a triangle ABC be the convex hull and two other points D and E lie inside the triangle. At least two of the triangles ADB, BDC and CDA have obtuse angles at the point D. Similarly, at least two of the triangles AEB, BEC and CEA are obtuse-angled. Thus there are at least four nonacute-angled triangles. (ii) Suppose that ABCD is the convex hull and that E is a point of its interior. At least one angle of the quadrilateral is not acute, determining one non-acute-angled triangle. Also, the point E lies in the interior of either △ABC or △CDA; hence, as in the previous case, it determines another two obtuse-angled triangles. (iii) It is easy to see that at least two of the angles of the pentagon are not acute. We may assume that these two angles are among the angles corresponding to vertices A, B, and C. Now consider the quadrilateral ACDE. At least one of its angles is not acute. Hence, there are at least three triangles that are not acute-angled. Now weconsider all combinations of 5 points chosen from the given 100. There are 100 such combinations, and for each of them there are at least three non5 acute-angled triangles with vertices in it. On the other hand, vertices of each of  100 97 the triangles are counted 97 times. Hence there are at least 3 / non2 5 2 acute-angled triangles with vertices in the given 100 points. Since the number of all triangles with vertices in the given points is 100 3 , the ratio between the number of acute-angled triangles and the number of all triangles cannot be greater than  3 100 5 1 − 97 100 = 0.7. 2 3 4.13 Shortlisted Problems 1971 389 4.13 Solutions to the Shortlisted Problems of IMO 1971 1. Assuming that a, b, c in (1) exist, let us find what their values should be. Since P2 (x) = x2 − 2, equation (1) for n = 1 becomes (x2 −4)2 = [a(x2 − 2)+ bx+ 2c]2 . Therefore, there are two possibilities for (a, b, c): (1, 0, −1) and (−1, 0, 1). In both cases we must prove that (x2 − 4)[Pn (x)2 − 4] = [Pn+1 (x) − Pn−1 (x)]2 . (2) It suffices to prove (2) for all x in the interval [−2, 2]. In this interval we can set x = 2 cost for some real t. We prove by induction that Pn (x) = 2 cosnt for all n. (3) This is trivial for n = 0, 1. Assume (3) holds for some n − 1 and n. Then Pn+1 (x) = 4 cost cos nt − 2 cos(n − 1)t = 2 cos(n + 1)t by the additive formula for the cosine. This completes the induction. Now (2) reduces to the obviously correct equality 16 sin2 t sin2 nt = (2 cos(n + 1)t − 2 cos(n − 1)t)2 . Second solution. If x is fixed, the linear recurrence relation Pn+1 (x) + Pn−1 (x) = xPn (x) can be solved in the standard way. The characteristic polynomial t 2 − xt + 1 has zeros t1,2 with t1 +t2 = x and t1 t2 = 1; hence, the general Pn (x) has the form at1n + bt2n for some constants a, b. From P0 = 2 and P1 = x we obtain that Pn (x) = t1n + t2n. Plugging in these values and using t1t2 = 1 one easily verifies (2). 2. We will construct such a set Sm of 2m points. Take vectors u1 , . . . , um in a given plane such that |ui | = 1/2 and 0 6= |c1 u1 + c2 u2 + · · · + cn un | = 6 1/2 for any choice of numbers ci equal to 0 or ±1 (where two or more of the numbers ci are nonzero). Such vectors are easily constructed by induction on m: For u1 , . . . , um−1 fixed, there are only finitely many vector values um that violate the upper condition, and we may set um to be any other vector of length 1/2. Let Sm be the set of all points M0 + ε1 u1 + ε2 u2 + · · · + εm um , where M0 is any fixed point in the plane and εi = ±1 for i = 1, . . . , m. Then Sm obviously satisfies the condition of the problem. 3. Let x, y, z be a solution of the given system with x2 + y2 + z2 = α < 10. Then xy + yz + zx = (x + y + z)2 − (x2 + y2 + z2 ) 9 − α = . 2 2 Furthermore, 3xyz = x3 + y3 + z3 − (x + y + z)(x2 + y2 + z2 − xy − yz − zx), which gives us xyz = 3(9 − α )/2 − 4. We now have 390 4 Solutions 35 = x4 + y4 + z4 = (x3 + y3 + z3 )(x + y + z) −(x2 + y2 + z2 )(xy + yz + zx) + xyz(x + y + z) α (9 − α ) 9(9 − α ) = 45 − + − 12. 2 2 The solutions in α are α = 7 and α = 11. Therefore α = 7, xyz = −1, xy + xz + yz = 1, and x5 + y5 + z5 = (x4 + y4 + z4 )(x + y + z) −(x3 + y3 + z3 )(xy + xz + yz) + xyz(x2 + y2 + z2 ) = 35 · 3 − 15 · 1 + 7 · (−1) = 83. 4. In the coordinate system in which the x-axis passes through the centers of the circles and the y-axis is their common tangent, the circles have equations x2 + y2 + 2r1 x = 0, x2 + y2 − 2r2 x = 0. Let p be the desired line with equation y = ax + b. The abscissas of points of intersection of p with both circles satisfy one of (1 + a2)x2 + 2(ab + r1)x + b2 = 0, (1 + a2 )x2 + 2(ab − r2)x + b2 = 0. Let us denote the lengths of the chords and their projections onto the x-axis by d and d1 , respectively. From these equations it follows that d12 = 4(ab + r1)2 4b2 4(ab − r2)2 4b2 − = − . (1 + a2)2 1 + a2 (1 + a2 )2 1 + a2 (1) Consider the point of intersection of p with the y-axis. This point has equal powers with respect to both circles. Hence, if that point divides the segment determined on p by the two circles into two segments of lengths x and y, this power equals x(x + d) = y(y + d), which implies x = y = d/2. Thus each of the equations in (1) has two roots, one of which is thrice the other. This fact gives us (ab + r1 )2 = 4(1 + a2)b2 /3. We can now use (1) to obtain r2 − r1 , 4b2 + a2 b2 = 3[(ab + r1)2 − a2 b2 ] = 3r1 r2 ; 2 14r1 r2 − r12 − r22 4(r2 − r1 )2 2 a2 = , b = ; 16 14r1 r2 − r12 − r22 ab = d12 = (14r1 r2 − r12 − r22 )2 . 36(r1 + r2 )2 Finally, since d 2 = d12 (1 + a2), we conclude that 1 (14r1 r2 − r12 − r22 ), 12 √ and that the problem is solvable if and only if 7 − 4 3 ≤ d2 = r1 r2 √ ≤ 7 + 4 3. 4.13 Shortlisted Problems 1971 391 5. Without loss of generality, we may assume that a ≥ b ≥ c ≥ d ≥ e. Then a − b = −(b − a) ≥ 0, a − c ≥ b − c ≥ 0, a − d ≥ b − d ≥ 0 and a − e ≥ b − e ≥ 0, and hence (a − b)(a − c)(a − d)(a − e) + (b − a)(b − c)(b − d)(b − e) ≥ 0. Analogously, (d − a)(d − b)(d − c)(d − e) + (e − a)(e − b)(e − c)(e − d) ≥ 0. Finally, (c − a)(c − b)(c − d)(c − e) ≥ 0 as a product of two nonnegative numbers, from which the inequality stated in the problem follows. Remark. The problem in an alternative formulation, accepted for the IMO, asked to prove that the analogous inequality (a1 − a2 )(a1 − a2 ) · · · (a1 − an ) + (a2 − a1)(a2 − a3) · · · (a2 − an ) + · · · +(an − a1)(an − a2) · · · (an − an−1) ≥ 0 holds for arbitrary real numbers ai if and only if n = 3 or n = 5. The case n = 3 is analogous to n = 5. For n = 4, a counterexample is a1 = 0, a2 = a3 = a4 = 1, while for n > 5 one can take a1 = a2 = · · · = an−4 = 0, an−3 = an−2 = an−1 = 2, an = 1 as a counterexample. 6. The proof goes by induction on n. For n = 2, the following labeling satisfies the conditions (i)–(iv): C1 = 11,C2 = 12,C3 = 22,C4 = 21. Suppose that n > 2, and that the numeration C1 ,C2 , . . . ,C2n−1 of a regular 2n−1 gon, in cyclical order, satisfies (i)–(iv). Then one can assign to the vertices of a 2n -gon cyclically the following numbers: 1C1 , 1C2 , . . . , 1C2n−1 , 2C2n−1 , . . . , 2C2 , 2C1 . The conditions (i), (ii) obviously hold, while (iii) and (iv) follow from the inductive assumption. 7. (a) Suppose that X ,Y, Z are fixed on segments AB, BC,CD. It is proven in a standard way that if ∠AT X 6= ∠ZT D, then ZT + T X can be reduced. It follows that if there exists a broken line XY ZT X of minimal length, then the following conditions hold: ∠DAB = π − ∠AT X − ∠AX T, ∠ABC = π − ∠BXY − ∠BY X = π − ∠AX T − ∠CY Z, ∠BCD = π − ∠CY Z − ∠CZY, ∠CDA = π − ∠DT Z − ∠DZT = π − ∠AT X − ∠CZY. Thus σ = 0. (b) Now let σ = 0. Let us cut the surface of the tetrahedron along the edges AC, CD, and DB and set it down into a plane. Consider the plane figure S = ACD′ BD′′C′ thus obtained made up of triangles BCD′ , ABC, ABD′′ , and AC′ D′′ , with Z ′ , T ′ , Z ′′ respectively on CD′ , AD′′ ,C′ D′′ (here C′ corresponds to C, etc.). Since ∠C′ D′′ A + ∠D′′ AB + ∠ABC + ∠BCD′ = 0 as an 392 4 Solutions oriented angle (because σ = 0), the lines CD′ and C′ D′′ are parallel and equally oriented; i.e., CD′ D′′C′ is a parallelogram. The broken line XY ZT X has minimal length if and only if Z ′′ , T ′ , X, Y , Z ′ C C′ are collinear (where Z ′ Z ′′ k CC′ ), α ′ ′′ 2 and then this length equals Z Z = A CC′ = 2AC sin(α /2). There is an in′ Z Z ′′ finity of such lines, one for every X line Z ′ Z ′′ parallel to CC′ that meets Y T′ the interiors of all the segments B ′′ ′ ′′ ′ CB, BA, AD . Such Z Z exist. D D′′ ′′ Indeed, the triangles CAB and D AB are acute-angled, and thus the segment AB has a common interior point with the parallelogram CD′ D′′C′ . Therefore the desired result follows. 8. Suppose that a, b, c,t satisfy all the conditions. Then abc 6= 0 and c x1 x2 = , a a x2 x3 = , b b x3 x1 = . c Multiplying these equations, we obtain x21 x22 x23 = 1, and hence x1 x2 x3 = ε = ±1. From the above equalities we get x1 = ε b/a, x2 = ε c/b, x3 = ε a/c. Substituting x1 in the first equation, we get ab2 /a2 + t ε b2/a + c = 0, which gives us b2 (1 + t ε ) = −ac. (1) Analogously, c2 (1 +t ε ) = −ab and a2 (1 +t ε ) = −bc, and therefore (1 + t ε )3 = −1; i.e., 1 + t ε = −1, since it is real. This also implies together with (1) that b2 = ac, c2 = ab, and a2 = bc, and consequently a = b = c. Thus the three equations in the problem are equal, which is impossible. Hence, such a, b, c,t do not exist. 9. We use induction. Since T1 = 0, T2 = 1, T3 = 2, T4 = 3, T5 = 5, T6 = 8, the statement is true for n = 1, 2, 3. Suppose that both formulas from the problem hold for some n ≥ 3. Then     17 n−1 12 n T2n+1 = 1 + T2n + 2n−1 = 2 + 2n−1 = 2 , 7 7     12 n−2 17 n T2n+2 = 1 + T2n−3 + 2n+1 = 2 + 2n+1 = 2 . 7 7 Therefore the formulas hold for n + 1, which completes the proof. 10. We use induction. Suppose that every two of the numbers a1 = 2n1 − 3, a2 = 2n2 − 3, . . . , ak = 2nk − 3, where 2 = n1 < n2 < · · · < nk , are coprime. Then one can construct ak+1 = 2nk+1 − 3 in the following way: 4.13 Shortlisted Problems 1971 393 Set s = a1 a2 . . . ak . Among the numbers 20 , 21 , . . . , 2s , two give the same residue upon division by s, say s | 2α − 2β . Since s is odd, it can be assumed w.l.o.g. that β = 0 (this is actually a direct consequence of Euler’s theorem). Let 2α − 1 = qs, q ∈ N. Since 2α +2 − 3 = 4qs + 1 is then coprime to s, it is enough to take nk+1 = α + 2. We obviously have nk+1 > nk . 11. We use induction. The statement for n = 1 is trivial. Suppose that it holds for n = k and consider n = k + 1. From the given condition, we have k ∑ |a j,1x1 + · · · + a j,kxk + a j,k+1| j=1 +|ak+1,1 x1 + · · · + ak+1,k xk + ak+1,k+1 | ≤ M, k ∑ |a j,1x1 + · · · + a j,kxk − a j,k+1| j=1 +|ak+1,1 x1 + · · · + ak+1,k xk − ak+1,k+1 | ≤ M for each choice of xi = ±1. Since |a + b| + |a − b| ≥ 2|a| for all a, b, we obtain k 2 ∑ |a j1 x1 + · · · + a jk xk | + 2|ak+1,k+1| ≤ 2M, that is, j=1 k ∑ |a j1x1 + · · · + a jk xk | ≤ M − |ak+1,k+1|. j=1 Now by the inductive assumption ∑kj=1 |a j j | ≤ M − |ak+1,k+1 |, which is equivalent to the desired inequality. 12. Let us start with the case A = A′ . If the triangles ABC and A′ B′C′ are oppositely oriented, then they are symmetric with respect to some axis, and the statement is true. Suppose that they are equally oriented. There is a rotation around A by 60◦ that maps ABB′ onto ACC′ . This rotation also maps the midpoint B0 of BB′ onto the midpoint C0 of CC′ , hence the triangle AB0C0 is equilateral. In the general case, when A 6= A′ , let us denote by T the translation that maps A onto A′ . Let X ′ be the image of a point X under the (unique) isometry mapping ABC onto A′ B′C′ , and X ′′ the image of X under T . Furthermore, let X0 , X0′ be the midpoints of segments X X ′ , X ′ X ′′ . Then X0 is the image of X0′ under the translation −(1/2)T . However, since it has already been proven that the triangle A′0 B′0C0′ is equilateral, its image A0 B0C0 under (1/2)T is also equilateral. The statement of the problem is thus proven. 13. Let p be the least of all the sums of elements in one row or column. If p ≥ n/2, then the sum of all elements of the array is s ≥ np ≥ n2 /2. Now suppose that p < n/2. Without loss of generality, one can assume that the sum of elements in the first row is p, and that exactly the first q elements of it are different from zero. Then the sum of elements in the last n − q columns is greater than or equal to (n − p)(n − q). Furthermore, the sum of elements in the first q columns is greater than or equal to pq. This implies that the sum of all elements in the array is 394 4 Solutions 1 1 1 s ≥ (n − p)(n − q) + pq = n2 + (n − 2p)(n − 2q) ≥ n2 , 2 2 2 since n ≥ 2p ≥ 2q. 14. Denote by V the figure made by a circle of radius 1 whose center moves along the broken line. From the condition of the problem, V contains the whole 50 ×50 square, and thus the area S(V ) of V is not less than 2500. Let L be the length of the broken line. We shall show that S(V ) ≤ 2L + π , from which it will follow that L ≥ 1250 − π /2 > 1248. For each segment li = Ai Ai+1 of the broken line, consider the figure Vi obtained by a circle of radius 1 whose center moves along it, and let Vi be obtained by cutting off the circle of radius 1 with center at the starting point of li . The area of Vi is equal to 2Ai Ai+1 . It is clear that the union of all the figures Vi together with a semicircle with center in A1 and a semicircle with center in An contains V completely. Therefore S(V ) ≤ π + 2A1 A2 + 2A2 A3 + · · · + 2An−1 An = π + 2L. This completes the proof. 15. Assume the opposite. Then one can numerate the cards 1 to 99, with a number ni written on the card i, so that n98 6= n99 . Denote by xi the remainder of n1 + n2 + · · · + ni upon division by 100, for i = 1, 2, . . . , 99. All xi must be distinct: Indeed, if xi = x j , i < j, then ni+1 + · · · + n j is divisible by 100, which is impossible. Also, no xi can be equal to 0. Thus, the numbers x1 , x2 , . . . , x99 take exactly the values 1, 2, . . . , 99 in some order. Let x be the remainder of n1 + n2 + · · · + n97 + n99 upon division by 100. It is not zero; hence it must be equal to xk for some k ∈ {1, 2, . . ., 99}. There are three cases: (i) x = xk , k ≤ 97. Then nk+1 + nk+2 + · · · + n97 + n99 is divisible by 100, a contradiction; (ii) x = x98 . Then n98 = n99 , a contradiction; (iii) x = x99 . Then n98 is divisible by 100, a contradiction. Therefore, all the cards contain the same number. 16. Denote by P′ the polyhedron defined as the image of P under the homothety with center at A1 and coefficient of similarity 2. It is easy to see that all Pi , i = 1, . . . , 9, are contained in P′ (indeed, if M ∈ Pk , then 1 −−→ 1 −−→ −−−→′ A1 M = (A1 Ak + A1 M ) 2 2 for some M ′ ∈ P, and the claim follows from the convexity of P). But the volume of P′ is exactly 8 times the volume of P, while the volumes of Pi add up to 9 times that volume. We conclude that not all Pi have disjoint interiors. 17. We use the following obvious consequences of (a + b)2 ≥ 4ab: 1 4 ≥ , (a1 + a2 )(a3 + a4 ) (a1 + a2 + a3 + a4 )2 4.13 Shortlisted Problems 1971 1 4 ≥ . (a1 + a4 )(a2 + a3 ) (a1 + a2 + a3 + a4 )2 Now we have a1 + a3 a 2 + a 4 a3 + a1 a4 + a2 + + + a1 + a2 a 2 + a 3 a3 + a4 a4 + a1 (a1 + a3)(a1 + a2 + a3 + a4 ) (a2 + a4 )(a1 + a2 + a3 + a4 ) = + (a1 + a2 )(a3 + a4 ) (a1 + a4 )(a2 + a3 ) 4(a1 + a3 ) 4(a2 + a4 ) ≥ + = 4. a1 + a2 + a3 + a 4 a1 + a2 + a3 + a4 395 396 4 Solutions 4.14 Solutions to the Shortlisted Problems of IMO 1972 1. Suppose that f (x0 ) 6= 0 and for a given y define the sequence xk by the formula  xk + y, if | f (xk + y)| ≥ | f (xk − y)|; xk+1 = xk − y, otherwise. It follows from (1) that | f (xk+1 )| ≥ |ϕ (y)|| f (xk )|; hence by induction, | f (xk )| ≥ |ϕ (y)|k | f (x0 )|. Since | f (xk )| ≤ 1 for all k, we obtain |ϕ (y)| ≤ 1. Second solution. Let M = sup | f (x)| ≤ 1, and xk any sequence, possibly constant, such that | f (xk )| → M, k → ∞. Then for all k, |ϕ (y)| = | f (xk + y) + f (xk − y)| 2M ≤ → 1, 2| f (xk )| 2| f (xk )| k → ∞. 2. We use induction. For n = 1 the assertion is obvious. Assume that it is true for a positive integer n. Let A1 , A2 , . . . , A3n+3 be given 3n + 3 points, and let w.l.o.g. A1 A2 . . . Am be their convex hull. Among all the points Ai distinct from A1 , A2 , we choose the one, say Ak , for which the angle ∠Ak A1 A2 is minimal (this point is uniquely determined, since no three points are collinear). The line A1 Ak separates the plane into two halfplanes, one of which contains A2 only, and the other one all the remaining 3n points. By the inductive hypothesis, one can construct n disjoint triangles with vertices in these 3n points. Together with the triangle A1 A2 Ak , they form the required system of disjoint triangles. 3. We have for each k = 1, 2, . . . , n that m ≤ xk ≤ M, which gives (M −xk )(m−xk ) ≤ 0. It follows directly that 0≥ But ∑nk=1 xk n n n k=1 k=1 k=1 ∑ (M − xk)(m − xk ) = nmM − (m + M) ∑ xk + ∑ x2k . = 0, implying the required inequality. 4. Choose in E a half-line s beginning at a point O. For every α in the interval [0, 180◦ ], denote by s(α ) the line obtained by rotation of s about O by α , and by g(α ) the oriented line containing s(α ) on which s(α ) defines the positive direction. For each P in Mi , i = 1, 2, let P(α ) be the foot of the perpendicular from P to g(α ), and lP (α ) the oriented (positive, negative or zero) distance of P(α ) from O. Then for i = 1, 2 one can arrange the lP (α ) (P ∈ Mi ) in ascending order, as l1 (α ), l2 (α ), . . . , l2ni (α ). Call Ji (α ) the interval [lni (α ), lni +1 (α )]. It is easy to see that any line perpendicular to g(α ) and passing through the point with the distance l in the interior of Ji (α ) from O, will divide the set Mi into two subsets of equal cardinality. Therefore it remains to show that for some α , the interiors of intervals J1 (α ) and J2 (α ) have a common point. If this holds for α = 0, then we have finished. Suppose w.l.o.g. that J1 (0) lies on g(0) to the left of J2 (0); then J1 (180◦ ) lies to 4.14 Shortlisted Problems 1972 397 the right of J2 (180◦ ). Note that J1 and J2 cannot simultaneously degenerate to a point (otherwise, we would have four collinear points in M1 ∪ M2 ); also, each of them degenerates to a point for only finitely many values of α . Since J1 (α ) and J2 (α ) move continuously, there exists a subinterval I of [0, 180◦] on which they are not disjoint. Thus, at some point of I, they are both nondegenerate and have a common interior point, as desired. 5. Lemma. If X ,Y, Z, T are points in space, then the lines X Z and Y T are perpendicular if and only if XY 2 + ZT 2 = Y Z 2 + T X 2 . Proof. Consider the plane π through X Z parallel to Y T . If Y ′ , T ′ are the feet of the perpendiculars to π from Y, T respectively, then and XY 2 + ZT 2 = XY ′ 2 + ZT ′ 2 + 2YY ′ 2 , Y Z 2 + T X 2 = Y ′ Z 2 + T ′ X 2 + 2YY ′ 2 . Since by the Pythagorean theorem XY ′ 2 + ZT ′ 2 = Y ′ Z 2 + T ′ X 2 , i.e., XY ′ 2 − Y ′ Z 2 = XT ′ 2 − T ′ Z 2 , if and only if Y ′ T ′ ⊥ X Z, the statement follows. Assume that the four altitudes intersect in a point P. Then we have DP ⊥ ABC ⇒ DP ⊥ AB and CP ⊥ ABD ⇒ CP ⊥ AB, which implies that CDP ⊥ AB, and CD ⊥ AB. By the lemma, AC2 + BD2 = AD2 + BC2 . Using the same procedure we obtain the relation AD2 + BC2 = AB2 + CD2 . Conversely, assume that AB2 + CD2 = AC2 + BD2 = AD2 + BC2 . The lemma implies that AB ⊥ CD, AC ⊥ BD, AD ⊥ BC. Let π be the plane containing CD that is perpendicular to AB, and let hD be the altitude from D to ABC. Since π ⊥ AB, we have π ⊥ ABC ⇒ hD ⊂ π and π ⊥ ABD ⇒ hC ⊂ π . The altitudes hD and hC are not parallel; thus they have an intersection point PCD . Analogously, hB ∩ hC = {PBC } and hB ∩ hD = {PBD }, where both these points belong to π . On the other hand, hB doesn’t belong to π ; otherwise, it would be perpendicular to both ACD and AB ⊂ π , i.e. AB ⊂ ACD, which is impossible. Hence, hB can have at most one common point with π , implying PBD = PCD . Analogously, PAB = PBD = PCD = PABCD . 6. Let n = 2α 5β m, where α = 0 or β = 0. These two cases are analogous, and we treat only α = 0, n = 5β m. The case m = 1 is settled by the following lemma. Lemma. For any integer β ≥ 1 there exists a multiple Mβ of 5β with β digits in decimal expansion, all different from 0. Proof. For β = 1, M1 = 5 works. Assume that the lemma is true for β = k. There is a positive integer Ck ≤ 5 such that Ck 2k + mk ≡ 0 (mod 5), where 5k mk = Mk , i.e. Ck 10k + Mk ≡ 0 (mod 5k+1 ). Then Mk+1 = Ck 10k + Mk satisfies the conditions, and proves the lemma. In the general case, consider, the sequence 1, 10β , 102β , . . . . It contains two numbers congruent modulo (10β − 1)m, and therefore for some k > 0, 10kβ ≡ 1 (mod (10β − 1)m) (this is in fact a consequence of Fermat’s theorem). The number 10kβ − 1 Mβ = 10(k−1)β Mβ + 10(k−2)β Mβ + · · · + Mβ 10β − 1 398 4 Solutions is a multiple of n = 5β m with the required property. 7. P1 R (a) Consider the circumscribing cube OQ1 PR1 O1 QP1 R (that is, the cube O1 Q in which the edges of the tetrahedron are √small diagonals), of side b = a 2/2. The left-hand side is the sum of squares of the projections of the edges of the P R1 tetrahedron onto a perpendicular O Q1 l to π . On the other hand, if l forms angles ϕ1 , ϕ2 , ϕ3 with OO1 , OQ1 , OR1 respectively, then the projections of OP and QR onto l have lengths b(cos ϕ2 + cos ϕ3 ) and b| cos ϕ2 − cos ϕ3 |. Summing up all these expressions, we obtain 4b2 (cos2 ϕ1 + cos2 ϕ2 + cos2 ϕ3 ) = 4b2 = 2a2 . (b) We construct a required tetrahedron of edge length a given in (a). Take O arbitrarily on π0 , and let p, q, r be the distances of O from π1 , π2 , π3 . Since a > p, q, r, |p − q|, we can choose P on π1 anywhere at distance a from O, and Q at one of the two points on π2 at distance a from both O and P. Consider the fourth vertex of the tetrahedron: its distance from π0 will satisfy the equation from (a); i.e., there are two values for this distance; clearly, one of them is r, putting R on π3 . 8. Let f (m, n) = (2m)!(2n)! m!n!(m+n)! . Then it is directly shown that f (m, n) = 4 f (m, n − 1) − f (m + 1, n − 1), and thus n may be successively reduced until one obtains f (m, n) = ∑r cr f (r, 0). Now f (r, 0) is a simple binomial coefficient, and the cr ’s are integers. Second solution. For each prime p, the greatest exponents of p that divide the numerator (2m)!(2n)! and denominator m!n!(m + n)! are respectively           2m 2n m n m+n and ; ∑ p k + pk ∑ pk + pk + pk k>0 k>0 hence it suffices to show that the first exponent is not less than the second one for every p. This follows from the fact that for each real x, [2x] + [2y] ≥ [x] + [y] + [x + y], which is straightforward to prove (for example, using [2x] = [x] + [x + 1/2]). 9. Clearly x1 = x2 = x3 = x4 = x5 is a solution. We shall show that this describes all solutions. Suppose that not all xi are equal. Then among x3 , x5 , x2 , x4 , x1 two consecutive are distinct: Assume w.l.o.g. that x3 6= x5 . Moreover, since (1/x1 , . . . , 1/x5 ) is a solution whenever (x1 , . . . , x5 ) is, we may assume that x3 < x5 . 4.14 Shortlisted Problems 1972 399 √ Consider first the case x1 ≤ x2 . We infer from (i) that x1 ≤ x3 x5 < x5 and x2 ≥ √ x3 x5 > x3 . Then x25 > x1 x3 , which together with (iv) gives x24 ≤ x1 x3 < x3 x5 ; but we also have x23 ≤ x5 x2 ; hence by (iii), x24 ≥ x5 x2 > x5 x3 , a contradiction. √ Consider next the case x1 > x2 . We infer from (i) that x1 ≥ x3 x5 > x3 and √ x2 ≤ x3 x5 < x5 . Then by (ii) and (v), x1 x4 ≤ max(x22 , x23 ) ≤ x3 x5 and x2 x4 ≥ min(x21 , x25 ) ≥ x3 x5 , which contradicts the assumption x1 > x2 . Second solution. 0 ≥ L1 = (x21 − x3 x5 )(x22 − x3 x5 ) = x21 x22 + x23 x25 − (x21 + x22 )x3 x5 1 ≥ x21 x22 + x23 x25 − (x21 x23 + x21 x25 + x22 x23 + x22 x25 ), 2 and analogously for L2 , . . . , L5 . Therefore L1 + L2 + L3 + L4 + L5 ≥ 0, with the only case of equality x1 = x2 = x3 = x4 = x5 . 10. Consider first a triangle. It can be decomposed into k = 3 cyclic quadrilaterals by perpendiculars from some interior point of it to the sides; also, it can be decomposed into a cyclic quadrilateral and a triangle, and it follows by induction that this decomposition is possible for every k. Since every triangle can be cut into two triangles, the required decomposition is possible for each n ≥ 6. It remains to treat the cases n = 4 and n = 5. n = 4. If the center O of the circumcircle is inside a cyclic quadrilateral ABCD, then the required decomposition is effected by perpendiculars from O to the four sides. Otherwise, let C and D be the vertices of the obtuse angles of the quadrilateral. Draw the perpendiculars at C and D to the lines BC and AD respectively, and choose points P and Q on them such that PQ k AB. Then the required decomposition is effected by CP, PQ, QD and the perpendiculars from P and Q to AB. n = 5. If ABCD is an isosceles trapezoid with AB k CD and AD = BC, then it is trivially decomposed by lines parallel to AB. Otherwise, ABCD can be decomposed into a cyclic quadrilateral and a trapezoid; this trapezoid can be cut into an isosceles trapezoid and a triangle, which can further be cut into three cyclic quadrilaterals and an isosceles trapezoid. Remark. It can be shown that the assertion is not true for n = 2 and n = 3. 11. Let ∠A = 2x, ∠B = 2y, ∠C = 2z. (a) Denote by Mi the center of Ki , i = 1, 2, . . . . If N1 , N2 are the projections of p M1 , M2 onto AB, we have AN1 = r1 cotx, N2 B = r2 cot y, and √ N1 N2 = (r1 + r2 )2 − (r1 − r2 )2 = 2 r1 r2 . The required relation between r1 , r2 follows from AB = AN1 + N1 N2 + N2 B. √ If this relation is further considered as a quadratic equation in r2 , then its discriminant, which equals ∆ = 4 (r(cot x + coty) cot y − r1 (cotx cot y − 1)), 400 4 Solutions must be nonnegative, and therefore r1 ≤ r cot y cot z. Then t1 ,t2 , . . . exist, and we can assume that ti ∈ [0, π /2]. (b) Substituting r1 = r cot y cotz sin2 t1 , r2 = r cot z cot x sin2 t2 in the relation of (a) we obtain that sin2 t1 + sin2 t2 + k2 + 2k sint1 sint2 = 1, where we set k = √ tan x tan y. It follows that (k + sint1 sin t2 )2 = (1 − sin2 t1 )(1 − sin2 t2 ) = cos2 t1 cos2 t2 , and hence √ cos(t1 + t2 ) = cost1 cost2 − sint1 sint2 = k = tan x tan y, which is constant. Writing the analogous relations for each ti ,ti+1 we conclude that t1 + t2 = t4 + t5 , t2 + t3 = t5 + t6 , and t3 + t4 = t6 + t7 . It follows that t1 = t7 , i.e., K1 = K7 . 12. First we observe that it is not essential to require the subsets to be disjoint (if they aren’t, one simply excludes their intersection). There are 210 − 1 = 1023 different subsets and at most 990 different sums. By the pigeonhole principle there are two different subsets with equal sums. 4.15 Shortlisted Problems 1973 401 4.15 Solutions to the Shortlisted Problems of IMO 1973 AP2 BP2 AP·PA1 + BP·PB1 to R2 − OP2 , i.e., 1. The condition of the point P can be written in the form CP2 DP2 CP·PC1 + DP·PD1 = 4. All the four denominators are equal the power of P with respect to S. Thus the condition becomes AP2 + BP2 + CP2 + DP2 = 4(R2 − OP2). + to (1) Let M and N be the midpoints of segments AB and CD respectively, and G the midpoint of MN, or the centroid of ABCD. By Stewart’s formula, an arbitrary point P satisfies 1 1 AP2 + BP2 + CP2 + DP2 = 2MP2 + 2NP2 + AB2 + CD2 2 2 1 2 2 2 = 4GP + MN + (AB +CD2 ). 2 Particularly, for P ≡ O we get 4R2 = 4OG2 + MN 2 + 12 (AB2 + CD2 ), and the above equality becomes AP2 + BP2 + CP2 + DP2 = 4GP2 + 4R2 − 4OG2 . Therefore (1) is equivalent to OG2 = OP2 + GP2 ⇔ ∠OPG = 90◦ . Hence the locus of points P is the sphere with diameter OG. Now the converse is easy. 2. Let D′ be the reflection of D across A. Since BCAD′ is then a parallelogram, the condition BD ≥ AC is equivalent to BD ≥ BD′ , which is in turn equivalent to ∠BAD ≥ ∠BAD′ , i.e. to ∠BAD ≥ 90◦ . Thus the needed locus is actually the locus of points A for which there exist points B, D inside K with ∠BAD = 90◦ . Such points B, D exist if and only if the two tangents from A to K, say AP and AQ, determine an obtuse angle. Then √ if P, Q ∈ K, we have ∠PAO = ∠QAO = OP ϕ > 45◦ ; hence OA = sin < OP 2. Therefore the locus of A is the interior of ϕ √ ′ the circle K with center O and radius 2 times the radius of K. 3. We use induction on odd numbers n. For n = 1 there is nothing to prove. Suppose that the result holds for n − 2 vectors, and let us be given vectors v1 , v2 , . . . , vn arranged clockwise. Set v′ = v2 + v3 + · · · + vn−1 , u = v1 + vn , and v = v1 + v2 + · · · + vn = v′ + u. By the inductive hypothesis we have |v′ | ≥ 1. Now if the angles between v′ and the vectors v1 , vn are α and β respectively, then the angle between u and v′ is |α − β |/2 ≤ 90◦ . Hence |v′ + u| ≥ |v′ | ≥ 1. Second solution. Again by induction, it can be easily shown that all possible values of the sum v = v1 + v2 + · · · + vn , for n vectors v1 , . . . , vn in the upper halfplane (with y ≥ 0), are those for which |v| ≤ n and |v − ke| ≥ 1 for every integer k for which n − k is odd, where e is the unit vector on the x axis. 4. Each of the subsets must be of the form {a2 , ab, ac, ad} or {a2 , ab, ac, bc}. It is now easy to count up the partitions. The result is 26460. 402 4 Solutions 5. Let O be the vertex of the trihedron, Z the center of a circle k inscribed in the trihedron, and A, B,C points in which the plane of the circle meets the edges of the trihedron. We claim that the distance OZ is constant. Set OA = x, OB = y, OC = z, BC = a, CA = b, AB = c, and let S and r = 1 be the area and inradius of △ABC. Since Z is the incenter of ABC, we have −→ − → −→ −→ (a + b + c)OZ = aOA + bOB + cOC. Hence − → −→ −→ (a + b + c)2OZ 2 = (aOA + bOB + cOC)2 = a2 x2 + b2y2 + c2 z2 . But since y2 + z2 = a2 , z2 + x2 = b2 and x2 + y2 = c2 , we obtain x2 = y2 = a2 −b2 +c2 2 ,z 2 = a2 +b2 −c2 . 2 (1) −a2 +b2 +c2 , 2 Substituting these values in (1) yields 2a2 b2 + 2b2c2 + 2c2 a2 − a4 − b4 − c4 2 = 8S2 = 2(a + b + c)2r2 . (a + b + c)2OZ 2 = √ √ Hence OZ = r 2 = 2, and Z belongs to a sphere σ with center O and radius √ 2. Moreover, the distances of Z from the faces of the trihedron do not exceed 1; hence Z belongs to a part of σ that lies inside the unit cube with three faces lying on the faces of the trihedron. It is easy to see that this part of σ is exactly the required locus. 6. Yes. Take for M the set of vertices of a cube ABCDEFGH and two points I, J symmetric to the center O of the cube with respect to the laterals ABCD and EFGH. Remark. We prove a stronger result: Given an arbitrary finite set of points S , then there is a finite set M ⊃ S with the described property. Choose a point A ∈ S and any point O such that AO k BC for some two points B,C ∈ S . Now let X ′ be the point symmetric to X with respect to O, and S ′ = {X , X ′ | X ∈ S}. Finally, take M = {X , X | X ∈ S′ }, where X denotes the point symmetric to X with respect to A. This M has the desired property: If X ,Y ∈ M and Y 6= X, then XY k XY ; otherwise, X X, i.e., X A is parallel to X ′ A′ if X 6= A′ , or to BC otherwise. 7. The result follows immediately from Ptolemy’s inequality. 8. Let fn be the required total number, and let fn (k) denote the number of sequences a1 , . . . , an of nonnegative integers such that a1 = 0, an = k, and |ai − ai+1 | = 1 for i = 1, . . . , n − 1. In particular, f1 (0) = 1 and fn (k) = 0 if k < 0 or k ≥ n. Since an−1 is either k − 1 or k + 1, we have fn (k) = fn−1 (k + 1) + f n−1(k − 1) for k ≥ 1. By successive application of (1) we obtain  r    r r fn (k) = ∑ − fn−r (k + r − 2i). i i−k−1 i=0 (1) (2) 4.15 Shortlisted Problems 1973 403 This can be verified by direct induction. Substituting r = n − 1 in (2), we get at most one nonzero summand, namely the one for which i = k+n−1 2 . Therefore   n−1 n−1 fn (n −1 − 2 j) = j − j−1 . Adding up these equalities for j = 0, 1, . . . , n−1 2  we obtain fn = [n−1 n−1 ] , as required. 2 −→ 9. Let a, b, c be vectors going along Ox, Oy, Oz, respectively, such that OG = a + − → −→ −→ b + c. Now let A ∈ Ox, B ∈ Oy, C ∈ Oz and let OA = α a, OB = β b, OC = γ c, where α , β , γ > 0. Point G belongs to a plane ABC with A ∈ Ox, B ∈ Oy, C ∈ Oz if and only if there exist positive real numbers λ , µ , ν with sum 1 such that − → −→ −→ −→ λ OA + µ OB + ν OC = OG, which is equivalent to λ α = µβ = νγ = 1. Such λ , µ , ν exist if and only if α,β , γ > 0 and 1 1 1 + + = 1. α β γ Since the volume of OABC is proportional to the product αβ γ , it is minimized when α1 · β1 · 1γ is maximized, which occurs when α = β = γ = 3 and G is the centroid of △ABC. 10. Let bk = a1 qk−1 + · · · + ak−1 q + ak + ak+1 q + · · · + an qn−k , k = 1, 2, . . . , n. We show that these numbers satisfy the required conditions. Obviously bk > ak . Further, bk+1 − qbk = −[(q2 − 1)ak+1 + · · · + qn−k−1 (q2 − 1)an] > 0 ; we analogously obtain qbk+1 − bk < 0. Finally, b1 + b2 + · · · + bn = a1 (qn−1 + · · · + q + 1) + . . . +ak (qn−k + · · · + q + 1 + q + · · ·+ qk−1 ) + . . . ≤ (a1 + a2 + · · · + an )(1 + 2q + 2q2 + · · · + 2qn−1) 1+q < (a1 + · · · + an). 1−q 11. Putting x + 1x = t we also get x2 + x12 = t 2 − 2, and the given equation reduces √ 2 to t 2 + at + b − 2 = 0. Since x = t± 2t −4 , x will be real if and only if |t| ≥ 2, t ∈ R. Thus we need the minimum value of a2 + b2 under the condition at + b = −(t 2 − 2), |t| ≥ 2. However, by the Cauchy–Schwarz inequality we have (a2 + b2 )(t 2 + 1) ≥ (at + b)2 = (t 2 − 2)2. 2 2 It follows that a2 + b2 ≥ h(t) = (tt 2−2) . Since h(t) = (t 2 + 1) + t 2 9+1 − 6 is in+1 creasing for t ≥ 2, we conclude that a2 + b2 ≥ h(2) = 45 . 404 4 Solutions The cases of equality are easy to examine: These are a = ± 45 and b = − 25 . Second solution. In fact, there was no need for considering x = t + 1/t. By the Cauchy–Schwarz inequality we have (a2 + 2b2 + a2 )(x6 + x4 /2 + x2 ) ≥ (ax3 + bx2 + ax)2 = (x4 + 1)2 . Hence a2 + b 2 ≥ (x4 + 1)2 2x6 + x4 + 2x2 4 ≥ , 5 with equality for x = 1. 12. Observe that the absolute values of the determinants of the given matrices are invariant under all the admitted operations. The statement follows from det A = 16 6= detB = 0. 13. Let S1 , S2 , S3 , S4 denote the areas of the faces of the tetrahedron, V its volume, h1 , h2 , h3 , h4 its altitudes, and r the radius of its inscribed sphere. Since 3V = S1 h1 = S2 h2 = S3 h3 = S4 h4 = (S1 + S2 + S3 + S4 )r, it follows that 1 1 1 1 1 + + + = . h 1 h2 h3 h 4 r In our case, h1 , h2 , h3 , h4 ≥ 1, hence r ≥ 1/4. On the other hand, it is clear that a sphere of radius greater than 1/4 cannot be inscribed in a tetrahedron all of whose altitudes have length equal to 1. Thus the answer is 1/4. 14. Suppose that the soldier starts at the C vertex A of the equilateral triangle ABC of side length a. Let ϕ , ψ be the arcs of ψ E F circles with centers B and C and radii N √ ϕ a 3/4 respectively, that lie inside the triangle. In order to check the vertices D B,C, he must visit some points D ∈ ϕ B A M and E ∈ ψ . Thus his path cannot be shorter than the path ADE (or AED) itself. √ The length of the path ADE is AD + DE ≥ AD + DC − a 3/4. Let F be the reflection of C across the line MN, where M, N are the midpoints of AB and BC. Then DC ≥ DF and hence AD + DC ≥ AD + DF ≥ AF. Consequently √ √ √ ! 3 7 3 AD + DE ≥ AF − a =a − , 4 2 4 with equality if and only if D is the midpoint of arc ϕ and E = (CD) ∩ ψ . Moreover, it is easy to verify that, in following the path ADE, the soldier will check the whole region. Therefore this path (as well as the one symmetric to it) is the shortest possible path that the soldier can take in order to check the entire field. 4.15 Shortlisted Problems 1973 405 π π 15. If z = cos θ + i sin θ , then z − z−1 = 2i sin θ . Now put z = cos 2n+1 + i sin 2n+1 . Using de Moivre’s formula we transform the required equality into n √ A = ∏ (zk − z−k ) = in 2n + 1. (1) k=1 On the other hand, the complex numbers z2k (k = −n, −n + 1, . . .,n) are the roots of x2n+1 − 1, and hence n ∏ (x − z2k)(x − z−2k) = k=1 x2n+1 − 1 = x2n + · · · + x + 1. x−1 (2) Now we go back to proving (1). We have n (−1)n zn(n+1)/2 A = ∏ (1 − z2k ) and k=1 n z−n(n+1)/2 A = ∏ (1 − z−2k ). k=1 Multiplying these two equalities, we√get (−1)n A2 = ∏nk=1 (1 − z2k )(1 − z−2k ) = 2n + 1, by (2). Therefore A = ±i−n 2n + 1. This actually implies that the re√ quired product is ± 2n + 1, but it must be positive, since all the sines are, and the result follows. 16. First, we have P(x) = Q(x)R(x) for Q(x) = xm − |a|m eiθ and R(x) = xm − |a|m e−iθ , where eiϕ means of course cos ϕ + i sin ϕ . It remains to factor both Q and R. Suppose that Q(x) = (x − q1 ) · · · (x − qm ) and R(x) = (x − r1 ) · · · (x − rm ). m Considering Q(x), we see that |qm k | = |a| and also |qk | = |a| for k = 1, . . . , m. m imβk . Thus we may put qk = |a|eiβk and obtain by de Moivre’s formula qm k = |a| e It follows that mβk = θ +2 jπ for some j ∈ Z, and we have exactly m possibilities π for βk modulo 2π : βk = θ +2(k−1) for k = 1, 2, . . . , m. m i β k Thus qk = |a|e ; analogously we obtain for R(x) that rk = |a|e−iβk . Consequently, m xm − |a|meiθ = ∏ (x − |a|eiβk ) k=1 and m xm − |a|me−iθ = ∏ (x − |a|e−iβk ). k=1 Finally, grouping the kth factors of both polynomials, we get m P(x) = = m ∏ (x − |a|eiβk )(x − |a|e−iβk ) = ∏ (x2 − 2|a|x cos βk + a2) k=1 m  ∏ k=1 k=1  θ + 2(k − 1)π 2 2 x − 2|a|x cos +a . m 17. Let f1 (x) = ax + b and f2 (x) = cx + d be two functions from F . We define g(x) = f1 ◦ f2 (x) = acx + (ad + b) and h(x) = f2 ◦ f1 (x) = acx + (bc + d). By the condition for F , both g(x) and h(x) belong to F . Moreover, there exists h−1 (x) = x−(bc+d) , and ac 406 4 Solutions h−1 ◦ g(x) = acx + (ad + b) − (bc + d) (ad + b) − (bc + d) = x+ ac ac belongs to F . Now it follows that we must have ad + b = bc + d for every b d f1 , f2 ∈ F , which is equivalent to 1−a = 1−c = k. But these formulas exactly b describe the fixed points of f1 and f2 : f1 (x) = ax + b = x ⇒ x = 1−a . Hence all the functions in F fix the point k. 4.16 Shortlisted Problems 1974 407 4.16 Solutions to the Shortlisted Problems of IMO 1974 1. Denote by n the number of exams. We have n(A + B+C) = 20 +10 +9 = 39, and since A, B,C are distinct, their sum is at least 6; therefore n = 3 and A + B +C = 13. Assume w.l.o.g. that A > B > C. Since Betty gained A points in arithmetic, but fewer than 13 points in total, she had C points in both remaining exams (in spelling as well). Furthermore, Carol also gained fewer than 13 points, but with at least B points on two examinations (on which Betty scored C), including spelling. If she had A in spelling, then she would have at least A + B + C = 13 points in total, a contradiction. Hence, Carol scored B and placed second in spelling. Remark. Moreover, it follows that Alice, Betty, and Carol scored B + A + A, A +C +C, and C + B + B respectively, and that A = 8, B = 4, C = 1. 2. We denote by qi the square with side 1i . Let us divide the big square into rectangles ri by parallel lines, where the size of ri is 32 × 21i for i = 2, 3, . . . 1 3 and 32 × 1 for i = 1 (this can be done because 1 + ∑∞ i=2 2i = 2 ). In rectangle r1 , one can put the squares q1 , q2 , q3 , as is done on the figure. Also, since 1 + · · · + 2i+11−1 < 2i · 21i = 1 < 32 , in each ri , i ≥ 2, one can put q2i , . . . , q2i+1 −1 . 2i This completes the proof. q8 , . . . , q15 q4 q5 q6 q7 q1 q3 q2 Remark. It can be shown that the squares q1 , q2 cannot fit in any square of side less than 32 . 3. For deg(P) ≤ 2 the statement is obvious, since n(P) ≤ deg(P2 ) = 2 deg(P) ≤ deg(P) + 2. Suppose now that deg(P) ≥ 3 and n(P) > deg(P) + 2. Then there is at least one integer b for which P(b) = −1, and at least one x with P(x) = 1. We may assume w.l.o.g. that b = 0 (if necessary, we consider the polynomial P(x + b) instead). If k1 , . . . , km are all integers for which P(ki ) = 1, then P(x) = Q(x)(x − k1 ) · · · (x − km ) + 1 for some polynomial Q(x) with integer coefficients. Setting x = 0 we obtain (−1)m Q(0)k1 · · · km = 1 − P(0) = 2. It follows that k1 · · · km | 2, and hence m is at most 3. The same holds for the polynomial −P(x), and thus P(x) = −1 also has at most 3 integer solutions. This counts for 6 solutions of P2 (x) = 1 in total, implying the statement for deg(P) ≥ 4. It remains to verify the statement for n = 3. If deg(P) = 3 and n(P) = 6, then it follows from the above consideration that P(x) is either −(x2 − 1)(x − 2) + 1 or (x2 − 1)(x + 2) + 1. It is directly checked that n(P) equals only 4 in both cases. 408 4 Solutions 4. Assume w.l.o.g. that a1 ≤ a2 ≤ a3 ≤ a4 ≤ a5 . If m is the least value of |ai − a j |, i 6= j, then ai+1 − ai ≥ m for i = 1, 2, . . . , 5, and consequently ai − a j ≥ (i − j)m for any i, j ∈ {1, . . . , 5}, i > j. Then it follows that ∑ (ai − a j )2 ≥ m2 ∑ (i − j)2 = 50m2. i> j i> j On the other hand, by the condition of the problem, 5 ∑(ai − a j )2 = 5 ∑ a2i − (a1 + · · · + a5)2 ≤ 5. i> j i=1 Therefore 50m2 ≤ 5; i.e., m2 ≤ 1 10 . 5. All the angles are assumed to be oriented and measured modulo 180◦ . Denote by αi , βi , γi the angles of triangle △i , at Ai , Bi ,Ci respectively. Let us determine the angles of △i+1 . If Di is the intersection of lines Bi Bi+1 and CiCi+1 , we have ∠Bi+1 Ai+1Ci+1 = ∠Di BiCi+1 = ∠Bi DiCi+1 + ∠DiCi+1 Bi = ∠Bi DiCi − ∠BiCi+1Ci = −2∠Bi AiCi . We conclude that αi+1 = −2αi , and analogously βi+1 = −2βi , γi+1 = −2γi . Therefore αr+t = (−2)t αr . However, since (−2)12 ≡ 1 (mod 45) and consequently (−2)14 ≡ (−2)2 (mod 180), it follows that α15 = α3 , since all values are modulo 180◦ . Analogously, β15 = β3 and γ15 = γ3 , and moreover, △3 and △15 are inscribed in the same circle; hence △3 ∼ = △15 . 6. We set     2n + 1 3k 1 n 2n + 1 √ 2k+1 x= ∑ 2 =√ ∑ 8 , 8 k=0 2k + 1 k=0 2k + 1   n  n  2n + 1 3k 2n + 1 √ 2k y= ∑ 2 =∑ 8 . 2k 2k k=0 k=0 n Both x and y are positive integers. Also, from the binomial formula we obtain  2n+1  √ √ 2n + 1 √ i y+x 8 = ∑ 8 = (1 + 8)2n+1 , i i=0 √ √ 2n+1 and similarly y − x 8 = (1 − 8) . √ √ Multiplying these equalities, we get y2 − 8x2 = (1 + 8)2n+1 (1 − 8)2n+1 = −72n+1 . Reducing modulo 5 gives us 3x2 − y2 ≡ 22n+1 ≡ 2 · (−1)n . Now we see that if x is divisible by 5, then y2 ≡ ±2 (mod 5), which is impossible. Therefore x is never divisible by 5. 4.16 Shortlisted Problems 1974 409 Second solution. Another standard way is considering recurrent formulas. If we set     m m k xm = ∑ 8k , ym = ∑ 8, 2k + 1 2k k k    then since ab = a−1 + a−1 b b−1 , it follows that xm+1 = xm + ym and ym+1 = 8xm + ym ; therefore xm+1 = 2xm + 7xm−1 . We need to show that none of x2n+1 are divisible by 5. Considering the sequence {xm } modulo 5, we get that xm = 0, 1, 2, 1, 1, 4, 0, 3, 1, 3, 3, 2, 0, 4, 3, 4, 4, 1, . . . . Zeros occur in the initial position of blocks of length 6, where each subsequent block is obtained by multiplying the previous one by 3 (modulo 5). Consequently, xm is divisible by 5 if and only if m is a multiple of 6, which cannot happen if m = 2n + 1. 7. Consider an arbitrary prime number p. If p | m, then there exists bi that is divisible by the same power of p as m. Then p divides neither ai bmi nor ai , because (ai , bi ) = 1. If otherwise p ∤ m, then bmi is not divisible by p for any i, hence p di  vides ai and ai bmi to the same power. Therefore (a1 , . . . , ak ) and a1 bm1 , . . . , ak bmk have the same factorization; hence they are equal. Second solution. For k = 2 we can easily verify the formula   a1 a 2 m 1 m ,m = (a1 b2 , a2 b1 ) = [b1 , b2 ](a1 , a2 )(b1 , b2 ) = (a1 , a2 ), b1 b 2 b 1 b2 b 1 b2 since [b1 , b2 ] · (b1, b2 ) = b1 b2 . We proceed by induction:     m m m m m a1 , . . . , ak , ak+1 = (a1 , . . . , ak ), ak+1 b1 bk bk+1 [b1 , . . . , bk ] bk+1 = (a1 , . . . , ak , ak+1 ). 8. It is clear that a b c d + + + <S a+b+c+d a+b+c+d a+b+c+d a+b+c+d and S < a b c d + + + , a+b a+b c+d c+d or equivalently, 1 < S < 2. On the other hand, all values from (1, 2) are attained. Since S = 1 for (a, b, c, d) = (0, 0, 1, 1) and S = 2 for (a, b, c, d) = (0, 1, 0, 1), due to continuity all the values from (1, 2) are obtained, for example, for (a, b, c, d) = (x(1 − x), x, 1 − x, 1), where x goes through (0, 1). Second solution. Set S1 = a c + a+b+d b+c+d and S2 = b d + . a+b+c a+c+d We may assume without loss of generality that a+b+c+d = 1. Putting a+c = x and b + d = y (then x + y = 1), we obtain that the set of values of 410 4 Solutions S1 = a c 2ac + x − x2 + = 1−c 1−a ac + 1 − x  2x is x, 2−x . Having the analogous result for S2 in mind, we conclude that the  i 2y 2x values that S = S1 + S2 can take are x + y, 2−x + 2−y . Since x + y = 1 and 2x 2y 4 − 4xy + = ≤2 2−x 2−y 2 + xy with equality for xy = 0, the desired set of values for S is (1, 2). 9. There exist real numbers a, b, c with tan a = x, tanb = y, tan c = z. Then using the additive formula for tangents we obtain tan(a + b + c) = x + y + z − xyz . 1 − xy − xz − yz We are given that xyz = x + y + z. In this case xy + yz + zx = 1 is impossible; otherwise, x, y, z would be the zeros of a cubic polynomial t 3 − λ t 2 + t − λ = (t 2 + 1)(t − λ ) (where λ = xyz), which has only one real root. It follows that x + y + z = xyz ⇐⇒ tan(a + b + c) = 0. (1) 3 3x−x Hence a + b + c = kπ for some k ∈ Z. We note that 1−3x 2 actually expresses tan 3a. Since 3a + 3b + 3c = 3kπ , the result follows from (1) for the numbers 3x−x3 3y−y3 3z−z3 , , . 1−3x2 1−3y2 1−3z2 10. If we set ∠ACD = γ1 and ∠BCD = γ2 for a point D on the segment AB, then by the sine theorem, f (D) = CD2 CD CD sin α sin β = · = . AD · BD AD BD sin γ1 sin γ2 The denominator of the last fraction is 1 (cos(γ1 − γ2 ) − cos(γ1 + γ2 )) 2 1 1 − cos γ γ = (cos(γ1 − γ2 ) − cos γ ) ≤ = sin2 . 2 2 2 h  α sin β Now we deduce that the set of values of f (D) is the interval sinsin 2 γ , +∞ . sin γ1 sin γ2 = 2 Hence f (D) = 1 (equivalently, CD2 = AD · BD) is possible if and only if sin α sin β ≤ sin2 γ2 , i.e., p γ sin α sin β ≤ sin . 2 Second solution. Let E be the second point of intersection of the line CD with the circumcircle k of ABC. Since AD · BD = CD · ED (power of D with respect to k), 4.16 Shortlisted Problems 1974 411 ED CD2 = AD · BD ie equivalent to ED = CD. Clearly the ratio CD (D ∈ AB) takes a maximal value when E is the midpoint of the arc AB not containing C. (This follows from ED : CD = E ′ D : C′ D when C′ and E ′ are respectively projections from C and E onto AB.) On the other hand, it is directly shown that in this case sin2 γ2 ED = , CD sin α sin β and the assertion follows. 11. First, we notice that a1 + a2 + · · · + a p = 32. The numbers ai are distinct, and consequently ai ≥ i and a1 + · · · + a p ≥ p(p + 1)/2. Therefore p ≤ 7. The number 32 can be represented as a sum of 7 mutually distinct positive integers in the following ways: (1) (2) (3) (4) (5) 32 = 1 + 2 + 3 + 4 + 5 + 6 + 11; 32 = 1 + 2 + 3 + 4 + 5 + 7 + 10; 32 = 1 + 2 + 3 + 4 + 5 + 8 + 9; 32 = 1 + 2 + 3 + 4 + 6 + 7 + 9; 32 = 1 + 2 + 3 + 5 + 6 + 7 + 8. The case (1) is eliminated because there is no rectangle with 22 cells on an 8 × 8 chessboard. In the other cases the partitions are realized as below. 20 8 14 2 10 4 10 18 2 4 6 8 16 6 Case (2) Case (3) 12 8 14 18 10 12 14 16 4 6 6 2 Case (4) 4 2 Case (5) 12. We say that a word is good if it doesn’t contain any nonallowed word. Let an be the number of good words of length n. If we prolong any good word of length n by adding one letter to its end (there are 3an words that can be so obtained), we get either (i) a good word of length n + 1, or (ii) an (n + 1)-letter word of the form XY , where X is a good word and Y a nonallowed word. The number of words of type (ii) with word Y of length k is exactly an+1−k ; hence the total number of words of kind (ii) doesn’t exceed an−1 + · · · + a1 + a0 (where a0 = 1). Hence an+1 ≥ 3an − (an−1 + · · · + a1 + a0 ), a0 = 1, a1 = 3. (1) We prove by induction that an+1 > 2an for all n. For n = 1 the claim is trivial. If it holds for i ≤ n, then ai ≤ 2i−n an ; thus we obtain from (1) 412 4 Solutions an+1 > an  1 1 1 3 − − 2 − ···− n 2 2 2  > 2an . Therefore an ≥ 2n for all n (moreover, one can show from (1) that an ≥ (n + 2)2n−1 ); hence there exist good words of length n. Remark. If there are two nonallowed words (instead of one) of each length greater than 1, the statement of the problem need not remain true. 4.17 Shortlisted Problems 1975 413 4.17 Solutions to the Shortlisted Problems of IMO 1975 1. First, we observe that there cannot exist three routes of the form (A, B,C), (A, B, D), (A,C, D), for if E, F are the remaining two ports, there can be only one route covering A, E, namely, (A, E, F). Thus if (A, B,C), (A, B, D) are two routes, the one covering A,C must be w.l.o.g. (A,C, E). The other roots are uniquely determined: These are (A, D, F), (A, E, F), (B, D, E), (B, E, F), (B,C, F), (C, D, E), (C, D, F ). 2. Since there are finitely many arrangements of the zi ’s, assume that z1 , . . . , zn is the one for which ∑ni=1 (xi − zi )2 is minimal. We claim that in this case if i < j and xi 6= x j then zi ≥ z j , from which the claim of the problem directly follows. Indeed, otherwise we would have (xi − z j )2 + (x j − zi )2 = (xi − zi )2 + (x j − z j )2 +2(xi zi + x j z j − xi z j − x j zi ) = (xi − zi )2 + (x j − z j )2 + 2(xi − x j )(zi − z j ) ≤ (xi − zi )2 + (x j − z j )2 , contradicting the assumption.   3. From (k + 1)2/3 + (k + 1)1/3k1/3 + k2/3 (k + 1)1/3 − k1/3 = 1 and 3k2/3 < (k + 1)2/3+ (k + 1)1/3k1/3 + k2/3 < 3(k + 1)2/3 we obtain     3 (k + 1)1/3 − k1/3 < k−2/3 < 3 k1/3 − (k − 1)1/3 . Summing from 1 to n we get   1 + 3 (n + 1)1/3 − 21/3 < In particular, for n = 109 n ∑ k−2/3 < 1 + 3(n1/3 − 1). k=1 this inequality gives   2997 < 1 + 3 (109 + 1)1/3 − 21/3 < h 9 109 ∑ k−2/3 < 2998. k=1 i −2/3 Therefore ∑10 = 2997. k=1 k 4. Put ∆ an = an − an+1 . By the imposed condition, ∆ an > ∆ an+1 . Suppose that for some n, ∆ an < 0: Then for each k ≥ n, ∆ ak < ∆ an ; hence an − an+m = ∆ an + · · · + ∆ an+m−1 < m∆ an . Thus for sufficiently large m it holds that an − an+m < −1, which is impossible. This proves the first part of the inequality. Next one observes that n≥ n n k=1 k=1 ∑ ak = nan+1 + ∑ k∆ ak ≥ (1 + 2 + · · ·+ n)∆ an = Hence (n + 1)∆ an ≤ 2. n(n + 1) ∆ an . 2 414 4 Solutions 5. There are exactly 8 · 9k−1 k-digit numbers in M (the first digit can be chosen in 8 ways, while any other position admits 9 possibilities). The least of them is 10k , and hence xj k k 1 1 1 =∑ <∑ ∑ ∑ 10i−1 x x j j i=1 10i−1 ≤x <10i i=1 10i−1 ≤x <10i <10k ∑ j k = ∑ i=1 8 · 9i−1 10i−1 j   9k = 80 1 − k < 80. 10 6. Let us denote by C the sum of digits of B. We know that 1616 ≡ A ≡ B ≡ C (mod 9). Since 1616 = 264 = 26·10+4 ≡ 24 ≡ 7, we get C ≡ 7 (mod 9). Moreover, 1616 < 10016 = 1032 , hence A cannot exceed 9·32 = 288; consequently, B cannot exceed 19 and C is at most 10. Therefore C = 7. 7. We use induction on m. Denote by Sm the left-hand side of the equality to be proved. First S0 = (1 − y)(1 + y + · · ·+ yn ) + yn+1 = 1, since x = 1 − y. Furthermore, Sm+1 − Sm       n  m + n + 1 m+1 n+1 m+1+ j m+ j j m+1 j = x y +x xy − y ∑ m+1 j j j=0   m + n + 1 m+1 n+1 = x y m+1        n m+1+ j j m+ j j m + 1 + j j+1 m+1 +x y − y − y ∑ j j j j=0 "     # n  m + n + 1 n+1 m+ j j m + j + 1 j+1 m+1 =x y +∑ y − y n j−1 j j=0 = 0; i.e., Sm+1 = Sm = 1 for every m. Second solution. Let us be given an unfair coin that, when tossed,  shows heads with probability x and tails with probability y. Note that xm+1 m+j j y j is the probability that until the moment when the (m + 1)th  i head appears, exactly j tails ( j < n + 1) have appeared. Similarly, yn+1 n+i i x is the probability that exactly i heads will appear before the (n + 1)th tail occurs. Therefore, the above sum is the probability that either m + 1 heads will appear before n + 1 tails, or vice versa, and this probability is clearly 1. 8. Denote by K and L the feet of perpendiculars from the points P and Q to the lines BC and AC respectively. 4.17 Shortlisted Problems 1975 415 C Let M and N be the points on AB (ordered A − N − M − B) such that the triP angle RMN is isosceles with ∠R = 90◦ . BM Q By sine theorem we have BM · BA◦ = BR L K ◦ BR sin 15 BK sin 45 sin 30◦ = BA = sin 45◦ . Since BC = cos 15◦ N M sin 15◦ B A sin 45◦ , we deduce that MK k AC and MK = AL. Similarly, NL k BC R − → −→ −→ −→ and NL = BK. It follows that the vectors RN, NL, and LQ are the images of RM, −→ −−→ KP, and MK respectively under a rotation of 90◦ , and consequently the same −→ − → holds for their sums RQ and RP. Therefore, QR = RP and ∠QRP = 90◦ . Second solution. Let ABS be the equilateral triangle constructed in the exterior of △ABC. Obviously, the triangles BPC, BRS, ARS, AQC are similar. Let f be the rotational homothety centered at B that maps P onto C, and let g be the rotational homothety about A that maps C onto Q. The composition h = g ◦ f is also a rotational homothety; its angle is ∠PBC + ∠CAQ = 90◦ , and the coefficient is BC AQ BP · AC = 1. Moreover, R is a fixed point of h because f (R) = S and g(S) = R. Hence R is the center of h, and the statement follows from h(P) = Q. Remark. There are two more possible approaches: One includes using complex numbers and the other one is mere calculating of RP, RQ, PQ by the cosine theorem. Second remark. The problem allows a generalization: Given that ∠CBP = ∠CAQ = α , ∠BCP = ∠ACQ = β , and ∠RAB = ∠RBA = 90◦ − α − β , show that RP = RQ and ∠PRQ = 2α . 9. Suppose n is the natural number with na ≤ 1 < (n + 1)a. Let fa (x) = f (x − a). If a function f with the desired properties exists, then fa (a) = 0 and let w.l.o.g. f (a) > 0, or equivalently, let the graph of fa lie below the graph of f . In this case also f (2a) > f (a), since otherwise, the graphs of f and fa would intersect between a and 2a. Continuing in this way we are led to 0 = f (0) < f (a) < f (2a) < · · · < f (na). Thus if na = 1, i.e., a = 1/n, such an f does not exist. On the other hand, if a 6= 1/n, then we similarly obtain f (1) > f (1 − a) > f (1 − 2a) > · · · > f (1 − na). Choosing values of f at ia, 1 − ia, i = 1, . . . , n, so that they satisfy f (1 − na) < · · · < f (1 − a) < 0 < f (a) < · · · < f (na), we can extend f to other values of [0, 1] by linear interpolation. A function obtained this way has the desired property. 10. We shall prove that for all x, y with x + y = 1 it holds that f (x, y) = x − 2y. In this case f (x, y) = f (x, 1 − x) can be regarded as a polynomial in z = x − 2y = 3x − 2, say f (x, 1 − x) = F(z). Putting in the given relation a = b = x/2, c = 1 − x, we obtain f (x, 1 − x) + 2 f (1 − x/2, x/2) = 0; hence F(z) + 2F(−z/2) = 0. Now F(1) = 1, and we get that for all k, F((−2)k ) = (−2)k . Thus F(z) = z for infinitely many values of z; hence F(z) ≡ z. Consequently f (x, y) = x − 2y if x + y = 1. 416 4 Solutions For general x, ywith x + y 6=   0, since f is homogeneous ,we have f (x, y) = (x + y y x x n n y) f x+y , x+y = (x + y) x+y − 2 x+y = (x + y)n−1 (x − 2y). The same is true for x + y = 0, because f is a polynomial. 11. Let (aki ) be the subsequence of (ak ) consisting of all ak ’s that give remainder r upon division by a1 . For every i > 1, aki ≡ ak1 (mod a1 ); hence aki = ak1 +ya1 for some integer y > 0. It follows that for every r = 0, 1, . . . , a1 − 1 there is exactly one member of the corresponding (aki )i≥1 that cannot be represented as xal + yam , and hence at most a1 + 1 members of (ak ) in total are not representable in the given form. 12. Since sin 2xi = 2 sin xi cosxi and sin(xi + xi+1 ) + sin(xi − xi+1 ) = 2 sin xi cos xi+1 , the inequality from the problem is equivalent to (cos x1 − cosx2 ) sin x1 + (cosx2 − cosx3 ) sin x2 + · · · π . (1) 4 Consider the unit circle with center at O(0, 0) and points Mi (cos xi , sin xi ) on it. Also, choose the points Ni (cos xi , 0) and Mi′ (cos xi+1 , sin xi ). It is clear that (cos xi − cosxi+1 ) sin xi is equal to the area of the rectangle Mi Ni Ni+1 Mi′ . Since all these rectangles are disjoint and lie inside the quarter circle in the first quadrant whose area is π4 , inequality (1) follows. · · · + (cosxν −1 − cosxν ) sin xν −1 < 13. Suppose that Ak Ak+1 ∩ Am Am+1 6= 0/ for some k, m > k + 1. Without loss of generality we may suppose that k = 0, m = n − 1 and that no two segments Ak Ak+1 and Am Am+1 intersect for 0 ≤ k < m − 1 < n − 1 except for k = 0, m = n − 1. Also, shortening A0 A1 , we may suppose that A0 ∈ An−1 An . Finally, we may reduce the problem to the case that A0 . . . An−1 is convex: Otherwise, the segment An−1 An can be prolonged so that it intersects some Ak Ak+1 , 0 < k < n − 2. If n = 3, then A1 A2 ≥ 2A0 A1 implies A0 A2 > A0 A1 , hence ∠A0 A1 A2 > ∠A1 A2 A3 , a contradiction. Let n = 4. From A3 A2 > A1 A2 we conclude that ∠A3 A1 A2 > ∠A1 A3 A2 . Using the inequality ∠A0 A3 A2 > ∠A0 A1 A2 we obtain that ∠A0 A3 A1 > ∠A0 A1 A3 implying A0 A1 > A0 A3 . Now we have A2 A3 < A3 A0 + A0 A1 + A1 A2 < 2A0 A1 + A1 A2 ≤ 2A1 A2 ≤ A2 A3 , which is not possible. Now suppose n ≥ 5. If αi is the exterior angle at Ai , then α1 > · · · > αn−1 ; ◦ ◦ ◦ hence αn−1 < 360 n−1 ≤ 90 . Consequently ∠An−2 An−1 A0 ≥ 90 and A0 An−2 > A n−1 An−2 . On the other  hand, A0 An−2 < A0 A1 + A1 A2 + · · · + An−3 An−2 < 1 1 1 + 2n−3 + · · · + 2 An−1 An−2 < An−1 An−2 , which contradicts the previous 2n−2 relation. √ √ 14. We shall prove that for every n ∈ N, 2n + 25 ≤ xn ≤ 2n + 25+ 0.1. Note that for n = 1000 this gives us exactly the desired inequalities. First, notice that the recurrent relation is equivalent to 2xk (xk+1 − xk ) = 2. (1) 4.17 Shortlisted Problems 1975 417 Since x0 < x1 < · · · < xk < · · · , from (1) we get x2k+1 − x2k = (xk+1 + xk )(xk+1 − xk ) > 2. Adding these up we obtain x2n ≥ x20 + 2n, which proves the first inequality. On the other hand, xk+1 = xk + x1 ≤ xk + 0.2 (for xk ≥ 5), and one also dek duces from (1) that x2k+1 −x2k − 0.2(xk+1 −xk ) = (xk+1 +xk −0.2)(xk+1 − xk ) ≤ 2. Again, adding these inequalities up, (k = 0, . . . , n − 1) yields x2n ≤ 2n + x20 + 0.2(xn − x0 ) = 2n + 24 + 0.2xn. Solving the corresponding quadratic equation, we obtain √ √ xn < 0.1 + 2n + 24.01 < 0.1 + 2n + 25. 15. Assume that the center of the circle is at the origin O(0, 0), and that the points A1 , A2 , . . . , A1975 are arranged on the upper half-circle so that ∠Ai OA1 = αi (α1 = α −α α α 0). The distance Ai A j equals 2 sin j 2 i = 2 sin 2j cos α2i − cos 2j sin α2i , and it αk αk will be rational if all sin 2 , cos 2 are rational. Finally, observe that there exist infinitely many angles α such that both sin α , cos α are rational, and that such α can be arbitrarily small. For example, take α 2 so that sin α = t 22t+1 and cos α = tt 2 −1 for any t ∈ Q. +1 418 4 Solutions 4.18 Solutions to the Shortlisted Problems of IMO 1976 1. Let r denote the common inradius. Some two of the four triangles with the inradii ρ have cross angles at M: Suppose these are △AMB1 and △BMA1 . We shall show that △AMB1 ∼ = △BMA1 . Indeed, the altitudes of these two triangles are both equal to r, the inradius of △ABC, and their interior angles at M are equal to some angle ϕ . If P is the point of tangency of the incircle of △A1 MB ρ A1 M+BM−A1 B 1B with MB, then ρr = A1 M+BM+A , which also implies r−2 = A1 B ρ = A1 B 2r cot(ϕ /2) 2r cot(ϕ /2) ρ = A1 B . Since similarly r−2 , we obtain A1 B = B1 A and ρ = B1 A ∼ consequently △AMB1 = △BMA1 . Thus ∠BAC = ∠ABC and CC1 ⊥ AB. There are two alternatives for the other two incircles: (i) If the inradii of AMC1 and AMB1 are equal to r, it is easy to obtain that △AMC1 ∼ = △AMB1 . Hence ∠AB1 M = ∠AC1 M = 90◦ , and △ABC is equilateral. (ii) The inradii of AMB1 and CMB1 are equal to r. Put x = ∠MAC1 = ∠MBC1 . SAMB AB1 In this case ϕ = 2x and ∠B1 MC = 90◦ − x. Now we have CB = SCMB1 = 1 2MP A1 B AM+MB1 +AB1 CM+MB1 +CB1 AB1 AB CB1 = BC = = AM+MB1 −AB1 CM+MB1 −CB1 = cot x . cot(45◦ −x/2) 1 On the other hand, we have 2 cos 2x. Thus we have an equation for x: tan(45◦ − x/2) = 2 cos 2x tan x, or equivalently   x  ◦ x x 2 tan 45◦ − sin 45 − cos 45◦ − = 2 cos 2x sin x. 2 2 2  Hence sin 3x − sin x = 2 sin2 45◦ − 2x = 1 − sin x, implying sin 3x = 1, i.e., x = 30◦ . Therefore △ABC is equilateral. 2. Let us put bi = i(n + 1 − i)/2, and let ci = ai − bi , i = 0, 1, . . . , n + 1. It is easy to verify that b0 = bn+1 = 0 and bi−1 − 2bi + bi+1 = −1. Subtracting this inequality from ai−1 − 2ai + ai+1 ≥ −1, we obtain ci−1 − 2ci + ci+1 ≥ 0, i.e., 2ci ≤ ci−1 + ci+1 . We also have c0 = cn+1 = 0. Suppose that there exists i ∈ {1, . . . , n} for which ci > 0, and let ck be the maximal such ci . Assuming w.l.o.g. that ck−1 < ck , we obtain ck−1 + ck+1 < 2ck , which is a contradiction. Hence ci ≤ 0 for all i; i.e., ai ≤ bi . Similarly, considering the sequence c′i = ai + bi one can show that c′i ≥ 0, i.e., ai ≥ −bi for all i. This completes the proof. 3. (a) Let ABCD be a quadrangle with 16 = d = AB + CD + AC, and let S be its area. Then S ≤ (AC ·AB+AC ·CD)/2 = AC(d −AC)/2 ≤ d 2 /8 = 32, where equality occurs if√and only if AB ⊥ AC ⊥ CD and AC = AB + CD = 8. In this case BD = 8 2. −→ − → (b) Let A′ be the point with DA′ = AC. The triangular inequality implies AD + √ BC ≥ AA′ = 8 5. Thus the perimeter attains its minimum for AB = CD = 4. (c) Let us assume w.l.o.g. that CD ≤ AB. Then C lies inside △BDA′ and hence BC + AD = BC +CA′ < BD + DA′ . The maximal value BD + DA′ of BC + AD is attained when C approaches D, making a degenerate quadrangle. 4.18 Shortlisted Problems 1976 419 4. The first few values are easily verified to be 2rn + 2−rn , where r0 = 0, r1 = r2 = 1, r3 = 3, r4 = 5, r5 = 11, . . . . Let us put un = 2rn + 2−rn (we will show that rn exists and is integer for each n). A simple calculation gives us un (u2n−1 − 2) = 2rn +2rn−1 + 2−rn −2rn−1 + 2rn−2rn−1 + 2−rn+2rn−1 . If an array qn , with q0 = 0 and q1 = 1, is set so as to satisfy the linear recurrence qn+1 = qn + 2qn−1, then it also satisfies qn − 2qn−1 = −(qn−1 − 2qn−2) = · · · = (−1)n−1(q1 − 2q0) = (−1)n−1 . Assuming inductively up to n ri = qi , the expression for un (u2n−1 − 2) = un+1 +u1 reduces to 2qn+1 +2−qn+1 + u1 . Therefore, rn+1 = qn+1 . The solution to this linear n n recurrence with r0 = 0, r1 = 1 is rn = qn = 2 −(−1) , and since [un ] = 2rn for 3 n ≥ 0, the result follows. n n Remark. One could simply guess that un = 2rn + 2−rn for rn = 2 −(−1) , and 3 then prove this result by induction. 5. If one substitutes an integer q-tuple (x1 , . . . , xq ) satisfying |xi | ≤ p for all i in an equation of the given system, the absolute value of the right-hand member never exceeds pq. So for the right-hand member of the system there are (2pq + 1) p possibilities There are (2p +1)q possible q-tuples (x1 , . . . , xq ). Since (2p + 1)q > (2pq + 1) p , there are at least two q-tuples (y1 , . . . , yq ) and (z1 , . . . , zq ) giving the same right-hand members in the given system. The difference (x1 , . . . , xq ) = (y1 − z1 , . . . , yq − zq ) thus satisfies all the requirements of the problem. √ 6. Suppose a1 ≤ a2 ≤ a3 are the dimensions of the box. If we set bi = [ai / 3 2], the condition of the problem is equivalent to ab11 · ab22 · ab33 = 5. We list some values of √ a, b = [a/ 3 2] and a/b: a 2 3 4 5 6 7 8 9 10 b 1 2 3 3 4 5 6 7 7 a/b 2 1.5 1.33 1.67 1.5 1.4 1.33 1.29 1.43 We note that if a > 2, then a/b ≤ 5/3, and if a > 5, then a/b ≤ 3/2. If a1 > 2, then ab11 · ab22 · ab33 < (5/3)3 < 5, a contradiction. Hence a1 = 2. If also a2 = 2, then √ a3 /b3 = 5/4 < 3 2, which is impossible. Also, if a2 ≥ 6, then ab22 · ab33 ≤ (1.5)2 < 2.5, again a contradiction. We thus have the following cases: (i) a1 = 2, a2 = 3, then a3 /b3 = 5/3, which holds only if a3 = 5; (ii) a1 = 2, a2 = 4, then a3 /b3 = 15/8, which is impossible; (iii) a1 = 2, a2 = 5, then a3 /b3 = 3/2, which holds only if a3 = 6. The only possible sizes of the box are therefore (2, 3, 5) and (2, 5, 6). 7. The map T transforms the interval (0, a] onto (1−a, 1] and the interval (a, 1] onto (0, 1 − a]. Clearly T preserves the measure. Since the measure of the interval [0, 1] is finite, there exist two positive integers k, l > k such that T k (J) and T l (J) are not disjoint. But the map T is bijective; hence T l−k (J) and J are not disjoint. 8. Every polynomial with real coefficients can be factored as a product of linear and quadratic polynomials with real coefficients. Thus it suffices to prove the result only for a quadratic polynomial P(x) = x2 − 2ax + b2, with a > 0 and b2 > a2 . Using the identity 420 4 Solutions (x2 + b2 )2n − (2ax)2n = (x2 − 2ax + b2) 2n−1 ∑ (x2 + b2)k (2ax)2n−k−1 k=0  we have solved the problem if we can choose n such that b2n 2n > 22n a2n . n 2n However, it is is easy to show that 2n n < 22n ; hence it is enough to take n such that (b/a)2n > 2n. Since limn→∞ (2n)1/(2n) = 1 < b/a, such an n always exists. 9. The equation Pn (x) = x is of degree 2n , and has at most 2n distinct roots. If x > 2, then by simple induction Pn (x) > x for all n. Similarly, if x < −1, then P1 (x) > 2, which implies Pn (x) > 2 for all n. It follows that all real roots of the equation Pn (x) = x lie in the interval [−2, 2], and thus have the form x = 2 cost. Observe that P1 (2 cost) = 4 cos2 t − 2 = 2 cos 2t, and in general Pn (2 cost) = 2 cos 2nt. Our equation becomes cos 2nt = cost, which indeed has 2n different solutions t = m t = 22nπ+1 (m = 1, 2, . . . , 2n−1 ). 2π m 2n −1 (m = 0, 1, . . . , 2n−1 − 1) and 10. Let a1 ≤ a2 ≤ · · · ≤ an be positive integers whose sum is 1976. Let M denote the maximal value of a1 a2 · · · an . We make the following observations: (1) a1 = 1 does not yield the maximum, since replacing 1, a2 by 1 + a2 increases the product. (2) a j − ai ≥ 2 does not yield the maximal value, since replacing ai , a j by ai + 1, a j − 1 increases the product. (3) ai ≥ 5 does not yield the maximal value, since 2(ai − 2) = 2ai − 4 > ai . Since 4 = 22 , we may assume that all ai are either 2 or 3, and M = 2k 3l , where 2k + 3l = 1976. (4) k ≥ 3 does not yield the maximal value, since 2 · 2 · 2 < 3 · 3. Hence k ≤ 2 and 2k ≡ 1976 (mod 3) gives us k = 1, l = 658 and M = 2 · 3658. k 11. We shall show by induction that 52 − 1 = 2k+2 qk for each k = 0, 1, . . . , where qk ∈ N. Indeed, is true for k = 0, and if it holds k then  k the statement  k   kfor some  k+1 2 2 2 k+3 2 5 − 1 = 5 + 1 5 − 1 = 2 dk+1 where dk+1 = 5 + 1 dk /2 is an integer by the inductive hypothesis. Let us now choose n = 2k + k + 2. We have 5n = 10k+2 qk + 5k+2 . It follows from 54 < 103 that 5k+2 has at most [3(k + 2)/4] + 2 nonzero digits, while 10k+2 qk ends in k + 2 zeros. Hence the decimal representation of 5n contains at least [(k + 2)/4] − 2 consecutive zeros. Now it suffices to take k > 4 · 1978. 12. Suppose the decomposition into k polynomials is possible. The sum of coefficients of each polynomial a1 x + a2 x2 + · · · + an xn equals 1 + · · · + n = n(n + 1)/2 while the sum of coefficients of 1976(x + x2 + · · · + xn ) is 1976n. Hence we must have 1976n = kn(n + 1)/2, which reduces to (n + 1) | 3952 = 24 · 13 · 19. In other words, n is of the form n = 2α 13β 19γ − 1, with 0 ≤ α ≤ 4, 0 ≤ β ≤ 1, 0 ≤ γ ≤ 1. 4.18 Shortlisted Problems 1976 421 We can immediately eliminate the values n = 0 and n = 3951 that correspond to α = β = γ = 0 and α = 4, β = γ = 1. We claim that all other values n are permitted. There are two cases. α ≤ 3. In this case k = 3952/(n + 1) is even. The simple choice of the polynomials P = x + 2x2 + · · · + nxn and P′ = nx + (n − 1)x2 + · · · + xn suffices, since k(P + P′)/2 = 1976(x + x2 + · · · + xn ). α = 4. Then k is odd. Consider (k − 3)/2 pairs (P, P′ ) of the former case and   n P1 = nx + (n − 1)x3 + · · · + n+1 2 x  n−1 2 4 n−1 + + n−3 ; 2 x + · · · + x  n+1 2 x n−1 3 n P2 = 2 x + 2 x + · · · + x  n−1 . + nx2 + (n − 1)x4 + · · · + n+3 2 x Then P + P1 + P2 = 3(n + 1)(x + x2 + · · · + xn )/2 and therefore (k − 3)(P + P′ )/2 + (P + P1 + P2) = 1976(x + x2 + · · · + xn ). It follows that the desired decomposition is possible if and only if 1 < n < 3951 and n + 1 | 2 · 1976. 422 4 Solutions 4.19 Solutions to the Longlisted Problems of IMO 1977 1. Let P be the projection of S onto the plane ABCDE. Obviously BS > CS is equivalent to BP > CP. The conditions of the problem imply that PA > PB and PA > PE. The locus of such points P is the region of the plane that is determined by the perpendicular bisectors of segments AB and AE and that contains the point diametrically opposite A. But since AB < DE, the whole of this region lies on one side of the perpendicular bisector of BC. The result follows immediately. Remark. The assumption BC < CD is redundant. 2. We shall prove by induction on n that f (x) > f (n) whenever x > n. The case n = 0 is trivial. Suppose that n ≥ 1 and that x > k implies f (x) > f (k) for all k < n. It follows that f (x) ≥ n holds for all x ≥ n. Let f (m) = minx≥n f (x). If we suppose that m > n, then m − 1 ≥ n and consequently f (m − 1) ≥ n. But in this case the inequality f (m) > f ( f (m − 1)) contradicts the minimality property of m. The inductive proof is thus completed. It follows that f is strictly increasing, so f (n + 1) > f ( f (n)) implies that n + 1 > f (n). But since f (n) ≥ n we must have f (n) = n. 3. Let v1 , v2 , . . . , vk be k persons who are not acquainted with each other. Let us denote by m the number of acquainted couples and by d j the number of acquaintances of person v j . Then m ≤ dk+1 + dk+2 + · · · + dn ≤ d(n − k) ≤ k(n − k) ≤  k + (n − k) 2 2 = n2 . 4 4. Consider any vertex vn from which the maximal number d of segments start, and suppose it is not a vertex of a triangle. Let A = {v1 , v2 , . . . , vd } be the set of points that are connected to vn , and let B = {vd+1 , vd+2 , . . . , vn } be the set of the other points. Since vn is not a vertex of a triangle, there is no segment both of whose vertices lie in A ; i.e., each segment has an end in B. Thus, if d j denotes the number of segments at v j and m denotes the total number of segments, we  have m ≤ dd+1 + dd+2 + · · · + dn ≤ d(n − d) ≤ n2 /4 = m. This means that each inequality must be equality, implying that each point in B is a vertex of d segments, and each of these segments has the other end in A . Then there is no triangle at all, which is a contradiction. 5. Let us denote by I and E the sets of interior boundary points and exterior boundary points. Let ABCD be the square inscribed in the circle k with sides parallel to the coordinate axes. Lines AB, BC,CD, DA divide the plane into 9 regions: R, RA , RB , RC , RD , RAB , RBC , RCD , RDA . There is a unique pair of lattice points AI ∈ R, AE ∈ RA that are opposite vertices of a RCD RD RDA AE A RA RC C D RBC R AI B RAB RB 4.19 Longlisted Problems 1977 423 unit square. We similarly define BI ,CI , DI , BE ,CE , DE . Let us form a graph G by connecting each point from E lying in RAB (respectively RBC , RCD , RDA ) to its upper (respectively left, lower, right) neighbor point (which clearly belongs to I). It is easy to see that: (i) All vertices from I other than AI , BI ,CI , DI have degree 1. (ii) AE is not in E if and only if AI ∈ I and degAI = 2. (iii) No other lattice points inside RA belong to E. Thus if m is the number of edges of the graph G and s is the number of points among AE , BE , CE , and DE that are in E, using (i)–(iii) we easily obtain |E| = m + s and |I| = m − (4 − s) = |E| + 4. 6. Let hyi denote the distance from y ∈ R to the closest even integer. We claim that h1 + cosxi ≤ sin x for all x ∈ [0, π ]. Indeed, if cos x ≥ 0, then h1 + cosxi = 1 − cos x ≤ 1 − cos2 x = sin2 x ≤ sin x; the proof is similar if cos x < 0. We note that hx + yi ≤ hxi + hyi holds for all x, y ∈ R. Therefore * + n n n j=1 j=1 j=1 ∑ sin x j ≥ ∑ h1 + cosx j i ≥ ∑ (1 + cosx j ) = 1. 7. Let us suppose that c1 ≤ c2 ≤ · · · ≤ cn and that c1 < 0 < cn . There exists k, 1 ≤ k < n, such that ck ≤ 0 < ck+1 . Then we have (n − 1)(c21 + c22 + · · · + c2n ) ≥ k(c21 + · · · + c2k ) + (n − k)(c2k+1 + · · · + c2n ) ≥ (c1 + · · · + ck )2 + (ck+1 + · · · + cn )2 = (c1 + · · · + cn )2 −2(c1 + · · · + ck )(ck+1 + · · · + cn ), from which we obtain (c1 + · · · + ck )(ck+1 + · · · + cn ) ≥ 0, a contradiction. Second solution. By the given condition and the inequality between arithmetic and quadratic mean we have (c1 + · · · + cn )2 = (n − 1)(c21 + · · · + c2n−1) + (n − 1)c2n ≥ (c1 + · · · + cn−1)2 + (n − 1)c2n, which is equivalent to 2(c1 + c2 + · · · + cn )cn ≥ nc2n . Similarly, 2(c1 + c2 + · · · + cn )ci ≥ nc2i for all i = 1, . . . , n. Hence all ci are of the same sign. 8. There is exactly one point satisfying the given condition on each face of the hexahedron. Namely, on the face ABD it is the point that divides the median from D in the ratio 32 : 3. √ 9. A necessary and sufficient condition for M to be nonempty is that 1/ 10 ≤ t ≤ 1. 424 4 Solutions 10. Integers a, b, q, r satisfy a2 + b2 = (a + b)q + r, 0 ≤ r < a + b, q2 + r = 1977. From q2 ≤ 1977 it follows that q ≤ 44, and consequently a2 + b2 < 45(a + b). Having in mind the inequality (a + b)2 ≤ 2(a2 +b2 ), we get (a + b)2 < 90(a + b), i.e., a +b < 90 and consequently r < 90. Now from q2 = 1977−r > 1977−90 = 1887 it follows that q > 43; hence q = 44 and r = 41. It remains to find positive integers a and b satisfying a2 + b2 = 44(a + b) + 41, or equivalently (a − 22)2 + (b − 22)2 = 1009. The Diophantine equation A2 + B2 = 1009 has only two pairs of positive solutions: (15, 28) and (28, 15). Hence (|a − 22|, |b − 22|) ∈ {(15, 28), (28, 15)}, which implies (a, b) ∈ {(7, 50), (37, 50), (50, 7), (50, 37)}. 11. (a) Suppose to the contrary that none of the numbers z0 , z1 , . . . , zn−1 is divisible by n. Then two of these numbers, say zk and zl (0 ≤ k < l ≤ n − 1), are congruent modulo n, and thus n | zl − zk = zk+1 zl−k−1 . But since (n, z) = 1, this implies n | zl−k−1 , which is a contradiction. (b) Again suppose the contrary, that none of z0 , z1 , . . . , zn−2 is divisible by n. Since (z − 1, n) = 1, this is equivalent to n ∤ (z − 1)z j , i.e., zk 6≡ 1 (mod n) for all k = 1, 2, . . . , n − 1. But since (z, n) = 1, we also have that zk 6≡ 0 (mod n). It follows that there exist k, l, 1 ≤ k < l ≤ n − 1 such that zk ≡ zl , i.e., zl−k ≡ 1 (mod n), which is a contradiction. 12. According to part (a) of the previous problem we can conclude that T = {n ∈ N | (n, z) = 1}. 13. The figure Φ contains two points A and B having maximum distance. Let h be the semicircle with diameter AB that lies in Φ , and let k be the circle containing h. Consider any point M inside k. The line passing through M that is orthogonal to AM meets h in some point P (because ∠AMB > 90◦ ). Let h′ and h′ be the two semicircles with diameter AP, where M ∈ h′ . Since h′ contains a point C such that BC > AB, it cannot be contained in Φ , implying that h′ ⊂ Φ . Hence M belongs to Φ . Since Φ contains no points outside the circle k, it must coincide with the disk determined by k. On the other hand, any disk has the required property. 14. We prove by induction on n that independently of the word w0 , the given algorithm generates all words of length n. This is clear for n = 1. Suppose now the statement is true for n − 1, and that we are given a word w0 = c1 c2 . . . cn of length n. Obviously, the words w0 , w1 , . . . , w2n−1 −1 all have the nth digit cn , and by the inductive hypothesis these are all words whose nth digit is cn . Similarly, by the inductive hypothesis w2n−1 , . . . , w2n −1 are all words whose nth digit is 1 − cn , and the induction is complete. 15. Each segment is an edge of at most two squares and a diagonal of at most one square. Therefore pk = 0 for k > 3, and we have to prove that p0 = p2 + 2p3. (1) 4.19 Longlisted Problems 1977 425 Let us calculate the number q(n) of considered squares. Each of these squares is inscribed in a square with integer vertices and sides parallel to the coordinate axes. There are (n − s)2 squares of side s with integer vertices and sides parallel to the coordinate axes, and each of them circumscribes exactly s of the consid2 2 2 ered squares. It follows that q(n) = ∑n−1 s=1 (n − s) s = n (n − 1)/12. Computing the number of edges and diagonals of the considered squares in two ways, we obtain that p1 + 2p2 + 3p3 = 6q(n). (2) On the other hand, the total number of segments with endpoints in the considered integer points is given by  2 n n2 (n2 − 1) p 0 + p1 + p2 + p3 = = = 6q(n). (3) 2 2 Now (1) follows immediately from (2) and (3). 16. For i = k and j = l the system is reduced to 1 ≤ i, j ≤ n, and has exactly n2 solutions. Let us assume that i 6= k or j 6= l. The points A(i, j), B(k, l), C(− j +k + l, i − k + l), D(i − j + l, i + j − k) are vertices of a negatively oriented square with integer vertices lying inside the square [1, n] × [1, n], and each of these squares corresponds to exactly 4 solutions to the system. By the previous problem there are exactly q(n) = n2 (n2 − 1)/12 such squares. Hence the number of solutions is equal to n2 + 4q(n) = n2 (n2 + 2)/3. 17. Centers of the balls that are tangent to K are vertices of a regular polyhedron with triangular faces, with edge length 2R and radius of circumscribed sphere r + R. Therefore the number n of these balls is 4, 6, or 20. It is straightforward to obtain that: √ √ (i) If n = 4, then r + R = 2R(√6/4), whence R = r(2 + √6). (ii) If n = 6, then r + R = 2R( p 2/2), whence R = r(1 + 2). √ (iii) If n =h20, p then√r + R = 2R√ 5 +i 5/8. In this case we can conclude that R=r 5 − 2 5+ (3 − 5)/2 . 18. Let U be the midpoint of thep segment AB. The point M belongs to CU and √ √ CM = ( 5 − 1)CU/2, r = CU 5 − 2. 19. We shall prove the statement by induction on m. For m = 2 it is trivial, since each power of 5 greater than 5 ends in 25. Suppose that the statement is true for some m ≥ 2, and that the last m digits of 5n alternate in parity. It can be shown m−2 by induction that the maximum power of 2 that divides 52 − 1 is 2m , and m−2 consequently the difference 5n+2 − 5n is divisible by 10m but not by 2 · 10m. m−2 It follows that the last m digits of the numbers 5n+2 and 5n coincide, but the digits at the position m+1 have opposite parity. Hence the last m+1 digits of one of these two powers of 5 alternate in parity. The inductive proof is completed. 426 4 Solutions 20. There exist u, v such that √ a cosx + b sin x = r√cos(x − u) and A cos 2x + B sin 2x = R cos 2(x− v), where r = a2 + b2 and R = A2 + B2. Then 1 − f (x) = r cos(x− u) + R cos2(x − v) ≤ 1 holds for all x ∈ R. There exists x ∈ R such that cos(x − u) ≥ 0 and cos 2(x − v) = 1 (indeed, either x = v or x = v + π works). √ It follows that R ≤ 1. Similarly, there exists x ∈ R such that cos(x − u) = 1/ 2 and√cos 2(x − v) ≥ 0 (either x = u − π /4 or x = u + π /4 works). It follows that r ≤ 2. Remark. The proposition of this problem contained as an addendum the following, more difficult, inequality: p p a2 + b2 + A2 + B2 ≤ 2. The proof follows from the existence of x ∈ R such that cos(x − u) ≥ 1/2 and cos 2(x − v) ≥ 1/2. 21. Let us consider the vectors v1 = (x1 , x2 , x3 ), v2 = (y1 , y2 , y3 ), v3 = (1, 1, 1) in space. The given equalities express the condition that these three vectors are mutually perpendicular. Also, x21 2 x1 +x22 +x23 , y21 2 y1 +y22 +y23 , and 1/3 are the squares of the projections of the vector (1, 0, 0) onto the directions of v1 , v2 , v3 , respectively. The result follows from the fact that the sum of squares of projections of a unit vector on three mutually perpendicular directions is 1. 22. Since the quadrilateral OA1 BB1 is cyclic, ∠OA1 B1 = ∠OBC. By using the analogous equalities we obtain ∠OA4 B4 = ∠OB3C3 = ∠OC2 D2 = ∠OD1 A1 = ∠OAB, and similarly ∠OB4 A4 = ∠OBA. Hence △OA4 B4 ∼ △OAB. Analogously, we have for the other three pairs of triangles △OB4C4 ∼ △OBC, △OC4 D4 ∼ △OCD, △OD4 A4 ∼ △ODA, and consequently ABCD ∼ A4 B4C4 D4 . 23. Every polynomial q(x1 , . . . , xn ) with integer coefficients can be expressed in the form q = r1 + x1 r2 , where r1 , r2 are polynomials in x1 , . . . , xn with integer coefficients in which the variable x1 occurs only with even exponents. Thus if q1 = r1 − x1 r2 , the polynomial qq1 = r12 − x21 r22 contains x1 only with even exponents. We can continue inductively constructing polynomials q j , j = 2, 3, . . . , n, such that qq1 q2 · · · q j contains each of the variables x1 , x2 , . . . , x j only with even exponents. Thus the polynomial qq1 · · · qn is a polynomial in x21 , . . . , x2n . The polynomials f and g exist for every n ∈ N. In fact, it suffices to construct q1 , . . . , qn for the polynomial q = x1 + · · · + xn and take f = q1 q2 · · · qn . 24. Setting x = y = 0 gives us f (0) = 0. Let us put g(x) = arctan f (x). The given functional equation becomes tang(x + y) = tan(g(x) + g(y)); hence g(x + y) = g(x) + g(y) + k(x, y)π , where k(x, y) is an integer function. But k(x, y) is continuous and k(0, 0) = 0, therefore k(x, y) = 0. Thus we obtain the classical Cauchy’s functional equation g(x + y) = g(x) + g(y) on the interval (−1, 1), all of whose continuous solutions are of the form g(x) = ax for some real a. Moreover, g(x) ∈ (−π , π ) implies |a| ≤ π /2. 4.19 Longlisted Problems 1977 427 Therefore f (x) = tan ax for some |a| ≤ π /2, and this is indeed a solution to the given equation. 25. Let   n fn (z) = z + a ∑ (a − kb)k−1 (z + kb)n−k . k=1 k n n We shall prove by induction on n that fn (z) = (z + a)n . This is trivial for n = 1. Suppose that the statement is true for some positive integer n − 1. Then n−1   n (n − k)(a − kb)k−1(z + kb)n−k−1 k k=1  n−1  n−1 = nzn−1 + na ∑ (a − kb)k−1 (z + kb)n−k−1 k k=1 fn′ (z) = nzn−1 + a ∑ = n fn−1 (z) = n(z + a)n−1 . It remains to prove that fn (−a) = 0. For z = −a we have by the lemma of (SL8113), n   n n fn (−a) = (−a) + a ∑ (−1)n−k (a − kb)n−1 k=1 k n   n =a∑ (−1)n−k (a − kb)n−1 = 0. k k=0 26. The result is an immediate consequence (for G = {−1, 1}) of the following generalization. (1) Let G be a proper subgroup of Z∗n (the multiplicative group of residue classes modulo n coprime to n), and let V be the union of elements of G. A number m ∈ V is called indecomposable in V if there do not exist numbers p, q ∈ V , p, q 6∈ {−1, 1}, such that pq = m. There exists a number r ∈ V that can be expressed as a product of elements indecomposable in V in more than one way. First proof. We shall start by proving the following lemma. Lemma. There are infinitely many primes not in V that do not divide n. Proof. There is at least one such prime: In fact, any number other than ±1 not in V must have a prime factor not in V , since V is closed under multiplication. If there were a finite number of such primes, say p1 , p2 , . . . , pk , then one of the numbers p1 p2 · · · pk + n, p21 p2 · · · pk + n is not in V and is coprime to n and p1 , . . . , pk , which is a contradiction. [This lemma is actually a direct consequence of Dirichlet’s theorem.] Let us consider two such primes p, q that are congruent modulo n. Let pk be the least power of p that is in V . Then pk , qk , pk−1 q, pqk−1 belong to V and are indecomposable in V . It follows that r = pk · qk = pk−1 q · pqk−1 428 4 Solutions has the desired property. Second proof. Let p be any prime not in V that does not divide n, and let pk be the least power of p that is in V . Obviously pk is indecomposable in V . Then the number r = pk · (pk−1 + n)(p + n) = p(pk−1 + n) · pk−1(p + n) has at least two different factorizations into indecomposable factors. 27. The result is a consequence of the generalization from the previous problem for G = {1}. Remark. There is an explicit example: r = (n − 1)2 · (2n − 1)2 = [(n − 1)(2n − 1)]2 . 28. The recurrent relations give us that     √ xi + [n/xi] xi + n/xi xi+1 = = ≥ [ n]. 2 2 √ On the other hand, if xi > [ n] for some i, then we have xi+1 < xi . This follows 2 from the fact that √ xi+1 < xi is equivalent to xi > (x √i + n/xi)/2, i.e., to xi > n. Therefore xi = [ n] holds for at least one i ≤ n − [ n] + 1. √ √ Remark. If n + 1√is a perfect square,√then xi = [ n] implies xi+1 = [ n] + 1. Otherwise, xi = [ n] implies xi+1 = [ n]. 29. Let us denote the midpoints of segments LM, AN, BL, MN, BK, CM, NK, CL, DN, KL, DM, AK by P1 , P2 , P3 , P4 , P5 , P6 , P7 , P8 , P9 , P10, P11 , P12 , respectively. D C We shall prove that the dodecagon P1 P2 P3 . . . P11 P12 is regular. From BL = BA and ∠ABL = 30◦ it follows that ∠BAL = 75◦ . Similarly O L N ∠DAM = 75◦ , and therefore ∠LAM = ◦ 60 , which together with the fact P1 AL = AM implies that △ALM is P2 equilateral. Now, from the triangles M A B OLM and ALN, we deduce OP1 = LM/2, OP2 = AL/2 and OP2 k AL. Hence OP1 = OP2 , ∠P1 OP2 = ∠P1 AL = 30◦ and ∠P2 OM = ∠LAD = 15◦ . The desired result follows from symmetry. 30. Suppose ∠SBA = x. By the trigonometric form of Ceva’s theorem we have sin(96◦ − x) sin 18◦ sin 6◦ = 1. sin x sin 12◦ sin 48◦ (1) We claim that x = 12◦ is a solution of this equation. To prove this, it is enough to show that sin 84◦ sin 6◦ sin 18◦ = sin 48◦ sin 12◦ sin 12◦ , which is equivalent to sin 18◦ = 2 sin 48◦ sin 12◦ = cos 36◦ − cos60◦ . The last equality can be checked directly. 4.19 Longlisted Problems 1977 429 Since the equation is equivalent to (sin 96◦ cot x − cos96◦ ) sin 6◦ sin 18◦ = sin 48◦ sin 12◦ , the solution x ∈ [0, π ) is unique. Hence x = 12◦ . Second solution. We know that if a, b, c, a′ , b′ , c′ are points on the unit circle in the complex plane, the lines aa′ , bb′ , cc′ are concurrent if and only if (a − b′)(b − c′ )(c − a′) = (a − c′)(b − a′)(c − b′). (1) We shall prove that x = 12◦ . We may suppose that ABC is the triangle in the π π complex plane with vertices a = 1, b = ε 9 , c = ε 14 , where ε = cos 15 + i sin 15 . ′ 12 ′ 28 ′ ′ ′ If a = ε , b = ε , c = ε , our task is the same as proving that lines aa , bb , cc′ are concurrent, or by (1) that (1 − ε 28)(ε 9 − ε )(ε 14 − ε 12 ) − (1 − ε )(ε 9 − ε 12 )(ε 14 − ε 28 ) = 0. The last equality holds, since the left-hand side is divisible by the minimum polynomial of ε : z8 + z7 − z5 − z4 − z3 + z + 1. 31. We obtain from (1) that f (1, c) = f (1, c) f (1, c); hence f (1, c) = 1 and consequently f (−1, c) f (−1, c) = f (1, c) = 1, i.e. f (−1, c) = 1. Analogously, f (c, 1) = f (c, −1) = 1. Clearly f (1, 1) = f (−1, 1) = f (1, −1) = 1. Now let us assume that a 6= 1. Observe that f (x−1 , y) = f (x, y−1 ) = f (x, y)−1 . Thus by (1) and (2) we get 1 = f (a, 1 − a) f (1/a, 1 − 1/a)     1 1−a = f (a, 1 − a) f a, = f a, = f (a, −a). 1 − 1/a 1 − 1/a We now have f (a, a) = f (a, −1) f (a, −a) = 1 · 1 = 1 and 1 = f (ab, ab) = f (a, ab) f (b, ab) = f (a, a) f (a, b) f (b, a) f (b, b) = f (a, b) f (b, a). 32. It is a known result that among six persons there are 3 mutually acquainted or 3 mutually unacquainted. By the condition of the problem the last case is excluded. If there is a man in the room who is not acquainted with four of the others, then these four men are mutually acquainted. Otherwise, each man is acquainted with at least five others, and since the sum of numbers of acquaintances of all men in the room is even, one of the men is acquainted with at least six men. Among these six there are three mutually acquainted, and they together with the first one make a group of four mutually acquainted men. √ 33. Let r be the radius of K and s > 2/r an integer. Consider the points Ak (ka1 − [ka1 ], ka2 − [ka2 ]), where k = 0, 1, 2, . . . , s2 . Since all these points are in the unit square, two of them, say √ A p , Aq , q > p, are in a small square with side 1/s, and consequently A p Aq ≤ 2/s < r. Therefore, for n = q − p, m1 = [qa1 ] − [pa1 ] and m2 = [qa2 ] − [pa2 ] the distance between the points n(a1 , a2 ) and (m1 , m2 ) is less then r, i.e., the point (m1 , m2 ) is in the circle K + n(a1 , a2 ). 430 4 Solutions 34. Let A be the set of the 2n sequences of n terms equal to ±1. Since there are k2 products ab with a, b ∈ B, by the pigeonhole principle there exists c ∈ A such that ab = c holds for at most k2 /2n pairs (a, b) ∈ B × B. Then cb ∈ B holds for at most k2 /2n values b ∈ B, which means that |B ∩ cB| ≤ k2 /2n . 35. The solutions are 0 and Nk = 10 99 . . .9} 89, where k = 0, 1, 2, . . .. | {z k Remark. If we omit the condition that at most one of the digits is zero, the solutions are numbers of the form Nk1 Nk2 . . . Nkr , where k1 = kr , k2 = kr−1 etc. The more general problem k ·a1 a2 . . . an = an . . . a2 a1 has solutions only for k = 9 and for k = 4 (namely 0, 2199 . . .978 and combinations as above). 36. It can be shown by simple induction that Sm (a1 , . . . , a2n ) = (b1 , . . . , b2n ), where m (mi) bk = ∏ ak+i (assuming that ak+2n = ak ). i=0 If we take m = 2n all the binomial coefficients will be even, and thus bk = ak ak+m = 1 for all k. m i apart from i = 0 and i = m 37. We look for a solution with xA1 1 = · · · = xAn n = nA1 A2 ···An x and xn+1 = ny . In order for this to be a solution we must have A1 A2 · · · An x + 1 = An+1 y. This equation has infinitely many solutions (x, y) in N, since A1 A2 · · · An and An+1 are coprime. 38. The condition says that the quadratic equation f (x) = 0 has distinct real solutions, where n n n j=1 j=1 j=1 f (x) = 3x2 ∑ m j − 2x ∑ m j (a j + b j + c j ) + ∑ m j (a j b j + b j c j + c j a j ). It is easy to verify that the function f is the derivative of n F(x) = ∑ m j (x − a j )(x − b j )(x − c j ). j=1 Since F(a1 ) ≤ 0 ≤ F(an ), F(b1 ) ≤ 0 ≤ F(bn ) and F(c1 ) ≤ 0 ≤ F(cn ), F(x) has three distinct real roots, and hence by Rolle’s theorem its derivative f (x) has two distinct real roots. 39. By the pigeonhole principle, we can find 5 distinct points among the given 37 such that their x-coordinates are congruent and their y-coordinates are congruent modulo 3. Now among these 5 points either there exist three with z-coordinates congruent modulo 3, or there exist three whose z-coordinates are congruent to 0, 1, 2 modulo 3. These three points are the desired ones. Remark. The minimum number n such that among any n integer points in space one can find three points whose barycenter is an integer point is n = 19. Each proof of this result seems to consist in studying a great number of cases. 4.19 Longlisted Problems 1977 431 40. Let us divide the chessboard into 16 squares Q1 , Q2 , . . . , Q16 of size 2 × 2. Let sk be the sum of numbers in Qk , and let us assume that s1 ≥ s2 ≥ · · · ≥ s16 . Since s4 + s5 + · · · + s16 ≥ 1 + 2 + · · · + 52 = 1378, we must have s4 ≥ 100 and hence s1 , s2 , s3 ≥ 100 as well. 41. The considered sums are congruent modulo N to Sk = ∑Ni=1 (i + k)ai , k = 0, 1, . . . , N − 1. Since Sk = S0 + k(a1 + · · · + an ) = S0 + k, all these sums give distinct residues modulo N and therefore are distinct. 42. It can be proved by induction on n that {an,k | 1 ≤ k ≤ 2n } = {2m | m = 3n + 3n−1 s1 + · · · + 31 sn−1 + sn (si = ±1)}. Thus the result is an immediate consequence of the following lemma. Lemma. Each positive integer s can be uniquely represented in the form s = 3n + 3n−1s1 + · · · + 31 sn−1 + sn , where si ∈ {−1, 0, 1}. (1) Proof. Both the existence and the uniqueness can be shown by simple induction on s. The statement is trivial for s = 1, while for s > 1 there exist q ∈ N, r ∈ {−1, 0, 1} such that s = 3q + r, and q has a unique representation of the form (1). h i  k+p+1 k+p 43. Since k(k + 1) · · · (k + p) = (p + 1)! k+p = (p + 1)! − p+1 p+2 p+2 , it follows that   n n+ p+1 n(n + 1) · · ·(n + p + 1) k(k + 1) · · · (k + p) = (p + 1)! = . ∑ p + 2 p+2 k=1 44. Let d(X , σ ) denote the distance from a point X to a plane σ . Let us consider the pair (A, π ) where A ∈ E and π is a plane containing some three points B,C, D ∈ E such that d(A, π ) is the smallest possible. We may suppose that B,C, D are selected such that △BCD contains no other points of E. Let A′ be the projection of A on π , and let lb , lc , ld be lines through B,C, D parallel to CD, DB, BC respectively. If A′ is in the half-plane determined by ld not containing BC, then d(D, ABC) ≤ d(A′ , ABC) < d(A, BCD), which is impossible. Similarly, A′ lies in the half-planes determined by lb , lc that contain D, and hence A′ is inside the triangle bordered by lb , lc , ld . The minimality property of (A, π ) and the way in which BCD was selected guarantee that E ∩ T = {A, B,C, D}. 45. As in the previous problem, let us choose the pair (A, π ) such that d(A, π ) is minimal. If π contains only three points of E, we are done. If not, there are four points in E ∩ P, say A1 , A2 , A3 , A4 , such that the quadrilateral Q = A1 A2 A3 A4 contains no other points of E. Suppose Q is not convex, and that w.l.o.g. A1 is inside the triangle A2 A3 A4 . If A0 is the projection of A on P, the point A1 belongs to one of the triangles A0 A2 A3 , A0 A3 A4 , A0 A4 A2 , say A0 A2 A3 . Then d(A1 , AA2 A3 ) ≤ d(A0 , AA2 A3 ) < AA0 , which is impossible. Hence Q is convex. Also, by the minimality property of (A, π ) the pyramid AA1 A2 A3 A4 contains no other points of E. 432 4 Solutions 46. We need to consider only the case t > |x|. There is no loss of generality in assuming x > 0. To obtain the estimate from below, set     x+t x+t a1 = f − − f (−(x + t)), a2 = f (0) − f − , 2  2  x+t x+t a3 = f − f (0), a4 = f (x + t) − f . 2 2 Since −(x + t) < x −t and x < (x + t)/2, we have f (x) − f (x − t) ≤ a1 + a2 + a3 . Since 2−1 < a j+1 /a j < 2, it follows that g(x,t) > a4 a3 /2 > = 14−1 . a1 + a2 + a3 4a3 + 2a3 + a3 To obtain the estimate from above, set   x+t b1 = f (0) − f − ,  3   2(x + t) x+t b3 = f −f , 3 3 b2 = f  x+t 3  − f (0),   2(x + t) b4 = f (x + t) − f . 3 If t < 2x, then x − t < −(x + t)/3 and therefore f (x) − f (x − t) ≥ b1 . If t ≥ 2x, then (x + t)/3 ≤ x and therefore f (x) − f (x − t) ≥ b2 . Since 2−1 < b j+1 /b j < 2, we get b2 + b3 + b4 b2 + 2b2 + 4b2 g(x,t) < < = 14. min{b1 , b2 } b2 /2 47. M lies on AB and N lies on BC. If CQ ≤ 2CD/3, then BM = CQ/2. If CQ > 2CD/3, then N coincides with C. 48. Let a plane cut the edges AB, BC,CD, DA at points K, L, M, N respectively. Let D′ , A′ , B′ be distinct points in the plane ABC such that the triangles BCD′ , CD′ A′ , D′ A′ B′ are equilateral, and B D′ B′ M ′ ∈ [CD′ ], N ′ ∈ [D′ A′ ], and K ′ ∈ L [A′ B′ ] such that CM′ = CM, A′ N ′ = M′ AN, and A′ K ′ = AK. The perimeter K′ K N′ P of the quadrilateral KLMN is equal to the length of the polygonal line A C A′ KLM′ N ′ K ′ , which is not less than KK ′ . It follows that P ≥ 2a. Let us consider all quadrilaterals KLMN that are obtained by intersecting the tetrahedron by a plane parallel to a fixed plane α . The lengths of the segments KL, LM, MN, NK are linear functions in AK, and so is P. Thus P takes its maximum at an endpoint of the interval, i.e., when the plane KLMN passes through one of the vertices A, B,C, D, and it is easy to see that in this case P ≤ 3a. 4.19 Longlisted Problems 1977 433 49. If one of p, q, say p, is zero, then −q is a perfect square. Conversely, (p, q) = (0, −t 2 ) and (p, q) = (−t 2 , 0) satisfy the conditions for t ∈ Z. We now assume that p, q are nonzero. If the trinomial x2 + px + q has two integer roots x1 , x2 , then |q| = |x1 x2 | ≥ |x1 | + |x2 | − 1 ≥ |p| − 1. Similarly, if x2 + qx + p has integer roots, then |p| ≥ |q| − 1 and q2 − 4p is a square. Thus we have two cases to investigate: (i) |p| = |q|. Then p2 − 4q = p2 ± 4p is a square, so (p, q) = (4, 4). (ii) |p| = |q| ± 1. The solutions for (p, q) are (t, −1 − t) for t ∈ Z and (5, 6), (6, 5). 50. Suppose that Pn (x) = n for x ∈ {x1 , x2 , . . . , xn }. Then Pn (x) = (x − x1 )(x − x2 ) · · · (x − xn ) + n. From Pn (0) = 0 we obtain n = |x1 x2 · · · xn | ≥ 2n−2 (because at least n − 2 factors are different from ±1) and therefore n ≥ 2n−2 . It follows that n ≤ 4. For each positive integer n ≤ 4 there exists a polynomial Pn . Here is the list of such polynomials: n = 1 : ±x, n = 3 : ±(x3 − x) + 3x2 , n = 2 : 2x2 , x2 ± x, −x2 ± 3x, n = 4 : −x4 + 5x2 . 51. We shall use the following algorithm: Choose a segment of maximum length (“basic” segment) and put on it unused segments of the opposite color without overlapping, each time of the maximum possible length, as long as it is possible. Repeat the procedure with remaining segments until all the segments are used. Let us suppose that the last basic segment is black. Then the length of the used part of any white basic segment is greater than the free part, and consequently at least one-half of the length of the white segments has been used more than once. Therefore all basic segments have total length at most 1.5 and can be distributed on a segment of length 1.51. On the other hand, if we are given two white segments of lengths 0.5 and two black segments of lengths 0.999 and 0.001, we cannot distribute them on a segment of length less than 1.499.   √ √ 52. The maximum and minimum are 2R 4 − 2k2 and 2R 1 + 1 − k2 respectively. 53. The discriminant of the given equation considered as a quadratic equation in b is 196 − 75a2. Thus 75a2 ≤ 196 and hence −1 ≤ a ≤ 1. Now the integer solutions of the given equation are easily found: (−1, 3), (0, 0), (1, 2). 54. We shall use the following lemma. Lemma. If a real function f is convex on the interval I and x, y, z ∈ I, x ≤ y ≤ z, then (y − z) f (x) + (z − x) f (y) + (x − y) f (z) ≤ 0. 434 4 Solutions Proof. The inequality is obvious for x = y = z. If x < z, then there exist p, r such that p + r = 1 and y = px + rz. Then by Jensen’s inequality f (px + rz) ≤ p f (x) + r f (z), which is equivalent to the statement of the lemma. By applying the lemma to the convex function − ln x we obtain xy yz zx ≥ yx zy xz for any 0 < x ≤ y ≤ z. Multiplying the inequalities ab bc ca ≥ ba cb ac and ac cd d a ≥ ca d c ad we get the desired inequality. a Remark. Similarly, for 0 < a1 ≤ a2 ≤ · · · ≤ an it holds that aa12 a23 · · · aan1 ≥ aa21 aa32 · · · aa1n . 55. The statement is true without the assumption that O ∈ BD. Let BP ∩ DN = {K}. − → −→ −→ If we denote AB = a, AD = b and AO = α a+ β b for some α , β ∈ R, 1/α +1/β 6= 1, by straightforward calculation we obtain that −→ AK = α β 1 −→ a+ b= AO. α + β − αβ α + β − αβ α + β − αβ Hence A, K, O are collinear. 56. See the solution to (LL67-36). 57. Suppose that there exists a sequence of 17 terms a1 , a2 , . . . , a17 satisfying the required conditions. Then the sum of terms in each row of the rectangular array below is positive, while the sum of terms in each column is negative, which is a contradiction. a1 a2 . . . a11 a2 a3 . . . a12 .. .. .. . . . a7 a8 . . . a17 On the other hand, there exist 16-term sequences with the required property. An example is 5, 5, −13, 5, 5, 5, −13, 5, 5, −13, 5, 5, 5, −13, 5, 5 which can be obtained by solving the system of equations ∑k+10 i=k ai = 1 (k = 1, 2, . . . , 6) and a = −1 (l = 1, 2, . . . , 10). ∑l+6 i i=l Second solution. We shall prove a stronger statement: If 7 and 11 in the question are replaced by any positive integers m, n, then the maximum number of terms is m + n − (m, n) − 1. Let a1 , a2 , . . . , al be a sequence of real numbers, and let us define s0 = 0 and sk = a1 + · · · + ak (k = 1, . . . , l). The given conditions are equivalent to sk > sk+m for 0 ≤ k ≤ l − m and sk < sk+n for 0 ≤ k ≤ l − n. Let d = (m, n) and m = m′ d, n = n′ d. Suppose that there exists a sequence (ak ) of length greater than or equal to l = m + n − d satisfying the required conditions. Then the m′ + n′ numbers s0 , sd , . . . , s(m′ +n′ −1)d satisfy n′ inequalities sk+m < sk and m′ inequalities sk < sk+n . Moreover, each term skd appears twice in these inequalities: once on the left-hand and once on the right-hand side. It follows that there exists a ring of inequalities si1 < si2 < · · · < sik < si1 , giving a contradiction. On the other hand, suppose that such a ring of inequalities can be made also for l = m + n − d − 1, say si1 < si2 < · · · < sik < si1 . If there are p inequalities of 4.19 Longlisted Problems 1977 435 the form ak+m < ak and q inequalities of the form ak+n > ak in the ring, then qn = rm, which implies m′ | q, n′ | p and thus k = p + q ≥ m′ + n′ . But since all i1 , i2 , . . . , ik are congruent modulo d, we have k ≤ m′ + n′ − 1, a contradiction. Hence there exists a sequence of length m + n − d − 1 with the required property. 58. The following inequality (Finsler and Hadwiger, 1938) is sharper than the one we have to prove: √ 2ab + 2bc + 2ca − a2 − b2 − c2 ≥ 4S 3. (1) First proof. Let us set 2x = b + c − a, 2y = c + a − b, 2z = a + b − c. Then x, y, z > 0 and the inequality (1) becomes y2 z2 + z2 x2 + x2 y2 ≥ xyz(x + y + z), which is equivalent to the obvious inequality (xy − yz)2 + (yz − zx)2 + (zx − xy)2 ≥ 0. Second proof. Using the known relations for a triangle a2 + b2 + c2 = 2s2 − 2r2 − 8rR, ab + bc + ca = s2 + r2 + 4rR, S = rs, where r and R are the radii of the incircle and the circumcircle, s the semiperimeter and S the area, we can transform (1) into √ s 3 ≤ 4R + r. The last inequality is a consequence of the inequalities 2r ≤ R and s2 ≤ 4R2 + 4Rr + 3r2 , where the last one follows from the equality HI 2 = 4R2 + 4Rr + 3r2 − s2 (H and I being the orthocenter and the incenter of the triangle). 59. Let us consider the set R of pairs of coordinates of the points from E reduced modulo 3. If some element of R occurs thrice, then the corresponding points are vertices of a triangle with integer barycenter. Also, no three elements from E can have distinct x-coordinates and distinct y-coordinates. By an easy discussion we can conclude that the set R contains at most four elements. Hence |E| ≤ 8. An example of a set E consisting of 8 points that satisfies the required condition is E = {(0, 0), (1, 0), (0, 1), (1, 1), (3, 6), (4, 6), (3, 7), (4, 7)}. 60. By Lagrange’s interpolation formula we have n F(x) = ∏i6= j (x − x j ) ∑ F(x j ) ∏i6= j (xi − x j ) . j=0 Since the leading coefficient in F(x) is 1, it follows that 436 4 Solutions n 1= F(x j ) ∑ ∏i6= j (xi − x j ) . j=0 Since ∏(xi − x j ) i6= j j−1 = ∏ |xi − x j | i=0 n ∏ i= j+1 |xi − x j | ≥ j!(n − j)!, we have n 1≤ ∑ j=0 |F(x j )| 1 ≤ n! ∏i6= j (xi − x j )   n 2n |F(x )| ≤ max|F(x j )|. j ∑ n! j=0 j n Now the required inequality follows immediately. 4.20 Shortlisted Problems 1978 437 4.20 Solutions to the Shortlisted Problems of IMO 1978 1. There exists an Ms that contains at least 2n/k = 2(k2 + 1) elements. It follows that Ms contains either at least k2 + 1 even numbers or at least k2 + 1 odd numbers. In the former case, consider the predecessors of those k2 + 1 numbers: 2 +1 among them, at least kk+1 > k, i.e., at least k + 1, belong to the same subset, say Mt . Then we choose s,t. The latter case is similar. Second solution. For all i, j ∈ {1, 2, . . . , k}, consider the set Ni j = {r | 2r ∈ Mi , 2r − 1 ∈ M j }. Then {Ni j | i, j} is a partition of {1, 2, . . ., n} into k2 subsets. For n ≥ k3 + 1 one of these subsets contains at least k + 1 elements, and the statement follows. Remark. The statement is not necessarily true when n = k3 . 2. Consider the transformation φ of the plane defined as the homothety H with center B and coefficient 2 followed by the rotation R about the center O through O an angle of 60◦ . Being direct, this mapping must be a rotational homoN A thety. We also see that H maps S B′ S into the point symmetric to S with A′ respect to OA, and R takes it back to S. Hence S is a fixed point, and M is consequently also the center of B φ . Therefore φ is the rotational ho◦ mothety about S with the angle 60 and coefficient 2. (In fact, this could also be seen from the fact that φ preserves angles of triangles and maps the segment SR onto SB, where R is the midpoint of AB.) Since φ (M) = B′ , we conclude that ∠MSB′ = 60◦ and SB′ /SM = 2. Similarly, ∠NSA′ = 60◦ and SA′ /SN = 2, so triangles MSB′ and NSA′ are indeed similar. Second solution. Probably the simplest way here is using complex numbers. Put the origin at O and complex numbers a, a′ at points A, A′ , and denote the primitive sixth root of 1 by ω . Then the numbers at B, B′ , S and N are ω a, ω a′ , (a + ω a)/3, and (a + ω a′ )/2 respectively. Now it is easy to verify that (n − s) = ω (a′ − s)/2, i.e., that ∠NSA′ = 60◦ and SA′ /SN = 2. 3. What we need are m, n for which 1978m(1978n−m − 1) is divisible by 1000 = 8 · 125. Since 1978n−m − 1 is odd, it follows that 1978m is divisible by 8, so m ≥ 3. Also, 1978n−m − 1 is divisible by 125, i.e., 1978n−m ≡ 1 (mod 125). Note that 1978 ≡ −2 (mod 5), and consequently also −2n−m ≡ 1. Hence 4 | n−m = 4k, k ≥ 1. It remains to find the least k such that 19784k ≡ 1 (mod 125). Since 19784 ≡ (−22)4 = 4842 ≡ (−16)2 = 256 ≡ 6, we reduce it to 6k ≡ 1. Now 6k = (1+5)k ≡ 1 + 5k + 25 2k (mod 125), which reduces to 125 | 5k(5k − 3). But 5k − 3 is not divisible by 5, and so 25 | k. Therefore 100 | n − m, and the desired values are m = 3, n = 103. 438 4 Solutions 4. Let γ , ϕ be the angles of T1 and T2 opposite to c and w respectively. By the cosine theorem, the inequality is transformed into a2 (2v2 − 2uv cos ϕ ) + b2 (2u2 − 2uv cos ϕ ) +2(a2 + b2 − 2ab cos γ )uv cos ϕ ≥ 4abuv sin γ sin ϕ . This is equivalent to 2(a2 v2 + b2 u2 ) − 4abuv(cos γ cos ϕ + sin γ sin ϕ ) ≥ 0, i.e., to 2(av − bu)2 + 4abuv(1 − cos(γ − ϕ )) ≥ 0, which is clearly satisfied. Equality holds if and only if γ = ϕ and a/b = u/v, i.e., when the triangles are similar, a corresponding to u and b to v. 5. We first explicitly describe the elements of the sets M1 , M2 . x 6∈ M1 is equivalent to x = a + (a + 1) + · · ·+ (a + n − 1) = n(2a + n − 1)/2 for some natural numbers n, a, n ≥ 2. Among n and 2a + n − 1, one is odd and the other even, and both are greater than 1; so x has an odd factor ≥ 3. On the other hand, for every x with an odd divisor p > 3 it is easy to see that there exist corresponding a, n. Therefore M1 = {2k | k = 0, 1, 2, . . . }. x 6∈ M2 is equivalent to x = a + (a + 2) + · · · + (a + 2(n − 1)) = n(a + n − 1), where n ≥ 2, i.e. to x being composite. Therefore M2 = {1} ∪ {p | p = prime}. x 6∈ M3 is equivalent to x = a + (a + 3) + · · · + (a + 3(n − 1)) = n(2a + 3(n − 1))/2. It remains to show that every c ∈ M3 can be written as c = 2k p with p prime. Suppose the opposite, that c = 2k pq, where p, q are odd and q ≥ p ≥ 3. Then there exist positive integers a, n (n ≥ 2) such that c = n(2a + 3(n − 1))/2 and hence c 6∈ M3 . Indeed, if k = 0, then n = 2 and 2a + 3 = pq work; otherwise, setting n = p one obtains a = 2k q − 3(p − 1)/2 ≥ 2q − 3(p − 1)/2 ≥ (p + 3)/2 > 1. 6. For fixed n and the set {ϕ (1), . . . , ϕ (n)}, there are finitely many possibilities for a mapping ϕ on {1, . . . , n}. Suppose ϕ is the one among these for which ∑nk=1 ϕ (k)/k2 is minimal. If i < j and ϕ (i) > ϕ ( j) for some i, j ∈ {1, . . . , n}, define ψ as ψ (i) = ϕ ( j), ψ ( j) = ϕ (i), and ψ (k) = ϕ (k) for all other k. Then     ϕ (k) ψ (k) ϕ (i) ϕ ( j) ϕ (i) ϕ ( j) ∑ k2 − ∑ k2 = i2 + j2 − j2 + i2 i+ j = (i − j)(ϕ ( j) − ϕ (i)) 2 2 > 0, i j which contradicts the assumption. This shows that ϕ (1) < · · · < ϕ (n), and consequently ϕ (k) ≥ k for all k. Hence n n n ϕ (k) k 1 ≥∑ 2 =∑ . 2 k k k=1 k=1 k=1 k ∑ 4.20 Shortlisted Problems 1978 439 7. Let x = OA, y = OB, z = OC, p α = ∠BOC, β = ∠COA, γ = ∠AOB. The conditions yield the equation x + y + x2 + y2 − 2xy cos γ = 2p, which transforms to (2p − x − y)2 = x2 + y2 − 2xy cos γ , i.e. (p − x)(p − y) = xy(1 − cos γ ). Thus p−x p−y · = 1 − cos γ , x y p−z p−z p−x and analogously p−y y · z = 1 − cos α , z · x = 1 − cos β . Setting u = p−z v = p−y y , w = z , the above system becomes uv = 1 − cos γ , vw = 1 − cos α , p−x x , wu = 1 − cos β . This system has a unique solution in positive real numbers u, v, w: s r (1 − cos β )(1 − cos γ ) (1 − cos γ )(1 − cos α ) u= , v= , 1 − cos α 1 − cos β s (1 − cos α )(1 − cos β ) w= . 1 − cos γ Finally, the values of x, y, z are uniquely determined from u, v, w. Remark. It is not necessary that the three lines be in the same plane. Also, there could be any odd number of lines instead of three. 8. Take the subset {ai } = {1, 7, 11, 13, 17, 19, 23, 29, . . ., 30m − 1} of S containing all the elements of S that are not multiples of 3. There are 8m such elements. Every element in S can be uniquely expressed as 3t ai for some i and t ≥ 0. In a subset of S with 8m + 1 elements, two of them will have the same ai , hance one will divide the other. On the other hand, for each i = 1, 2, . . . , 8m choose t ≥ 0 such that 10m < bi = 3t ai < 30m. Then there are 8m bi ’s in the interval (10m, 30m), and the quotient of any two of them is less than 3, so none of them can divide any other. Thus the answer is 8m. 9. Since the nth missing number (gap) is f ( f (n)) + 1 and f ( f (n)) is a member of the sequence, there are exactly n − 1 gaps less than f ( f (n)). This leads to f ( f (n)) = f (n) + n − 1. (1) Since 1 is not a gap, we have f (1) = 1. The first gap is f ( f (1)) + 1 = 2. Two consecutive integers cannot both be gaps (the predecessor of a gap is of the form f ( f (m))). Now we deduce f (2) = 3; a repeated application of the formula above gives f (3) = 3 +1 = 4, f (4) = 4 +2 = 6, f (6) = 9, f (9) = 14, f (14) = 22, f (22) = 35, f (35) = 56, f (56) = 90, f (90) = 145, f (145) = 234, f (234) = 378. Also, f ( f (35)) + 1 = 91 is a gap, so f (57) = 92. Then by (1), f (92) = 148, f (148) = 239, f (239) = 386. Finally, here f ( f (148)) + 1 = 387 is a gap, so f (240) = 388. 440 4 Solutions Second solution. As above, we arrive at formula (1). Then by simple induction it follows that f (Fn + 1) = Fn+1 + 1, where Fk is the Fibonacci sequence (F1 = F2 = 1). We now prove by induction (on n) that f (Fn + x) = Fn+1 + f (x) for all x with 1 ≤ x ≤ Fn−1 . This is trivially true for n = 0, 1. Supposing that it holds for n − 1, we shall prove it for n: (i) If x = f (y) for some y, then by the inductive assumption and (1) f (Fn + x) = f (Fn + f (y)) = f ( f (Fn−1 + y)) = Fn + f (y) + Fn−1 + y − 1 = Fn+1 + f (x). (ii) If x = f ( f (y)) + 1 is a gap, then f (Fn + x − 1) + 1 = Fn+1 + f (x − 1) + 1 is a gap also: Fn+1 + f (x) + 1 = Fn+1 + f ( f ( f (y))) + 1 = f (Fn + f ( f (y))) + 1 = f ( f (Fn−1 + f (y))) + 1. It follows that f (Fn + x) = Fn+1 + f (x − 1) + 2 = Fn+1 + f (x). Now, since we know that each positive integer x is expressible as x = Fk1 + Fk2 + · · · + Fkr , where 0 < kr 6= 2, ki ≥ ki+1 + 2, we obtain f (x) = Fk1 +1 + Fk2 +1 + · · · + Fkr +1 . Particularly, 240 = 233 + 5 + 2, so f (240) = 377 + 8 + 3 = 388. √ Remark. It can be shown that f (x) = [α x], where α = (1 + 5)/2. 10. Assume the opposite. One of the countries, say A, contains at least 330 members a1 , a2 , . . . , a330 of the society ( 6 ·329 = 1974). Consider the differences a330 − ai , i = 1, 2, . . . , 329: the members with these numbers are not in A, so at least 66 of them, a330 − ai1 , . . . , a330 − ai66 , belong to the same country, say B. Then the differences (ai66 − a330) − (ai j − a330 ) = ai66 − ai j , j = 1, 2, . . . , 65, are neither in A nor in B. Continuing this procedure, we find that 17 of these differences are in the same country, say C, then 6 among 16 differences of themselves in a country D, and 3 among 5 differences of themselves in E; finally, two differences of these 3 differences belong to country F, so that the difference of themselves cannot be in any country. This is a contradiction. Remark. The following stronger ([6!e] = 1957) statement can be proved in the same way. Schur’s lemma. If n is a natural number and e the logarithm base, then for every partition of the set {1, 2, . . . , [en!]} into n subsets one of these subsets contains some two elements and their difference. 11. Set F(x) = f1 (x) f2 (x) · · · fn (x): we must prove concavity of F 1/n . By the assumption, n F(θ x + (1 − θ )y) ≥ ∏[θ fi (x) + (1 − θ ) f (y)] i=1 n = ∑ θ k (1 − θ )n−k ∑ fi1 (x) . . . fik (x) fik+1 (y) fin (y), k=0 4.20 Shortlisted Problems 1978 441 n where the second sum goes through all k k-subsets {i1 , . . . , ik } of {1, . . . , n}. The inequality between the arithmetic and geometric means now gives us   n f (x) f (x) · · · f (x) f (y) f (y) ≥ F(x)k/n F(y)(n−k)/n . ik ik+1 in ∑ i1 i2 k Inserting this in the above inequality and using the binomial formula, we finally obtain   n k n−k n F(θ x + (1 − θ )y) ≥ ∑ θ (1 − θ ) F(x)k/n F(y)(n−k)/n k k=0  n = θ F(x)1/n + (1 − θ )F(y)1/n , which proves the assertion. 12. Let O be the center of the smaller circle, T its contact point with the circumcircle of ABC, and J the midpoint of segment BC. The figure is symmetric with respect to the line through A, O, J, T . A homothety centered at A taking T into J will take the smaller circle into the incircle of ABC, hence will take O into the incenter I. On the other hand, ∠ABT = ∠ACT = 90◦ implies that the quadrilaterals ABTC and APOQ are similar. Hence the above homothety also maps O to the midpoint of PQ. This finishes the proof. Remark. The assertion is true for a nonisosceles triangle ABC as well, and this (more difficult) case is a matter of SL93-3. 13. Lemma. If MNPQ is a rectangle and O any point in space, then OM2 + OP2 = ON 2 + OQ2 . Proof. Let O1 be the projection of O onto MNPQ, and m, n, p, q denote the distances of O1 from MN, NP, PQ, QM, respectively. Then OM2 = OO21 + q2 + m2 , ON 2 = OO21 + m2 + n2 , OP2 = OO21 + n2 + p2 , OQ2 = OO21 + p2 + q2 , and the lemma follows immediately. Now we return to the problem. Let O be the center of the given sphere S, and X the point opposite P in the face of the parallelepiped through P, A, B. By the lemma, we have OP2 + OQ2 = OC2 + OX 2 and OP2 +√OX 2 = OA2 +OB2 . Hence 2OP2 + OQ2 = OA2 + OB2 + OC2 = 3R2 , i.e. OQ = √3R2 − OP2 > R. We claim that the locus of Q is the whole sphere (O, 3R2 − OP2 ). Choose any point Q on this sphere. Since OQ > R > OP, the sphere with diameter PQ intersects S on a circle. Let C be an arbitrary point on this circle, and X the point opposite C in the rectangle PCQX. By the lemma, OP2 + OQ2 = OC2 + OX 2 , hence OX 2 = 2R2 − OP2 > R2 . The plane passing through P and perpendicular to PC intersects S in a circle γ ; both P, X belong to this plane, P being inside and X outside the circle, so that the circle with diameter PX intersects γ at some point B. Finally, we choose A to be the point opposite B in the rectangle PBX A: we deduce that OA2 + OB2 = OP2 + OX 2 , and consequently A ∈ S. By the construction, there is a rectangular parallelepiped through P, A, B,C, X, Q. 442 4 Solutions 14. We label the cells of the cube by (a1 , a2 , a3 ), ai ∈ {1, 2, . . . , 2n + 1}, in a natural way: for example, as Cartesian coordinates of centers of the cells ((1, 1, 1) is one corner, etc.). Notice that there should be (2n+1)3 −2n(2n+1)·2(n+1) = 2n+1 void cells, i.e., those not covered by any piece of soap. n = 1. In this case, six pieces of soap 1 × 2 × 2 can be placed on the following positions: [(1, 1, 1), (2, 2, 1)], [(3, 1, 1), (3, 2, 2)], [(2, 3, 1), (3, 3, 2)] and the symmetric ones with respect to the center of the box. (Here [A, B] denotes the rectangle with opposite corners at A, B.) n is even. Each of the 2n + 1 planes Pk = {(a1 , a2 , k) | ai = 1, . . . , 2n + 1} can receive 2n pieces of soap: In fact, Pk can be partitioned into four n × (n + 1) rectangles at the corners and the central cell, while an n × (n + 1) rectangle can receive n/2 pieces of soap. n is odd, n > 1. Let us color a cell (a1 , a2 , a3 ) blue, red, or yellow if exactly three, two or one ai respectively is equal to n + 1. Thus there are 1 blue, 6n red, and 12n2 yellow cells. We notice that each piece of soap must contain at least one colored cell (because 2(n + 1) > 2n + 1). Also, every piece of soap contains an even number (actually, 1 · 2, 1(n + 1), or 2(n + 1)) of cells in Pk . On the other hand, 2n + 1 cells are void, i.e., one in each plane. There are several cases for a piece of soap S: (i) S consists of 1 blue, n + 1 red and n yellow cells; (ii) S consists of 2 red and 2n yellow cells (and no blue cells); (iii) S contains 1 red cell, n + 1 yellow cells, and the rest are uncolored; (iv) S contains 2 yellow cells and no blue or red ones. From the descriptions of the last three cases, we can deduce that if S contains r red cells and no blue, then it contains exactly 2 + (n − 1)r red ones. (∗) Now, let B1 , . . . , Bk be all boxes put in the cube, with a possible exception for the one covering the blue cell: thus k = 2n(2n + 1) if the blue cell is void, or k = 2n(2n + 1) − 1 otherwise. Let ri and yi respectively be the numbers of red and yellow cells inside Bi . By (∗) we have y1 + · · · + yk = 2k + (n − 1)(r1 + · · · + rk ). If the blue cell is void, then r1 + · · · + rk = 6n and consequently y1 + · · ·+yk = 4n(2n + 1)+ 6n(n − 1) = 14n2 − 2n, which is impossible because there are only 12n2 < 14n2 − 2n yellow cells. Otherwise, r1 + · · · + rk ≥ 5n − 2 (because n + 1 red cells are covered by the box containing the blue cell, and one can be void) and consequently y1 + · · · + yk ≥ 4n(2n + 1) − 2 + (n − 1)(5n − 2) = 13n2 − 3n; since there are n more yellow cells in the box containing the blue one, this counts for 13n2 − 2n > 12n2 (n ≥ 3), again impossible. Remark. The following solution of the case n odd is simpler, but does not work for n = 3. For k = 1, 2, 3, let mk be the number of pieces whose long sides are perpendicular to the plane πk (ak = n + 1). Each of these mk pieces covers exactly 2 cells of πk , while any other piece covers n + 1, 2(n + 1), or none. It follows that 4n2 + 4n − 2mk is divisible by n + 1, and so is 2mk . This further implies that 4.20 Shortlisted Problems 1978 443 2m1 + 2m2 + 2m3 = 4n(2n + 1) is a multiple of n + 1, which is impossible for each odd n except n = 1 and n = 3. 15. Let Cn = {a1 , . . . , an } (C0 = 0) / and Pn = { f (B) | B ⊆ Cn }. We claim that Pn contains at least n + 1 distinct elements. First note that P0 = {0} contains one element. Suppose that Pn+1 = Pn for some n. Since Pn+1 ⊇ {an+1 + r | r ∈ Pn }, it follows that for each r ∈ Pn , also r + an+1 ∈ Pn . Then obviously 0 ∈ Pn implies kan+1 ∈ Pn for all k; therefore Pn = P has at least p ≥ n + 1 elements. Otherwise, if Pn+1 ⊃ Pn for all n, then |Pn+1 | ≥ |Pn | + 1 and hence |Pn | ≥ n + 1, as claimed. Consequently, |Pp−1 | ≥ p . (All the operations here are performed modulo p.) √ 16. Clearly |x| ≤ 1. As x runs over [−1, 1], the vector u = (ax, a 1 − x2 ) runs over all vectors of length a nonnegative vertical component. p a in the plane having √ Putting v = (by, b 1 − y2), w = (cz, c 1 − z2 ), the system becomes u + v = w, with vectors u, v, w of lengths a, b, c respectively in the upper half-plane. Then a, b, c are sides of a (possibly degenerate) triangle; i.e, |a − b| ≤ c ≤ a + b is a necessary condition. Conversely, if a, b, c satisfy this condition, one constructs a triangle OMN with −−→ −→ OM = a, ON = b, MN = c. If the vectors OM, ON have a positive nonnegative −−→ component, then so does their sum. For every such triangle, putting u = OM, −→ −−→ −→ v = ON, and w = OM + ON gives a solution, and every solution is given by one such triangle. This triangle is uniquely determined up to congruence: α = ∠MON = ∠(u, v) and β = ∠(u, w). Therefore, all solutions of the system are x = cost, y = cos(t + α ), z = y = cos(t + β ), x = cost, y = cos(t − α ), z = y = cos(t − β ), t ∈ [0, π − α ] or t ∈ [α , π ]. 17. Let z0 ≥ 1 be a positive integer. Supposing that the statement is true for all triples (x, y, z) with z < z0 , we shall prove that it is true for z = z0 too. If z0 = 1, verification is trivial, while x0 = y0 is obviously impossible. So let there be given a triple (x0 , y0 , z0 ) with z0 > 1 and x0 < y0 , and define another triple (x, y, z) by x = z0 , y = x0 + y0 − 2z0 , and z = z0 − x0 . Then x, y, z are positive integers. This is clear for x, z, while y = x0 + y0 − 2z0 ≥ √ 2( x0 y0 − z0 ) > 2(z0 − z0 ) = 0. Moreover, xy − z2 = x0 (x0 + y0 − 2z0 ) − (z0 − x0 )2 = x0 y0 − z20 = 1 and z < z0 , so that by the assumption, the statement holds for x, y, z. Thus for some nonnegative integers a, b, c, d we have x = a 2 + b2 , y = c2 + d 2 , z = ac + bd. But then we obtain representations of this sort for x0 , y0 , z0 too: x0 = a2 + b2 , y0 = (a + c)2 + (b + d)2, z0 = a(a + c) + b(b + d). For the second part of the problem, we note that for z = (2q)!, 444 4 Solutions z2 = (2q)!(2q)(2q − 1) · · ·1 ≡ (2q)! · (−(2q + 1))(−(2q + 2)) · · ·(−4q) = (−1)2q (4q)! ≡ −1 (mod p), by Wilson’s theorem. Hence p | z2 + 1 = py for some positive integer y > 0. Now it follows from the first part that there exist integers a, b such that x = p = a2 +b2 . Second solution. Another possibility is using arithmetic of Gaussian integers. Lemma. Suppose m, n, p, q are elements of Z or any other unique factorization domain, with mn = pq. then there exist elements a, b, c, d such that m = ab, n = cd, p = ac, q = bd. Proof is direct, for example using factorization of a, b, c, d into primes. We now apply this lemma to the Gaussian integers in our case (because Z[i] has the unique factorization property), having in mind that xy = z2 + 1 = (z + i)(z − i). We obtain (1) x = ab, (2) y = cd, (3) z + i = ac, (4) z − i = bd for some a, b, c, d ∈ Z[i]. Let a = a1 + a2i, etc. By (3) and (4), gcd(a1 , a2 ) = · · · = gcd(d1 , d2 ). Then (1) and (2) give us b = a, c = d. The statement follows at once: x = ab = aa = a21 + a22 , y = dd = d12 + d22 and z + i = (a1 d1 + a2 d2 ) + ı(a2 d1 − a1 d 2 ) ⇒ z = a1 d1 + a2 d2 . 4.21 Shortlisted Problems 1979 445 4.21 Solutions to the Shortlisted Problems of IMO 1979 1. We prove more generally, by induction on n, that any 2n-gon with equal edges and opposite edges parallel to each other can be dissected. For n = 2 the only possible such 2n-gon is a single lozenge, so our theorem holds in this case. We will now show that it holds for general n. Assume by induction that it holds for n − 1. Let A1 A2 . . . A2n be an arbitrary 2n-gon with equal edges and opposite edges parallel to each other. Then we can construct points Bi for i = 3, 4, . . . , n −−→ −−→ −−−−−−→ such that Ai Bi = A2 A1 = An+1 An+2 . We set B2 = A2n+1 = A1 and Bn+1 = An+2 . It follows that Ai Bi Bi+1 Ai+1 for i = 2, 3, 4, . . . , n are all lozenges. It also follows that Bi Bi+1 for i = 2, 3, 4, . . . , n are equal to the edges of A1 A2 . . . A2n and parallel to Ai Ai+1 and hence to An+i An+i+1 . Thus B2 . . . Bn+1 An+3 . . . A2n is a 2(n − 1)gon with equal edges and opposite sides parallel and hence, by the induction hypothesis, can be dissected into lozenges. We have thus provided a dissection for A1 A2 . . . A2n . This completes the proof. 2. The only way to arrive at the latter alternative is to draw four different socks in the first drawing or to draw only one pair in the first drawing and then draw two different socks in the last drawing. We will call these probabilities respectively p1 , p2 , p3 . We calculate them as follows:  5 4 5 42 22 8 4 4 4 4 2 p1 = 10 = , p2 = 10 = , p3 = 6 = . 21 7 15 4 4 2 We finally calculate the desired probability: P = p1 + p2 p3 = 8 15 . 3. An obvious solution is f (x) = 0. We now look for nonzero solutions. We note that plugging in x = 0 we get f (0)2 = f (0); hence f (0) = 0 or f (0) = 1. If f (0) = 0, then f is of the form f (x) = xk g(x), where g(0) 6= 0. Plugging this formula into f (x) f (2x2 ) = f (2x3 + x) we get 2k x2k g(x)g(2x2 ) = (2x2 + 1)k g(2x3 + x). Plugging in x = 0 gives us g(0) = 0, which is a contradiction. Hence f (0) = 1. For an arbitrary root α of the polynomial f , 2α 3 + α must also be a root. Let α be a root of the largest modulus. If |α | > 1 then |2α 3 + α | > 2|α |3 − |α | > |α |, which is impossible. It follows that |α | ≤ 1 and hence all roots of f have modules less than or equal to 1. But the product of all roots of f is | f (0)| = 1, which implies that all the roots have modulus 1. Consequently, for a root α it holds that |α | = |2α 3 − α | = 1. This is possible only if α = ±ı. Since the coefficients of f are real it follows that f must be of the form f (x) = (x2 + 1)k where k ∈ N0 . These polynomials satisfy the original formula. Hence, the solutions for f are f (x) = 0 and f (x) = (x2 + 1)k , k ∈ N0 . 4. Let us prove first that the edges A1 A2 , A2 A3 , . . . , A5 A1 are of the same color. Assume the contrary, and let w.l.o.g. A1 A2 be red and A2 A3 be green. Three of the segments A2 Bl (l = 1, 2, 3, 4, 5), say A2 Bi , A2 B j , A2 Bk , have to be of the same 446 4 Solutions color, let it w.l.o.g. be red. Then A1 Bi , A1 B j , A1 Bk must be green. At least one of the sides of triangle Bi B j Bk , say Bi B j , must be an edge of the prism. Then looking at the triangles A1 Bi B j and A2 Bi B j we deduce that Bi B j can be neither green nor red, which is a contradiction. Hence all five edges of the pentagon A1 A2 A3 A4 A5 have the same color. Similarly, all five edges of B1 B2 B3 B4 B5 have the same color. We now show that the two colors are the same. Assume otherwise, i.e., that w.l.o.g. the A edges are painted red and the B edges green. Let us call segments of the form Ai B j diagonal (i and j may be equal). We now count the diagonal segments by grouping the red segments based on their A point, and the green segments based on their B point. As above, the assumption that three of Ai B j for fixed i are red leads to a contradiction. Hence at most two diagonal segments out of each Ai may be red, which counts up to at most 10 red segments. Similarly, at most 10 diagonal segments can be green. But then we can paint at most 20 diagonal segments out of 25, which is a contradiction. Hence all edges in the pentagons A1 A2 A3 A4 A5 and B1 B2 B3 B4 B5 have the same color. 5. Let A = {x | (x, y) ∈ M} and B = {y | (x, y) ∈ M. Then A and B are disjoint and hence  2 (|A| + |B|)2 n |M| ≤ |A| · |B| ≤ ≤ . 4 4 These cardinalities can be achieved for M = {(a, b) | a = 1, 2, . . . , [n/2], b = [n/2] + 1, . . ., n} . 6. Setting q = x2 + x − p, the given equation becomes q q q (x + 1)2 − 2q + (x + 2)2 − q = (2x + 3)2 − 3q. (1) p Taking squares of both sides we get 2 ((x + 1)2 − 2q)((x + 2)2 − q) = 2(x + 1)(x + 2). Taking squares again we get  q 2q − 2(x + 2)2 − (x + 1)2 = 0. If 2q = 2(x + 2)2 + (x + 1)2 , at least one of the expressions under the three square roots in (1) is negative, and in that case the square root is not well-defined. Thus, we must have q = 0. Now (1) is equivalent to |x + 1| + |x + 2| = |2x + 3|, which holds if and only if x 6∈ (−2, −1). The number of real solutions x of q = x2 + x − p = 0 which are not in the interval (−2, −1) is zero if p < −1/4, one if p = −1/4 or 0 < p < 2, and two otherwise. Hence, the answer is −1/4 < p ≤ 0 or p ≥ 2. 7. We denote the sum mentioned above by S. We have the following equalities: 4.21 Shortlisted Problems 1979 447 1 1 1 1 1 + − + ···− + 2 3 4 1318 1319   1 1 1 1 1 1 + + ···+ −2 + + ···+ 2 1319 2 4 1318   1 1 1 1 1 + + ···+ − 1 + + ···+ 2 1319 2 659 1 1 1 + + ···+ 660 661 1319 989 989 1 1 1979 ∑ i + 1979 − i = ∑ i · (1979 − i) i=660 i=660 S = 1− = = = = Since no term in the sum contains a denominator divisible by 1979 (1979 is a prime number), it follows that when S is represented as p/q the numerator p will have to be divisible by 1979. 8. By the definition of f , it holds that f (0.b1 b2 . . . ) = 3b1 /4 + f (0.b2 b3 . . . )/4 = 0.b1 b1 + f (0.b2 b3 . . . )/4. Continuing this argument we obtain f (0.b1 b2 b3 . . . ) = 0.b1 b1 . . . bn bn + 1 f (0.bn+1 bn+2 . . . ). 22n (1) The binary representation of every rational number is eventually periodic. Let us first determine f (x) for a rational x with the periodic representation x = 0.b1 b2 . . . bn . Using (1) we obtain f (x) = 0.b1 b1 . . . bn bn + f (x)/22n , and hence n f (x) = 2n2−1 0.b1 b1 . . . bn bn = 0.b1 b1 . . . bn bn . Now let x = 0.a1 a2 . . . ak b1 b2 . . . bn be an arbitrary rational number. Then it follows from (1) that f (x) = 0.a1a1 . . . ak ak + 1 f (0.b1 b2 . . . bn ) = 0.a1 a1 . . . ak ak b1 b1 . . . bn bn . 22n Hence f (0.b1 b2 . . . ) = 0.b1 b1 b2 b2 . . . for every rational number 0.b1 b2 . . . . 9. Let us number the vertices, starting from S and moving clockwise. In that case S = 1 and F = 5. After an odd number of moves to a neighboring point we can be only on an even point, and hence it follows that a2n−1 = 0 for all n ∈ N. Let us define respectively zn and wn as the number of paths from S to S in 2n moves and the number of paths from S to points 3 and 7 in 2n moves. We easily derive the following recurrence relations: a2n+2 = wn , wn+1 = 2wn + 2zn , zn+1 = 2zn + wn , n = 0, 1, 2, . . . . By subtracting the second equation from the third we get zn+1 = wn+1 − wn . By plugging this equation into the formula for wn+2 we get wn+2 −4wn+1 +2w √n = 0. 2 − 4r + 2 = 0 are x = 2 + 2 and The roots of the characteristic equation r √ y = 2 − 2. From the √ conditions w0 = 0 and w1 = 2 we easily obtain a2n = wn−1 = (xn−1 − yn−1 )/ 2 . 448 4 Solutions → − → − 10. In the cases a = 0 , b = 0 , and a k b the inequality is trivial. Otherwise, let us − → − → consider a triangle ABC such that CB = a and CA = b. From this point on we shall refer to α , β , γ as angles √ of ABC. Since |a × b| = |a||b| sin γ , our inequality reduces to |a||b| sin3 γ ≤ 3 3|c|2 /8, which is further reduced to √ 3 3 sin α sin β sin γ ≤ 8 using the sine law. The last inequality follows immediately from Jensen’s inequality applied to the function f (x) = ln sin x, which is concave for 0 < x < π because f ′ (x) = cot x is strictly decreasing. 11. Let us define yi = x2i . We thus have y1 + y2 + · · · + yn = 1, yi ≥ 1/n2 , and P = √ y1 y2 . . . yn . The upper bound is obtained immediately from √ the AM–GM inequality: P ≤ √ 1/nn/2 , where equality holds when xi = yi = 1/ n. For the lower bound, let us assume w.l.o.g. that y1 ≥ y2 ≥ · · · ≥ yn . We note that if a ≥ b ≥ 1/n2 and s = a + b > 2/n2 is fixed, then ab = (s2 − (a − b)2 )/4 is minimized when |a − b| is maximized, i.e., when b = 1/n2. Hence y1 y2 · · · yn is minimal when√y2 = y3 = · · · = yn = 1/n2 . Then y1 = (n2 − n + 1)/n2 and therefore Pmin = n2 − n + 1/nn . 12. The first criterion ensures that all sets in an S-family are distinct. Since the number of different families of subsets is finite, h has to exist. In fact, we will show that h = 11. First of all, if there exists X ∈ F such that |X | ≥ 5, then by (3) there exists Y ∈ F such that X ∪ Y = R. In this case |F| is at most 2. Similarly, for |X | = 4, for the remaining two elements either there exists a subset in F that contains both, in which case we obtain the previous case, or there exist different Y and Z containing them, in which case X ∪Y ∪ Z = R, which must not happen. Hence we can assume |X | ≤ 4 for all X ∈ F. Assume |X | = 1 for some X . In that case other sets must not contain that subset and hence must be contained in the remaining 5-element subset. These elements must not be subsets of each other. From elementary combinatorics, the largest  number of subsets of a 5-element set of which none is subset of another is 52 = 10. This occurs when we take all 2-element subsets. These subsets also satisfy (2). Hence |F|max = 11 in this case. Otherwise, let us assume |X | = 3 for some X . Let us define the following families of subsets: G = {Z = Y \ X | Y ∈ F} and H = {Z = Y ∩ X | Y ∈ F}. Then no two sets in G must complement each other in R \ X , and G must cover this set. Hence G contains exactly the sets of each of the remaining 3 elements. For each element of G no two sets in H of which one is a subset of another may be paired with it. There can be only 3 such subsets selected within a 3-element set X . Hence the number of remaining sets is smaller than 3·3 = 9. Hence in this case  |F|max = 10. In the remaining case all subsets have two elements. There are 62 = 15 of them. But for every three that complement each other one must be discarded; hence the maximal number for F in this case is 2 · 15/3 = 10. It follows that h = 11. 4.21 Shortlisted Problems 1979 449 13. From elementary trigonometry we have sin 3t = 3 sint − 4 sin3 t. Hence, if we √ denote y = sin 20◦ , we have 3/2 = sin 60◦ = 3y − 4y3 . Obviously 0 < y < 1/2 = sin 30◦ . The function f (x) = 3x − 4x3 is strictly increasing on [0, 1/2) because f ′ (x) = 3 − 12x2 > 0 for 0 ≤ x < 1/2. Now the desired inequality 20 60 = 1 21 7 ◦ 3 < sin 20 < 60 = 20 follows from   √   1 3 7 f < <f , 3 2 20 which is directly verified. 14. Let us assume that a ∈ R\ {1} is such that there exist a and x such that x = loga x, or equivalently f (x) := ln x/x = ln a. Then a is a value of the function f (x) for x ∈ R+ \ {1}, and the converse also holds. First we observe that f (x) tends to −∞ as x → 0 and f (x) tends to 0 as x → 1. Since f (x) > 0 for x > 1, the function f (x) takes its maximum at a point x for which f ′ (x) = (1 − ln x)/x2 = 0. Hence max f (x) = f (e) = e1/e . It follows that the set of values of f (x) for x ∈ R+ is the interval (−∞, e1/e ), and consequently the desired set of bases a of logarithms is (0, 1) ∪ (1, e1/e ]. 15. We note that 5 5 5 5 i=1 i=1 i=1 i=1 ∑ i(a − i2)2 xi = a2 ∑ ixi − 2a ∑ i3 xi + ∑ i5 xi = a2 · a − 2a · a2 + a3 = 0. Since the terms in the sum on the left are all nonnegative, it follows that all the terms have to be 0. Thus, either xi = 0 for all i, in which case a = 0, or a = j2 for some j and xi = 0 for i 6= j. In this case, x j = a/ j = j. Hence, the only possible values of a are {0, 1, 4, 9, 16, 25}. 16. Obviously, no two elements of F can be complements of each other. If one of the sets has one element, then the conclusion is trivial. If there exist two different 2-element sets, then they must contain a common element, which in turn must then be contained in all other sets. Thus we can assume that there exists at most one 2-element subset of K in F. Since there can be at most 6 subsets of more than 3 elements of a 5-element set, it follows that at least 9 out of 10 possible 3-element subsets of K belong to F. Let us assume, without loss of generality, that all sets but {c, d, e} belong to F. Then sets {a, b, c}, {a, d, e}, and {b, c, d} have no common element, which is a contradiction. Hence it follows that all sets have a common element. 17. Let K, L, and M be intersections of CQ and BR, AR and CP, and AQ and BP, respectively. Let ∠X denote the angle of the hexagon KQMPLR at the vertex X , where X is one of the six points. By an elementary calculation of angles we get ∠K = 140◦, ∠L = 130◦ , ∠M = 150◦ , ∠P = 100◦ , ∠Q = 95◦ , ∠R = 105◦ . 450 4 Solutions Since ∠KBC = ∠KCB, it follows that K is on the symmetry line of ABC through A. Analogous statements hold for L and M. Let KR and KQ be points symmetric to K with respect to AR and AQ, respectively. Since ∠AKQ Q = ∠AKQ KR = 70◦ C and ∠AKR R = ∠AKR KQ = 70◦ , it follows that the points KR , R, Q, and 25o 20o KQ are collinear. Hence ∠QRK = ◦ 2∠R − 180 and ∠RQK = 2∠Q − KR 180◦ . In the same way we conclude ◦ R that ∠PRL = 2∠R−180 , ∠RPL = K L 2∠P − 180◦ , ∠QPM = 2∠P − 180◦ Q P and ∠PQM = 2∠Q − 180◦. From these 25o 20o M KQ o formulas we easily get ∠RPQ = 60◦ , 15 15o B A ∠RQP = 75◦ , and ∠QRP = 45◦ . 18. Let us write all ai in binary representation. For S ⊆ {1, 2, . . .,m} let us define b(S) as the number in whose binary representation ones appear in exactly the slots where ones appear in all ai where i ⊆ S and don’t appear in any other ai . Some b(S), including b(0), / will equal 0, and hence there are fewer than 2m different positive b(S). We note that no two positive b(S1 ) and b(S2 ) (S1 6= S2 ) have ones in the same decimal places. Hence sums of distinct b(S)’s are distinct. Moreover ai = ∑ b(S) i∈S and hence the positive b(S) are indeed the numbers b1 , . . . , bn whose existence we had to prove. 19. Let us define i j for two positive integers i and j in the following way: i1 = i and i j+1 = ii j for all positive integers j. Thus we must find the smallest m such that 100m > 3100 . Since 1001 = 100 > 27 = 32 , we inductively have 100 j = 10100 j−1 > 3100 j−1 > 33 j = 3 j+1 and hence m ≤ 99. We now prove that m = 99 by proving 10098 < 3100 . We note that (1001 )2 = 104 < 274 = 312 < 327 = 33 . We also note for d > 12 (which trivially holds for all d = 100i ) that if c > d 2 , then we have 2 3c > 3d > 312d = (312 )d > 10000d = (100d )2 . Hence from 33 > (1001)2 it inductively follows that 3 j > (100 j−2 )2 > 100 j−2 and hence that 10099 > 3100 > 10098. Hence m = 99. 20. Let xk = max{x1 , x2 , . . . , xn }. Then xi xi+1 ≤ xi xk for i = 1, 2, . . . ,k − 1 and xi xi+1 ≤ xk xi+1 for i = k, . . . , n − 1. Summing up these inequalities for i = 1, 2, . . . , n − 1 we obtain n−1 ∑ ≤ xk (x1 + · · · + xk−1 + xk+1 + · · · + xn) = xk (a − xk) ≤ i=1 a2 . 4 We note that the value a2 /4 is attained for x1 = x2 = a/2 and x3 = · · · = xn = 0. Hence a2 /4 is the required maximum. 4.21 Shortlisted Problems 1979 451 21. Denote m = 106 and let f (n) be the number of different ways n ∈ N can be expressed as x2 + y3 with x, y ∈ {0, 1, . . . , m}. Clearly f (n) = 0 for n < 0 or n > m2 + m3 . The first equation can be written as x2 + t 3 = y2 + z3 = n, whereas the second equation can be written as x2 + t 3 = n + 1, y2 + z3 = n. Hence we obtain the following formulas for M and N: m M = ∑ f (i)2 , m−1 N= i=0 ∑ f (i) f (i + 1) . i=0 Using the AM–GM inequality we get m−1 N= ∑ f (i) f (i + 1) ∑ f (i)2 + f (i + 1)2 f (0)2 m−1 f (m)2 = + ∑ f (i)2 + <M. 2 2 2 i=1 i=0 m−1 ≤ i=0 The last inequality is strict, since f (0) = 1 > 0. This completes our proof. 22. Let the centers of the two circles be denoted by O and O1 and their respective M(t) radii by r and r1 , and let the positions N(t) of the points on the circles at time t be A denoted by M(t) and N(t). Let Q be the point such that OAO1 Q is a paralωt ωt φ φ lelogram. We will show that Q is the O O1 point P we are looking for, i.e., that P=Q QM(t) = QN(t) for all t. We note that OQ = O1 A = r1 , O1 Q = OA = r and ∠QOA = ∠QO1 A = φ . Since the two points return to A at the same time, it follows that ∠M(t)OA = ∠N(t)O1 A = ω t. Therefore ∠QOM(t) = ∠QO1 N(t) = φ + ω t, from which it follows that △QOM(t) ∼ = △QO1 N(t). Hence QM(t) = QN(t), as we claimed. 23. It is easily verified that no solutions exist for n ≤ 8. Let us now assume that n > 8. We note that 28 + 211 + 2n = 28 · (9 + 2n−8 ). Hence 9 + 2n−8 must also be a square, say 9 + 2n−8 = x2 , x ∈ N, i.e., 2n−8 = x2 − 9 = (x − 3)(x + 3). Thus x − 3 and x + 3 are both powers of 2, which is possible only for x = 5 and n = 12. Hence, n = 12 is the only solution. 24. Clearly O is the midpoint of BC. Let M and N be the points of tangency of the circle with AB and AC, respectively, and let ∠BAC = 2ϕ . Then ∠BOM = ∠CON = ϕ . Let us assume that PQ touches the circle in X. If we set ∠POM = ∠POX = x and ∠QON = ∠QOX = y, then 2x + 2y = ∠MON = 180◦ − 2ϕ , i.e., y = 90◦ − ϕ − x. It follows that ∠OQC = 180◦ − ∠QOC −∠OCQ = 180◦ −(ϕ +y)−(90◦ − ϕ ) = 90◦ − y = x + ϕ = ∠BOP. Hence the triangles BOP and CQO are similar, and consequently BP ·CQ = BO · CO = (BC/2)2 . 452 4 Solutions Conversely, let BP ·CQ = (BC/2)2 and let Q′ be the point on (AC) such that PQ′ is tangent to the circle. Then BP ·CQ′ = (BC/2)2 , which implies Q ≡ Q′ . 25. Let us first look for such a point R on a ray l in π going through P. Let ∠QPR = 2θ . Consider a point Q′ on the extension of l beyond P such that Q′ P = QP. Then we have QP + PR RQ′ sin ∠Q′ QR = = . QR QR sin ∠QQ′ R Since ∠QQ′ R is fixed, the maximum of the expression occurs when ∠Q′ QR = 90◦ , i.e., when PR = PQ. In this case, (QP + PR)/QR = 1/sin θ . Looking at all possible rays l, we see that θ is minimal when l contains the projection of PQ onto π . Hence, if PQ 6⊥ π , the desired point R is the point on the projection of ray PQ onto π such that PR = PQ; otherwise, R is any point of the circle k(P, PQ). 26. Let us assume that f (x + y) = f (x) + f (y) for all reals. In this case we trivially apply the equation to get f (x + y + xy) = f (x + y) + f (xy) = f (x) + f (y) + f (xy). Hence the equivalence is proved in the first direction. Now let us assume that f (x+ y+xy) = f (x)+ f (y)+ f (xy) for all reals. Plugging in x = y = 0 we get f (0) = 0. Plugging in y = −1 we get f (x) = − f (−x). Plugging in y = 1 we get f (2x + 1) = 2 f (x) + f (1) and hence f (2(u + v + uv) + 1) = 2 f (u + v + uv) + f (1) = 2 f (uv) + 2 f (u) + 2 f (v) + f (1) for all real u and v. On the other hand, plugging in x = u and y = 2v+1 we get f (2(u+v+uv)+1) = f (u+ (2v+ 1)+ u(2v+1)) = f (u)+2 f (v)+ f (1)+ f (2uv+ u). Hence it follows that 2 f (uv) + 2 f (u) + 2 f (v) + f (1) = f (u) + 2 f (v) + f (1) + f (2uv + u), i.e., f (2uv + u) = 2 f (uv) + f (u). (1) Plugging in v = −1/2 we get 0 = 2 f (−u/2) + f (u) = −2 f (u/2) + f (u). Hence, f (u) = 2 f (u/2) and consequently f (2x) = 2 f (x) for all reals. Now (1) reduces to f (2uv + u) = f (2uv) + f (u). Plugging in u = y and x = 2uv, we obtain f (x) + f (y) = f (x + y) for all nonzero reals x and y. Since f (0) = 0, it trivially holds that f (x + y) = f (x) + f (y) when one of x and y is 0. Second solution. Assume that f (x + y + xy) = f (x) + f (y) + f (xy) for all x, y. Substituting (x, −y) in the functional equation and adding it to the original equation yields f (x − t) + f (x + t) = 2 f (x), where t = y(x + 1). (2) Thus (2) holds whenever x 6= −1. Similarly, for t 6= −1 we have f (t − x) + f (x + t) = 2 f (t). Summing these two equalities and using f (y) = − f (−y) as shown above we obtain f (x) + f (t) = f (x + t) for all x,t 6= −1. The case x = −1 or t = −1 is easy to handle with, as f (−1) = 2 f (− 12 ). 4.22 Shortlisted Problems 1981 453 4.22 Solutions to the Shortlisted Problems of IMO 1981 1. Assume that the set {a − n + 1, a − n + 2, . . . , a} of n consecutive numbers satisfies the condition a | lcm[a − n + 1, . . . , a − 1]. Let a = pα1 1 pα2 2 . . . pαr r be the canonic representation of a, where p1 < p2 < · · · < pr are primes and α1 , · · · , αr > 0. Then for each j = 1, 2, . . . , r, there exists m, m = 1, 2, . . . , n − 1, α α α such that p j j | a − m, i.e., such that p j j | m. Thus p j j ≤ n − 1. If r = 1, then a = pα1 1 ≤ n − 1, which is impossible. Therefore r ≥ 2. But then there must exist two distinct prime numbers less than n; hence n ≥ 4. For n = 4, we must have pα1 1 , pα2 2 ≤ 3, which leads to p1 = 2, p2 = 3, α1 = α2 = 1. Therefore a = 6, and {3, 4, 5, 6} is a unique set satisfying the condition of the problem. For every n ≥ 5 there exist at least two such sets. In fact, for n = 5 we easily find two sets: {2, 3, 4, 5, 6} and {8, 9, 10, 11, 12}. Suppose that n ≥ 6. Let r, s,t be natural numbers such that 2r ≤ n−1 < 2r+1 , 3s ≤ n−1 < 3s+1 , 5t ≤ n−1 < 5t+1 . Taking a = 2r · 3s and a = 2r · 5t we obtain two distinct sets with the required property. Thus the answers are (a) n ≥ 4 and (b) n = 4. 2. Lemma. Let E, F, G, H, I, and K be points on edges AB, BC, CD, DA, AC, and BD of a tetrahedron. Then there is a sphere that touches the edges at these points if and only if AE = AH = AI, BE = BF = BK, CF = CG = CI, DG = DH = DK. (∗) Proof. The “only if” side of the equivalence is obvious. We now assume (∗). Denote by D ε , φ , γ , η , ι , and κ planes through G E, F, G, H, I, K perpendicular to H AB, BC, CD, DA, AC and BD respectively. Since the three planes ε , C η , and ι are not mutually parallel, F A they intersect in a common point O. E B ∼ ∼ Clearly, △AEO = △AHO = △AIO; hence OE = OH = OI = r, and the sphere σ (O, r) is tangent to AB, AD, AC. To prove that σ is also tangent to BC,CD, BD it suffices to show that planes φ , γ , and κ also pass through O. Without loss of generality we can prove this for just φ . By the conditions for E, F, I, these are exactly the points of tangency of the incircle of △ABC and its sides, and if S is the incenter, then SE ⊥ AB, SF ⊥ BC, SI ⊥ AC. Hence ε , ι , and φ all pass through S and are perpendicular to the plane ABC, and consequently all share the line l through S perpendicular to ABC. Since l = ε ∩ ι , the point O will be situated on l, and hence φ will also contain O. This completes our proof of the lemma. Let AH = AE = x, BE = BF = y, CF = CG = z, and DG = DH = w. If the sphere is also tangent to AC at some point I, then AI = x and IC = z. Using the stated lemma it suffices to prove that if AC = x + z, then BD = y + w. 454 4 Solutions Let EF = FG = GH = HI = t, ∠BAD = α , ∠ABC = β , ∠BCD = γ , and ∠ADC = δ . We get t 2 = EH 2 = AE 2 + AH 2 − 2 · AE · AH cos α = 2x2 (1 − cos α ). We similarly conclude that t 2 = 2y2 (1 − cos β ) = 2z2 (1 − cos γ ) = 2w2 (1 − cos δ ). Further, using that AB = x + y, BC = y + z, cos β = 1 −t 2 /2y2 , we obtain    x z AC2 = AB2 + BC2 − 2AB · BC cos β = (x − z)2 + t 2 +1 +1 . y y Analogously, from the triangle ADC we get AC2 = (x − z)2 + t 2 (x/w + 1)(z/w + 1), which gives (x/y + 1)(z/y + 1) = (x/w + 1)(z/w + 1). Since f (s) = (x/s + 1)(z/s + 1) is a decreasing function in s, it follows that y = w; similarly x = z. Hence CF = CG = x and DG = DH = y. Hence AC k EF and AC : t = AC : EF = AB : EB = (x + y) : y; i.e., AC = t(x + y)/y. Similarly, from the triangle ABD, we get that BD = t(x + y)/x. Hence if AC = x + z = 2x, it follows that 2x = t(x + y)/y ⇒ 2xy = t(x + y) ⇒ BD = t(x + y)/x = 2y = y + w. This completes the proof. Second solution. Without loss of generality, assume that EF = 2. Consider the Cartesian system in which points O, E, F, G, H respectively have coordinates (0, 0, 0), (−1, −1, a), (1, −1, a), (1, 1, a), (−1, 1, a). Line AH is perpendicular to OH and AE is perpendicular to OE; hence from Pythagoras’s theorem AO2 = AH 2 + HO2 = AE 2 + EO2 = AE 2 + HO2 , which implies AH = AE. Therefore the y-coordinate of A is zero; analogously the x-coordinates of B and D and the y-coordinate of C are 0. Let A have coordinates (x0 , 0, z1 ): then −→ −→ −→ −→ EA(x0 + 1, 1, z1 − a) ⊥ EO(1, 1, −a), i.e., EA · EO = x0 + 2 + a(a − z1) = 0. Similarly, for B(0, y0 , z2 ) we have y0 + 2 + a(a − z2) = 0. This gives us z1 = x0 + a2 + 2 , a z2 = y0 + a2 + 2 . a (1) We haven’t used yet that A(x0 , 0, z1 ), E(−1, −1, a) and B(0, y0 , z2 ) are collinear, so let A′ , B′ , E ′ be the feet of perpendiculars from A, B, E to the plane xy. The line A′ B′ , given by y0 x + x0 y = x0 y0 , z = 0, contains the point E ′ (−1, −1, 0), from which we obtain (x0 + 1)(y0 + 1) = 1. (2) In the same way, from the points B and C we get relations similar to (1) and (2) and conclude that C has the coordinates C(−x0 , 0, z1 ). Similarly we get D(0, −y0 , z2 ).√The condition that AC√is tangent to the sphere σ (O, OE) is equiv2 2 + 2 − (a2 + 2). But then (2) implies that alent to z√ 1 = a + 2, i.e., to x0 = a a √ 2 2 y0 = −a a + 2 − (a + 2) and z2 = − a2 + 2, which means that the sphere σ is tangent to BD as well. This finishes the proof. 3. Denote max(a + b + c, b + c + d, c + d + e, d + e + f , e + f + g) by p. We have (a + b + c) + (c + d + e) + (e + f + g) = 1 + c + e ≤ 3p, 4.22 Shortlisted Problems 1981 455 which implies that p ≥ 1/3. However, p = 1/3 is achieved by taking (a, b, c, d, e, f , g) = (1/3, 0, 0, 1/3, 0, 0, 1/3). Therefore the answer is 1/3. Remark. In fact, one can prove a more general statement in the same way. Given positive integers n, k, n ≥ k, if a1 , a2 , . . . , an are nonnegative real numbers with sum 1, then the minimum value of maxi=1,...,n−k+1 {ai + ai+1 + · · · + ai+k−1 } is 1/r, where r is the integer with k(r − 1) < n ≤ kr. 4. We shall use the known formula for the Fibonacci sequence 1 fn = √ (α n − (−1)n α −n ), 5 √ 1+ 5 where α = . 2 (1) (a) Suppose that a fn + b fn+1 = fkn for all n, where kn > 0 is an integer depending on n. By (1), this is equivalent to a(α n − (−1)n α −n ) + b(α n+1 + (−1)n α −n−1 ) = α kn − (−1)kn α −kn , i.e., α kn −n = a + bα − α −2n (−1)n (a − bα −1 − (−α )n−kn ) → a + bα (2) as n → ∞. Hence, since kn is an integer, kn − n must be constant from some point on: kn = n + k and α k = a + bα . Then it follows from (2) that α −k = a−bα −1 , and from (1) we conclude that a fn +b fn+1 = fk+n holds for every n. Putting n = 1 and n = 2 in the previous relation and solving the obtained system of equations we get a = fk−1 , b = fk . It is easy to verify that such a and b satisfy the conditions. 2 = f for all n. This leads to (b) As in (a), suppose that u fn2 + v fn+1 ln √ u + vα 2 − 5α ln −2n = 2(u − v)(−1)nα −2n √ −(uα −4n + vα −4n−2 + (−1)ln 5α −ln −2n ) → 0, √ as n → ∞. Thus u + vα 2 = 5α ln −2n , and ln − 2n = k is equal to a constant. Putting this into the above equation and multiplying by α 2n we get u − v → 0 as n → ∞, i.e., u = v. Finally, substituting n = 1 and n = 2 in u fn2 + 2 = f we easily get that the only possibility is u = v = 1 and k = 1. It u fn+1 ln is easy to verify that such u and v satisfy the conditions. 5. There are four types of small cubes upon disassembling: (1) 8 cubes with three faces, painted black, at one corner; (2) 12 cubes with two black faces, both at one edge; (3) 6 cubes with one black face; (4) 1 completely white cube. All cubes of type (1) must go to corners, and be placed in a correct way (one of three): for this step we have 38 · 8! possibilities. Further, all cubes of type (2) must go in a correct way (one of two) to edges, admitting 212 · 12! possibilities; similarly, there are 46 · 6! ways for cubes of type (3), and 24 ways for the cube of type (4). Thus the total number of good reassemblings is 38 8! · 212 12! · 46 6! · 456 4 Solutions 24, while the number of all possible reassemblings is 2427 · 27!. The desired 8 12 12!·46 6!·24 probability is 3 8!·22427 . It is not necessary to calculate these numbers to ·27! find out that the blind man practically has no chance to reassemble the cube in a right way: in fact, the probability is of order 1.8 · 10−37. 6. Assume w.l.o.g. that n = degP ≥ degQ, and let P0 = {z1 , z2 , . . . , zk }, P1 = {zk+1 , zk+2 , . . . zk+m }. The polynomials P and Q match at k + m points z1 , z2 , . . . , zk+m ; hence if we prove that k + m > n, the result will follow. By the assumption, P(x) = (x − z1 )α1 · · · (x − zk )αk = (x − zk+1 )αk+1 · · · (x − zk+m )αk+m + 1 for some positive integers α1 , . . . , αk+m . Let us consider P′ (x). As we know, it is divisible by (x − zi )αi −1 for i = 1, 2, . . . , k + m; i.e., k+m ∏ (x − zi )αi −1 | P′ (x). i=1 αi −1 ≤ degP′ = n − 1, i.e., k + m ≥ n +1, Therefore 2n −k −m = deg ∏k+m i=1 (x− zi ) as we claimed. 7. We immediately find that f (1, 0) = f (0, 1) = 2. Then f (1, y + 1) = f (0, f (1, y)) = f (1, y) + 1; hence f (1, y) = y + 2 for y ≥ 0. Next we find that f (2, 0) = f (1, 1) = 3 and f (2, y + 1) = f (1, f (2, y)) = f (2, y) + 2, from which f (2, y) = 2y + 3. Particularly, f (2, 2) = 7. Further, f (3, 0) = f (2, 1) = 5 and f (3, y + 1) = f (2, f (3, y)) = 2 f (3, y) + 3. This gives by induction f (3, y) = 2y+3 − 3. For y = 3, f (3, 3) = 61. Finally, from f (4, 0) = f (3, 1) = 13 and f (4, y + 1) = f (3, f (4, y)) = 2 f (4,y)+3 − 3, we conclude that f (4, y) = 2 2. 2 .. −3 (y + 3 twos). 8. Since the number k, k = 1, 2, . . . , n − r + 1, is the minimum in exactly element subsets of {1, 2, . . . , n}, it follows that Using the equality n−r+1 ∑ k=1 r+ j j    1 n−r+1 n − k  f (n, r) = n ∑ k . r−1 k=1 r j = ∑i=0 r+i−1 r−1  , we get  ! r+i−1 ∑ ∑ r−1 j=0 i=0      n−r  r+ j n+1 n+1 n = ∑ = = . r r+1 r+1 r j=0   n−k k = r−1 n−r Therefore f (n, r) = (n + 1)/(r + 1). j n−k r−1 r- 4.22 Shortlisted Problems 1981 457 9. If we put 1 + 24an = b2n , the given recurrent relation becomes 2 2 3 b2 2 bn+1 = + n + bn = 3 2 6 3  3 bn + 2 2 2 , i.e., bn+1 = 3 + bn , 2 (1) where b1 = 5. To solve this recurrent equation, we set cn = 2n−1 bn . From (1) we obtain cn+1 = cn + 3 · 2n−1 = · · · = c1 + 3(1 + 2 + 22 + · · · + 2n−1 ) = 5 + 3(2n − 1) = 3 · 2n + 2. Therefore bn = 3 + 2−n+2 and consequently      b2 − 1 1 3 1 1 1 1 an = n = 1 + n + 2n−1 = 1 + n−1 1+ n . 24 3 2 2 3 2 2 10. It is easy to see that partitioning into p = 2k squares is possible for k ≥ 2 (Fig. 1). Furthermore, whenever it is possible to partition the square into p squares, there is a partition of the square into p + 3 squares: namely, in the partition into p squares, divide one of them into four new squares. x-y y y p=8 Fig. 1 x y Fig. 2 This implies that both p = 2k and p = 2k + 3 are possible if k ≥ 2, and therefore all p ≥ 6 are possible. On the other hand, partitioning the square into 5 squares is not possible. Assuming it is possible, one of its sides would be covered by exactly two squares, which cannot be of the same size (Fig. 2). The rest of the big square cannot be partitioned into three squares. Hence, the answer is n = 6. 11. Let us denote the center of the semicircle by O, and ∠AOB = 2α , ∠BOC = 2β , AC = m, CE = n. We claim that a2 + b2 + n2 + abn = 4. Indeed, since a = 2 sin α , b = 2 sin β , n = 2 cos(α + β ), we have a2 + b2 + n2 + abn = 4(sin2 α + sin2 β + cos2 (α + β ) + 2 sin α sin β cos(α + β ))   cos2α cos2β = 4+4 − − + cos(α + β ) cos(α − β ) 2 2 = 4 + 4 (cos(α + β ) cos(α − β ) − cos(α + β ) cos(α − β )) = 4. 458 4 Solutions Analogously, c2 + d 2 + m2 + cdm = 4. By adding both equalities and subtracting m2 + n2 = 4 we obtain a2 + b2 + c2 + d 2 + abn + cdm = 4. Since n > c and m > b, the desired inequality follows. 12. We will solve the contest problem (in which m, n ∈ {1, 2, . . . , 1981}). For m = 1, n can be either 1 or 2. If m > 1, then n(n − m) = m2 ± 1 > 0; hence n − m > 0. Set p = n − m. Since m2 − mp − p2 = m2 − p(m + p) = −(n2 − nm − m2), we see that (m, n) is a solution of the equation if and only if (p, m) is a solution too. Therefore, all the solutions of the equation are given as two consecutive members of the Fibonacci sequence 1, 1, 2, 3, 5, 8, 13, 21, 34, 55, 89, 144, 233, 377, 610, 987, 1597, 2584, . . . . So the required maximum is 9872 + 15972. 13. Lemma. For any polynomial P of degree at most n,   n+1 ∑ (−1) i P(i) = 0. i=0 n+1 i (1) Proof. We shall use induction on n. For n = 0 it is trivial. Assume that it is true for n = k and suppose that P(x) is a polynomial of degree k + 1. Then P(x) − P(x + 1) clearly has degree at most k; hence (1) gives  k+1 (P(i) − P(i + 1)) i i=0     k+1 k+2 k+1 k+1 = ∑ (−1)i P(i) + ∑ (−1)i P(i) i i−1 i=0 i=1   k+2 k+2 = ∑ (−1)i P(i). i i=0 k+1 0= ∑ (−1)i  This completes the proof of the lemma. Now we apply the lemma to obtain the value of P(n + 1). Since P(i) = for i = 0, 1, . . . , n, we have    n+1 1, 2 | n; i n+1 n+1 0 = ∑ (−1) P(i) = (−1) P(n + 1) + 0, 2 ∤ n. i i=0 It follows that P(n + 1) =  1, 0,  n+1 −1 i 2 | n; 2 ∤ n. 14. We need the following lemma. Lemma. If a convex quadrilateral PQRS satisfies PS = QR and ∠SPQ ≥ ∠RQP, then ∠QRS ≥ ∠PSR. 4.22 Shortlisted Problems 1981 459 Proof. If the lines PS and QR are parallel, then this quadrilateral is a parallelogram, and the statement is trivial. Otherwise, let X be the point of intersection of lines PS and QR. Assume that ∠SPQ + ∠RQP > 180◦ . Then ∠X PQ ≤ ∠X QP implies that XP ≥ XQ, and consequently X S ≥ X R. Hence, ∠QRS = ∠X RS ≥ ∠X SR = ∠PSR. Similarly, if ∠SPQ + ∠RQP < 180◦ , then ∠X PQ ≥ ∠X QP, from which it follows that XP ≤ XQ, and thus X S ≤ X R; hence ∠QRS = 180◦ − ∠X RS ≥ 180◦ − ∠XSR = ∠PSR. Now we apply the lemma to the quadrilateral ABCD. Since ∠B ≥ ∠C and AB = CD, it follows that ∠CDA ≥ ∠BAD, which together with ∠EDA = ∠EAD gives ∠D ≥ ∠A. Thus ∠A = ∠B = ∠C = ∠D. Analogously, by applying the lemma to BCDE we obtain ∠E ≥ ∠B, and hence ∠B = ∠C = ∠D = ∠E. 15. Set BC = a, CA = b, AB = c, and denote the area of △ABC by P, and a/PD + b/PE + c/PF by S. Since a · PD + b · PE + c · PF = 2P, by the Cauchy–Schwarz inequality we have   a b c 2PS = (a · PD + b · PE + c · PF) + + ≥ (a + b + c)2, PD PE PF with equality if and only if PD = PE = PF, i.e., P is the incenter of △ABC. In that case, S attains its minimum: Smin = (a + b + c)2 . 2P 16. The sequence {un } is bounded, whatever u1 is. Indeed, assume the opposite, and let um be the first member of the sequence such that |um | > max{2, |u1 |}. Then |um−1 | = |u3m − 15/64| > |um |, which is impossible. Next, let us see for what values of um , um+1 is greater, equal, or smaller, respectively. If um+1 = um , then um = u3m+1 − 15/64 = u3m − 15/64; i.e., um is a root of x3 − x − 15/64 = 0. This equation factors as (x + 1/4)(x2 − x/4 − 15/16) √ √ = 0, and hence um is equal to x1 = (1 − 61)/8, x2 = −1/4, or x3 = (1 + 61)/8, and these are the only possible limits of the sequence. Each of um+1 > um , um+1 < um is equivalent to u3m − um − 15/64 < 0 and u3m − um − 15/64 > 0 respectively. Thus the former is satisfied for um in the interval I1 = (−∞, x1 ) or I3 = (x2 , x3 ), while the latter is satisfied p for um in I2 = (x1 , x2 ) or I4 = (x3 , ∞). Moreover, since the function f (x) = 3 x + 15/64 is strictly increasing with fixed points x1 , x2 , x3 , it follows that um will never escape from the interval I1 , I2 , I3 , or I4 to which it belongs initially. Therefore: (1) if u1 is one of x1 , x2 , x3 , the sequence {um } is constant; (2) if u1 ∈ I1 , then the sequence is strictly increasing and tends to x1 ; (3) if u1 ∈ I2 , then the sequence is strictly decreasing and tends to x1 ; (4) if u1 ∈ I3 , then the sequence is strictly increasing and tends to x3 ; (5) if u1 ∈ I4 , then the sequence is strictly decreasing and tends to x3 . 460 4 Solutions 17. Let us denote by SA , SB , SC the centers of the given circles, where SA lies on the bisector of ∠A, etc. Then SA SB k AB, SB SC k BC, SC SA k CA, so that the inner bisectors of the angles of triangle ABC are also inner bisectors of the angles of △SA SB SC . These two triangles thus have a common incenter S, which is also the center of the homothety χ mapping △SA SB SC onto △ABC. The point O is the circumcenter of triangle SA SB SC , and so is mapped by χ onto the circumcenter P of ABC. This means that O, P, and the center S of χ are collinear. 18. Let C be the convex hull of the set of the planets: its border consists of parts of planes, parts of cylinders, and parts of the surfaces of some planets. These parts of planets consist exactly of all the invisible points; any point on a planet that is inside C is visible. Thus it remains to show that the areas of all the parts of planets lying on the border of C add up to the area of one planet. As we have seen, an invisible part of a planet is bordered by some main spherical arcs, parallel two by two. Now fix any planet P, and translate these arcs onto arcs on the surface of P. All these arcs partition the surface of P into several parts, each of which corresponds to the invisible part of one of the planets. This correspondence is bijective, and therefore the statement follows. 19. Consider the partition of plane π into regular hexagons, each having inradius 2. Fix one of these hexagons, denoted by γ . For any other hexagon x in the partition, there exists a unique translation τx taking it onto γ . Define the mapping ϕ : π → γ as follows: If A belongs to the interior of a hexagon x, then ϕ (A) = τx (A) (if A is on the border of some hexagon, it does not actually matter where its image is). The total area of the images√of the union of the given circles equals S, while the area of the hexagon γ is 8 3. Thus there exists a point B of γ that is covered S at least 8√ times, i.e., such that ϕ −1 (B) consists of at least 8√S 3 distinct points 3 of the plane that belong to some of the circles. For any of these points, take a circle that contains it. All these circles are disjoint, with total area not less than π √ S ≥ 2S/9. 8 3 Remark. The statement becomes false if the constant 2/9 is replaced by any number greater than 1/4. In that case a counterexample is, for example, a set of unit circles inside a circle of radius 2 covering a sufficiently large part of its area. 4.23 Shortlisted Problems 1982 461 4.23 Solutions to the Shortlisted Problems of IMO 1982 1. From f (1) + f (1) ≤ f (2) = 0 we obtain f (1) = 0. Since 0 < f (3) ≤ f (1) + f (2) + 1, it follows that f (3) = 1. Note that if f (3n) ≥ n, then f (3n + 3) ≥ f (3n) + f (3) ≥ n + 1. Hence by induction f (3n) ≥ n holds for all n ∈ N. Moreover, if the inequality is strict for some n, then it is so for all integers greater than n as well. Since f (9999) = 3333, we deduce that f (3n) = n for all n ≤ 3333. By the given condition, we have 3 f (n) ≤ f (3n) ≤ 3 f (n) + 2. Therefore f (n) = [ f (3n)/3] = [n/3] for n ≤ 3333. In particular, f (1982) = [1982/3] = 660. 2. Since K does not contain a lattice point other than O(0, 0), it is bounded by four lines u, v, w, x that pass through the points U(1, 0), V (0, 1), W (−1, 0), X (0, −1) respectively. Let PQRS be the quadrilateral formed by these lines, where U ∈ SP, V ∈ PQ, W ∈ QR, X ∈ RS. If one of the quadrants, say Q1 , contains no vertices of PQRS, then K ∩ Q1 is contained in △OUV and hence has area less than 1/2. Consequently the area of K is less than 2. Let us now suppose that P, Q, R, S lie in different quadrants. One of the angles of PQRS is at least 90◦: let it be ∠P. Then SUPV ≤ PU · PV /2 ≤ (PU 2 + PV 2 )/4 ≤ UV 2 /4 = 1/2, which implies that SK∩Q1 < SOUPV ≤ 1. Hence the area of K is less than 4.  3. (a) By the Cauchy–Schwarz inequality we have x20 /x1 + · · · + x2n−1/xn ·(x1 + · · · + xn ) ≥ (x0 + · · · + xn−1 )2 . Let us set Xn−1 = x1 + x2 + · · · + xn−1 . Using x0 = 1, the last inequality can be rewritten as x2 x20 (1 + Xn−1)2 4Xn−1 4 + · · · + n−1 ≥ ≥ = . x1 xn Xn−1 + xn Xn−1 + xn 1 + xn /Xn−1 (1) Since xn ≤ xn−1 ≤ · · · ≤ x1 , it follows that Xn−1 ≥ (n − 1)xn . Now (1) yields x20 /x1 + · · · + x2n−1/xn ≥ 4(n − 1)/n, which exceeds 3.999 for n > 4000. (b) The sequence xn = 1/2n obviously satisfies the required condition. Second solution to part (a). For each n ∈ N, let us find a constant cn such that the inequality x20 /x1 + · · · + x2n−1 /xn ≥ cn x0 holds for any sequence x0 ≥ x1 ≥ · · · ≥ xn > 0. For n = 1 we can take c1 = 1. Assuming that cn exists, we have  2  q x20 x2 √ x1 x2n + + ···+ ≥ 0 + cn x1 ≥ 2 x20 cn = x0 · 2 cn . x1 x2 xn+1 x1 √ n−2 Thus we can take cn+1 = 2 cn . Then inductively cn = 22−1/2 , and since cn → 4 as n → ∞, the result follows. Third solution. Since {xn } is decreasing, there exists limn→∞ xn = x ≥ 0. If x > 0, then x2n−1 /xn ≥ xn ≥ x holds for each n, and the result is trivial. If otherwise x = 0, then we note that x2n−1 /xn ≥ 4(xn−1 − xn ) for each n, with equality if and only if xn−1 = 2xn . Hence 462 4 Solutions n x2k−1 ≥ lim ∑ ∑ 4(xk−1 − xk) = 4x0 = 4. n→∞ n→∞ k=1 xk k=1 n lim Equality holds if and only if xn−1 = 2xn for all n, and consequently xn = 1/2n . 4. Suppose that a satisfies the requirements of the problem and that x, qx, q2 x, q3 x are the roots of the given equation. Then x 6= 0 and we may assume that |q| > 1, so that |x| < |qx| < |q2 x| < |q3 x|. Since the equation is symmetric, 1/x is also a root and therefore 1/x = q3 x, i.e., q = x−2/3 . It follows that the roots are x, x1/3 , x−1/3 , x−1 . Now by Viète’s formula we have x + x1/3 + x−1/3 + x−1 = a/16 and x4/3 + x2/3 + 2 + x−2/3 + x−4/3 = (2a + 17)/16. On setting z = x1/3 + x−1/3 these equations become z3 − 2z = a/16, (z2 − 2)2 + z2 − 2 = (2a + 17)/16. Substituting a = 16(z3 − 2z) in the second equation leads to z4 − 2z3 − 3z2 + 4z + 15/16 = 0. We observe that this polynomial factors as (z + 3/2)(z − 5/2)(z2 − z − 1/4). Since |z| = |x1/3 + x−1/3 | ≥ 2, the only viable value is z = 5/2. Consequently a = 170 and the roots are 1/8, 1/2, 2, 8. 5. Notice that △A5 B4 A4 ∼ = △A3 B2 A2 . A5 A4 We know that ∠A5 A3 A2 = 90◦ and B4 that ∠A2 B4 A4 is equal to the sum of the angles ∠A2 B4 A3 and ∠A3 B4 A4 . A6 A3 O Clearly, ∠A2 B4 A3 = 90◦ − ∠B2 A2 A3 B2 and ∠A3 B4 A4 = ∠B4 A5 A4 + ∠A5 A4 B4 . Hence we conclude that ∠A2 B4 A4 = A1 A2 90◦ + ∠B4A5 A4 = 120◦. Hence B4 belongs to the circle with center A√ 3 and radius A3 A4 , so A3 A4 = A3 B4 . Thus λ = A3 B4 /A3 A5 = A3 A4 /A3 A5 = 1/ 3. 6. Denote by d(U,V ) the distance between points or sets of points U and V . For P, Q ∈ L we shall denote by LPQ the part of L between points P and Q and by lPQ the length of this part. Let us denote by Si (i = 1, 2, 3, 4) the vertices of S and by Ti points of L such that Si Ti ≤ 1/2 in such a way that lA0 T1 is the least of the lA0 Ti ’s, S2 and S4 are neighbors of S1 , and lA0 T2 < lA0 T4 . Now we shall consider the points of the segment S1 S4 . Let D and E be the sets of points defined as follows: D = {X ∈ [S1 S4 ] | d(X , LA0 T2 ) ≤ 1/2} and E = {X ∈ [S1 S4 ] | d(X , LT2 An ) ≤ 1/2}. Clearly D and E are closed, nonempty (indeed, S1 ∈ D and S4 ∈ E) subsets of [S1 S4 ]. Since their union is a connected set S1 S4 , it follows that they must have a nonempty intersection. Let P ∈ D ∩ E. Then there exist points X ∈ LA0 T2 and Y ∈ LT2 An such that d(P, X) ≤ 1/2, d(P,Y ) ≤ 1/2, and consequently d(X ,Y ) ≤ 1. On the other hand, T2 lies between X and Y on L, and thus LXY = LXT2 + LT2Y ≥ X T2 + T2Y ≥ (PS2 − X P − S2 T2 ) + (PS2 − Y P − S2 T2 ) ≥ 99 + 99 = 198. 4.23 Shortlisted Problems 1982 463 7. Let a, b, ab be the roots of the cubic polynomial p(x) = (x − a)(x − b)(x − ab). Observe that 2p(−1) = −2(1 + a)(1 + b)(1 + ab); p(1) + p(−1) − 2(1 + p(0)) = −2(1 + a)(1 + b). The statement of the problem is trivial if both the expressions are equal to 2p(−1) zero. Otherwise, the quotient p(1)+p(−1)−2(1+p(0)) = 1 + ab is rational and consequently ab is rational. But since (ab)2 = −p(0) is an integer, it follows that ab is also an integer. This completes the proof. 8. Let F be the given figure. Consider any chord AB of the circumcircle γ that supports F . The other supporting lines to F from A and B intersect γ again at D and C respectively so that ∠DAB = ∠ABC = 90◦ . Then ABCD is a rectangle, and hence CD must support F as well, from which it follows that F is inscribed in the rectangle ABCD touching each of its sides. We easily conclude that F is the intersection of all such rectangles. Now, since the center O of γ is the center of symmetry of all these rectangles, it must be so for their intersection F as well. 9. Let X and Y be the midpoints of the segments AP and BP. Then DY PX is a C parallelogram. Since X and Y are the circumcenters of the triangles APM and BPL, we conclude that X M = L XP = DY and Y L = Y P = DX . FurP M thermore, we have ∠DXM = ∠DX P + ∠PXM = ∠DXP + 2∠PAM. SimiY X larly, ∠DY L = ∠DY P + 2∠PBL hence ∠DXM = ∠DY L. Therefore, the trianB A D gles DXM and LY D are congruent, implying DM = DL. 10. If the two balls taken from the box are both white, then the number of white balls decreases by two; otherwise, it remains unchanged. Hence the parity of the number of white balls does not change during the procedure. Therefore if p is even, the last ball cannot be white; the probability is 0. If p is odd, the last ball has to be white; the probability is 1. 11. (a) Suppose {a1 , a2 , . . . , an } is the arrangement that yields the maximal value Qmax of Q. Note that the value of Q for the rearrangement {a1 , . . . , ai−1 , a j , a j−1 , . . . , ai , a j+1 , . . . , an } equals Qmax − (ai − a j )(ai−1 − a j+1), where 1 < i < j < n. Hence (ai − a j )(ai−1 − a j+1 ) ≥ 0 for all 1 < i < j < n. We may suppose w.l.o.g. that a1 = 1. Let ai = 2. If 2 < i < n, then (a2 − ai )(a1 − ai+1 ) < 0, which is impossible. Therefore i is either 2 or n; let w.l.o.g. an = 2. Further, if a j = 3 for 2 < j < n, then (a1 − a j+1 )(a2 − a j ) < 0, which is impossible; therefore a2 = 3. Continuing this argument we obtain that A = {1, 3, 5, . . . , 2[(n − 1)/2] + 1, 2[n/2], . . ., 4, 2}. (b) A similar argument leads to the minimizing rearrangement {1, n, 2, n − 1, . . . , [n/2] + 1}. 464 4 Solutions 12. Let y be the line perpendicular to L passing through the center of C. It can be shown by a continuity argument that there exists a point Y ∈ y such that an b inversion Ψ centered at Y maps C and L onto two concentric circles Cb and L. Let Xb denote the image of an object X under Ψ . Then the circles Cbi touch Cb externally and b L internally, and all have the same radius. Let us now rotate the c3 passes through Y . picture around the common center Z of Cb and b L so that C Applying the inversion Ψ again on the picture thus obtained, Cb and b L go back to c3 goes to a line C′ parallel to L, while the images of C c1 and C c2 go C and L, but C 3 ′ ′ ′ to two equal circles C1 and C2 touching L, C3 , and C. This way we have achieved that C3 becomes a line. C3′ Denote by O1 , O2 , O respectively the O 1 centers of the circles C1′ ,C2′ ,C and by T x x ′ the point of tangency of the circles C1′ C1 C2′ O1 O2 T and C2′ . If x is the common radius of the circles C1′ and C2′ , then from △O1 T O L we obtain that (x − 1)2 + x2 = (x + 1)2 , and thus x = 4. Hence the distance of O from L equals 2x − 1 = 7. 13. The points S1 , S2 , S3 clearly lie A3 c deon the inscribed circle. Let XY note the oriented arc XY . The arcs T1 d Td 2 S1 and T1 T3 are equal, since they S2 are symmetric with respect to the S1 T2 bisector of ∠A1 . Similarly, Td 3 T2 = d d d Sd 2 T1 . Hence T3 S1 = T3 T2 + T2 S1 = S3 d d d S2 T1 + T1 T3 = S2 T3 . It follows that A1 A2 T3 S1 S2 is parallel to A1 A2 , and consequently S1 S2 k M1 M2 . Analogously S1 S3 k M1 M3 and S2 S3 k M2 M3 . Since the circumcircles of △M1 M2 M3 and △S1 S2 S3 are not equal, these triangles are not congruent and hence they must be homothetic. Then all the lines Mi Si pass through the center of homothety. Second solution. Set the complex plane so that the incenter of △A1 A2 A3 is the unit circle centered at the origin. Let ti , si respectively denote the complex numbers of modulus 1 corresponding to Ti , Si . Clearly t1t1 = t2t2 = t3 t3 = 1. Since T2 T3 and T1 S1 are parallel, we obtain t2 t3 = t1 s1 , or s1 = t2 t3t1 . Similarly s2 = t1 t3 t2 , s3 = t1t2 t3 , from which it follows that s2 − s3 = t1 (t3 t2 − t2t3 ). Since the number in parentheses is strictly imaginary, we conclude that OT1 ⊥ S2 S3 and consequently S2 S3 k A2 A3 . We proceed as in the first solution. 14. (a) If any two of A1 , B1 ,C1 , D1 coincide, say A1 ≡ B1 , then ABCD is inscribed in a circle centered at A1 and hence all A1 , B1 ,C1 , D1 coincide. Assume now the opposite, and let w.l.o.g. ∠DAB + ∠DCB < 180◦ . Then A is outside the circumcircle of △BCD, so A1 A > A1C. Similarly, C1C > C1 A. Hence the perpendicular bisector lAC of AC separates points A1 and C1 . Since B1 , D1 lie on lAC , this means that A1 and C1 are on opposite sides 4.23 Shortlisted Problems 1982 465 B1 D1 . Similarly one can show that B1 and D1 are on opposite sides of A1C1 . (b) Since A2 B2 ⊥ C1 D1 and C1 D1 ⊥ AB, it follows that A2 B2 k AB. Similarly A2C2 k AC, A2 D2 k AD, B2C2 k BC, B2 D2 k BD, and C2 D2 k CD. Hence △A2 B2C2 ∼ △ABC and △A2 D2C2 ∼ △ADC, and the result follows. 15. Let a = k/n, where n, k ∈ N, n ≥ k. Putting t n = s, the given inequality becomes 1−t k n k/n−1 , or equivalently 1−t n ≤ (1 + t ) (1 + t + · · · + t k−1 )n (1 + t n )n−k ≤ (1 + t + · · · + t n−1 )n . This is clearly true for k = n. Therefore it is enough to prove that the left-hand side of the above inequality is an increasing function of k. We are led to show that (1 + t + · · · + t k−1 )n (1 + t n )n−k ≤ (1 + t + · · · + t k )n (1 + t n )n−k−1 . This is 1+t+···+t k equivalent to 1 + t n ≤ An , where A = 1+t+···+t k−1 . But this easily follows, since An − t n = (A − t)(An−1 + An−2t + · · · + t n−1) ≥ (A − t)(1 + t + · · · + t n−1 ) = 1 + t + · · · + t n−1 ≥ 1. 1 + t + · · · + t k−1 Remark. The original problem asked to prove the inequality for real a. 16. It is easy to verify that whenever (x, y) is a solution of the equation x3 − 3xy2 + y3 = n, so are the pairs (y−x, −x) and (−y, x−y). No two of these three solutions are equal unless x = y = n = 0. Observe that 2891 ≡ 2 (mod 9). Since x3 , y3 ≡ 0, ±1 (mod 9), x3 − 3xy2 + y3 cannot give the remainder 2 when divided by 9. Hence the above equation for n = 2891 has no integer solutions. 17. Let A be the origin of the Cartesian plane. Suppose that BC : AC = k and that (a, b) and (a1 , b1 ) are coordinates of the points C and C1 , respectively. Then the coordinates of the point B are (a, b) + k(−b, a) = (a − kb, b + ka), while the coordinates of B1 are (a1 , b1 ) + k(b1 , −a1 ) = (a + kb1 , b1 − ka1 ). Thus the lines x−(a−kb) x−(a1 +kb1 ) x−a 1 BC1 and CB1 are given by the equations x−a y−b1 = y−(b+ka) and y−b = y−(b1 −ka1 ) respectively. After multiplying, these equations transform into the forms BC1 : CB1 : kax + kby = kaa1 + kbb1 + ba1 − ab1 − (b − b1)x + (a − a1)y ka1 x + kb1y = kaa1 + kbb1 + ba1 − ab1 − (b − b1)x + (a − a1)y. The coordinates (x0 , y0 ) of the point M satisfy these equations, from which we deduce that kax0 + kby0 = ka1 x0 + kb1 y0 . This yields xy00 = − ba11 −b −a , implying that the lines CC1 and AM are perpendicular. 18. Set the coordinate system with the axes x, y, z along the lines l1 , l2 , l3 respectively. The coordinates (a, b, c) of M satisfy a2 + b2 + c2 = R2 , and so SM is given by the equation (x − a)2 + (y − b)2 + (z − c)2 = R2 . Hence the coordinates of P1 are (x, 0, 0) with (x − a)2 + b2 + c2 = R2 , implying that either x = 2a or x = 0. 466 4 Solutions Thus by the definition we obtain x = 2a. Similarly, the coordinates of P2 and P3 are (0, 2b, 0) and (0, 0, 2c) respectively. Now, the centroid of △P1 P2 P3 has the coordinates (2a/3, 2b/3, 2c/3). Therefore the required locus of points is the sphere with center O and radius 2R/3. √ √ 19. Let us set x = m/n. Since f (x) = (m + n)/ m2 + n2 = (x + 1)/ 1 + x2 is a continuous function of x, f (x) takes all values between any two values of f ; moreover, the corresponding x can be rational. This completes the proof. Remark. Since f is increasing for x ≥ 1, 1 ≤ x < z < y implies f (x) < f (z) < f (y). 20. Since MN is the image of AC under rotation about B for 60◦ , we have MN = AC. Similarly, PQ is the image of AC under rotation about D through 60◦ , from which it follows that PQ k MN. Hence either M, N, P, Q are collinear or MNPQ is a parallelogram. 4.24 Shortlisted Problems 1983 467 4.24 Solutions to the Shortlisted Problems of IMO 1983 1. Suppose that there are n airlines A1 , . . . , An and N > 2n cities. We shall prove that there is a round trip by at least one Ai containing an odd number of stops. For n = 1 the statement is trivial, since one airline serves at least 3 cities and hence P1 P2 P3 P1 is a round trip with 3 landings. We use induction on n, and assume that n > 1. Suppose the contrary, that all round trips by An consist of an even number of stops. Then we can separate the cities into two nonempty classes Q = {Q1 , . . . , Qr } and R = {R1 , . . . , Rs } (where r + s = N), so that each flight by An runs between a Q-city and an R-city. (Indeed, take any city Q1 served by An ; include each city linked to Q1 by An in R, then include in Q each city linked by An to any R-city, etc. Since all round trips are even, no contradiction can arise.) At least one of r, s is larger than 2n−1 , say r > 2n−1 . But, only A1 , . . . , An−1 run between cities in {Q1 , . . . , Qr }; hence by the induction hypothesis at least one of them flies a round trip with an odd number of landings, a contradiction. It only remains to notice that for n = 10, 2n = 1024 < 1983. Remark. If there are N = 2n cities, there is a schedule with n airlines that contain no odd round trip by any of the airlines. Let the cities be Pk , k = 0, . . . , 2n − 1, and write k in the binary system as an n-digit number a1 . . . an (e.g., 1 = (0 . . . 001)2 ). Link Pk and Pl by Ai if the ith digits k and l are distinct but the first i − 1 digits are the same. All round trips under Ai are even, since the ith digit alternates. 2. By definition, σ (n) = ∑d|n d = ∑d|n n/d = n ∑d|n 1/d, hence σ (n)/n = ∑d|n 1/d. In particular, σ (n!)/n! = ∑d|n! 1/d ≥ ∑nk=1 1/k. It follows that the sequence σ (n)/n is unbounded, and consequently there exist an infinite number of integers n such that σ (n)/n is strictly greater than σ (k)/k for k < n. 3. (a) A circle is not Pythagorean. Indeed, consider the partition into two semicircles each closed at one and open at the other end. (b) An equilateral triangle, call it PQR, is Pythagorean. Let P′ , Q′ , and R′ be the points on QR, RP, and PQ such that PR′ : R′ Q = QP′ : P′ R = RQ′ : Q′ P = 1 : 2. Then Q′ R′ ⊥ PQ, etc. Suppose that PQR is not Pythagorean, and consider a partition into A, B, neither of which contains the vertices of a right-angled triangle. At least two of P′ , Q′ , and R′ belong to the same class, say P′ , Q′ ∈ A. Then [PR] \ {Q′ } ⊂ B and hence R′ ∈ A (otherwise, if R′′ is the foot of the perpendicular from R′ to PR, △RR′ R′′ is right-angled with all vertices in B). But this implies again that [PQ] \ {R′ } ⊂ B, and thus B contains vertices of a rectangular triangle. This is a contradiction. 4. The rotational homothety centered at C that sends B to R also sends A to Q; hence the triangles ABC and QRC are similar. For the same reason, △ABC and △PBR are similar. Moreover, BR = CR; hence △CRQ ∼ = △RBP. Thus PR = QC = AQ and QR = PB = PA, so APQR is a parallelogram. 5. Each natural number p can be written uniquely in the form p = 2q (2r − 1). We call 2r − 1 the odd part of p. Let An = (a1 , a2 , . . . , an ) be the first sequence. Clearly the terms of An must have different odd parts, so those parts must be at 468 4 Solutions least 1, 3, . . . , 2n − 1. Being the first sequence, An must have the numbers 2n − 1, 2n − 3, . . ., 2k + 1 as terms, where k = [n + 1/3] (then 3(2k − 1) < 2n − 1 < 3(2k + 1)). Smaller odd numbers 2s + 1 (with s < k) obviously cannot be terms of An . In this way we have obtained the n − k odd numbers of An . The other k terms must be even, and by the same reasoning as above they must be precisely the terms of 2Ak (twice the terms of Ak ). Therefore An is defined recursively as A0 = 0, / A1 = {1}, A2 = {3, 2}; An = {2n − 1, 2n − 3, . . ., 2k + 1} ∪ 2Ak . 6. The existence of r: Let S = {x1 + x2 + · · · + xi − 2i | i = 1, 2, . . . , n}. Let max S be attained for the first time at r′ . If r′ = n, then x1 + x2 + · · · + xi − 2i < 2 for 1 ≤ i ≤ n − 1, so one can take r = r′ . Suppose that r′ < n. Then for l < n − r′ we have xr′ +1 + xr′ +2 + · · · + xr′ +l = (x1 + · · · + xr′ +l − 2(r′ + l)) − (x1 + · · · + xr′ − 2r′ ) + 2l ≤ 2l; also, for i < r′ we have (xr′ +1 + · · · + xn ) + (x1 + · · · + xi − 2i) < (xr′ +1 + · · · + xn ) + (x1 + · · · + xr′ − 2r′ ) = (x1 + · · · + xn ) − 2r′ = 2(n − r′ ) + 2 ⇒ xr′ +1 + · · · + xn + x1 + · · · + xi ≤ 2(n + i − r′) + 1, so we can again take r = r′ . For the second part of the problem, we relabel the sequence so that r = 0 works. Suppose that the inequalities are strict. We have x1 + x2 + · · · + xk ≤ 2k, k = 1, . . . , n − 1. Now, 2n + 2 = (x1 + · · · + xk ) + (xk+1 + · · · + xn ) ≤ 2k + xk+1 + · · · + xn ⇒ xk+1 + · · · + xn ≥ 2(n − k) + 2 > 2(n − k) + 1. So we cannot begin with xk+1 for any k > 0. Now assume that there is an equality for some k. There are two cases: (i) Suppose x1 + x2 + · · · + xi ≤ 2i (i = 1, . . . , k) and x1 + · · · + xk = 2k + 1, x1 + · · · + xk+l ≤ 2(k + l) + 1 (1 ≤ l ≤ n − 1 − k). For i ≤ k − 1 we have xi+1 + · · · + xn = 2(n + 1) − (x1 + · · · + xi ) > 2(n − i) + 1, so we cannot take r = i. If there is a j ≥ 1 such that x1 + x2 + · · · + xk+ j ≤ 2(k + j), then also xk+ j+1 + · · · + xn > 2(n − k − j) + 1. If (∀ j ≥ 1) x1 + · · · + xk+ j = 2(k + j) + 1, then xn = 3 and xk+1 = · · · = xn−1 = 2. In this case we directly verify that we cannot take r = k + j. However, we can also take r = k: for k + l ≤ n − 1, xk+1 + · · · + xk+l ≤ 2(k + l) + 1 − (2k + 1) = 2l, also xk+1 + · · · + xn = 2(n − k) + 1, and moreover x1 ≤ 2, x1 + x2 ≤ 4, . . . . (ii) Suppose x1 + · · ·+ xi ≤ 2i (1 ≤ i ≤ n − 2) and x1 + · · ·+ xn−1 = 2n − 1. Then we can obviously take r = n − 1. On the other hand, for any 1 ≤ i ≤ n − 2, xi+1 + · · · + xn−1 + xn = (x1 + · · · + xn−1 ) − (x1 + · · · + xi ) + 3 > 2(n − i) + 1, so we cannot take another r 6= 0. √ √ √ √ √ 7. Clearly, each an is positive and an+1 = an a + 1 + an + 1 a. Notice that √ √ √ √ √ an+1 + 1 = a + 1 an + 1 + a an . Therefore √ √ p √ ( a + 1 − a)( an + 1 − an ) p p √ √ √ √ √ √ = ( a + 1 an + 1 + a an ) − ( an a + 1 + an + 1 a) p √ = an+1 + 1 − an+1 . 4.24 Shortlisted Problems 1983 √ √ √ By induction, an+1 − an = √ √ n a + 1 + a . Hence, √ an = 469 √ n √ √ a + 1 − a . Similarly, an+1 + an = √ n √ √ n i 1 h√ a +1+ a − a +1− a , 2 from which the result follows. 8. Situations in which the condition of the statement is fulfilled are the following: S1 : N1 (t) = N2 (t) = N3 (t) S2 : Ni (t) = N j (t) = h, Nk (t) = h + 1, where (i, j, k) is a permutation of the set {1, 2, 3}. In this case the first student to leave must be from row k. This leads to the situation S1 . S3 : Ni (t) = h, N j (t) = Nk (t) = h + 1, ((i, j, k) is a permutation of the set {1, 2, 3}). In this situation the first student leaving the room belongs to row j (or k) and the second to row k (or j). After this we arrive at the situation S1 . Hence, the initial situation is S1 and after each triple of students leaving the room the situation S1 must recur. We shall compute the probability Ph that from a situation S1 with 3h students in the room (h ≤ n) one arrives at a situation S1 with 3(h − 1) students in the room: Ph = (3h) · (2h) · h 3!h3 = . (3h) · (3h − 1) · (3h − 2) 3h(3h − 1)(3h − 2) Since the room becomes empty after the repetition of n such processes, which are independent, we obtain for the probability sought n P = ∏ Ph = h=1 (3!)n (n!)3 . (3n)! 9. For any triangle of sides a, b, c there exist 3 nonnegative numbers x, y, z such that a = y + z, b = z + x, c = x + y (these numbers correspond to the division of the sides of a triangle by the point of contact of the incircle). The inequality becomes (y + z)2 (z + x)(y − x) + (z + x)2(x + y)(z − y) + (x + y)2(y + z)(x − z) ≥ 0. Expanding, we get xy3 + yz3 + zx3 ≥ xyz(x + y + z). This follows from Cauchy’s 2 √ inequality (xy3 + yz3 + zx3 )(z + x + y) ≥ xyz(x + y + z) with equality if and only if xy3 /z = yz3 /x = zx3 /y, or equivalently x = y = z, i.e., a = b = c.  10. Choose P(x) = qp (qx − 1)2n+1 + 1 , I = [1/2q, 3/2q]. Then all the coefficients of P are integers, and P(x) − p p p 1 = (qx − 1)2n+1 ≤ , q q q 22n+1 for x ∈ I. The desired inequality follows if n is chosen large enough. 470 4 Solutions 11. First suppose that the binary representation of x is finite: x = 0, a1 a2 . . . an = ∑nj=1 a j 2− j , ai ∈ {0, 1}. We shall prove by induction on n that  n −b if ak = 0, f (x) = ∑ b0 . . . b j−1 a j , where bk = 1 − b if ak = 1. j=1 (Here a0 = 0.) Indeed, by the recursion formula, n−1 a1 = 0 ⇒ f (x) = b f ∑ a j+12 j=1 −j ! n−1 = b ∑ b1 . . . b j a j+1 j=1 n−1 hence f (x) = ∑ b0 . . . b j a j+1 as b0 = b1 = b; j=0 n−1 a1 = 1 ⇒ f (x) = b + (1 − b) f ∑ a j+12− j j=1 ! n−1 = ∑ b0 . . . b j a j+1, j=0 as b0 = b, b1 = 1 − b. Clearly, f (0) = 0, f (1) = 1, f (1/2) = b > 1/2. Assume x = ∑nj=0 a j 2− j , and for k ≥ 2, v = x + 2−n−k+1 , u = x + 2−n−k = (v + x)/2. Then f (v) = f (x) + b0 . . . bn bk−2 and f (u) = f (x) + b0 . . . bn bk−1 > ( f (v) + f (x))/2. This means that the point (u, f (u)) lies above the line joining (x, f (x)) and (v, f (v)). By induction, every (x, f (x)), where x has a finite binary expansion, lies above the line joining (0, 0) and (1/2, b) if 0 < x < 1/2, or above the line joining (1/2, b) and (1, 1) if 1/2 < x < 1. It follows immediately that f (x) > x. For the second inequality, observe that f (x) − x = < ∞ ∑ (b0 . . . b j−1 − 2− j )a j j=1 ∞ ∑ (b j − 2− j )a j < j=1 ∞ b ∑ (b j − 2− j ) = 1 − b − 1 = c. j=1 By continuity, these inequalities also hold for x with infinite binary representations. 12. Putting y = x in (i) we see that there exist positive real numbers z such that f (z) = z (this is true for every z = x f (x)). Let a be any of them. Then f (a2 ) = f (a f (a)) = a f (a) = a2 , and by induction, f (an ) = an . If a > 1, then an → +∞ as n → ∞, and we have a contradiction with (ii). Again, a = f (a) = f (1 · a) = a f (1), so f (1) = 1. Then, a f (a−1 ) = f (a−1 f (a)) = f (1) = 1, and by induction, f (a−n ) = a−n . This shows that a 6< 1. Hence, a = 1. It follows that x f (x) = 1, i.e., f (x) = 1/x for all x. This function satisfies (i) and (ii), so f (x) = 1/x is the unique solution. 13. Given any coloring of the 3 × 1983 − 2 points of the axes, we prove that there is a unique coloring of E having the given property and extending this coloring. 4.24 Shortlisted Problems 1983 471 The first thing to notice is that given any rectangle R1 parallel to a coordinate plane and whose edges are parallel to the axes, there is an even number r1 of red vertices on R1 . Indeed, let R2 and R3 be two other rectangles that are translated from R1 orthogonally to R1 and let r2 , r3 be the numbers of red vertices on R2 and R3 respectively. Then r1 + r2 , r1 + r3 , and r2 + r3 are multiples of 4, so r1 = (r1 + r2 + r1 + r3 − r2 − r3 )/2 is even. Since any point of a coordinate plane is a vertex of a rectangle whose remaining three vertices lie on the corresponding axes, this determines uniquely the coloring of the coordinate planes. Similarly, the coloring of the inner points of the parallelepiped is completely determined. The solution is hence 23×1983−2 = 25947. 14. Let Tn be the set of all nonnegative integers whose ternary representations consist of at most n digits and do not contain a digit 2. The cardinality of Tn is 2n , and the greatest integer in Tn is 11 . . . 1 = 30 + 31 +· · ·+ 3n−1 = (3n − 1)/2. We claim that there is no arithmetic triple in Tn . To see this, suppose x, y, z ∈ Tn and 2y = x + z. Then 2y has only 0’s and 2’s in its ternary representation, and a number of this form can be the sum of two integers x, z ∈ Tn in only one way, namely x = z = y. But |T10 | = 210 = 1024 and max T10 = (310 − 1)/2 = 29524 < 30000. Thus the answer is yes. 15. There is no such set. Suppose that M satisfies the conditions (i) and (ii) and let qn = |{a ∈ M : a ≤ n}|. Consider the differences b − a, where a, b ∈ M and 10 < a < b ≤ k. They are all positive and less √ implies that they  than k, and (ii) 10 different integers. Hence qk −q10 < k, so q ≤ are qk −q 2k + 10. It follows k 2 2 from (i) that among the numbers of the form a + b, where a, b ∈ M, a ≤ b ≤ n, or a ≤ n < b ≤ 2n, there are all integers from the √ interval [2, 2n + 1]. Thus qn +1 + q (q − q ) ≥ 2n for every n ∈ N. Set Q = 2k + 10. We have n n 2n k 2   qn + 1 1 1 + qn (q2n − qn ) = qn + qn (2q2n − qn ) 2 2 2 1 1 ≤ qn + qn (2Q2n − qn ) 2 2 1 1 ≤ Qn + Qn (2Q2n − Qn ) 2√ 2 √ √ ≤ 2( 2 − 1)n + (20 + 2/2) n + 55, which is less than n for n large enough, a contradiction. 16. Set hn,i (x) = xi + · · · + xn−i , 2i ≤ n. The set F(n) is the set of linear combinations with nonnegative coefficients of the hn,i ’s. This is a convex cone. Hence, it suffices to prove that hn,i hm, j ∈ F(m + n). Indeed, setting p = n − 2i and q = m − 2 j and assuming p ≤ q we obtain hn,i (x)hm, j (x) = (xi + · · · + xi+p )(x j + · · · + x j+q ) = which proves the claim. n−i+ j ∑ k=i+ j hm+n,k , 472 4 Solutions 17. Set a = minPi Pj , b = max Pi Pj . We use the following lemma. √ Lemma. There exists a disk of radius less than or equal to b/ 3 containing all the Pi ’s. Assuming that this is proved, the disks √ with center Pi and radius a/2 are disjoint and included in a disk of radius b/ 3 + a/2; hence comparing areas,  2 √ √ a2 b nπ · < π · √ + a/2 and b > 3/2 · ( n − 1)a. 4 3 Proof of the lemma. If a nonobtuse triangle with sides a ≥ b√≥ c has a circumscribed circle of radius R, we have R = a/(2 sin α ) ≤ a/ 3. Now we show that there exists a disk D of radius R containing A = {P1 , . . . , Pn } whose border C is such that C ∩ A is not included in an open semicircle, and hence contains either two diametrically opposite points and R ≤ b/2, or an acute√ angled triangle and R ≤ b/ 3. Among all disks whose borders pass through three points of A and that contain all of A, let D be the one of least radius. Suppose that C ∩A is contained in an arc of central angle less than 180◦ , and that Pi , Pj are its endpoints. Then there exists a circle through Pi , Pj of smaller radius that contains A, a contradiction. Thus D has the required property, and the assertion follows. 18. Let (x0 , y0 , z0 ) be one solution of bcx + cay + abz = n (not necessarily nonnegative). By subtracting bcx0 + cay0 + abz0 = n we get bc(x − x0 ) + ca(y − y0) + ab(z − z0) = 0. Since (a, b) = (a, c) = 1, we must have a|x − x0 or x − x0 = as. Substituting this in the last equation gives bcs + c(y − y0) + b(z − z0) = 0. Since (b, c) = 1, we have b|y − y0 or y − y0 = bt. If we substitute this in the last equation we get bcs + bct + b(z − z0 ) = 0, or cs + ct + z − z0 = 0, or z − z0 = −c(s +t). In x = x0 + as and y = y0 + bt, we can choose s and t such that 0 ≤ x ≤ a − 1 and 0 ≤ y ≤ b − 1. If n > 2abc − bc − ca − ab, then abz = n − bcx − acy > 2abc − ab − bc − ca − bc(a − 1) − ca(b − 1) = −ab or z > −1, i.e., z ≥ 0. Hence, it is representable as bcx + cay + abz with x, y, z ≥ 0. Now we prove that 2abc − bc − ca − ab is not representable as bcx + cay + abz with x, y, z ≥ 0. Suppose that bcx + cay + abz = 2abc − ab − bc − ca with x, y, z ≥ 0. Then bc(x + 1) + ca(y + 1) + ab(z + 1) = 2abc with x + 1, y + 1, z + 1 ≥ 1. Since (a, b) = (a, c) = 1, we have a|x + 1 and thus a ≤ x + 1. Similarly b ≤ y + 1 and c ≤ z + 1. Thus bca + cab + abc ≤ 2abc, a contradiction. 19. For all n, there exists a unique polynomial Pn of degree n such that Pn (k) = Fk for n + 2 ≤ k ≤ 2n + 2 and Pn (2n + 3) = F2n+3 − 1. For n = 0, we have F1 = 4.24 Shortlisted Problems 1983 473 F2 = 1, F3 = 2, P0 = 1. Now suppose that Pn−1 has been constructed and let Pn be the polynomial of degree n satisfying Pn (X + 2) − Pn (X + 1) = Pn−1 (X ) and Pn (n + 2) = Fn+2 . (The mapping Rn [X ] → Rn−1 [X ] × R, P 7→ (Q, P(n + 2)), where Q(X) = P(X + 2) − P(X + 1), is bijective, since it is injective and those two spaces have the same dimension; clearly degQ = degP−1.) Thus for n+2 ≤ k ≤ 2n + 2 we have Pn (k + 1) = Pn (k) + Fk−1 and Pn (n + 2) = Fn+2 ; hence by induction on k, Pn (k) = Fk for n + 2 ≤ k ≤ 2n + 2 and Pn (2n + 3) = F2n+2 + Pn−1(2n + 1) = F2n+3 − 1. Finally, P990 is exactly the polynomial P of the terms of the problem, for P990 − P has degree less than or equal to 990 and vanishes at the 991 points k = 992, . . ., 1982. 20. If (x1 , x2 , . . . , xn ) satisfies the system with parameter a, then (−x1 , −x2 , . . . , −xn ) satisfies the system with parameter −a. Hence it is sufficient to consider only a ≥ 0. Let (x1 , . . . , xn ) be a solution. Suppose x1 ≤ a, x2 ≤ a, . . . , xn ≤ a. Summing the equations we get (x1 − a)2 + · · · + (xn − a)2 = 0 and see that (a, a, . . . , a) is the only such solution. Now suppose that xk ≥ a for some k. According to the kth equation, xk+1 |xk+1 | = x2k − (xk − a)2 = a(2xk − a) ≥ a2 , which implies that xk+1 ≥ a as well (here xn+1 = x1 ). Consequently, all x1 , x2 , . . . , xn are greater than or equal to a, and as above (a, a, . . . , a) is the only solution. 21. Using the identity an − bn = (a − b) n−1 ∑ an−m−1bm m=0 with a = k1/n and b = (k − 1)1/n one obtains   1 < k1/n − (k − 1)1/n nk1−1/n for all integers n > 1 and k ≥ 1.  This gives us the inequality k1/n−1 < n k1/n − (k − 1)1/n if n > 1 and k ≥ 1. In  a similar way one proves that n (k + 1)1/n − k1/n < k1/n−1 if n > 1 and k ≥ 1. Hence for n > 1 and m > 1 it holds that  m  n ∑ (k + 1)1/n − k1/n < k=1 m ∑ k1/n−1 k=1 m   < n ∑ k1/n − (k − 1)1/n + 1, k=2 or equivalently, 474 4 Solutions   n (m + 1)1/n − 1 < The choice n = 1983 and m = m ∑ k1/n−1 < n k=1 21983 then gives   m1/n − 1 + 1. 21983 1983 < ∑ k1/1983−1 < 1984. k=1 Therefore the greatest integer less than or equal to the given sum is 1983. 22. Decompose n into n = st, where the greatest common divisor of s and t is 1 and where s > 1 and t > 1. For 1 ≤ k ≤ n put k = vs + u, where 0 ≤ v ≤ t − 1 and 1 ≤ u ≤ s, and let ak = avs+u be the unique integer in the set {1, 2, 3, . . ., n} such that vs + ut − avs+u is a multiple of n. To prove that this construction gives a permutation, assume that ak1 = ak2 , where ki = vi s + ui, i = 1, 2. Then (v1 − v2 )s + (u1 − u2 )t is a multiple of n = st. It follows that t divides (v1 − v2 ), while |v1 − v2 | ≤ t − 1, and that s divides (u1 − u2 ), while |u1 − u2 | ≤ s − 1. Hence, v1 = v2 , u1 = u2 , and k1 = k2 . We have proved that a1 , . . . , an is a permutation of {1, 2, . . . , n} and hence  ! n 2π ak t−1 s 2π v 2π u . ∑ k cos n = ∑ ∑ (vs + u) cos t + s v=0 u=1 k=1 Using ∑su=1 cos(2π u/s) = ∑su=1 sin(2π u/s) = 0 and the additive formulas for cosine, one finds that ! n t−1 2π ak 2π v s 2π u 2π v s 2π u ∑ k cos n = ∑ cos t ∑ u cos s − sin t ∑ u sin s v=0 u=1 u=1 k=1 ! ! s t−1 2π u 2π v = ∑ u cos ∑ cos t s u=1 v=0 ! ! s t−1 2π u 2π v − ∑ u sin ∑ sin t = 0. s u=1 v=0 23. We note that ∠O1 KO2 = ∠M1 KM2 is equivalent to ∠O1 KM1 = ∠O2 KM2 . Let S be the intersection point of the common tangents, and let L be the second P1 point of intersection of SK and W1 . L P2 Since △SO1 P1 ∼ △SP1M1 , we have K 2 SK · SL = SP1 = SO1 · SM1 which imS plies that points O1 , L, K, M1 lie on a O1 M1 O2 M2 circle. Hence ∠O1 KM1 = ∠O1 LM1 = Q2 ∠O2 KM2 . Q1 24. See the solution of (SL91-15). 4.24 Shortlisted Problems 1983 475 25. Suppose the contrary, that R3 = P1 ∪ P2 ∪ P3 is a partition such that a1 ∈ R+ is not realized by P1 , a2 ∈ R+ is not realized by P2 and a3 ∈ R+ not realized by P3 , where w.l.o.g. a1 ≥ a2 ≥ a3 . If P1 = 0/ = P2 , then P3 = R3 , which is impossible. If P1 = 0, / and X ∈ P2 , the sphere centered at X with radius a2 is included in P3 and a3 ≤ a2 is realized, which is impossible. If P1 6= 0, / let X1 ∈ P1 . The sphere S centered in X1 , of radius a1 is included in P2 ∩ P3 . Since a1 ≥ a3 , S 6⊂ P3 . Let X2 ∈ P2 ∩ S. The circle q {Y ∈ S | d(X2 ,Y ) = a2 } is √ included in P3 , but a2 ≤ a1 ; hence it has radius r = a2 1 − a22/(4a21 ) ≥ a2 3/2 √ and a3 ≤ a2 ≤ a2 3 < 2r; hence a3 is realized by P3 . 476 4 Solutions 4.25 Solutions to the Shortlisted Problems of IMO 1984 1. This is the same problem as (SL83-20). 2. (a) For m = t(t − 1)/2 and n = t(t + 1)/2 we have 4mn − m − n = (t 2 − 1)2 − 1. (b) Suppose that 4mn − m − n = p2 , or equivalently, (4m − 1)(4n − 1) = 4p2 + 1. The number 4m − 1 has at least one prime divisor, say q, that is of the form 4k + 3. Then 4p2 ≡ −1 (mod q). However, by Fermat’s theorem we have  q−1 q−1 1 ≡ (2p)q−1 = 4p2 2 ≡ (−1) 2 (mod q), which is impossible since (q − 1)/2 = 2k + 1 is odd. 3. From the equality n = d62 + d72 − 1 we see that d6 and d7 are relatively prime and d7 | d62 − 1 = (d6 − 1)(d6 + 1), d6 | d72 − 1 = (d7 − 1)(d7 + 1). Suppose that d6 = ab, d7 = cd with 1 < a < b, 1 < c < d. Then n has 7 divisors smaller than d7 , namely 1, a, b, c, d, ab, ac, which is impossible. Hence, one of the two numbers d6 and d7 is either a prime p or the square p2 of a prime p 6= 2. Let it be di , {i, j} = {6, 7}; then di | (d j − 1)(d j + 1) implies that d j ≡ ±1 (mod di ), and consequently (di2 − 1)/d j ≡ ±1 as well. But either d j or (di2 − 1)/d j is less than di , and therefore equals di − 1 or equals 1. The only nontrivial possibilities are (di2 − 1)/d j = 1 and d j = di ± 1. In the first case we get di < d j ; hence d7 = d62 − 1 = (d6 − 1)(d6 + 1); hence d6 + 1 is a divisor of n that is between d6 and d7 . This is impossible. We thus conclude that d7 = d6 + 1. Setting d6 = x, d7 = x + 1 we obtain that n = x2 + (x + 1)2 − 1 = 2x(x + 1) is even. (i) Assume that one of x, x+1 is a prime p. The other one has at most 6 divisors and hence must be of the form 23 , 24 , 25 , 2q, 2q2, 4q, where q is an odd prime. The numbers 23 and 24 are easily eliminated, while 25 yields the solution x = 31, x + 1 = 32, n = 1984. Also, 2q is eliminated because n = 4pq then has only 4 divisors less than x; 2q2 is eliminated because n = 4pq2 has at least 6 divisors less than x; 4q is also eliminated because n = 8pq has 6 divisors less than x. (ii) Assume that one of x, x + 1 is p2 . The other one has at most 5 divisors (p excluded), and hence is of the form 23 , 24 , 2q, where q is an odd prime. The number 23 yields the solution x = 8, x + 1 = 9, n = 144, while 24 is easily eliminated. Also, the number 2q is eliminated because n = 4p2 q has 6 divisors less than x. Thus there are two solutions in total: 144 and 1984. 4. Consider the convex n-gon A1 A2 . . . An (the indices are considered modulo n). For any diagonal Ai A j we have Ai A j + Ai+1 A j+1 > Ai Ai+1 + A j A j+1 . Summing all such n(n − 3)/2 inequalities, we obtain 2d > (n − 3)p, proving the first inequality. Let us now prove the second inequality. We notice that for each diagonal Ai Ai+ j (we may assume w.l.o.g. that j ≤ [n/2]) the following relation holds: Ai Ai+ j < Ai Ai+1 + · · · + Ai+ j−1 Ai+ j . (1) 4.25 Shortlisted Problems 1984 477 If n = 2k + 1, then summing the inequalities (1) for j = 2, 3, . . . , k and i = 1, 2, . . . , n yields d < (2 + 3 + · · ·+ k)p = ([n/2][n + 1/2] − 2) p/2. If n = 2k, then summing the inequalities (1) for j = 2, 3, . . . , k − 1, i = 1, 2, . . . , n and for j = k, i = 1, 2, . . . , k again yields d < (2 + 3 + · · · + (k − 1) + k/2)p = 1 2 ([n/2][n + 1/2] − 2) p. 5. Let f (x, y, z) = xy + yz + zx − 2xyz. The first inequality follows immediately by adding xy ≥ xyz, yz ≥ xyz, and zx ≥ xyz (in fact, a stronger inequality xy + yz + zx − 9xyz ≥ 0 holds). Assume w.l.o.g. that z is the smallest of x, y, z. Since xy ≤ (x + y)2 /4 = (1 − z)2 /4 and z ≤ 1/2, we have xy + yz + zx − 2xyz = (x + y)z + xy(1 − 2z) (1 − z)2 (1 − 2z) ≤ (1 − z)z + 4 7 (1 − 2z)(1 − 3z)2 7 = − ≤ . 27 108 27 6. From the given recurrence we infer fn+1 − fn = fn − fn−1 + 2. Consequently, fn+1 − fn = ( f2 − f1 ) + 2(n − 1) = c − 1 + 2(n − 1). Summing up for n = 1, 2, . . . , k − 1 yields the explicit formula fk = f1 + (k − 1)(c − 1) + (k − 1)(k − 2) = k2 + bk − b, where b = c − 4. Now we easily obtain fk fk+1 = k4 + 2(b + 1)k3 + (b2 + b + 1)k2 − (b2 + b)k − b. We are looking for an r for which the last expression equals fr . Setting r = k2 + pk + q we get by a straightforward calculation that p = b + 1, q = −b, and r = k2 + (b + 1)k − b = fk + k. Hence fk fk+1 = f fk +k for all k. 7. It clearly suffices to solve the problem for the remainders modulo 4 (16 of each kind). (a) The remainders can be placed as shown in Figure 1, so that they satisfy the conditions. 10321032 23012301 32103210 p 01230123 qr s 10321032 t 23012301 32103210 01230123 Fig. 1 Fig. 2 (b) Suppose that the required numbering exists. Consider a part of the chessboard as in Figure 2. By the stated condition, all the numbers p + q + r + s, q + r + s + t, p + q + r + t, p + r + s + t give the same remainder modulo 4, and so do p, q, r, s. We deduce that all numbers on black cells of the 478 4 Solutions board, except possibly the two corner cells, give the same remainder, which is impossible. 8. Suppose that the statement of the problem is false. Consider two arbitrary circles R = (O, r) and S = (O, s) with 0 < r < s < 1. The point X ∈ R with α (X ) = r(s − r) < 2π satisfies that C(X ) = S. It follows that the color of the point X does not appear on S. Consequently, the set of colors that appear on R is not the same as the set of colors that appear on S. Hence any two distinct circles with center at O and radii less than 1 have distinct sets of colors. This is a contradiction, since there are infinitely many such circles but only finitely many possible sets of colors. 9. Let us show first that the system has at most one solution. Suppose that (x, y, z) and (x′ , y′ , z′ ) are two distinct solutions and that w.l.o.g. x < x′ . Then the √ second ′ and z > z′ , but then √y − a + z − a > and third equation imply that y > y √ √ ′ y − a + z′ − a, which is a contradiction. We shall now prove the existence of at least one solution. be an arbitrary √ Let P √ √ point in the plane and K, L, M points such that PK = a, PL = b, PM = c, and ∠KPL = ∠LPM = ∠MPK = 120◦ . The lines through K, L, M perpendicular respectively to PK, PL, PM form an equilateral √ triangle ABC, where K ∈ BC, L ∈ 2 AC,and M ∈ AB. Since its area equals AB 3/4 = S△BPC + S△APC + S△APB = √ √ √  AB a + b + c /2, it follows that AB = 1. Therefore x = PA2 , y = PB2 , and √ √ √ z√= PC2 is a solution of the system (indeed, y − a + z − a = PB2 − PK 2 + PC2 − PK 2 = BK +CK = 1, etc.). 10. Suppose that the product of some five consecutive numbers is a square. It is easily seen that among them at least one, say n, is divisible neither by 2 nor 3. Since n is coprime to the remaining four numbers, it is itself a square of a number m of the form 6k ± 1. Thus n = (6k ± 1)2 = 24r + 1, where r = k(3k ± 1)/2. Note that neither of the numbers 24r − 1, 24r + 5 is one of our five consecutive numbers because it is not a square. Hence the five numbers must be 24r, 24r + 1, . . . , 24r +4. However, the number 24r +4 = (6k ±1)2 +3 is divisible by 6r +1, which implies that it is a square as well. It follows that these two squares are 1 and 4, which is impossible. 11. Suppose that an integer x satisfies the equation. Then the numbers x − a1 , x − a2 , . . . , x − a2n are 2n distinct integers whose product is 1 · (−1) · 2 · (−2) · · · n · (−n). From here it is obvious that the numbers x − a1 , x − a2 , . . . , x − a2n are some reordering of the numbers −n, −n + 1, . . ., −1, 1, . . . , n − 1, n. It follows that their sum is 0, and therefore x = (a1 + a2 + · · · + a2n)/2n. This is the only solution if {a1 , a2 , . . . , a2n } = {x − n, . . ., x − 1, x + 1, . . .,x + n} for some x ∈ N. Otherwise there is no solution. 12. By the binomial formula we have (a + b)7 − a7 − b7 = 7ab[(a5 + b5 ) + 3ab(a3 + b3 ) + 5a2b2 (a + b)] = 7ab(a + b)(a2 + ab + b2)2 . 4.25 Shortlisted Problems 1984 479 Thus it will be enough to find a and b such that 7 ∤ a, b and 73 | a2 + ab + b2. Such numbers must satisfy (a + b)2 > a2 + ab + b2 ≥ 73 = 343, implying a + b ≥ 19. Trying a = 1 we easily find the example (a, b) = (1, 18). 13. Let Z be the given cylinder of radius r, altitude h, and volume π r2 h = 1, k1 and k2 the circles surrounding its bases, and V the volume of an inscribed tetrahedron ABCD. We claim that there is no loss of generality in assuming that A, B,C, D all lie on k1 ∪ k2 . Indeed, if the vertices A, B,C are fixed and D moves along a segment EF parallel to the axis of the cylinder (E ∈ k1 , F ∈ k2 ), the maximum distance of D from the plane ABC (and consequently the maximum value of V ) is achieved either at E or at F. Hence we shall consider only the following two cases: (i) A, B ∈ k1 and C, D ∈ k2 . Let P, Q be the projections of A, B on the plane of k2 , and R, S the projections of C, D on the plane of k1 , respectively. Then V is one-third of the volume V ′ of the prism ARBSCPDQ with bases ARBS and CPDQ. The area of the quadrilateral ARBS inscribed in k1 does not exceed the area of the square inscribed therein, which is 2r2 . Hence 3V = V ′ ≤ 2r2 h = 2/π . (ii) A, B,C ∈ k1 and D ∈ k2 . The area of the triangle ABC does√not exceed the area of an equilateral triangle inscribed in k1 , which is 3 3r2 /4. Conse√ √ 3 2 3 2 quently, V ≤ 4 r h = 4π < 3π . 14. Let M and N be the midpoints of AB and CD, and let M ′ , N ′ be their projections on CD and AB, respectively. We know that MM ′ = AB, and hence 1 1 SABCD = SAMD + SBMC + SCMD = (SABD + SABC ) + AB ·CD. (1) 2 4 The line AB is tangent to the circle with diameter CD if and only if NN ′ = CD/2, or equivalently, 1 1 SABCD = SAND + SBNC + SANB = (SBCD + SACD) + AB ·CD. 2 4 By (1), this is further equivalent to SABC +SABD = SBCD + SACD . But since SABC + SACD = SABD + SBCD = SABCD , this reduces to SABC = SBCD , i.e., to BC k AD. 15. (a) Since rotation by 60◦ around A transforms the triangle CAF into △EAB, it follows that ∡(CF, EB) = 60◦ . We similarly deduce that ∡(EB, AD) = ∡(AD, FC) = 60◦ . Let S be the intersection point of BE and AD. Since ∡CSE = ∡CAE = 60◦ , we have that EASC is cyclic. Therefore ∡(AS, SC) = 60◦ = ∡(AD, FC), which implies that S lies on CF as well. (b) A rotation of EASC around E by 60◦ transforms A into C and S into a point T for which SE = ST = SC +CT = SC + SA. Summing the equality SE = SC + SA and the analogous equalities SD = SB + SC and SF = SA + SB yields the result. 16. From the first two conditions we can easily conclude that a + d > b + c (indeed, (d + a)2 − (d − a)2 = (c + b)2 − (c − b)2 = 4ad = 4bc and d − a > c − b > 0). Thus k > m. 480 4 Solutions From d = 2k − a and c = 2m − b we get a(2k − a) = b(2m − b), or equivalently, (b + a)(b − a) = 2m (b − 2k−ma). (1) Since 2k−m a is even and b is odd, the highest power of 2 that divides the righthand side of (1) is m. Hence (b + a)(b − a) is divisible by 2m but not by 2m+1 , which implies b + a = 2m1 p and b − a = 2m2 q, where m1 , m2 ≥ 1, m1 + m2 = m, and p, q are odd. Furthermore, b = (2m1 p + 2m2 q)/2 and a = (2m1 p − 2m2 q)/2 are odd, so either m1 = 1 or m2 = 1. Note that m1 = 1 is not possible, since it would imply that b−a = 2m−1 q ≥ 2m−1 , although b+c = 2m and b < c imply that b < 2m−1 . Hence m2 = 1 and m1 = m − 1. Now since a + b < b + c = 2m , we obtain a + b = 2m−1 and b − a = 2q, where q is an odd integer. Substituting these into (1) and dividing both sides by 2m we get q = 2m−2 + q − 2k−ma =⇒ 2k−m a = 2m−2 . Since a is odd and k > m, it follows that a = 1. Remark. Now it is not difficult to prove that all quadruples (a, b, c, d) that satisfy the given conditions are of the form (1, 2m−1 − 1, 2m−1 + 1, 22m−2 − 1), where m ∈ N, m ≥ 3. 17. For any m = 0, 1, . . . , n − 1, we shall find the number of permutations (x1 , x2 , . . . , xn ) with exactly k discordant pairs such that xn = n − m. This xn is a member of exactly m discordant pairs, and hence the permutation (x1 , . . . , xn−1 ) of the set {1, 2, . . ., n} \ {m} must have exactly k − m discordant pairs: there are d(n − 1, k − m) such permutations. Therefore d(n, k) = d(n − 1, k) + d(n − 1, k − 1) · · ·+ d(n − 1, k − n + 1) = d(n − 1, k) + d(n, k − 1)  (note that d(n, k) is 0 if k < 0 or k > n2 ). We now proceed to calculate d(n, 2) and d(n, 3). Trivially, d(n, 0) = 1. It follows that d(n, 1) = d(n − 1, 1) + d(n, 0) = d(n − 1, 1) + 1, which yields d(n, 1) = d(1, 1) + n − 1 = n − 1. Further, d(n, 2) = d(n − 1, 2) + d(n, 1) = d(n − 1, 2) + n − 1 = d(2, 2) + 2 + 3 + · · · + n − 1 = (n2 − n − 2)/2. Finally, using the known formula 12 + 22 + · · · + k2 = k(k + 1)(2k + 1)/6, we have d(n, 3) = d(n − 1, 3) + d(n, 2) = d(n − 1, 3) + (n2 − n − 2)/2 = d(2, 3) + ∑ni=3 (n2 − n − 2)/2 = (n3 − 7n + 6)/6. 18. Suppose that circles k1 (O1 , r1 ), k2 (O2 , r2 ), and k3 (O3 , r3 ) touch the edges of the angles ∠BAC, ∠ABC, and ∠ACB, respectively. Denote also by O and r the center and radius of the incircle. Let P be the point of tangency of the incircle with AB and let F be the foot of the perpendicular from O1 to OP. From △O1 FO we ob√ √ tain cot(α /2) = 2 rr1 /(r − r1 ) and analogously cot(β /2) = 2 rr2 /(r − r2 ), √ cot(γ /2) = 2 rr3 /(r − r3 ). We will now use a well-known trigonometric identity for the angles of a triangle: 4.25 Shortlisted Problems 1984 cot 481 α β γ α β γ + cot + cot = cot · cot · cot . 2 2 2 2 2 2 (This identity follows from tan(γ /2) = cot(α /2 + β /2) and the formula for the cotangent of a sum.) Plugging in the obtained cotangents, we get √ √ √ √ √ √ 2 rr1 2 rr2 2 rr3 2 rr1 2 rr2 2 rr3 + + = · · ⇒ r − r1 r − r2 r − r3 r − r r − r2 r − r3 √ 1 √ r1 (r − r2 )(r − r3 ) + r2 (r − r1 )(r − r3 ) √ √ + r3 (r − r1 )(r − r2 ) = 4r r1 r2 r3 . For r1 = 1, r2 = 4, and r3 = 9 we get (r − 4)(r − 9) + 2(r − 1)(r − 9) + 3(r − 1)(r − 4) = 24r ⇒ 6(r − 1)(r − 11) = 0. Clearly, r = 11 is the only viable value for r. 19. First, we shall prove that the numbers in the nth row are exactly the numbers 1 , n−1 0 n 1 n n−1 1 , 1 n n−1 2 , ... , 1 n n−1 n−1 . (1) The proof of this fact can be done by induction. For small n, the statement can be easily verified. Assuming that the statement is true for some n, we have that the kth element in the (n + 1)st row is, as is directly verified, 1 n − n−1 k−1 1 (n + 1) n  k−1 = 1 . (n + 1) nk Thus (1) is proved. Now the geometric mean of the elements of the nth row becomes: n q n 1 n−1 n−1 n−1 0 · 1 · · · n−1 1 1  = n−1 . ≥  n−1 n−1 2 ( 0 )+( 1 )+···+(n−1 ) n−1 n n The desired result follows directly from substituting n = 1984. 20. Define the set S = R+ r {1}. The given inequality is equivalent to ln b/ln a < ln (b + 1)/ln(a + 1). If b = 1, it is obvious that each a ∈ S satisfies this inequality. Suppose now that b is also in S. Let us define on S a function f (x) = ln(x + 1)/lnx. Since ln (x + 1) > lnx and 1/x > 1/x + 1 > 0, we have f ′ (x) = ln x x+1 − ln (x+1) x 2 ln x <0 for all x. 482 4 Solutions Hence f is always decreasing. We also note that f (x) < 0 for x < 1 and that f (x) > 0 for x > 1 (at x = 1 there is a discontinuity). Let us assume b > 1. From ln b/ln a < ln (b + 1)/ln (a + 1) we get f (b) > f (a). This holds for b > a or for a < 1. Now let us assume b < 1. This time we get f (b) < f (a). This holds for a < b or for a > 1. Hence all the solutions to loga b < loga+1 (b + 1) are {b = 1, a ∈ S}, {a > b > 1}, {b > 1 > a}, {a < b < 1}, and {b < 1 < a}. 4.26 Shortlisted Problems 1985 483 4.26 Solutions to the Shortlisted Problems of IMO 1985 1. Since there are 9 primes (p1 = 2 < p2 = 3 < · · · < p9 = 23) less than 26, each a number x j ∈ M is of the form ∏9i=1 pi i j , where 0 ≤ ai j . Now, x j xk is a square if ai j + aik ≡ 0 (mod 2) for i = 1, . . . , 9. Since the number of distinct ninetuples modulo 2 is 29 , any subset of M with at least 513 elements contains two elements with square product. Starting from M and eliminating such pairs, one obtains (1985 − 513)/2 = 736 > 513 distinct two-element subsets of M each having a square as the product of elements. Reasoning as above, we find at least one (in fact many) pair of such squares whose product is a fourth power. 2. The polyhedron has 3 · 12/2 = 18 edges, and by Euler’s formula, 8 vertices. Let v1 and v2 be the numbers of vertices at which respectively 3 and 6 edges meet. Then v1 + v2 = 8 and 3v1 + 6v2 = 2 · 18, implying that v1 = 4. Let A, B,C, D be the vertices at which three edges meet. Since the dihedral angles are equal, all the edges meeting at A, say AE, AF, AG, must have equal length, say x. (If x = AE = AF 6= AG = y, and AEF, AFG, and AGE are isosceles, ∠EAF 6= ∠FAG, in contradiction to the equality of the dihedral angles.) It is easy to see that at E, F, and G six edges meet. One proceeds to conclude that if H is the fourth vertex of this kind, EFGH must be a regular tetrahedron of edge length y, and the other vertices A, B, C, and D are tops of isosceles pyramids based on EFG, EFH, FGH, and GEH. Let the plane through A, B,C meet EF, HF, and GF, at ′ is a regular hexagon, and since x = FA = FE ′ , E ′ , H ′ , and G′ . Then AE ′ BH ′CG√ ′ ′ ′ we have E G = x and AE = x/ 3. From the isosceles triangles AEF and FAE ′ we obtain finally, with ∡EFA = α , √ y = cos α = 1 − 2 sin2 (α /2), x/(2x 3) = sin(α /2), 2x and y/x = 5/3. 3. We shall write P ≡ Q for two polynomials P and Q if P(x) − Q(x) has even coefficients. m m We observe that (1 + x)2 ≡ 1 + x2 for every m ∈ N. Consequently, for every polynomial p with degree less than k = 2m , w(p · qk ) = 2w(p). Now we prove the inequality from the problem by induction on in . If in ≤ 1, the inequality is trivial. Assume it is true for any sequence with i1 < · · · < in < 2m (m ≥ 1), and let there be given a sequence with k = 2m ≤ in < 2m+1 . Consider two cases. (i) i1 ≥ k. Then w(qi1 + · · · + qin ) = 2w(qi1 −k + · · · + qin −k ) ≥ 2w(qi1 −k ) = w(qi1 ). (ii) i1 < k. Then the polynomial p = qi1 + · · · + qin has the form p= k−1 k−1 i=0 i=0 k−1 h ∑ ai xi + (1 + x)k ∑ bi xi ≡ ∑ i=0 i (ai + bi )xi + bi xi+k . Whenever some ai is odd, either ai + bi or bi in the above sum will be odd. It follows that w(p) ≥ w(qi1 ), as claimed. 484 4 Solutions The proof is complete. 4. Let hxi denote the residue of an integer x modulo n. Also, we write a ∼ b if a and b receive the same color. We claim that all the numbers hi ji, i = 1, 2, . . . , n − 1, are of the same color. Since j and n are coprime, this will imply the desired result. We use induction on i. For i = 1 the statement is trivial. Assume now that the statement is true for i = 1, . . . , k − 1. For 1 < k < n we have hk ji 6= j. If hk ji > j, then by (ii), hk ji ∼ hk ji − j = h(k − 1) ji. If otherwise hk ji < j, then by (ii) and (i), hk ji ∼ j − hk ji ∼ n − j + hk ji = h(k − 1) ji. This completes the induction. 5. Let w.l.o.g. circle C have unit radius. For each m ∈ R, the locus of points M such that f (M) = m is the circle Cm with radius rm = m/(m + 1), that is tangent to C at A. Let Om be the center of Cm . We have to show that if M ∈ Cm and N ∈ Cn , where m, n > 0, then the midpoint P of MN lies inside the circle C(m+n)/2 . This is trivial if m = n, so let m 6= n. For fixed M, P is in the image Cn′ of Cn under the homothety with center M and coefficient 1/2. The center of the circle Cn′ is at the midpoint of On M. If we let both M and N vary, P will be on the union of circles with radius rn /2 and centers in the image of Cm under the homothety with center On and coefficient 1/2. Hence P is not outside the circle centered at the midpoint Om On and with radius (rm + rn )/2. It remains to show that r(m+n)/2 > (rm + rn )/2. But this inequality is easily reduced to (m − n)2 > 0, which is true. 6. Let us set xn,i = r i i+ q i+1 √ i + 1 + · · · + n n, i−2 i−1 yn,i = xi−1 n+1,i + xn+1,ixn,i + · · · + xn,i . In particular, xn,2 = xn and xn,i = 0 for i > n. We observe that for n ≥ i ≥ 2, xn+1,i − xn,i = xin+1,i − xin,i yn,i = xn+1,i+1 − xn,i+1 . yn,i 1+(i−1)/i ≥ i3/2 and x Since yn,i > ixi−1 n+1,n+1 − xn,n+1 = n,i ≥ i induction gives √ n+1 n+1 1 xn+1 − xn ≤ < for n > 2. n! (n!)3/2 √ n + 1, simple n+1 The inequality for n = 2 is directly verified. 7. Let ki ≥ 0 be the largest integer such that pki | xi , i = 1, . . . , n, and yi = xi /pki . We may assume that k = k1 + · · · + kn . All the yi must be distinct. Indeed, if yi = y j and ki > k j , then xi ≥ px j ≥ 2xi ≥ 2x1 , which is impossible. Thus y1 y2 . . . yn = P/pk ≥ n!. 4.26 Shortlisted Problems 1985 485 If equality holds, we must have yi = 1, y j = 2 and yk = 3 for some i, j, k. Thus p ≥ 5, which implies that either yi /y j ≤ 1/2 or yi /y j ≥ 5/2, which is impossible. Hence the inequality is strict. 8. Among ten consecutive integers that divide n, there must exist numbers divisible by 23 , 32 , 5, and 7. Thus the desired number has the form n = 2α1 3α2 5α3 7α4 11α5 · · · , where α1 ≥ 3, α2 ≥ 2, α3 ≥ 1, α4 ≥ 1. Since n has (α1 + 1)(α2 + 1)(α3 + 1) · · · distinct factors, and (α1 + 1)(α2 + 1)(α3 + 1)(α4 + 1) ≥ 48, we must have (α5 + 1) · · · ≤ 3. Hence at most one α j , j > 4, is positive, and in the minimal n this must be α5 . Checking through the possible combinations satisfying (α1 + 1)(α2 + 1) · · ·(α5 + 1) = 144 one finds that the minimal n is 25 · 32 · 5 · 7 · 11 = 110880. →→− − → − → −→ −→ −→ → → 9. Let − a , b ,− c , d denote the vectors OA, OB, OC, OD respectively. Then |− a|= → − → − → → − → − → − − → | b | = | c | = | d | = 1. The centroids of the faces are ( b + c + d )/3, (− a + → − → − → − → − → − c + d )/3, etc., and each of these is at distance 1/3 from P = ( a + b + c + → − d )/3; hence the required radius is 1/3. To compute |P| as a function of the → − 2 − → − → edges of ABCD, observe that AB2 = ( b − → a ) = 2 − 2− a · b etc. Now → − − → − → |− a + b +→ c + d |2 9 16 − 2(AB2 + BC2 + AC2 + AD2 + BD2 +CD2 ) = . 9 P2 = 10. If M is at a vertex of the regular tetraB hedron ABCD (AB = 1), then one can M5 take M ′ at the center of the opposite C C A A face of the tetrahedron. M6 M4 Let M be on the face (ABC) of B D B the tetrahedron, excluding the vertices. Consider a continuous map f c′ M M3 A of C onto the surface S of ABCD C M1 iπ /3 that maps m + ne for m, n ∈ Z A C M2 onto A, B, C, D if (m, n) ≡ (1, 1), (1, 0), (0, 1), (0, 0) (mod 2) respectB ively, and maps each unit equilateral triangle with vertices of the form m+neiπ /3 isometrically onto the corresponding face of ABCD. The point M then has one preimage M j , j = 1, 2, . . . , 6, in each of the six preimages of △ABC having two vertices on the unit circle. The M j ’s form a convex centrally symmetric (possibly degenerate) hexagon. Of the triangles formed by two adjacent sides of this hexagon consider the one, say M1 M2 M3 , c′ its circumcenter. Then with the smallest radius of circumcircle and denote by M ′ ′ c c′ , M2 M c′ , we can choose M = f (M ). Indeed, the images of the segments M1 M c′ are three different shortest paths on S from M to M ′ . M3 M 486 4 Solutions 11. Let −x1 , . . . , −x6 be the roots of the polynomial. Let sk,i (k ≤ i ≤ 6) denote the sum of all products of k of the numbers x1 , . . . , xi . By Vieta’s formula we have ak = sk,6 for k = 1, . . . , 6. Since sk,i = sk−1,i−1 xi + sk,i−1 , one can compute the ak by the following scheme (the horizontal and vertical arrows denote multiplications and additions respectively): x1 ↓ s1,2 ↓ s1,3 ↓ s1,4 ↓ s1,5 ↓ a1 → s2,2 ↓ → s2,3 ↓ → s2,4 ↓ → s2,5 ↓ → a2 → s3,3 → s4,4 → s5,5 → a6 ↓ ↓ ↓ → s3,4 → s4,5 → a5 ↓ ↓ → s3,5 → a4 ↓ → a3 12. We shall prove by induction on m that Pm (x, y, z) is symmetric and that (x + y)Pm (x, z, y + 1) − (x + z)Pm(x, y, z + 1) = (y − z)Pm(x, y, z) (1) holds for all x, y, z. This is trivial for m = 0. Assume now that it holds for m = n − 1. Since obviously Pn (x, y, z) = Pn (y, x, z), the symmetry of Pn will follow if we prove that Pn (x, y, z) = Pn (x, z, y). Using (1) we have Pn (x, z, y) − Pn (x, y, z) = (y + z)[(x + y)Pn−1(x, z, y + 1) − (x + z)Pn−1(x, y, z + 1)] − (y2 − z2 )Pn−1 (x, y, z) = (y + z)(y − z)Pn−1(x, y, z) − (y2 − z2 )Pn−1 (x, y, z) = 0. It remains to prove (1) for m = n. Using the already established symmetry we have (x + y)Pn (x, z, y + 1) − (x + z)Pn(x, y, z + 1) = (x + y)Pn (y + 1, z, x) − (x + z)Pn(z + 1, y, x) = (x + y)[(y + x + 1)(z + x)Pn−1(y + 1, z, x + 1) − x2Pn−1 (y + 1, z, x)] −(x + z)[(z + x + 1)(y + x)Pn−1(z + 1, y, x + 1) − x2Pn−1 (z + 1, y, x)] = (x + y)(x + z)(y − z)Pn−1(x + 1, y, z) − x2 (y − z)Pn−1 (x, y, z) = (y − z)Pn (z, y, x) = (y − z)Pn(x, y, z), as claimed. 13. If m and n are relatively prime, there exist positive integers p, q such that pm = qn + 1. Thus by putting m balls in some boxes p times we can achieve that one box receives q + 1 balls while all others receive q balls. Repeating this process sufficiently many times, we can obtain an equal distribution of the balls. Now assume gcd(m, n) > 1. If initially there is only one ball in the boxes, then after k operations the number of balls will be 1 + km, which is never divisible by n. Hence the task cannot be done. 4.26 Shortlisted Problems 1985 487 14. It suffices to prove the existence of a good point in the case of exactly 661 −1’s. We prove by induction on k that in any arrangement with 3k + 2 points k of which are −1’s a good point exists. For k = 1 this is clear by inspection. Assume that the assertion holds for all arrangements of 3n + 2 points and consider an arrangement of 3(n + 1) + 2 points. Now there exists a sequence of consecutive −1’s surrounded by two +1’s. There is a point P which is good for the arrangement obtained by removing the two +1’s bordering the sequence of −1’s and one of these −1’s. Since P is out of this sequence, clearly the removal either leaves a partial sum as it was or diminishes it by 1, so P is good for the original arrangement. Second solution. Denote the number on an arbitrary point by a1 , and the numbers on successive points going in the positive direction by a2 , a3 , . . . (in particular, ak+1985 = ak ). We define the partial sums s0 = 0, sn = a1 + a2 + · · · + an for all positive integers n; then sk+1985 = sk + s1985 and s1985 ≥ 663. Since s1985m ≥ 663m and 3 · 663m > 1985(m + 2) + 1 for large m, not all values 0, 1, 2, . . . 663m can appear thrice among the 1985(m + 2) + 1 sums s−1985 , s−1984 , . . . , s1985(m+1) (and none of them appears out of this set). Thus there is an integral value s > 0 that appears at most twice as a partial sum, say sk = sl = s, k < l. Then either ak or al is a good point. Actually, si > s must hold for all i > l, and si < s for all i < k (otherwise, the sum s would appear more than twice). Also, for the same reason there cannot exist indices p, q between k and l such that s p > s and sq < s; i.e., for k < p < l, s p ’s are either all greater than or equal to s, or smaller than or equal to s. In the former case ak is good, while in the latter al is good. C′ 15. There is no loss of generality if we as′ ′ ′ ′ sume K = ABCD, K = AB C D , and 2 G E B H′ C that K ′ is obtained from K by a clockF wise rotation around A by φ , 0 ≤ φ ≤ 3 5 4 B′ π /4. Let C′ D′ , B′C′ , and the parallel 1 ′ H D to AB through D′ meet the line BC at C′′ D′′ 5 4 E, F, and G respectively. Let us now G′ E′ 3 choose points E ′ ∈ AB′ , G′ ∈ AB, C′′ ∈ 2 AD′ , and E ′′ ∈ AD such that the trianD E ′′ A gles AE ′ G′ and AC′′ E ′′ are translates of the triangles D′ EG and FC′ E respectively. Since AE ′ = D′ E and AC′′ = FC′ , we have C′′ E ′′ = C′ E = B′ E ′ and C′′ D′ = B′ F, which imply that △E ′′C′′ D′ is a translate of △E ′ B′ F, and consequently E ′′ D′ = E ′ F and E ′′ D′ k E ′ F. It follows that there exist points H ∈ CD, H ′ ∈ BF, and D′′ ∈ E ′ G′ such that E ′′ D′ HD is a translate of E ′ FH ′ D′′ . The remaining parts of K and K ′ are the rectangles D′ GCH and D′′ H ′ BG′ of equal area. We shall now show that two rectangles with parallel sides and equal areas can be decomposed into translation invariant parts. Let the sides of the rectangles XY ZT and X ′Y ′ Z ′ T ′ (XY k X ′Y ′ ) satisfy X ′Y ′ < XY , Y ′ Z ′ > Y Z, and X ′Y ′ ·Y ′ Z ′ = XY · Y Z. Suppose that 2X ′Y ′ > XY (otherwise, we may cut off congruent rectangles from both the original ones until we reduce them to the case of 2X ′Y ′ > XY ). 488 4 Solutions Let U ∈ XY and V ∈ ZT be points such that YU = TV = X ′Y ′ and W ∈ XV be a point such that UW k X T . Then translating △XUW to a triangle V ZR and △XV T to a triangle W RS results in a rectangle UY RS congruent to X ′Y ′ Z ′ T ′ . Thus we have partitioned K and K ′ into translation-invariant parts. Although not all the parts are triangles, we may simply triangulate them. 16. Let the three circles be α (A, a), β (B, b), and γ (C, c), and assume c ≤ a, b. We denote by RX,ϕ the rotation around X through an angle ϕ . Let PQR be an equilateral triangle, say of positive orientation (the case of negatively oriented △PQR is analogous), with P ∈ α , Q ∈ β , and R ∈ γ . Then Q = RP,−60◦ (R) ∈ RP,−60◦ (γ ) ∩ β . Since the center of RP,−60◦ (γ ) is RP,−60◦ (C) = RC,60◦ (P) and it belongs to RC,60◦ (α ), the union of circles RP,−60◦ (γ ) as P varies on α is the annulus U with center A′ = RC,60◦ (A) and radii a − c and a + c. Hence there is a solution if and only if U ∩ β is nonempty. 17. The statement of the problem is equivalent to the statement that there is one and only one a such that 1 − 1/n < fn (a) < 1 for all n. We note that each fn is a polynomial with positive coefficients, and therefore increasing and convex in R+ . Define xn and yn by fn (xn ) = 1 − 1/n and fn (yn ) = 1. Since     1 2 1 1 1 fn+1 (xn ) = 1 − + 1− = 1− n n n n and fn+1 (yn ) = 1 + 1/n, it follows that xn < xn+1 < yn+1 < yn . Moreover, the convexity of fn together with the fact that fn (x) > x for all x > 0 implies that yn − xn < fn (yn ) − fn (xn ) = 1/n. Therefore the sequences have a common limit a, which is the only number lying between xn and yn for all n. By the definition of xn and yn , the statement immediately follows. x2 18. Set yi = xi+1 ixi+2 , where xn+i = xi . Then ∏ni=1 yi = 1 and the inequality to be yi proved becomes ∑ni=1 1+y ≤ n − 1, or equivalently i n 1 ∑ 1 + yi ≥ 1. i=1 We prove this inequality by induction on n. 1 Since 1+y + 1+y1 −1 = 1, the inequality is true for n = 2. Assume that it is true for n − 1, and let there be given y1 , . . . , yn > 0 with ∏ni=1 yi = 1. Then 1+y1n−1 + 1 1 1+yn > 1+yn−1 yn , which is equivalent to 1 + yn yn−1 (1 + yn + yn−1 ) > 0. Hence by the inductive hypothesis n 1 n−2 1 1 ∑ 1 + yi ≥ ∑ 1 + yi + 1 + yn−1yn ≥ 1. i=1 i=1 4.26 Shortlisted Problems 1985 489 Remark. The constant n − 1 is best possible (take for example xi = ai with a arbitrarily large). 19. Suppose that for some n > 6 there is a regular n-gon with vertices having integer coordinates, and that A1 A2 . . . An is the smallest such n-gon, of side length a. If −−→ − −−−→ O is the origin and Bi the point such that OBi = Ai−1 Ai , i = 1, 2, . . . , n (where A0 = An ), then Bi has integer coordinates and B1 B2 . . . Bn is a regular polygon of side length 2a sin(π /n) < a, which is impossible. It remains to analyze the cases n ≤ 6. If P is a regular n-gon with n = 3, 5, 6, then its center C has rational coordinates. We may suppose that C also has integer coordinates and then rotate P around C thrice through 90◦ , thus obtaining a regular 12-gon or 20-gon, which is impossible. Hence we must have n = 4 which is indeed a solution. 20. Let O be the center of the circle touching the three sides of BCDE and let F ∈ (ED) be the point such that EF = EB. Then ∠EFB = 90◦ − ∠E/2 = ∠C/2 = ∠OCB, which implies that B,C, F, O lie on a circle. It follows that ∠DFC = ∠OBC = ∠B/2 = 90◦ − ∠D/2 and consequently ∠DCF = ∠DFC. Hence ED = EF + FD = EB + CD. Second solution. Let r be the radius of the small circle and let M, N be the points of tangency of the circle with BE and CD respectively. Then EM = rcot E, DN = rcot D, MB = r cot and ED = EO + OD = r r + . sin D sin E ∠B ∠D ∠E = r tan , NC = r tan , 2 2 2 The statement follows from the identity cotx + tan(x/2) = 1/sin x. 21. Let B1 and C1 be the points on the rays AC and AB respectively such that X B1 = XC = XB = XC1 . Then ∠X B1C = ∠XCB1 = ∠ABC and ∠XC1 B = ∠X BC1 = ∠ACB, which imply that B1 , X ,C1 are collinear and △AB1C1 ∼ △ABC. Moreover, X is the midpoint of B1C1 because X B1 = XC1 , from which we conclude that △AXB1 ∼ △AMB. Therefore ∠CAX = ∠BAM and AM BM BM = = = cos α . AX X B1 BC 22. Assume that △ABC is acute (the case of an obtuse △ABC is similar). Let S and R be the centers of the circumcircles of △ABC and △KBN, respectively. Since ∠BNK = ∠BAC, the triangles BNK and BAC are similar. Now we have ∠CBR = ∠ABS = 90◦ − ∠ACB, which gives us BR ⊥ AC and consequently BR k OS. Similarly BS ⊥ KN implies that BS k OR. Hence BROS is a parallelogram. Let L be the point symmetric to B with respect to R. Then RLOS is also a parallelogram, and since SR ⊥ BM, we obtain OL ⊥ BM. However, we also have LM ⊥ BM, from which we conclude that O, L, M are collinear and OM ⊥ BM. 490 4 Solutions Second solution. The lines BM, NK, and CA are the radical axes of pairs of the three circles, and hence they intersect at a single point P. Also, the quadrilateral MNCP is cyclic. Let OA = OC = OK = ON = r. We then have BM · BP = BN · BC = OB2 − r2 , P PM · PB = PN · PK = OP2 − r2 . It follows that OB2 − OP2 = BP(BM− PM) = BM 2 − PM2 , which implies that OM ⊥ MB. B M R N L K S O C A 4.27 Shortlisted Problems 1986 491 4.27 Solutions to the Shortlisted Problems of IMO 1986 1. If w > 2, then setting in (i) x = w−2, y = 2, we get f (w) = f ((w−2) f (w)) f (2) = 0. Thus f (x) = 0 if and only if x ≥ 2. Now let 0 ≤ y < 2 and x ≥ 0. The LHS in (i) is zero if and only if x f (y) ≥ 2, while the RHS is zero if and only if x + y ≥ 2. It follows that x ≥ 2/ f (y) if and only if x ≥ 2 − y. Therefore  2 for 0 ≤ y < 2; f (y) = 2−y 0 for y ≥ 2. The confirmation that f satisfies the given conditions is straightforward. 2. No. If a were rational, its decimal expansion would be periodic from some point. Let p be the number of decimals in the period. Since f (102p ) has 2np zeros, it contains a full periodic part; hence the period would consist only of zeros, which is impossible. 3. Let E be the point where the boy turned westward, reaching the shore at D. Let the ray DE cut AC at F and the shore again at G. Then EF = AE = x (because AEF is an equilateral triangle) and FG = DE = y. From AE · EB = DE · EG we obtain x(86 − x) = y(x + y). If x is odd, then x(86 − x) is odd, while y(x + y) is even. Hence x is even, and so y must also be even. Let y = 2y1 . The above equation can be rewritten as (x + y1 − 43)2 + (2y1)2 = (43 − y1 )2 . Since y1 < 43, we have (2y1 , 43 − y1) = 1, and thus (|x + y1 − 43|, 2y1 , 43 − y1) is a primitive Pythagorean triple. Consequently there exist integers a > b > 0 such that y1 = ab and 43 − y1 = a2 + b2 . We obtain that a2 + b2 + ab = 43, which has the unique solution a = 6, b = 1. Hence y = 12 and x = 2 or x = 72. Remark. The Diophantine equation x(86 − x) = y(x + y) can be also solved directly. Namely, we have that x(344 − 3x) = (2y + x)2 is a square, and since x is even, we have (x, 344 − 3x) = 2 or 4. Consequently x, 344 − 3x are either both squares or both two times squares. The rest is easy. 4. Let x = pα x′ , y = pβ y′ , z = pγ z′ with p ∤ x′ y′ z′ and α ≥ β ≥ γ . From the given equation it follows that pn (x + y) = z(xy − pn ) and consequently z′ | x + y. Since also pγ | x + y, we have z | x + y, i.e., x + y = qz. The given equation together with the last condition gives us xy = pn (q + 1) and x + y = qz. Conversely, every solution of (1) gives a solution of the given equation. For q = 1 and q = 2 we obtain the following classes of n + 1 solutions each: (1) 492 4 Solutions i n−i i n−i q = 1 : (x, y, z) = (2p  , p , 2p + p ) q = 2 : (x, y, z) = 3p j for i = 0, 1, 2, . . . , n; j n− j , pn− j , 3p +p 2 for j = 0, 1, 2, . . . , n. For n = 2k these two classes have a common solution (2pk , pk , 3pk ); otherwise, all these solutionsare distinct. One further solution is given by (x, y, z) = 1, pn (pn + 3)/2, p2 + 2 , not included in the above classes for p > 3. Thus we have found 2(n + 1) solutions. Another type of solution is obtained if we put q = pk + pn−k . This yields the solutions (x, y, z) = (pk , pn + pn−k + p2n−2k , pn−k + 1) for k = 0, 1, . . . , n. For k < n these are indeed new solutions. So far, we have found 3(n + 1) − 1 or 3(n + 1) solutions. One more solution is given by (x, y, z) = (p, pn + pn−1 , pn−1 + pn−2 + 1). 5. Suppose that for every a, b ∈ {2, 5, 13, d}, a 6= b, the number ab − 1 is a perfect square. In particular, for some integers x, y, z we have 2d − 1 = x2 , 5d − 1 = y2 , 13d − 1 = z2 . Since x is clearly odd, d = (x2 + 1)/2 is also odd because 4 ∤ x2 + 1. It follows that y and z are even, say y = 2y1 and z = 2z1 . Hence (z1 − y1 )(z1 + y1 ) = (z2 − y2 )/4 = 2d. But in this case one of the factors z1 − y1 , z1 + y1 is odd and the other one is even, which is impossible. 6. There are five such numbers: 69300 = 22 · 32 · 52 · 7 · 11 : 50400 = 25 · 32 · 52 · 7 : 60480 = 26 · 33 · 5 · 7 : 55440 = 24 · 32 · 5 · 7 · 11 : 65520 = 24 · 32 · 5 · 7 · 13 : 3 · 3 · 3 · 2 · 2 = 108 divisors; 6 · 3 · 3 · 2 = 108 divisors; 7 · 4 · 2 · 2 = 112 divisors; 5 · 3 · 2 · 2 · 2 = 120 divisors; 5 · 3 · 2 · 2 · 2 = 120 divisors. 7. Let P(x) = (x − x0 )(x − x1 ) · · · (x − xn )(x − xn+1 ). Then P′ (x) = n+1 P(x) ∑ x − xj and P′′ (x) = j=0 Therefore n+1 P(x) ∑ ∑ (x − x j )(x − xk ) . j=0 k6= j 1 j6=i (xi − x j ) P′′ (xi ) = 2P′ (xi ) ∑ for i = 0, 1, . . . , n + 1, and the given condition implies P′′ (xi ) = 0 for i = 1, 2, . . . , n. Consequently, x(x − 1)P′′(x) = (n + 2)(n + 1)P(x). (1) 4.27 Shortlisted Problems 1986 493 It is easy to observe that there is a unique monic polynomial of degree n + 2 satisfying differential equation (1). On the other hand, the polynomial Q(x) = (−1)n P(1 − x) also satisfies this equation, is monic, and degQ = n + 2. Therefore (−1)n P(1 − x) = P(x), and the result follows. 8. We shall solve the problem in the alternative formulation. Let LG (v) denote the length of the longest directed chain of edges in the given graph G that begins in a vertex v and is arranged decreasingly relative to the numbering. By the pigeonhole principle it suffices to show that ∑v L(v) ≥ 2q in every such graph. We do this by induction on q. For q = 1 the claim is obvious. We assume that it is true for q − 1 and consider a graph G with q edges numbered 1, . . . , q. Let the edge number q connect vertices u and w. Removing this edge, we get a graph G′ with q − 1 edges. We then have LG (u) ≥ LG′ (w) + 1, LG (w) ≥ LG′ (u) + 1, LG (v) ≥ LG′ (v) for other v. Since ∑ LG′ (v) ≥ 2(q − 1) by inductive assumption, it follows that ∑ LG (v) ≥ 2(q − 1) + 2 = 2q as desired. Second solution. Let us place a spider at each vertex of the graph. Let us now interchange the positions of the two spiders at the endpoints of each edge, listing the edges increasingly with respect to the numbering. This way we will move spiders exactly 2q times (two for each edge). Hence there is a spider that will be moved at least 2q/n times. All that remains is to notice that the path of each spider consists of edges numbered in increasing order. Remark. A chain of the stated length having all vertices distinct does not necessarily exist. An example is n = 4, q = 6 with the numbering following the order ab, cd, ac, bd, ad, bc. 9. We shall use induction on the number n of points. The case n = 1 is trivial. Let us suppose that the statement is true for all 1, 2, . . . , n − 1, and that we are given a set T of n points. If there exists a point P ∈ T and a line l that is parallel to an axis and contains P and no other points of T , then by the inductive hypothesis we can color the set T \ {P} and then use a suitable color for P. Let us now suppose that whenever a line parallel to an axis contains a point of T , it contains another point of T . It follows that for an arbitrary point P0 ∈ T we can choose points P1 , P2 , . . . such that Pk Pk+1 is parallel to the x-axis for k even, and to the y-axis for k odd. We eventually come to a pair of integers (r, s) of the same parity, 0 ≤ r < s, such that lines Pr Pr+1 and Ps Ps+1 coincide. Hence the closed polygonal line Pr+1 Pr+2 . . . Ps Pr+1 is of even length. Thus we may color the points of this polygonal line alternately and then apply the inductive assumption for the rest of the set T . The induction is complete. Second solution. Let P1 , P2 , . . . , Pk be the points lying on a line l parallel to an axis, going from left to right or from up to down. We draw segments joining P1 with P2 , P3 with P4 , and generally P2i−1 with P2i . Having this done for every such line l, we obtain a set of segments forming certain polygonal lines. If one 494 4 Solutions of these polygonal lines is closed, then it must have an even number of vertices. Thus, we can color the vertices on each of the polygonal lines alternately (a point not lying on any of the polygonal lines may be colored arbitrarily). The obtained coloring satisfies the conditions. 10. The set X = {1, . . . , 1986} splits into triads T1 , . . . , T662 , where T j = {3 j − 2, 3 j − 1, 3 j}. Let F be the family of all k-element subsets P such that |P∩T j | = 1 or 2 for some index j. If j0 is the smallest such j0 , we define P′ to be the k-element set obtained from P by replacing the elements of P ∩ T j0 by the ones following cyclically inside T j0 . Let s(P) denote the remainder modulo 3 of the sum of elements of P. Then s(P), s(P′ ), s(P′′ ) are distinct, and P′′′ = P. Thus the operator ′ gives us a bijective correspondence between the sets X ∈ F with s(P) = 0, those with s(P) = 1, and those with s(P) = 2. If 3 ∤ k is not divisible by 3, then each k-element subset of X belongs to F , and the game is fair. If 3 | k, then k-element subsets not belonging to F are those that are unions of several triads. Since every such subset has the sum of elements divisible by 3, it follows that player A has the advantage. 11. Let X be a finite set in the plane and lk a line containing exactly k points of X (k = 1, . . . , n). Then ln contains n points, ln−1 contains at least n − 2 points not lying on ln , ln−2 contains at least n − 4 points not lying on ln or ln−1 , etc. It follows that   |X | ≥ g(n) = n + (n − 2) + (n − 4) + · · ·+ n − 2 n2 .    n+2  Hence f (n) ≥ g(n) = n+1 2 2 , where the last equality is easily proved by induction. We claim that f (n) = g(n). To prove this, we shall inductively construct a set Xn of cardinality g(n) with the required property. For n ≤ 2 a one-point and twopoint set satisfy the requirements. Assume that Xn is a set of g(n) points and that lk is a line containing exactly k points of Xn , k = 1, . . . , n. Consider any line l not parallel to any of the lk ’s and not containing any point of Xn or any intersection point of the lk . Let l intersect lk in a point Pk , k = 1, . . . , n, and let Pn+1 , Pn+2 be two points on l other than P1 , . . . , Pn . We define Xn+2 = Xn ∪ {P1 , . . . , Pn+2 }. The set Xn+2 consists of g(n)+ (n+ 2) = g(n + 2) points. Since the lines l, ln , . . . , l2 , l1 meet Xn in n + 2, n + 1, . . ., 3, 2 points respectively (and there clearly exists a line containing only one point of Xn+2 ), this set also meets the demands. 12. We define f (x1 , . . . , x5 ) = ∑5i=1 (xi+1 − xi−1 )2 (x0 = x5 , x6 = x1 ). Assuming that x3 < 0, according to the rules the lattice vector X = (x1 , x2 , x3 , x4 , x5 ) changes into Y = (x1 , x2 + x3 , −x3 , x4 + x3 , x5 ). Then f (Y ) − f (X) = (x2 + x3 − x5 )2 + (x1 + x3 )2 + (x2 − x4 )2 +(x3 + x5 )2 + (x1 − x3 − x4 )2 − (x2 − x5 )2 −(x3 − x1 )2 − (x4 − x2 )2 − (x5 − x3 )2 − (x1 − x4 )2 = 2x3 (x1 + x2 + x3 + x4 + x5 ) = 2x3 S < 0. 4.27 Shortlisted Problems 1986 495 Thus f strictly decreases after each step, and since it takes only nonnegative integer values, the number of steps must be finite. Remark. One could inspect the behavior of g(X ) = ∑5i=1 ∑5j=1 |xi + xi+1 + · · · + x j−1 | instead. Then g(Y ) − g(X ) = |S + x3 | − |S − x3| > 0. 13. Let us consider the infinite integer lattice and assume that having reached a point (x, n) or (n, y), the particle continues moving east and north following the rules of the game. The required probability pk is equal to the probability of getting to one of the points E1 (n, n + k), E2 (n + k, n), but without passing through (n, n + k − 1) or (n + k − 1, n). Thus p is equal to the probability p1 of getting to E1 (n, n + k) via D1 (n − 1, n + k) plus the probability p2 of getting to E2 (n + k, n)  via D2 (n + 2n+k−1 −2n−k k, n − 1). Both p1 and p2 are easily seen to be equal to n−1 2 , and  −2n−k+1 therefore p = 2n+k−1 2 . n−1 14. We shall use the following simple fact. Lemma. If b k is the image of a circle k under an inversion centered at a point Z, and O1 , O2 are centers of k and b k, then O1 , O2 , and Z are collinear. Proof. The result follows immediately from the symmetry with respect to the line ZO1 . Let I be the center of the inscribed circle i. Since IX ·IA = IE 2 , the inversion with respect to i takes points A into X, and analogously B,C into Y, Z respectively. It follows from the lemma that the center of circle ABC, the center of circle XY Z, and point I are collinear. 15. (a) This is the same problem as SL82-14. ∠D cos∠B ′ (b) If S is the midpoint of AC, we have B′ S = AC 2cos sin ∠D , D S = AC 2 sin∠B , B′ D′ = AC sin(∠B+∠D) 2 sin∠B sin∠D . These formulas are true also if ∠B > 90◦ or ∠D > 90◦ . We similarly obtain that A′′C′′ = B′ D′ sin(∠A′ +∠C′ ) 2 sin∠A′ sin ∠C′ . Therefore sin2 (∠A + ∠C) . 4 sin ∠A sin ∠B sin ∠C sin ∠D 16. Let Z be the center of the polygon. Suppose that at some moment we have A ∈ Pi−1 Pi and B ∈ Pi Pi+1 , where Zi Pi+1 O Pi−1 , Pi , Pi+1 are adjacent vertices of Z the polygon. Since ∠AOB = 180◦ − ∠Pi−1 Pi Pi+1 , the quadrilateral APi BO B is cyclic. Hence ∠APi O = ∠ABO = Pi−1 A Pi ∠APi Z, which means that O ∈ Pi Z. over, from OPi = 2r sin ∠Pi AO, where r is the radius of circle APi BO, we obtain that ZPi ≤ OPi ≤ ZPi /cos(π /n). Thus O traces a segment ZZi as A and B move along Pi−1 Pi and Pi Pi+1 respectively, where Zi is a point on the ray Pi Z with Pi Zi cos(π /n) = Pi Z. When A, B move along the whole circumference of the polygon, O traces an asterisk consisting of n segments of equal length emanating from Z and pointing away from the vertices. A′′C′′ = AC 496 4 Solutions 17. We use complex numbers to represent the position of a point in the plane. For convenience, let A1 , A2 , A3 , A4 , A5 , . . . be A, B,C, A, B, . . . respectively, and let P0 be the origin. After the kth step, the position of Pk will be Pk = Ak + (Pk−1 − Ak )u, k = 1, 2, 3, . . ., where u = e4π ı/3 . We easily obtain Pk = (1 − u)(Ak + uAk−1 + u2 Ak−2 + · · · + uk−1 A1 ). The condition P0 ≡ P1986 is equivalent to A1986 + uA1985 + · · · + u1984 A2 + u1985 A1 = 0, which, having in mind that A1 = A4 = A7 = · · · , A2 = A5 = A8 = · · · , A3 = A6 = A9 = · · · , reduces to 662(A3 + uA2 + u2 A1 ) = (1 + u3 + · · · + u1983)(A3 + uA2 + u2 A1 ) = 0. It follows that A3 − A1 = u(A1 − A2), and the assertion follows. Second solution. Let fP denote the rotation with center P through 120◦ clockwise. Let f1 = fA . Then f1 (P0 ) = P1 . Let B′ = f1 (B), C′ = f1 (C), and f2 = fB′ . Then f2 (P1 ) = P2 and f2 (AB′C′ ) = A′ B′C′′ . Finally, let f3 = fC′′ and f3 (A′ B′C′′ ) = A′′ B′′C′′ . Then g = f3 f2 f1 is a translation sending P0 to P3 and C to C′′ . Now P1986 = P0 implies that g662 is the identity, and thus C = C′′ . Let K be such that ABK is equilateral and positively oriented. We observe that f2 f1 (K) = K; therefore the rotation f2 f1 satisfies f2 f1 (P) 6= P for P 6= K. Hence f2 f1 (C) = C′′ = C implies K = C. 18. We shall use the following criterion for a quadrangle to be circumscribable. Lemma. The quadrangle AY DZ is circumscribable if and only if DB − DC = AB − AC. Proof. Suppose that AY DZ is circumscribable and that the incircle is tangent to AZ, ZD, DY , YA at M, N, P, Q respectively. Then DB − DC = PB − NC = MB − QC = AB − AC. Conversely, assume that DB − DC = AB − AC and let A a tangent from D to the incircle of the triangle ACZ meet CZ and CA Q at D′ 6= Z and Y ′ 6= A respectively. M According to the first part we have Y Z D′ B − D′C = AB − AC. It follows N D P that |D′ B − DB| = |D′C − DC| = X DD′ , implying that D′ ≡ D. B C Let us assume that DZBX and DXCY are circumscribable. Using the lemma we obtain DC − DA = BC − BA and DA − DB = CA − CB. Adding these two inequalities yields DC − DB = AC − AB, and the statement follows from the lemma. 19. Let M and N be the midpoints of segments AB and CD, respectively. The given conditions imply that △ABD ∼ = △BAC and △CDA ∼ = △DCB; hence MC = MD and NA = NB. It follows that M and N both lie on the perpendicular bisectors of AB and CD, and consequently MN is the common perpendicular bisector of AB and CD. Points B and C are symmetric to A and D with respect to MN. Now if P 4.27 Shortlisted Problems 1986 497 is a point in space and P′ the point symmetric to P with respect to MN, we have BP = AP′ , CP = DP′ , and thus f (P) = AP + AP′ + DP + DP′ . Let PP′ intersect MN in Q. Then AP + AP′ ≥ 2AQ and DP + DP′ ≥ 2DQ, from which it follows that f (P) ≥ 2(AQ + DQ) = f (Q). It remains to minimize f (Q) with Q moving along the line MN. Let us rotate point D around MN to a point D′ that belongs to the plane AMN, on the side of MN opposite to A. Then f (Q) = 2(AQ + D′Q) ≥ AD′ , and equality occurs when Q is the intersection of AD′ and MN. Thus min f (Q) = AD′ . We note that 4MD2 = 2AD2 + 2BD2 − AB2 = 2a2 + 2b2 − AB2 and 4MN 2 = 4MD2 − CD2 = 2a2 + 2b2 − AB2 − CD2 . Now, AD′ 2 = (AM + D′ N)2 + MN 2 , which together with AM + D′ N = (a + b)/2 gives us a2 + b2 + AB ·CD a2 + b2 + c2 = . 2 2 p We conclude that min f (Q) = (a2 + b2 + c2 )/2. 2 AD′ = 20. If the faces of the tetrahedron ABCD are congruent triangles, we must have AB = CD, AC = BD, and AD = BC. Then the sum of angles at A is ∠BAC + ∠CAD + ∠DAB = ∠BDC + ∠CBD + ∠DCB = 180◦. We now assume that the sum of angles at each vertex is 180◦ . Let us construct triangles BCD′ ,CAD′′ , ABD′′′ in the plane ABC, exterior to △ABC, such that △BCD′ ∼ = △BCD, △CAD′′ ∼ = △CAD, and △ABD′′′ ∼ = △ABD. Then by the as′′ ′′′ sumption, A ∈ D D , B ∈ D′′′ D′ , and C ∈ D′ D′′ . Since also D′′ A = D′′′ A = DA, etc., A, B,C are the midpoints of segments D′′ D′′′ , D′′′ D′ , D′ D′′ respectively. Thus the triangles ABC, BCD′ , CAD′′ , ABD′′′ are congruent, and the statement follows. 21. Since the sum of all edges of ABCD is 3, the statement of the problem is an immediate consequence of the following statement: Lemma. √ Let r be the inradius of a triangle with sides a, b, c. Then a + b + c ≥ 6 3 · r, with equality if and only if the triangle is equilateral. Proof. If S and p denotes the area and semiperimeter of the triangle, by Heron’s formula and the AM–GM inequality we have p pr = S = p(p − a)(p − b)(p − c) s r   (p − a) + (p − b) + (p − c) 3 p4 p2 ≤ p = = √ , 3 27 3 3 √ i.e., p ≥ 3 3 · r, which is equivalent to the claim. 498 4 Solutions 4.28 Solutions to the Shortlisted Problems of IMO 1987 1. By (ii), f (x) = 0 has at least one solution, and there is the greatest among them, say x0 . Then by (v), for any x, 0 = f (x) f (x0 ) = f (x f (x0 ) + x0 f (x) − x0 x) = f (x0 ( f (x) − x)). (1) It follows that x0 ≥ x0 ( f (x) − x). Suppose x0 > 0. By (i) and (iii), since f (x0 ) − x0 < 0 < f (0) − 0, there is a number z between 0 and x0 such that f (z) = z. By (1), 0 = f (x0 ( f (z) − z)) = f (0) = 1, a contradiction. Hence, x0 < 0. Now the inequality x0 ≥ x0 ( f (x) − x) gives f (x) − x ≥ 1 for all x; so, f (1987) ≥ 1988. Therefore f (1987) = 1988. 2. Let di denote the number of cliques of which person i is a member. Clearly di ≥ 2. We now distinguish two cases: (i) For some i, di = 2. Suppose that i is a member of two cliques, Cp and Cq . Then |Cp | = |Cq | = n, since for each couple other than i and his/her spouse, one member is in Cp and one in Cq . There are thus (n − 1)(n − 2) pairs (r, s) of nonspouse persons distinct from i, where r ∈ C p , s ∈ Cq . We observe that each such pair accounts for a different clique. Otherwise, we find two members of C p or Cq who belong to one other clique. It follows that k ≥ 2 + (n − 1)(n − 2) ≥ 2n for n ≥ 4. (ii) For every i, di ≥ 3. Suppose that k < 2n. For i = 1, 2, . . . , 2n assign to person i an indeterminant xi , and for j = 1, 2, . . . , k set y = ∑i∈C j xi . From linear algebra, we know that if k < 2n, then there exist x1 , x2 , . . . , x2n , not all zero, such that y1 = y2 = · · · = yk = 0. On the other hand, suppose that y1 = y2 = · · · = yk = 0. Let M be the set of the couples and M ′ the set of all other pairs of persons. Then 0= k 2n j=1 i=1 ∑ y2j = ∑ di x2i + 2 ∑ xi x j (i, j)∈M ′ 2n = ∑ (di − 2)x2i + (x1 + x2 + · · · + x2n)2 + ∑ i=1 (i, j)∈M (xi − x j )2 2n ≥ ∑ x2i > 0, i=1 if not all x1 , x2 , . . . , x2n are zero, which is a contradiction. Hence k ≥ 2n. Remark. The condition n ≥ 4 is essential. For a party attended by 3 couples {(1, 4), (2, 5), (3, 6)}, there is a collection of 4 cliques satisfying the conditions: {(1, 2, 3), (3, 4, 5), (5, 6, 1), (2, 4, 6)}. 3. The answer: yes. Set p(k, m) = k + [1 + 2 + · · ·+ (k + m)] = It is obviously of the desired type. (k + m)2 + 3k + m . 2 4.28 Shortlisted Problems 1987 499 − → −→ −→ 4. Setting x1 = AB, x2 = AD, x3 = AE, we have to prove that |x1 + x2 | + |x2 + x3 | + |x3 + x1 | ≤ |x1 | + |x2 | + |x3 | + |x1 + x2 + x3 |. We have (|x1 | + |x2 | + |x3 |)2 − |x1 + x2 + x3 |2 = 2 ∑ (|xi ||x j | − hxi , x j i) = 1≤i< j≤3 ∑ = ∑ h i 2 (|xi | + |x j |)2 − xi + x j 1≤i< j≤3 (|xi | + |x j | + |xi + x j |)(|xi | + |x j | − |xi + x j |). 1≤i< j≤3 The following two inequalities are obvious: |xi | + |x j | − |xi + x j | ≥ 0, (1) |xi | + |x j | + |xi + x j | ≤ |x1 | + |x2 | + |x3 | + |x1 + x2 + x3 |. (2) It follows that (|x1 | + |x2 | + |x3 |)2 − |x1 + x2 + x3 |2 ! ≤ 3 3 i=1 i=1 ∑ |xi | + ∑ xi 3 2 ∑ |xi | − i=1 ∑ 1≤i< j≤3 ! |xi + x j | , and dividing by the positive number ∑3i=1 |xi | + ∑3i=1 xi we obtain 3 3 i=1 i=1 ∑ |xi | − ∑ xi 3 ≤ 2 ∑ |xi | − i=1 ∑ 1≤i< j≤3 |xi + x j |. The inequality is proven. Let us analyze the cases of equality. If one of the vectors is null, then equality obviously holds. Suppose that xi 6= 0, i = 1, 2, 3. For every i, j, at least one of (1) and (2) is equality. Equality in (1) holds if and only if xi and x j are collinear with the same direction, while in (2) it holds if and only if −xk and x1 + x2 + x3 are collinear with the same direction. If not all the vectors are collinear, then there are at least two distinct pairs xi , x j , i < j, for which (2) is an equality, so at least two of xi are collinear with x1 + x2 + x3 , but then so is the third; hence, the sum x1 + x2 + x3 must be 0. Thus the cases of equality are (a) the vectors are collinear with the same direction; (b) the vectors are collinear, two of them have the same direction, say xi , x j , and |xk | ≥ |xi | + |x j |; (c) one of the vectors is 0; (d) their sum is 0. Second solution. The following technique, although not quite elementary, is often used to effectively reduce geometric inequalities of first degree, like this one, to the one-dimensional case. Let σ be a fixed sphere with center O. For an arbitrary segment d in space, and any line l, we denote by πl (d) the length of the projection of d onto l. Consider the integral of lengths of these projections on all possible directions of OP, with 500 4 Solutions R P moving on the sphere: σ πOP (d) d σ . It is clear that this value depends only on the length of d (because of symmetry); hence Z σ πOP d σ = c · |d| for some constant c 6= 0. (1) Notice that by the one-dimensional case, for any point P ∈ σ , πOP (x1 ) + πOP (x2 ) + πOP (x3 ) + πOP (x1 + x2 + x3) ≥ πOP (x1 + x2 ) + πOP (x1 + x3 ) + πOP (x2 + x3 ). By integration on σ , using (1), we obtain c(|x1 | + |x2 | + |x3 | + |x1 + x2 + x3 |) ≥ c(|x1 + x2 | + |x1 + x3 | + |x2 + x3 |). 5. Assuming the notation a = BC, b = AC, c = AB; x = BL, y = CM, z = AN, from the Pythagorean theorem we obtain (a − x)2 + (b − y)2 + (c − z)2 = x2 + y2 + z2 = x2 + (a − x)2 + y2 + (b − y)2 + z2 + (c − z)2 . 2 Since x2 + (a − x)2 = a2 /2 + (a − 2x)2/2 ≥ a2 /2 and similarly y2 + (b − y)2 ≥ b2 /2 and z2 + (c − z)2 ≥ c2 /2, we get x2 + y2 + z2 ≥ a2 + b2 + c2 . 4 Equality holds if and only if P is the circumcenter of the triangle ABC, i.e., when x = a/2, y = b/2, z = c/2. 6. Suppose w.l.o.g. that a ≥ b ≥ c. Then 1/(b + c) ≥ 1/(a + c) ≥ 1/(a + b). Chebyshev’s inequality yields   an bn cn 1 1 1 1 + + ≥ (an + bn + cn ) + + . (1) b+c a+c a+b 3 b+c a+c a+b By the Cauchy-Schwarz inequality we have   1 1 1 2(a + b + c) + + ≥ 9, b+c a+c a+b and the mean inequality yields (an + bn + cn )/3 ≥ [(a + b + c)/3]n . We obtain from (1) that  n   an bn cn a+b+c 1 1 1 + + ≥ + + b+c a+c a+b 3 b+c a+c a+b    n−2 3 a + b + c n−1 2 ≥ = Sn−1 . 2 3 3 4.28 Shortlisted Problems 1987 501 7. For all real numbers v the following inequality holds: 4 ∑ (vi − v j )2 ≤ 5 ∑ (vi − v)2 . 0≤i< j≤4 (1) i=0 Indeed, ∑ (vi − v j )2 = 0≤i< j≤4 ∑ [(vi − v) − (v j − v)]2 0≤i< j≤4 4 = 5 ∑ (vi − v) − 2 i=0 4 ∑ (vi − v) i=0 !2 4 ≤ 5 ∑ (vi − v)2 . i=0 Let us first take vi ’s, satisfying condition (1), so that w.l.o.g. v0 ≤ v1 ≤ v2 ≤ v3 ≤ v4 ≤ 1 + v0 . Defining v5 = 1 + v0 , we see that one of the differences v j+1 − v j , j = 0, . . . , 4, is at most 1/5. Take v = (v j+1 + v j )/2, and then place the other three v j ’s in the segment [v − 1/2, v + 1/2]. Now we have |v − v j | ≤ 1/10, |v − v j+1 | ≤ 1/10, and |v − vk | ≤ 1/2, for any k different from j, j + 1. The vi ’s thus obtained have the required property. In fact, using the inequality (1), we obtain  2  2 ! 1 1 2 = 3.85 < 4. ∑ (vi − v j ) ≤ 5 2 10 + 3 2 0≤i< j≤4 Remark. The best possible estimate for the right-hand side is 2. 8. (a) Consider ai = ik + 1, i = 1, 2, . . . , m; b j = jm + 1, j = 1, 2, . . . , k. Assume that mk | ai b j − as bt = (ik + 1)( jm + 1) − (sk + 1)(tm + 1) = km(i j − st) + m( j − t) + k(i − s). Since m divides this sum, we get that m | k(i − s), or, together with gcd(k, m) = 1, that i = s. Similarly j = t, which proves part (a). (b) Suppose the opposite, i.e., that all the residues are distinct. Then the residue 0 must also occur, say at a1 b1 : mk | a1 b1 ; so, for some a′ and b′ , a′ | a1 , b′ | b1 , and a′ b′ = mk. Assuming that for some i, s 6= i, a′ | ai − as , we obtain mk = a′ b′ | ai b1 − as b1 , a contradiction. This shows that a′ ≥ m and similarly b′ ≥ k, and thus from a′ b′ = mk we have a′ = m, b′ = k. We also get (∗): all ai ’s give distinct residues modulo m = a′ , and all b j ’s give distinct residues modulo k = b′ . Now let p be a common prime divisor of m and k. By (∗), exactly p−1 p m of ai ’s and exactly (p−1)2 mk p2 p−1 p k of b j ’s are not divisible by p. Therefore there are precisely products ai b j that are not divisible by p, although from the assumption that they all give distinct residues it follows that the number (p−1)2 of such products is p−1 mk. We have arrived at a contradiction, p mk 6= p2 thus proving (b). 502 4 Solutions 9. The answer is yes. Consider the curve C = {(x, y, z) | x = t, y = t 3 , z = t 5 , t ∈ R}. Any plane defined by an equation of the form ax + by + cz + d = 0 intersects the curve C at points (t,t 3 ,t 5 ) with t satisfying ct 5 + bt 3 + at + d = 0. This last equation has at least one but only finitely many solutions. 10. Denote by r, R (take w.l.o.g. r < R) the radii and by A, B the centers of the spheres S1 , S2 respectively. Let s be the common radius of the spheres in the ring, C the center of one of them, say S, and D the foot of the perpendicular from C to AB. The centers of the spheres in the ring form a regular n-gon with center D, and thus sin(π /n) = s/CD. Using the Heron’s formula on the triangle ABC, we obtain (r + R)2CD2 = 4rRs(r + R + s), hence sin2 π s2 (r + R)2 s = = . 2 n CD 4(r + R + s)rR v A D r E1 C E s R E2 B (1) Choosing the unit of length so that r + R = 2, for simplicity of writing, we write (1) as 1/sin2 (π /n) = rR (1 + 2/s). Let now v be half the angle at the top of the cone. Then clearly R − r = (R + r) sin v = 2 sin v, giving us R = 1 + sin v, r = 1 − sinv. It follows that   1 2 = 1 + cos2 v. (2) s sin2 πn We need to express s as a function of R and r. Let E1 , E2 , E be collinear points of√tangency S2 , and S with the cone. Obviously, E1 E2 = E1 E + E2 E, i.e., √ of S1 ,√ 2 rs + 2 Rs = 2 Rr = (R + r) cosv = 2 cos v. Hence, √ √ √ cos2 v = s( R + r)2 = s(R + r + 2 Rr) = s(2 + 2 cosv). Substituting this into (2), we obtain 2 + cos v = 1/sin(π /n). Therefore 1/3 < sin(π /n) < 1/2, and we conclude that the possible values for n are 7, 8, and 9. 11. Let A1 be the set that contains 1, and let the minimal element of A2 be less than that of A3 . We shall construct the partitions with required properties by allocating successively numbers to the subsets that always obey the rules. The number 1 must go to A1 ; we show that for every subsequent number we have exactly two possibilities. Actually, while A2 and A3 are both empty, every successive number can enter either A1 or A2 . Further, when A2 is no longer empty, we use induction 4.28 Shortlisted Problems 1987 503 on the number to be placed, denote it by m: if m can enter Ai or A j but not Ak , and it enters Ai , then m + 1 can be placed in Ai or Ak , but not in A j . The induction step is finished. This immediately gives us that the final answer is 2n−1 . ′ B 12. Here all angles will be oriented and measured counterclockwise. Note that ∡CA′ B = ∡AB′C = ∡BC′ A = π /3. A Let a′ , b′ , c′ denote respectively the inner bisectors of angles A′ , B′ ,C′ in triangle A′ B′C′ . The lines a′ , b′ , c′ M X P K meet at the centroid X of A′ B′C′ , ′ C L and ∡(a′ , b′ ) = ∡(b′ , c′ ) = ∡(c′ , a′ ) = B C 2π /3. Now let K, L, M be the points such that KB = KC, LC = LA, MA = MB, and ∡BKC = ∡CLA = ∡AMB = A′ 2π /3, and let C1 , C2 , C3 be the circles circumscribed about triangles BKC, CLA, and AMB respectively. These circles are characterized by C1 = {Z | ∡BZC = 2π /3}, etc.; hence we deduce that they meet at a point P such that ∡BPC = ∡CPA = ∡APB = 2π /3 (Torricelli’s point). Points A′ , B′ ,C′ run over C1 r{P}, C2 r{P}, C3 r{P} respectively. As for a′ , b′ , c′ , we see that K ∈ a′ , L ∈ b′ , M ∈ c′ , and also that they can take all possible directions except KP, LP, MP respectively (if K = P, KP is assumed to be the corresponding tangent at K). Then, since ∡KXL = 2π /3, X runs over the circle defined by {Z | ∡KZL = 2π /3}, without P. But analogously, X runs over the circle {Z | ∡LZM = 2π /3}, from which we can conclude that these two circles are the same, both equal to the circumcircle of KLM, and consequently also that triangle KLM is equilateral (which is, anyway, a well-known fact). Therefore, the locus of the points X is the circumcircle of KLM minus point P. 13. We claim thatpthe points Pi (i, i2 ), i = 1, 2, . . . p , 1987, satisfy the conditions. In fact: (i) Pi Pj = (i − j)2 + (i2 − j2 )2 = |i − j| 1 + (i√+ j)2. It is known that for each positive p integer n, n is either an p integer or an irrational number. Since i + j < 1 + (i + j)2 < i + j + 1, 1 + (i + j)2 is not an integer, it is irrational, and so is Pi Pj . (ii) The area A of the triangle Pi Pj Pk , for distinct i, j, k, is given by i2 + j 2 j 2 + k2 k2 + i2 (i − j) + ( j − k) + (k − i) 2 2 2 (i − j)( j − k)(k − i) = ∈ Q r {0}, 2 A= also showing that this triangle is nondegenerate. 14. Let xn be the total number of counted words of length n, and yn , zn , un , zn , yn the numbers of counted words of length n starting with 0, 1, 2, 3, 4, respectively (indeed, by symmetry, words starting with 0 are equally numbered as those starting 504 4 Solutions with 4, etc.). We have the clear relations (1) yn = zn−1 ; (3) un = 2zn−1 ; (2) zn = yn−1 + un−1 ; (4) xn = 2yn + 2zn + un . From (1), (2), and (3) we get zn = zn−2 + 2zn−2 = 3zn−2 , with z1 = 1, z2 = 2, which gives z2n = 2 · 3n−1 , z2n+1 = 3n . Then (1), (3), and (4) obviously imply y2n = 3n−1 , u2n = 2 · 3n−1, x2n = 8 · 3n−1, y2n+1 = 2 · 3n−1; u2n+1 = 4 · 3n−1; x2n+1 = 14 · 3n−1; with the initial number x1 = 5. 15. Since x21 + x22 + · · · + x2n = 1, we get by the Cauchy-Schwarz inequality q √ |x1 | + |x2| + · · · + |xn| ≤ n(x21 + x22 + · · · + x2n ) = n. Hence all kn sums of the form e1 x1 + e2 x2 + · · · + en xn , with ei√∈ {0, 1, 2, . . . , k − 1}, must lie in some closed interval ℑ of length (k − 1) n.√ This interval can be covered with kn − 1 closed subintervals of length kk−1 n. By the pin −1 geonhole principle there must be two of these sums lying in the same subinterval. Their difference, which is of the form e1 x1 + e2 x2 + · · · + en xn where ei ∈ {0, ±1, . . . , ±(k − 1)}, satisfies √ (k − 1) n |e1 x1 + e2x2 + · · · + en xn | ≤ . kn − 1 16. We assume that S = {1, 2, . . ., n}, and use the obvious fact n ∑ pn (k) = n! (0) k=0 (a) To each permutation π of S we assign an n-vector (e1 , e2 , . . . , en ), where ei is 1 if i is a fixed point of π , and 0 otherwise. Since exactly pn (k) of the assigned vectors contain exactly k “1”s, the considered sum ∑nk=0 kpn (k) counts all the “1”s occurring in all the n! assigned vectors. But for each i, 1 ≤ i ≤ n, there are exactly (n − 1)! permutations that fix i; i.e., exactly (n − 1)! of the vectors have ei = 1. Therefore the total number of “1”s is n · (n − 1)! = n!, implying n ∑ kpn (k) = n!. k=0 (1) 4.28 Shortlisted Problems 1987 505 (b) In this case, to each permutation π of S we assign a vector (d1 , . . . , dn ) instead, with di = k if i is a fixed point of π , and di = 0 otherwise, where k is the number of fixed points of π . Let us count the sum Z of all components di for all the n! permutations. There are pn (k) such vectors with exactly k components equal to k, and sums of components equal to k2 . Thus, Z = ∑nk=0 k2 pn (k). On the other hand, we may first calculate the sum of all components di for fixed i. In fact, the value di = k > 0 will occur exactly pn−1 (k − 1) times, so that the sum of the di ’s is ∑nk=1 kpn−1 (k − 1) = ∑n−1 k=0 (k + 1)pn−1 (k) = 2(n − 1)!. Summation over i yields Z = ∑nk=0 k2 pn (k) = 2n!. (2) From (0), (1), and (2), we conclude that n n n n k=0 k=0 k=0 k=0 ∑ (k − 1)2 pn (k) = ∑ k2 pn(k) − 2 ∑ kpn (k) + ∑ pn(k) = n!. Remark. Only the first part of this problem was given on the IMO. 17. The number of 4-colorings of the set M is equal to 41987. Let A be the number of arithmetic progressions in M with 10 terms. The number of colorings containing a monochromatic arithmetic progression with 10 terms is less than 4A · 41977. So, if A < 49 , then there exist 4-colorings with the required property. Now we estimate the value of A. If the first term of a 10-term progression is k  and the difference is d, then 1 ≤ k ≤ 1978 and d ≤ 1987−k ; hence 9 A= 1978  ∑ k=1  1987 − k 1986 + 1985 + · · ·+ 9 1995 · 1978 < = < 49 . 9 9 18 18. Note first that the statement that some a + x, a + y, a + x + y belong to a class C is equivalent to the following statement: (1) There are positive integers p, q ∈ C such that p < q ≤ 2p. Indeed, given p, q, take simply x = y = q − p, a = 2p − q; conversely, if a, x, y (x ≤ y) exist such that a + x, a + y, a + x + y ∈ C, take p = a + y, q = a + x + y: clearly, p < q ≤ 2p. We will show that h(r) = 2r. Let {1, 2, . . ., 2r} = C1 ∪ C2 ∪ · · · ∪Cr be an arbitrary partition into r classes. By the pigeonhole principle, two among the r + 1 numbers r, r + 1, . . . , 2r belong to the same class, say i, j ∈ Ck . If w.l.o.g. i < j, then obviously i < j ≤ 2i, and so by (1) this Ck has the required property. On the other hand, we consider the partition {1, 2, . . ., 2r − t} = r−t [ k=1 {k, k + r} ∪ {r − t + 1} ∪ · · · ∪ {r} and prove that (1), and thus also the required property, does not hold. In fact, none of the classes in the partition contains p and q with p < q ≤ 2p, because k + r > 2k. 506 4 Solutions 19. The facts given in the problem allow us to draw a triangular pyramid with angles 2α , 2β , 2γ at the top and lateral edges of length 1/2. At the base there is a triangle whose side lengths are exactly sin α , sin β , sin γ . The area of this triangle does not exceed the sum of areas of the lateral sides, which equals (sin 2α + sin 2β + sin 2γ )/8. 20. Let y be the smallest nonnegative integer with y ≤ p − 2 for which f (y) is a composite number. Denote by q the smallest prime divisor of f (y). We claim that y < q. Suppose the contrary, that y ≥ q. Let r be a positive integer such that y ≡ r (mod q). Then f (y) ≡ f (r) ≡ 0 (mod q), and since q ≤ y ≤ p − 2 ≤ f (r), we conclude that q | f (r), which is a contradiction to the minimality of y. Now, we will prove that q > 2y. Suppose the contrary, that q ≤ 2y. Since f (y) − f (x) = (y − x)(y + x + 1), we observe that f (y) − f (q − 1 − y) = (2y − q + 1)q, from which it follows that f (q − 1 − y) is divisible by q. But by the assumptions, q − 1 − y < y, implying that f (q − 1 − y) is prime and therefore equal to q. This is impossible, because f (q − 1 − y) = (q − 1 − y)2 + (q − 1 − y) + p > q + p − y − 1 ≥ q. Therefore q ≥ 2y + 1. Now, since f (y), being composite, cannot be equal to q, and q is its smallest prime divisor, we obtain that f (y) ≥ q2 . Consequently, y2 + y + p ≥ q2 ≥ (2y + 1)2 = 4y2 + 4y + 1 ⇒ 3(y2 + y) ≤ p − 1, p and from this we easily conclude that y < p/3, which contradicts the condition of the problem. In this way, all the numbers f (0), f (1), . . . , f (p − 2) must be prime. A 21. Let P be the second point of intersection of segment BC and the circle circumscribed about quadrilateral AKLM. Denote by E the intersection point of the lines KN and BC and by F the intersection point of the lines M MN and BC. Then ∠BCN = ∠BAN K and ∠MAL = ∠MPL, as angles on E P B C F the same arc. Since AL is a bisector, ∠BCN = ∠BAL = ∠MAL = ∠MPL, N and consequently PM k NC. Similarly we prove KP k BN. Then the quadrilaterals BKPN and NPMC are trapezoids; hence SBKE = SNPE and SNPF = SCMF . Therefore SABC = SAKNM . 4.28 Shortlisted Problems 1987 507 22. Suppose that there exists such function f . Then we obtain f (n + 1987) = f ( f ( f (n))) = f (n) + 1987 for all n ∈ N, and from here, by induction, f (n + 1987t) = f (n) + 1987t for all n,t ∈ N. Further, for any r ∈ {0, 1, . . .,1986}, let f (r) = 1987k + l, k, l ∈ N, l ≤ 1986. We have r + 1987 = f ( f (r)) = f (l + 1987k) = f (l) + 1987k, and consequently there are two possibilities: (i) k = 1 ⇒ f (r) = l + 1987 and f (l) = r; (ii) k = 0 ⇒ f (r) = l and f (l) = r + 1987; in both cases, r 6= l. In this way, the set {0, 1, . . . , 1986} decomposes into pairs {a, b} such that f (a) = b and f (b) = a + 1987, or f (b) = a and f (a) = b + 1987. But the set {0, 1, . . . , 1986} has an odd number of elements, and cannot be decomposed into pairs. Contradiction. 23. If we prove the existence of p, q ∈ N such that the roots r, s of f (x) = x2 − kp · x + kq = 0 are irrational real numbers with 0 < s < 1 (and consequently r > 1), then we are done, because from r + s, rs ≡ 0 (mod k) we get rm + sm ≡ 0 (mod k), and 0 < sm < 1 yields the assertion. To prove the existence of such natural numbers p and q, we can take them such that f (0) > 0 > f (1), i.e., kq > 0 > k(q − p) + 1 ⇒ p > q > 0. The irrationality of r can be obtained by taking q = p − 1, because the discriminant D = (kp)2 − 4kp + 4k, for (kp − 2)2 < D < (kp − 1)2 , is not a perfect square for p ≥ 2. 508 4 Solutions 4.29 Solutions to the Shortlisted Problems of IMO 1988 1. Assume that p and q are real and b0 , b1 , b2 , . . . is a sequence such that bn = pbn−1 + qbn−2 for all n > 1. From the equalities bn = pbn−1 + qbn−2, bn+1 = pbn + qbn−1, bn+2 = pbn+1 + qbn , eliminating bn+1 and bn−1 we obtain that bn+2 = (p2 + 2q)bn − q2 bn−2 . So the sequence b0 , b2 , b4 , . . . has the property b2n = Pb2n−2 + Qb2n−4, P = p2 + 2q, Q = −q2 . (1) We shall solve the problem by induction. The sequence an has p = 2, q = 1, and hence P = 6, Q = −1. Let k = 1. Then a0 = 0, a1 = 1, and an is of the same parity as an−2 ; i.e., it is even if and only if n is even. Let k ≥ 1. We assume that for n = 2k m, the numbers an are divisible by 2k , but divisible by 2k+1 if and only if m is even. We assume also that the sequence c0 , c1 , . . . , with cm = am·2k , satisfies the condition cn = pcn−1 − cn−2 , where p ≡ 2 (mod 4) (for k = 1 it is true). We shall prove the same statement for k + 1. According to (1), c2n = Pc2n−2 − c2n−4 , where P = p2 − 2. Obviously P ≡ 2 (mod 4). Since P = 4s + 2 for some integer s, and c2n = 2k+1 d2n , c0 = 0, c1 ≡ 2k (mod 2k+1 ), and c2 = pc1 ≡ 2k+1 (mod 2k+2 ), we have c2n = (4s + 2)2k+1d2n−2 − c2n−4 ≡ c2n−4 (mod 2k+2 ), i.e., 0 ≡ c0 ≡ c4 ≡ c8 ≡ · · · and 2k+1 ≡ c2 ≡ c6 ≡ · · · (mod 2k+2 ), which proves the statement. Second solution. The recursion is solved by       √ √  1  n n n an = √ (1 + 2)n − (1 − 2)n = +2 + 22 + ··· . 1 3 5 2 2 Let n = 2k m with m odd; then for p > 0 the summand     n m n−1 p k+p (n − 1) . . .(n − 2p) k+p 2 =2 m =2 2p + 1 (2p + 1)! 2p + 1 2p is divisible by 2k+p , because the denominator 2p + 1 is odd. Hence   n p an = n + ∑ 2 = 2k m + 2k+1N 2p + 1 p>0 for some integer N, so that an is exactly divisible by 2k . Third solution. It can be proven by induction that a2n = 2an (an + an+1 ). The required result follows easily, again by induction on k. 2. For polynomials f (x), g(x) with integer coefficients, we use the notation f (x) ∼ g(x) if all the coefficients of f − g are even. Let n = 2s . It is immediately shown s s+1 s by induction that (x2 + x + 1)2 ∼ x2 + x2 + 1, and the required number for s n = 2 is 3. Let n = 2s − 1. If s is odd, then n ≡ 1 (mod 3), while for s even, n ≡ 0 (mod 3). Consider the polynomial 4.29 Shortlisted Problems 1988 Rs (x) = 509  (x + 1)(x2n−1 + x2n−4 + · · · + xn+3 ) + xn+1     +xn + xn−1 + (x + 1)(xn−4 + xn−7 + · · · + 1), 2 ∤ s;  (x + 1)(x2n−1 + x2n−4 + · · · + xn+2 ) + xn    +(x + 1)(xn−3 + xn−6 + · · · + 1), s+1 2 | s. s s It is easily checked that (x2 + x + 1)Rs (x) ∼ x2 + x2 + 1 ∼ (x2 + x + 1)2 , s so that Rs (x) ∼ (x2 + x + 1)2 −1 . In this case, the number of odd coefficients is s+2 s (2 − (−1) )/3. Now we pass to the general case. Let the number n be represented in the binary system as n = 11 . . . 1} 00 . . . 0} 11 . . . 1} 00 . . . 0} . . . 11 . . . 1} 00 . . .0}, | {z | {z | {z | {z | {z | {z ak ak−1 bk a1 bk−1 b1 bi > 0 (i > 1), b1 ≥ 0, and ai > 0. Then n = ∑ki=1 2si (2ai − 1), where si = b1 + a1 + b2 + a2 + · · · + bi , and hence k un (x) = (x2 + x + 1)n = ∏(x2 + x + 1)2 i (2 s i=1 si ri,di ri,1 2 (x ) ∼ x +· · ·+x ; clearly r ai −1) k ∼ ∏ Rai (x2 i ). s i=1 Let Rai and ri, j ≤ 2si+1 (2ai − i, j s i+1 1) < 2 , so that for any j, ri, j can have nonzero binary digits only in some position t, si ≤ t ≤ si+1 − 1. Therefore, in k is divisible by 2si k k di ∏ Rai (x2 i ) ∼ ∏(xri,1 + · · · + xri,di ) = ∑ ∑ xr1,p1 +r2,p2 +···+rk,pk i=1 s i=1 i=1 pi =1 all the exponents r1,p1 + r2,p2 + · · · + rk,pk are different, so that the number of odd coefficients in un (x) is k k 2ai +2 − (−1)ai . 3 i=1 ∏ di = ∏ i=1 3. Let R be the circumradius, r the inradius, s the semiperimeter, ∆ the area of ABC and ∆ ′ the area of A′ B′C′ . The angles of triangle A′ B′C′ are A′ = 90◦ − A/2, B′ = 90◦ − B/2, and C′ = 90◦ −C/2, and hence ∆ = 2R2 sin A sin B sinC and ∆ ′ = 2R2 sin A′ sin B′ sinC′ = 2R2 cos Hence, A B C cos cos . 2 2 2 ∆ sin A sin B sinC A B C 2r = = 8 sin sin sin = . ∆ ′ cos A2 cos B2 cos C2 2 2 2 R Here we used that r = AI sin(A/2) = · · · = 4R sin(A/2) · sin(B/2) · sin(C/2). Euler’s inequality 2r ≤ R shows that ∆ ≤ ∆ ′ . Second solution. Let H be orthocenter of triangle ABC, and Ha , Hb , Hc points symmetric to H with respect to BC,CA, AB, respectively. Since ∠BHaC = 510 4 Solutions ∠BHC = 180◦ − ∠A, points Ha , Hb , Hc lie on the circumcircle of ABC, and the area of the hexagon AHc BHaCHb is double the area of ABC. (1) Let us apply the analogous result for the triangle A′ B′C′ . Since its orthocenter is the incenter I of ABC, and the point symmetric to I with respect to B′C′ is the point A, we find by (1) that the area of the hexagon AC′ BA′CB′ is double the area of A′ B′C′ . But it is clear that the area of ∆ CHa B is less than or equal to the area of ∆ CA′ B etc.; hence, the area of AHc BHaCHb does not exceed the area of AC′ BA′CB′ . The statement follows immediately. 4. Suppose that the numbers of any two neighboring squares differ by at most n −1. For k = 1, 2, . . . , n2 − n, let Ak , Bk , and Ck denote, respectively, the sets of squares numbered by 1, 2, . . . , k; of squares numbered by k + n, k + n + 1, . . ., n2 ; and of squares numbered by k + 1, . . . ,k + n − 1. By the assumption, the squares from Ak and Bk have no edge in common; Ck has n − 1 elements only. Consequently, for each k there exists a row and a column all belonging either to Ak , or to Bk . For k = 1, it must belong to Bk , while for k = n2 − n it belongs to Ak . Let k be the smallest index such that Ak contains a whole row and a whole column. Since Bk−1 has that property too, it must have at least two squares in common with Ak , which is impossible. 5. Let n = 2k and let A = {A1 , . . . , A2k+1 } denote the family of sets with the desired properties. Since every S element of their union B belongs to at least two sets of A, it follows that A j = i6= j Ai ∩ A j holds for every 1 ≤ j ≤ 2k + 1. Since each intersection in the sum has at most one element and A j has 2k elements, it follows that every element of A j , i.e., in general of B, is a member of exactly two sets. We now prove that k is even, assuming that the marking described in the problem exists. We have already shown that for every two indices 1 ≤ j ≤ 2k +1 and i 6= j there exists a unique element contained in both Ai and A j . On a 2k × 2k matrix let us mark in the ith column and jth row for i 6= j the number that was joined to the element of B in Ai ∩ A j . In the ith row and column let us mark the number of the element of B in Ai ∩ A2k+1 . In each row from the conditions of the marking there must be an even number of zeros. Hence, the total number of zeros in the matrix is even. The matrix is symmetric with respect to its main diagonal; hence it has an even number of zeros outside its main diagonal. Hence, the number of zeros on the main diagonal must also be even and this number equals the number of elements in A2k+1 that are marked with 0, which is k. Hence k must be even. For even k we note that the dimensions of a 2k × 2k matrix are divisible by 4. Tiling the entire matrix with the 4 × 4 submatrix   0101 1 0 1 0  Q=  0 1 1 0 , 1001 we obtain a marking that indeed satisfies all the conditions of the problem; hence we have shown that the marking is possible if and only if k is even. 4.29 Shortlisted Problems 1988 511 6. Let ω be the plane through AB, parallel to CD. Define the point transformation f : X 7→ X ′ in space as follows. If X ∈ KL, then X ′ = X ; otherwise, let ωX be the plane through X parallel to ω : then X ′ is the point symmetric to X with respect to the intersection point of KL with ωX . Clearly, f (A) = B, f (B) = A, f (C) = D, f (D) = C; hence f maps the tetrahedron onto itself. We shall show that f preserves volumes. Let s : X 7→ X ′′ denote the symmetry with respect to KL, and g the transformation mapping X ′′ into X ′ ; then f = g ◦ s. If points X1′′ = s(X1 ) and X2′′ = s(X2 ) have the property that X1′′ X2′′ is parallel to KL, then the segments X1′′ X2′′ and X1′ X2′ have the same length and lie on the same line. Then by Cavalieri’s principle g preserves volume, and so does f . Now, if α is any plane containing the line KL, the two parts of the tetrahedron on which it is partitioned by α are transformed into each other by f , and therefore have the same volumes. Second solution. Suppose w.l.o.g. that the plane α through KL meets the interiors of edges AC and BD at X −→ − → − → and Y . Let AX = λ AC and BY = −→ µ BD, for 0 ≤ λ , µ ≤ 1. Then the − → − → −→ −→ vectors KX = λ AC − AB/2, KY = → → −→ − − → − −→ µ BD + AB/2, KL = AC/2 + BD/2 are coplanar; i.e., there exist real numbers a, b, c, not all zero, such that D L Y C X A K B − → − → → −→ −→ − → −→ b − a − 0 = aKX + bKY + cKL = (λ a + c/2)AC + (µ b + c/2)BD+ AB. 2 − → −→ − → Since AC, BD, AB are linearly independent, we must have a = b and λ = µ . We need to prove that the volume of the polyhedron KX LY BC, which is one of the parts of the tetrahedron ABCD partitioned by α , equals half of the volume V of ABCD. Indeed, we obtain 1 1 1 VKXLY BC = VKXLC +VKBYLC = (1 − λ )V + (1 + µ )V = V. 4 4 2 7. The algebraic equation x3 − 3x2 + 1 = 0 admits three real roots β , γ , a, with √ −0.6 < β < −0.5, 0.6 < γ < 0.7, 8 < a < 3. Define, for all integers n, un = β n + γ n + an . It holds that un+3 = 3un+2 − un . Obviously, 0 < β n + γ n < 1 for all n ≥ 2, and we see that un − 1 = [an ] for n ≥ 2. It is now a question whether u1788 − 1 and u1988 − 1 are divisible by 17. Working modulo 17, we get u0 ≡ 3, u1 ≡ 3, u2 ≡ 9, u3 ≡ 7, u4 ≡ 1, . . . , u16 = 3, u17 = 3, u18 = 9. Thus, un is periodic modulo 17, with period 16. Since 1788 = 512 4 Solutions 16 · 111 + 12, 1988 = 16 · 124 + 4, it follows that u1788 ≡ u12 ≡ 1 and u1988 ≡ u4 = 1. So, [a1788 ] and [a1988 ] are divisible by 17. Second solution. The polynomial x3 − 3x2 + 1 allows the factorization modulo 17 as (x − 4)(x − 5)(x + 6). Hence it is easily seen that un ≡ 4n + 5n + (−6)n . Fermat’s theorem gives us 4n ≡ 5n ≡ (−6)n ≡ 1 for 16 | n, and the rest follows easily. Remark. In fact, the roots of x3 − 3x2 + 1 = 0 are 1 1 2 sin 10◦ , 2 sin 50◦ , and − 2 sin170◦ . 8. Consider first the case that the vectors are on the same line. Then if e is a unit vector, we can write u1 = x1 e, . . . , un = xn e for scalars xi , |xi | ≤ 1, with zero sum. It is now easy to permute x1 , x2 , . . . , xn into z1 , z2 , . . . zn so that |z1 | ≤ 1, |z1 + z2 | ≤ 1, . . . , |z1 + z2 + · · · + zn−1 | ≤ 1. Indeed, suppose w.l.o.g. that z1 = x1 ≥ 0; then we choose z2 , . . . , zr from the xi ’s to be negative, until we get to the first r with x1 + x2 + · · · + xr ≤ 0; we continue successively choosing positive z j ’s from the remaining xi ’s until we get the first partial sum that is positive, and so on. It is easy to verify that |z1 + z2 + · · · + z j | ≤ 1 for all j = 1, 2, . . . , n. Now we pass to the general case. Let s be the longest vector that can be obtained by summing a subset of u1 , . . . , um , and assume w.l.o.g. that s = u1 + · · · + u p . Further, let δ and δ ′ respectively be the lines through the origin O in the direction of s and perpendicular to s, and e, e′ respectively the unit vectors on δ and δ ′ . Put ui = xi e + yi e′ , i = 1, 2, . . . , m. By the definition of δ and δ ′ , we have |xi |, |yi | ≤ 1; x1 + · · · + xm = y1 + · · ·+ym = 0; y1 +· · · +y p = y p+1 + · · ·+ym = 0; we also have x p+1 , . . . , xm ≤ 0 (otherwise, if xi > 0 for some i, then |s + vi | > |s|), and similarly x1 , . . . , x p ≥ 0. Finally, suppose by the one-dimensional case that y1 , . . . , y p and y p+1 , . . . , ym are permuted in such a way that all the sums y1 + · · · + yi and y p+1 + · · · + y p+i are ≤ 1 in absolute value. We apply the construction of the one-dimensional case to x1 , . . . , xm taking, as described above, positive zi ’s from x1 , x2 , . . . , x p and negative ones from x p+1 , . . . , xm , but so that the order is preserved; this way we get a permutation xσ1 , xσ2 , . . . , xσm . It is then clear that each sum yσ1 + yσ2 + · · · + yσk decomposes into the sum (y1 + y2 + · · · + yl ) + (y p+1 + · · · + y p+n ) (because of the preservation of order), and that each of these sums is less than or equal to 1 in absolute value. Thus each sum uσ1 + · · · + uσk is composed of a vector of length at most 2√and an orthogonal vector of length at most 1, and so is itself of length at most 5. 2 2 +b 9. Let us assume aab+1 = k ∈ N. We then have a2 − kab + b2 = k. Let us assume that k is not an integer square, which implies k ≥ 2. Now we observe the minimal pair (a, b) such that a2 − kab + b2 = k holds. We may assume w.l.o.g. that a ≥ b. For a = b we get k = (2 − k)a2 ≤ 0; hence we must have a > b. Let us observe the quadratic equation x2 − kbx + b2 − k = 0, which has solutions a and a1 . Since a + a1 = kb, it follows that a1 ∈ Z. Since a > kb implies k > a + b2 > kb and a = kb implies k = b2 , it follows that a < kb and thus b2 > k. Since 2 2 aa1 = b2 − k > 0 and a > 0, it follows that a1 ∈ N and a1 = b a−k < a a−1 < a . We have thus found an integer pair (a1 , b) with 0 < a1 < a that satisfies the original 4.29 Shortlisted Problems 1988 513 equation. This is a contradiction of the initial assumption that (a, b) is minimal. Hence k must be an integer square. 10. We claim that if the family {A1 , . . . , At } separates the n-set N, then 2t ≥ n. The proof goes by induction. The case t = 1 is clear, so suppose that the claim holds for t − 1. Since At does not separate elements of its own or its complement, it follows that {A1 , . . . , At−1 } is separating for both At and N r At , so that |At |, |N r At | ≤ 2t−1 . Then |N| ≤ 2 · 2t−1 = 2t , as claimed. Also, if the set N with N = 2t is separated by {A1 , . . . , At }, then (precisely) one element of N is not covered. To show this, we again use induction. This is trivial for t = 1, so let t ≥ 1. Since A1 , . . . , At−1 separate both At and N r At , N r At must have exactly 2t−1 elements, and thus one of its elements is not covered by A1 , . . . , At−1 , and neither is covered by At . We conclude that a separating and covering family of t subsets can exist only if n ≤ 2t − 1. We now construct such subsets for the set N if 2t−1 ≤ n ≤ 2t − 1, t ≥ 1. For t = 1, put A1 = {1}. In the step from t to t +1, let N = N ′ ∪N ′′ ∪{y}, where |N ′ |, |N ′′ | ≤ 2t−1 ; let A′1 , . . . , At′ be subsets covering and separating N ′ and A′′1 , . . . , At′′ such subsets for N ′′ . Then the subsets Ai = A′i ∪ A′′i (i = 1, . . . ,t) and At+1 = N ′′ ∪ {y} obviously separate and cover N. The answer: t = [log2 n] + 1. Second solution. Suppose that the sets A1 , . . . , At cover and separate N. Label each element x ∈ N with a string (x1 x2 . . . xt ) of 0’s and 1’s, where xi is 1 when x ∈ Ai , 0 otherwise. Since the Ai ’s separate, these strings are distinct; since they cover, the string (00 . . . 0) does not occur. Hence n ≤ 2t − 1. Conversely, for 2t−1 ≤ n < 2t , represent the elements of N in base 2 as strings of 0’s and 1’s of length t. For 1 ≤ i ≤ t, take Ai to be the set of numbers in N whose binary string has a 1 in the ith place. These sets clearly cover and separate. 11. The answer is 32. Write the combinations as triples k = (x, y, z), 0 ≤ x, y, z ≤ 7. Define the sets K1 = {(1, 0, 0), (0, 1, 0), (0, 0, 1), (1, 1, 1)}, K2 = {(2, 0, 0), (0, 2, 0), (0, 0, 2), (2, 2, 2)}, K3 = {(0, 0, 0), (4, 4, 4)}, and K = {k = k1 + k2 + k3 | ki ∈ Ki , i = 1, 2, 3}. There are 32 combinations in K. We shall prove that these combinations will open the safe in every case. Let t = (a, b, c) be the right combination. Set k3 = (0, 0, 0) if at least two of a, b, c are less than 4, and k3 = (4, 4, 4) otherwise. In either case, the difference t − k3 contains two nonnegative elements not greater than 3. Choosing a suitable k2 we can achieve that t − k3 − k2 contains two elements that are 0, 1. So, there exists k1 such that t − k3 − k2 − k1 = t − k contains two zeros, for k ∈ K. This proves that 32 is sufficient. Suppose that K is a set of at most 31 combinations. We say that k ∈ K covers the combination k1 if k and k1 differ in at most one position. One of the eight sets Mi = {(i, y, z) | 0 ≤ y, z ≤ 7}, i = 0, 1, . . . , 7, contains at most three elements of K. Suppose w.l.o.g. that this is M0 . Further, among the eight sets N j = {(0, j, z) | 0 ≤ z ≤ 7}, j = 0, . . . , 7, there are at least five, say w.l.o.g. N0 , . . . , N4 , not containing any of the combinations from K. 514 4 Solutions Of the 40 elements of the set N = {(0, y, z) | 0 ≤ y ≤ 4, 0 ≤ z ≤ 7}, at most 5 · 3 = 15 are covered by K ∩ M0 , and at least 25 aren’t. Consequently, the intersection of K with L = {(x, y, z) | 1 ≤ x ≤ 7, 0 ≤ y ≤ 4, 0 ≤ z ≤ 7} contains at least 25 elements. So K has at most 31 − 25 = 6 elements in the set P = {(x, y, z) | 0 ≤ x ≤ 7, 5 ≤ y ≤ 7, 0 ≤ z ≤ 7}. This implies that for some j ∈ {5, 6, 7}, say w.l.o.g. j = 7, K contains at most two elements in Q j = {(x, y, z) | 0 ≤ x, z ≤ 7, y = j}; denote them by l1 , l2 . Of the 64 elements of Q7 , at most 30 are covered by l1 and l2 . But then there remain 34 uncovered elements, which must be covered by different elements of K r Q7 , having itself at most 29 elements. Contradiction. 12. Let E(XY Z) stand for the area of a triangle XY Z. We have E1 E(AMR) E(AMK) E(ABK) MR AM BK = · · = · · E E(AMK) E(ABK) E(ABC) MK AB BC We similarly obtain  E1 E 1/3 1 ≤ 3  MR AM BK + + MK AB BC  .  E2 E 1/3 1 ≤ 3  KR BM CK + + MK AB BC  . ⇒ √ √ √ 1/3 1/3 Therefore (E1 /E) +√ (E2 /E)√ ≤ 1,√i.e., 3 E1 + 3 E2 ≤ 3 E. Analogously, √ √ √ 3 E3 + 3 E4 ≤ 3 E and 3 E5 + 3 E6 ≤ 3 E; hence p 8 6 E 1 E2 E3 E4 E5 E6 p √ p p √ √ = 2( 3 E1 3 E2 )1/2 · 2( 3 E3 3 E4 )1/2 · 2( 3 E5 3 E6 )1/2 p p p √ √ √ ≤ ( 3 E1 + 3 E2 ) · ( 3 E3 + 3 E4 ) · ( 3 E5 + 3 E6 ) ≤ E. 13. Let AB = c, AC = b, ∠CBA = β , BC = a, and AD = h. Let r1 and r2 be the inradii of ABD and ADC respectively and O1 and O2 the centers of the respective incircles. A It is obvious that√ r1 /r2 = c/b. √ We also have DO1 = 2r1 , DO2 = 2r2 , P and ∠O1 DA = ∠O2 DA = 45◦ . Hence L ◦ ∠O1 DO2 = 90 and DO1 /DO2 = c/b K O2 from which it follows that △O1 DO2 ∼ O1 △BAC. We now define P as the B D C intersection of the circumcircle of △O1 DO2 with DA. From the above similarity we have ∠DPO2 = ∠DO1 O2 = β = ∠DAC. It follows that PO2 k AC and from ∠O1 PO2 = 90◦ it also follows that PO1 k AB. We also have ∠PO1 O2 = ∠PO2 O1 = 45◦ ; hence ∠LKA = ∠KLA = 45◦ , and thus AK = AL. From ∠O1 KA = ∠O1 DA = 45◦, O1 A = O1 A, and ∠O1 KA = ∠O1 DA we have △O1 KA ∼ = △O1 DA and hence AL = AK = AD = h. Thus 4.29 Shortlisted Problems 1988 515 E ah/2 a a2 b2 + c2 = 2 = = = ≥2. E1 h /2 h ah bc Remark. It holds that for an arbitrary triangle ABC, AK = AL if and only if AB = AC or ∡BAC = 90◦ . 14. Consider an array [ai j ] of the given property and denote the sums of the rows and the columns by ri and c j respectively. Among the ri ’s and c j ’s, one element of [−n, n] is missing, so that there are at least n nonnegative and n nonpositive sums. By permuting rows and columns we can obtain an array in which r1 , . . . , rk and c1 , . . . , cn−k are nonnegative. Clearly n n n i=1 j=1 r=−n ∑ |ri | + ∑ |c j | ≥ ∑ |r| − n = n2 . But on the other hand, n n k n i=1 j=1 i=1 i=k+1 ∑ |ri | + ∑ |c j | = ∑ ri − ∑ = n−k ri + ∑ c j − j=1 ∑ ai j − ∑ a i j + ∑ i≤k k n−k i>k = 2 ∑ ∑ ai j − 2 i=1 j=1 j≤n−k n ∑ n ∑ cj = j=n−k+1 ai j − ∑ ai j = j>n−k n ∑ i=k+1 j=n−k+1 ai j ≤ 4k(n − k). This yields n2 ≤ 4k(n − k), i.e., (n − 2k)2 ≤ 0, and thus n must be even. We proceed to show by induction that for all even n an array of the given type exists. For n = 2 the array in Fig. 1 is good. Let such an n × n array be given for some even n ≥ 2, with c1 = n, c2 = −n + 1, c3 = n − 2, . . ., cn−1 = 2, cn = −1 and r1 = n − 1, r2 = −n + 2, . . .,rn−1 = 1, rn = 0. Upon enlarging this array as indicated in Fig. 2, the positive sums are increased by 2, the nonpositive sums are decreased by 2, and the missing sums −1, 0, 1, 2 occur in the new rows and columns, so that the obtained array (n + 2) × (n + 2) is of the same type. 1 0 1 -1 1 -1 n×n 1 -1 1 -1 1 -1 1 1 -1 1 -1 1 Fig. 1 Fig. 2 1 -1 1 -1 0 -1 15. Referring to the description of LA , we have ∠AMN = ∠AHN = 90◦ − ∠HAC = ∠C, and similarly ∠ANM = ∠B. Since the triangle ABC is acute-angled, the line LA lies inside the angle A. Hence if P = LA ∩ BC and Q = LB ∩ AC, we get ∠BAP = 90◦ − ∠C; hence AP passes through the circumcenter O of ∆ ABC. 516 4 Solutions Similarly we prove that LB and LC contains the circumcenter O also. It follows that LA , LB and LC intersect at the point O. Remark. Without identifying the point of intersection, one can prove the concurrence of the three lines using Ceva’s theorem, in usual or trigonometric form. k 16. Let f (x) = ∑70 k=1 x−k . For all integers i = 1, . . . , 70 we have that f (x) tends to plus infinity as x tends downward to i, and f (x) tends to minus infinity as x tends upward to i. As x tends to infinity, f (x) tends to 0. Hence it follows that there exist x1 , x2 , . . . , x70 such that 1 < x1 < 2 < x2 < 3 < · · · < x69 < 70 < x70 and f (xi ) = 54 for all i = 1, . . . , 70. Then the solution to the inequality is given by S S = 70 i=1 (i, xi ]. For numbers x for which f (x) is well-defined, the equality f (x) = 54 is equivalent to 70 4 70 70 p(x) = ∏ (x − j) − ∑ k ∏(x − j) = 0. 5 k=1 j=1 j=1 j6=k The numbers x1 , x2 , . . . , x70 are then the zeros of this polynomial. The sum  70 4 69 ∑70 i=1 xi is then equal to minus the coefficient of x in p, which is ∑i=1 i + 5 i . Finally, 70 4 70 4 70 · 71 |S| = ∑ (xi − i) = · ∑ i = · = 1988 . 5 i=1 5 2 i=1 17. Let AC and AD meet BE in R, S, respectively. Then by the conditions of the problem, ∠AEB = ∠EBD = ∠BDC = ∠DBC = ∠ADB = ∠EAD = α , ∠ABE = ∠BEC = ∠ECD = ∠CED = ∠ACE = ∠BAC = β , ∠BCA = ∠CAD = ∠ADE = γ . Since ∠SAE = ∠SEA, it follows that AS = SE, and analogously BR = RA. But BSDC and REDC are parallelograms; hence BS = CD = RE, giving us BR = SE and AR = AS. Then also AC = AD, because RS k CD. We deduce that 2β = ∠ACD = ∠ADC = 2α , i.e., α = β . It will be sufficient to show that α = γ , since that will imply α = β = γ = 36◦ . We have that the sum of the interior angles of ACD is 4α + γ = 180◦ . We have sin γ AE AE AE sin(2α + γ ) = = = = , sin α DE CD RE sin(α + γ ) i.e., cos α − cos(α + 2γ ) = 2 sin γ sin(α + γ ) = 2 sin α sin(2α + γ ) = cos(α + γ ) − cos(3α + γ ). From 4α + γ = 180◦ we obtain − cos(3α + γ ) = cos α . Hence γ 2α + 3γ cos(α + γ ) + cos(α + 2γ ) = 2 cos cos = 0, 2 2 so that 2α + 3γ = 180◦ . It follows that α = γ . 4.29 Shortlisted Problems 1988 517 Second solution. We have ∠BEC = ∠ECD = ∠DEC = ∠ECA = ∠CAB, and hence the trapezoid BAEC is cyclic; consequently, AE = BC. Similarly AB = ED, and ABCD is cyclic as well. Thus ABCDE is cyclic and has all sides equal; i.e., it is regular. 2 2 2 18. (a) Define ∠APO = pφ and S = AB + AC + BC . We calculate PA = 2r cos φ 2 2 2 and PB, PC = R − r cos φ ± r sin φ . We also have AB2 = PA2 + PB2 , AC2 = PA2 + PC2 and BC = BP + PC. Combining all these we obtain S = AB2 + AC2 + BC2 = 2(PA2 + PB2 + PC2 + PB · PC) = 2(4r2 cos2 φ + 2(R2 − r2 cos2 φ + r2 sin2 φ ) + R2 − r2 ) = 6R2 + 2r2. Hence it follows that S is constant; i.e., it does not depend on φ . (b) Let B1 and C1 respectively be points such that APBB1 and APCC1 are rectangles. It is evident that B1 and C1 lie on the larger circle and that −−→ −→ 1 −−→ −→ PU = 2 PB1 and PV = 12 PC1 . It is evident that we can arrange for an arbitrary point on the larger circle to be B1 or C1 . Hence, the locus of U and V is equal to the circle obtained when the larger circle is shrunk by a factor of 1/2 with respect to point P. 19. We will show that f (n) = n for every n (thus also f (1988) = 1988). Let f (1) = r and f (2) = s. We obtain respectively the following equalities: f (2r) = f (r + r) = 2; f (2s) = f (s + s) = 4; f (4) = f (2 + 2) = 4r; f (8) = f (4 + 4) = 4s; f (5r) = f (4r + r) = 5; f (r + s) = 3; f (8) = f (5 + 3) = 6r + s. Then 4s = 6r + s, which means that s = 2r. Now we prove by induction that f (nr) = n and f (n) = nr for every n ≥ 4. First we have that f (5) = f (2 + 3) = 3r + s = 5r, so that the statement is true for n = 4 and n = 5. Suppose that it holds for n − 1 and n. Then f (n + 1) = f (n − 1 + 2) = (n−1)r +2r = (n+1)r, and f ((n+1)r) = f ((n−1)r +2r) = (n−1)+2 = n+1. This completes the induction. Since 4r ≥ 4, we have that f (4r) = 4r2 , and also f (4r) = 4. Then r = 1, and consequently f (n) = n for every natural number n. Second solution. f ( f (1) + n + m) = f ( f (1) + f ( f (n) + f (m))) = 1 + f (n) + f (m), so f (n) + f (m) is a function of n + m. Hence f (n + 1) + f (1) = f (n) + f (2) and f (n + 1) − f (n) = f (2) − f (1), implying that f (n) = An + B for some constants A, B. It is easy to check that A = 1, B = 0 is the only possibility. 20. Suppose that An = {1, 2, . . . , n} is partitioned into Bn and Cn , and that neither Bn nor Cn contains 3 distinct numbers one of which is equal to the product of the other two. If n ≥ 96, then the divisors of 96 must be split up. Let w.l.o.g. 2 ∈ Bn . There are four cases. (i) 3 ∈ Bn , 4 ∈ Bn . Then 6, 8, 12 ∈ Cn ⇒ 48, 96 ∈ Bn . A contradiction for 96 = 2 · 48. (ii) 3 ∈ Bn , 4 ∈ Cn . Then 6 ∈ Cn , 24 ∈ Bn , 8, 12, 48 ∈ Cn . A contradiction for 48 = 6 · 8. 518 4 Solutions (iii) 3 ∈ Cn , 4 ∈ Bn . Then 8 ∈ Cn , 24 ∈ Bn , 6, 48 ∈ Cn . A contradiction for 48 = 6 · 8. (iv) 3 ∈ Cn , 4 ∈ Cn . Then 12 ∈ Bn , 6, 24 ∈ Cn . A contradiction for 24 = 4 · 6. If n = 95, there is a very large number of ways of partitioning An . For example, Bn = {1, p, p2 , p3 q2 , p4 q, p2 qr | p, q, r are distinct primes}, Cn = {p3 , p4 , p5 , p6 , pq, p2 q, p3 q, p2 q2 , pqr | p, q, r are distinct primes}. Then B95 = {1, 2, 3, 4, 5, 7, 9, 11, 13, 17, 19, 23, 25, 29, 31, 37, 41, 43, 47, 48, 49, 53, 59, 60, 61, 67, 71, 72, 73, 79, 80, 83, 84, 89, 90}. 21. Let X be the set of all ordered triples a = (a1 , a2 , a3 ) for ai ∈ {0, 1, . . . , 7}. Write a ≺ b if ai ≤ bi for i = 1, 2, 3 and a 6= b. Call a subset Y ⊂ X independent if there are no a, b ∈ Y with a ≺ b. We shall prove that an independent set contains at most 48 elements. For j = 0, 1, . . . , 21 let X j = {(a1 , a2 , a3 ) ∈ X | a1 + a2 + a3 = j}. If x ≺ y and x ∈ X j , y ∈ X j+1 for some j, then we say that y is a successor of x, and x a predecessor of y. Lemma. If A is an m-element subset of X j and j ≤ 10, then there are at least m distinct successors of the elements of A. Proof. For k = 0, 1, 2, 3 let X j,k = {(a1 , a2 , a3 ) ∈ X j | min(a1 , a2 , a3 , 7 − a1 , 7 − a2 , 7 − a3) = k}. It is easy to verify that every element of X j,k has at least two successors in X j+1,k and every element of X j+1,k has at most two predecessors in X j,k . Therefore the number of elements of A ∩ X j,k is not greater than the number of their successors. Since X j is a disjoint union of X j,k , k = 0, 1, 2, 3, the lemma follows. Similarly, elements of an m-element subset of X j , j ≥ 11, have at least m predecessors. Let Y be an independent set, and let p, q be integers such that p < 10 < q. We can transform Y by replacing all the elements of Y ∩ X p with their successors, and all the elements of Y ∩ Xq with their predecessors. After this transformation Y will still be independent, and by the lemma its size will not be reduced. Every independent set can be eventually transformed in this way into a subset of X10 , and X10 has exactly 48 elements. p 22. Set X = ∑i=1 xi and w.l.o.g. assume that X ≥ 0 (if (x1 , . . . , x p ) is a solution, then (−x1 , . . . , −x p ) is a solution too). Since x2 ≥ x for all integers x, it follows that p ∑i=1 x2i ≥ X . If the last inequality is an equality, then all xi ’s are 0 or 1; then, taking that there 4 are a 1’s, the equation becomes 4p + 1 = 4(a + 1) + a−1 , which forces p = 6 and a = 5. p 4 Otherwise, we have X + 1 ≤ ∑i=1 x2i = 4p+1 X 2 + 1, so X ≥ p + 1. Also, by the p Cauchy–Schwarz inequality, X 2 ≤ p ∑i=1 x2i = X ≤ 2p. Thus 1 ≤ X /p ≤ 2. However, 4p 2 4p+1 X + p, so X 2 ≤ 4p2 + p and 4.29 Shortlisted Problems 1988 p ∑ i=1  xi − X p 2 2X p = ∑ x2i − ∑ xi + = ∑ x2i − p p2 X2 519 X2 p = 1− X2 < 1, p(4p + 1) and we deduce that − 1 < xi − X /p < 1 for all i. This finally gives xi ∈ {1, 2}. Suppose there are b 2’s. Then 3b + p = 4(b + p)2 /(4p + 1) + 1, so p = b + 1/(4b − 3), which leads to p = 2, b = 1. Thus there are no solutions for any p 6∈ {2, 6}. Remark. The condition p = n(n + 1), n ≥ 3, was unnecessary in the official solution, too (its only role was to simplify showing that X 6= p − 1). 23. Denote by R the intersection point of lines AQ and BC. We know that BR : RC = c : b and AQ : QR = (b + c) : a. By applying Stewart’s theorem to ∆ PBC and ∆ PAR we obtain a · AP2 + b · BP2 + c ·CP2 = aPA2 + (b + c)PR2 + (b + c)RB · RC = (a + b + c)QP2 + (b + c)RB · RC + (a + b + c)QA · QR. (1) On the other hand, putting P = Q into (1), we get that a · AQ2 + b · BQ2 + c · CQ2 = (b + c)RB · RC + (a + b + c)QA · QR, and the required statement follows. Second solution. At vertices A, B,C place weights equal to a, b, c in some units respectively, so that Q is the center of gravity of the system. The left side of the equality to be proved is in fact the moment of inertia of the system about the axis through P and perpendicular to the plane ABC. On the other side, the right side expresses the same, due to the parallel axes theorem. Alternative approach. Analytical geometry. The fact that all the variable segments appear squared usually implies that this is a good approach. Assign coordinates A(xa , ya ), B(xb , yb ), C(xc , yc ), and P(x, y), use that (a + b + c)Q = aA + bB + cC, and calculate. Alternatively, differentiate f (x, y) = a · AP2 + b · BP2 + c ·CP2 − (a + b + c)QP2 and show that it is constant. 24. The first condition means in fact that ak − ak+1 is decreasing. In particular, if ak − ak+1 = −δ < 0, then ak − ak+m = (ak − ak+1 ) + · · · + (ak+m−1 − ak+m ) < −mδ , which implies that ak+m > ak + mδ , and consequently ak+m > 1 for large enough m, a contradiction. Thus ak − ak+1 ≥ 0 for all k. Suppose that ak − ak+1 > 2/k2 . Then for all i < k, ai − ai+1 > 2/k2 , so that ai − ak+1 > 2(k + 1 − i)/k2 , i.e., ai > 2(k + 1 − i)/k2 , i = 1, 2, . . . , k. But this implies a1 + a2 + · · · + ak > 2/k2 + 4/k2 + · · · + 2k/k2 = k(k + 1)/k2 , which is impossible. Therefore ak − ak+1 ≤ 2/k2 for all k. 25. Observe that 1001 = 7 · 143, i.e., 103 = −1 + 7a, a = 143. Then by the binomial theorem, 1021 = (−1 + 7a)7 = −1 + 72b for some integer b, so that we also have 520 4 Solutions 9 1021n ≡ −1 (mod 49) for any odd integer n > 0. Hence N = 49 (1021n + 1) is an 2 3 21n 21n integer of 21n digits, and N(10 + 1) = 7 (10 + 1) is a double number that is a perfect square. 26. In the sequel, a1 a2 . . . aα will be used to representation a number whose binary digits are a1 , . . . , aα . We will show by induction that if n = ck ck−1 . . . c0 = ∑ki=0 ci 2i is the binary representation of n (ci ∈ {0, 1}), then f (n) = c0 c1 . . . ck = ∑ki=0 ci 2k−i is the number whose binary representation is the palindrome of the binary representation of n. This evidently holds for n ∈ {1, 2, 3}. Let us assume that the claim holds for all numbers up to n − 1 and show it holds for n = ck ck−1 . . . c0 . We observe three cases: (i) c0 = 0 ⇒ n = 2m ⇒ f (n) = f (m) = 0c1 . . . ck = c0 c1 . . . ck . (ii) c0 = 1, c1 = 0 ⇒ n = 4m + 1 ⇒ f (n) = 2 f (2m + 1) − f (m) = 2 · 1c2 . . . ck − c2 . . . ck = 2k + 2 · c2 . . . ck − c2 . . . ck = 10c2 . . . ck = c0 c1 . . . ck . (iii) c0 = 1, c1 = 1 ⇒ n = 4m+3 ⇒ f (n) = 3 f (2m+1)−2 f (m) = 3·1c2 . . . ck − 2 · c2 . . . ck = 2k + 2k−1 + 3 · c2 . . . ck − 2 · c2 . . . ck = 11c2 . . . ck = c0 c1 . . . ck . We thus have to find the number of palindromes in binary representation smaller than 1988 = 11111000100. We note that for all m ∈ N the numbers of 2m- and (2m − 1)-digit binary palindromes are both equal to 2m−1 . We also note that 11111011111 and 11111111111 are the only 11-digit palindromes larger than 1988. Hence we count all palindromes of up to 11 digits and exclude the largest two. The number of n ≤ 1988 such that f (n) = n is thus equal to 1 + 1 + 2 + 2 + 4 + 4 + 8 + 8 + 16 + 16 + 32 − 2 = 92. 27. Consider a Cartesian system with the x-axis on the line BC and origin at the foot of the perpendicular from A to BC, so that A lies on the y-axis. Let A be (0, α ), B(−β , 0), C(γ , 0), where α , β , γ > 0 (because ABC is acute-angled). Then tan B = α , β tanC = α γ and tan A = − tan(B +C) = α (β + γ ) ; α2 − β γ here tan A > 0, so α 2 > β γ . Let L have equation x cos θ + y sin θ + p = 0. Then u2 tan A + v2 tan B + w2 tanC α (β + γ ) α α = 2 (α sin θ + p)2 + (−β cos θ + p)2 + (γ cos θ + p)2 α −βγ β γ α ( β + γ ) α (β + γ ) 2 = (α 2 sin2 θ + 2α p sin θ + p2 ) 2 + α (β + γ ) cos2 θ + p α −βγ βγ α (β + γ ) = (α 2 p2 + 2α pβ γ sin θ + α 2 β γ sin2 θ + β γ (α 2 − β γ ) cos2 θ ) β γ (α 2 − β γ )  α (β + γ )  = (α p + β γ sin θ )2 + β γ (α 2 − β γ ) ≥ α (β + γ ) = 2∆ , 2 β γ (α − β γ ) with equality when α p + β γ sin θ = 0, i.e., if and only if L passes through (0, β γ /α ), which is the orthocenter of the triangle. 4.29 Shortlisted Problems 1988 521 28. The sequence is uniquely determined by the conditions, and a1 = 2, a2 = 7, a3 = 25, a4 = 89, a5 = 317, . . . ; it satisfies an = 3an−1 + 2an−2 for n = 3, 4, 5. We show that the sequence bn given by b1 = 2, b2 = 7, bn = 3bn−1 + 2bn−2 has the same inequality property, i.e., that bn = an : bn+1 bn−1 − b2n = (3bn + 2bn−1)bn−1 − bn (3bn−1 + 2bn−2 ) = −2(bn bn−2 − b2n−1 ) for n > 2 gives that bn+1 bn−1 − b2n = (−2)n−2 for all n ≥ 2. But then bn+1 − b2n 2n−2 1 = < , bn−1 bn−1 2 since it is easily shown that bn−1 > 2n−1 for all n. It is obvious that an = bn are odd for n > 1. 29. Let the first train start from Signal 1 at time 0, and let t j be the time it takes for the jth train in the series to travel from one signal to the next. By induction on k, we show that Train k arrives at signal n at time sk + (n − 2)mk , where sk = t1 + · · · + tk and mk = max j=1,...,k t j . For k = 1 the statement is clear. We now suppose that it is true for k trains and for every n, and add a (k + 1)th train behind the others at Signal 1. There are two cases to consider: (i) tk+1 ≥ mk , i.e., mk+1 = tk+1 . Then Train k + 1 leaves Signal 1 when all the others reach Signal 2, which by the induction happens at time sk . Since by the induction hypothesis Train k arrives at Signal i + 1 at time sk + (i − 1)mk ≤ sk + (i − 1)tk+1 , Train k + 1 is never forced to stop. The journey finishes at time sk + (n − 1)tk+1 = sk+1 + (n − 2)mk+1. (ii) tk+1 < mk , i.e., mk+1 = mk . Train k + 1 leaves Signal 1 at time sk , and reaches Signal 2 at time sk + tk+1 , but must wait there until all the other trains get to Signal 3, i.e., until time sk + mk (by the induction hypothesis). So it reaches Signal 3 only at time sk + mk + tk+1 . Similarly, it gets to Signal 4 at time sk + 2mk + tk+1 , etc. Thus the entire schedule finishes at time sk + (n − 2)mk + tk+1 = sk+1 + (n − 2)mk+1. 30. Let ∆1 , s1 , r′ denote the area, semiperimeter, and inradius of triangle ABM, ∆2 , s2 , r′ the same quantities for triangle MBC, and ∆ , s, r those for △ABC. Also, let P′ and Q′ be the points of tangency of the incircle of △ABM with the side AB and of the incircle of △MBC with the side BC, respectively, and let P, Q be the points of tangency of the incircle of △ABC with the sides AB, BC. We have ∆1 = s1 r′ , ∆2 = s2 r′ , ∆ = sr, so that sr = (s1 + s2 )r′ . Then s1 + s2 = s + BM ⇒ r′ s = . r s + BM (1) On the other hand, from the similarity of the triangles it follows that AP′ /AP = CQ′ /CQ = r′ /r. By a well-known formula we find that AP = s − BC, CQ = s − AB, AP′ = s1 − BM, CQ′ = s2 − BM, and therefore deduce that 522 4 Solutions r′ s1 − BM s2 − BM r′ s1 + s2 − 2BM s − BM = = ⇒ = = . r s − BC s − AB r 2s − AB − BC AC (2) It follows from (1) and (2) that (s − BM)/AC = s/(s + BM), giving us s2 − BM2 = s · AC. Finally, BM2 = s(s − AC) = s · BP = s · r cot B B = ∆ cot . 2 2 31. Denote the number of participants by 2n, and assign to each seat one of the numbers 1, 2, . . . , 2n. Let the participant who was sitting at the seat k before the break move to seat π (k). It suffices to prove that for every permutation π of the set {1, 2, . . .,2n}, there exist distinct i, j such that π (i) − π ( j) = ±(i − j), the differences being calculated modulo 2n. If there are distinct i and j such that π (i) − i = π ( j) − j modulo 2n, then we are done. Suppose that all the differences π (i) − i are distinct modulo 2n. Then they take values 0, 1, . . . , 2n − 1 in some order, and consequently 2n ∑ (π (i) − i) = 0 + 1 + · · ·+ (2n − 1) ≡ n(2n − 1) (mod 2n). i=1 On the other hand, ∑2n i=1 (π (i) − i) = ∑ π (i) − ∑ i = 0, which is a contradiction because n(2n − 1) is not divisible by 2n. Remark. For an odd number of participants, the statement is false. For example, the permutation (a, 2a, . . . , (2n + 1)a) of (1, 2, . . . , 2n + 1) modulo 2n + 1 does not satisfy the statement when gcd(a2 − 1, 2n + 1) = 1. Check that such an a always exists. 4.30 Shortlisted Problems 1989 523 4.30 Solutions to the Shortlisted Problems of IMO 1989 1. Let I denote the intersection of the three internal bisectors. Then IA1 = A1 A0 . B0 One way to prove this is to realA ize that the circumcircle of △ABC C0 B1 is the nine-point circle of △A0 B0C0 , so it bisects IA0 , since I is the orC1 thocenter of A0 B0C0 . Another way I is through noting that IA1 = A1 B. This is a consequence of ∠A1 IB = B C A1 ∠IBA1 = (∠A + ∠B)/2, and A1 B = A1 A0 which follows from ∠A1 A0 B = A0 ∠A1 BA0 = 90◦ − ∠IBA1 . Hence, we obtain SIA1 B = SA0 A1 B . Repeating this argument for the six triangles that have a vertex at I and adding them up gives us SA0 B0C0 = 2SAC1 BA1CB1 . To prove SAC1 BA1CB1 ≥ 2SABC , draw the three altitudes in triangle ABC intersecting in H. Let X, Y , and Z be the symmetric points of H with respect the sides BC, CA, and AB, respectively. Then X ,Y, Z are points on the circumcircle of △ABC (because ∠BXC = ∠BHC = 180◦ − ∠A). Since A1 is the midpoint of the arc BC, we have SBA1C ≥ SBXC . Hence SAC1 BA1CB1 ≥ SAZBXCY = 2(SBHC + SCHA + SAHB ) = 2SABC . 2. Let the carpet have width x, length y. Suppose that the carpet EFGH lies in a room ABCD, E being on AB, F on BC, G on CD, and H on DA. Then △AEH ≡ △CGF ∼ △BFE ≡ △DHG. Let yx = k, AE = a and AH = b. In that case BE = kb and DH = ka. Thus a + kb = 50, ka + b = 55, whence a = 55k−50 and b = 50k−55 . Hence x2 = k2 −1 k2 −1 a2 + b2 = 5525k2 −11000k+5525 , (k2 −1)2 i.e., x2 (k2 − 1)2 = 5525k2 − 11000k + 5525. Similarly, from the equations for the second storeroom, we get x2 (k2 − 1)2 = 4469k2 − 8360k + 4469. Combining the two equations, we get 5525k2 − 11000k + 5525 = 4469k2 − 8360k + 4469, which implies k = 2 or 1/2. Without loss of generality we have √ y = 2x and a+ 2b = 50, 2a +b = 55; hence a = 20, b = 15, x = 152 + 202 = 25, and y = 50. We have thus shown that the carpet is 25 feet by 50 feet. 3. Let the carpet have width x, length y. Let the length of the storerooms be q. Let y/x = k. Then, as in the previous problem, (kq − 50)2 + (50k − q)2 = (kq − 38)2 + (38k − q)2, i.e., kq = 22(k2 + 1). (1)  2  2 Also, as before, x2 = kq−50 + 50k−q , i.e., k2 −1 k2 −1 524 4 Solutions x2 (q2 − 1)2 = (k2 + 1)(q2 − 1900), (2) which, together with (1), yields x2 k2 (k2 − 1)2 = (k2 + 1)(484k4 − 932k2 + 484). Since k is rational, let k = c/d, where c and d are integers with gcd(c, d) = 1. Then we obtain x2 c2 (c2 − d 2 )2 = c2 (484c4 − 448c2d 2 − 448d 4) + 484d 6. We thus have c2 | 484d 6, but since (c, d) = 1, we have c2 | 484 ⇒ c | 22. 1 1 2 11 Analogously, d | 22; thus k = 1, 2, 11, 22, 12 , 11 , 22 , 11 , 2 . Since reciprocals lead  to the same solution, we need only consider k ∈ 1, 2, 11, 22, 11 2 , yielding q = 44, 55, 244, 485, 125, respectively. We can test these values by substituting them into (2). Only k = 2 gives us an integer solution, namely x = 25, y = 50. 4. First we note that for every integer k > 0 and prime number p, pk doesn’t divide k!. This follows from the fact that the highest exponent r of p for which pr |k! is     k k k k k r= + 2 + ··· < + 2 + ··· = < k. p p p p p−1 Now suppose that α is a rational root of the given equation. Then αn + n! n! n! α n−1 + · · · + α 2 + α + n! = 0, (n − 1)! 2! 1! (1) from which we can conclude that α must be an integer, not equal to ±1. Let p be a prime divisor of n and let r be the highest exponent of p for which pr |n!. Then p | α . Since pk |α k and pk ∤ k!, we obtain that pr+1 | n!α k /k! for k = 1, 2, . . . , n. But then it follows from (1) that pr+1 | n!, a contradiction. 5. According to the Cauchy–Schwarz inequality, n ∑ ai i=1 !2 n ≤ ∑ i=1 a2i ! n ∑1 i=1 2 ! n =n ∑ i=1 a2i ! . Since r1 + · · · + rn = −n, applying this inequality we obtain r12 + . . . + rn2 ≥ n, and applying it three more times, we obtain r116 + · · · + rn16 ≥ n, with equality if and only if r1 = r2 = . . . = rn = −1 and p(x) = (x + 1)n . 6. Let us denote the measures of the inner angles of the triangle ABC by α , β , γ . Then P = r2 (sin 2α + sin 2β + sin 2γ )/2. Since the inner angles of the triangle A′ B′C′ are (β + γ )/2, (γ + α )/2, (α + β )/2, we also have Q = r2 [sin (β + γ ) + 4.30 Shortlisted Problems 1989 525 sin (γ + α ) + sin (α + β )]/2. Applying the AM–GM mean inequality, we now obtain 16 6 r (sin (β + γ ) + sin (γ + α ) + sin (α + β ))3 8 ≥ 54r6 sin (β + γ ) sin (γ + α )sin (α + β ) = 27r6 [cos(α − β ) − cos(α + β + 2γ )] sin(α + β ) 16Q3 = = 27r6 [cos(α − β ) + cos γ ] sin(α + β ) 27 6 = r [sin(α + β + γ ) + sin(α + β − γ ) + sin2α + sin 2β ] 2 27 6 = r [sin(2γ ) + sin 2α + sin 2β ] = 27r4 P. 2 This completes the proof. 7. Assume that P1 and P2 are points inside E, and that the line P1 P2 intersects the perimeter of E at Q1 and Q2 . If we prove the statement for Q1 and Q2 , we are done, since these arcs can be mapped homothetically to join P1 and P2 . Let V1 ,V2 be two vertices of E. Then applying two homotheties to the inscribed circle of E one can find two arcs (one of them may be a side of E) joining these two points, both tangent to the sides of E that meet at V1 and V2 . If A is any point of the side V2V3 , two homotheties with center V1 take the arcs joining V1 to V2 and V3 into arcs joining V1 to A; their angle of incidence at A remains (1 − 2/n) π . Next, for two arbitrary points Q1 and Q2 on two different sides V1V2 and V3V4 , we join V1 and V2 to Q2 with pairs of arcs that meet at Q2 and have an angle of incidence (1 − 2/n) π . The two arcs that meet the line Q1 Q2 again outside E meet at Q2 at an angle greater than or equal to (1 − 2/n) π . Two homotheties with center Q2 carry these arcs to ones meeting also at Q1 with the same angle of incidence. 8. Let A, B,C, D denote the vertices of R. We consider the set S of all points E of the plane that are vertices of at least one rectangle, and its subset S ′ consisting of those points in S that have both coordinates integral in the orthonormal coordinate system with point A as the origin and lines AB, AD as axes. First, to each E ∈ S we can assign an integer nE as the number of rectangles Ri with one vertex at E. It is easy to check that nE = 1 if E is one of the vertices A, B,C, D; in all other cases nE is either 2 or 4. Furthermore, for each rectangle Ri we define f (Ri ) as the number of vertices of Ri that belong to S ′ . Since every Ri has at least one side of integer length, f (Ri ) can take only values 0, 2, or 4. Therefore we have n ∑ f (Ri ) ≡ 0 (mod 2). i=1 On the other hand, ∑ni=1 f (Ri ) is equal to ∑E∈S ′ nE , implying that ∑ E∈S ′ nE ≡ 0 (mod 2). 526 4 Solutions However, since nA = 1, at least one other nE , where E ∈ S ′ , must be odd, and that can happen only for E being B, C, or D. We conclude that at least one of the sides of R has integral length. Second solution. Consider the coordinate system introduced above. If D is a rectangle whose sides are parallel to the axes of the system, it is easy to prove that Z sin(2π (x + α ))sin(2π (y + β ))dxdy = 0, for all α , β ∈ R D if and only if at least one side of D has integral length. This holds for all Ri ’s therefore, adding up these equalities for each α and β we get Z R sin (2π (x + α ))sin (2π (y + β )) dx dy = 0. Thus, R also has a side of integral length. √ √ √ √ √ √ 9. From an+1 + bn+1 3 2 + cn+1 3 4 = (an + bn 3 2 + cn 3 4)(1 + 4 3 2 − 4 3 4) we obtain an+1 = an − 8bn + 8cn. Since a0 = 1, an is odd for all n. For an integer k > 0, we can write k = 2l k′ , k′ being odd and l a nonnegative integer. Let us set v(k) = l, and define βn = v(bn ), γn = v(cn ). We prove the following lemmas: Lemma 1. For every integer p ≥ 0, b2 p and c2 p are nonzero, and β2 p = γ2 p = p + 2. Proof. By induction on p. For p = 0, b1 = 4 and c1 = −4, so the assertion is true. Suppose that it holds for p. Then √ √ √ √ p+1 3 3 3 3 (1 + 4 2 − 4 4)2 = (a + 2 p+2(b′ 2 + c′ 4))2 with a, b′ , and c′ odd. √ √ p+1 √ √ Then we easily obtain that (1 + 4 3 2 −4 3 4)2 = A+ 2 p+3 (B 3 2 +C 3 4), where A, B = ab′ + 2 p+1E,C = ac′ + 2 p+1 F are odd. Therefore Lemma 1 holds for p + 1. Lemma 2. Suppose that for integers n, m ≥ 0, βn = γn = λ > βm = γm = µ . Then bn+m , cn+m are nonzero √ and β√n+m = γn+m = µ .√ √ Proof. Calculating (a′ + 2λ (b′ 3 2 + c′ 3 4))(a′′ + 2 µ (b′′ 3 2 + c′′ 3 √ 4)), with a′ , √ 3 3 ′ ′ ′′ ′′ ′′ µ b , c , a , b , c odd, we easily obtain the product A + 2 (B 2 + C 4), where A, B = a′ b′′ + 2λ −µ E, and C = a′ c′′ + 2λ −µ F are odd, which proves Lemma 2. Since every integer n > 0 can be written as n = 2 pr + · · · + 2 p1 , with 0 ≤ p1 < · · · < pr , from Lemmas 1 and 2 it follows that cn is nonzero, and that γn = p1 + 2. Remark. b1989 and c1989 are divisible by 4, but not by 8. 10. Plugging in wz + a instead of z into the functional equation, we obtain f (wz + a) + f (w2 z + wa + a) = g(wz + a). (1) By repeating this process, this time in (1), we get f (w2 z + wa + a) + f (z) = g(w2 z + wa + a). (2) 4.30 Shortlisted Problems 1989 527 Solving the system of linear equations (1), (2) and the original functional equation, we easily get g(z) + g(w2 z + wa + a) − g(wz + a) . 2 This function thus uniquely satisfies the original functional equation. f (z) = 11. Call a binary sequence S of length n repeating if for some d | n, d > 1, S can be split into d identical blocks. Let xn be the number of nonrepeating binary sequences of length n. The total number of binary sequences of length n is obviously 2n . Any sequence of length n can be produced by repeating its unique longest nonrepeating initial block according to need. Hence, we obtain the recursion relation ∑d|n xd = 2n . This, along with x1 = 2, gives us an = xn for all n. We now have that the sequences counted by xn can be grouped into groups of n, the sequences in the same group being cyclic shifts of each other. Hence, n | x n = an . 12. Assume that each car starts with a unique ranking number. Suppose that while turning back at a meeting point two cars always exchanged their ranking numbers. We can observe that ranking numbers move at a constant speed and direction. One hour later, after several exchanges, each starting point will be occupied by a car of the same ranking number and proceeding in the same direction as the one that started from there one hour ago. We now give the cars back their original ranking numbers. Since the sequence of the cars along the track cannot be changed, the only possibility is that the original situation has been rotated, maybe onto itself. Hence for some d | n, after d hours each car will be at its starting position and orientation. 13. Let us construct the circles σ1 with center A and radius R1 = AD, σ2 with center B and radius R2 = BC, and σ3 with center P and radius x. The points C and D lie on σ2 and σ1 respectively, and CD is tangent to σ3 . From this it is plain that the greatest value of x occurs when CD is also tangent to σ1 and σ2 . We shall show that in this case the required inequality is really an equality, i.e., that √1 = √1 + √1 . Then the inequality will immediately follow. x BC AD Denote the point pof tangency of CD with σ3 by√M. By the Pythagorean theorem we have CD =√ (R1 + R√2 )2 − (R1 − R2 )2 = 2 R1 R2 . On the other hand, CD = CM + MD = 2 R2 x + 2 R1 x. Hence, we obtain √1x = √1R + √1R . 1 2 14. Lemma 1. In a quadrilateral ABCD circumscribed about a circle, with points of tangency P, Q, R, S on DA, AB, BC,CD respectively, the lines AC, BD, PR, QS concur. Proof. Follows immediately, for example, from Brianchon’s theorem. Lemma 2. Let a variable chord XY of a circle C(I, r) subtend a right angle at a fixed point Z within the circle. Then the locus of the midpoint P of XY is p a circle whose center is at the midpoint M of IZ and whose radius is r2 /2 − IZ 2/4. 528 4 Solutions −→ − → − → − → − → − → Proof. From ∠XZY = 90◦ follows ZX · ZY = (IX − IZ) · (IY − IZ) = 0. Therefore, 1 − −→2 − → → − → − → − → MP = (MI + IP)2 = (−IZ + IX + IY )2 4 1 − → → − − → → − 2 2 = (IX + IY − IZ 2 + 2(IX − IZ) · (IY − IZ)) 4 1 1 = r2 − IZ 2 . 2 4 Lemma 3. Using notation as in Lemma 1, if ABCD is cyclic, PR is perpendicular to QS. Proof. Consider the inversion in C(I, r), mapping A to A′ etc. (P, Q, R, S are fixed). As is easily seen, A′ , B′ ,C′ , D′ will lie at the midpoints of PQ, QR, RS, SP, respectively. A′ B′C′ D′ is a parallelogram, but also cyclic, since inversion preserves circles; thus it must be a rectangle, and so PR ⊥ QS. Now we return to the main result. Let I and O be the incenter and circumcenter, Z the intersection of the diagonals, and P, Q, R, S, A′ , B′ ,C′ , D′ points as defined in Lemmas 1 and 3. From Lemma 3, the chords PQ, QR, RS, SP subtend 90◦ at Z. Therefore by Lemma 2 the points A′ , B′ ,C′ , D′ lie on a circle whose center is the midpoint Y of IZ. Since this circle is the image of the circle ABCD under the considered inversion (centered at I), it follows that I, O,Y are collinear, and hence so are I, O, Z. Remark. This is the famous Newton’s theorem for bicentric quadrilaterals. √ √ 15. By Cauchy’s inequality, 44 < 1989 < a + b + c + d ≤ 2 · 1989 < 90. Since m2 = a + b + c + d is of the same parity as a2 + b2 + c2 + d 2 = 1989, m2 is either 49 or 81. Let d = max{a, b, c, d}. Suppose that m2 = 49. Then (49− d)2 = (a +b + c)2 > a2 + b2 + c2 = 1989− d 2, 2 and so pd − 49d + 206 > 0. This inequality does not hold for 5 ≤ d ≤ 44. Since d ≥ 1989/4 > 22, d must be at least 45, which is impossible because 452 > 1989. Thus we must √ have m2 = 81 and m = 9. Now, 4d > 81 implies d ≥ 21. On the other hand, d < 1989, and hence d = 25 or d = 36. Suppose that d = 25 and put a = 25 − p, b = 25 − q, c = 25 − r with p, q, r ≥ 0. From a + b + c = 56 it follows that p + q + r = 19, which, together with (25 − p)2 + (25 − q)2 + (25 − r)2 = 1364, gives us p2 + q2 + r2 = 439 > 361 = (p + q + r)2, a contradiction. Therefore d = 36 and n = 6. Remark. A little more calculation yields the unique solution a = 12, b = 15, c = 18, d = 36. 16. Define Sk = ∑ki=0 ai (k = 0, 1, . . . , n) and S−1 = 0. We note that Sn−1 = Sn . Hence 4.30 Shortlisted Problems 1989 Sn = n−1 n−1 n−1 k=0 k=0 i=k ∑ ak = nc + ∑ ∑ ai−k (ai + ai+1) n−1 i n−1 i i=0 k=0 i=0 k=0 = nc + ∑ 529 ∑ ai−k (ai + ai+1) = nc + ∑ (ai + ai+1) ∑ ai−k n−1 = nc + ∑ (Si+1 − Si−1 )Si = nc + Sn2, i=0 Sn2 − Sn + nc = 0. i.e., Since Sn is real, the discriminant of the quadratic equation 1 must be positive, and hence c ≤ 4n . 17. A figure consisting of 9 lines is shown below. b HH  b Hb b b HH  @ @HH  HHb @ b  Now we show that 8 lines are not sufficient. Assume the opposite. By the pigeonhole principle, there is a vertex, say A, that is joined to at most 2 other vertices. Let B,C, D, E denote the vertices to which A is not joined, and F, G the other two vertices. Then any two vertices of B,C, D, E must be mutually joined for an edge to exist within the triangle these two points form with A. This accounts for 6 segments. Since only two segments remain, among A, F, and G at least two are not joined. Taking these two and one of B,C, D, E that is not joined to any of them (it obviously exists), we get a triple of points, no two of which are joined; a contradiction. Second solution. Since (i) is equivalent to the fact that no three points make a “blank triangle,” by Turan’s theorem the number of “blank edges” cannot exceed [72 /4] = 12, leaving at least 7 · 6/2 − 12 = 9 segments. For general n, the answer is [(n − 1)/2]2 . 18. Consider the triangle MAi Mi . Obviously, the point Mi is the image of Ai under α /2−90◦ 2 sin(α /2) the composition C of rotation RM and homothety HM . Therefore, the polygon M1 M2 . . . Mn is obtained as the image of A1 A2 . . . An under the rotational homothety C with coefficient 2 sin(α /2). Therefore SM1 M2 ...Mn = 4 sin2 (α /2)· S. 19. Let us color the board in a chessboard fashion. Denote by Sb and Sw respectively the sum of numbers in the black and in the white squares. It is clear that every allowed move leaves the difference Sb − Sw unchanged. Therefore a necessary condition for annulling all the numbers is Sb = Sw . We now show it is sufficient. Assuming Sb = Sw let us observe a triple of (different) cells a, b, c with respective values xa , xb , xc where a and c are both adjacent to b. We first prove that we can reduce xa to be 0 if xa > 0. If xa ≤ xb , we subtract xa from both a and b. If xa > xb , we add xa − xb to b and c and proceed as in the previous case. Applying the reduction in sequence, along the entire board, we 530 4 Solutions reduce all cells except two neighboring cells to be 0. Since Sb = Sw is invariant, the two cells must have equal values and we can thus reduce them both to 0. √ 20. Suppose k ≥ 1/2 + 2n. Consider a point P in S. There are  at least k points in S having all the same distance to P, so there are at least 2k pairs of points A, B  with AP = BP. Since this is true for every point P ∈ S, there are at least n 2k triples of points (A, B, P) for which AP = BP holds. However,      √ k k(k − 1) n √ 1 1 n =n ≥ 2n + 2n − 2 2 2 2 2     n 1 n = 2n − > n(n − 1) = 2 . 2 4 2  Since n2 is the number of all possible pairs (A, B) with A, B ∈ S, there must exist a pair of points A, B with more than two points Pi such that APi = BPi . These points Pi are collinear (they lie on the perpendicular bisector of AB), contradicting condition (i). 21. In order to obtain a triangle as the intersection we must have three points P, Q, R on three sides of the tetrahedron passing through one vertex, say T . It is clear that we may suppose w.l.o.g. that P is a vertex, and Q and R lie on the edges T P1 and T P2 (P1 , P2 are vertices) or on their extensions respectively. Suppose −→ −−→ −→ −−→ that T Q = λ T P1 and T R = µ T P2 , where λ , µ > 0. Then −→ − → PQ · PR (λ − 1)(µ − 1) + 1 p cos ∠QPR = = √ . PQ · PR 2 λ 2 − λ + 1 µ 2 − µ + 1 In order to obtain an √ obtuse angle (with cos < 0) p we must choose µ < 1 and µ λ > 2− λ 2 − λ + 1 > λ − 1 and µ 2 − µ + 1 > 1 − µ , we get 1− µ > 1. Since that for (λ − 1)(µ − 1) + 1 < 0, cos ∠QPR > 1 − (1 − µ )(λ − 1) 1 >− ; 2(1 − µ )(λ − 1) 2 hence ∠QPR < 120◦ . Remark. After obtaining the formula for cos∠QPR, the official solution was as follows: For fixed µ0 < 1 and λ > 1, cos ∠QPR is a decreasing function of λ : indeed, ∂ cos ∠QPR µ − (3 − µ )λ = < 0. 2 ∂λ 4(λ − λ + 1)3/2(µ 2 − µ + 1)1/2 Similarly, for a fixed, sufficiently large λ0 , cos ∠QPR is decreasing for µ decreasing to 0. Since limλ →0,µ →0+ cos ∠QPR = −1/2, we conclude that ∠QPR < 120◦ . 22. The statement remains valid if 17 is replaced by any divisor k of 1989 = 32 · 13 · 17, 1 < k < 1989, so let k beSone such divisor. The set {1, 2, . . . , 1989} can be partitioned as {1, 2, . . ., 3k} ∪ Lj=1 {(2 j + 1)k + 1, (2 j + 1)k + 2, . . ., (2 j + 1)k + 2k} = X ∪Y1 ∪ · · · ∪YL , where L = (1989 − 3k)/2k. The required statement will be an obvious consequence of the following two claims. 4.30 Shortlisted Problems 1989 531 Claim 1. The set X = {1, 2, . . . , 3k} can be partitioned into k disjoint subsets, each having 3 elements and the same sum. Proof. Since k is odd, let t = k − 1/2 and X = {1, 2, . . . , 6t + 3}. For l = 1, 2, . . . ,t, define X2l−1 = {l, 3t + 1 + l, 6t + 5 − 2l}, X2l = {t + 1 + l, 2t + 1 + l, 6t + 4 − 2l} X2t+1 = Xk = {t + 1, 4t + 2, 4t + 3}. It is easily seen that these three subsets are disjoint and that the sum of elements in each set is 9t + 6. Claim 2. Each Y j = {(2 j + 1)k + 1, . . ., (2 j + 1)k + 2k} can be partitioned into k disjoint subsets, each having 2 elements and the same sum. Proof. The obvious partitioning works: Y j = {(2 j + 1)k + 1, (2 j + 1)k + 2k} ∪ · · · ∪ {(2 j + 1)k + k, (2 j + 1)k + (k + 1)}. 23. Two numbers x, y ∈ {1, . . . , 2n} will be called twins if |x − y| = n. Then the set {1, . . . , 2n} splits into n pairs of twins. A permutation (x1 , . . . , x2n ) of this set is said to be of type Tk if |xi − xi+1 | = n holds for exactly k indices i (thus a permutation of type T0 contains no pairs of neighboring twins). Denote by Fk (n) the number of Tk -type permutations of {1, . . . , 2n}. Let (x1 , . . . , x2n ) be a permutation of type T0 . Removing x2n and its twin, we obtain a permutation of 2n − 2 elements consisting of n − 1 pairs of twins. This new permutation is of one of the following types: (i) type T0 : x2n can take 2n values, and its twin can take any of 2n − 2 positions; (ii) type T1 : x2n can take any one of 2n values, but its twin must be placed to separate the unique pair of neighboring twins in the new permutation. The recurrence formula follows: F0 (n) = 2n[(2n − 2)F0(n − 1) + F1(n − 1)]. (1) Now let (x1 , . . . , x2n ) be a permutation of type T1 , and let (x j , x j+1 ) be the unique neighboring twin pair. Similarly, on removing this pair we get a permutation of 2n − 2 elements, either of type T0 or of type T1 . The pair (x j , x j+1 ) is chosen out of n twin pairs and can be arranged in two ways. Also, in the first case it can be placed anywhere (2n − 1 possible positions), but in the second case it must be placed to separate the unique pair of neighboring twins. Hence, F1 (n) = 2n[(2n − 1)F0(n − 1) + F1(n − 1)] = F0 (n) + 2nF0(n − 1). (2) This implies that F0 (n) < F1 (n). Therefore the permutations with at least one neighboring twin pair are more numerous than those with no such pairs. Remark 1. As in the official solution, formulas (1) and (2) together give for F0 the recurrence F0 (n) = 2n[(2n − 1)F0(n − 1) + (2n − 2)F0(n − 2)]. 532 4 Solutions For the ratio pn = F0 (n)/(2n)!, simple algebraic manipulation yields pn = pn−2 pn−1 + (2n−3)(2n−1) . Since p1 = 0, we get pn < pn−1 + 1 1 1 1 = pn−1 + − < ··· < . (2n − 3)(2n − 1) 2(2n − 3) 2(2n − 1) 2 Remark 2. Using the inclusion–exclusion principle, the following formula can be obtained:       0 n 1 n 2 n F0 (n) = 2 (2n)! − 2 (2n − 1)! + 2 (2n − 2)! − · · · 0 1 2   n · · · + (−1)n−12n n!. n One consequence is that in fact, limn→∞ pn = 1/e. Second solution. Let f : T0 → T1 be the mapping defined as follows: if (x1 , x2 , . . . , x2n ) ∈ T0 and xk , k > 2, is the twin of x1 , then f (x1 , x2 , . . . , x2n ) = (x2 , . . . , xk−1 , x1 , xk , . . . , x2n ). The mapping f is injective, but not surjective. Thus F0 (n) < F1 (n). 24. Instead of Euclidean distance, we will use the angles ∠Ai OA j , O denoting the center of the sphere. Let {A1 , . . . , A5 } be any set for which mini6= j ∠Ai OA j ≥ π /2 (such a set exists: take for example five vertices of an octagon). We claim that two of the Ai ’s must be antipodes, thus implying that mini6= j ∠Ai OA j is exactly √ equal to π /2, and consequently that mini6= j Ai A j = 2. Suppose no two of the five points are antipodes. Visualize A5 as the south pole. Then A1 , . . . , A4 lie in the northern hemisphere, including the equator (but excluding the north pole). No two of A1 , . . . , A4 can lie in the interior of a quarter of this hemisphere, which means that any two of them differ in longitude by at least π /2. Hence, they are situated on four meridians that partition the sphere into quarters. Finally, if one of them does not lie on the equator, its two neighbors must. Hence, in any case there will exist an antipodal pair, giving us a contradiction. 25. We may assume w.l.o.g. that a > 0 (because a, b < 0 is impossible, and a, b 6= 0 from the condition of the problem). Let (x0 , y0 , z0 , w0 ) 6= (0, 0, 0, 0) be a solution of x2 − ay2 − bz2 + abw2 . Then x20 − ay20 = b(z20 − aw20 ). Multiplying both sides by (z20 − aw20 ), we get (x20 − ay20 )(z20 − aw20) − b(z20 − aw20)2 = 0 ⇔ (x0 z0 − ay0 w0 )2 − a(y0 z0 − x0 w0 )2 − b(z20 − aw20 )2 = 0. Hence, for x1 = x0 z0 − ay0 w0 , y1 = y0 z0 − x0 w0 , z1 = z20 − aw20 , we have 4.30 Shortlisted Problems 1989 533 x21 − ay21 − bz21 = 0. If (x1 , y1 , z1 ) is the trivial solution, then z1 = 0 implies z0 = w0 = 0 and similarly x0 = y0 = 0 because a is not a perfect square. This contradicts the initial assumption. 26. By the Cauchy–Schwarz inequality, n ∑ xi i=1 Since ∑ni=1 xi = a − x0 and ∑ni=1 x2i !2 n ≤ n ∑ x2i . i=1 = b − x20 , we have (a − x0 )2 ≤ n(b − x20), i.e., (n + 1)x20 − 2ax0 + (a2 − nb) ≤ 0.   The discriminant of this quadratic is D = 4n(n + 1) b − a2 /(n + 1) , so we conclude that (i) if a2 > (n + 1)b, then such an x0 does not exist; (ii) if a2 = (n + 1)b, then x0 √ = a/n + 1; and √ D/2 D/2 (iii) if a2 < (n + 1)b, then a−n+1 ≤ x0 ≤ a+n+1 . It is easy to see that these conditions for x0 are also sufficient. 27. Let n be the required exponent, and suppose n = 2k q, where q is an odd integer. Then we have k mn − 1 = (m2 − 1)[(m2 k (q−1) k k + · · · + m2 + 1] = (m2 − 1)A, k where A is odd. Therefore mn − 1 and m2 − 1 are divisible by the same power of 2, and so n = 2k . Next, we observe that k m2 − 1 = (m2 k−1 − 1)(m2 k−1 + 1) = . . . = (m2 − 1)(m2 + 1)(m4 + 1) · · ·(m2 k−1 + 1). Let s be the maximal positive integer for which m ≡ ±1 (mod 2s ). Then m2 − 1 is divisible by 2s+1 and not divisible by 2s+2 . All the numbers m2 + 1, m4 + k−1 k 1, . . . , m2 + 1 are divisible by 2 and not by 4. Hence m2 − 1 is divisible by s+k s+k+1 2 and not by 2 . It follows from the above consideration that the smallest exponent n equals 21989−s if s ≤ 1989, and n = 1 if s > 1989. 28. Assume w.l.o.g. that the rays OA1 , OA2 , OA3 , OA4 are arranged clockwise. Setting OA1 = a, OA2 = b, OA3 = c, OA4 = d, and ∠A1 OA2 = x, ∠A2 OA3 = y, ∠A3 OA4 = z, we have 1 1 S1 = σ (OA1 A2 ) = ab| sin x|, S2 = σ (OA1 A3 ) = ac| sin(x + y)|, 2 2 1 1 S3 = σ (OA1 A4 ) = ad| sin(x + y + z)|, S4 = σ (OA2 A3 ) = bc| sin y|, 2 2 1 1 S5 = σ (OA2 A4 ) = bd| sin(y + z)|, S6 = σ (OA3 A4 ) = cd| sin z|. 2 2 534 4 Solutions Since sin(x + y + z) sin y + sin x sin z = sin(x + y) sin(y + z), it follows that there exists a choice of k, l ∈ {0, 1} such that S1 S6 + (−1)k S2 S5 + (−1)l S3 S4 = 0. For example (w.l.o.g.), if S3 S4 = S1 S6 + S2 S5 , we have  2 max Si ≥ S3 S4 = S1 S6 + S2 S5 ≥ 1 + 1 = 2, √ i.e., max1≤i≤6 Si ≥ 2 as claimed. 1≤i≤6 29. Let Pi , sitting at the place A, and Pj sitting at B, be two birds that can see each other. Let k and l respectively be the number of birds visible from B but not from A, and the number of those visible from A but not from B. Assume that k ≥ l. Then if all birds from B fly to A, each of them will see l new birds, but won’t see k birds anymore. Hence the total number of mutually visible pairs does not increase, while the number of distinct positions occupied by at least one bird decreases by one. Repeating this operation as many times as possible one can arrive at a situation in which two birds see each other if and only if they are in the same position. The number of such distinct positions is at most 35, while the total number of mutually visible pairs is not greater than at the beginning. Thus the problem is equivalent to the following one: (1) If xi ≥ 0 are integers with ∑35 j=1 x j = 155, find the least possible value of 35 2 ∑ j=1 (x j − x j )/2. If x j ≥ xi + 2 for some i, j, then the sum of (x2j − x j )/2 decreases (for x j − xi − 2) if xi , x j are replaced with xi + 1, x j − 1. Consequently, our sum attains its minimum when the xi ’s differ from each other by at most 1. In this case, all the xi ’s are equal to either [155/35] = 4 or [155/35] + 1 = 5, where 155 = 20 · 4 + 15 · 5. It follows that the (minimum possible) number of mutually visible 5·4 pairs is 20 · 4·3 2 + 15 · 2 = 270. Second solution for (1). Considering the graph consisting of birds as vertices and pairs of mutually nonvisible birds as edges, we see that there is no complete 36subgraph. Turan’s theorem gives the answer immediately. (See problem (SL8917).) 30. For all n such N exists. For a given n choose N = (n + 1)!2 + 1. Then 1 + j is a proper factor of N + j for 1 ≤ j ≤ n. So if N + j = pm is a power of a prime p, then 1 + j = pr for some integer r, 1 ≤ r < m. But then pr+1 divides both (n + 1)!2 = N − 1 and pm = N + j, implying that pr+1 | 1 + j, which is impossible. Thus none of N + 1, N + 2, . . ., N + n is a power of a prime. Second solution. Let p1 , p2 , . . . , p2n be distinct primes. By the Chinese remainder theorem, there exists a natural number N such that p1 p2 | N + 1, p3 p4 | N + 2, . . . , p2n−1 p2n | N + n, and then obviously none of the numbers N + 1, . . . , N + n can be a power of a prime. 4.30 Shortlisted Problems 1989 535 31. Let us denote by N pqr the number of solutions for which a p /x p ≥ aq /xq ≥ ar /xr , where (p, q, r) is one of six permutations of (1, 2, 3). It is clearly enough to prove that Npqr + Nqpr ≤ 2a1 a2 (3 + ln(2a1 )). First, from 3a p a p aq ar ap ≥ + + =1 and <1 xp x p xq xr xp we get a p + 1 ≤ x p ≤ 3a p . Similarly, for fixed x p we have   2aq aq ar ap aq ap ap ≥ + = 1− and ≤ min ,1− , xq xq xr xp xq xp xp  which gives max aq · x p /a p, aq · x p /(x p − a p) ≤ xq ≤ 2aq · x p /(x p − a p ), i.e., if a p + 1 ≤ x p ≤ 2a p there are at most aq · x p /(x p − a p ) + 1/2 possible values for xq (because there are [2x] − [x] = [x + 1/2] integers between x and 2x), and if 2a p + 1 ≤ x p ≤ 3a p , at most 2aq · x p /(x p − a p) − aq · x p /a p + 1 possible values. Given x p and xq , xr is uniquely determined. Hence    3a p aq · x p 2aq · x p aq · x p 1 ≤ ∑ + + ∑ − +1 2 ap x p =a p +1 x p − a p x p =2a p +1 x p − a p   ap  3a p k + ap 2(k + 2a p) k + 2a p = + aq ∑ + − 2 k k + ap ap k=1   ap  3a p k 1 2 = + aq ∑ 1 − + a p + 2 ap k k + ap k=1 ! ap  3a p aq 1 1 2 = − + a p aq +∑ + 2 2 2 k=1 k k + ap   3 1 5 ≤ a p aq − + ln(2a p ) + − ln 2 . 2aq 2a p 2 2a p N pqr  Here we have used ∑nk=1 (1/k + 2/(k + n)) ≤ ln(2n) + 2 − ln 2 (this can be proved by induction). Hence, Npqr + Nqpr ≤ 2a p aq (1 + 0.5 + ln(2a p) + 2 − ln2) < 2a1 a2 (2.81 + ln(2a1 )). Remark. The official solution was somewhat simpler, but used that the interval (x, 2x], for real x, cannot contain more than x integers, which is false in general. Thus it could give only a weaker estimate N ≤ 6a1 a2 (9/2 − ln2 + ln(2a1 )). 32. Let CC′ be an altitude, and R the circumradius. Then, since AH = R, we have AC′ = |R sin B| and hence (1) CC′ = |R sin B tan A|. On the other hand, CC′ = |BC sin B| = 2|R sin A sin B|, which together with (1) yields 2| sin A| = | tan A| ⇒ | cos A| = 1/2. Hence, ∠A is 60◦ . (Without the condition that the triangle is acute, ∠A could also be 120◦ .) 536 4 Solutions Second Solution. For a point X , let X denote the vector OX . Then |A| = |B| = |C| = R and H = A + B +C, and moreover, 2 2 R2 = (H − A)2 = (B +C)2 = 2B + 2C − (B − C)2 = 4R2 − BC2. √ ◦ It follows that sin A = BC 2R = 3/2, i.e., that ∠A = 60 . Third Solution. Let A1 be the midpoint of BC. It is well known that AH = 2OA1 , and since AH = AO = BO, it means that in the right-angled triangle BOA1 the relation BO = 2OA1 holds. Thus ∠BOA1 = ∠A = 60◦ . 4.31 Shortlisted Problems 1990 537 4.31 Solutions to the Shortlisted Problems of IMO 1990 1. Let N be a number that can be written as a sum of 1990 consecutive integers and as a sum of consecutive positive integers in exactly 1990 ways. The former requirement gives us N = m + (m + 1) + · · · + (m + 1989) = 995(2m + 1989) for some m. Thus 2 ∤ N, 5 | N, and 199 | N. The latter requirement tells us that there are exactly 1990 ways to express N as n + (n + 1) + · · · + (n + k), or equivalently, express 2N as (k + 1)(2n + k). Since N is odd, it follows that one of the factors k + 1 and 2n + k is odd and the other is divisible by 2, but not by 4. Evidently k + 1 < 2n + k. On the other hand, every factorization 2N = ab, 1 < a < b, corresponds to a single pair (n, k), where n = b−a+1 (which is an integer) and 2 k = a − 1. The number of such factorizations is equal to d(2N)/2 − 1 because a = b is impossible (here d(x) denotes the number of positive divisors of an x ∈ N). Hence we must have d(2N) = 2 · 1991 = 3982. Now let 2N = 2 · 5e1 · 199e2 · pe33 · · · per r be a factorization of 2N into prime numbers, where p3 , . . . , pr are distinct primes other than 2, 5, and 199 and e1 , · · · , er are positive integers. Then d(2N) = 2(e1 + 1)(e2 + 1) · · · (er +1), from which we deduce (e1 + 1)(e2 + 1) · · · (er + 1) = 1991 = 11 · 181. We thus get {e1 , e2 } = {10, 180} and e3 = · · · = er = 0. Hence N = 510 · 199180 and N = 5180 · 19910 are the only possible solutions. These numbers indeed satisfy the desired properties. 2. We will call a cycle with m committees and n countries an (m, n) cycle. We will number the delegates from each country with numbers 1, 2, 3 and denote committees by arrays of these integers (of length n) defining which of the delegates from each country is in the committee. We will first devise methods of constructing larger cycles out of smaller cycles. Let A1 , . . . , Am be an (m, n) cycle, where m is odd. Then the following is a (2m, n + 1) cycle: (A1 , 1), (A2 , 2), . . . , (Am , 1), (A1 , 2), (A2 , 1), . . . , (Am , 2). Also, let A1 , . . . , Am be an (m, n) cycle and k ≤ m an even integer. Then the cycle (A1 , 3), (A2 , 1), (A3 , 2), . . . , (Ak−2 , 1), (Ak−1 , 2), (Ak , 3), (Ak−1 , 1), (Ak−2 , 2), . . . , (A2 , 2) is a (2(k − 1), n + 1) cycle. Starting from the ((1),(2),(3)) cycle with parameters (3, 1) we can sequentially construct larger cycles using the shown methods. The obtained cycles have parameters as follows: (6, 2), (10, 3), . . . , (2k + 2, k), . . . , (1026, 10), (1990, 11). Thus there exists a cycle of 1990 committees with 11 countries. 3. A segment connecting two points which divides the given circle into two arcs one of which contains exactly n points in its interior we will call a good segment. Good segments determine one or more closed polygonal lines that we will call 538 4 Solutions stars. Let us compute the number of stars. Note first that gcd(n + 1, 2n − 1) = gcd(n + 1, 3). (i) Suppose that 3 ∤ n + 1. Then the good segments form a single star. Among any n points, two will be adjacent vertices of the star. On the other hand, we can select n − 1 alternate points going along the star, and in this case no two points lie on a good segment. Hence N   = n.   (ii) If 3 | n + 1, we obtain three stars of 2n−1 vertices. If more than 2n−1 = 3 6 n−2 3 points are chosen on any of the stars, then two of them will be connected with a good segment. On the other hand, we can select n−2 3 alternate points on each star, which adds up to n − 2 points in total, no two of which lie on a good segment. Hence N = n − 1. To sum up, N = n for 3 ∤ 2n − 1 and N = n − 1 for 3 | 2n − 1. 4. Assuming that A1 is not such a set Ai , it follows that for every m there exist m consecutive numbers not in A1 . It follows that A2 ∪ A3 ∪ · · · ∪ Ar contains arbitrarily long sequences of numbers. Inductively, let us assume that A j ∪ A j+1 ∪ · · · ∪ Ar contains arbitrarily long sequences of consecutive numbers and none of A1 , A2 , . . . , A j−1 is the desired set Ai . Let us assume that A j is also not Ai . Hence for each m there exists k(m) such that among k(m) elements of A j there exist two consecutive elements that differ by at least m. Let us consider m · k(m) consecutive numbers in A j ∪ · · · ∪ Ar , which exist by the induction hypothesis. Then either A j contains fewer than k(m) of these integers, in which case A j+1 ∪ · · · ∪ Ar contains m consecutive integers by the pigeonhole principle or A j contains k(m) integers among which there exists a gap of length m of consecutive integers that belong to A j+1 ∪ · · · ∪ Ar . Hence we have proven that A j+1 ∪ · · · ∪ Ar contains sequences of integers of arbitrary length. By induction, assuming that A1 , A2 , . . . , Ar−1 do not satisfy the conditions to be the set Ai , it follows that Ar contains sequences of consecutive integers of arbitrary length and hence satisfies the conditions necessary for it to be the set Ai . 5. Let O be the circumcenter of ABC, E the midpoint of OH, and R and r the radii of the circumcircle and incircle respectively. We use the following facts from −→ −→ elementary geometry: OH = 3OG, OK 2 = R2 − 2Rr, and KE = R2 − r. Hence −→ −→ −→ −→ −→ −→ KO KH = 2KE − KO and KG = 2KE+ . We then obtain 3 2 −→ −→ 1 KH · KG = (4KE 2 − KO2 ) = − r(R − 2r) < 0 . 3 3 Hence cos ∠GKH < 0 ⇒ ∠GKH > 90◦ . 6. Let W denote the set of all n0 for which player A has a winning strategy, L the set of all n0 for which player B has a winning strategy, and T the set of all n0 for which a tie is ensured. Lemma. Assume {m, m + 1, . . .1990} ⊆ W and that there exists s ≤ 1990 such that s/pr ≥ m, where pr√is the largest degree of a prime that divides s. Then all integers x such that s ≤ x < m also belong in W . 4.31 Shortlisted Problems 1990 539 Proof. Starting from x, player A can choose s, and by definition of s, player B cannot choose a number smaller than m. This ensures player A the victory. We now have trivially that since 452 = 2025 > 1990, it follows that for n0 ∈ {45, . . . , 1990} the player A can choose 1990 in the first move. Therefore 2 {45, . . . , 1990} ⊆ W . Using m = 45 and selecting √ s = 420 = 2 · 3 · 5 · 7 we apply the lemma to get that all integers x such that 420 < 21 ≤ x ≤ 1990 are in W . Again, using m = 21 and selecting s = 168 = 23 · 3 · 7 we apply the lemma to get √ that all integers x such that 168 < 13 ≤ x ≤ 1990 are in W . Selecting s = 105 we obtain the new value for m at m = 11. Selecting s = 60 we obtain m = 8. Thus {8, . . . , 1990} ⊆ W . For n0 > 1990 there exists r ∈ N such that 2r · 32 < n0 ≤ 2r+1 · 32 < n20 . Player A can take n1 = 2r+1 · 32 . The number player B selects has to satisfy 8 ≤ n2 < n0 . After finitely many steps he will select 8 ≤ n2r ≤ 1990, and A will have a winning strategy. Hence all m ≥ 8 belong to W . Now let us consider the case n0 ≤ 5. Since the smallest number divisible by three different primes is 30 and n20 ≤ 52 = 25 < 30, it follows that n1 is of the form n1 = pr or n1 = pr · qs , where p and q are two different primes. In the first case player B can choose 1 and win, while in the second case he can select √ the smaller of pr , qs , which is also smaller than n1 ≤ n0 . Thus player B can eventually reach n2k = 1. Thus {2, 3, 4, 5} ⊆ L. Finally, for n0 = 6 or n0 = 7 player A must select a number divisible by at least three primes, which must be 30 = 2 · 3 · 5 or 42 = 2 · 3 · 7; otherwise, B can select a degree of a prime smaller than n0 , yielding n2 < 6 and victory for B. Player B must select a number smaller than 8. Hence, he has to select 6 in both cases. Afterwards, to avoid losing the game, player A will always choose 30 and player B always 6. In this case we would have a tie. Hence T ⊆ {6, 7}. Considering that we have accounted for all integers n0 > 1, the final solution is L = {2, 3, 4, 5}, T = {6, 7}, and W = {x ∈ N | x ≥ 8}. 2 7. Let f (n) = g(n)2n for all n. The recursion then transforms into g(n+2)−2g(n+ 1) + g(n) = n · 16−n−1 for n ∈ N0 . By summing this equation from 0 to n − 1, we get 1 g(n + 1) − g(n) = 2 · (1 − (15n + 1)16−n). 15 By summing up again from 0 to n − 1 we get g(n) = 1513 · (15n − 32 + (15n + 2)16−n+1). Hence f (n) = 1 2 · (15n + 2 + (15n − 32)16n−1) · 2(n−2) . 3 15 Now let us look at the values of f (n) modulo 13: f (n) ≡ 15n + 2 + (15n − 32)16n−1 ≡ 2n + 2 + (2n − 6)3n−1. We have 33 ≡ 1 (mod 13). Plugging in n ≡ 1 (mod 13) and n ≡ 1 (mod 3) for n = 1990 gives us f (1990) ≡ 0 (mod 13). We similarly calculate f (1989) ≡ 0 and f (1991) ≡ 0 (mod 13). 540 4 Solutions 8. Since 21990 < 8700 < 10700, we have f1 (21990 ) < (9 · 700)2 < 4 · 107 . We then have f2 (21990 ) < (3 + 9 · 7)2 < 4900 and finally f3 (21990 ) < (3 + 9 · 3)2 = 302 . It is easily shown that fk (n) ≡ fk−1 (n)2 (mod 9). Since 26 ≡ 1 (mod 9), we have 21990 ≡ 24 ≡ 7 (all congruences in this problem will be mod 9). It follows that f1 (21990) ≡ 72 ≡ 4 and f2 (21990 ) ≡ 42 ≡ 7. Indeed, it follows that f2k (21990 ) ≡ 7 and f2k+1 (21990 ) ≡ 4 for all integer k > 0. Thus f3 (21990 ) = r2 where r < 30 is an integer and r ≡ f2 (21990 ) ≡ 7. It follows that r ∈ {7, 16, 25} and hence f3 (21990) ∈ {49, 256, 625}. It follows that f4 (21990 ) = 169, f5 (21990 ) = 256, and inductively f2k (21990 ) = 169 and f2k+1 (21990 ) = 256 for all integer k > 1. Hence f1991 (21990 ) = 256. 9. Let a, b, c be the lengths of the sides of △ABC, s = a+b+c 2 , r the inradius of the triangle, and c1 and b1 the lengths of AB2 and AC2 respectively. As usual we will denote by S(XY Z) the area of △XY Z. We have AC1 · AB2 c1 rs S(ABC) = , AC · AB 2b c1 r cr S(AKB2 ) = , S(AC1K) = . 2 4 1 rs From S(AC1 B2 ) = S(AKB2 )+ S(AC1 K) we get c2b = c21 r + cr4 ; therefore (a − b + c)c1 = bc. By looking at the area of △AB1C2 we similarly obtain (a + b − c)b1 = bc. From these two equations and from S(ABC) = S(AB2C2 ), from which we have b1 c1 = bc, we obtain S(AC1 B2 ) = a2 − (b − c)2 = bc ⇒ b 2 + c2 − a 2 1 = cos(∠BAC) = ⇒ ∠BAC = 60◦ . 2bc 2 10. Let r be the radius of the base and h the height of the cone. We may assume w.l.o.g. that r = 1. Let A be the top of the cone, BC the diameter of the circumference of the base such that the plane touches the circumference at B, O the center of the base, and H the midpoint of OA (also belonging to the plane). Let BH cut the sheet of the cone at D. By applying Menelaus’s theorem to △AOC AD OH 1 HD HA OC 1 and △BHO, we conclude that DC = CB BO · HA = 2 and DB = AO · CB = 4 . The plane cuts the cone in an ellipse whose major axis is BD. Let E be the center BE of this ellipse and FG its minor axis. We have ED = 12 . Let E ′ , F ′ , G′ be radial projections of E, F, G from A onto the base of the cone. Then E sits on BC. Let h(X ) denote the height of a point X with respect to the base of the cone. We have h(E) = h(D)/2 = h/3. Hence EF = 2E ′ F ′ /3. Applying Menelaus’s theorem to √ ′ BE HA 2 3 √1 . △BHO we get OE = · = 1. Hence EF = = E′B EH AO 3 2 3 Let d denote the distance from A to the plane. Let V1 and V denote the volume of the cone above the plane (on the same side of the plane as A) and the total volume of the cone. We have √ V1 BE · EF · d (2BH/3)(1/ 3)(2SAHB /BH) = = V h √ h (2/3)(1/ 3)(h/2) 1 = = √ . h 3 3 4.31 Shortlisted Problems 1990 541 Since this ratio is smaller than 1/2, we have indeed selected the correct volume for our ratio. 11. Assume B(A, E, M, B). Since A, B,C, D lie on a circle, we have ∠GCE = ∠MBD and ∠MAD = ∠FCE. Since FD is tangent to the circle around △EMD at E, we have ∠MDE = ∠FEB = ∠AEG. Consequently, ∠CEF = 180◦ − ∠CEA − ∠FEB = 180◦ − ∠MED − ∠MDE = ∠EMD and ∠CEG = 180◦ − ∠CEF = 180◦ − ∠EMD = ∠DMB. Therefore △CEF ∼ △AMD and △CEG ∼ △BMD. From the first similarity we obtain CE · C MD = AM · EF, and from the second we obtain CE · MD = BM · EG. Hence F AM · EF = BM · EG ⇒ A GE AM λ = = . EF BM 1−λ If B(A, M, E, B), interchanging the roles of A and B we similarly obtain E M B D G GE EF = λ 1−λ . 12. Let d(X , l) denote the distance of a point X from a line l. Using the elemena tary facts that AF : FC = c : a and BD : DC = c : b, we obtain d(F, L) = a+c hc b and d(D, L) = b+c hc , where ha is the altitude of △ABC from A. We also have ∠FGC = β /2, ∠DEC = α /2. It follows that DE = d(D, L) sin(α /2) and FG = d(F, L) . sin(β /2) (1) x Now suppose that a > b. Since the function f (x) = x+c is strictly increasing, we deduce d(F, L) > d(D, L). Furthermore, sin(α /2) > sin(β /2), so we get from (1) that FG > DE. Similarly, a < b implies FG < DE. Hence we must have a = b, i.e., AC = BC. 13. We will call the ground the “zeroth” rung. We will prove that the minimum n is n = a + b − (a, b). It is plain that if (a, b) = k > 1, the scientist can climb only onto the rungs divisible by k and we can just observe these rungs to obtain the situation equivalent to a′ = a/k, b′ = b/k, and n′ = a′ + b′ −1. Thus let us assume that (a, b) = 1 and show that n = a + b − 1. We obviously have n > a. Consider n = a + b − k, k ≥ 1, and let us assume without loss of generality that a > b (otherwise, we can reverse the problem starting from the top rung in our round trip). Then we can uniquely define the numbers ri , 0 ≤ ri < b, by ri ≡ ia (mod b). We now describe the only possible sequence of moves. From a position 0 ≤ p ≤ b − k we can move only a rungs upward and for p > b − 1 we can move only b rungs downward. If we end up at b − k < p ≤ b − 1, we are stuck. Hence, given that we are at ri , if ri ≤ b − k, we can move to a + ri , and when we descend as far as we can go we will end up at ri+1 ≡ a + ri (mod b). 542 4 Solutions If the mathematician climbs to the highest rung and then comes back to ri = 0, then we deduce b | ia, so i ≥ b. But since (a, b) = 1, there exists 0 < j < b such that r j ≡ ja ≡ b − 1 (mod b). Thus the mathematician has visited the position b − 1. For him not to get stuck we must have k ≤ 1 and n ≥ a + b − 1. For n = a + b − 1 by induction he can come to any position ri , i ≥ 0, so he eventually comes to r j = b − 1, climbs to the highest rung, and then continues until he gets to rb = 0. Hence the answer to the problem is n = a + b − 1. 14. Let V be the set of all midpoints of bad sides, and E the set of segments connecting two points in V that belong to the same triangle. Each edge in E is parallel to exactly one good side and thus is parallel to the coordinate grid and has halfinteger coordinates. Thus, the edges of E are a subset of the grid formed by joining the centers of the squares in the original grid to each other. Let G be a graph whose set of vertices is V and set of edges is E. The degree of each vertex X , denoted by d(X ), is 0, 1, or 2. We observe the following cases: (i) d(X ) = 0 for some X. Then both triangles containing X have two good sides. (ii) d(X ) = 1 for some X . Since ∑X∈V d(X) = 2|E| is even, it follows that at least another vertex Y has the degree 1. Hence both X and Y belong to triangles having two good sides. (iii) d(X ) = 2 for all X ∈ V . We will show that this case cannot occur. We prove first that centers of all the squares of the m × n board belong to V ∪ E. A bad side contains no points with half-integer coordinates in its interior other than its midpoint. Therefore either a point X is in V , or it lies on the segment connecting the midpoints of the two bad sides. Evidently, the graph G can be partitioned into disjoint cycles. Each center of a square is passed exactly once in exactly one cycle. Let us color the board black and white in a standard chessboard fashion. Each cycle passes through centers that must alternate in color, and hence it contains an equal number of black and white centers. Consequently, the numbers of black and white squares on the entire board must be equal, contradicting the condition that m and n are odd. Our proof is thus completed. 15. Let S(Z) denote the sum of all the elements of a set Z. We have S(X ) = (k + 1) · 1990 + k(k+1) 2 . To partition the set into two parts with equal sums, S(X ) must be even and hence k(k+1) must be even. Hence k is of the form 4r or 4r + 3, where 2 r is an integer. For k = 4r + 3 we can partition X into consecutive fourtuplets {1990+4l, 1990+ 4l + 1, 1990 + 4l + 2, 1990 + 4l + 3} for 0 ≤ l ≤ r and put 1990 + 4l, 1990 + 4l + 3 ∈ A and 1990 + 4l + 1, 1990 + 4l + 2 ∈ B for all l. This would give us S(A) = S(B) = (3980 + 4r + 3)(r + 1). For k = 4r the numbers of elements in A and B must differ. Let us assume without loss of generality |A| < |B|. Then S(A) ≤ (1990 + 2r + 1) + (1990 + 2r + 2) + · · · + (1990 + 4r) and S(B) ≥ 1990 + 1991 + · · ·+ (1990 + 2r). Plug- 4.31 Shortlisted Problems 1990 543 ging these inequalities into the condition S(A) = S(B) gives us r ≥ 23 and consequently k ≥ 92. We note that B = {1990, 1991, . . ., 2034, 2052, 2082} and A = {2035, 2036, . . ., 2051, 2053, . . ., 2081} is a partition for k = 92 that satisfies S(A) = S(B). To construct a partition out of higher k = 4r we use the k = 92 partition for the first 93 elements and construct for the remaining elements as was done for k = 4r + 3. Hence we can construct a partition exactly for the integers k of the form k = 4r + 3, r ≥ 0, and k = 4r, r ≥ 23. 16. Let A0 A1 . . . A1989 be the desired 1990-gon. We also define A1990 = A0 . Let −−→ O be an arbitrary point. For 1 ≤ i ≤ 1990 let Bi be a point such that OBi = −−−−→ Ai−1 Ai . We define B0 = B1990 . The points Bi must satisfy the following prop2π erties: ∠Bi OBi+1 = 1990 , 0 ≤ i ≤ 1989, lengths of OBi are a permutation of −−→ − → 2 2 2 2 1 , 2 , . . . , 1989 , 1990 , and ∑1989 i=0 OBi = 0 . Conversely, any such set of points Bi corresponds to a desired 1990-gon. Hence, our goal is to construct vectors −−→ OBi satisfying all the stated properties. Let us group vectors of lengths (2n − 1)2 and (2n)2 into pairs and put them diametrically opposite each other. The length of the resulting vectors is 4n − 1. The problem thus reduces to arranging vectors of lengths 3, 7, 11, . . .,3979 at mu→ − 2π tual angles of 995 such that their sum is 0 . We partition the 995 directions into 199 sets of five directions at mutual angles 25π . The directions when intersected with a unit circle form a regular pentagon. We group the set of lengths of vectors 3, 7, . . . , 3979 into 199 sets of five consecutive elements of the set. We place each group of lengths on directions belonging to the same group of directions, −−→ −−→ thus constructing five vectors. We use that OC1 + · · · + OCn = 0 where O is the center of a regular n-gon C1 . . .Cn . In other words, vectors of equal lengths along directions that form a regular n-gon cancel each other out. Such are the groups of five directions. Hence, we can assume for each group of five lengths for its lengths to be {0, 4, 8, 12, 16}. We place these five lengths in a random fashion on 2π a single group of directions. We then rotate the configuration clockwise by 199 to cover other groups of directions and repeat until all groups of directions are exhausted. It follows that all vectors of each of the lengths {0, 4, 8, 12, 16} will form a regular 199-gon and will thus cancel each other out. We have thus constructed a way of obtaining points Bi and have hence shown the existence of the 1990-gon satisfying (i) and (ii). 17. Let us set a coordinate system denoting the vertices of the block. The vertices of the unit cubes of the block can be described as {(x, y, z) | 0 ≤ x ≤ p, 0 ≤ y ≤ q, 0 ≤ z ≤ r}, and we restrict our attention to only these points. Suppose the point A is fixed at (a, b, c). Then for every other necklace point (x, y, z) numbers x − a, y − b, and z − c must be of equal parity. Conversely, every point (x, y, z) such that x − a, y − b, and z − c are of the same parity has to be a necklace point. Consider the graph G whose vertices are all such points and edges are all diagonals of the unit cubes through these points. In part (a) we are looking for an open or closed Euler path, while in part (b) we are looking for a closed Euler path. 544 4 Solutions Necklace points in the interior of the (p, q, r) box have degree 8, points on the surface have degree 4, points on the edge have degree 2, and points on the corner have degree 1. A closed Euler path can be formed if and only if all vertices are of an even degree, while an open Euler path can be formed if and only if exactly two vertices have an odd degree. Hence the problem in part (a) amounts to being able to choose a point A such that 0 or 2 corner vertices are necklace vertices, whereas in part (b) no corner points can be necklace vertices. We distinguish two cases. (i) At least two of p, q, r, say p, q, are even. We can choose a = 1, b = c = 0. In this case none of the corners is a necklace point. Hence a closed Euler path exists. (ii) At most one of p, q, r is even. However one chooses A, exactly two necklace points are at the corners. Hence, an open Euler path exists, but it is impossible to form a closed path. Hence, in part (a), a box can be made of all (p, q, r) and in part (b) only those (p, q, r) where at least two of the numbers are even. 18. Clearly, it suffices to consider the case (a, b) = 1. Let S be the set of integers such that M − b ≤ x ≤ M + a − 1. Then f (S) ⊆ S and 0 ∈ S. Consequently, f k (0) ∈ S. Let us assume for k > 0 that f k (0) = 0. Since f (m) = m + a or f (m) = m − b, it follows that k can be written as k = r + s, where ra − sb = 0. Since a and b are relatively prime, it follows that k ≥ a + b. Let us now prove that f a+b (0) = 0. In this case a+b = r +s and hence f a+b (0) = (a + b − s)a − sb = (a + b)(a − s). Since a + b | f a+b (0) and f a+b (0) ∈ S, it follows that f a+b (0) = 0. Thus for (a, b) = 1 it follows that k = a + b. For other a+b a and b we have k = (a,b) . 19. Let d1 , d2 , d3 , d4 be the distances of the point P to the tetrahedron. Let d be the height of the regular tetrahedron. Let xi = di /d. Clearly, x1 + x2 + x3 + x4 = 1, and given this condition, the parameters vary freely as we vary P within the tetrahedron. The four tetrahedra have volumes x31 , x32 , x33 , and x34 , and the four parallelepipeds have volumes of 6x2 x3 x4 , 6x1 x3 x4 , 6x1 x2 x4 , and 6x1 x2 x3 . Hence, using x1 + x2 + x3 + x4 = 1 and setting g(x) = x2 (1 − x), we directly verify that 4 f (P) = f (x1 , x2 , x3 , x4 ) = 1 − ∑ x3i − 6 i=1 ∑ xi x j xk 1≤i< j<k≤4 = 3(g(x1 ) + g(x2 ) + g(x3 ) + g(x4 )) . We note that g(0) = 0 and g(1) = 0. Hence, as x1 tends to 1 and other variables tend to 0, f (x1 , x2 , x3 , x4 ) = 0. Thus f (P) is sharply bounded downwards at 0. We now find an upper bound. We note that g(xi + x j ) = (xi + x j )2 (1 − x1 − x2 )   3 = g(xi ) + g(x j ) + 2xi x j 1 − (xi + x j ) ; 2 4.31 Shortlisted Problems 1990 545 thus for xi + x j ≤ 2/3 and xi , x j > 0 we have g(xi + x j ) + g(0) ≥ g(xi ) + g(x j ). Equality holds only when xi + x j = 2/3. Assuming without loss of generality x1 ≥ x2 ≥ x3 ≥ x4 , we have g(x1 ) + g(x2 ) + g(x3 ) + g(x4 ) < g(x1 ) + g(x2 ) + g(x3 + x4 ). Assuming y1 + y2 + y3 = 1 and y1 ≥ y2 ≥ y3 , we have g(y1 ) + g(y2 ) + g(y3 ) ≤ g(y1 ) + g(y2 + y3 ). Hence g(x1 ) + g(x2 ) + g(x3) + g(x4 ) < g(x) + g(1 − x) for some x. We also have g(x) + g(1 − x) = x(1 − x) ≤ 1/4. Hence f (P) ≤ 3/4. Equality holds for x1 = x2 = 1/2, x3 = x4 = 0 (corresponding to the midpoint of an edge), and as the variables converge to these values, f (P) converges to 3/4. Hence the bounds for f (P) are 3 0 < f (P) < . 4 20. Let n be the unique integer such that 2n−1 ≤ k < 2n . Let S(n) be the set of numbers less than 10n that are written with only the digits {0, 1} in the decimal system. Evidently |S(n)| = 2n > k and hence there exist two numbers x, y ∈ S(n) such that k | x − y. Let us show that w = |x − y| is the desired number. By definition k | w. We also have √ √ w < 1.2 · 10n−1 ≤ 1.2 · (23 2)n−1 ≤ 1.2 · k3 k ≤ k4 . Finally, since x, y ∈ S(n), it follows that w = |x − y| can be written using only the digits {0, 1, 8, 9}. This completes the proof. 21. We must solve the congruence (1 + 2 p + 2n−p )N ≡ 1 (mod 2n ). Since (1 + 2 p + 2n−p ) and 2n are coprime, there clearly exists a unique N satisfying this equation and 0 < N < 2n .   j 2 j p ≡ 1 (mod 2n ) Let us assume n = mp. Then we have (1 + 2 p) ∑m−1 (−1) j=0 and (1 + 2n−p)(1 − 2n−p) ≡ 1 (mod 2n ). By multiplying the two congruences we obtain ! (1 + 2 p)(1 + 2n−p)(1 − 2n−p) (1 + 2 p)(1 + 2n−p) m−1 ∑ (−1) j 2 j p j=0 ≡ 1 (mod 2n ) . (mod 2n ), it follows that N ≡ (1 −  ≡ j j p (mod 2n ). The integer N = m−1 (−1) j 2 j p − 2n−p + 2n 2n−p ) ∑m−1 ∑ j=0 j=0 (−1) 2 satisfies the congruence and 0 < N ≤ 2n . Using that for a > b we have in binary representation 2a − 2b = 11 . . 11} 00 . . 00}, | .{z | .{z Since  (1 + 2 p + 2n−p) a−b times b times the binary representation of N is calculated as follows:  11 . . 11} 11 . . 11} 00 . . 00} . . . 11 . . 11} 00 . . 00} 1,  | .{z | .{z | .{z | .{z | .{z    p times p times p times p−1 times p times N=  11 . . . 11 00 . . . 00 11 . . . 11 00 . . . 00 . . . 11 . . . 11 00 . . . 00 1,    | {z } | {z } | {z } | {z } | {z } | {z } p−1 times p+1 times p times p times p times p−1 times 2 ∤ np , 2 | np . 546 4 Solutions 22. We can assume without loss of generality that each connection is serviced by only one airline and the problem reduces to finding two disjoint monochromatic cycles of the same color and of odd length on a complete graph of 10 points colored by two colors. We use the following two standard lemmas: Lemma 1. Given a complete graph on six points whose edges are colored with two colors there exists a monochromatic triangle. Proof. Let us denote the vertices by c1 , c2 , c3 , c4 , c5 , c6 . By the pigeonhole principle at least three vertices out of c1 , say c2 , c3 , c4 , are of the same color, let us call it red. Assuming that at least one of the edges connecting points c2 , c3 , c4 is red, the connected points along with c1 form a red triangle. Otherwise, edges connecting c2 , c3 , c4 are all of the opposite color, let us call it blue, and hence in all cases we have a monochromatic triangle. Lemma 2. Given a complete graph on five points whose edges are colored with two colors there exists a monochromatic triangle or a monochromatic cycle of length five. Proof. Let us denote the vertices by c1 , c2 , c3 , c4 , c5 . Assume that out of a point ci three vertices are of the same color. We can then proceed as in Lemma 1 to obtain a monochromatic triangle. Otherwise, each point is connected to other points with exactly two red and two blue vertices. Hence, we obtain monochromatic cycles starting from a single point and moving along the edges of the same color. Since each cycle must be of length at least three (i.e., we cannot have more than one cycle of one color), it follows that for both red and blue we must have one cycle of length five of that color. We now apply the lemmas. Let us denote the vertices by c1 , c2 , . . . , c10 . We apply Lemma 1 to vertices c1 , . . . , c6 to obtain a monochromatic triangle. Out of the seven remaining vertices we select 6 and again apply Lemma 1 to obtain another monochromatic triangle. If they are of the same color, we are done. Otherwise, out of the nine edges connecting the two triangles of opposite color at least 5 are of the same color, we can assume blue w.l.o.g., and hence a vertex of a red triangle must contain at least two blue edges whose endpoints are connected with a blue edge. Hence there exist two triangles of different colors joined at a vertex. These take up five points. Applying Lemma 2 on the five remaining points, we obtain a monochromatic cycle of odd length that is of the same color as one of the two joined triangles and disjoint from both of them. 23. Let us assume n > 1. Obviously n is odd. Let p ≥ 3 be the smallest prime divisor of n. In this case (p − 1, n) = 1. Since 2n + 1 | 22n − 1, we have that p | 22n − 1. Thus it follows from Fermat’s little theorem and elementary number theory that p | (22n − 1, 2 p−1 − 1) = 2(2n,p−1) − 1. Since (2n, p − 1) ≤ 2, it follows that p | 3 and hence p = 3. Let us assume now that n is of the form n = 3k d, where 2, 3 ∤ d. We first prove that k = 1. Lemma. If 2m − 1 is divisible by 3r , then m is divisible by 3r−1 . Proof. This is the lemma from (SL97-14) with p = 3, a = 22 , k = m, α = 1, and β = r. 4.31 Shortlisted Problems 1990 547 Since 32k divides n2 | 22n − 1, we can apply the lemma to m = 2n and r = 2k to conclude that 32k−1 | n = 3k d. Hence k = 1. Finally, let us assume d > 1 and let q be the smallest prime factor of d. Obviously q is odd, q ≥ 5, and (n, q − 1) ∈ {1, 3}. We then have q | 22n − 1 and q | 2q−1 − 1. Consequently, q | 2(2n,q−1) − 1 = 22(n,q−1) − 1, which divides 26 −1 = 63 = 32 ·7, so we must have q = 7. However, in that case we obtain 7 | n | 2n + 1, which is a contradiction, since powers of two can only be congruent to 1,2 and 4 modulo 7. It thus follows that d = 1 and n = 3. Hence n > 1 ⇒ n = 3. It is easily verified that n = 1 and n = 3 are indeed solutions. Hence these are the only solutions. 24. Let us denote A = b + c + d, B = a + c + d, C = a + b + d, D = a + b + c. Since ab + bc + cd + da = 1 the numbers A, B,C, D are all positive. By trivially applying the AM-GM inequality we have: a2 + b2 + c2 + d 2 ≥ ab + bc + cd + da = 1 . We will prove the inequality assuming only that A, B,C, D are positive and a2 + b2 + c2 + d 2 ≥ 1. In this case we may assume without loss of generality that a ≥ b ≥ c ≥ d ≥ 0. Hence a3 ≥ b3 ≥ c3 ≥ d 3 ≥ 0 and A1 ≥ B1 ≥ C1 ≥ D1 > 0. Using the Chebyshev and Cauchy inequalities we obtain: a3 b3 c 3 d 3 + + + A B C D   1 3 1 1 1 1 3 3 3 ≥ (a + b + c + d ) + + + 4 A B C D   1 2 1 1 1 1 2 2 2 ≥ (a + b + c + d )(a + b + c + d) + + + 16 A B C D   1 2 1 1 1 1 1 2 2 2 = (a + b + c + d )(A + B + C + D) + + + ≥ . 48 A B C D 3 This completes the proof. 25. Plugging in x = 1 we get f ( f (y)) = f (1)/y and hence f (y1 ) = f (y2 ) implies y1 = y2 i.e. that the function is bijective. Plugging in y = 1 gives us f (x f (1)) = f (x) ⇒ x f (1) = x ⇒ f (1) = 1. Hence f ( f (y)) = 1/y. Plugging in y = f (z) implies 1/ f (z) = f (1/z). Finally setting y = f (1/t) into the original equation gives us f (xt) = f (x)/ f (1/t) = f (x) f (t). Conversely, any functional equation on Q+ satisfying (i) f (xt) = f (x) f (t) and (ii) f ( f (x)) = 1x for all x,t ∈ Q+ also satisfies the original functional equation: f (x f (y)) = f (x) f ( f (y)) = f (x) y . Hence it suffices to find a function satisfying (i) and (ii). We note that all elements q ∈ Q+ are of the form q = ∏ni=1 pai i where pi are prime and ai ∈ Z. The criterion (i) implies f (q) = f (∏ni=1 pai i ) = ∏ni=1 f (pi )ai . Thus it is sufficient to define the function on all primes. For the function to satisfy (ii) it is necessary and sufficient for it to satisfy f ( f (p)) = 1p for all primes p. Let qi denote the i-th smallest prime. We define our function f as follows: 548 4 Solutions f (q2k−1 ) = q2k , f (q2k ) = 1 , k∈N. q2k−1 Such a function clearly satisfies (ii) and along with the additional condition f (xt) = f (x) f (t) it is well defined for all elements of Q+ and it satisfies the original functional equation. 26. We note that |P(x)/x| → ∞. Hence, there exists an integer number M such that M > |q1 | and |P(x)| ≤ |x| ⇒ |x| < M. It follows that |qi | < M for all i ∈ N because assuming |qi | ≥ M for some i we get |qi−1 | = |P(qi )| > |qi | ≥ M and this ultimately contradicts |q1 | < M. 3 2 Let us define q1 = rs and P(x) = ax +bxe +cx+d where r, s, a, b, c, d, e are all integers. For N = sa we shall prove by induction that Nqi is an integer for all i ∈ N. By definition N 6= 0. For i = 1 this obviously holds. Assume it holds for some i ∈ N. Then using qi = P(qi+1 ) we have that Nqi+1 is a zero of the polynomial  e  x Q(x) = N 3 P − qi a N = x3 + (sb)x2 + (s2 ac)x + (s3 a2 d − s2 ae(Nqi )) . Since Q(x) is a monic polynomial with integer coefficients (a conclusion for which we must assume the induction hypothesis) and Nqi+1 is rational it follows by the rational root theorem that Nqi+1 is an integer. It follows that all qi are multiples of 1/N. Since −M < qi < M we conclude that qi can take less than T = 2M|N| distinct values. Therefore for each j there are m j and m j + k j (k j > 0) both belonging to the set { jT + 1, jT + 2, . . . , jT + T } such that qm j = qm j +k j . Since k j < T for all k j it follows that there exists a positive integer k which appears an infinite number of times in the sequence k j , i.e. there exist infinitely many integers m such that qm = qm+k . Moreover, qm = qm+k clearly implies qn = qn+k for all n ≤ m. Hence qn = qn+k holds for all n. 27. Let us denote by An (k) the n-digit number which consists of n − 1 ones and one digit seven in the k + 1-th rightmost position (0 ≤ k < n). Then An (k) = (10n + 54 · 10k − 1)/9. We note that if 3 | n we have that 3 | An (k) for all k. Hence n cannot be divisible by 3. Now let 3 ∤ n. We claim that for each such n ≥ 5, there exists k < n for which 7 | An (k). We see that An (k) is divisible by 7 if and only if 10n − 1 ≡ 2 · 10k (mod 7). There are several cases. n ≡ 1 (mod 6). Then 10n − 1 ≡ 2 ≡ 2 · 100, so 7 | An (0). n ≡ 2 (mod 6). Then 10n − 1 ≡ 1 ≡ 2 · 104, so 7 | An (4). n ≡ 4 (mod 6). Then 10n − 1 ≡ 3 ≡ 2 · 105, so 7 | An (5). n ≡ 5 (mod 6). Then 10n − 1 ≡ 4 ≡ 2 · 102, so 7 | An (2). The remaining cases are n = 1, 2, 4. For n = 4 the number 1711 = 29 · 59 is composite, while it is easily checked that n = 1 and n = 2 are solutions. Hence the answer is n = 1, 2. 4.31 Shortlisted Problems 1990 549 28. Let us first prove the following lemma. Lemma. Let (b′ /a′ , d ′ /c′ ) and (b′′ /a′′ , d ′′ /c′′ ) be two points with rational coordinates where the fractions given are irreducible. If both a′ and c′ are odd and the distance between the two points is 1 then it follows that a′′ and c′′ are odd, and that b′ + d ′ and b′′ + d ′′ are of a different parity. Proof. Let b/a and d/c be irreducible fractions such that b′ /a′ − b′′ /a′′ = b/a and d ′ /c′ − d ′′ /c′′ = d/c. Then it follows that b2 /a2 + d 2 /c2 = 1 ⇒ b2 c2 + a2 d 2 = a2 c2 . Since (a, b) = 1 and (c, d) = 1 it follows that a | c, c | a and hence a = c. Consequently b2 + d 2 = a2 . Since a is mutually co-prime to b and d it follows that a and b + d are odd. From b′′ /a′′ = b/a + b′/a′ we get that a′′ | aa′ , so a′′ is odd. Similarly, c′′ is odd as well. Now it follows that b′′ ≡ b + b′ and similarly d ′′ ≡ d + d ′ (mod 2). Hence b′′ + d ′′ ≡ b′ + d ′ + b + d ≡ b′ + d ′ + 1 (mod 2), from which it follows that b′ + d ′ and b′′ + d ′′ are of a different parity. Without loss of generality we start from the origin of the coordinate system (0/1, 0/1). Initially b + d = 0 and after moving to each subsequent point along the broken line b + d changes parity by the lemma. Hence it will not be possible to return to the origin after an odd number of steps since b + d will be odd. 550 4 Solutions 4.32 Solutions to the Shortlisted Problems of IMO 1991 1. All the angles ∠PP1C, ∠PP2C, ∠PQ1C, ∠PQ2C are right, hence P1 , P2 , A Q1 , Q2 lie on the circle with diameter PC. The result now follows immediately from Pascal’s theorem apP1 plied to the hexagon P1 PP2 Q1CQ2 . It X Q2 tells us that the points of intersection P of the three pairs of lines P1C, PQ1 (intersection A), P1 Q2 , P2 Q1 (intersection B P2 Q1 C X ) and PQ2 , P2C (intersection B) are collinear. 2. Let HQ meet PB at Q′ and HR meet PC at R′ . From MP = MB = MC A we have ∠BPC = 90o . So PR′ HQ′ is a rectangle. Since PH is perpendicular X Y to BC, it follows that the circle with diameter PH, through P, R′ , H, Q′ , is tanP gent to BC. It is now sufficient to show R ′ ′ that QR is parallel to Q R . Let CP meet Q ′ R Q′ AB at X , and BP meet AC at Y . Since P is on the median, it follows (for example, by Ceva’s theorem) that AX/X B = H M B C AY /YC, i.e. that XY is parallel to BC. Therefore, PY /BP = PX/CP. Since HQ is parallel to CX, we have QQ′ /HQ′ = PX/CP and similarly RR′ /HR′ = PY /BP. It follows that QQ′ /HQ′ = RR′ /HR′ , hence QR is parallel to Q′ R′ as required. Second solution. It suffices to show that ∠RHC = ∠RQH, or equivalently RH : QH = PC : PB. We assume PC : PB = 1 : x. Let X ∈ AB and Y ∈ AC be points such that MX ⊥ PB and MY ⊥ PC. Since MX bisects ∠AMB and MY bisects AMC, we deduce AX : XB = AM : MB = AY : YC ⇒ XY k BC ⇒ ⇒ △XY M ∼ △CBP ⇒ X M : MY = 1 : x. Now from CH : HB = 1 : x2 we obtain RH : MY = CH : CM = 1 : QH : MX = BH : BM = x2 : 1+x2 1+x2 2 and . Therefore 2 2x2 RH : QH = MY : MX = 1 : x. 2 1+x 1 + x2 2 3. Consider the problem with the unit circle on the complex plane. For convenience, we use the same letter for a point in the plane and its corresponding complex number. Lemma 1. Line l(S, PQR) contains the point Z = P+Q+R+S . 2 4.32 Shortlisted Problems 1991 551 Proof. Suppose P′ , Q′ , R′ are the feet of perpendiculars from S to QR, RP, PQ respectively. It suffices to show that P′ , Q′ , R′ , Z are on the same line. Let us  ′  ′ −Q −Q first represent P′ by Q, R, S. Since P′ ∈ QR, we have PR−Q = PR−Q , that is, (P′ − Q)(R − Q) = (P′ − Q)(R − Q). (1) ′ −S On the other hand, since SP′ ⊥ QR, the ratio PR−Q is purely imaginary. Thus ′ (P − S)(R − Q) = −(P′ − S)(R − Q). (2) Eliminating P′ from (1) and (2) and using the fact that X = X −1 for X on the unit circle, we obtain P′ = (Q + R + S − QR/S)/2 and analogously Q′ = (P + R + S − PR/S)/2 and R′ = (P + Q + S − PQ/S)/2. Hence Z − P′ = (P + QR/S)/2, Z − Q′ = (Q + PR/S)/2 and Z − R′ = (R +PQ/S)/2.  ps qr Setting P = p2 , Q = q2 , R = r2 , S = s2 we obtain Z − P′ = pqr + 2s qr ps ,     qs pqr rs pq pr ′ Z − Q′ = pqr 2s pr + qs and Z − P = 2s pq + rs . Since x + x−1 = 2Re x is real for all x on the unit circle, it follows that the ratio of every pair of these differences is real, which means that Z, P′ , Q′ , R′ belong to the same line. Lemma 2. If P, Q, R, S are four different points on a circle, then the lines l(P, QRS), l(Q, RSP), l(R, SPQ), l(S, PQR) intersect at one point. Proof. By Lemma 1, they all pass through P+Q+R+S . 2 Now we can find the needed conditions for A, B, . . . , F. In fact, the lines l(A, BDF), l(D, ABF) meet at Z1 = A+B+D+F , and l(B, ACE), l(E, ABC) meet 2 at Z2 = A+B+C+E . Hence, Z ≡ Z if and only if D − C = E − F ⇔ CDEF is a 1 2 2 rectangle. Remark. The line l(S, PQR) is widely known as Simson’s line; the proof that the feet of perpendiculars are collinear is straightforward. The key claim, Lemma 1, is a known property of Simson’s lines, and can be shown elementarily: ∗ l(S, PQR) passes through the midpoint X of HS, where H is the orthocenter of PQR. 4. Assume the contrary, that ∠MAB, ∠MBC, ∠MCA are all greater than 30◦ . By the sine Ceva theorem, it holds that sin ∠MAC sin ∠MBA sin ∠MCB = sin ∠MAB sin ∠MBC sin ∠MCA > sin3 30◦ = 1 . 8 (∗) On the other hand, since ∠MAC + ∠MBA + ∠MCB < 180◦ − 3 · 30◦ = 90◦ , Jensen’s inequality applied on the concave function ln sin x (x ∈ [0, π ]) gives us sin ∠MAC sin ∠MBA sin ∠MCB < sin3 30◦ , contradicting (∗). Second solution. Denote the intersections of PA, PB, PC with BC,CA, AB by A1 , B1 ,C1 , respectively. Suppose that each of the angles ∠PAB, ∠PBC, ∠PCA is greater than 30o and denote PA = 2x, PB = 2y, PC = 2z. Then PC1 > x, PA1 > y, PB1 > z. On the other hand, we know that 552 4 Solutions PC1 PA1 PB1 SABP SPBC SAPC + + = + + = 1. PC + PC1 PA + PA1 PB + PB1 SABC SABC SABC y t x z Since the function p+t is increasing, we obtain 2z+x + 2x+y + 2y+z < 1. But on the contrary, Cauchy-Schwartz inequality (or alternatively Jensen’s inequality) yields x y z (x + y + z)2 + + ≥ = 1. 2z + x 2x + y 2y + z x(2z + x) + y(2x + y) + z(2y + z) 5. Let P1 be the point on the side BC such that ∠BFP1 = β /2. Then ∠BP1 F = sin(3β /2) BF 180o − 3β /2, and the sine law gives us BP = sin(β /2) = 3 − 4 sin2 (β /2) = 1 + 1 2 cos β . o o Now we calculate BF BP . We have ∠BIF = 120 − β /2, ∠BFI = 60 and ∠BIC = o o 120 , ∠BCI = γ /2 = 60 − β /2. By the sine law, BF = BI sin(120o − β /2) , sin 60o 1 sin 120o BP = BC = BI . 3 3 sin(60o − β /2) 3 sin(60o −β /2) sin(60o +β /2) This implies BF BP = sin2 60o 2(cos β − cos120o ) = 2 cos β + 1 = BF BP1 . = 4 sin(60o − β /2) sin(60o + β /2) = Therefore P ≡ P1 . 6. Let a, b, c be sides of the triangle. Let A1 be the intersection of line AI with BC. ac By the known fact, BA1 : A1C = c : b and AI : IA1 = AB : BA1 , hence BA1 = b+c AI AB b+c AI b+c and IA1 = BA1 = a . Consequently lA = a+b+c . Put a = n + p, b = p + m, c = m + n: it is obvious that m, n, p are positive. Our inequality becomes 2< (2m + n + p)(m + 2n + p)(m + n + 2p) 64 ≤ . (m + n + p)3 27 The right side inequality immediately follows from the inequality between arithmetic and geometric means applied on 2m + n + p, m + 2n + p and m + n + 2p. For the left side inequality, denote by T = m + n + p. Then we can write (2m + n + p)(m + 2n + p)(m + n + 2p) = (T + m)(T + n)(T + p) and (T + m)(T + n)(T + p) = T 3 + (m + n + p)T 2 + (mn + np + pn)T + mnp > 2T 3 . Remark. The inequalities cannot be improved. In fact, AI·BI·CI lA lB lC is equal to 8/27 for a = b = c, while it can be arbitrarily close to 1/4 if a = b and c is sufficiently small. 7. The given equations imply AB = CD, AC = BD, AD = BC. Let L1 , M1 , N1 be the midpoints of AD, BD,CD respectively. Then the above equalities yield L1 M1 = AB/2 = LM, L1 M1 k AB k LM; L1 M = CD/2 = LM1 , L1 M k CD k LM1 . 4.32 Shortlisted Problems 1991 553 D Thus L, M, L1 , M1 are coplanar and LML1 M1 is a rhombus as well as N1 MNM1 N1 and LNL1 N1 . Then the segM1 ments LL1 , MM1 , NN1 have the comL1 mon midpoint Q and QL ⊥ QM, Q M QL ⊥ QN, QM ⊥ QN. We also inC L fer that the line NN1 is perpendicular N B A to the plane LML1 M1 and hence to the line AB. Thus QA = QB, and similarly, QB = QC = QD, hence Q is just the center O, and ∠LOM = ∠MON = ∠NOL = 90◦ . 8. Let P1 (x1 , y1 ), P2 (x2 , y2 ), . . . , Pn (xn , yn ) be the n points of S in the coordinate plane. We may assume x1 < x2 < · · · < xn (choosing adequate axes and renumbering the points if necessary). Define d to be half the minimum distance of Pi from the line Pj Pk , where i, j, k go through all possible combinations of mutually distinct indices. First we define a set T containing 2n − 4 points: T = {(xi , yi − d), (xi , yi + d) | i = 2, 3, . . . , n − 1}. Consider any triangle Pk Pl Pm , where k < l < m. Its interior contains at least one of the two points (xl , yl ± d), so T is a set of 2n − 4 points with the required property. However, at least one of the points of T is useless. The convex hull of S is a polygon with at least three points in S as vertices. Let Pj be a vertex of that hull distinct from P1 and Pn . Clearly one of the points (x j , y j ± d) lies outside the convex hull, and thus can be left out. The remaining set of 2n − 5 points satisfies the conditions. 9. Let A1 , A2 be two points of E which are joined. In E \ {A1 , A2 }, there are at most 397 points to which A1 is not joined, and at most as much to which A2 is not joined. Consequently, there exists a point A3 which is joined to both A1 and A2 . There are at most 3 · 397 = 1191 points of E \ {A1 , A2 , A3 } to which at least one of A1 , A2 , A3 is not joined, hence it is possible to choose a point A4 joined to A1 , A2 , A3 . Similarly, there exists a point A5 which is joined to all A1 , A2 , A3 , A4 . Finally, among the remaining 1986 points, there are at most 5· 397 = 1985 which are not joined to one of the points A1 , . . . , A5 . Thus there is at least one point A6 joined to all A1 , . . . , A5 . It is clear that A1 , . . . , A6 are pairwise joined. Solution of the alternative version. Let be given 1991 points instead. Number the points from 1 to 1991, and join i and j if and only if i − j is not a multiple of 5. Then each i is joined to 1592 or 1593 other points, and obviously among any six points there are two which are not joined. 10. We start at some vertex v0 and walk along distinct edges of the graph, numbering them 1, 2, . . . in the order of appearance, until this is no longer possible without reusing an edge. If there are still edges which are not numbered, one of them has a vertex which has already been visited (else G would not be connected). Starting from this vertex, we continue to walk along unused edges resuming the 554 4 Solutions numbering, until we eventually get stuck. Repeating this procedure as long as possible, we shall number all the edges. Let v be a vertex which is incident with e ≥ 2 edges. If v = v0 , then it is on the edge 1, so the gcd at v is 1. If v 6= v0 , suppose that it was reached for the first time by the edge r. At that time there was at least one unused edge incident with v (as e ≥ 2), hence one of them was labelled by r + 1. The gcd at v again equals gcd(r, r + 1) = 1. h i  n−m n−m−1 1 11. To start with, observe that n−m = 1n n−m . m m + m−1  [n/2] m m−1 For n = 1, 2, . . . set Sn = ∑m=0 (−1)m n−m + m . Using the identity k = k  m−1 we obtain the following relation for S : n k−1   n−m+1 Sn+1 = ∑(−1)m m m    n − m m m n−m = ∑(−1) + ∑(−1) = Sn − Sn−1. m m−1 m m Since the initial members of the sequence Sn are 1, 1, 0, −1, −1, 0, 1, 1, . . ., we thus find that Sn is periodic with period 6. Now the sum from the problem reduces to           1 1991 1 1990 1989 1 996 995 − + + ···− + 1991 0 1991 1 0 1991 995 994 1 1 1 (S1991 − S1989) = (0 − (−1)) = . 1991 1991 1991 12. Let Am be the set of those elements of S which are divisible by m. By the inclusion-exclusion principle, the number of elements divisible by 2, 3, 5 or 7 equals = |A2 ∪ A3 ∪ A5 ∪ A7 | = |A2 | + |A3 | + |A5| + |A7| − |A6 | − |A10| − |A14| − |A15| −|A21 | − |A35| + |A30| + |A42| + |A70| + |A105| − |A210| = 140 + 93 + 56 + 40 − 46 − 28 − 20 − 18 −13 − 8 + 9 + 6 + 4 + 2 − 1 = 216. Among any five elements of the set A2 ∪A3 ∪A5 ∪A7 , one of the sets A2 , A3 , A5 , A7 contains at least two, and those two are not relatively prime. Therefore n > 216. We claim that the answer is n = 217. First notice that the set A2 ∪ A3 ∪ A5 ∪ A7 consists of four prime (2, 3, 5, 7) and 212 composite numbers. The set S \ A contains exactly 8 composite numbers: namely, 112 , 11 · 13, 11 · 17, 11 · 19, 11 · 23, 132, 13 · 17, 13 · 19. Thus S consists of the unity, 220 composite numbers and 59 primes. Let A be a 217-element subset of S, and suppose that there are no five pairwise relatively prime numbers in A. Then A can contain at most 4 primes (or unity and three primes) and at least 213 composite numbers. Hence the set S \ A contains 4.32 Shortlisted Problems 1991 555 at most 7 composite numbers. Consequently, at least one of the following 8 fiveelement sets is disjoint with S \ A, and is thus entirely contained in A: {2 · 23, 3 · 19, 5 · 17, 7 · 13, 11 · 11}, {2 · 31, 3 · 29, 5 · 23, 7 · 19, 11 · 17}, {2 · 41, 3 · 37, 5 · 31, 7 · 29, 11 · 23}, {2 · 47, 3 · 43, 5 · 41, 7 · 37, 13 · 19}, {2 · 29, 3 · 23, 5 · 19, 7 · 17, 11 · 13}, {2 · 37, 3 · 31, 5 · 29, 7 · 23, 11 · 19}, {2 · 43, 3 · 41, 5 · 37, 7 · 31, 13 · 17}, {2 · 2, 3 · 3, 5 · 5, 7 · 7, 13 · 13}. As each of these sets consists of five numbers relatively prime in pairs, the claim is proved. 13. Call a sequence e1 , . . . , en good if e1 a1 + · · · + en an is divisible by n. Among the sums s0 = 0, s1 = a1 , s2 = a1 + a2 , . . . , sn = a1 + · · · + an , two give the same remainder modulo n, and their difference corresponds to a good sequence. To show that, permuting the ai ’s, we can find n − 1 different sequences, we use the following Lemma. Let A be a k × n (k ≤ n − 2) matrix of zeros and ones, whose every row contains at least one 0 and at least two 1’s. Then it is possible to permute columns of A is such a way that in any row 1’s do not form a block. Proof. We will use the induction on k. The case k = 1 and arbitrary n ≥ 3 is trivial. Suppose that k ≥ 2 and that for k − 1 and any n ≥ k + 1 the lemma is true. Consider a k × n matrix A, n ≥ k + 2. We mark an element ai j if either it is the only zero in the i-th row, or one of the 1’s in the row if it contains exactly two 1’s. Since n ≥ 4, every row contains at most two marked elements, which adds up to at most 2k < 2n marked elements in total. It follows that there is a column with at most one marked element. Assume w.l.o.g. that it is the first column and that a1 j isn’t marked for j > 1. The matrix B, obtained by omitting the first row and first column from A, satisfies the conditions of the lemma. Therefore, we can permute columns of B and get the required form. Considered as a permutation of column of A, this permutation may leave a block of 1’s only in the first row of A. In the case that it is so, if a11 = 1 we put the first column in the last place, otherwise we put it between any two columns having 1’s in the first row. The obtained matrix has the required property. Suppose now that we have got k different nontrivial good sequences ei1 , . . . , ein , i = 1, . . . , k, and that k ≤ n − 2. The matrix A = (eij ) fulfills the conditions of Lemma, hence there is a permutation σ from Lemma. Now among the sums s0 = 0, s1 = aσ (1) , s2 = aσ (1) + aσ (2) , . . . , sn = aσ (1) + · · · + aσ (n), two give the same remainder modulo n. Let s p ≡ sq (mod n), p < q. Then n | sq − s p = aσ (p+1) + · · · + aσ (q), and this yields a good sequence e1 , . . . , en with eσ (p+1) = · · · = eσ (q) = 1 and other e’s equal to zero. Since from the construction we see that none of the sequences eσ ( j)i has all 1’s in a block, in this way we have got a new nontrivial good sequence, and we can continue this procedure until there are n − 1 sequences. Together with the trivial 0, . . . , 0 sequence, we have found n good sequences. 556 4 Solutions 14. Suppose that f (x0 ), f (x0 + 1), . . . , f (x0 + 2p − 2) are squares. If p | a and p ∤ b, then f (x) ≡ bx + c (mod p) for x = x0 , . . . , x0 + p − 1 form a complete system of residues modulo p. However, a square is always congruent to exactly one of the p+1 p−1 2 2 2 2 numbers 0, 1 , 2 , . . . , ( 2 ) and thus cannot give every residue modulo p. Also, if p | a and p | b, then p | b2 − 4ac. We now assume p ∤ a. The following identities hold for any quadric polynomial: and 4a · f (x) = (2ax + b)2 − (b2 − 4ac) (1) f (x + p) − f (x) = p(2ax + b) + p2a. (2) Suppose that there is an y, x0 ≤ y ≤ x0 + p − 2, for which f (y) is divisible by p. Then both f (y) and f (y + p) are squares divisible by p, and therefore both are divisible by p2 . But relation (2) implies that p | 2ay + b, and hence by (1) b2 − 4ac is divisible by p as well. Therefore it suffices to show that such an y exists, and for that aim we prove that there are two such y in [x0 , x0 + p − 1]. Assume the opposite. Since for x = x0 , x0 + 1, . . . , x0 + p − 1 f (x) is congruent modulo p to one of the p−1 2 num 2 bers 12 , 22 , . . . , p−1 , it follows by the pigeon-hole principle that for some mu2 tually distinct u, v, w ∈ {x0 , . . . , x0 + p − 1} we have f (u) ≡ f (v) ≡ f (w) (mod p). Consequently the difference f (u) − f (v) = (u − v)(a(u + v) + b) is divisible by p, but it is clear that p ∤ u − v, hence a(u + v) ≡ −b (mod p). Similarly a(u + w) ≡ −b (mod p), which together with the previous congruence yields p | a(v − w) ⇒ p | v − w which is clearly impossible. It follows that p | f (y1 ) for at least one y1 , x0 ≤ y1 < x0 + p. If y2 , x0 ≤ y2 < x0 + p is such that a(y1 + y2 ) + b ≡ 0 (mod p), we have p | f (y1 ) − f (y2 ) ⇒ p | f (y2 ). If y1 = y2 , then by (1) p | b2 − 4ac. Otherwise, among y1 , y2 one belongs to [x0 , x0 + p − 2] as required.   Second solution. Using Legendre’s symbols ap for quadratic residues we can prove a stronger statement for p ≥ 5. It can be shown that p−1  ∑ x=0 ax2 + bx + c p  =−   a p if p ∤ b2 − 4ac, 2 hence for at most p+3 2 values of x between x0 and x0 + p − 1 inclusive, ax + bx + c is a quadratic residue or 0 modulo p. Therefore, if p ≥ 5 and f (x) is a square 2 for p+5 2 consecutive values, then p | b − 4ac. 15. Assume that the sequence has the period T . We can find integers k > m > 0, as large as we like, such that 10k ≡ 10m (mod T ), using for example Euler’s theorem. It is obvious that a10k −1 = a10k and hence, taking k sufficiently large and using the periodicity, we see that a2·10k −10m −1 = a10k −1 = a10k = a2·10k −10m . 4.32 Shortlisted Problems 1991 557 Since (2 · 10k − 10m)! = (2 · 10k − 10m )(2 · 10k − 10m − 1)! and the last nonzero digit of 2 ·10k − 10m is nine, we must have a2·10k −10m −1 = 5 (if s is a digit, the last digit of 9s is s only if s = 5). But this means that 5 divides n! with a greater power than 2 does, which is impossible. Indeed, if the exponents of these powers are α2 , α5 respectively, then α5 = [n/5] + [n/52] + · · · ≤ α2 = [n/2] + [n/22] + · · ·. 16. Let p be the least prime number that does not divide n: thus a1 = 1 and a2 = p. Since a2 − a1 = a3 − a2 = · · · = r, the ai ’s are 1, p, 2p − 1, 3p − 2, . . .. We have the following cases: p = 2. Then r = 1 and the numbers 1, 2, 3, . . . , n − 1 are relatively prime to n, hence n is a prime. p = 3. Then r = 2, so every odd number less than n is relatively prime to n, from which we deduce that n has no odd divisors. Therefore n = 2k for some k ∈ N. p > 3. Then r = p − 1 and ak+1 = a1 + k(p − 1) = 1 + k(p − 1). Since n − 1 also must belong to the progression, we have p − 1 | n − 2. Let q be any prime divisor of p − 1. Then also q | n − 2. On the other hand, since q < p, it must divide n too, therefore q | 2, i.e. q = 2. This means that p − 1 has no prime divisors other than 2 and thus p = 2l + 1 for some l ≥ 2. But in order for p to be prime, l must be even (because 3 | 2l + 1 for l odd). Now we recall that 2p − 1 is also relatively prime to n; but 2p − 1 = 2l+1 + 1 is divisible by 3, which is a contradiction because 3 | n. 17. Taking the equation 3x + 4y = 5z (x, y, z > 0) modulo 3, we get that 5z ≡ 1 (mod 3), hence z is even, say z = 2z1 . The equation then becomes 3x = 52z1 − 4y = (5z1 − 2y )(5z1 + 2y ). Each factor 5z1 − 2y and 5z1 + 2y is a power of 3, for which the only possibility is 5z1 + 2y = 3x and 5z1 − 2y = 1. Again modulo 3 these equations reduce to (−1)z1 + (−1)y = 0 and (−1)z1 − (−1)y = 1, implying that z1 is odd and y is even. Particularly, y ≥ 2. Reducing the equation 5z1 + 2y = 3x modulo 4 we get that 3x ≡ 1, hence x is even. Now if y > 2, modulo 8 this equation yields 5 ≡ 5z1 ≡ 3x ≡ 1, a contradiction. Hence y = 2, z1 = 1. The only solution of the original equation is x = y = z = 2. 18. For integers a > 0, n > 0 and α ≥ 0, we shall write aα k n when aα | n and aα +1 ∤ n. Lemma. For every odd number a ≥ 3 and an integer n ≥ 0 it holds that n an+1 k (a + 1)a − 1 and n an+1 k (a − 1)a + 1. Proof. We shall prove the first relation by induction (the second is analogous). For n = 0 the statement is obvious. Suppose that it holds for some n, i.e. n that (1 + a)a = 1 + Nan+1, a ∤ N. Then   a 2 2n+2 an+1 n+1 a n+1 (1 + a) = (1 + Na ) = 1 + a · Na + N a + Ma3n+3 2 558 4 Solutions  for some integer M. Since a2 is divisible by a for a odd, we deduce that the part of the above sum behind 1 + a · Nan+1 is divisible by an+3. Hence n+1 (1 + a)a = 1 + N ′ an+2 , where a ∤ N ′ . It follows immediately from Lemma that 19911993 k 19901991 1992 +1 and 19911991 k 19921991 1990 − 1. Adding these two relations we obtain immediately that k = 1991 is the desired value. 19. Set x = cos(π a). The given equation is equivalent to 4x3 + 4x2 − 3x − 2 = 0, which factorizes as (2x + 1)(2x2 + x − 2) = 0. The case 2x + 1 = 0 yields cos(π a) = −1/2 and a = 2/3. It remains to show that if x satisfies 2x2 + x − 2 = 0 then a is not rational. The polynomial equation √ 2x2 + x − 2 = 0 has two real roots, x1,2 = −1±4 17 , and since |x| ≤ 1 we must have x = cos π a = √ −1+ 17 . 4 √ We now prove by induction that, for every integer n ≥ 0, cos(2n π a) = an +b4n 17 for some odd integers an , bn . The case n = 0 is trivial. Also, if cos(2n π a) = √ an +bn 17 , then 4 cos(2n+1 π a) = 2 cos2 (2n π a) − 1 √   √ 1 a2n + 17b2n − 8 an+1 + bn+1 17 = + an bn 17 = . 4 2 4 By the inductive step that an , bn are odd, it is obvious that an+1 , bn+1 are also odd. This proves the claim. Note also that, since an+1 = 12 (a2n + 17b2n − 8) > an , the sequence {an } is strictly n increasing. √ Hence the set of values of cos(2 π a), n = 0, 1, 2, . . ., is infinite (because 17 is irrational). However, if a were rational, then the set of values of cos mπ a, m = 1, 2, . . . , would be finite, a contradiction. Therefore the only possible value for a is 2/3. 20. We prove the result with 1991 replaced by any positive integer k. For natural numbers p, q, let ε = (α p − [α p])(α q − [α q]). Then 0 < ε < 1 and ε = α 2 pq − α (p[α q] + q[α p]) + [α p][α q]. Multiplying this equality by α − k and using α 2 = kα + 1, i.e. α (α − k) = 1, we get (α − k)ε = α (pq + [α p][α q]) − (p[α q] + q[α p] + k[α p][α q]). Since 0 < (α − k)ε < 1, we have [α (p ∗ q)] = p[α q] + q[α p] + k[α p][α q]. Now (p ∗ q) ∗ r = (p ∗ q)r + [α (p ∗ q)][α r] = = pqr + [α p][α q]r + [α q][α r]p + [α r][α p]q + k[α p][α q][α r]. Since the last expression is symmetric, the same formula is obtained for p ∗ (q ∗ r). 4.32 Shortlisted Problems 1991 559 21. The polynomial g(x) factorizes as g(x) = f (x)2 − 9 = ( f (x) − 3)( f (x) + 3). If one of the equations f (x) + 3 = 0 and f (x) − 3 = 0 has no integer solutions, then the number of integer solutions of g(x) = 0 clearly does not exceed 1991. Suppose now that both f (x) + 3 = 0 and f (x) − 3 = 0 have integer solutions. Let x1 , . . . , xk be distinct integer solutions of the former, and xk+1 , . . . , xk+l be distinct integer solutions of the latter equation. There exist monic polynomials p(x), q(x) with integer coefficients such that f (x) + 3 = (x − x1 )(x − x2 ) . . . (x − xk )p(x) and f (x) − 3 = (x − xk+1 )(x − xk+2 ) . . . (x − xk+l )q(x). Thus we obtain (x − x1 )(x − x2 ) . . . (x − xk )p(x) − (x − xk+1)(x − xk+2 ) . . . (x − xk+l )q(x) = 6. Putting x = xk+1 we get (xk+1 − x1 )(xk+1 − x2 ) · · · (xk+1 − xk ) | 6, and since the product of more than four distinct integers cannot divide 6, this implies k ≤ 4. Similarly l ≤ 4; hence g(x) = 0 has at most 8 distinct integer solutions. Remark. The proposer provided a solution for the upper bound of 1995 roots which was essentially the same as that of (IMO74-6). 22. Suppose w.l.o.g. that the center of the square is at the origin O(0, 0). We denote the curve y = f (x) = x3 + ax2 + bx + c by γ and the vertices of the square by A, B,C, D in this order. At first, the symmetry with respect to the point O maps γ into the curve γ (y = f (−x) = x3 − ax2 + bx − c). Obviously γ also passes through A, B,C, D, and thus has four different intersection points with γ . Then 2ax2 + 2c has at least four distinct solution, which implies a = c = 0. Particularly, γ passes through O and intersects all quadrants, and hence b < 0. Further, the curve γ ′ , obtained by rotation of γ around O for 90◦ , has an equation −x = f (y) and also contains the points A, B,C, D and O. The intersection points (x, y) of γ ∩ γ ′ are determined by −x = f ( f (x)), and hence they are roots of a polynomial p(x) = f ( f (x)) + x of 9-th degree. But the number of times that one cubic actually crosses the other in each quadrant is in the general case even (draw the picture!), and since ABCD is the only square lying on γ ∩ γ ′ , the intersection points A, B,C, D must be double. It follows that p(x) = x[(x − r)(x + r)(x − s)(x + s)]2, (1) where r, s are the x-coordinates of A and B. On the other hand, p(x) is defined by (x3 + bx)3 + b(x3 + bx) + x, and therefore equating of coefficients with (1) yields 3b = −2(r2 + s2 ), 3b2 = (r2 + s2 )2 + 2r2 s2 , b(b2 + 1) = −2r2 s2 (r2 + s2 ), b 2 + 1 = r 4 s4 . √ Straightforward solving this system of equations gives b = − 8 and r2 + s2 = √ 18. The line segment from O to (r, of the square, and thus a side ps) is half a diagonal √ of the square has length a = 2(r2 + s2 ) = 4 72. 23. From (i), replacing m by f ( f (m)), we get 560 4 Solutions analogously f ( f ( f (m)) + f ( f (n)) ) = − f ( f ( f ( f (m)) + 1)) − n; f ( f ( f (n)) + f ( f (m)) ) = − f ( f ( f ( f (n)) + 1)) − m. From these relations we get f ( f ( f ( f (m)) + 1)) − f ( f ( f ( f (n)) + 1)) = m − n. Again from (i), f ( f ( f ( f (m)) + 1)) = f (−m − f ( f (2)) ) and f ( f ( f ( f (n)) + 1)) = f (−n − f ( f (2)) ). Setting f ( f (2)) = k we obtain f (−m − k) − f (−n − k) = m − n for all integers m, n. This implies f (m) = f (0) − m. Then also f ( f (m)) = m, and using this in (i) we finally get f (n) = −n − 1 for all integers n. Particularly f (1991) = −1992. From (ii) we obtain g(n) = g(−n − 1) for all integers n. Since g is a polynomial, it must also satisfy g(x) = g(−x − 1) for all real x. Let us now express g as a polynomial on x + 1/2: g(x) = h(x + 1/2). Then h satisfies h(x + 1/2) = h(−x − 1/2), i.e. h(y) = h(−y), hence it is a polynomial in y2 ; thus g is a polynomial in (x + 1/2)2 = x2 + x + 1/4. Hence g(n) = p(n2 + n) (for some polynomial p) is the most general form of g. 24. Let yk = ak − ak+1 + ak+2 − · · · + ak+n−1 for k = 1, 2, . . . , n, where we define xi+n = xi for 1 ≤ i ≤ n. We then have y1 + y2 = 2a1 , y2 + y3 = 2a2 , . . . , yn + y1 = 2an . (i) Let n = 4k − 1 for some integer k > 0. Then for each i = 1, 2, . . . ,n we have that yi = (ai + ai+1 + · · · + ai−1 ) − 2(ai+1 + ai+3 + · · · + ai−2 )=1 + 2 + · · · + (4k − 1) − 2(ai+1 + ai+3 + · · · + ai−2 ) is even. Suppose now that a1 , . . . , an is a good permutation. Then each yi is positive and even, so yi ≥ 2. But for some t ∈ {1, . . . , n} we must have at = 1, and thus yt + yt+1 = 2at = 2 which is impossible. Hence the numbers n = 4k − 1 are not good. (ii) Let n = 4k + 1 for some integer k > 0. Then 2, 4, . . . , 4k, 4k + 1, 4k − 1, . . . , 3, 1 is a permutation with the desired property. Indeed, in this case y1 = y4k+1 = 1, y2 = y4k = 3, . . . , y2k = y2k+2 = 4k − 1, y2k+1 = 4k + 1. Therefore all nice numbers are given by 4k + 1, k ∈ N. 25. Since replacing x1 by 1 can only reduce the set of indices i for which the desired inequality holds, we may assume x1 = 1. Similarly we may assume xn = 0. Now we can let i be the largest index such that xi > 1/2. Then xi+1 ≤ 1/2, hence xi (1 − xi+1) ≥ 1 1 = x1 (1 − xn ). 4 4 26. Without loss of generality we can assume b1 ≥ b2 ≥ · · · ≥ bn . We denote by Ai the product a1 a2 . . . ai−1 ai+1 . . . an . If for some i < j holds Ai < A j , then bi Ai + b j A j ≤ bi A j + b j Ai (or equivalently (bi − b j )(Ai − A j ) ≤ 0). Therefore the sum ∑ni=1 bi Ai does not decrease when we rearrange the numbers a1 , . . . , an so that A1 ≥ · · · ≥ An , and consequently a1 ≤ · · · ≤ an . Further, for fixed ai ’s and ∑ bi = 4.32 Shortlisted Problems 1991 561 1, the sum ∑ni=1 bi Ai is maximal when b1 takes the largest possible value, i.e. b1 = p, b2 takes the remaining largest possible value b2 = 1 − p, whereas b3 = · · · = bn = 0. In this case n ∑ biAi = pA1 + (1 − p)A2 = a3 . . . an(pa2 + (1 − p)a1) i=1 ≤ p(a1 + a2 )a3 . . . an ≤ p , (n − 1)n−1 using the inequality between the geometric and arithmetic means for a3 , . . . , an , a1 + a2 . 27. Write F(x1 , . . . , xn ) = ∑i< j xi x j (xi +x j ). Choose an n-tuple (x1 , . . . , xn ), ∑ni=1 xi = 1, xi ≥ 0 with at least three nonzero components, and assume w.l.o.g. that x1 ≥ · · · ≥ xk−1 ≥ xk ≥ xk+1 = · · · = xn = 0. We claim that replacing xk−1 , xk with xk−1 + xk , 0 the value of F increases. Write for brevity xk−1 = a, xk = b. Then F(. . . , a + b, 0, 0, . . .) − F(. . . , a, b, 0, . . .) = k−2 k−2 i=1 i=1 ∑ xi (a + b)(xi + a + b) − ∑ [xi a(xi + a) + xib(xi + b)] − ab(a + b) k−2 ! = ab 2 ∑ xi − a − b = ab(2 − 3(a + b)) > 0, i=1 because xk−1 + xk ≤ 23 (x1 + xk−1 + xk−2 ) ≤ 23 . Repeating this procedure we can reduce the number of nonzero xi ’s to two, increasing the value of F in each step. It remains to maximize F over n-tuples (x1 , x2 , 0, . . . , 0) with x1 , x2 ≥ 0, x1 + x2 = 1: in this case F equals x1 x2 and attains its maximum value 14 when x1 = x2 = 12 , x3 = . . . , xn = 0. √ √ √ 28. Let √xn = c(n 2 − [n 2]) for some constant c > 0. For i > j, putting p = [i 2] − [ j 2], we have √ |2(i − j)2 − p2 |c c c √ √ |xi − x j | = c|(i − j) 2 − p| = ≥ ≥ , (i − j) 2 + p (i − j) 2 + p 4(i − j) √ because p < (i − j) 2 + 1. Taking c = 4, we obtain that for any i > j, (i − j)|xi − x j | ≥ 1. Of course, this implies (i − j)a |xi − x j | ≥ 1 for any a > 1. √ Remark. The constant 4 can be replaced with 3/2 + 2. Second solution. Another example of a sequence {xn } is constructed in the following way: x1 = 0, x2 = 1, x3 = 2 and x3k i+m = xm + 3ik for i = 1, 2 and 1 ≤ m ≤ 3k . It is easily shown that |i − j| · |xi − x j | ≥ 1/3 for any i 6= j. Third solution. If n = b0 + 2b1 + · · · + 2k bk , bi ∈ {0, 1}, then one can set xn a = b0 + 2−a b1 + · · · + 2−ka bk . In this case it holds that |i − j|a |xi − x j | ≥ 22a −2 −1 . 562 4 Solutions 29. One easily observes that the following sets are super-invariant: one-point set, its complement, closed and open half-lines or their complements, and the whole real line. To show that these are the only possibilities, we first observe that S is superinvariant if and only if for each a > 0 there is a b such that x ∈ S ⇔ ax + b ∈ S. (i) Suppose that for some a there are two such b’s: b1 and b2 . Then x ∈ S ⇔ ax + b1 ∈ S and x ∈ S ⇔ ax + b2 ∈ S, which implies that S is periodic: 2 y ∈ S ⇔ y + b1 −b ∈ S. Since S is identical to a translate of any stretching a of S, all positive numbers are periods of S. Therefore S ≡ R. (ii) Assume that, for each a, b = f (a) is unique. Then for any a1 and a2 , x ∈ S ⇔ a1 x + f (a1 ) ∈ S ⇔ a1 a2 x + a2 f (a1 ) + f (a2 ) ∈ S ⇔ a2 x + f (a2 ) ∈ S ⇔ a1 a2 x + a1 f (a2 ) + f (a1 ) ∈ S. As above it follows that a1 f (a2 )+ f (a1 ) = a2 f (a1 )+ f (a2 ), or equivalently f (a1 )(a2 − 1) = f (a2 )(a1 − 1). Hence (for some c), f (a) = c(a − 1) for all a. Now x ∈ S ⇔ ax+ c(a − 1) ∈ S actually means that y−c ∈ S ⇔ ay− c ∈ S for all a. Then it is easy to conclude that {y − c | y ∈ S} is either a half-line or the whole line, and so is S. 30. Let a and b be the integers written by A and B respectively, and let x < y be the two integers written by the referee. Suppose that none of A and B ever answers “yes”. Initially, regardless of a, A knows that 0 ≤ b ≤ y and answers “no”. In the second step, B knows that A obtained 0 ≤ b ≤ y, but if a were greater than x, A would know that a + b = y and would thus answer “yes”. So B concludes 0 ≤ a ≤ x but answers “no”. The process continues. Suppose that, in the n-th step, A knows that B obtained rn−1 ≤ a ≤ sn−1 . If b > x − rn−1 , B would know that a + b > x and hence a + b = y, while if b < y − sn−1 , B would know that a + b < y, i.e. a + b = x: in both cases he would be able to guess a. However, B answered “no”, from which A concludes y − sn−1 ≤ b ≤ x − rn−1 . Put rn = y − sn−1 and sn = x − rn−1. Similarly, in the next step B knows that A obtained rn ≤ b ≤ sn and, since A answered “no”, concludes y−sn ≤ a ≤ x−rn . Put rn+1 = y−sn and sn+1 = x−rn . Notice that in both cases si+1 − ri+1 = si − ri − (y − x). Since y − x > 0, there exists an m for which sm − rm < 0, a contradiction. 4.33 Shortlisted Problems 1992 563 4.33 Solutions to the Shortlisted Problems of IMO 1992 1. Assume that a pair (x, y) with x < y satisfies the required conditions. We claim 2 that the pair (y, x1 ) also satisfies the conditions, where x1 = y +m x (note that x1 > y is a positive integer). This will imply the desired result, since starting from the pair (1, 1) we can obtain arbitrarily many solutions. First, we show that gcd(x1 , y) = 1. Suppose to the contrary that gcd(x1 , y) = d > 1. Then d | x1 | y2 + m ⇒ d | m, which implies d | y | x2 + m ⇒ d | x. But this last is impossible, since gcd(x, y) = 1. Thus it remains to show that x1 | y2 + m and y | x21 + m. The former relation is obvious. Since gcd(x, y) = 1, the latter is equivalent to y | (xx1 )2 + mx2 = y4 + 2my2 + m2 + mx2 , which is true because y | m(m + x2 ) by the assumption. Hence (y, x1 ) indeed satisfies all the required conditions. Remark. The original problem asked to prove the existence of a pair (x, y) of positive integers satisfying the given conditions such that x + y ≤ m + 1. The problem in this formulation is trivial, since the pair x = y = 1 satisfies the conditions. Moreover, this is sometimes the only solution with x + y ≤ m + 1. For example, for m = 3 the least nontrivial solution is (x0 , y0 ) = (1, 4). 2. Let us define xn inductively as xn = f (xn−1 ), where x0 ≥ 0 is a fixed real number. It follows from the given equation in f that xn+2 = −axn+1 + b(a + b)xn . The general solution to this equation is of the form xn = λ1 bn + λ2 (−a − b)n, where λ1 , λ2 ∈ R satisfy x0 = λ1 + λ2 and x1 = λ1 b − λ2 (a + b). In order to have xn ≥ 0 for all n we must have λ2 = 0. Hence x0 = λ1 and f (x0 ) = x1 = λ1 b = bx0 . Since x0 was arbitrary, we conclude that f (x) = bx is the only possible solution of the functional equation. It is easily verified that this is indeed a solution. 3. Consider two squares AB′CD′ and A′ BC′ D. Since AC ⊥ BD, these two squares are homothetic, which implies that the lines AA′ , BB′ ,CC′ , DD′ are concurrent at a certain point O. F Since the rotation about A by 90◦ takes ∆ ABK into ∆ AFD, it follows that K BK ⊥ DF. Denote by T the intersecA tion of BK and DF. The rotation about E some point X by 90◦ maps BK into DF L T if and only if T X bisects an angle beD tween BK and DF. Therefore ∠FTA = ◦ B ∠AT K = 45 . Moreover, the quadriC A′ lateral BA′ DT is cyclic. Therefore ∠BTA′ = BDA′ = 45◦ and consequently that the points A, T, A′ are collinear. It follows that the point O lies on a bisector of ∠BT D and therefore the rotation R about O by 90◦ takes BK into DF. Analogously, R maps the lines CE, DG, AI into AH, BJ,CL. Hence the quadrilateral P1 Q1 R1 S1 is the image of the quadrilateral P2 Q2 R2 S2 , and the result follows. 564 4 Solutions 4. There are 36 possible edges in total. If not more than 3 edges are left undrawn, then we can choose 6 of the given 9 points no two of which are connected by an undrawn edge. These 6 points together with the edges between them form a two-colored complete graph, and thus by a well-known result there exists at least one monochromatic triangle. It follows that n ≤ 33. In order to show that n = 33, we shall give an example of a graph with 32 edges that does not contain a monochromatic triangle. Let us start with a complete graph C5 with 5 vertices. Its edges can be colored in two colors so that there is no monochromatic triangle (Fig. 1). Furthermore, given a graph H with k vertices without monochromatic triangles, we can add to it a new vertex, join it to all vertices of H except A, and color each edge BX in the same way as AX. The obtained graph obviously contains no monochromatic triangles. Applying this construction four times to the graph C5 we arrive to an example like that shown on Fig. 2. Fig. 1 Fig. 2 Second solution. For simplicity, we call the colors red and blue. Let r(k, l) be the least positive integer r such that each complete r-graph whose edges are colored in red and blue contains either a complete red k-graph or a complete blue l-graph. Also, let t(n, k) be the greatest possible number of edges in a graph with n vertices that does not contain a complete k-graph. These numbers exist by the theorems of Ramsey and Turán. Let us assume that r(k, l) < n. Every graph with n vertices and t(n, r(k, l)) +1 edges contains a complete subgraph with r(k, l) vertices, and this subgraph contains either a red complete k-graph or a blue complete l-graph. We claim that t(n, r(k, l)) + 1 is the smallest number of edges with the above property. By the definition of r(k, l) there exists a coloring of the complete graph H with r(k, l) − 1 vertices in two colors such that no red complete k-graph or blue complete l-graph exists. Let ci j be the color in which the edge (i, j) of H is colored, 1 ≤ i < j ≤ r(k, l) − 1. Consider a complete r(k, l) − 1-partite graph G with n vertices and exactly t(n, r(k, l)) edges and denote its partitions by Pi , i = 1, . . . , r(k, l) − 1. If we color each edge of H between Pi and Pj ( j < i) in the color ci j , we obviously obtain a graph with n vertices and t(n, r(k, l)) edges in two colors that contains neither a red complete k-graph nor a blue complete l-graph. Therefore the answer to our problem is t(9, r(3, 3)) + 1 = t(9, 6) + 1 = 33. 4.33 Shortlisted Problems 1992 565 5. Denote by K, L, M, and N the midpoints of the sides AB, BC,CD, and DA, respectively. The quadrilateral KLMN is a rhombus. We shall prove that O1 O3 k KM. Similarly, O2 O4 k LN, and the desired result follows immediately. −−−→ −−→ −−→ −−→ We have O1 O3 = KM + O1 K + MO3 . Assume that ABCD is positively ori√ ented. A rotational homothety R with angle −90◦ and coefficient 1/ 3 takes −→ −−→ −−→ −→ the vectors BK and CM into O1 K and MO3 respectively. Therefore −−−→ −−→ −−→ −−→ −−→ −→ −→ O1 O3 = KM + (O1 K + MO3 ) = KM + R(BK + CM) → −→ −−→ 1 − −−→ − → = KM + R(BA + CD) = KM + R(LN). 2 Since LN ⊥ KM, it follows that R(LN) is parallel to KM and so is O1 O3 . 6. It is easy to see that f is injective and surjective. From f (x2 + f (y)) = f ((−x)2 + f (y)) it follows that f (x)2 = ( f (−x))2 , which implies f (−x) = − f (x) because f is injective. Furthermore, there exists z ∈ R such that f (z) = 0. From f (−z) = − f (z) = 0 we deduce that z = 0. Now we√have f (x2 ) = f (x2 + f (0)) = 0 + ( f (x))2 = f (x)2 , and consequently f (x) = f ( x)2 > 0 for all x > 0. It also follows that f (x) < 0 for x < 0. In other words, f preserves sign. Now setting x > 0 and y = − f (x) in the given functional equation we obtain √ 2 √ f (x − f (x)) = f ( x + f (−x)) = −x + f ( x)2 = −(x − f (x)). But since f preserves sign, this implies that f (x) = x for x > 0. Moreover, since f (−x) = − f (x), it follows that f (x) = x for all x. It is easily verified that this is indeed a solution. 7. Let G1 , G2 touch the chord BC at P, Q and touch the circle G at R, S respectively. Let D be the midpoint of the complementary arc BC of G. The homothety centered at R mapping G1 onto G also maps the line BC onto a tangent of G parallel to BC. It follows that this line touches G at point D, which is therefore the image of P under the homothety. Hence R, P, and D are collinear. Since ∠DBP = ∠DCB = ∠DRB, it follows that △DBP ∼ △DRB and consequently that DP · DR = DB2 . Similarly, points S, Q, D are collinear and satisfy DQ · DS = DB2 = DP · DR. Hence D lies on the radical axis of the circles G1 and G2 , i.e., on their common tangent AW , which also implies that AW bisects the √ angle BAD. Furthermore, since DB = DC = DW = DP · DR, it follows from the lemma of (SL99-14) that W is the incenter of △ABC. Remark. According to the third solution of (SL93-3), both PW and QW contain the incenter of △ABC, and the result is immediate. The problem can also be solved by inversion centered at W . 8. For simplicity, we shall write n instead of 1992. Lemma. There exists a tangent n-gon A1 A2 . . . An with sides A1 A2 = a1 , A2 A3 = a2 , . . . , An A1 = an if and only if the system x1 + x2 = a1 , x2 + x3 = a2 , , . . . , xn + x1 = an (1) 566 4 Solutions has a solution (x1 , . . . , xn ) in positive reals. Proof. Suppose that such an n-gon A1 A2 . . . An exists. Let the side Ai Ai+1 touch the inscribed circle at point Pi (where An+1 = A1 ). Then x1 = A1 Pn = A1 P1 , x2 = A2 P1 = A2 P2 , . . . , xn = An Pn−1 = An Pn is clearly a positive solution of the system (1). Now suppose that the system (1) has a positive real solution (x1 , . . . , xn ). Let us draw a polygonal line A1 A2 . . . An+1 touching a circle of radius r at points P1 , P2 , . . . , Pn respectively such that A1 P1 = An+1 Pn = x1 and Ai Pi = Ai Pi−1 = xi for i = 2, . . . , n. Observe that q OA1 = OAn+1 = x21 + r2 An+1 and the function f (r) = ∠A1 OA2 + A1 Pn ∠A2 OA3 + · · · + ∠An OAn+1 = 2 · P1 (arctan xr1 + · · · + arctan xrn ) is conAn O A2 tinuous. Thus A1 A2 . . . An+1 is a Pn−1 P2 closed simple polygonal line if and An−1 only if f (r) = 360◦. But such an r exists, since f (r) → 0 when r → ∞, A3 and f (r) → ∞ when r → 0. This proves the second direction of the lemma. For n = 4k, the system (1) is solvable in positive reals if ai = i for i ≡ 1, 2 (mod 4), ai = i + 1 for i ≡ 3 and ai = i − 1 for i ≡ 0 (mod 4). Indeed, one solution is given by xi = 1/2 for i ≡ 1, xi = 3/2 for i ≡ 3 and xi = i − 3/2 for i ≡ 0, 2 (mod 4). Remark. For n = 4k + 2 there is no such n-gon. In fact, solvability of the system (1) implies a1 + a3 + · · · = a2 + a4 + · · · , while in the case n = 4k + 2 the sum a1 + a2 + · · · + an is odd. √ 9. Since the equation x3 − x − c = 0 has only one real root for every c > 2/(3 3) , α is the unique real root of x3 − x − 331992 = 0. Hence f n (α ) = f (α ) = α . Remark. Consider any irreducible polynomial g(x) in the place of x3 − x − 331992. The problem amounts to proving that if α and f (α ) are roots of g, then any f (n) (α ) is also a root of g. In fact, since g( f (x)) vanishes at x = α , it must be divisible by the minimal polynomial of α , that is, g(x). It follows by induction that g( f (n) (x)) is divisible by g(x) for all n ∈ N, and hence g( f (n) (α )) = 0. 10. Let us set S(x) = {(y, z) | (x, y, z) ∈ V }, Sy (x) = {z | (x, z) ∈ Sy } and Sz (x) = {y | (x, y) ∈ Sz }. Clearly S(x) ⊂ Sx and S(x) ⊂ Sy (x) × Sz (x). It follows that q |V | = ∑ |S(x)| ≤ ∑ |Sx ||Sy (x)||Sz (x)| x qx (1) p = |Sx | ∑ |Sy (x)||Sz (x)|. x 4.33 Shortlisted Problems 1992 567 Using the Cauchy–Schwarz inequality we also get q q r r |S (x)||S (x)| ≤ |S (x)| |S (x)| = |Sy ||Sz |. z ∑ y ∑ y ∑ z x x Now (1) and (2) together yield |V | ≤ (2) x p |Sx ||Sy ||Sz |. 11. Let I be the incenter of △ABC. Since 90◦ + α /2 = ∠BIC = ∠DIE = 138◦ , we obtain that ∠A = 96◦ . A E S B E′ D′ I D′ D C E′ Let and be the points symmetric to D and E with respect to CE and BD respectively, and let S be the intersection point of ED′ and BD. Then ∠BDE ′ = 24◦ and ∠D′ DE ′ = ∠D′ DE − ∠E ′ DE = 24◦ ,which means that DE ′ bisects the angle SDD′ . Moreover, ∠E ′ SB = ∠ESB = ∠EDS + ∠DES = 60◦ and hence SE ′ bisects the angle D′ SB. It follows that E ′ is the excenter of △D′ DS and consequently ∠D′ DC = ∠DD′C = ∠SD′ E ′ = (180◦ − 72◦ )/2 = 54◦ . Finally, ∠C = 180◦ − 2 · 54◦ = 72◦ and ∠B = 12◦ . 12. Let us set deg f = n and deg g = m. We shall prove the result by induction on n. If n < m, then degx [ f (x) − f (y)] < degx [g(x) − g(y)], which implies that f (x) − f (y) = 0, i.e., that f is constant. The statement trivially holds. Assume now that n ≥ m. Transition to f1 (x) = f (x) − f (0) and g1 (x) = g(x) − g(0) allows us to suppose that f (0) = g(0) = 0. Then the given condition for y = 0 gives us f (x) = f1 (x)g(x), where f1 (x) = a(x, 0) and deg f1 = n − m. We now have a(x, y)(g(x) − g(y)) = f (x) − f (y) = f1 (x)g(x) − f1 (y)g(y) = [ f1 (x) − f1 (y)]g(x) + f1 (y)[g(x) − g(y)]. Since g(x) is relatively prime to g(x) − g(y), it follows that f1 (x) − f1 (y) = b(x, y)(g(x) − g(y)) for some polynomial b(x, y). By the induction hypothesis there exists a polynomial h1 such that f1 (x) = h1 (g(x)) and consequently f (x) = g(x) · h1 (g(x)) = h(g(x)) for h(t) = th1 (t). Thus the induction is complete. 13. Let us define (pqr − 1) (p − 1)(q − 1)(r − 1) 1 1 1 = 1+ + + p−1 q−1 r−1 1 1 1 + + + . (p − 1)(q − 1) (q − 1)(r − 1) (r − 1)(p − 1) F(p, q, r) = 568 4 Solutions Obviously F is a decreasing function of p, q, r. Suppose that 1 < p < q < r are integers for which F(p, q, r) is an integer. Observe that p, q, r are either all even or all odd. Indeed, if for example p is odd and q is even, then pqr − 1 is odd while (p − 1)(q − 1)(r − 1) is even, which is impossible. Also, if p, q, r are even then F(p, q, r) is odd. If p ≥ 4, then 1 < F(p, q, r) ≤ F(4, 6, 8) = 191/105 < 2, which is impossible. Hence p ≤ 3. Let p = 2. Then q, r are even and 1 < F(2, q, r) ≤ F(2, 4, 6) = 47/15 < 4. Therefore F(2, q, r) = 3. This equality reduces to (q − 3)(r − 3) = 5, with the unique solution q = 4, r = 8. Let p = 3. Then q, r are odd and 1 < F(3, q, r) ≤ F(3, 5, 7) = 104/48 < 3. Therefore F(3, q, r) = 2. This equality reduces to (q − 4)(r − 4) = 11, which leads to q = 5, r = 15. Hence the only solutions (p, q, r) of the problem are (2, 4, 8) and (3, 5, 15). 14. We see that x1 = 20 . Suppose that for some m, r ∈ N we have xm = 2r . Then inductively xm+i = 2r−i (2i + 1) for i = 1, 2, . . . , r and xm+r+1 = 2r+1 . Since every natural number can be uniquely represented as the product of an odd number and a power of two, we conclude that every natural number occurs in our sequence exactly once. Moreover, it follows that 2k − 1 = xk(k+1)/2 . Thus xn = 1992 = 23 · 249 implies that xn+3 = 255 = 2 · 128 − 1 = x128·129/2 = x8256 . Hence n = 8253. 15. The result follows from the following lemma by taking n = 1992·1993 and M = 2 {d, 2d, . . . , 1992d}. Lemma. For every n ∈ N there exists a natural number d such that all the numbers d, 2d, . . . , nd are of the form mk (m, k ∈ N, k ≥ 2). Proof. Let p1 , p2 , . . . , pn be distinct prime numbers. We shall find d in the form d = 2α2 3α3 · · · nαn , where αi ≥ 0 are integers such that kd is a perfect pk th power. It is sufficient to find αi , i = 2, 3, . . . , n, such that αi ≡ 0 (mod p j ) if i 6= j and αi ≡ −1 (mod p j ) if i = j. But the existence of such αi ’s is an immediate consequence of the Chinese remainder theorem. 16. Observe that x4 + x3 + x2 + x + 1 = (x2 + 3x + 1)2 − 5x(x + 1)2 . Thus for x = 525 we have N = x4 + x3 + x2 + x + 1 = (x2 + 3x + 1 − 513(x + 1))(x2 + 3x + 1 + 513(x + 1)) = A · B. Clearly, both A and B are positive integers greater than 1. 17. (a) Let n = ∑ki=1 2ai , so that α (n) = k. Then n2 = ∑ 22ai + ∑ 2ai +a j +1 i i< j  k(k+1) has at most k + 2k = 2 binary ones. (b) The above inequality is an equality for all numbers nk = 2k . 4.33 Shortlisted Problems 1992 m m 569 j (c) Put nm = 22 −1 − ∑mj=1 22 −2 , where m > 1. It is easy to see that α (nm ) = 2m − m. On the other hand, squaring and simplifying yields n2m = 1 + m+1 i j m(m+1) ∑i< j 22 +1−2 −2 . Hence α (n2m ) = 1 + 2 and thus α (n2m ) 2 + m(m + 1) = → 0 as m → ∞. α (nm ) 2(2m − m) Solution to the alternative parts. i i+1 i j (d) Let n = ∑ni=1 22 . Then n2 = ∑ni=1 22 + ∑i< j 22 +2 +1 has exactly α (n2 ) k(k+1) 2 2k binary ones, and therefore α (n) = k(k+1) → ∞. (e) Consider the sequence ni constructed in part (c). Let θ > 1 be a constant to be chosen later, and let Ni = 2mi ni − 1 where mi > α (ni ) is such that mi /α (ni ) → θ as i → ∞. Then α (Ni ) = α (ni ) + mi − 1, whereas Ni2 = 22mi n2i − 2mi +1 ni + 1 and α (Ni2 ) = α (n2i ) − α (ni ) + mi . It follows that α (Ni2 ) α (n2i ) + (θ − 1)α (ni ) θ − 1 lim = lim = , i→∞ α (Ni ) i→∞ (1 + θ )α (ni ) θ +1 γ which is equal to γ ∈ [0, 1] for θ = 1+ 1−γ (for γ = 1 we set mi /α (ni ) → ∞). 2 (f) Let be given a sequence (ni )∞ i=1 with α (ni )/α (ni ) → γ . Taking mi > α (ni ) m i and Ni = 2 ni + 1 we easily find that α (Ni ) = α (ni ) + 1 and α (Ni2 ) = α (n2i )+ α (ni )+ 1. Hence α (Ni2 )/α (Ni ) = γ + 1. Continuing this procedure we can construct a sequence ti such that α (ti2 )/α (ti ) = γ +k for an arbitrary k ∈ N. 1 18. Let us define inductively f 1 (x) = f (x) = x+1 and f n (x) = f ( f n−1 (x)), and let 2 n gn (x) = x + f (x) + f (x) + · · · + f (x). We shall prove first the following statement. Lemma. The function gn (x) is strictly increasing on [0, 1], and gn−1 (1) = F1 /F2 + F2 /F3 + · · · + Fn /Fn+1. y−x Proof. Since f (x) − f (y) = (1+x)(1+y) is smaller in absolute value than x − y, it follows that x > y implies f 2k (x) > f 2k (y) and f 2k+1 (x) < f 2k+1 (y), and moreover that for every integer k ≥ 0, [ f 2k (x) − f 2k (y)] + [ f 2k+1 (x) − f 2k+1 (y)] > 0. Hence if x > y, we have gn (x) − gn (y) = (x − y) + [ f (x) − f (y)] + · · · + [ f n (x) − f n (y)] > 0, which yields the first part of the lemma. The second part follows by simple induction, since f k (1) = Fk+1 /Fk+2 . If some xi = 0 and consequently x j = 0 for all j ≥ i, then the problem reduces to the problem with i − 1 instead of n. Thus we may assume that all x1 , . . . , xn are different from 0. If we write ai = [1/xi ], then xi = ai +x1 i+1 . Thus we can regard xi as functions of xn depending on a1 , . . . , an−1 . Suppose that xn , an−1 , . . . , a3 , a2 are fixed. Then x2 , x3 , . . . , xn are all fixed, and 1 x1 = a1 +x is maximal when a1 = 1. Hence the sum S = x1 + x2 + · · · + xn is 2 maximized for a1 = 1. 570 4 Solutions We shall show by induction on i that S is maximized for a1 = a2 = · · · = ai = 1. In fact, assuming that the statement holds for i − 1 and thus a1 = · · · = ai−1 = 1, having xn , an−1 , . . . , ai+1 fixed we have that xn , . . . , xi+1 are also fixed, and that xi−1 = f (xi ), . . . , x1 = f i−1 (xi ). Hence by the lemma, S = gi−1 (xi ) + xi+1 + · · · + xn is maximal when xi = ai +x1 i+1 is maximal, that is, for ai = 1. Thus the induction is complete. It follows that x1 + · · · + xn is maximal when a1 = · · · = an−1 = 1, so that x1 + · · · + xn = gn−1 (x1 ). By the lemma, the latter does not exceed gn−1 (1). This completes the proof. Remark. The upper bound is the best possible, because it is approached by taking 1 1 xn close to 1 and inductively (in reverse) defining xi−1 = 1+x = ai +x . i i √ 4 2 + 3)2 − 8(x2 − 1)2 = [x4 + 2(1 − 2)x2 + 3 + 19. Observe that f (x) √ √ = (x + 2x √ 2 2][x4 + 2(1 + 2)x2 + 3 − 2 2]. Now it is easy to find that the roots of f are √  √  4 4 x1,2,3,4 = ±i i 2 ± 1 and x5,6,7,8 = ±i 2±1 . In other words, xk = αi + β j , where αi2 = −1 and β j4 = 2. We claim that any root of f can be obtained from any other using rational functions. In fact, we have x3 = −αi − 3β j + 3αi β j2 + β j3 , x5 = 11αi + 7β j − 10αi β j2 − 10β j3 x7 = −71αi − 49β j + 35αi β j2 + 37β j3 , from which we easily obtain that αi = 24−1 (127x + 5x3 + 19x5 + 5x7 ), β j = 24−1 (151x + 5x3 + 19x5 + 5x7 ). Since all other values of α and β can be obtained as rational functions of αi and β j , it follows that all the roots xl are rational functions of a particular root xk . We now note that if x1 is an integer such that f (x1 ) is divisible by p, then p > 3 and x1 ∈ Z p is a root of the polynomial f . By the previous consideration, all remaining roots x2 , . . . , x8 of f over the field Z p are rational functions of x1 , since 24 is invertible in Z p . Then f (x) factors as f (x) = (x − x1 )(x − x2) · · · (x − x8 ), and the result follows. 20. Denote by U the point of tangency of the circle C and the line l. Let X and U ′ be the points symmetric to U with respect to S and M respectively; these points do not depend on the choice of P. Also, let C′ be the excircle 4.33 Shortlisted Problems 1992 P of △PQR corresponding to P, S′ the center of C′ , and W,W ′ the points of X tangency of C and C′ with the line W S PQ respectively. Obviously, △W SP ∼ △W ′ S′ P. Since SX k S′U ′ and SX : U M U′ Q S′U ′ = SW : S′W ′ = SP : S′ P, we deduce that ∆ SXP ∼ ∆ S′U ′ P, and consequently that P lies on the line XU ′ . On the other hand, it is easy to show that each point P of the ray U ′ X over X sat- W ′ isfies the required condition. Thus the S′ desired locus is the extension of U ′ X over X . 571 R 21. (a) Representing n2 as a sum of n2 − 13 squares is equivalent to representing 13 as a sum of numbers of the form x2 − 1, x ∈ N, such as 0, 3, 8, 15, . . .. But it is easy to check that this is impossible, and hence s(n) ≤ n2 − 14. (b) Let us prove that s(13) = 132 − 14 = 155. Observe that 132 = 82 + 82 + 42 + 42 + 32 = 82 + 82 + 4 2 + 4 2 + 22 + 22 + 12 = 82 + 82 + 4 2 + 3 2 + 32 + 22 + 12 + 1 2 + 1 2 . Given any representation of n2 as a sum of m squares one of which is even, we can construct a representation as a sum of m + 3 squares by dividing the even square into four equal squares. Thus the first equality enables us to construct representations with 5, 8, 11, . . ., 155 squares, the second to construct ones with 7, 10, 13, . . ., 154 squares, and the third with 9, 12, . . . , 153 squares. It remains only to represent 132 as a sum of k = 2, 3, 4, 6 squares. This can be done as follows: 132 = 122 + 52 = 122 + 42 + 32 = 112 + 42 + 42 + 42 = 122 + 32 + 22 + 22 + 22 + 22 . (c) We shall prove that whenever s(n) = n2 − 14 for some n ≥ 13, it also holds that s(2n) = (2n)2 − 14. This will imply that s(n) = n2 − 14 for any n = 2t · 13. If n2 = x21 + · · · + x2r , then we have (2n)2 = (2x1 )2 + · · · + (2xr )2 . Replacing (2xi )2 with x2i + x2i + x2i + x2i as long as it is possible we can obtain representations of (2n)2 consisting of r, r + 3, . . . , 4r squares. This gives representations of (2n)2 into k squares for any k ≤ 4n2 − 62. Further, we observe that each number m ≥ 14 can be written as a sum of k ≥ m numbers of the form x2 − 1, x ∈ N, which is easy to verify. Therefore if 572 4 Solutions k ≤ 4n2 − 14, it follows that 4n2 − k is a sum of k numbers of the form x2 − 1 (since k ≥ 4n2 − k ≥ 14), and consequently 4n2 is a sum of k squares. Remark. One can find exactly the value of s(n) for each n:  if n has no prime divisor congruent to 1 mod 4;  1, s(n) = 2, if n is of the form 5 · 2k , k a positive integer;  2 n − 14, otherwise. 4.34 Shortlisted Problems 1993 573 4.34 Solutions to the Shortlisted Problems of IMO 1993 1. First we notice that for a rational point O (i.e., with rational coordinates), there exist 1993 rational points in each quadrant of the unit circle centered at O. In fact, it suffices to take   2   t −1 2t X = O+ ± 2 ,± t = 1, 2, . . . , 1993 . t + 1 t2 + 1 1993 2 Now consider the set A = {(i/q, j/q) | i, j = 0, 1, . . . , 2q}, where q = ∏t=1 (t + 1). We claim that A gives a solution for the problem. Indeed, for any P ∈ A there is a quarter of the unit circle centered at P that is contained in the square [0, 2] × [0, 2]. As explained above, there are 1993 rational points on this quarter circle, and by definition of q they all belong to A. Remark. Substantially the same problem was proposed by Bulgaria for IMO 71: see (SL71-2), where we give another possible construction of a set A. 2 2. It is well known that r ≤ 12 R. Therefore 13 (1 + r)2 ≤ 13 1 + 12 = 34 . It remains only to show that p ≤ 14 . We note that p does not exceed one half of the circumradius of △A′ B′C′ . However, by the theorem on the nine-point circle, this circumradius is equal to 12 R, and the conclusion follows. Second solution. By a well-known relation we have cos A + cos B + cosC = 1 + Rr (= 1 + r when R = 1). Next, recalling that the incenter of △A′ B′C′ is at the orthocenter of △ABC, we easily obtain p = 2 cos A cos B cosC. Cosines of angles of a triangle satisfy the identity cos2 A + cos2 B + cos2 C + 2 cos A cos B cosC = 1 (the proof is straightforward: see (SL81-11)). Thus 1 1 p + (1 + r)2 = 2 cosA cos B cosC + (cos A + cosB + cosC)2 3 3 ≤ 2 cosA cos B cosC + cos2 A + cos2 B + cos2 C = 1. 3. Let O1 and ρ be the center and radius of kc . It is clear that C, I, O1 are collinear and CI/CO1 = r/ρ . By Stewart’s theorem applied to △OCO1 ,   r r OI 2 = OO21 + 1 − OC2 −CI · IO1 . (1) ρ ρ Since OO1 = R − ρ , OC = R and by Euler’s formula OI 2 = R2 −2Rr, substituting these values in (1) gives CI · IO1 = rρ , or equivalently CO1 · IO1 = ρ 2 = DO21 . Hence the triangles CO1 D and DO1 I are similar, implying ∠DIO1 = 90◦ . Since CD = CE and the line CO1 bisects the segment DE, it follows that I is the midpoint of DE. Second solution. Under the inversion with center C and power ab, kc is transbBC b corresponding to C. Thus CD = ab , where s is formed into the excircle of A s bBC, b and consequently the distance the common semiperimeter of △ABC and △A 2SABC from D to BC is ab = 2r. The statement follows immediately. s sinC = s 574 4 Solutions Third solution. We shall prove a stronger statement: Let ABCD be a convex quadrilateral inscribed in a circle k, and k′ the circle that is tangent to segments BO, AO at K, L respectively (where O = BD ∩AC), and internally to k at M. Then KL contains the incenters I, J of △ABC and △ABD. Let K ′ , K ′′ , L′ , L′′ , N denote the midpoints of arcs BC, BD, AC, AD, AB that don’t contain M; X ′ , X ′′ the points on k defined by X ′ N = NX ′′ = K ′ K ′′ = L′ L′′ (as oriented arcs); and set S = AK ′ ∩ BL′′ , M = NS ∩ k, K = K ′′ M ∩ BO, L = L′ M ∩ AO. It is clear that I = AK ′ ∩BL′ , J = AK ′′ ∩BL′′ . Furthermore, X ′ M contains I (to see this, use the fact that for A, B,C, D, E, F on k, lines AD, BE, CF are concurrent if and only if AB · CD · EF = BC · DE · FA, and then express AM/MB by applying this rule to AMBK ′ NL′′ and show that AK ′ , MX ′ , BL′ are concurrent). Similarly, X ′′ M contains J. Now the points X ′′ D N X′ C ′ B, K, I, S, M lie on a circle (∠BKM = L K ′′ ∠BIM = ∠BSM), and points A, L, J, O S, M do so as well. Lines IK, JL are L′′ ′′ ′ parallel to K L (because ∠MKI = K′ ′′ ′ J I ∠MBI = ∠MK L ). On the other hand, L K the quadrilateral ABIJ is cyclic, and S simple calculation with angles shows that IJ is also parallel to K ′′ L′ . Hence B A K, I, J, L are collinear. Finally, K ≡ K, L ≡ L, and M ≡ M because the homoM thety centered at M that maps k′ to k ′′ ′ ′′ ′ sends K to K and L to L (thus M, K, K , as well as M, L, L , must be collinear). As is seen now, the deciphered picture yields many other interesting properties. Thus, for example, N, S, M are collinear, i.e., ∠AMS = ∠BMS. Fourth solution. We give an alternative proof of the more general statement in the third solution. Let W be the foot of the perpendicular from B to AC. We define q = CW , h = BW , t = OL = OK, x = AL, θ = ∡W BO (θ is negative if B(O,W, A), θ = 0 if W = O), and as usual, a = BC, b = AC, c = AB. Let α = ∡KLC and β = ∡ILC (both angles must be acute). Our goal is to prove α = β . We note that 90◦ − θ = 2α . One easily gets tan α = cos θ , tan β = 1 + sin θ 2SABC a+b+c b+c−a −x 2 . (1) ′ Applying Casey’s  theorem to A, B,C, k , we get AC · BK + AL · BC = AB · CL, h i.e., b cos θ − t + xa = c(b − x). Using that t = b − x − q − h tan θ we get  b(b + c − q) − bh cos1 θ + tan θ x= . a+b+c (2) Plugging (2) into the second equation of (1) and using bh = 2SABC and c2 = b2 + a2 − 2bq, we obtain tan α = tan β , i.e., α = β , which completes our proof. 4.34 Shortlisted Problems 1993 575 4. Let h be the altitude from A and ϕ = ∠BAD. We have BM = 12 (BD + AB − AD) and MD = 12 (BD − AB + AD), so 1 1 BD 4BD + = = MB MD MB · MD BD2 − AB2 − AD2 + 2AB · AD 4BD 2BD sin ϕ = = 2AB · AD(1 − cos ϕ ) 2SABD (1 − cos ϕ ) 2BD sin ϕ 2 = = . BD · h(1 − cos ϕ ) h tan ϕ2 It follows that as well. 1 MB 1 + MD depends only on h and ϕ . Specially, 1 NC 1 + NE = h tan(2ϕ /2) 5. For n = 1 the game is trivially over. If n = 2, it can end, for example, in the following way: • • • • −→ • −→ • • Fig. 1 The sequence of moves shown in Fig. 2 enables us to remove three pieces placed in a 1 × 3 rectangle, using one more piece and one more free cell. In that way, for any n ≥ 4 we can reduce an (n + 3) × (n + 3) square to an n × n square (Fig. 3). Therefore the game can end for every n that is not divisible by 3. • • • • −→ • • • −→ Fig. 2 • • −→ • Fig. 3 Suppose now that one can play the game on a 3k × 3k square so that at the end only one piece remains. Denote the cells by (i, j), i, j ∈ {1, . . . , 3k}, and let S0 , S1 , S2 denote the numbers of pieces on those squares (i, j) for which i + j gives remainder 0, 1, 2 respectively upon division by 3. Initially S0 = S1 = S2 = 3k2 . After each move, two of S0 , S1 , S2 diminish and one increases by one. Thus each move reverses the parity of the Si ’s, so that S0 , S1 , S2 are always of the same parity. But in the final position one of the Si ’s must be equal to 1 and the other two must be 0, which is impossible. √ 6. Notice that for α = 1+2 5 , α 2 n = α n + n for all n ∈ N. We shall show that f (n) =   α n + 12 (the closest integer to α n) satisfies the requirements. Observe that f 576 4 Solutions is strictly increasing and f (1) = 2. By the definition of f , | f (n) − α n| ≤ f ( f (n)) − f (n) − n is an integer. On the other hand, 1 2 and | f ( f (n)) − f (n) − n| = | f ( f (n)) − f (n) − α 2 n + α n| = | f ( f (n)) − α f (n) + α f (n) − α 2 n − f (n) + α n| = |(α − 1)( f (n) − α n) + ( f ( f (n)) − α f (n))| ≤ (α − 1)| f (n) − α n| + | f ( f (n)) − α f (n)| 1 1 1 ≤ (α − 1) + = α < 1, 2 2 2 which implies that f ( f (n)) − f (n) − n = 0. 7. Multiplying by a and c the equation ax2 + 2bxy + cy2 = Pk n, (1) gives (ax + by)2 + Py2 = aPk n and (bx + cy)2 + Px2 = cPk n. It follows immediately that M(n) is finite; moreover, (ax + by)2 and (bx + cy)2 are divisible by P, and consequently ax +by, bx + cy are divisible by P because P is not divisible by a square greater than 1. Thus there exist integers X,Y such that bx+ cy = PX, ax+ by = −PY . Then x = −bX − cY and y = aX + bY . Introducing these values into (1) and simplifying the expression obtained we get aX 2 + 2bXY + cY 2 = Pk−1 n. (2) Hence (x, y) 7→ (X ,Y ) is a bijective correspondence between integral solutions of (1) and (2), so that M(Pk n) = M(Pk−1 n) = · · · = M(n). 8. Suppose that f (n) = 1 for some n > 0. Then f (n + 1) = n + 2, f (n + 2) = 2n + 4, f (n + 3) = n + 1, f (n + 4) = 2n + 5, f (n + 5) = n, and so by induction f (n + 2k) = 2n + 3 + k, f (n + 2k − 1) = n + 3 − k for k = 1, 2, . . . , n + 2. Particularly, n′ = 3n + 3 is the smallest value greater than n for which f (n′ ) = 1. It follows that all numbers n with f (n) = 1 are given by n = bi , where b0 = 1, bn = 3bn−1 + 3. Furthermore, bn = 3 + 3bn−1 = 3 + 32 + 32bn−2 = · · · = 3 + 32 + · · · + 3n + 3n = = 12 (5 · 3n − 3). It is seen from above that if n ≤ bi , then f (n) ≤ f (bi − 1) = bi + 1. Hence if f (n) = 1993, then n ≥ bi ≥ 1992 for some i. The smallest such bi is b7 = 5466, and f (bi + 2k − 1) = bi + 3 − k = 1993 implies k = 3476. Thus the least integer in S is n1 = 5466 + 2 · 3476 − 1 = 12417. All the elements of S are given by ni = bi+6 + 2k − 1, where bi+6 + 3 − k = 1993, i.e., k = bi+6 − 1990. Therefore ni = 3bi+6 − 3981 = 12 (5 · 3i+7 − 7971). Clearly n S is infinite and limi→∞ i+1 ni = 3. 9. We shall first complete the “multiplication table” for the sets A, B,C. It is clear that this multiplication is commutative and associative, so that we have the following relations: AC = (AB)B = BB = C; A2 = AA = (AB)C = BC = A; C2 = CC = B(BC) = BA = B. 4.34 Shortlisted Problems 1993 577 (a) Now put 1 in A and distribute the primes arbitrarily in A, B,C. This distribution uniquely determines the partition of Q+ with the stated property. Indeed, if an arbitrary rational number α β β γ γm x = pα1 1 · · · pk k q1 1 · · · ql l r11 · · · rm is given, where pi ∈ A, qi ∈ B, ri ∈ C are primes, it is easy to see that x belongs to A, B, or C according as β1 + · · · + βl + 2γ1 + · · · + 2γm is congruent to 0, 1, or 2 (mod 3). (b) In every such partition, cubes all belong to A. In fact, A3 = A2 A = AA = A, B3 = B2 B = CB = A, C3 = C2C = BC = A. (c) By (b) we have 1, 8, 27 ∈ A. Then 2 6∈ A, and since the problem is symmetric with respect to B,C, we can assume 2 ∈ B and consequently 4 ∈ C. Also 7 6∈ A, and also 7 6∈ B (otherwise, 28 = 4 · 7 ∈ A and 27 ∈ A), so 7 ∈ C, 14 ∈ A, 28 ∈ B. Further, we see that 3 6∈ A (since otherwise 9 ∈ A and 8 ∈ A). Put 3 in C. Then 5 6∈ B (otherwise 15 ∈ A and 14 ∈ A), so let 5 ∈ C too. Consequently 6, 10 ∈ A. Also 13 6∈ A, and 13 6∈ C because 26 6∈ A, so 13 ∈ B. Now it is easy to distribute the remaining primes 11, 17, 19, 23, 29, 31: one possibility is A = {1, 6, 8, 10, 14, 19, 23, 27, 29, 31, 33, . . .}, C = {3, 4, 5, 7, 18, 22, 24, 26, 30, 32, 34, . . .}, B = {2, 9, 11, 12, 13, 15, 16, 17, 20, 21, 25, 28, 35, . . .}. Remark. It can be proved that min{n ∈ N | n ∈ A, n + 1 ∈ A} ≤ 77. 10. (a) Let n = p be a prime and let p | a p − 1. By Fermat’s theorem p | a p−1 − 1, so that p | agcd(p,p−1) − 1 = a − 1, i.e., a ≡ 1 (mod p). Since then ai ≡ 1 (mod p), we obtain p | a p−1 + · · · + a + 1 and hence p2 | a p − 1 = (a − 1)(a p−1 + · · · + a + 1). (b) Let n = p1 · · · pk be a product of distinct primes and let n | an − 1. Then from pi | an − 1 = (a(n/pi) ) pi − 1 and part (a) we conclude that p2i | an − 1. Since this is true for all indices i, we also have n2 | an − 1; hence n has the property P. 11. Due to the extended Eisenstein criterion, f must have an irreducible factor of degree not less than n − 1. Since f has no integral zeros, it must be irreducible. Second solution. The proposer’s solution was as follows. Suppose that f (x) = g(x)h(x), where g, h are nonconstant polynomials with integer coefficients. Since | f (0)| = 3, either |g(0)| = 1 or |h(0)| = 1. We may assume |g(0)| = 1 and that g(x) = (x − α1 ) · · · (x − αk ). Then |α1 · · · αk | = 1. Since αin−1 (αi + 5) = −3, taking the product over i = 1, 2, . . . , k yields |(α1 + 5) · · · (αk + 5)| = |g(−5)| = 3k . But f (−5) = g(−5)h(−5) = 3, so the only possibility is degg = k = 1. This is impossible, because f has no integral zeros. Remark. Generalizing this solution, it can be shown that if a, m, n are positive integers and p < a − 1 is a prime, then F(x) = xm (x + a)n + p is irreducible. The details are left to the reader. 578 4 Solutions 12. Let x1 < x2 < · · · < xn be the elements of S. We use induction on n. The result is trivial for k = 1 or n = k, so assume that it is true for n − 1 numbers. Then there exist m = (k − 1)(n − k) + 1 distinct sums of k − 1 numbers among x2 , . . . , xn ; call these sums Si , S1 < S2 < · · · < Sm . Then x1 + S1 , x1 + S2 , . . . , x1 + Sm are distinct sums of k of the numbers x1 , x2 , . . . , xn . However, the biggest of these sums is x1 + Sm ≤ x1 + xn−k+2 + xn−k+3 + · · · + xn ; hence we can find n − k sums that are greater and thus not included here: x2 + xn−k+2 + · · ·+ xn , x3 + xn−k+2 + · · ·+ xn , . . . , xn−k+1 + xn−k+2 + · · · + xn . This counts for k(n − k) + 1 sums in total. Remark. Equality occurs if S is an arithmetic progression. 13. For an odd integer N > 1, let SN = {(m, n) ∈ S | m+n = N}. If f (m, n) = (m1 , n1 ), then m1 + n1 = m + n with m1 odd and m1 ≤ n2 < N2 < n1 , so f maps SN to SN . Also f is bijective, since if f (m, n) = (m1 , n1 ), then n is uniquely determined as the even number of the form 2k m1 that belongs to the interval [ N+1 2 , N], and this also determines m.   Note that SN has at most N+1 elements, with equality if and only if N is prime. 4   Thus if (m, n) ∈ SN , there exist s, r with 1 ≤ s < r ≤ N+5 such that f s (m, n) = 4   f r (m, n). Consequently f t (m, n) = (m, n), where t = r − s, 0 < t ≤ N+1 = 4  m+n+1  . 4 Suppose that (m, n) ∈ SN and t is the least positive integer with f t (m, n) = (m, n). We write (m, n) = (m0 , n0 ) and f i (m, n) = (mi , ni ) for i = 1, . . . ,t. Then there exist positive integers ai such that 2ai mi = ni−1 , i = 1, . . . ,t. Since mt = m0 , multiplying these equalities gives 2a1 +a2 +···+at m0 m1 · · · mt−1 = n0 n1 · · · nt−1 ≡ (−1)t m0 m1 · · · mt−1 (mod N). (1) It follows that N | 2k ± 1 and consequently N | 22k − 1, where k = a1 + · · · + at . On the other hand, it also follows that 2k | n0 n1 · · · nt−1 | (N − 1)(N − 3) · · · (N − 2[N/4]). But since   (N − 1)(N − 3) · · · N − 2 N4 N−1 2 · 4 · · ·(N − 1)  N−2   = =2 2 , N−1 1 · 3··· 2 4 + 1 1 · 2··· 2 we conclude that 0 < k ≤ N−1 2 , where equality holds if and only if {n1 , . . . , nt } N+1 is the set of all even integers from N+1 2 to N − 1, and consequently t = 4 . h Now if N ∤ 2 −1 for 1 ≤ h < N −1, we must have 2k = N −1. Therefore t = N+1 4 . 14. We first assume that all angles of triangle ABC are less than 120◦ . Consider the Torricelli point T of the triangle. It holds that ∠(AT, EF) = ∠(BT, FD) = ∠(CT, DE) = θ for some angle θ . Therefore 2S = 2(SAET F + SBFT D + SCDT E ) = (AT · EF + BT · FD + CT · DE) sin θ = (AT + BT + CT )DE sin θ ≤ (AT + BT +CT )DE. (1) 4.34 Shortlisted Problems 1993 579 On the other hand, by the cosine theorem we get AT 2 + AT · BT + BT 2 = c2 , BT 2 + BT ·CT +CT 2 = a2 , CT 2 + CT · AT + AT 2 = b2 , √ 3(AT · BT + BT ·CT +CT · AT ) = 4 3(SAT B + SBTC + SCTA ) √ = 4 3S. Adding these four equalities, we obtain 2(AT + BT + CT )2 = a2 + b2 + c2 + √ 4 3S, which together with (1) implies the desired inequality. Assume now that ∠C ≥ 120◦ and take T to be the point lying on the same side of AB as C such that ∠BTC = ∠CTA = 60◦ (if ∠C = 120◦ , take T ≡ C). In this case it is shown as above that and 2S ≤ (AT + BT −CT )DE √ 2(AT + BT − CT )2 = a2 + b2 + c2 + 4 3S, and the inequality follows as before. 15. Denote by d(PQR) the diameter of a triangle PQR. It is clear that d(PQR) · m(PQR) = 2SPQR. So if the point X lies inside the triangle ABC or on its boundary, we have d(ABX), d(BCX), d(CAX) ≤ d(ABC), which implies 2SABX 2SBCX 2SCAX + + d(ABX) d(BCX) d(CAX) 2SABX + 2SBCX + 2SCAX ≥ d(ABC) 2SABC = = m(ABC). d(ABC) m(ABX) + m(BCX) + m(CAX) = If X is outside △ABC but inside the angle BAC, consider the point Y of intersection of AX and BC. Then m(ABX) + m(BCX ) + m(CAX ) ≥ m(ABY ) + m(BCY ) + m(CAY ) ≥ m(ABC). Also, if X is inside the opposite angle of ∠BAC (i.e., ∠DAE, where B(D, A, B) and B(E, A,C)), then m(ABX) + m(BCX) + m(CAX) ≥ m(BCX) ≥ m(ABC). Since these are essentially all possible different positions of point X, we have finished the proof. 16. Let Sn = {A = (a1 , . . . , an ) | 0 ≤ ai < i}. For each A = (a1 , . . . , an ), denote A′ = (a1 , . . . , an−1 ), so we can write A = (A′ , an ). The proof of the statement from the problem will be given by induction on n. For n = 2 there are two possibilities for A0 , so one directly checks that A2 = A0 . Now assume that n ≥ 3 and that A0 = (A′0 , a0n ) ∈ Sn . It is clear that then any Ai is in Sn too. By the induction hypothesis there exists k ∈ N such that A′k = A′k+2 = A′k+4 = · · · and A′k+1 = A′k+3 = · · · . Observe that if we increase (decrease) akn , ak+1,n will decrease (respectively increase), and this will also increase (respectively decrease) ak+2,n . Hence akn , ak+2,n , ak+4,n , . . . is monotonically increasing or decreasing, 580 4 Solutions and since it is bounded (by 0 and n − 1), it follows that we will eventually have ak+2i,n = ak+2i+2,n = · · · . Consequently Ak+2i = Ak+2i+2 . 17. We introduce the rotation operation Rot to the left by one, so that Step j = Rot− j ◦ Step0 ◦ Rot j . Now writing Step∗ = Rot ◦ Step0 , the problem is transformed into the question whether there is an M(n) such that all lamps are on again after M(n) successive applications of Step∗ . We operate in the field Z2 , representing off by 0 and on by 1. So if the status of L j at some moment is given by v j ∈ Z2 , the effect of Step j is that v j is replaced by v j + v j−1 . With the n-tuple v0 , . . . , vn−1 we associate the polynomial P(x) = vn−1 xn−1 + v0 xn−2 + v1 xn−3 + · · · + vn−2 . By means of Step∗ , this polynomial is transformed into the polynomial Q(x) over Z of degree less than n that satisfies Q(x) ≡ xP(x) (mod xn + xn−1 + 1). From now on, the sign ≡ always stands for congruence with this modulus. (a) It suffices to show the existence of M(n) with xM(n) ≡ 1. Because the number of residue classes is finite, there are r, q, with r < q, such that xq ≡ xr , i.e., xr (xq−r − 1) = 0. One can take M(n) = q − r. (Or simply note that there are only finitely many possible configurations; since each operation is bijective, the configuration that reappears first must be on, on, . . . , on.) 2 2 (b) We shall prove that if n = 2k , then xn −1 ≡ 1. We have xn ≡ (xn−1 + 1)n ≡ 2 xn −n + 1, because all binomial coefficients of order n = 2k are even, apart 2 2 2 from the first one and the last one. Since also xn ≡ xn −1 + xn −n , this is what we wanted. 2 2 (c) Now if n = 2k +1, we prove that xn −n+1 ≡ 1. We have xn −1 ≡ (xn+1 )n−1 ≡ 2 (x + xn )n−1 ≡ xn−1 + xn −n (again by evenness of binomial coefficients of 2 2 2 2 order n − 1 = 2k ). Together with xn ≡ xn −1 + xn −n , this leads to xn ≡ xn−1 . 18. Let Bn be the set of sequences with the stated property (Sn = |Bn |). We shall prove by induction on n that Sn ≥ 32 Sn−1 for every n. n−i Suppose that for every i ≤ n, Si ≥ 32 Si−1 , and consequently Si ≤ 23 Sn . Let us consider the 2Sn sequences obtained by putting 0 or 1 at the end of any sequence from Bn . If some sequence among them does not belong to Bn+1 , then for some k ≥ 1 it can be obtained by extending some sequence from Bn+1−6k by a sequence of k terms repeated six times. The number of such sequences is 2k Sn+1−6k . Hence the number of sequences not satisfying our condition is not greater than  6k−1 3 2(2/3)6 192 1 k k 2 ∑ 2 Sn+1−6k ≤ ∑ 2 3 Sn = 2 Sn 1 − 2(2/3)6 = 601 Sn < 2 Sn. k≥1 k≥1 n Therefore Sn+1 is not smaller than 2Sn − 12 Sn = 32 Sn . Thus we have Sn ≥ 32 . 19. Let s be the minimum number of nonzero digits that can appear in the b-adic representation of any number divisible by bn − 1. Among all numbers divisible by bn − 1 and having s nonzero digits in base b, we choose the number A with 4.34 Shortlisted Problems 1993 581 the minimum sum of digits. Let A = a1 bn1 + · · · + as bns , where 0 < ai ≤ b − 1 and n1 > n2 > · · · > ns . First, suppose that ni ≡ n j (mod n), i 6= j. Consider the number B = A − aibni − a j bn j + (ai + a j )bn j +kn , with k chosen large enough so that n j + kn > n1 : this number is divisible by bn − 1 as well. But if ai + a j < b, then B has s − 1 digits in base b, which is impossible; on the other hand, ai + a j ≥ b is also impossible, for otherwise B would have sum of digits less for b − 1 than that of A (because B would have digits 1 and ai + a j − b in the positions n j + kn + 1, n j + kn). Therefore ni 6≡ n j if i 6= j. Let ni ≡ ri , where ri ∈ {0, 1, . . . , n − 1} are distinct. The number C = a1 br1 + · · · + as brs also has s digits and is divisible by bn − 1. But since C < bn , the only possibility is C = bn − 1 which has exactly n digits in base b. It follows that s = n. 20. For every real x we shall denote by ⌊x⌋ and ⌈x⌉ the greatest integer less than or equal to x and the smallest integer greater than or equal hto x respectively. i ci i The condition ci + nki ∈ [1 − n, n] is equivalent to ki ∈ Ii = 1−c n − 1, 1 − n . For every two integers (not necessarily distinct), namely l ci , this m interval contains 1−ci ci  pi = n − 1 ≤ qi = 1 − n . In order to show that there exist integers ki ∈ Ii with ∑ni=1 ki = 0, it is sufficient to show that ∑ni=1 pi ≤ 0 ≤ ∑ni=1 qi . i Since pi < 1−c n , we have n n ci ≤ 1, i=1 n ∑ pi < 1 − ∑ i=1 and consequently qi > − cni implies ∑ni=1 pi ≤ 0 because the pi ’s are integers. On the other hand, n n i=1 i=1 ci ∑ qi > − ∑ n ≥ −1, which leads to ∑ni=1 qi ≥ 0. The proof is complete. 21. Assume that S is a circle with center O that cuts Si diametrically in points Pi , Qi , i ∈ {A, B,C}, and denote by ri , r the radii of Si and S respectively. Since OA is perpendicular to PA QA , it follows by Pythagoras’s theorem that OA2 + APA2 = OPA2 , i.e., rA2 + OA2 = r2 . Analogously rB2 + OB2 = r2 and rC2 + OC2 = r2 . Thus if OA , OB , OC are the feet of perpendiculars from O to BC, CA, AB respectively, then OC A2 − OC B2 = rB2 − rA2 . Since the left-hand side is a monotonic function of OC ∈ AB, the point OC is uniquely determined by the imposed conditions. The same holds for OA and OB . If A, B,C are not collinear, then the positions of OA , OB , OC uniquely determine the point O, and therefore the circle S also. On the other hand, if A, B,C are collinear, all one can deduce is that O lies on the lines lA , lB , lC through OA , OB , OC , perpendicular to BC,CA, AB respectively. 582 4 Solutions Hence, lA , lB , lC are parallel, so O can be either anywhere on the line if these lines coincide, or nowhere if they don’t coincide. So if there exists more than one circle S, A, B,C lie on a line and the foot O′ of the perpendicular from O to the line ABC is fixed. If X ,Y are the intersection points of S and the line ABC, then r2 = OX 2 = OA2 + rA2 and consequently O′ X 2 = O′ A2 + rA2 , which implies that X ,Y are fixed. SC lB lA C SB OB SA OC A OA O B S lC C 22. Let M be the point inside ∠ADB that satisfies DM = DB and DM ⊥ DB. T Then ∠ADM = ∠ACB and AD/DM = AC/CB. It follows that the triangles U ADM, ACB are similar; hence ∠CAD = D ∠BAM (because ∠CAB = ∠DAM) and AB/AM = AC/AD. Consequently the B A triangles CAD, BAM are similar and CD M √CD . Hence therefore√AC AB = BM = 2BD AB·CD AC·BD = 2. Let CT,CU be the tangents at C to the circles ACD, BCD respectively. Then (in oriented angles) ∠TCU = ∠TCD + ∠DCU = ∠CAD + ∠CBD = 90◦ , as required. Second solution to the first part. Denote by E, F, G the feet of the perpendiculars from D to BC,CA, AB. Consider the pedal triangle EFG. Since FG = AD sin ∠A, from the sine theorem we have FG : GE : EF = (CD · AB) : (BD · AC) : (AD · BC). Thus EG = FG. On the other hand, √ ∠EGF = ∠EGD + ∠DGF = ∠CBD √ + ∠CAD = 90◦ implies that EF : EG = 2 : 1; hence the required ratio is 2. Third solution to the first part. Under inversion centered at C and with power r2 = CA · CB, the triangle DAB maps into a right-angled isosceles triangle D∗ A∗ B∗ , where AD · BC ∗ ∗ AC · BD ∗ ∗ AB ·CD ,D B = ,A B = . CD CD CD √ Thus D∗ B∗ : A∗ B∗ = 2, and this is the required ratio. D∗ A ∗ = 23. Let the given numbers be a1 , . . . , an . Put s = a1 +· · ·+an and m = lcm(a1 , . . . , an ) and write m = 2k r with k ≥ 0 and r odd. Let the binary expansion of r be r = 2k0 + 2k1 + · · · + 2kt , with 0 = k0 < · · · < kt . Adjoin to the set {a1 , . . . , an } the numbers 2ki s, i = 1, 2, . . . ,t. The sum of the enlarged set is rs. Finally, adjoin rs, 2rs, 22 rs, . . . , 2l−1 rs for l = max{k, kt }. The resulting set has sum 2l rs, which is divisible by m and so by each of a j , and also by the 2i s above and by rs, 2rs, . . . , 2l−1 rs. Therefore this is a DS-set. 4.34 Shortlisted Problems 1993 583 Second solution. We show by induction that there is a DS-set containing 1 and n. For n = 2, 3, take {1, 2, 3}. Assume that {1, n, b1, . . . , bk } is a DS-set. Then {1, n + 1, n, 2(n + 1)n, 2(n + 1)b1, . . . , 2(n + 1)bk } is a DS-set too. For given a1 , . . . , an let m be a sufficiently large common multiple of the ai ’s such that u = m − (a1 + · · · + an ) 6= ai for all i. There exist b1 , . . . , bk such that {1, u, b1 , . . . , bk } is a DS-set. It is clear that {a1, . . . , an , u, mu, mb1 , . . . , mbk } is a DS-set containing a1 , . . . , an . 24. By the Cauchy–Schwarz inequality, if x1 , x2 , . . . , xn and y1 , y2 , . . . , yn are positive numbers, then ! ! ! n xi ∑ yi i=1 n ∑ xi yi i=1 n ≥ ∑ xi 2 . i=1 Applying this to the numbers a, b, c, d and b + 2c + 3d, c + 2d + 3a, d + 2a + 3b, a + 2b + 3c (here n = 4), we obtain a b c d + + + b + 2c + 3d c + 2d + 3a d + 2a + 3b a + 2b + 3c ≥ (a + b + c + d)2 2 ≥ . 4(ab + ac + ad + bc + bd + cd) 3 The last inequality follows, for example, from (a − b)2 + (a − c)2 + · · · + (c − d)2 ≥ 0. Equality holds if and only if a = b = c = d. Second solution. Putting A = b + 2c + 3d, B = c + 2d + 3a, C = d + 2a + 3b, D = a + 2b + 3c, our inequality transforms into −5A + 7B +C+ D −5B + 7C + D + A + 24A 24B + −5C + 7D + A + B −5D + 7A + B +C 2 + ≥ . 24C 24D 3 This follows from the arithmetic-geometric mean inequality, since A D ≥ 4, etc. B A + CB + D C + 25. We need only consider the case a > 1 (since the case a < −1 is reduced to a > 1 by taking a′ = −a, x′i = −xi ). Since the left sides of the equations are nonnegative, we have xi ≥ − 1a > −1, i = 1, . . . , 1000. Suppose w.l.o.g. that x1 = max{xi }. In particular, x1 ≥ x2 , x3 . If x1 ≥ 0, then we deduce that x21000 ≥ 1 ⇒ x1000 ≥ 1; further, from this we deduce that x999 > 1 etc., so either xi > 1 for all i or xi < 0 for all i. (i) xi > 1 for every i. Then x1 ≥ x2 implies x21 ≥ x22 , so x2 ≥ x3 . Thus x1 ≥ x2 ≥ · · · ≥ x1000 ≥ x1 , and consequently x1 = · · · = x1000 . In this case the only √ solution is xi = 12 (a + a2 + 4) for all i. (ii) xi < 0 for every i. Then x1 ≥ x3 implies x21 ≤ x23 ⇒ x2 ≤ x4 . Similarly, this leads to x3 ≥ x5 , etc. Hence x1 ≥ x3 ≥ x5 ≥ · · · ≥ x999 ≥ x1 and x2 ≤ x4 ≤ · · · ≤ x2 , so we deduce that x1 = x3 = · · · and x2 = x4 = · · · . Therefore the 584 4 Solutions 26. Set system is reduced to x21 = ax2 +1, x22 = ax1 +1. Subtracting these equations, one obtains (x1 − x2 )(x1 + x2 + a) = 0. There √ are two possibilities: (1) If x1 = x2 , then x1 = x2 = · · · = 12 (a − a2 + 4). (2) x1 + x2 + a = 0 is equivalent to x21 + ax1 + (a2 − 1) = 0. The discriminant of the last equation is 4 − 3a2 . Therefore if a > √23 , this case √ yields no solutions, while if a ≤ √23 , we obtain x1 = 12 (−a− 4 − 3a2), √ x2 = 12 (−a + 4 − 3a2), or vice versa. 176 f (a, b, c, d) = abc + bcd + cda + dab − abcd 27   176 = ab(c + d) + cd a + b − ab . 27 If a+b− 176 a b ≤ 0, by the arithmetic-geometric inequality we have f (a, b, c, d) ≤ 1 ab(c + d) ≤ 27 . On the other hand, if a + b − 176 27 ab > 0, the value of f increases if c, d are c+d c+d replaced by 2 , 2 . Consider now the following fourtuplets:     c+d c+d a+b a+b c+d c+d P0 (a, b, c, d), P1 a, b, , , P2 , , , , 2 2 2 2 2 2     1 a+b c+d 1 1 1 1 1 P3 , , , , P4 , , , 4 2 2 4 4 4 4 4 From the above considerations we deduce that for i = 0, 1, 2, 3 either f (Pi ) ≤ f (Pi+1 ), or directly f (Pi ) ≤ 1/27. Since f (P4 ) = 1/27, in every case we are led to 1 f (a, b, c, d) = f (P0 ) ≤ . 27 Equality occurs only in the cases (0, 1/3, 1/3, 1/3) (with permutations) and (1/4, 1/4, 1/4, 1/4). Remark. Lagrange multipliers also work. On the boundary of the set one of the numbers a, b, c, d is 0, and the inequality immediately follows, while for an extremum point in the interior, among a, b, c, d there are at most two distinct values, in which case one easily verifies the inequality. 4.35 Shortlisted Problems 1994 585 4.35 Solutions to the Shortlisted Problems of IMO 1994 1. Obviously a0 > a1 > a2 > · · · . Since ak − ak+1 = 1 − a 1+1 , we have an = a0 + k (a1 − a0 ) + · · ·+ (an − an−1) = 1994 − n + a01+1 + · · · + an−11 +1 > 1994 − n. Also, for 1 ≤ n ≤ 998, 1 1 n 998 + ···+ < < <1 a0 + 1 an−1 + 1 an−1 + 1 a997 + 1 because as above, a997 > 997. Hence ⌊an ⌋ = 1994 − n. 2. We may assume that a1 > a2 > · · · > am . We claim that for i = 1, . . . , m, ai + am+1−i ≥ n + 1. Indeed, otherwise ai + am+1−i , . . . , ai + am−1 , ai + am are i different elements of A greater than ai , which is impossible. Now by adding for i = 1, . . . , m we obtain 2(a1 + · · · + am ) ≥ m(n + 1), and the result follows. 3. The last condition implies that f (x) = x has at most one solution in (−1, 0) and at most one solution in (0, ∞). Suppose that for u ∈ (−1, 0), f (u) = u. Then putting x = y = u in the given functional equation yields f (u2 + 2u) = u2 + 2u. Since u ∈ (−1, 0) ⇒ u2 + 2u ∈ (−1, 0), we deduce that u2 + 2u = u, i.e., u = −1 or u = 0, which is impossible. Similarly, if f (v) = v for v ∈ (0, ∞), we are led to the same contradiction. However, for all x ∈ S we have f (x + (1 + x) f (x)) = x + (1 + x) f (x), x so we must have x + (1 + x) f (x) = 0. Therefore f (x) = − 1+x for all x ∈ S. It is directly verified that this function satisfies all the conditions. 4. Suppose that α = β . The given functional equation for x = y yields f (x/2) = x−α f (x)2 /2; hence the functional equation can be written as 1 1 f (x) f (y) = xα y−α f (y)2 + yα x−α f (x)2 , 2 2  2 (x/y)α /2 f (y) − (y/x)α /2 f (x) = 0. i.e., Hence f (x)/xα = f (y)/yα for all x, y ∈ R+ , so f (x) = λ xα for some λ . Substituting into the functional equation we obtain that λ = 21−α or λ = 0. Thus either f (x) ≡ 21−α xα or f (x) ≡ 0. Now let α 6= β . Interchanging x with y in the given equation and subtracting these equalities from each other, we get (xα − xβ ) f (y/2) = (yα − yβ ) f (x/2), so for some constant λ ≥ 0 and all x 6= 1, f (x/2) = λ (xα − xβ ). Substituting this into the given equation, we obtain that only λ = 0 is possible, i.e., f (x) ≡ 0. 5. If f (n) (x) = pn (x) qn (x) for some positive integer n and polynomials pn , qn , then f (n+1) (x) = f  pn (x) qn (x)  = pn (x)2 + qn (x)2 . 2pn (x)qn (x) 586 4 Solutions Note that f (0) (x) = x/1. Thus f (n) (x) = als pn , qn is defined recursively by pn (x) , qn (x) where the sequence of polynomi- p0 (x) = x, q0 (x) = 1, and pn+1 (x) = pn (x)2 + qn(x)2 , qn+1 (x) = 2pn (x)qn (x). Furthermore, p0 (x) ± q0 (x) = x ± 1 and pn+1 (x) ± qn+1 (x) = pn (x)2 + qn (x)2 ± n 2pn (x)qn (x) = (pn (x) ± qn (x))2 , so pn (x) ± qn(x) = (x ± 1)2 for all n. Hence n pn (x) = (x + 1)2 + (x − 1)2 2 n n and qn (x) = n (x + 1)2 − (x − 1)2 . 2 Finally, n n f (n) (x) pn (x)qn+1 (x) 2pn (x)2 ((x + 1)2 + (x − 1)2 )2 = = = (n+1) qn (x)pn+1 (x) pn+1 (x) (x + 1)2n+1 + (x − 1)2n+1 f (x)  2n 2 x+1 1 x−1 = 1+  n+1 = 1 +  x+1 2n  . x+1 2 f x−1 1 + x−1 6. Call the first and second player M and N respectively. N can keep A ≤ 6. Indeed, let 10 dominoes be placed as e shown in the picture, and whenever M d marks a 1 in a cell of some domino, c let N mark 0 in the other cell of that b domino if it is still empty. Since any a 3 × 3 square contains at least three 1 2 3 4 5 complete dominoes, there are at least three 0’s inside. Hence A ≤ 6. We now show that M can make A = 6. Let him start by marking 1 in c3. By symmetry, we may assume that N’s response is made in row 4 or 5. Then M marks 1 in c2. If N puts 0 in c1, then M can always mark two 1’s in b × {1, 2, 3} as well as three 1’s in {a, d} × {1, 2, 3}. Thus either {a, b, c} × {1, 2, 3} or {b, c, d} × {1, 2, 3} will contain six 1’s. However, if N does not play his second move in c1, then M plays there, and thus he can easily achieve to have six 1’s either in {a, b, c} × {1, 2, 3} or {c, d, e} × {1, 2, 3}. 7. Let a1 , a2 , . . . , am be the ages of the male citizens (m ≥ 1). We claim that the age of each female citizen can be expressed in the form c1 a1 + · · · + cm am for some constants ci ≥ 0, and we will prove this by induction on the number n of female citizens. The claim is clear if n = 1. Suppose it holds for n and consider the case of n + 1 female citizens. Choose any of them, say A of age x who knows k citizens (at least one male). By the induction hypothesis, the age of each of the other n females is expressible as c1 a1 + · · · + cm am + c0 x, where ci ≥ 0 and c0 + c1 + · · · + cm = 1. Consequently, the sum of ages of the k citizens who know A is 4.35 Shortlisted Problems 1994 8. 9. 587 kx = b1 a1 + · · · + bm am + b0 x for some constants bi ≥ 0 with sum k. But A knows at least one male citizen (who does not contribute to the coefficient of x), so m am b0 ≤ k − 1. Hence x = b1 a1 +···+b , and the claim follows. k−b0 (a) Let a, b, c, a ≤ b ≤ c be the amounts of money in dollars in Peter’s first, second, and third account, respectively. If a = 0, then we are done, so suppose that a > 0. Let Peter make transfers of money into the first account as follows. Write b = aq + r with 0 ≤ r < a and let q = m0 + 2m1 + · · · + 2k mk be the binary representation of q (mi ∈ {0, 1}, mk = 1). In the ith transfer, i = 1, 2, . . . , k + 1, if mi = 1 he transfers money from the second account, while if mi = 0 he does so from the third. In this way he has transferred exactly (m0 + 2m1 + · · · + 2k mk )a dollars from the second account, thus leaving r dollars in it, r < a. Repeating this procedure, Peter can diminish the amount of money in the smallest account to zero, as required. (b) If Peter has an odd number of dollars, he clearly cannot transfer his money into one account. (a) For i = 1, . . . , n, let di be 0 if the card i is in the ith position, and 1 otherwise. Define b = d1 + 2d2 + 22 d3 + · · · + 2n−1 dn , so that 0 ≤ b ≤ 2n − 1, and b = 0 if and only if the game is over. After each move some digit dl changes from 1 to 0 while dl+1 , dl+2 , . . . remain unchanged. Hence b decreases after each move, and consequently the game ends after at most 2n − 1 moves. (b) Suppose the game lasts exactly 2n − 1 moves. Then each move decreases b for exactly one, so playing the game in reverse (starting from the final configuration), every move is uniquely determined. It follows that if the configuration that allows a game lasting 2n − 1 moves exists, it must be unique. Consider the initial configuration 0, n, n −1, . . .,2, 1. We prove by induction that the game will last exactly 2n − 1 moves, and that the card 0 will get to the 0th position only in the last move. This is trivial for n = 1, so suppose that the claim is true for some n = m − 1 ≥ 1 and consider the case n = m. Obviously the card 0 does not move until the card m gets to the 0-th position. But if we ignore the card 0 and consider the card m to be the card 0, the induction hypothesis gives that the card m will move to the 0th position only after 2m−1 − 1 moves. After these 2m−1 − 1 moves, we come to the configuration 0, m − 1, . . . , 2, 1, m. The next move yields m, 0, m − 1, . . . , 2, 1, so by the induction hypothesis again we need 2m−1 − 1 moves more to finish the game. 10. (a) The case n > 1994 is trivial. Suppose that n = 1994. Label the girls G1 to G1994 , and let G1 initially hold all the cards. At any moment give to each card the value i, i = 1, . . . , 1994, if Gi holds it. Define the characteristic C of a position as the sum of all these values. Initially C = 1994. In each move, if Gi passes cards to Gi−1 and Gi+1 (where G0 = G1994 and G1995 = G1 ), C changes for ±1994 or does not change, so that it remains divisible by 588 4 Solutions 1994. But if the game ends, the characteristic of the final position will be C = 1 + 2 + · · ·+ 1994 = 997 · 1995, which is not divisible by 1994. (b) Whenever a card is passed from one girl to another for the first time, let the girls sign their names on it. Thereafter, if one of them passes a card to her neighbor, we shall assume that the passed card is exactly the one signed by both of them. Thus each signed card is stuck between two neighboring girls, so if n < 1994, there are two neighbors who never exchange cards. Consequently, there is a girl G who played only a finite number of times. If her neighbor plays infinitely often, then after her last move, G will continue to accumulate cards indefinitely, which is impossible. Hence every girl plays finitely many times. 11. Tile the table with dominoes and numbers as shown in the picture. The second player will not lose if whenever the first player plays in a cell of a domino, he plays in the other cell of the same domino. However, if the first player plays in a cell with a number, the second plays in the cell with same number that is diagonally adjacent. 2 1 1 2 2 1 1 2 4 3 2 1 1 2 3 4 4 3 2 1 3 4 4 3 2 1 1 2 3 4 4 3 2 1 3 4 4 3 2 1 1 2 3 4 1 2 3 4 4 3 1 2 3 4 4 3 12. Define Sn recursively as follows: Let S2 = {(0, 0), (1, 1)} and Sn+1 = Sn ∪ Tn , where Tn = {(x + 2n−1, y + Mn ) | (x, y) ∈ Sn }, with Mn chosen large enough so that the entire set Tn lies above every line passing through two points of Sn . By definition, Sn has exactly 2n−1 points and contains no three collinear points. We claim that no 2n points of this set are the vertices of a convex 2n-gon. Consider an arbitrary convex polygon P with vertices in Sn . Join by a diagonal d the two vertices of P having the smallest and greatest x-coordinates. This diagonal divides P into two convex polygons P1 , P2 , the former lying above d. We shall show by induction that both P1 , P2 have at most n vertices. Assume to the contrary that P1 has at least n + 1 vertices A1 (x1 , y1 ), . . . , An+1 (xn+1 , yn+1 ) in Sn , with x1 < · · · < xn+1 . It follows that y2 − y1 yn+1 − yn > ··· > . x2 − x1 xn+1 − xn By the induction hypothesis, not more than n − 1 of these vertices belong to Sn−1 or Tn−1 , so let Ak−1 , Ak ∈ Sn−1 , Ak+1 ∈ Tn−1 . But by the construction of y −y y −yk Tn−1 , xk+1 > xkk −xk−1 , which gives a contradiction. Similarly, P2 has no more k+1 −xk k−1 than n vertices, and therefore P itself has at most 2n − 2 vertices. 13. Extend AD and BC to meet at P, and let Q be the foot of the perpendicular from P to AB. Denote by O the center of Γ . Since △PAQ ∼ △OAD and △PBQ ∼ AQ PQ PQ AQ BC PD △OBC, we obtain AD = OD = OC = BQ BC . Therefore QB · CP · DA = 1, so by the converse Ceva theorem, AC, BD, and PQ are concurrent. It follows that Q ≡ F. 4.35 Shortlisted Problems 1994 589 Finally, since the points O,C, P, D, F are concyclic, we have ∠DFP = ∠DOP = ∠POC = ∠PFC. 14. Although it does not seem to have been noticed at the jury, the statement of the problem is false. For A(0, 0), B(0, 4),C(1, 4), D(7, 0), we have M(4, 2), P(2, 1), Q(2, 3) and N(9/2, 1/2) 6∈ △ABM. The official solution, if it can be called so, actually shows that N lies inside ABCD and goes as follows: The case AD = BC is trivial, so let AD > BC. Let L be the midpoint of AB. Complete the parallelograms ADMX and BCMY . Now N = DX ∩ CY , so let CY and DX intersect AB at K and H respectively. From LX = LY and HL HA LA KB KB KL = < < < = LX AD AD AD BC LY we get HL < KL, and the statement follows. 15. We shall prove that AD is a common tangent of ω and ω2 . Denote by K, L the points of tangency of ω with l1 and l2 respectively. Let r, r1 , r2 be the radii of ω , ω1 , ω2 respectively, and set KA = x, LB = y. It will be enough if we show that xy = 2r2 , since this will imply that △KLB and △AKO are similar, where O is the center of ω , and consequently that OA ⊥ KD (because D ∈ KB). Now if O1 is the center of ω1 , we have x2 = KA2 = OO21 − (KO − AO1 )2 = (r + r1 )2 − (r − r1 )2 = 4rr1 and analogously y2 = 4rr2 . But we also have (r1 + r2 )2 = O1 O22 = (x − y)2 + (2r − r1 − r2 )2 , so x2 − 2xy + y2 = 4r(r1 + r2 − r), from which we obtain xy = 2r2 as claimed. Hence AD is tangent to both ω , ω2 , and similarly BC is tangent to ω , ω1 . It follows that Q lies on the radical axes of pairs of circles (ω , ω1 ) and (ω , ω2 ). Therefore Q also lies on the radical axis of (ω1 , ω2 ), i.e., on the common tangent at E of ω1 and ω2 . Hence QC = QD = QE. Second solution. An inversion with center at D maps ω and ω2 to parallel lines, ω1 and l2 to disjoint equal circles touching ω , ω2 , and l1 to a circle externally tangent to ω1 , l2 , and to ω . It is easy to see that the obtained picture is symmetric (with respect to a diameter of l1 ), and that line AD is parallel to the lines ω and ω2 . Going back to the initial picture, this means that AD is a common tangent of ω and ω2 . The end is like that in the first solution. 16. First, assume that ∠OQE = 90◦ . Extend PN to meet AC at R. Then OEPQ and ORFQ are cyclic quadrilaterals; hence we have ∠OEQ = ∠OPQ = ∠ORQ = ∠OFQ. It follows that △OEQ ∼ = △OFQ and QE = QF. A Now suppose QE = QF. Let S be the point symmetric to A with respect to Q, so that the quadrilateral AESF is a parF allelogram. Draw the line E ′ F ′ through Q P R Q so that ∠OQE ′ = 90◦ and E ′ ∈ AB, N E F ′ ∈ AC. By the first part QE ′ = QF ′ ; ′ ′ O hence AE SF is also a parallelogram. C B It follows that E ≡ E ′ , F ≡ F ′ , and S ∠OQE = 90◦ . 590 4 Solutions 17. We first prove that AB cuts OE in a fixed point H. Note that ∠OAH = ∠OMA = ∠OEA (because O, A, E, M lie on a circle); hence △OAH ∼ △OEA. This implies OH · OE = OA2 , i.e., H is fixed. Let the lines AB and CD meet at K. Since EAOBM and ECDM are cyclic, we have ∠EAK = ∠EMB = ∠ECK, so ECAK is cyclic. l E K F M C D A H O B Therefore ∠EKA = 90◦ , hence EKBD is also cyclic and EK k OM. Then ∠EKF = ∠EBD = ∠EOM = ∠OEK, from which we deduce that KF = FE. However, since ∠EKH = 90◦ , the point F is the midpoint of EH; hence it is fixed. 18. Since for each of the subsets {1, 4, 9}, {2, 6, 12}, {3, 5, 15} and {7, 8, 14} the product of its elements is a square and these subsets are disjoint, we have |M| ≤ 11. Suppose that |M| = 11. Then 10 ∈ M and none of the disjoint subsets {1, 4, 9}, {2, 5}, {6, 15}, {7, 8, 14} is a subset of M. Consequently {3, 12} ⊂ M, so none of {1}, {4}, {9}, {2, 6}, {5, 15}, and {7, 8, 14} is a subset of M: thus |M| ≤ 9, a contradiction. It follows that |M| ≤ 10, and this number is attained in the case M = {1, 4, 5, 6, 7, 10, 11, 12, 13, 14}. 19. Since mn − 1 and m3 are relatively prime, mn − 1 divides n3 + 1 if and only if it divides m3 (n3 + 1) = (m3 n3 − 1) + m3 + 1. Thus n3 + 1 m3 + 1 ∈Z⇔ ∈ Z; mn − 1 mn − 1 3 1 hence we may assume that m ≥ n. If m = n, then nn2 +1 = n + n−1 is an integer, −1 2 so m = n = 2. If n = 1, then m−1 ∈ Z, which happens only when m = 2 or m = 3. Now suppose m > n ≥ 2. Since m3 + 1 ≡ 1 and mn − 1 ≡ −1 (mod n), n3 +1 we deduce mn−1 = kn − 1 for some integer k > 0. On the other hand, kn − 1 < n3 +1 n2 −1 1 = n + n−1 ≤ 2n − 1 gives that k = 1, and therefore n3 + 1 = (mn − 1)(n − 1). 2 +1 2 This yields m = nn−1 = n + 1 + n−1 ∈ N, so n ∈ {2, 3} and m = 5. The solutions with m < n are obtained by symmetry. There are 9 solutions in total: (1, 2), (1, 3), (2, 1), (3, 1), (2, 2), (2, 5), (3, 5), (5, 2), (5, 3). 20. Let A be the set of all numbers of the form p1 p2 . . . p p1 , where p1 < p2 < · · · < p p1 are primes. In other words, A = {2 · 3, 2 · 5, . . .} ∪ {3 · 5 · 7, 3 · 5 · 11, . . .} ∪ {5 · 7 · 11 · 13 · 17, . . .} ∪ · · ·. This set satisfies the requirements of the problem. Indeed, for any infinite set of primes P = {q1 , q2 , . . . } (where q1 < q2 < · · · ) we have m = q1 q2 · · · qq1 ∈ A and n = q2 q3 · · · qq1 +1 6∈ A. 4.35 Shortlisted Problems 1994 591 21. Note first that yn = 2k (k ≥ 2) and zk ≡ 1 (mod 4) for all n, so if xn is odd, xn+1 will be even. Further, it is shown by induction on n that yn > zn when xn−1 is even and 2yn > zn > yn when xn−1 is odd. In fact, n = 1 is the trivial case, while if it holds for n ≥ 1, then yn+1 = 2yn > zn = zn+1 if xn is even, and 2yn+1 = 2yn > yn + zn = zn+1 if xn is odd (since then xn−1 is even). If x1 = 0, then x0 = 3 is good. Suppose xn = 0 for some n ≥ 2. Then xn−1 is odd and xn−2 is even, so that yn−1 > zn−1 . We claim that a pair (yn−1 , zn−1 ), where 2k = yn−1 > zn−1 > 0 and zn−1 ≡ 1 (mod 4), uniquely determines x0 = f (yn−1 , zn−1 ). We see that xn−1 = 12 yn−1 + zn−1 , and define (xk , yk , zk ) backwards as follows, until we get (yk , zk ) = (4, 1). If yk > zk , then xk−1 must have been even, so we define (xk−1 , yk−1 , zk−1 ) = (2xk , yk /2, zk ); otherwise xk−1 must have been odd, so we put (xk−1 , yk−1 , zk−1 ) = (xk − yk /2 + zk , yk , zk − yk ). We eventually arrive at (y0 , z0 ) = (4, 1) and a good integer x0 = f (yn−1 , zn−1 ), as claimed. Thus for example (yn−1 , zn−1 ) = (64, 61) implies xn−1 = 93, (xn−2 , yn−2 , zn−2 ) = (186, 32, 61) etc., and x0 = 1953, while in the case of (yn−1 , zn−1 ) = (128, 1) we get x0 = 2080. Note that y′ > y ⇒ f (y′ , z′ ) > f (y, z) and z′ > z ⇒ f (y, z′ ) > f (y, z). Therefore there are no y, z for which 1953 < f (y, z) < 2080. Hence all good integers less than or equal to 1994 are given as f (y, z), y = 2k ≤ 64 and 0 < z ≡ 1 (mod 4), and the number of such (y, z) equals 1 + 2 + 4 + 8 + 16 = 31. So the answer is 31. 22. (a) Denote by b(n) the number of 1’s in the binary representation of n. Since b(2k + 2) = b(k + 1) and b(2k + 1) = b(k) + 1, we deduce that  f (k) + 1, if b(k) = 2; f (k + 1) = (1) f (k), otherwise. The set of k’s with b(k) = 2 is infinite, so it follows that f (k) is unbounded. Hence f takes all natural values. (b) Since f is increasing, k is a unique solution of f (k) = m if and only if f (k − 1) < f (k) < f (k + 1). By (1), this inequality is equivalent to b(k − 1) = b(k) = 2. It is easy to see that then k − 1 must be of the form 2t + 1 for some t. In this case, {k + 1, . . . , 2k} contains the number 2t+1 + 3 = 10 . . . 0112 and t(t−1) binary (t + 1)-digit numbers with three 1’s, so m = 2 t(t−1) f (k) = 2 + 1. 23. (a) Let p be a prime divisor of xi , i > 1, and let x j ≡ u j (mod p) where 0 ≤ u j ≤ p − 1 (particularly ui ≡ 0). Then u j+1 ≡ u j u j−1 + 1 (mod p). The number of possible pairs (u j , u j+1 ) is finite, so u j is eventually periodic. We claim that for some d p > 0, ui+d p = 0. Indeed, suppose the contrary and let (um , um+1 , . . . , um+d−1 ) be the first period for m ≥ i. Then m 6= i. By the assumption um−1 6≡ um+d−1 , but um−1 um ≡ um+1 − 1 ≡ um+d+1 − 1 ≡ um+d−1 um+d ≡ um+d−1 um (mod p), which is impossible if p ∤ um . Hence there is a d p with ui = ui+d p = 0 and moreover ui+1 = ui+d p +1 = 1, so the sequence u j is periodic with period d p starting from ui . Let m be the least 592 4 Solutions common multiple of all d p ’s, where p goes through all prime divisors of xi . Then the same primes divide every xi+km , k = 1, 2, . . . , so for large enough k and j = i + km, xii | x jj . (b) If i = 1, we cannot deduce that xi+1 ≡ 1 (mod p). The following example shows that the statement from (a) need not be true in this case. Take x1 = 22 and x2 = 9. Then xn is even if and only if n ≡ 1 (mod 3), but modulo 11 the sequence {xn } is 0, 9, 1, 10, 0, 1, 1, 2, 3, 7, 0, . . ., so 11 | xn (n > 1) if and only if n ≡ 5 (mod 6). Thus for no n > 1 can we have 22 | xn . 24. A multiple of 10 does not divide any wobbly number. Also, if 25 | n, then every multiple of n ends with 25, 50, 75, or 00; hence it is not wobbly. We now show that every other number n divides some wobbly number. (i) Let n be odd and not divisible by 5. For any k ≥ 1 there exists l such that (10k − 1)n divides 10l − 1, and thus also divides 10kl − 1. Consequently, kl −1 vk = 10 is divisible by n, and it is wobbly when k = 2 (indeed, v2 = 10k −1 101 . . . 01). If n is divisible by 5, one can simply take 5v2 instead. (ii) Let n be a power of 2. We prove by induction on m that 22m+1 has a wobbly multiple wm with exactly m nonzero digits. For m = 1, take w1 = 8. Suppose that for some m ≥ 1 there is a wobbly wm = 22m+1 dm . Then the numbers a · 102m + wm are wobbly and divisible by 22m+1 when a ∈ {2, 4, 6, 8}. Moreover, one of these numbers is divisible by 22m+3 . Indeed, it suffices to choose a such that a2 + dm is divisible by 4. This proves the induction step. (iii) Let n = 2m r, where m ≥ 1 and r is odd, 5 ∤ r. Then v2m wm is wobbly and divisible by both 2m and r (using notation from (i), r | v2m ). 4.36 Shortlisted Problems 1995 593 4.36 Solutions to the Shortlisted Problems of IMO 1995 1. Let x = 1a , y = 1b , z = 1c . Then xyz = 1 and S= 1 1 1 x2 y2 z2 + + = + + . a3 (b + c) b3 (c + a) c3 (a + b) y + z z + x x + y We must prove that S ≥ 32 . From the Cauchy–Schwarz inequality, x+y+z [(y + z) + (z + x) + (x + y)] · S ≥ (x + y + z)2 ⇒ S ≥ . 2 √ It follows from the A-G mean inequality that x+y+z ≥ 32 3 xyz = 32 ; hence the 2 proof is complete. Equality holds if and only if x = y = z = 1, i.e., a = b = c = 1. 2 2 2 y x z Remark. After reducing the problem to y+z + z+x + x+y ≥ 32 , we can solve the problem using Jensen’s inequality applied to the function g(u, v) = u2 /v. The problem can also be solved using Muirhead’s inequality. 2. We may assume c ≥ 0 (otherwise, we may simply put −yi in the place of yi ). Also, we may assume a ≥ b. If b ≥ c, it is enough to take n = a + b − c, x1 = · · · = xa = 1, y1 = · · · = yc = ya+1 = · · · = ya+b−c = 1, and the other xi ’s and yi ’s equal to 0, so we need only consider the case a > c > b. We proceed to prove the statement of the problem by induction on a + b. The case a + b = 1 is trivial. Assume that the statement is true when a + b ≤ N, and let a + b = N + 1. The triple (a + b − 2c, b, c − b) satisfies the condition (since (a + b − 2c)b − (c − b)2 = ab − c2 ), so by the induction hypothesis there are n-tuples (xi )ni=1 and (yi )ni=1 with the wanted property. It is easy to verify that (xi + yi )ni=1 and (yi )ni=1 give a solution for (a, b, c). a2 +a2 −a2 2a a i i+1 i+1 i+2 3. Write Ai = aii +ai+1 −ai+2 = ai + ai+1 + ai+2 − ai +ai+1 −ai+2 . Since 2ai ai+1 ≥ 4(ai + ai+1 − 2) (which is equivalent to  (ai − 2)(ai+1 − 2) ≥ 0), itfollows that  Ai ≤ ai + ai+2 −2 ai+2 −2 ai+1 + ai+2 − 4 1 + ai +ai+1 −ai+2 ≤ ai + ai+1 + ai+2 − 4 1 + 4 , because n 1 ≤ ai + ai+1 − ai+2 ≤ 4. Therefore Ai ≤ ai + ai+1 − 2, so ∑i=1 Ai ≤ 2s − 2n as required. 4. The second equation is equivalent to √b , z1 zx = a2 yz 2 2 + bzx + cxy + abc xyz = 4. Let x1 = √c . Then x2 +y2 + z2 + x1 y1 z1 1 1 1 xy √a , y1 yz = = 4, where 0 < x1 , y1 , z1 < 2. Regarding this as a quadratic equation in z1 , the discriminant (4 − x21 )(4 − y21 ) suggests that we let x1 = 2 sin u, y1 = 2 sin v, 0 < u, v < π /2. Then it is directly shown that z1 will be exactly 2 cos(u + v) as the only positive solution of the quadratic equation. √ √ √ Thus a = 2 yz sin u, b = 2 xz sin v, c = 2 xy(cos u cosv − sin u sin v), so from x + y + z − a − b − c = 0 we obtain √ √ √ √ √ ( x cos v − y cos u)2 + ( x sin v + y sin u − z)2 = 0, 594 4 Solutions which implies √ z= √ √ 1 √ √ 1 x sin v + y sin u = (y1 x + x1 y) = 2 2   b √ a √ √ x+ √ y . zx yz c+a Therefore z = a+b x = b+c 2 . Similarly, 2 and y = 2 . It is clear that the triple  b+c c+a a+b (x, y, z) = 2 , 2 , 2 is indeed a (unique) solution of the given system of equations. c+a a+b b+c Second solution. Put x = b+c 2 − u, y = 2 − v, z = 2 − w, where u ≤ 2 , c+a a+b 2 2 2 v ≤ 2 , w ≤ 2 and u + v + w = 0. The equality abc + a x + b y + c z = 4xyz becomes 2(au2 + bv2 + cw2 + 2uvw) = 0. Now uvw > 0 is clearly impossible. On the other hand, if uvw ≤ 0, then two of u, v, w are nonnegative, say u, v ≥ 0. Taking into account w = −u −v, the above equality reduces to 2[(a + c−2v)u2 + (b + c − 2u)v2 + 2cuv] = 0, so u = v = 0. Third solution. The fact that we are given two equations and three variables suggests that this is essentially a problem on inequalities. Setting f (x, y, z) = 4xyz − a2 x − b2 y − c2 z, we should show that max f (x, y, z) = abc, for 0 < x, y, z, x + y + z = a + b + c, and find when this value is attained. Thus we apply Lagrange multipliers to F(x, y, z) = f (x, y, z) − λ (x + y + z − a − b − c), and obtain that f takes a maximum at (x, y, z) such that 4yz − a2 = 4zx − b2 = 4xy − c2 = λ and x + y+ z = a + b + c. The only solution of this system is c+a a+b (x, y, z) = b+c 2 , 2 , 2 . 5. Suppose that a function f satisfies the condition, and let c be the least upper bound of { f (x) | x ∈ R}. We have c ≥ 2, since f (2) = f (1 + 1/12 ) = f (1) + f (1)2 = 2. Also, since c is the least upper bound, for each k = 1, 2, . . . there is an xk ∈ R such that f (xk ) ≥ c − 1/k. Then c≥ f  1 xk + 2 xk  1 ≥ c− + f k  1 xk 2 =⇒ f  1 xk  1 ≥ −√ . k On the other hand,       1 1 1 1 2 2 2 c≥ f + xk = f + f (xk ) ≥ − √ + c − . xk xk k k It follows that   1 1 2 √ − 2 ≥ c c−1− , k k k which cannot hold for k sufficiently large. Second solution. Assume that f exists and let n be the least integer such that f (x) ≤ n4 for all x. Since f (2) = 2, we have n ≥ 8. Let f (x) > n−1 4 . Then f (1/x) = f (x + 1/x2 ) − f (x) < 1/4, so f (1/x) > −1/2. On the other hand, this implies  n−1 2 < f (x)2 = f (1/x + x2 ) − f (1/x) < n4 + 12 , which is impossible when 4 n ≥ 8. 4.36 Shortlisted Problems 1995 6. Let yi = xi+1 + · · · + xn , Y = ∑nj=2 ( j − 1)x j , and zi = n(n − 1) ∑ xi x j − 2 i< j n−1 ∑ (n − i)xi i=1 ! Y = n(n−1) 2 yi − (n − i)Y. 595 Then n−1 n(n − 1) n−1 xi yi − ∑ (n − i)xiY ∑ 2 i=1 i=1 n−1 = ∑ xi zi , i=1 n−1 n−1 so it remains to show that ∑n−1 i=1 xi zi > 0. Since ∑i=1 yi = Y and ∑i=1 (n − i) = n(n−1) n(n−1) n 2 , we have ∑ zi = 0. Note that Y < ∑ j=2 ( j − 1)xn = 2 xn , and conse- quently zn−1 = n(n−1) 2 xn − Y > 0. Furthermore, we have   zi+1 zi n(n − 1) yi+1 yi − = − > 0, n−i−1 n−i 2 n−i−1 n−i z z1 z2 which means that n−1 < n−2 < · · · < n−1 1 . Therefore there is a k for which z1 , . . . , zk ≤ 0 and zk+1 , . . . , zn−1 > 0. But then zi (xi − xk ) ≥ 0, i.e., xi zi ≥ xk zi n−1 for all i, so ∑n−1 i=1 xi zi > ∑i=1 xk zi = 0 as required. n Second solution. Set X = ∑n−1 j=1 (n − j)x j and Y = ∑ j=2 ( j − 1)x j . Since 4XY = 2 2 (X + Y ) − (X − Y ) , the RHS of the inequality becomes  !2 !2  n n 1 XY = (n − 1)2 ∑ xi − ∑ (2i − 1 − n)xi  . 4 i=1 i=1 The LHS is 1 4   (n − 1)2 (∑ni=1 xi )2 − (n − 1) ∑i< j (x j − xi )2 . Since ∑ni=1 (2i−1− n)xi = ∑i< j (x j − xi ) also holds, we must prove that !2 ∑ (x j − xi) i< j > (n − 1) ∑(x j − xi )2 . (1) i< j Putting xi+1 − xi = di > 0 (so, x j − xi = di + di+1 + · · · + d j−1 ) and expanding the obtained expressions, we reduce this inequality to ∑k k2 (n − k)2 dk2 + 2 ∑k<l kl(n − k)(n − l)dk dl > ∑k (n − 1)k(n − k)dk2 + 2 ∑k<l (n − 1)k(n − l)dk dl , which is verified immediately by comparing coefficients. Remark. An inequality significantly stronger than (1) in the second solution has appeared later, as IMO 03-5. 7. The result is trivial if O coincides with X or Y , so let us assume it does not. From OB · ON = OC · OM = OX · OY we deduce that BCMN is a cyclic quadrilateral. Further, if O lies between X and Y , then ∠MAD + ∠MND = ∠MAD + ∠MNB + ∠BND = ∠MAD + ∠MCA + ∠AMC = 180◦ . Similarly, we also have ∠MAD + ∠MND = 180◦ if O is not on the segment XY . Therefore ADNM is cyclic. Now let AM and DN intersect at Z and let the line ZX intersect the two circles at Y1 596 4 Solutions and Y2 . Then ZX · ZY1 = ZM · ZA = ZN · ZD = ZX · ZY2 . Hence Y1 = Y2 = Y , implying that Z lies on XY . Second solution. Let Z1 , Z2 be the points in which AM, DN respectively meet XY , and P = BC ∩ XY . Then, from △OPC ∼ △APZ1 , we have PZ1 = PA·PC PO = PX 2 PO and analogously PZ2 = ′ ′ ′ PX 2 PO . Hence, we conclude that Z1 ≡ Z2 . 8. Let A , B ,C be the points symmetric to A, B,C with respect to the midpoints of BC,CA, AB respectively. From the condition on X we have X B2 − XC2 = AC2 − AB2 = A′ B2 − A′C2 , and hence X must lie on the line through A′ perpendicular to BC. Similarly, X lies on the line through B′ perpendicular to CA. It follows that there is a unique position for X , namely the orthocenter of △A′ B′C′ . It easily follows that this point X satisfies the original equations. 9. If EF is parallel to BC, △ABC must be isosceles and E,Y are symmetric to F, Z with respect to AD, so the result follows. Now suppose that EF meets BC at P. BP AE BD By Menelaus’s theorem, CP = BF FA · EC = DC (since BD = BF, CD = CE, AE = AF). It follows that the point P depends only on D and not on A. In particular, the same point is obtained as the intersection of ZY with BC. Therefore PE · PF = PD2 = PY · PZ, from which it follows that EFZY is a cyclic quadrilateral. Second solution. Since CD = CY = CE and BD = BZ = BF, all angles of EFZY can be calculated in terms of angles of ABC and Y ZBC. In fact, ∠FEY = 12 (∠A + ∠C + ∠BCY ) and ∠FZY = 12 (180◦ + ∠B + ∠BCY ), which gives us ∠FEY + ∠FZY = 180◦ . 10. Let the two triangles be X1Y1 Z1 , X2Y2 Z2 , with X1 = BB1 ∩CC1 , Y1 = CC1 ∩ AA1 , Z1 =AA1 ∩ BB1 , X2 = BB2 ∩CC2 , Y2 = CC2 ∩ AA2 , Z2 = AA2 ∩ BB2 . First, let us observe that ∠ABB2 = C ∠ACC1 and ∠ABB1 = ∠ACC2 . We A2 now obtain that ∠BZ1 A1 = ∠BAA1 + B1 ∠ABB1 = ∠BCC2 + ∠C2CA = ∠C and Y2 X1 A1 similarly ∠AZ2 B2 = ∠C, ∠AY1C1 = Z1 B 2 Z2 ∠CY2 A2 = ∠B. Also the triangles ABB2 and ACC1 are similar; hence Y1 X2 AC1 /AC = AB2 /AB. From the law of sines we obtain B A C1 C2 AZ1 AB AB AC AC = = = = sin ∠ABZ1 sin ∠AZ1 B sin ∠C sin ∠B sin ∠AY2C AY2 = =⇒ AZ1 = AY2 . sin ∠ACY2 Analogously, BX1 = BZ2 and CY1 = CX2 . Furthermore, again from the sine formula, 4.36 Shortlisted Problems 1995 597 AY1 AC1 AC1 AC = = sin ∠AC1Y1 sin ∠AY1C1 AC sin ∠B AB2 AB AB2 AZ2 = = = . AB sin ∠C sin ∠AZ2 B2 sin ∠AB2 Z2 Hence, AY1 = AZ2 and, analogously, BZ1 = BX2 and CX1 = CY2 . We deduce that Y1 Z2 k BC and Z2 X1 k AC, which gives us ∠Y1 Z2 X1 = 180◦ − ∠C = 180◦ − ∠Y1 Z1 X1 . It follows that Z2 lies on the circle circumscribed about △X1Y1 Z1 . Similarly, so do X2 and Y2 . Second solution. Let H be the orthocenter of △ABC. Triangles AHB, BHC, CHA, ABC have the same circumradius R. Additionally, ∠HAAi = ∠HBBi = ∠HCCi = θ (i = 1, 2). Since ∠HBX1 = ∠HCX1 = θ , BCX1 H is concyclic and therefore HX1 = 2R sin θ . The same holds for HY1 , HZ1 , HX2 , HY2 , HZ2 . Hence Xi ,Yi , Zi (i = 1, 2) lie on a circle centered at H. 11. Triangles BCD and EFA are equilateral, and hence BE is an axis of symmetry of ABDE. Let C′ , F ′ respectively be the points symmetric to C, F with respect to BE. The points G and H lie on the circumcircles of ABC′ and DEF ′ respectively (because, for instance, ∠AGB = 120◦ = 180◦ − ∠AC′B); hence from Ptolemy’s theorem we have AG + GB = C′ G and DH + HE = HF ′ . Therefore AG + GB + GH + DH + HE = C′ G + GH + HF ′ ≥ C′ F ′ = CF, with equality if and only if G and H both lie on C′ F ′ . Remark. Since by Ptolemy’s inequality AG + GB ≥ C′ G and DH + HE ≥ HF ′ , the result holds without the condition ∠AGB = ∠DHE = 120◦. 12. Let O be the circumcenter and R the circumradius of A1 A2 A3 A4 . We have OA2i = −→ −→ −→ −→ −−→ (OG + (OAi − OG))2 = OG2 + GA2i + 2OG · GAi . Summing up these equalities − − → → − for i = 1, 2, 3, 4 and using that ∑4i=1 GAi = 0 , we obtain 4 4 ∑ OA2i = 4OG2 + ∑ GA2i i=1 i=1 4 ⇐⇒ ∑ GA2i = 4(R2 − OG2). (1) i=1 Now we have that the potential of G with respect to the sphere equals GAi ·GA′i = R2 − OG2 . Plugging in these expressions for GA′i , we reduce the inequalities we must prove to GA1 · GA2 · GA3 · GA4 ≤ (R2 − OG2 )2 (2) and 4 4 1 ≥ ∑ GAi . i=1 GAi i=1 (R2 − OG2 ) ∑ (3) Inequality (2) immediately follows from (1) and the quadratic-geometric mean inequality for GAi . From the Cauchy–Schwarz inequality we have ∑4i=1 GA4i ≥ 598 4 Solutions  2  1 ≥ 16, hence the inequality (3) fol∑4i=1 GAi and ∑4i=1 GAi ∑4i=1 GA i lows from (1) and from 1 4 4 ∑ i=1 GA2i ! 4 1 ∑ GAi i=1 ! 1 ≥ 4 4 ∑ GAi i=1 !2 4 1 ∑ GAi i=1 ! 4 ≥ 4 ∑ GAi . i=1 13. If O lies on √ AC, then √ ABCD, √ AKON, and OLCM are similar; hence AC = AO + OC implies S = S1 + S2 . D Assume that O does not lie on AC and that w.l.o.g. it lies inside triangle ADC. M N T2 Let us denote by T1 , T2 the areas of parC A allelograms KBLO, NOMD respectS1 S2 ively. Consider a line through O that X W O Z Y intersects AD, DC, CB, BA respectT1 ively at X , Y , Z, W so that OW /OX = K L OZ/OY (such a line exists by a continuity argument: the left side is smaller B when W = X = A, but greater when √ Y = Z = C). The desired inequality is equivalent to T1 + T2 ≥ 2 S1√ S2 . Since triangles W KO, OLZ, W BZ are similar and WO + OZ = W Z, we have √ √ √ √ SW KO + SOLZ = SW BZ√= SW KO + SOLZ + T1 , which implies T1 = 2 SW KO SOLZ . Similarly, T2 = 2 SXNO SOMY . Since OW /OZ = OX/OY , we have SW KO /SXNO = SOLZ /SOMY . Therefore we obtain p p T1 + T2 = 2 SW KO SOLZ + 2 SXNO SOMY p p = 2 (SW KO + SXNO )(SOLZ + SOMY ) ≥ 2 S1 S2 . Second solution. Using an affine transformation of the plane one can transform any nondegenerate quadrilateral into a cyclic one, thereby preserving parallelness and ratios of areas. Thus we may assume w.l.o.g. that ABCD is cyclic. By a well-known formula, the area of a cyclic quadrilateral with sides a, b, c, d and semiperimeter p is given by p S = (p − a)(p − b)(p − c)(p − d). Let us set AK = a1 , KB = b1 , BL = a2 , LC = b2 , CM = a3 , MD = b3 , DN = a4 , NA = b4 . Then the sides of quadrilateral AKON are ai , the sides of CLOM are bi , and the sides of ABCD are ai + bi (i = 1, 2, 3, 4). If p and q are the semiperimeters √ of AKON and CLOM, and xi = p − ai , yi = q − bi , then we have S1 = x1 x2 x3 x4 , p √ S2 = y1 y2 y3 y4 , and S = (x1 + y1 )(x2 + y2 )(x3 + y3)(x4 + y4 ) . Thus we need to show that p √ √ 4 x x x x + 4 y y y y ≤ 4 (x + y )(x + y )(x + y )(x + y ) . 1 2 3 4 1 2 3 4 1 1 2 2 3 3 4 4 4.36 Shortlisted Problems 1995 599 By setting yi = ti xi we reduce this inequality to p √ 1 + 4 t1t2t3 t4 ≤ 4 (1 + t1 )(1 + t2 )(1 + t3 )(1 + t4 ) . One way to prove the last inequality is to apply the simple inequality p √ 1 + uv ≤ (1 + u)(1 + v) √ √ to t1 t2 , t3 t4 and then to t1 ,t2 and t3 ,t4 . 14. Let BB′ cut CC′ at P. Since ∠B′ BC′ = ∠B′CC′ , it follows that ∠PBH = ∠PCH. Let D and E be points such that BPCD and HPCE are parallelograms (consequently, so is BHED). Triangles BAC and C′ AB′ are similar, from which we A deduce that △B′ H ′C′ and △BHC ′ ′ H ′ B′ are similar, as well as △B PC and ′ C △BDC. Hence B′ PC′ H ′ and BDCH P are similar, from which we obtain ∠H ′ PB′ = ∠HDB. Now ∠CDE = H ∠PBH = ∠PCH = ∠CHE implies that B C HCED is a cyclic quadrilateral. Therefore ∠BPH = ∠DCE = ∠DHE = E ∠HDB = ∠H ′ PB′ ; hence HH ′ also D passes through P. Second solution. Let us start with observations △HBC ∼ △H ′ B′C′ , ∠PBH = ∠PCH, and ∠PB′ H ′ = ∠PC′ H ′ . By Ceva’s theorem in trigonometric form applied to △BPC and the point H, sin ∠BPH sin ∠HBP sin ∠HCB sin ∠HCB we have sin ∠HPC = sin ∠HBC · sin ∠HCP = sin ∠HBC . Similarly, Ceva’s theorem for ′ PH ′ sin ∠B ∠H ′ C′ B′ △B′ PC′ and point H ′ yields sin∠H ′ PC′ = sin sin ∠H ′ B′C′ . Thus it follows that sin ∠B′ PH ′ sin ∠BPH = , sin ∠H ′ PC′ sin ∠HPC which finally implies that ∠BPH = ∠B′ PH ′ . 15. We show by induction on k that there exists a positive integer ak for which a2k ≡ −7 (mod 2k ). The statement of the problem follows, since every ak + r2k (r = 0, 1, . . . ) also satisfies this condition. Note that for k = 1, 2, 3 one can take ak = 1. Now suppose that a2k ≡ −7 (mod 2k ) for some k > 3. Then either a2k ≡ −7 (mod 2k+1 ) or a2k ≡ 2k − 7 (mod 2k+1 ). In the former case, take ak+1 = ak . In the latter case, set ak+1 = ak + 2k−1 . Then a2k+1 = a2k + 2k ak + 22k−2 ≡ a2k + 2k ≡ −7 (mod 2k+1 ) because ak is odd. 16. If A is odd, then every number in M1 is of the form x(x + A) + B ≡ B (mod 2), while numbers in M2 are congruent to C modulo 2. Thus it is enough to take C ≡ B + 1 (mod 2). 2 2 If A is even, then all numbers in M1 have the form X + A2 + B − A4 and are 2 2 congruent to B − A4 or B − A4 + 1 modulo 4, while numbers in M2 are congruent 2 to C modulo 4. So one can choose any C ≡ B − A4 + 2 (mod 4). 600 4 Solutions 17. For n = 4, the vertices of a unit square A1 A2 A3 A4 and p1 = p2 = p3 = p4 = 16 satisfy the conditions. We claim that there are no solutions for n = 5 (and thus for any n ≥ 5). Suppose to the contrary that points Ai and pi , i = 1, . . . , 5, satisfy the conditions. Denote the area of △Ai A j Ak by Si jk = pi + p j + pk , 1 ≤ i < j < k ≤ 5. Observe that all the pi ’s must be distinct. Indeed, if p4 = p5 , then S124 = S125 and S234 = S235 , which implies that A4 A5 is parallel to A1 A2 and A2 A3 , so A1 , A2 , A3 are collinear, which is impossible. Also note that if Ai A j Ak Al is convex, then Si jk + Sikl = Si jl + S jkl gives pi + pk = p j + pl . Now consider the convex hull of A1 , A2 , A3 , A4 , A5 . There are three cases. (i) The convex hull is the pentagon A1 A2 A3 A4 A5 . We deduce that the quadrilaterals A1 A2 A3 A4 and A1 A2 A3 A5 are convex, so we have p1 + p3 = p2 + p4 and p1 + p3 = p2 + p5 . Hence p4 = p5 , a contradiction. (ii) The convex hull is w.l.o.g. the quadrilateral A1 A2 A3 A4 . Assume that A5 lies within A1 A3 A4 . Then A1 A2 A3 A5 is also convex, so as in (1) we get p4 = p5 . (iii) The convex hull is w.l.o.g. the triangle A1 A2 A3 . Since S124 + S134 + S234 = S125 + S135 + S235, we conclude that again p4 = p5 . 18. Let x = za and y = zb, where a and b are relatively prime. The given Diophantine equation becomes a + zb2 + z2 = z2 ab, so a = zc for some c ∈ Z. We obtain 2 +z c + b2 + z = z2 cb, or c = zb2 b−1 . 2 +1 2 (i) If z = 1, then c = bb−1 = b + 1 + b−1 , so b = 2 or b = 3. These values yield two solutions: (x, y) = (5, 2) and (x, y) = (5, 3). 2 +32 33 (ii) If z = 2, then 16c = 16b 4b−1 = 4b + 1 + 4b−1 , so b = 1 or b = 3. In this case (x, y) = (4, 2) or (x, y) = (4, 6). 2 2 +z3 3 b+z3 (iii) Let z ≥ 3. First, we see that z2 c = zz2bb−1 = b + zb+z 2 b−1 . Thus z2 b−1 must 2 −z+1 be a positive integer, so b + z3 ≥ z2 b − 1, which implies b ≤ z z−1 . It 2 2 2 follows that b ≤ z. But then b +z ≤ z +b < z b−1, with the last inequality 2 +z because (z2 − 1)(b − 1) > 2. Therefore c = zb2 b−1 < 1, a contradiction. The only solutions for (x, y) are (4, 2), (4, 6), (5, 2), (5, 3). 19. For each two people let n be the number of people exchanging greetings with both of them. To determine n in terms of k, we shall count in two ways the number of triples (A, B,C) of people such that A exchanged greetings with both B and C, but B and C mutually did not. There are 12k possibilities for A, and for each A there are (3k + 6) possibilities for B. Since there are n people who exchanged greetings with both A and B, there are 3k + 5 − n who did so with A but not with B. Thus the number of triples (A, B,C) is 12k(3k + 6)(3k + 5 − n). On the other hand, there are 12k possible choices of B, and 12k − 1 − (3k + 6) = 9k − 7 possible choices of C; for every B,C, A can be chosen in n ways, so the number of considered triples equals 12kn(9k − 7). 4.36 Shortlisted Problems 1995 Hence (3k + 6)(3k + 5 − n) = n(9k − 7), i.e., n = 4n 3 12k2 +44k+40 12k−1 3(k+2)(3k+5) . 12k−1 601 This gives us that 3k−44 12k−1 = = k+4− is an integer too. It is directly verified that only k = 3 gives an integer value for n, namely n = 6. Remark. The solution is complete under the assumption that such a k exists. We give an example of such a party with 36 persons, k = 3. Let the people sit in a 6 × 6 array [Pi j ]6i, j=1 , and suppose that two persons Pi j , Pkl exchanged greetings if and only if i = k or j = l or i − j ≡ k − l (mod 6). Thus each person exchanged greetings with exactly 15 others, and it is easily verified that this party satisfies the conditions. 20. We shall consider the set M = {0, 1, . . . , 2p − 1} instead. Let M1 = {0, 1, . . . , p − 1} and M2 = {p, p + 1, . . .,2p − 1}. We shall denote by |A| and σ (A) the number of elements and the sum of elements of the set A; also, let Cp be the family of all p-element subsets of M. Define the mapping T : Cp → Cp as T (A) = {x + 1 | x ∈ A ∩ M1 } ∪ {A ∩ M2 }, the addition being modulo p. There are exactly two fixed points of T : these are M1 and M2 . Now if A is any subset from C p distinct from M1 , M2 , and k = |A ∩ M1 | with 1 ≤ k ≤ p − 1, then for i = 0, 1, . . . , p − 1, σ (T i (A)) = σ (A) + ik (mod p). Hence subsets A, T (A), . . . , T p−1 (A) are distinct, and exactly one of them has sum of elements divisible by p. Since σ (M1 ), σ (M2 ) are divisible by p and Cp \ {M1 , M2 } dep−1 composes into families of the form {A, T (A),  . .. , T  (A)}, we conclude that the required number is 1p (|C p | − 2) + 2 = 1p 2p p − 2 + 2. Second solution. Let Ck be the family of all k-element subsets of {1, 2, . . . , 2p}. Denote by Mk (k = 1, 2, . . . , p) the family of p-element multisets with k distinct elements from {1, 2, . . . , 2p}, exactly one of which appears more than once, that have sum of elements divisible by p. It is clear that every subset from Ck , k < p, can be complemented to a multiset from Mk ∪ Mk+1 in exactly two ways, since the equation (p − k)a ≡ 0 (mod p) has exactly two solutions in {1, 2, . . . , 2p}. On the other hand, every multiset from Mk can be obtained by completing exactly one subset from Ck . Additionally, a multiset from Mk can be obtained from exactly one subset from Ck−1 if k < p, and from exactly p subsets from Ck−1 if k = p. Therefore |Mk | + |Mk+1 | = 2|Ck | = 2 2p k for k = 1, 2, . . . , p − 2, and 2p  |M p−1 | + p|M p | = 2|Cp−1 | = 2 p−1 . Since M1 = 2p, it is not difficult to show    using recursion that |M p | = 1p 2p − 2 + 2. p i p 2 Third solution. Let ω = cos 2pπ + i sin 2pπ . We have ∏2p i=1 (x − ω ) = (x − 1) = x2p − 2x p + 1; hence comparing the coefficients at x p , we obtain ∑ ω i1 +···+i p = p−1 ∑i=0 ai ω i = 2, where the first sum runs over all p-subsets {i1 , . . . , i p } of the set {1, . . . , 2p}, and ai is the number of such subsets for which i1 + · · · + i p ≡ i (mod p−1 p). Setting q(x) = −2 + ∑i=0 ai xi , we obtain q(ω j ) = 0 for j = 1, 2, . . . , p − 1. p−1 Hence 1 + x + · · · + x | q(x), and since deg q = p − 1, we have q(x) = −2 + p−1 ∑i=0 ai xi = c(1 + x + · · · + x p−1 ) for some constant c. Thus a0 −2 = a1 =· · · =  2p 1 a p−1 , which together with a0 + · · · + a p−1 = 2p p yields a0 = p p − 2 + 2. 602 4 Solutions 21. We shall show that there is no such n. Certainly, n = 2 does not work, so suppose n ≥ 3. Let a, b be distinct elements of A1 , and c any integer greater than −a and −b. We claim that a + c, b + c belong to the same subsets. Suppose to the contrary that a + c ∈ A1 and b + c ∈ A2 , and take arbitrary elements xi ∈ Ai , i = 3, . . . , n. The number b + x3 + · · · + xn is in A2 , so that s = (a + c) + (b + x3 + · · ·+xn )+x4 +· · ·+ xn must be in A3 . On the other hand, a + x3 + · · ·+xn ∈ A2 , so s = (a + x3 + · · ·+ xn ) + (b + c) + x4 + · · ·+ xn is in A1 , a contradiction. Similarly, if a + c ∈ A2 and b + c ∈ A3 , then s = a + (b + c) + x4 + · · · + xn belongs to A2 , but also s = b + (a + c) + x4 + · · · + xn ∈ A3 , which is impossible. For i = 1, . . . , n choose xi ∈ Ai ; set s = x1 + · · · + xn and yi = s − xi . Then yi ∈ Ai . By what has been proved above, 2xi = xi + xi belongs to the same subset as xi + yi = s does. It follows that all numbers 2xi , i = 1, . . . , n, are in the same subset. Since we can arbitrarily take xi from each set Ai , it follows that all even numbers belong to the same set, say A1 . Similarly, 2xi + 1 = (xi + 1) + xi is in the subset to which (xi + 1) + yi = s + 1 belongs for all i = 1, . . . , n; hence all odd numbers greater than 1 are in the same subset, say A2 . By the above considerations, 3 − 2 = 1 ∈ A2 also. But then nothing remains in A3 , . . . , An , a contradiction. √ √ √ √ √ 22. Let √ u = 2p − x − y and v = u(2 2p − u) = 2p − ( 2p − u)2 = 2p − x − y − 4xy for x, y ∈ N, x ≤ y. Obviously u ≥ 0 if and only if v ≥ 0, and u, v attain minimum Note that v 6= 0. Otherwise u = 0 too, √ positive √ √ values simultaneously. so y = ( 2p − x)2 = 2p − x − 2 2px, which implies that 2px is a square, and consequently x is divisible by 2p, which is impossible. √ Now let z be the smallest integer √ greater than 4xy. We have z2 − 1 ≥ 4xy, √ √ z ≤ 2p − x − y, and z ≤ p because 4xy ≤ ( x + y)2 < 2p. It follows that v = 2p − x − y − p p 4xy ≥ z − z2 − 1 = 1 1 √ p ≥ . 2 z+ z −1 p + p2 − 1 Equality holds if and only if z = x + y = p and 4xy = p2 − 1, which is satisfied p+1 only when x = p−1 2 and y = 2 . Hence for these values of x, y, both u and v attain positive minima. 23. By putting F(1) = 0 and F(361) = 1, condition (c) becomes F(F(n163 )) = F(F(n)) for n ≥ 2. For n = 2, 3, . . . , 360 let F(n) = n, and inductively define F(n) for n ≥ 362 as follows:  F(m), if n = m163 , m ∈ N; F(n) = the least number not in {F(k) | k < n} , otherwise. Obviously, (a) each nonnegative integer appears in the sequence because there are infinitely many numbers not of the form m163 , and (b) each positive integer appears infinitely often because F(m163 ) = F(m). Since F(n163 ) = F(n), (c) also holds. Second solution. Another example of such a sequence is as follows: If n = α pα1 1 pα2 2 · · · pk k is the factorization of n into primes, we put F(n) = α1 + α2 + 4.36 Shortlisted Problems 1995 603 · · · + αk and F(1) = 0. Conditions (a) and (b) are evidently satisfied for this F, while (c) follows from F(F(n163 )) = F(163F(n)) = F(F(n)) + 1 (because 163 is a prime) and F(F(361)) = F(F(192 )) = F(2) = 1. 24. The given condition is equivalent to (2xi − xi−1 )(xi xi−1 − 1) = 0, so either xi = 1 1 kn en 2 xi−1 or xi = x . We shall show by induction on n that for any n ≥ 0, xn = 2 x0 i−1 for some integer kn , where |kn | ≤ n and en = (−1)n−kn . Indeed, this is true for n = 0. If it holds for some n, then xn+1 = 12 xn = 2kn −1 xe0n (hence kn+1 = kn − 1 n and en+1 = en ) or xn+1 = x1n = 2−kn x−e (hence kn+1 = −kn and en+1 = −en ). 0 e k 1995 Thus x0 = x1995 = 2 1995 x0 . Note that e1995 = 1 is impossible, since in that case k1995 would be odd, although it should equal 0. Therefore e1995 = −1, which gives x20 = 2k1995 ≤ 21994 , so the maximal value that x0 can have is 2997 . This value is attained in the case xi = 2997−i for i = 0, . . . , 997 and xi = 2i−998 for i = 998, . . ., 1995. Second solution. First we show that there is an n, 0 ≤ n ≤ 1995, such that xn = 1. Suppose the contrary. Then each of xn belongs to one of the intervals I−i−1 = [2−i−1 , 2−i ) or Ii = (2i , 2i+1 ], where i = 0, 1, 2, . . .. Let xn ∈ Iin . Note that by the formula for xn , in and in−1 are of different parity. Hence i0 and i1995 are also of different parity, contradicting x0 = x1995 . It follows that for some n, xn = 1. Now if n ≤ 997, then x0 ≤ 2997 , while if n ≥ 998, we also have x0 = x1995 ≤ 2997 . 25. By the definition of q(x), it divides x for all integers x > 0, so f (x) = xp(x)/q(x) is a positive integer too. Let {p0 , p1 , p2 , . . . } be all prime numbers in increasing order. Since it easily follows by induction that all xn ’s are square-free, we can assign to each of them a unique code according to which primes divide it: if pm is the largest prime dividing xn , the code corresponding to xn will be . . . 0sm sm−1 . . . s0 , with si = 1 if pi | xn and si = 0 otherwise. Let us investigate how f acts on these codes. If the code of xn ends with 0, then xn is odd, so the code of f (xn ) = xn+1 is obtained from that of xn by replacing s0 = 0 by s0 = 1. Furthermore, if the code of xn ends with 011 . . .1, then the code of xn+1 ends with 100 . . .0 instead. Thus if we consider the codes as binary numbers, f acts on them as an addition of 1. Hence the code of xn is the binary representation of n and thus xn uniquely determines n. Specifically, if xn = 1995 = 3·5·7·19, then its code is 10001110 and corresponds to n = 142. 26. For n = 1 the result is trivial, since x1 = 1. Suppose now that n ≥ 2 and let i fn (x) = xn − ∑n−1 i=0 x . Note that xn is the unique positive real root of f n , because fn (x) 1 = x − 1 − 1x − · · · − xn−1 is strictly increasing on R+ . xn−1 Consider gn (x) = (x − 1) fn (x) = (x − 2)xn + 1. Obviously gn (x) has no positive n roots other than 1 and xn > 1. Observe that 1 − 21n > 1 − 2nn ≥ 12 for n ≥ 2 (by Bernoulli’s inequality). Since then      n 1 1 1 n 1 gn 2 − n = − n 2 − n + 1 = 1 − 1 − n+1 > 0, 2 2 2 2 604 4 Solutions and  gn 2 − 1 2n−1   n   1 1 n = − n−1 2 − n−1 + 1 = 1 − 2 1 − n < 0, 2 2 2 1 1 we conclude that xn is between 2 − 2n−1 and 2 − 21n , as required. Remark. Moreover, limn→∞ 2n (2 − xn) = 1. 27. Computing the first few values of f (n), we observe the following pattern: f (4k) = k, k ≥ 3, f (4k + 1) = 1, k ≥ 4, f (4k + 2) = k − 3, k ≥ 7, f (8) = 3; f (5) = f (13) = 2; f (2) = 1, f (6) = f (10) = 2, f (14) = f (18) = 3, f (26) = 4; f (4k + 3) = 2. We shall prove these statements simultaneously by induction on n, having verified them for k ≤ 7. (i) Let n = 4k. Since f (3) = f (7) = · · · = f (4k − 1) = 2, we have f (4k) ≥ k. But f (n) ≤ maxm<n f (m) + 1 ≤ (k − 1) + 1, so f (4k) = k. (ii) Let n = 4k + 1, k ≥ 7. Since f (4k) = k and f (m) < k for m < 4k, we deduce that f (4k + 1) = 1. (iii) Let n = 4k +2, k ≥ 7. Since f (17) = f (21) = · · · = f (4k +1) = 1, we obtain f (4k + 2) ≥ k − 3. On the other hand, if f (4k + 1) = f (4k + 1 −d) = 1, then d ≥ 8, and 4k + 1 − 8(k − 3) < 0. So f (4k + 2) = k − 3. (iv) Let n = 4k + 3, k ≥ 7. We have f (4k + 2) = k − 3 and f (m) = k − 3 for exactly one m < 4k + 2 (namely for m = 4k − 12); hence f (4k + 3) = 2. Therefore, for example, f (4n + 8) = n + 2 for all n; hence we can take a = 4 and b = 8. 28. Let F(x) = f (x) − 95 for x ≥ 1. Writing k for m + 95, the given condition becomes F(k + F(n)) = F(k) + n, k ≥ 96, n ≥ 1. (1) Thus for x, z ≥ 96 and an arbitrary y we have F(x + y) + z = F(x + y + F(z)) = F(x + F(F(y) + z)) = F(x) + F(y) + z, and consequently F(x + y) = F(x) + F(y) whenever x ≥ 96. Moreover, since then F(x + y) + F(96) = F(x + y + 96) = F(x) + F(y + 96) = F(x) + F(y) + F(96) for any x, y, we obtain F(x + y) = F(x) + F(y), x, y ∈ N. (2) It follows by induction that F(n) = nc for all n, where F(1) = c. Equation (1) becomes ck + c2 n = ck + n, and yields c = 1. Hence F(n) = n and f (n) = n + 95 for all n. Finally, ∑19 k=1 f (k) = 96 + 97 + · · ·+ 114 = 1995. Second solution. First we show that f (n) > 95 for all n. If to the contrary f (n) ≤ 95, we have f (m) = n + f (m + 95 − f (n)), so by induction f (m) = kn + f (m + k(95 − f (n))) ≥ kn for all k, which is impossible. Now for m > 95 we have 4.36 Shortlisted Problems 1995 605 f (m + f (n) − 95) = n + f (m), and again by induction f (m + k( f (n) − 95)) = kn + f (m) for all m, n, k. It follows that with n fixed, (∀m) lim k→∞ f (m + k( f (n) − 95)) n = ; m + k( f (n) − 95) f (n) − 95 hence lim s→∞ f (s) n = . s f (n) − 95 n Hence f (n)−95 does not depend on n, i.e., f (n) ≡ cn + 95 for some constant c. It is easily checked that only c = 1 is possible. 606 4 Solutions 4.37 Solutions to the Shortlisted Problems of IMO 1996 1. We have a5 + b5 − a2 b2 (a + b) = (a3 − b3 )(a2 − b2 ) ≥ 0, i.e. a5 + b5 ≥ a2 b2 (a + b). Hence ab a5 + b5 + ab ≤ ab a2 b2 (a + b) + ab = abc2 a2 b2 c2 (a + b) + abc2 = c . a+b+c c a Now, the left side of the inequality to be proved does not exceed a+b+c + a+b+c + b = 1. Equality holds if and only if a = b = c. a+b+c 2. Clearly a1 > 0, and if p 6= a1 , we must have an < 0, |an | > |a1 |, and p = −an . But then for sufficiently large odd k, −akn = |an |k > (n − 1)|a1 |k , so that ak1 + · · · + akn ≤ (n − 1)|a1|k − |an |k < 0, a contradiction. Hence p = a1 . Now let x > a1 . From a1 + · · · + an ≥ 0 we deduce ∑nj=2 (x − a j ) ≤ (n − a1  1) x + n−1 , so by the AM–GM inequality,   a1 n−1 (x − a2) · · · (x − an) ≤ x + ≤ xn−1 + xn−2a1 + · · · + an−1 (1) 1 . n−1  The last inequality holds because n−1 ≤ (n − 1)r for all r ≥ 0. Multiplying (1) r by (x − a1) yields the desired inequality. 3. Since a1 > 2, it can be written as a1 = b + b−1 for some b > 0. Furthermore, a21 − 2 = b2 + b−2 and hence a2 = (b2 + b−2 )(b + b−1). We prove that      n−1 n−1 an = b + b−1 b2 + b−2 b4 + b−4 · · · b2 + b−2  2  n−1 2 n n a an 2 −2n−1 by induction. Indeed, n+1 = − 2 = b + b − 2 = b2 + b−2 . an an−1 Now we have n 1 ∑ ai i=1 b b3 + + ··· b2 + 1 (b2 + 1)(b4 + 1) n b2 −1 ··· + 2 . n (b + 1)(b4 + 1) . . . (b2 + 1) = 1+ (1) √ Note that 12 (a + 2 − a2 − 4 ) = 1 + 1b ; hence we must prove that the right side in (1) is less than 1b . This follows from the fact that k b2 k (b2 + 1)(b4 + 1) · · · (b2 + 1) 1 1 = − ; k−1 k 2 4 2 2 4 (b + 1)(b + 1) · · · (b + 1) (b + 1)(b + 1) · · · (b2 + 1)   1 hence the right side in (1) equals 1b 1 − (b2 +1)(b4 +1)...(b 2n +1) , and this is clearly less than 1/b . 4.37 Shortlisted Problems 1996 607 4. Consider the function f (x) = a1 a2 an + 2 + ···+ n . x x x Since f is strictly decreasing from +∞ to 0 on the interval (0, +∞), there exists exactly one R > 0 for which f (R) = 1. This R is also the only positive real root of the given polynomial. Since ln x is a concave function on (0, +∞), Jensen’s inequality gives us !   n n aj aj A A ln j ≤ ln ∑ · j = ln f (R) = 0. ∑ R j=1 A j=1 A R Therefore ∑nj=1 a j (ln A − j ln R) ≤ 0, which is equivalent to A ln A ≤ B ln R, i.e., AA ≤ RB . 5. Considering the polynomials ±P(±x) we may assume w.l.o.g. that a, b ≥ 0. We have four cases: (1) c ≥ 0, d ≥ 0. Then |a| + |b| + |c| + |d| = a + b + c + d = P(1) ≤ 1. (2) c ≥ 0, d < 0. Then |a| + |b| + |c| + |d| = a + b + c − d = P(1) − 2P(0) ≤ 3. (3) c < 0, d ≥ 0. Then |a| + |b| + |c| + |d| = a + b − c + d 4 1 8 8 = P(1) − P(−1) − P(1/2) + P(−1/2) ≤ 7. 3 3 3 3 (4) c < 0, d < 0. Then |a| + |b| + |c| + |d| = a + b − c − d 5 4 = P(1) − 4P(1/2) + P(−1/2) ≤ 7. 3 3 Remark. It can be shown that the maximum of 7 is attained only for P(x) = ±(4x3 − 3x). 6. Let f (x), g(x) be polynomials with integer coefficients such that f (x)(x + 1)n + g(x)(xn + 1) = k0 . (1) r Write n = 2r m for m odd and note that xn + 1 = (x2 + 1)B(x), where B(x) = r r r x2 (m−1) − x2 (m−2) + · · · − x2 + 1. Moreover, B(−1) = 1; hence B(x) − 1 = (x + 1)c(x) and thus R(x)B(x) + 1 = (B(x) − 1)n = (x + 1)n c(x)n (2) for some polynomials c(x) and R(x). r The zeros of the polynomial x2 + 1 are ω j , with ω1 = cos 2πr + i sin 2πr , and ω j = ω 2 j−1 for 1 ≤ j ≤ 2r . We have 608 4 Solutions (ω1 + 1)(ω2 + 1) · · · (ω2r+1 + 1) = 2. (3) From (1) we also get f (ω j )(ω j + 1)n = k0 for j = 1, 2, . . . , 2r . Since A = f (ω1 ) f (ω2 ) · · · f (ω2r ) is a symmetric polynomial in ω1 , . . . , ω2r with integer coefficients, A is an integer. Consequently, taking the product over j = 1, 2, . . . , 2r r r and using (3) we deduce that 2n A = k02 is divisible by 2n = 22 m . Hence 2m | k0 . Furthermore, since ω j + 1 = (ω1 + 1)p j (ω1 ) for some polynomial p j with inr teger coefficients, (3) gives (ω1 + 1)2 p(ω1 ) = 2, where p(x) = p2 (x) · · · p2r (x) r has integer coefficients. But then the polynomial (x + 1)2 p(x) − 2 has a zero r x = ω1 , so it is divisible by its minimal polynomial x2 + 1. Therefore r r (x + 1)2 p(x) = 2 + (x2 + 1)q(x) (4) for some polynomial q(x). Raising (4) to the mth power we get (x + 1)n p(x)n = r 2m + (x2 + 1)Q(x) for some polynomial Q(x) with integer coefficients. Now using (2) we obtain r r r (x + 1)nc(x)n (x2 + 1)Q(x) = (x2 + 1)Q(x) + (x2 + 1)Q(x)B(x)R(x) = (x + 1)n p(x)n − 2m + (xn + 1)Q(X )R(x). Therefore (x+1)n f (x)+(xn +1)g(x) = 2m for some polynomials f (x), g(x) with integer coefficients, and k0 = 2m . 7. We are given that f (x + a + b) − f (x + a) = f (x + b) − f (x), where a = 1/6 and b = 1/7. Summing up these equations for x, x + b, . . . , x + 6b we obtain f (x + a + 1) − f (x + a) = f (x + 1) − f (x). Summing up the new equations for x, x + a, . . . , x + 5a we obtain that f (x + 2) − f (x + 1) = f (x + 1) − f (x). It follows by induction that f (x + n) − f (x) = n[ f (x + 1) − f (x)]. If f (x + 1) 6= f (x), then f (x + n) − f (x) will exceed in absolute value an arbitrarily large number for a sufficiently large n, contradicting the assumption that f is bounded. Hence f (x + 1) = f (x) for all x. 8. Putting m = n = 0 we obtain f (0) = 0 and consequently f ( f (n)) = f (n) for all n. Thus the given functional equation is equivalent to f (m + f (n)) = f (m) + f (n), f (0) = 0 . Clearly one solution is (∀x) f (x) = 0. Suppose f is not the zero function. We observe that f has nonzero fixed points (for example, any f (n) is a fixed point). Let a be the smallest nonzero fixed point of f . By induction, each ka (k ∈ N) is a fixed point too. We claim that all fixed points of f are of this form. Indeed, suppose that b = ka + i is a fixed point, where i < a. Then b = f (b) = f (ka + i) = f (i + f (ka)) = f (i) + f (ka) = f (i) + ka; hence f (i) = i. Hence i = 0. 4.37 Shortlisted Problems 1996 609 Since the set of values of f is a set of its fixed points, it follows that for i = 0, 1, . . . , a − 1, f (i) = ani for some integers ni ≥ 0 with n0 = 0. Let n = ka + i be any positive integer, 0 ≤ i < a. As before, the functional equation gives us f (n) = f (ka + i) = f (i) + ka = (ni + k)a. Besides the zero function, this is the general solution of the given functional equation. To verify this, we plug in m = ka + i, n = la + j and obtain f (m + f (n)) = f (ka + i + f (la + j)) = f ((k + l + n j )a + i) = (k + l + n j + ni )a = f (m) + f (n). 9. From the definition of a(n) we obtain  1 if n ≡ 0 or n ≡ 3 (mod 4); a(n) − a([n/2]) = −1 if n ≡ 1 or n ≡ 2 (mod 4). Let n = bk bk−1 . . . b1 b0 be the binary representation of n, where we assume bk = 1. If we define p(n) and q(n) to be the number of indices i = 0, 1, . . . , k − 1 with bi = bi+1 and the number of i = 0, 1, . . . , k − 1 with bi 6= bi+1 respectively, we get a(n) = p(n) − q(n). (1) (a) The maximum value of a(n) for n ≤ 1996 is 9 when p(n) = 9 and q(n) = 0, i.e., in the case n = 11111111112 = 1023. The minimum value is −10 and is attained when p(n) = 0 and q(n) = 10, i.e., only for n = 101010101012 = 1365. (b) From (1) we have that a(n) = 0 is equivalent to p(n) = q(n) = k/2. Hence k must be even, and the k/2 indices i for which bi = bi+1 can be chosen in k  exactly k/2 ways. Thus the number of positive integers n < 211 = 2048 with a(n) = 0 is equal to             0 2 4 6 8 10 + + + + + = 351. 0 1 2 3 4 5 But five of these numbers exceed 1996: these are 2002 = 111110100102 , 2004 = 111110101002 , 2006 = 111110101102 , 2010 = 111110110102 , 2026 = 111111010102 . Therefore there are 346 numbers n ≤ 1996 for which a(n) = 0. 10. We first show that H is the common orthocenter of the triangles ABC and AQR. 610 4 Solutions Q Let G, G′ , H ′ be respectively the cenR troid of △ABC, the centroid of △PBC, and the orthocenter of △PBC. Since A the triangles ABC and PBC have a X E common circumcenter, from the prop−−→′ erties of the Euler line we get HH = H −−→ − → 3GG′ = AP. But △AQR is exactly the B image of △PBC under translation by C − → AP; hence the orthocenter of AQR coP incides with H. (Remark: This can be shown by noting that AHBQ is cyclic.) Now we have that RH ⊥ AQ; hence ∠AX H = 90◦ = ∠AEH. It follows that AXEH is cyclic; hence ∠EXQ = 180◦ − ∠AHE = 180◦ − ∠BCA = 180◦ − ∠BPA = ∠PAQ (as oriented angles). Hence EX k AP. 11. Let X ,Y, Z respectively be the feet of the perpendiculars from P to BC, CA, AB. Examining the cyclic quadrilaterals AZPY , BX PZ, CY PX , one can easily see that ∠XZY = ∠APB − ∠C and XY = PC sin ∠C. The first relation gives that XY Z is isosceles with XY = XZ, so from the second relation PB sin ∠B = PC sin ∠C. Hence AB/PB = AC/PC. This implies that the bisectors BD and CD of ∠ABP and ∠ACP divide the segment AP in equal ratios; i.e., they concur with AP. Second solution. Take that X,Y, Z are the points of intersection of AP, BP,CP with the circumscribed circle of ABC instead. We similarly obtain XY = X Z. If we write AP · PX = BP · PY = CP · PZ = k, from the similarity of △APC and △ZPX we get AC AP AP ·CP = = , XZ PZ k k·AC·BP i.e., XZ = AP·BP·CP . It follows again that AC/AB = PC/PB. Third solution. Apply an inversion with center at A and radius r, and denote by Q the image of any point Q. Then the given condition becomes ∠BCP = ∠CBP, i.e., BP = PC. But r2 PB = PB, AP · AB so AC/AB = PC/PB. Remark. Moreover, it follows that the locus of P is an arc of the circle of Apollonius through C. 12. It is easy to see that P lies on the segment AC. Let E be the foot of the altitude BH and Y, Z the midpoints of AC, AB respectively. Draw the perpendicular HR to FP (R ∈ FP). Since Y is the circumcenter of △FCA, we have ∠FYA = 180◦ − 2∠A. Also, OFPY is cyclic; hence ∠OPF = ∠OY F = 2∠A − 90◦. Next, △OZF and △HRF are similar, so OZ/OF = HR/HF. 4.37 Shortlisted Problems 1996 This leads to HR · OF = HF · OZ = 1 1 2 HF · HC = 2 HE · HB = HE · OY . This implies that HR/HE = OY /OF. Moreover, ∠EHR = ∠FOY ; hence the triangles EHR and FOY are similar. Consequently ∠HPC = ∠HRE = ∠OY F = 2∠A − 90◦ . We finally get ∠FHP = ∠HPC + ∠HCP = ∠A. 611 C Y E P R A O H F B Z Second solution. As before, ∠HFY = 90◦ − ∠A, so it suffices to show that HP ⊥ FY . The points O, F, P,Y lie on a circle, say Ω 1 with center at the midpoint Q of OP. Furthermore, the points F,Y lie on the nine-point circle Ω of △ABC with center at the midpoint N of OH. The segment FY is the common chord of Ω1 and Ω , from which we deduce that NQ ⊥ FY . However, NQ k HP, and the result follows. Third solution. Let H ′ be the point symmetric to H with respect to AB. Then H ′ lies on the circumcircle of ABC. Let the line FP meet the circumcircle at U,V and meet H ′ B at P′ . Since OF ⊥ UV , F is the midpoint of UV . By the butterfly theorem, F is also the midpoint of PP′ . Therefore △H ′ FP′ ∼ = FHP; hence ∠FHP = ∠FH ′ B = ∠A. Remark. It is possible to solve the problem using trigonometry. For example, sin(A−B) sin(A−B) FZ FK CF ZO = KP = cosC , where K is on CF with PK ⊥ CF. Then KP = cosC + tan A, from which one obtains formulas for KP and KH. Finally, we can calculate KP tan ∠FHP = KH = · · · = tan A. Second remark. Here is what happens when BC ≤ CA. If ∠A > 45◦ , then ∠FHP = ∠A. If ∠A = 45◦ , the point P escapes to infinity. If ∠A < 45◦ , the point P appears on the extension of AC over C, and ∠FHP = 180◦ − ∠A. 13. By the law of cosines applied to △CA1 B1 , we obtain A1 B21 = A1C2 + B1C2 − A1C · B1C ≥ A1C · B1C. Analogously, B1C12 ≥ B1 A · C1 A and C1 A21 ≥ C1 B · A1 B, so that multiplying these inequalities yields A1 B21 · B1C12 ·C1 A21 ≥ A1 B · A1C · B1 A · B1C ·C1 A · C1 B. (1) Now, the lines AA1 , BB1 ,CC1 concur, so by Ceva’s theorem, A1 B · B1C · C1 A = AB1 · BC1 · CA1 , which together with (1) gives the desired inequality. Equality holds if and only if CA1 = CB1 , etc. 14. Let a, b, c, d, e, and f denote the lengths of the sides AB, BC, CD, DE, EF, and FA respectively. Note that ∠A = ∠D, ∠B = ∠E, and ∠C = ∠F. Draw the lines PQ and RS through A and D perpendicular to BC and EF Q F e E S f d a c A P D B b C R 612 4 Solutions respectively (P, R ∈ BC, Q, S ∈ EF). Then BF ≥ PQ = RS. Therefore 2BF ≥ PQ + RS, or 2BF ≥ (a sin B + f sinC) + (c sinC + d sinB), and similarly, 2BD ≥ (c sin A + b sinB) + (e sin B + f sin A), 2DF ≥ (e sinC + d sinA) + (a sinA + b sinC). (1) Next, we have the following formulas for the considered circumradii: RA = BF , 2 sin A RC = BD , 2 sinC RE = DF . 2 sin E It follows from (1) that     1 sin B sin A 1 sinC sin B RA + RC + RE ≥ a + + b + + ··· 4 sin A sin B 4 sin B sinC 1 P ≥ (a + b + · · ·) = , 2 2 with equality if and only if ∠A = ∠B = ∠C = 120◦ and FB ⊥ BC etc., i.e., if and only if the hexagon is regular. Second solution. Let us construct points A′′ ,C′′ , E ′′ such that ABA′′ F, CDC′′ B, and EFE ′′ D are parallelograms. It follows that A′′ ,C′′ , B are collinear and also C′′ , E ′′ , B and E ′′ , A′′ , F. Furthermore, E′ ′ denote by A be the intersection of the perpendiculars through F and B E to FA′′ and BA′′ , respectively, and let D C′ and E ′ be analogously defined. F ′′ C Since A′ FA′′ B is cyclic with the diameter being A′ A′′ and since △FA′′ B ∼ = A′′ E ′′ C A ′ ′′ △BAF, it follows that 2RA = A A = ′ C′ A x. Similarly, 2RC = C′C′′ = y and B 2RE = E ′ E ′′ = z. We also have AB = AB = FA′′ = ya , A′′ B = za , CD = C′′ B = zc , CB = C′′ D = xc , EF = E ′′ D = xe , and ED = E ′′ F = ye . The original inequality we must prove now becomes x + y + z ≥ ya + za + zc + xc + xe + ye . (1) We now follow and generalize the standard proof of the Erdős–Mordell inequality (for the triangle A′C′ E ′ ), which is what (1) is equivalent to when A′′ = C′′ = E ′′ . We set C′ E ′ = a, A′ E ′ = c and A′C′ = e. Let A1 be the point symmetric to A′′ with respect to the bisector of ∠E ′ A′C′ . Let F1 and B1 be the feet of the perpendiculars from A1 to A′C′ and A′ E ′ , respectively. In that case, A1 F1 = A′′ F = ya and A1 B1 = A′′ B = za . We have ax = A′ A1 · E ′C′ ≥ 2SA′ E ′ A1C′ = 2SA′E ′ A1 + 2SA′C′ A1 = cza + eya . 4.37 Shortlisted Problems 1996 613 Similarly, cy ≥ exc + azc and ez ≥ aye + cxe . Thus c a e c a e za + zc + xc + xe + ye + ya a c  c e e a c a z +z  c az − z  a c a c = + + − + ··· . a c 2 a c 2 x+y+z ≥ (2) ye −ya za −zc ′′ ′′ ′′ ′ ′ ′ e Let us set a1 = xc −x 2 , c1 = 2 , e1 = 2 . We note that △A C E ∼ △A C E c a e c a e and hence a1 /a = c1 /c = e1 /e= k. Thus a − c e1 + c − e a1 + e − a c1 = ae ea ca ac ec k ce a − c + c − e + e − a = 0. Equation (2) reduces to  c az + z  e cx + x  a c e c x+y+z ≥ + + + a c 2 c e 2 a ey + y  a e + + . e a 2 Using c/a + a/c, e/c + c/e, a/e + e/a ≥ 2 we finally get x + y + z ≥ ya + za + zc + xc + xe + ye . Equality holds if and only if a = c = e and A′′ = C′′ = E ′′ = center of △A′C′ E ′ , i.e., if and only if ABCDEF is regular. Remark. From the second proof it is evident that the Erdős–Mordell inequality is a special case of the problem. If Pa , Pb , Pc are the feet of the perpendiculars from a point P inside △ABC to the sides BC,CA, AB, and Pa PPb Pc′ , Pb PPc Pa′ , Pc PPa Pb′ parallelograms, we can apply the problem to the hexagon Pa Pc′ Pb Pa′ Pc Pb′ to prove the Erdős–Mordell inequality for △ABC and point P. 15. Denote by ABCD and EFGH the two rectangles, where AB = a, BC = b, EF = c, and FG = d. Obviously, the first rectangle can be placed within the second one with the angle α between AB and EF if and only if a cos α + b sin α ≤ c, a sin α + b cos α ≤ d. (1) Hence ABCD can be placed within EFGH if and only if there is an α ∈ [0, π /2] for which (1) holds. The lines l1 (ax + by = c) and l2 (bx + ay = d) and the axes x and y bound a region R. By (1), the desired placement of the rectangles is possible if and only if R contains some point (cos α , sin α ) of the unit circle centered at the origin (0, 0). This in turn holds if and only if the intersection point L ofl1 and l2 lies  outside bd−ac bc−ad the unit circle. It is easily computed that L has coordinates b2 −a2 , b2 −a2 . Now L being outside the unit circle is exactly equivalent to the inequality we want to prove. Remark. If equality holds, there is exactly one way of placing. This happens, for example, when (a, b) = (5, 20) and (c, d) = (16, 19). Second remark. This problem is essentially very similar to (SL89-2). 614 4 Solutions 16. Let A1 be the point of intersection of OA′ and BC; similarly define B1 and C1 . From the similarity of triangles OBA1 and OA′ B we obtain OA1 · OA′ = R2 . Now it is enough to show that 8OA1 · OB′ · OC′ ≤ R3 . Thus we must prove that λ µν ≤ 1 , 8 where OA1 OB1 OC1 = λ, = µ, = ν. OA OB OC (1) On the other hand, we have λ µ ν SOBC SAOC SABO + + = + + = 1. 1+λ 1+µ 1+ν SABC SABC SABC Simplifying this relation, we get 1 = λ µ + µν + νλ + 2λ µν ≥ 3(λ µν )2/3 + 2λ µν , which cannot hold if λ µν > 18 . Hence λ µν ≤ 18 , with equality if and only if λ = µ = ν = 12 . This implies that O is the centroid of ABC, and consequently, that the triangle is equilateral. Second solution. In the official solution, the inequality to be proved is transformed into cos(A − B) cos(B − C) cos(C − A) ≥ 8 cosA cos B cosC. B tanC+1 Since cos(B−C) = − cos(B−C) = tan cos A tan B tanC−1 , the last inequality becomes (xy + cos(B+C) 1)(yz + 1)(zx + 1) ≥ 8(xy − 1)(yz − 1)(zx − 1), where we write x, y, z for tan A, tan B, tanC. Using the relation x + y + z = xyz, we can reduce this inequality to (2x + y + z)(x + 2y + z)(x + y + 2z) ≥ 8(x + y)(y + z)(z + x). This p follows from the AM–GM inequality: 2x + y + z = (x + y) + (x + z) ≥ 2 (x + y)(x + z), etc. 17. Let the diagonals AC and BD meet in X. Either ∠AX B or ∠AX D is greater than or equal to 90◦ , so we assume w.l.o.g. that ∠AX B ≥ 90◦ . Let α , β , α ′ , β ′ denote ∠CAB, ∠ABD, ∠BDC, ∠DCA. These angles are all acute and satisfy α + β = α ′ + β ′ . Furthermore, RA = AD , 2 sin β RB = BC , 2 sin α RC = BC , 2 sin α ′ RD = AD . 2 sin β ′ Let ∠B + ∠D = 180◦ . Then A, B,C, D are concyclic and trivially RA + RC = RB + RD . Let ∠B + ∠D > 180◦ . Then D lies within the circumcircle of ABC, which implies that β > β ′ . Similarly α < α ′ , so we obtain RA < RD and RC < RB . Thus RA + RC < RB + RD. Let ∠B + ∠D < 180◦ . As in the previous case, we deduce that RA > RD and RC > RB , so RA + RC > RB + RD . 4.37 Shortlisted Problems 1996 615 18. We first prove the result in the simplest case. Given a 2-gon ABA and a point O, let a, b, c, h denote OA, OB, AB, and the distance of O from AB. Then D = a + b, P = 2c, and H = 2h, so we should show that (a + b)2 ≥ 4h2 + c2 . (1) Indeed, let l be the line through O parallel to AB, and D the point symmetric to B with respect to l. Then (a + b)2 = (OA + OB)2 = (OA + OD)2 ≥ AD2 = c2 + 4h2. Now we pass to the general case. Let A1 A2 . . . An be the polygon F and denote by di , pi , and hi respectively OAi , Ai Ai+1 , and the distance of O from Ai Ai+1 (where An+1 = A1 ). By the case proved above, we have for each i, di + di+1 ≥ q 4h2i + p2i . Summing these inequalities for i = 1, . . . , n and squaring, we obtain  2 q 4D2 ≥ ∑ni=1 4h2i + p2i . It remains only to prove that q ∑ 4h2i + p2i ≥ n i=1 s n ∑ (4h2i + p2i ) = i=1 p 4H 2 + D2 . But this follows immediately from the Minkowski inequality. Equality holds if and only if it holds in (1) and in the Minkowski inequality, i.e., if and only if d1 = · · · = dn and h1 /p1 = · · · = hn /pn . This means that F is inscribed in a circle with center at O and p1 = · · · = pn , so F is a regular polygon and O its center. 19. It is easy to check that after 4 steps we will have all a, b, c, d even. Thus |ab − cd|, |ac − bd|, |ad − bc| remain divisible by 4, and clearly are not prime. The answer is no. Second solution. After one step we have a + b + c + d = 0. Then ac − bd = ac + b(a + b + c) = (a + b)(b + c) etc., so |ab − cd| · |ac − bd| · |ad − bc| = (a + b)2(a + c)2(b + c)2 . However, the product of three primes cannot be a square, hence the answer is no. 20. Let 15a + 16b = x2 and 16a − 15b = y2 , where x, y ∈ N. Then we obtain x4 + y4 = (15a + 16b)2 + (16a − 15b)2 = (152 + 162 )(a2 + b2) = 481(a2 + b2 ). In particular, 481 = 13 · 37 | x4 + y4 . We have the following lemma. Lemma. Suppose that p | x4 + y4 , where x, y ∈ Z and p is an odd prime, where p 6≡ 1 (mod 8). Then p | x and p | y. Proof. Since p | x8 − y8 and by Fermat’s theorem p | x p−1 − y p−1 , we deduce that p | xd − yd , where d = (p − 1, 8). But d 6= 8, so d | 4. Thus p | x4 − y4 , which implies that p | 2y4 , i.e., p | y and p | x. 616 4 Solutions In particular, we can conclude that 13 | x, y and 37 | x, y. Hence x and y are divisible by 481. Thus each of them is at least 481. On the other hand, x = y = 481 is possible. It is sufficient to take a = 31 · 481 and b = 481. Second solution. Note that 15x2 + 16y2 = 481a2. It can be directly verified that the divisibility of 15x2 + 16y2 by 13 and by 37 implies that both x and y are divisible by both primes. Thus 481 | x, y. 21. (a) It clearly suffices to show that for every integer c there exists a quadratic sequence with a0 = 0 and an = c, i.e., that c can be expressed as ±12 ± 22 ± · · · ± n2 . Since (n + 1)2 − (n + 2)2 − (n + 3)2 + (n + 4)2 = 4, we observe that if our claim is true for c, then it is also true for c ± 4. Thus it remains only to prove the claim for c = 0, 1, 2, 3. But one immediately finds 1 = 12 , 2 = −12 − 22 − 32 + 42 , and 3 = −12 + 22 , while the case c = 0 is trivial. (b) We have a0 = 0 and an = 1996. Since an ≤ 12 + 22 + · · · + n2 = 16 n(n + 1)(2n + 1), we get a17 ≤ 1785, so n ≥ 18. On the other hand, a18 is of the same parity as 12 + 22 + · · · + 182 = 2109, so it cannot be equal to 1996. Therefore we must have n ≥ 19. To construct a required sequence with n = 19, we note that 12 + 22 + · · · + 192 = 2470 = 1996 + 2 · 237; hence it is enough to write 237 as a sum of distinct squares. Since 237 = 142 + 52 +42 , we finally obtain 1996 = 12 + 22 + 32 − 42 − 52 + 62 + · · · + 132 − 142 + 152 + · · · + 192. 22. Let a, b ∈ N satisfy the given equation. It is not possible that a = b (since it leads to a2 + 2 = 2a), so we assume w.l.o.g. that a > b. Next, for a > b = 1 the equation becomes a2 = h 2a, i and one obtains h 2 i a solution (a, b) = (2, 1). a2 Let b > 1. If b = α and ba = β , then we trivially have ab ≥ αβ . Since also a2 +b2 ab ≥ 2, we obtain α + β ≥ αβ + 2, or equivalently (α − 1)(β − 1) ≤ −1. But α ≥ 1, and therefore β = 0. It follows that a >hb2 ,ii.e.,ha = b2 + c for isome c > 0. 2 4 2 c+b2 +c2 Now the given equation becomes b3 + 2bc + cb = b +2bb3 +bc + b3 + bc, which reduces to  2  2  c b (c + 1) + c2 (c − 1)b + = . (1) b b3 + bc If c = 1, then (1) always holds, since both sides are 0. We obtain a family of solutions (a, b) = (n, n2 + 1) or (a, b) = (n2 + 1, n). Note that the solution (1, 2) found earlier is obtained for n = 1. 2 2 If c > 1, then (1) implies that b (c+1)+c ≥ (c − 1)b. This simplifies to b3 +bc c2 (b2 − 1) + b2(c(b2 − 2) − (b2 + 1)) ≤ 0. (2) 4.37 Shortlisted Problems 1996 617 Since c ≥ 2 and b2 − 2 ≥ 0, the only possibility is b = 2. But then (2) becomes 3c2 + 8c − 20 ≤ 0, which does not hold for c ≥ 2. Hence the only solutions are (n, n2 + 1) and (n2 + 1, n), n ∈ N. 23. We first observe that the given functional equation is equivalent to   (3m + 1)(3n + 1) − 1 4f + 1 = (4 f (m) + 1)(4 f (n) + 1). 3 This gives us the idea  of introducing a function g : 3N0 + 1 → 4N0 + 1 defined as g(x) = 4 f x−1 + 1. By the above equality, g will be multiplicative, i.e., 3 g(xy) = g(x)g(y) for all x, y ∈ 3N0 + 1. Conversely, any multiplicative bijection g from 3N0 + 1 onto 4N0 + 1 gives us a g(3x+1)−1 function f with the required property: f (x) = . 4 It remains to give an example of such a function g. Let P1 , P2 , Q1 , Q2 be the sets of primes of the forms 3k + 1, 3k + 2, 4k + 1, and 4k + 3, respectively. It is well known that these sets are infinite. Take any bijection h from P1 ∪ P2 onto Q1 ∪ Q2 that maps P1 bijectively onto Q1 and P2 bijectively onto Q2 . Now define g as follows: g(1) = 1, and for n = p1 p2 · · · pm (pi s need not be different) define g(n) = h(p1 )h(p2 ) · · · h(pm ). Note that g is well-defined. Indeed, among the pi s an even number are of the form 3k + 2, and consequently an even number of h(pi )s are of the form 4k + 3. Hence the product of the h(pi )s is of the form 4k + 1. Also, it is obvious that g is multiplicative. Thus, the defined g satisfies all the required properties. 24. We shall work on the array of lattice points defined by A = {(x, y) ∈ Z2 | 0 ≤ x ≤ 19, 0 ≤ y ≤ 11}. Our task is to move from (0, 0) to (19, 0) via the points of A so that each move has the form (x, y) → (x + a, y + b), where a, b ∈ Z and a2 + b2 = r. (a) If r is even, then a + b is even whenever a2 + b2 = r (a, b ∈ Z). Thus the parity of x + y does not change after each move, so we cannot reach (19, 0) from (0, 0). If 3 | r, then both a and b are divisible by 3, so if a point (x, y) can be reached from (0, 0), we must have 3 | x. Since 3 ∤ 19, we cannot get to (19, 0). (b) We have r = 73 = 82 + 32 , so each move is either (x, y) → (x ± 8, y ± 3) or (x, y) → (x ± 3, y ± 8). One possible solution is shown in Fig. 1. (c) We have 97 = 92 + 42 . Let us partition A as B ∪ C , where B = {(x, y) ∈ A | 4 ≤ y ≤ 7}. It is easily seen that moves of the type (x, y) → (x± 9, y± 4) always take us from the set B to C and vice versa, while the moves (x, y) → (x ± 4, y ± 9) always take us from C to C . Furthermore, each move of the type (x, y) → (x ± 9, y ± 4) changes the parity of x, so to get from (0, 0) to (19, 0) we must have an odd number of such moves. On the other hand, with an odd number of such moves, starting from C we can end up only in B, although the point (19, 0) is not in B. Hence, the answer is no. 618 4 Solutions Remark. Part (c) can also be solved by examining all cells that can be reached from (0, 0). All these cells are marked in Fig. 2. ◦· ◦· ············ ·· ◦········· ································ ·· · ··············◦·· · ·· ······◦······················ ·············◦····· ·· · · · · · · · · · ·· · · · · · · · · ◦······· ···· ···· ·· ············ ··· ····◦··········· ···············◦· ········ ······◦· · ·······•· •·· Fig. 1 • • • • • • • • • • • • • • • • • • • • • • • • • • • • • • • • • • • • • • • • • • • • • • • • • • • • • • • • • • • • Fig. 2 25. Let the vertices in the bottom row be assigned an arbitrary coloring, and suppose that some two adjacent vertices receive the same color. The number of such colorings equals 2 n − 2. It is easy to see that then the colors of the remaining vertices get fixed uniquely in order to satisfy the requirement. So in this case there are 2n − 2 possible colorings. Next, suppose that the vertices in the bottom row are colored alternately red and blue. There are two such colorings. In this case, the same must hold for every row, and thus we get 2n possible colorings. It follows that the total number of considered colorings is (2n − 2) + 2n = 2n+1 − 2. n(n+1) 26. Denote the required maximum size by Mk (m, n). If m < 2 , then trivially M = k, so from now on we assume that m ≥ n(n+1) 2 . First we give a lower bound for M. Let r = rk (m, n) be the largest integer such n(n−1) that r + (r + 1) + · · · + (r + n − 1) ≤ m. This is equivalent to nr ≤ m − 2 ≤  m n−1  n(r + 1), so r = n − 2 . Clearly no n elements from {r + 1, r + 2, . . .,k} add up to m, so   m n−1 M ≥ k − rk (m, n) = k − − . (1) n 2 We claim that M is actually equal to k − rk (m, n). To show this, we shall prove by induction on n that if no n elements of a set S ⊆ {1, 2, . . . , k} add up to m, then |S| ≤ k − rk (m, n).   For n = 2 the claim is true, because then for each i = 1, . . . , rk (m, 2) = m−1 2 at least one of i and m − i must be excluded from S. Now let us assume that n > 2 and that the result holds for n − 1. Suppose that S ⊆ {1, 2, . . .,k} does not contain n distinct elements with the sum m, and let x be the smallest element of S. We may assume that x ≤ rk (m, n), because otherwise the statement is clear. Consider the set S′ = {y − x | y ∈ S, y 6= x}. Then S′ is a subset of {1, 2, . . . , k − x} no n − 1 elements of which have the sum m − nx. Also, it is easily checked that n − 1 ≤ m − nx − 1 ≤ k − x, so we may apply the induction hypothesis, which yields that   m−x n |S| ≤ 1 + k − x − rk(m − nx, n − 1) = k − − . (2) n−1 2 4.37 Shortlisted Problems 1996 619 n(n−1)  m−nx− 2 n On the other hand, m−x ≥ 0 because x ≤ rk (m, n); n−1 − 2 −rk (m, n) = n(n−1) hence (2) implies |S| ≤ k − rk (m, n) as claimed. 27. Suppose that such sets of points A , B exist. First, we observe that there exist five points A, B,C, D, E in A such that their convex hull does not contain any other point of A . Indeed, take any point A ∈ A . Since any two points of A are at distance at least 1, the number of points X ∈ A with XA ≤ r is finite for every r > 0. Thus it is enough to choose four points B,C, D, E of A that are closest to A. Now consider the convex hull C of A, B,C, D, E. Suppose that C is a pentagon, say ABCDE. Then each of the disjoint triangles ABC, ACD, ADE contains a point of B. Denote these points by P, Q, R. Then △PQR contains some point F ∈ A , so F is inside ABCDE, a contradiction. Suppose that C is a quadrilateral, say ABCD, with E lying within ABCD. Then the triangles ABE, BCE,CDE, DAE contain some points P, Q, R, S of B that form two disjoint triangles. It follows that there are two points of A inside ABCD, which is a contradiction. Finally, suppose that C is a triangle with two points of A inside. Then C is the union of five disjoint triangles with vertices in A , so there are at least five points of B inside C . These five points make at least three disjoint triangles containing three points of A . This is again a contradiction. It follows that no such sets A , B exist. 28. Note that w.l.o.g., we can assume that p and q are coprime. Indeed, otherwise it suffices to consider the problem in which all xi ’s and p, q are divided by gcd(p, q). Let k, l be the number of indices i with xi+1 − xi = p and the number of those i with xi+1 − xi = −q (0 ≤ i < n). From x0 = xn = 0 we get kp = lq, so for some integer t > 1, k = qt, l = pt, and n = (p + q)t. Consider the sequence yi = xi+p+q − xi , i = 0, . . . , n− p − q. We claim that at least one of the yi ’s equals zero. We begin by noting that each yi is of the form up− vq, where u +v = p +q; therefore yi = (u + v)p − v(p +q) = (p −v)(p +q) is always divisible by p + q. Moreover, yi+1 − yi = (xi+p+q+1 − xi+p+q) − (xi+1 − xi ) is 0 or ±(p + q). We conclude that if no yi is 0 then all yi ’s are of the same sign. But this is in contradiction with the relation y0 + y p+q + · · · + yn−p−q = xn − x0 = 0. Consequently some yi is zero, as claimed. Second solution. As before we assume (p, q) = 1. Let us define a sequence of points Ai (yi , zi ) (i = 0, 1, . . . , n) in N20 inductively as follows. Set A0 = (0, 0) and define (yi+1 , zi+1 ) as (yi , zi + 1) if xi+1 = xi + p and (yi + 1, zi ) otherwise. The points Ai form a trajectory L in N20 continuously moving upwards and rightwards by steps of length 1. Clearly, xi = pzi − qyi for all i. Since xn = 0, it follows that (zn , yn ) = (kq, kp), k ∈ N. Since yn + zn = n > p + q, it follows that k > 1. We observe that xi = x j if and only if Ai A j k A0 An . We shall show that such i, j with i < j and (i, j) 6= (0, n) must exist. 620 4 Solutions If L meets A0 An in an interior point, then our statement trivially holds. From now on we assume the opposite. Let Pi j be the rectangle with sides parallel to the coordinate axes and with vertices at (ip, jq) and ((i + 1)p, ( j + 1)q). Let Li j be the part of the trajectory L lying inside Pi j . We may assume w.l.o.g. that the endpoints of L00 lie on the vertical sides of P00 . Then there obviously exists d ∈ {1, . . . , k − 1} such that the endpoints of Ldd lie on the horizontal sides of Pdd . Consider the translate L′dd of Ldd for the vector −d(p, q). The endpoints of L′dd lie on the vertical sides of P00 . Hence L00 and L′dd have some point X 6= A0 in common. The translate Y of point X for the vector d(p, q) belongs to L and satisfies XY k A0 An . 29. Let the squares be indexed serially by the integers: . . . ,−1, 0, 1, 2, . . . . When a bean is moved from i to i+1 or from i +1 to i for the first time, we may assign the index i to it. Thereafter, whenever some bean is moved in the opposite direction, we shall assume that it is exactly the one marked by i, and so on. Thus, each pair of neighboring squares has a bean stuck between it, and since the number of beans is finite, there are only finitely pairs of neighboring squares, and thus finitely many squares on which moves are made. Thus we may assume w.l.o.g. that all moves occur between 0 and l ∈ N and that all beans exist at all times within [0, l]. Defining bi to be the number of beans in the ith cell (i ∈ Z) and b the total number of beans, we define the semi-invariant S = ∑i∈Z i2 bi . Since all moves occur above 0, the semi-invariant S increases by 2 with each move, and since we always have S < b · l 2 , it follows that the number of moves must be finite. We now prove the uniqueness of the final configuration and the number of moves for some initial configuration {bi }. Let xi ≥ 0 be the number of moves made in the ith cell (i ∈ Z) during the game. Since the game is finite, only finitely many of xi ’s are nonzero. Also, the number of beans in cell i, denoted as ei , at the end is (∀i ∈ Z) ei = bi + xi−1 + xi+1 − 2xi ∈ {0, 1} . (1) Thus it is enough to show that given bi ≥ 0, the sequence {xi }i∈Z of nonnegative integers satisfying (1) is unique. Suppose the assertion is false, i.e., that there exists at least one sequence bi ≥ 0 for which there exist distinct sequences {xi } and {x′i } satisfying (1). We may choose such a {bi } for which min{∑i∈Z xi , ∑i∈Z x′i } is minimal (since ∑i∈Z xi is always finite). We choose any index j such that b j > 1. Such an index j exists, since otherwise the game is over. Then one must make at least one move in the jth cell, which implies that x j , x′j ≥ 1. However, then the sequences {xi } and {x′i } with x j and x′j decreased by 1 also satisfy (1) for a sequence {bi } where b j−1 , b j , b j+1 is replaced with b j−1 + 1, b j − 2, b j+1 + 1. This contradicts the assumption of minimal min{∑i∈Z xi , ∑i∈Z x′i } for the initial {bi }. 30. For convenience, we shall write f 2 , f g, . . . for the functions f ◦ f , f ◦ g, . . . . We need two lemmas. Lemma 1. If f (x) ∈ S and g(x) ∈ T , then x ∈ S ∩ T . 4.37 Shortlisted Problems 1996 621 Proof. The given condition means that f 3 (x) = g2 f (x) and g f g(x) = f g2 (x). Since x ∈ S ∪ T = U, we have two cases: x ∈ S. Then f 2 (x) = g2 (x), which also implies f 3 (x) = f g2 (x). Therefore g f g(x) = f g2 (x) = f 3 (x) = g2 f (x), and since g is a bijection, we obtain f g(x) = g f (x), i.e., x ∈ T . x ∈ T . Then f g(x) = g f (x), so g2 f (x) = g f g(x). It follows that f 3 (x) = g2 f (x) = g f g(x) = f g2 (x), and since f is a bijection, we obtain x ∈ S. Hence x ∈ S ∩T in both cases. Similarly, f (x) ∈ T and g(x) ∈ S again imply x ∈ S ∩T. Lemma 2. f (S ∩ T ) = g(S ∩ T ) = S ∩ T . Proof. By symmetry, it is enough to prove f (S ∩ T ) = S ∩ T , or in other words that f −1 (S ∩T ) = S ∩T . Since S ∩T is finite, this is equivalent to f (S ∩T ) ⊆ S∩T. Let f (x) ∈ S ∩ T . Then if g(x) ∈ S (since f (x) ∈ T ), Lemma 1 gives x ∈ S ∩ T ; similarly, if g(x) ∈ T , then by Lemma 1, x ∈ S ∩ T . Now we return to the problem. Assume that f (x) ∈ S. If g(x) 6∈ S, then g(x) ∈ T , so from Lemma 1 we deduce that x ∈ S ∩ T . Then Lemma 2 claims that g(x) ∈ S ∩ T too, a contradiction. Analogously, from g(x) ∈ S we are led to f (x) ∈ S. This finishes the proof. 622 4 Solutions 4.38 Solutions to the Shortlisted Problems of IMO 1997 1. Let ABC be the given triangle, with ∠B = 90◦ and AB = m, BC = n. For an arbitrary polygon P we denote by w(P) and b(P) respectively the total areas of the white and black parts of P. (a) Let D be the fourth vertex of the rectangle ABCD. When m and n are of the same parity, the coloring of the rectangle ABCD is centrally symmetric with respect to the midpoint of AC. It follows that w(ABC) = 12 w(ABCD) and b(ABC) = 12 b(ABCD); thus f (m, n) = 12 |w(ABCD)−b(ABCD)|. Hence f (m, n) equals 12 if m and n are both odd, and 0 otherwise. (b) The result when m, n are of the same parity follows from (a). Suppose that m > n, where m and n are of different parity. Choose a point E on AB such that AE = 1. Since by (a) |w(EBC) − b(EBC)| = f (m − 1, n) ≤ 12 , n we have f (m, n) ≤ 12 + |w(EAC) − b(EAC)| ≤ 12 + S(EAC) = 12 + n−1 2 = 2. 1 Therefore f (m, n) ≤ 2 min(m, n). (c) Let us calculate f (m, n) for m = 2k + 1, n = 2k, k ∈ N. With E defined as in (b), we have BE = BC = 2k. If the square at B is w.l.o.g. white, CE passes only through black squares. The white part of △EAC then consists of 2k i i2 similar triangles with areas 12 2ki 2k+1 = 4k(2k+1) , where i = 1, 2, . . . , 2k. The total white area of EAC is 1 4k + 1 (12 + 22 + · · · + (2k)2 ) = . 4k(2k + 1) 12 Therefore the black area is (8k − 1)/12, and f (2k + 1, 2k) = (2k − 1)/6, which is not bounded. 2. For any sequence X = (x1 , x2 , . . . , xn ) let us define X = (1, 2, . . . , x1 , 1, 2, . . . , x2 , . . . , 1, 2, . . . , xn ). Also, for any two sequences A, B we denote their concatenation by AB. It clearly holds that AB = A B. The sequences R1 , R2 , . . . are given by R1 = (1) and Rn = Rn−1 (n) for n > 1. We consider the family of sequences Qni for n, i ∈ N, i ≤ n, defined as follows: Qn1 = (1), and Qnn = (n), Qni = Qn−1,i−1 Qn−1,i if 1 < i < n. These sequences form a Pascal-like triangle, as shown in the picture below: Q1i : Q2i : Q3i : Q4i : Q5i : 1 1 1 1 1 2 12 3 112 123 4 1112 112123 1234 5 We claim that Rn is in fact exactly Qn1 Qn2 . . . Qnn . Before proving this, we observe that Qni = Qn−1,i . This follows by induction, since Qni = Qn−1,i−1 Qn−1,i = 4.38 Shortlisted Problems 1997 623 Qn−2,i−1 Qn−2,i = Qn−1,i for n ≥ 3, i ≥ 2 (the cases i = 1 and n = 1, 2 are trivial). Now R1 = Q11 and Rn = Rn−1 (n) = Qn−1,1 . . . Qn−1,n−1(n) = Qn,1 . . . Qn,n−1 Qn,n for n ≥ 2, which justifies our claim by induction. Now we know enough about the sequence Rn to return to the question of the problem. We use induction on n once again. The result is obvious for n = 1 and n = 2. Given any n ≥ 3, consider the kth elements of Rn from the left, say u, and from the right, say v. Assume that u is a member of Qn j , and consequently that v is a member of Qn,n+1− j . Then u and v come from symmetric positions of Rn−1 (either from Qn−1, j , Qn−1,n− j , or from Qn−1, j−1 , Qn−1,n+1− j ), and by the inductive hypothesis exactly one of them is 1. 3. (a) For n = 4, consider a convex quadrilateral ABCD in which AB = BC = − → − → −→ −→ AC = BD and AD = DC, and take the vectors AB, BC, CD, DA. For n = 5, − → − → −→ −→ −→ take the vectors AB, BC, CD, DE, EA for any regular pentagon ABCDE. (b) Let us draw the vectors of V as originated from the same point O. Consider any maximal subset B ⊂ V , and denote by u the sum of all vectors from B. If l is the line through O perpendicular to u, then B contains exactly those vectors from V that lie on the same side of l as u does, and no others. Indeed, if any v 6∈ B lies on the same side of l, then |u + v| ≥ |u|; similarly, if some v ∈ B lies on the other side of l, then |u − v| ≥ |u|. Therefore every maximal subset is determined by some line l as the set of vectors lying on the same side of l. It is obvious that in this way we get at most 2n sets. 4. (a) Suppose that an n × n coveralls matrix A exists for some n > 1. Let x ∈ {1, 2, . . .,2n − 1} be a fixed number that does not appear on the fixed diagonal of A. Such an element must exist, since the diagonal can contain at most n different numbers. Let us call the union of the ith row and the ith column the ith cross. There are n crosses, and each of them contains exactly one x. On the other hand, each entry x of A is contained in exactly two crosses. Hence n must be even. However, 1997 is an odd number; hence no coveralls matrix existsfor n = 1997. 12 (b) For n = 2, A2 = is a coveralls matrix. For n = 4, one such matrix is, 31   for example, 1256 3 1 7 5  A4 =   4 6 1 2. 7431 This construction can be generalized. Suppose that we are given an n × n coveralls matrix An . Let Bn be the matrix obtained from An by adding 2n to each entry, and Cn the matrix obtained from Bn by replacing each diagonal entry (equal to 2n + 1 by induction) with 2n. Then the matrix 624 4 Solutions  An Bn A2n = Cn An  is coveralls. To show this, suppose that i ≤ n (the case i > n is similar). The ith cross is composed of the ith cross of An , the ith row of Bn , and the ith column of Cn . The ith cross of Ai covers 1, 2, . . . , 2n − 1. The ith row of Bn covers all numbers of the form 2n + j, where j is covered by the ith row of An (including j = 1). Similarly, the ith column of Cn covers 2n and all numbers of the form 2n + k, where k > 1 is covered by the ith column of An . Thus we see that all numbers are accounted for in the ith cross of A2n , and hence A2n is a desired coveralls matrix. It follows that we can find a coveralls matrix whenever n is a power of 2. Second solution for part b. We construct a coveralls matrix explicitly for n = 2k . We consider the coordinates/cells of the matrix elements modulo n throughout the solution. We define the i-diagonal (0 ≤ i < n) to be the set of cells of the form ( j, j + i), for all j. We note that each cross contains exactly one cell from the 0-diagonal (the main diagonal) and two cells from each i-diagonal. For two cells within an i diagonal, x and y, we define x and y to be related if there exists a cross containing both x and y. Evidently, for every cell x not on the 0-diagonal there are exactly two other cells related to it. The relation thus breaks up each i-diagonal (i > 0) into cycles of length larger than 1. Due to the diagonal translational symmetry (modulo n), all the cycles within a given i-diagonal must be of equal length and thus of an even length, since n = 2k . The construction of a coveralls matrix is now obvious. We select a number, say 1, to place on all the cells of the 0-diagonal. We pair up the remaining numbers and assign each pair to an i-diagonal, say (2i, 2i + 1). Going along each cycle within the i-diagonal we alternately assign values of 2i and 2i + 1. Since the cycle has an even length, a cell will be related only to a cell of a different number, and hence each cross will contain both 2i and 2i + 1. 5. We shall prove first the 2-dimensional analogue: Lemma. Given an equilateral triangle ABC and two points M, N on the sides AB and AC respectively, there exists a triangle with sides CM, BN, MN. Proof. Consider a regular tetrahedron ABCD. Since CM = DM and BN = DN, one such triangle is DMN. Now, to solve the problem for a regular tetrahedron ABCD, we consider a 4-dimensional polytope ABCDE whose faces ABCD, ABCE, ABDE, ACDE, BCDE are regular tetrahedra. We don’t know what it looks like, but it yields a desired triangle: for M ∈ ABC and N ∈ ADC, we have DM = EM and BN = EN; hence the desired triangle is EMN. Remark. A solution that avoids embedding in R4 is possible, but no longer so short. 6. (a) One solution is 2 x = 2n 3n+1 , y = 2n 2 −n 3n , z = 2n 2 −2n+2 3n−1 . 4.38 Shortlisted Problems 1997 625 (b) Suppose w.l.o.g. that gcd(c, a) = 1. We look for a solution of the form x = pm , y = pn , z = qpr , p, q, m, n, r ∈ N. Then xa + yb = pma + pnb and zc = qc prc , and we see that it is enough to assume ma − 1 = nb = rc (there are infinitely many such triples (m, n, r)) and qc = p + 1. 7. Let us set AC = a, CE = b, EA = c. Applying Ptolemy’s inequality for the quadrilateral ACEF we get AC · EF + CE · AF ≥ AE ·CF. FA c BC a Since EF = AF, this implies FC ≥ a+b . Similarly BE ≥ b+c and Now, BC DE FA a b c + + ≥ + + . BE DA FC b + c c + a a + b Hence it is enough to prove that a b c 3 + + ≥ . b+c c+a a+b 2 DE DA ≥ b c+a . (1) If we now substitute x = b + c, y = c + a, z = a + b and S = a + b + c the inequality (1) becomes equivalent to S(1/x + 1/y + 1/y) − 3 ≥ 3/2 which follows immediately form 1/x + 1/y + 1/z ≥ 9/(x + y + z) = 9/(2S). Equality occurs if it holds in Ptolemy’s inequalities and also a = b = c. The former happens if and only if the hexagon is cyclic. Hence the only case of equality is when ABCDEF is regular. 8. (a) Denote by b and c the perpendicular bisectors of AB and AC respectively. If w.l.o.g. b and AD do not intersect (are parallel), then ∠BCD = ∠BAD = 90◦ , a contradiction. Hence V,W are well-defined. Now, ∠DW B = 2∠DAB and ∠DVC = 2∠DAC as oriented angles, and therefore ∠(W B,VC) = 2(∠DVC − ∠DW B) = 2∠BAC = 2∠BCD is not equal to 0. Consequently CV and BW meet at some T with ∠BTC = 2∠BAC. (b) Let B′ be the second point of intersection of BW with Γ . Clearly AD = BB′ . But we also have ∠BTC = 2∠BAC = 2∠BB′C, which implies that CT = T B′ . It follows that AD = BB′ = |BT ± T B′ | = |BT ±CT |. Remark. This problem is also solved easily using trigonometry. 9. For i = 1, 2, 3 (all indices in this problem will be modulo 3) we denote by Oi the center of Ci and by Mi the midpoint of the arc Ai+1 Ai+2 that does not contain Ai . First we have that Oi+1 Oi+2 is the perpendicular bisector of IBi , and thus it contains the circumcenter Ri of Ai Bi I. Additionally, it is easy to show R1 A3 B2 R3 A1 I B1 B3 A2 626 4 Solutions that Ti+1 Ai = Ti+1 I and Ti+2 Ai = Ti+2 I which implies that Ri lies on the line Ti+1 Ti+2 . Therefore Ri = Oi+1 Oi+2 ∩ Ti+1 Ti+2 . Now, the lines T1 O1 , T2 O2 , T3 O3 are concurrent at I. By Desargues’s theorem, the points of intersection of Oi+1 Oi+2 and Ti+1 Ti+2 , i.e., the Ri ’s, lie on a line for i = 1, 2, 3. Second solution. The centers of three circles passing through the same point I and not touching each other are collinear if and only if they have another common point. Hence it is enough to show that the circles Ai Bi I have a common point other than I. Now apply inversion at center I and with an arbitrary power. We shall denote by X ′ the image of X under this inversion. In our case, the image of the circle Ci is the line B′i+1 B′i+2 while the image of the line Ai+1 Ai+2 is the circle IA′i+1 A′i+2 that is tangent to B′i B′i+2 , and B′i B′i+2 . These three circles have equal radii, so their centers P1 , P2 , P3 form a triangle also homothetic to △B′1 B′2 B′3 . Consequently, points A′1 , A′2 , A′3 , that are the reflections of I across the sides of P1 P2 P3 , are vertices of a triangle also homothetic to B′1 B′2 B′3 . It follows that A′1 B′1 , A′2 B′2 , A′3 B′3 are concurrent at some point J ′ , i.e., that the circles Ai Bi I all pass through J. 10. Suppose that k ≥ 4. Consider any polynomial F(x) with integer coefficients such that 0 ≤ F(x) ≤ k for x = 0, 1, . . . , k + 1. Since F(k + 1) − F(0) is divisible by k + 1, we must have F(k + 1) = F(0). Hence F(x) − F(0) = x(x − k − 1)Q(x) for some polynomial Q(x) with integer coefficients. In particular, F(x) − F(0) is divisible by x(k + 1 − x) > k + 1 for every x = 2, 3, . . . , k − 1, so F(x) = F(0) must hold for any x = 2, 3, . . . , k − 1. It follows that F(x) − F(0) = x(x − 2)(x − 3) · · ·(x − k + 1)(x − k − 1)R(x) for some polynomial R(x) with integer coefficients. Thus k ≥ |F(1) − F(0)| = k(k − 2)!|R(1)|, although k(k − 2)! > k for k ≥ 4. In this case we have F(1) = F(0) and similarly F(k) = F(0). Hence, the statement is true for k ≥ 4. It is easy to find counterexamples for k ≤ 3. These are, for example,  for k = 1,  x(2 − x) for k = 2, F(x) = x(3 − x)  x(2 − x)2 (4 − x) for k = 3. 11. All real roots of P(x) (if any) are negative: say −a1 , −a2 , . . . , −ak . Then P(x) can be factored as P(x) = C(x + a1 ) · · · (x + ak )(x2 − b1x + c1 ) · · · (x2 − bm x + cm ), (1) where x2 − bi x + ci are quadratic polynomials without real roots. Since the product of polynomials with positive coefficients is again a polynomial with positive coefficients, it will be sufficient to prove the result for each of the factors in (1). The case of x + a j is trivial. It remains only to prove the claim for every polynomial x2 − bx + c with b2 < 4c. 4.38 Shortlisted Problems 1997 627 From the binomial formula, we have for any n ∈ N, n 2 (1 + x) (x − bx + c) = where n+2  ∑ i=0      n+2 n n n −b +c xi = ∑ Ci xi , i−2 i−1 i i=0  n! (b + c + 1)i2 − ((b + 2c)n + (2b + 3c + 1))i + c(n2 + 3n + 2) xi Ci = . i!(n − i + 2)! The coefficients Ci of xi appear in the form of a quadratic polynomial in i depending on n. We claim that for large enough n this polynomial has negative discriminant, and is thus positive for every i. Indeed, this discriminant equals D = ((b + 2c)n + (2b + 3c + 1))2 − 4(b + c + 1)c(n2 + 3n + 2) = (b2 − 4c)n2 − 2Un + V, where U = 2b2 + bc + b − 4c and V = (2b + c + 1)2 − 4c, and since b2 − 4c < 0, for large n it clearly holds that D < 0. 12. Lemma. For any polynomial P of degree at most n, the following equality holds:   n+1 i n+1 (−1) P(i) = 0. ∑ i i=0 Proof. See (SL81-13). Suppose to the contrary that the degree of f is at most p − 2. Then it follows from the lemma that   p−1 p−1 i p−1 0 = ∑ (−1) f (i) ≡ ∑ f (i) (mod p), i i=0 i=0  (p−1)(p−2)···(p−i) since p−1 = ≡ (−1)i (mod p). But this is clearly impossible i i! if f (i) equals 0 or 1 modulo p and f (0) = 0, f (1) = 1. p−1 Remark. In proving the essential relation ∑i=0 f (i) ≡ 0 (mod p), it is clearly enough to show that Sk = 1k + 2k + · · · + (p − 1)k is divisible by p for every k ≤ p − 2. This can be shown in two other ways. (1) By induction. Assume that S0 ≡ · · · ≡ Sk−1 (mod p). By the binomial formula we have  p−1 k−1  k+1 k+1 k+1 0 ≡ ∑ [(n + 1) − n ] ≡ (k + 1)Sk + ∑ Si (mod p), i n=0 i=0 and the inductive step follows. (2) Using the primitive root g modulo p. Then Sk ≡ 1 + gk + · · · + gk(p−2) = gk(p−1) − 1 ≡ 0 (mod p). gk − 1 628 4 Solutions 13. Denote A(r) and B(r) by A(n, r) and B(n, r) respectively. The numbers A(n, r) can be found directly: one can choose r girls and r boys in n2 ways, and pair them in r! ways. Hence r  2 n n!2 A(n, r) = · r! = . r (n − r)!2r! Now we establish a recurrence relation between the B(n, r)’s. Let n ≥ 2 and 2 ≤ r ≤ n. There are two cases for a desired selection of r pairs of girls and boys: (i) One of the girls dancing is gn . Then the other r − 1 girls can choose their partners in B(n − 1, r − 1) ways and gn can choose any of the remaining 2n − r boys. Thus, the total number of choices in this case is (2n − r)B(n − 1, r − 1). (ii) gn is not dancing. Then there are exactly B(n − 1, r) possible choices. Therefore, for every n ≥ 2 it holds that B(n, r) = (2n − r)B(n − 1, r − 1) + B(n − 1, r) for r = 2, . . . ,n. Here we assume that B(n, r) = 0 for r > n, while B(n, 1) = 1 + 3 + · · · + (2n − 1) = n2 . It is directly verified that the numbers A(n, r) satisfy the same initial conditions and recurrence relations, from which it follows that A(n, r) = B(n, r) for all n and r ≤ n. 14. We use the following nonstandard notation: (1◦ ) for x, y ∈ N, x ∼ y means that x and y have the same prime divisors; (2◦ ) for a prime p and integers r ≥ 0 and x > 0, pr k x means that x is divisible by pr , but not by pr+1 . First, bm − 1 ∼ bn − 1 is obviously equivalent to bm − 1 ∼ gcd(bm − 1, bn − 1) = bd − 1, where d = gcd(m, n). Setting bd = a and m = kd, we reduce the condition of the problem to ak −1 ∼ a −1. We are going to show that this implies that a +1 is a power of 2. This will imply that d is odd (for even d, a + 1 = bd + 1 cannot be divisible by 4), and consequently b + 1, as a divisor of a + 1, is also a power of 2. But before that, we need the following important lemma (Theorem 2.129). Lemma. Let a, k be positive integers and p an odd prime. If α ≥ 1 and β ≥ 0 are such that pα k a − 1 and pβ k k, then pα +β k ak − 1. k −1 Proof. We use induction on β . If β = 0, then aa−1 = ak−1 + · · · + a + 1 ≡ k (mod p) (because a ≡ 1), and it is not divisible by p. Suppose that the lemma is true for some β ≥ 0, and let k = pβ +1t where β p ∤ t. By the induction hypothesis, ak/p = a p t = mpα +β + 1 for some m not divisible by p. Furthermore, ak − 1 = (mpα +β + 1) p − 1 = (mpα +β ) p + · · · +   p (mpα +β )2 + mpα +β +1. 2  Since p | 2p = p(p−1) , all summands except for the last one are divisible 2 by pα +β +2. Hence pα +β +1 k ak − 1, completing the induction. 4.38 Shortlisted Problems 1997 629 Now let ak − 1 ∼ a − 1 for some a, k > 1. Suppose that p is an odd prime divisor β of k, with pβ k k. Then putting X = a p −1 + · · · + a + 1 we also have (a − 1)X = β a p − 1 ∼ a − 1; hence each prime divisor q of X must also divide a − 1. But then ai ≡ 1 (mod q) for each i ∈ N0 , which gives us X ≡ pβ (mod q). Therefore q | pβ , i.e., q = p; hence X is a power of p. On the other hand, since p | a − 1, we put pα k a − 1. From the lemma we obtain β pα +β k a p − 1, and deduce that pβ k X. But X has no prime divisors other than p, so we must have X = pβ . This is clearly impossible, because X > pβ for a > 1. Thus our assumption that k has an odd prime divisor leads to a contradiction: in other words, k must be a power of 2. Now ak −1 ∼ a −1 implies a − 1 ∼ a2 − 1 = (a −1)(a + 1), and thus every prime divisor q of a + 1 must also divide a − 1. Consequently q = 2, so it follows that a + 1 is a power of 2. As we explained above, this gives that b + 1 is also a power of 2. Remark. In fact, one can continue and show that k must be equal to 2. It is not possible for a4 − 1 ∼ a2 − 1 to hold. Similarly, we must have d = 1. Therefore all possible triples (b, m, n) with m > n are (2s − 1, 2, 1). 15. Let a + bt, t = 0, 1, 2, . . . , be a given arithmetic progression that contains a square and a cube (a, b > 0). We use induction on the progression step b to prove that the progression contains a sixth power. (i) b = 1: this case is trivial. (ii) b = pm for some prime p and m > 0. The case pm | a trivially reduces to the previous case, so let us have pm ∤ a. Suppose that gcd(a, p) = 1. If x, y are integers such that x2 ≡ y3 ≡ a (here all the congruences will be mod pm ), then x6 ≡ a3 and y6 ≡ a2 . Consider an integer y1 such that yy1 ≡ 1. It satisfies a2 (xy1 )6 ≡ x6 y6 y61 ≡ x6 ≡ a3 , and consequently (xy1 )6 ≡ a. Hence a sixth power exists in the progression. If gcd(a, p) > 1, we can write a = pk c, where k < m and p ∤ c. Since the arithmetic progression xt = a + bt = pk (c + pm−k t) contains a square, k must be even; similarly, it contains a cube, so 3 | k. It follows that 6 | k. The progression c + pm−k t thus also contains a square and a cube; hence by the previous case it contains a sixth power and thus xt does also. (iii) b is not a power of a prime, and thus can be expressed as b = b1 b2 , where b1 , b2 > 1 and gcd(b1 , b2 ) = 1. It is given that progressions a + b1t and a + b2 t both contain a square and a cube, and therefore by the inductive hypothesis they both contain sixth powers: say z61 and z62 , respectively. By the Chinese remainder theorem, there exists z ∈ N such that z ≡ z1 (mod b1 ) and z ≡ z2 (mod b2 ). But then z6 belongs to both of the progressions a + b1t and a + b2t. Hence z6 is a member of the progression a + bt. 16. Let da (X ), db (X), dc (X ) denote the distances of a point X interior to △ABC from the lines BC,CA, AB respectively. We claim that X ∈ PQ if and only if da (X ) + db (X) = dc (X). Indeed, if X ∈ PQ and PX = kPQ then da (X ) = kda (Q), db (X ) = (1 − k)db (P), and dc (X ) = (1 − k)dc (P) + kdc (Q), and simple substitution yields 630 4 Solutions da (X) + db (X ) = dc (X ). The converse follows easily. In particular, O ∈ PQ if and only if da (O) + db (O) = dc (O), i.e., cos α + cos β = cos γ . We shall now show that I ∈ DE if and only if AE + BD = DE. Let K be the point on the segment DE such that AE = EK. Then ∠EKA = 12 ∠DEC = 12 ∠CBA = ∠IBA; hence the points A, B, I, K are concyclic. The point I lies on DE if and only if ∠BKD = ∠BAI = 12 ∠BAC = 12 ∠CDE = ∠DBK, which is equivalent to KD = BD, i.e., to AE + BD = DE. But since AE = AB cos α , BD = AB cos β , and DE = AB cos γ , we have that I ∈ DE ⇔ cos α + cos β = cos γ . The conditions for O ∈ PQ and I ∈ DE are thus equivalent. Second solution. We know that three points X ,Y, Z are collinear if and only if for −→ − → − → −→ − → some λ , µ ∈ R with sum 1, we have λ CX + µ CY = CZ. Specially, if CX = pCA − → − → − → − → − → and CY = qCB for some p, q, and CZ = kCA + lCB, then Z lies on XY if and only if kq + l p = pq. Using known relations in a triangle we directly obtain sin β − → CB, sin β + sin γ − → − → −→ sin 2α · CA + sin 2β · CB CO = ; sin 2α + sin 2β + sin 2γ −→ tan β − → CE = CA, tan β + tan γ − → CP = sin α − → CA, sin α + sin γ −→ tan β − → CD = CB, tan β + tan γ − → − → − → sin α · CA + sin β · CB CI = . sin α + sin β + sin γ −→ CQ = Now by the above considerations we get that the conditions (1) P, Q, O are collinear and (2) D, E, I are collinear are both equivalent to cos α + cos β = cos γ . 17. We note first that x and y must be powers of the same positive integer. Indeed, α β β if x = pα1 1 · · · pk k and y = p1 1 · · · pk k (some of αi and βi may be 0, but not both 2 for the same index i), then xy = yx implies αβi = yx2 = qp for some p, q > 0 with α /p α /p i gcd(p, q) = 1, so for a = p1 1 · · · pk k we can take x = a p and y = aq . If a = 1, then (x, y) = (1, 1) is the trivial solution. Let a > 1. The given equa2q p tion becomes a pa = aqa , which reduces to pa2q = qa p . Hence p 6= q, so we distinguish two cases: (i) p > q. Then from a2q < a p we deduce p > 2q. We can rewrite the equation as p = a p−2qq, and putting p = 2q + d, d > 0, we obtain d = q(ad − 2). By induction, 2d − 2 > d for each d > 2, so we must have d ≤ 2. For d = 1 we get q = 1 and a = p = 3, and therefore (x, y) = (27, 3), which is indeed a solution. For d = 2 we get q = 1, a = 2, and p = 4, so (x, y) = (16, 2), which is another solution. (ii) p < q. As above, we get q/p = a2q−p , and setting d = 2q − p > 0, this is d transformed to ad = a(2a −1)p , or equivalently to d = (2ad − 1)p. However, this equality cannot hold, because 2ad − 1 > d for each a ≥ 2, d ≥ 1. The only solutions are thus (1, 1), (16, 2), and (27, 3). 4.38 Shortlisted Problems 1997 631 18. By symmetry, assume that AB > AC. The point D lies between M and P as well as between Q and R, and if we show that DM · DP = DQ · DR, it will imply that M, P, Q, R lie on a circle. Since the triangles ABC, AEF, AQR are similar, the points B,C, Q, R lie on a circle. Hence DB · DC = DQ · DR, and it remains to prove that DB · DC = DM · DP. However, the points B,C, E, F are concyclic, but so are the points E, F, D, M (they lie on the nine-point circle), and we obtain PB · PC = PE · PF = PD · PM. 2xy Set PB = x and PC = y. We have PM = x+y 2 and hence PD = x+y . It follows that DB = PB − PD = y(x−y) (x−y)2 x+y , and DM = 2(x+y) , 2 = DM · DP = xy(x−y) , as needed. (x+y)2 x(x−y) x+y , immediately obtain DB · DC DC = from which we 19. Using that an+1 = 0 we can transform the desired inequality into √ a1 + a2 + · · · + an+1 √ √ √ √ √ √ √ √ ≤ 1 a1 + ( 2 − 1) a2 + · · · + ( n + 1 − n) an+1 . (1) We shall prove by induction on n that (1) holds for any a1 ≥ a2 ≥ · · · ≥ an+1 ≥ 0, i.e., not only when an+1 = 0. For n = 0 the inequality is obvious. For the inductive step from n − 1 to n, where n ≥ 1, we need to prove the inequality √ √ √ √ √ a1 + · · · + an+1 − a1 + · · · + an ≤ ( n + 1 − n) an+1 . (2) Putting S = a1 + a2 + · · · + an , this simplifies to p √ √ √ S + an+1 − S ≤ nan+1 + an+1 − nan+1. For an+1 = 0 the inequality For an+1 > 0 we have that the func√ is obvious. √ a tion f (x) = x + an+1 − x = √x+an+1 +√x is strictly decreasing on R+ ; hence n+1 (2) will follow if we show that S ≥ nan+1 . However, the latter is true because a1 , . . . , an ≥ an+1 . Equality holds if and only if a1 = a2 = · · · = ak and ak+1 = · · · = an+1 = 0 for some k. √ √ Second solution. Setting bk = ak − ak+1 for k = 1, . . . , n we have ai = (bi + · · · + bn )2 , so the desired inequality after squaring becomes n ∑ kb2k + 2 ∑ k=1 1≤k<l≤n n kbk bl ≤ ∑ k b2k + 2 ∑ k=1 √ kl bk bl , 1≤k<l≤n which clearly holds. 20. To avoid dividing into cases regarding the position of the point X , we use oriented angles. 632 4 Solutions Let R be the foot of the perpendicular from X to BC. It is well known that the points P, Q, R lie on the corresponding Simson line. This line is a tangent to γ (i.e., the circle XDR) if and only if ∠PRD = ∠RX D. We have and ∠PRD = ∠PXB = 90◦ − ∠X BA = 90◦ − ∠X BC + ∠ABC = 90◦ − ∠DAC + ∠ABC ∠RXD = 90◦ − ∠ADB = 90◦ + ∠BCA − ∠DAC; hence ∠PRD = ∠RXD if and only if ∠ABC = ∠BCA, i.e, AB = AC. 21. For any permutation π = (y1 , y2 , . . . , yn ) of (x1 , x2 , . . . , xn ), denote by S(π ) the sum y1 + 2y2 + · · · + nyn . Suppose, contrary to the claim, that |S(π )| > n+1 2 for any π . Further, we note that if π ′ is obtained from π by interchanging two neighboring elements, say yk and yk+1 , then S(π ) and S(π ′ ) differ by |yk + yk+1 | ≤ n + 1, and consequently they must be of the same sign. Now consider the identity permutation π0 = (x1 , . . . , xn ) and the reverse permutation π0 = (xn , . . . , x1 ). There is a sequence of permutations π0 , π1 , . . . , πm = π0 such that for each i, πi+1 is obtained from πi by interchanging two neighboring elements. Indeed, by successive interchanges we can put xn in the first place, then xn−1 in the second place, etc. Hence all S(π0 ), . . . , S(πm ) are of the same sign. However, since |S(π0 ) + S(πm)| = (n + 1)|x1 + · · · + xn | = n + 1, this implies that one of S(π0) and S(π0 ) is smaller than n+1 2 in absolute value, contradicting the initial assumption. 22. (a) Suppose that f and g are such functions. From g( f (x)) = x3 we have f (x1 ) 6= f (x2 ) whenever x1 6= x2 . In particular, f (−1), f (0), and f (1) are three distinct numbers. However, since f (x)2 = f (g( f (x))) = f (x3 ), each of the numbers f (−1), f (0), f (1) is equal to its square, and so must be either 0 or 1. This contradiction shows that no such f , g exist. (b) The answer is yes. We begin with constructing functions F, G : (1, ∞) → (1, ∞) with the property F(G(x)) = x2 and G(F(x)) = x4 for x > 1. Define t ϕ (t) t ψ (t) the functions ϕ , ψ by F(22 ) = 22 and G(22 ) = 22 . These functions determine F and G on the entire interval (1, ∞), and satisfy ϕ (ψ (t)) = t + 1 and ψ (ϕ (t)) = t + 2. It is easy to find examples of ϕ and ψ : for example, ϕ (t) = 12 t + 1, ψ (t) = 2t. Thus we also arrive at an example for F, G: 1 log log x+1 2 2 F(x) = 22 2 = 22 √ log2 x , G(x) = 22 2 log2 log2 x 2 = 2log2 x . It remains only to extend these functions to the whole of R. This can be done as follows:   for x > 1,  G(x) for x > 1,  F(x) fe(x) = 1/F(1/x) for 0 < x < 1, ge(x) = 1/G(1/x) for 0 < x < 1,   x for x ∈ {0, 1}; x for x ∈ {0, 1}; 4.38 Shortlisted Problems 1997 and then f (x) = fe(|x|), g(x) = ge(|x|) 633 for x ∈ R. It is directly verified that these functions have the required property. 23. Let K, L, M, and N be the projections of O onto the lines AB, BC,CD, and DA, and let α1 , α2 , α3 , α4 , β1 , β2 , β3 , β4 denote the angles OAB, OBC, OCD, ODA, OAD, OBA, OCB, ODC, respectively. We start with the following observation: Since NK is a chord of the circle with diameter OA, we have OA sin ∠A = NK = ON cos α1 + OK cos β1 (because ∠ONK = α1 and ∠OKN = β1 ). Analogous equalities also hold: OB sin ∠B = KL = OK cos α2 + OL cos β2 , OC sin ∠C = LM = OL cos α3 + OM cos β3 and OD sin ∠D = MN = OM cos α4 + ON cos β4 . Now the condition in the problem can be restated as NK + LM = KL + MN (i.e., KLMN is circumscribed), i.e., OK(cos β1 − cos α2 ) + OL(cos α3 − cos β2 ) +OM(cos β3 − cos α4 ) + ON(cos α1 − cos β4 ) = 0. (1) To prove that ABCD is cyclic, it suffices to show that α1 = β4 . Assume the contrary, and let w.l.o.g. α1 > β4 . Then point A lies inside the circle BCD, which is further equivalent to β1 > α2 . On the other hand, from α1 + β2 = α3 + β4 we deduce α3 > β2 , and similarly β3 > α4 . Therefore, since the cosine is strictly decreasing on (0, π ), the left side of (1) is strictly negative, yielding a contradiction. 24. There is a bijective correspondence between representations in the given form of 2k and 2k + 1 for k = 0, 1, . . . , since adding 1 to every representation of 2k, we obtain a representation of 2k + 1, and conversely, every representation of 2k + 1 contains at least one 1, which can be removed. Hence, f (2k + 1) = f (2k). Consider all representations of 2k. The number of those that contain at least one 1 equals f (2k − 1) = f (2k − 2), while the number of those not containing a 1 equals f (k) (the correspondence is given by division of summands by 2). Therefore f (2k) = f (2k − 2) + f (k). (1) Summing these equalities over k = 1, . . . , n, we obtain f (2n) = f (0) + f (1) + · · · + f (n). (2) We first prove the right-hand inequality. Since f is increasing, and f (0)+ f (1) = f (2), (2) yields f (2n) ≤ n f (n) for n ≥ 2. Now f (23 ) = f (0) + · · · + f (4) = 2 10 < 23 /2 , and one can easily conclude by induction that f (2n+1 ) ≤ 2n f (2n ) < 2 2 2n · 2n /2 < 2(n+1) /2 for each n ≥ 3. We now derive the lower estimate. It follows from (1) that f (x + 2) − f (x) is increasing. Consequently, for each m and k < m we have f (2m + 2k) − f (2m) ≥ f (2m + 2k − 2) − f (2m − 2) ≥ · · · ≥ f (2m) − f (2m − 2k), so f (2m + 2k) + f (2m−2k) ≥ 2 f (2m). Adding all these inequalities for k = 1, 2, . . . , m, we obtain f (0)+ f (2)+ · · ·+ f (4m) ≥ (2m+ 1) f (2m). But since f (2) = f (3), f (4) = f (5) 634 4 Solutions etc., we also have f (1) + f (3) + · · · + f (4m − 1) > (2m − 1) f (2m), which together with the above inequality gives f (8m) = f (0) + f (1) + · · · + f (4m) > 4m f (2m). (3) 2 Finally, we have that the inequality f (2n ) > 2n /4 holds for n = 2 and n = 3, while 2 2 for larger n we have by induction f (2n ) > 2n−1 f (2n−2 ) > 2n−1+(n−2) /4 = 2n /4 . This completes the proof. Remark. Despite the fact that the lower estimate is more difficult, it is much weaker than the upper estimate. It can be shown that f (2n ) eventually (for large 2 n) exceeds 2cn for any c < 12 . 25. Let MR meet the circumcircle of triangle ABC again at a point X. We claim that X is the common point of the lines KP, LQ, MR. By symmetry, it will be enough to show that X lies on KP. It is easy to see that X and P lie on the same side of AB as K. Let Ia = AK ∩ BP be the excenter of △ABC corresponding to A. It is easy to calculate that ∠AIa B = γ /2, from which we get ∠RPB = ∠AIa B = ∠MCB = ∠RXB. Therefore R, B, P, X are concyclic. Now if P and K are on distinct sides Ia that Ti+1 Ai = Ti+1 I and Ti+2 Ai = Ti+2 I other case is similar), we have K C ∠RXP = 180◦ − ∠RBP = 90◦ − β /2 = ◦ ∠MAK = 180 − ∠RXK, from which X Q L it follows that K, X , P are collinear, as P claimed. Remark. It is not essential for the statement of the problem that R be an internal point of AB. Work with cases can be avoided using oriented angles. A R B M 26. Let us first examine the case that all the inequalities in the problem are actually equalities. Then an−2 = an−1 + an , an−3 = 2an−1 + an , . . . , a0 = Fn an−1 + Fn−1 an = 1, where Fn is the nth Fibonacci number. Then it is easy to see (from F1 + F2 +· · · + Fk = Fk+2 ) that  a0 + · · · + an = (Fn+2 − 1)an−1 + Fn+1 an = Fn+2 −1 Fn−1 (Fn+2 −1) F (F −1) + Fn+1 − an . Since n−1 Fn+2 ≤ Fn+1 , it follows that Fn Fn n −1 a0 + a1 + · · · + an ≥ Fn+2 Fn , with equality holding if and only if an = 0 and an−1 = F1n . We denote by Mn the required minimum in the general case. We shall prove by F −1 induction that Mn = n+2 Fn . For M1 = 1 and M2 = 2 it is easy to show that the formula holds; hence the inductive basis is true. Suppose that n > 2. The sequences 1, aa21 , . . . , aan1 and 1, aa32 , . . . , aan2 also satisfy the conditions of the problem. Hence we have   a2 an a0 + · · · + a n = a0 + a1 1 + + · · · + ≥ 1 + a1Mn−1 a1 a1 and 4.38 Shortlisted Problems 1997 635   a3 an a0 + · · · + a n = a0 + a1 + a2 1 + + · · · + ≥ 1 + a1 + a2 Mn−2 . a2 a2 Multiplying the first inequality by Mn−2 − 1 and the second one by Mn−1 , adding the inequalities and using that a1 + a2 ≥ 1, we obtain (Mn−1 + Mn−2 + 1)(a0 + · · · + an ) ≥ Mn−1 Mn−2 + Mn−1 + Mn−2 + 1, so Mn ≥ Since Mn−1 = Fn+2 −1 Fn . Fn+1 −1 Fn−1 Mn−1 Mn−2 + Mn−1 + Mn−2 + 1 . Mn−1 + Mn−2 + 1 and Mn−2 = Fn −1 Fn−2 , the above inequality easily yields Mn ≥ However, we have shown above that equality can occur; hence indeed the required minimum. Fn+2 −1 Fn is 636 4 Solutions 4.39 Solutions to the Shortlisted Problems of IMO 1998 1. We begin with the following observation: Suppose that P lies in △AEB, where E is the intersection of AC and BD (the other cases are similar). Let M, N be the feet of the perpendiculars from P to AC and BD respectively. We have SABP = SABE − SAEP − SBEP = 12 (AE · BE − AE · EN − BE · EM) = 12 (AM · BN − EM · EN). Similarly, SCDP = 12 (CM · DN − EM · EN). Therefore, we obtain SABP − SCDP = AM · BN − CM · DN . 2 (1) D Now suppose that ABCD is cyclic. C Then P is the circumcenter of ABCD; E hence M and N are the midpoints of AC N and BD. Hence AM = CM and BN = M DN; thus (1) gives us SABP = SCDP . P On the other hand, suppose that ABCD B is not cyclic and let w.l.o.g. PA = PB A > PC = PD. Then we must have AM > CM and BN > DN, and consequently by (1), SABP > SCDP . This proves the other implication. Second solution. Let F and G denote the midpoints of AB and CD, and assume that P is on the same side of FG as B and C. Since PF ⊥ AB, PG ⊥ CD, and ∠FEB = ∠ABE, ∠GEC = ∠DCE, a direct computation yields ∠FPG = ∠FEG = 90◦ + ∠ABE + ∠DCE. Taking into account that SABP = 12 AB · FP = FE · FP, we note that SABP = SCDP is equivalent to FE · FP = GE · GP, i.e., to FE/EG = GP/PF. But this last is equivalent to triangles EFG and PGF being similar, which holds if and only if EFPG is a parallelogram. This last is equivalent to ∠EFP = ∠EGP, or 2∠ABE = 2∠DCE. Thus SABP = SCDP is equivalent to ABCD being cyclic. Remark. The problems also allows an analytic solution, for example putting the x and y axes along the diagonals AC and BD. 2. If AD and BC are parallel, then ABCD is an isosceles trapezoid with AB = CD, so P is the midpoint of EF. Let M and N be the midpoints of AB and CD. Then MN k BC, and the distance d(E, MN) equals the distance d(F, MN) because B and D are the same distance from MN and EM/BM = FN/DN. It follows that the midpoint P of EF lies on MN, and consequently SAPD : SBPC = AD : BC. If AD and BC are not parallel, then they meet at some point Q. It is plain that △QAB ∼ △QCD, and since AE/AB = CF/CD, we also deduce that △QAE ∼ △QCF. Therefore ∠AQE = ∠CQF. Further, from these similarities we obtain QE/QF = QA/QC = AB/CD = PE/PF, which in turn means that QP is the internal bisector of ∠EQF. But since ∠AQE = ∠CQF, this is also the internal bisector of ∠AQB. Hence P is at equal distances from AD and BC, so again SAPD : SBPC = AD : BC. 4.39 Shortlisted Problems 1998 637 Remark. The part AB k CD could also be regarded as a limiting case of the other part. Second solution. Denote λ = AE AB , AB = a, BC = b, CD = c, DA = d, ∠DAB = α , c·d(E,AD)+a·d(F,AD) +aSFAD , we have SAPD = cSEADa+c = a+c λ cSABD +(1−λ )aSACD 1 1 . Since SABD = 2 ad sin α and SACD = 2 cd sin β , we are led to a+c acd abc SAPD = a+c [λ sin α + (1 − λ ) sin β ], and analogously SBPC = a+c [λ sin α + (1 − λ ) sin β ]. Thus we obtain SAPD : SBPC = d : b. ∠ABC = β . Since d(P, AD) = 3. Lemma. If U,W,V are three points on a line l in this order, and X a point in the plane with XW ⊥ UV , then ∠UXV < 90◦ if and only if XW 2 > UW ·VW . Proof. Let XW 2 > UW ·VW , and let X0 be a point on the segment XW such that X0W 2 ≥ UW · VW . Then X0W /UW = VW /X0W , so that triangles X0WU and VW X0 are similar. Thus ∠UX0V = ∠UX0W + ∠WUX0 = 90◦ , which immediately implies that ∠UXV < 90◦ . Similarly, if XW 2 ≤ UW ·VW , then ∠UXV ≥ 90◦ . Since BI ⊥ RS, it will be enough by the lemma to show that BI 2 > BR · BS. Note that △BKR ∼ △BSL: in fact, we have ∠KBR = ∠SBL = 90◦ − β /2 and ∠BKR = ∠AKM = ∠KLM = ∠BSL = 90◦ − α /2. In particular, we obtain BR/BK = BL/BS = BK/BS, so that BR · BS = BK 2 < BI 2 . Second solution. Let E, F be the midpoints of KM and LM respectively. The quadrilaterals RBIE and SBIF are inscribed in the circles with diameters IR and IS. Now we have ∠RIS = ∠RMS + ∠IRM + ∠ISM = 90◦ − β /2 + ∠IBE + ∠IBF = 90◦ − β /2 + ∠EBF. On the other hand, BE and BF are medians in △BKM and △BLM in which BM > BK and BM > BL. We conclude that ∠MBE < 12 ∠MBK and ∠MBF < 1 2 ∠MBL. Adding these two inequalities gives ∠EBF < β /2. Therefore ∠RIS < 90◦ . Remark. It can be shown (using vectors) that the statement remains true for an arbitrary line t passing through B. 4. Let K be the point on the ray BN with ∠BCK = ∠BMA. Since ∠KBC = ∠ABM, we get △BCK ∼ △BMA. It follows that BC/BM = BK/BA, which implies that also △BAK ∼ △BMC. The quadrilateral ANCK is cyclic, because ∠BKC = ∠BAM = ∠NAC. Then by Ptolemy’s theorem we obtain AC · BK = AC · BN + AN ·CK +CN · AK. On the other hand, from the similarities noted above we get CK = BC · AM AB ·CM AB · BC , AK = and BK = . BM BM BM After substitution of these values, the equality (1) becomes AB · BC · AC BC · AM · AN AB ·CM ·CN = AC · BN + + , BM BM BM which is exactly the equality we must prove multiplied by AB·BC·CA . BM (1) 638 4 Solutions 5. Let G be the centroid of △ABC and H the homothety with center G and ratio − 12 . It is well-known that H C2 A B2 D′ maps H into O. For every other point O F′ X , let us denote by X ′ its image under E C′ B′ H . Also, let A2 B2C2 be the triangle in which A, B,C are the midpoints of G E′ B2C2 , C2 A2 , and A2 B2 , respectively. B C A′ It is clear that A′ , B′ , and C′ are the midpoints of BC, CA, and AB respectF ively. We also have that D′ is the reH flection of A′ across B′C′ . Thus D′ D must lie on B2C2 and A′ D′ ⊥ B2C2 . A2 ′ However, it also holds that OA and B2C2 are orthogonal, so we conclude that O, D′ , A′ are collinear and D′ is the projection of O on B2C2 . Analogously, E ′ , F ′ are the projections of O on C2 A2 and A2 B2 . Now we apply Simson’s theorem. It claims that D′ , E ′ , F ′ are collinear (which is equivalent to D, E, F being collinear) if and only if O lies on the circumcircle of A2 B2C2 . However, this circumcircle is centered at H with radius 2R, so the last condition is equivalent to HO = 2R. 6. Let P be the point such that △CDP and △CBA are similar and equally oriented. BC Since then ∠DCP = ∠BCA and CA = DC CP , it follows that ∠ACP = ∠BCD and AC BC BC DB CP = CD , so △ACP ∼ △BCD. In particular, CA = PA . Furthermore, by the conditions of the problem we have ∠EDP = 360◦ − ∠B − PD PD CD AB CD AF ∠D = ∠F and DE = CD · DE = BC · DE = FE . Therefore △EDP ∼ △EFA as well, so that similarly as above we conclude that △AEP ∼ △FED and conseAE PA quently EF = FD . BC AE FD PA FD Finally, CA · EF · DB = DB PA · FD · DB = 1. Second solution. Let a, b, c, d, e, f be the complex coordinates of A, B, C, D, E, c−d e− f F, respectively. The condition of the problem implies that a−b b−c · d−e · f −a = −1. On the other hand, since (a − b)(c − d)(e − f ) + (b − c)(d − e)( f − a) = (b − c)(a − e)( f − d) + (c − a)(e − f )(d − b) holds identically, we immediately dea−e f −d BC AE FD duce that b−c c−a · e− f · d−b = −1. Taking absolute values gives CA · EF · DB = 1. 7. We shall use the following result. Lemma. In a triangle ABC with BC = a, CA = b, and AB = c, (a) ∠C = 2∠B if and only if c2 = b2 + ab; (b) ∠C + 180◦ = 2∠B if and only if c2 = b2 − ab. Proof. (a) Take a point D on the extension of BC over C such that CD = b. The condition ∠C = 2∠B is equivalent to ∠ADC = 12 ∠C = ∠B, and thus to AD = AB = c. This is further equivalent to triangles CAD and ABD being similar, so CA/AD = AB/BD, i.e., c2 = b(a + b). 4.39 Shortlisted Problems 1998 639 (b) Take a point E on the ray CB such that CE = b. As above, ∠C + 180◦ = 2∠B if and only if △CAE ∼ △ABE, which is equivalent to EB/BA = EA/AC, or c2 = b(b − a). Let F, G be points on the ray CB such that CF = 13 a and CG = 43 a. Set BC = a, CA = b, AB = c, EC = b1 , and EB = c1 . By the lemma it follows that c2 = b2 + ab. Also b1 = AG and c1 = AF, so Stewart’s theorem gives us c21 = 23 b2 + 1 2 2 2 1 2 2 1 2 4 2 2 2 c + 49 a2 = b2 + 43 ab + 49 a2 . It 3 c − 9 a = b + 3 ab − 9 a and b1 = − 3 b + 3  2 2 follows that b1 = 3 a + b and c21 = b21 − ab + 3 a2 = b21 − ab1. The statement of the problem follows immediately from the lemma. 8. Let M be the point of intersection of AE and BC, and let N be the point on ω diametrically opposite A. A Since ∠B < ∠C, points N and B are ω on the same side of AE. Furthermore, Y Z ∠NAE = ∠BAX = 90◦ − ∠ABE; hence the triangles NAE and BAX are similar. B M C D Consequently, △BAY and △NAM are X E also similar, since M is the midpoint N of AE. Thus ∠ANZ = ∠ABZ = ∠ABY = ∠ANM, implying that N, M, Z are collinear. Now we have ∠ZMD = 90◦ − ∠ZMA = ∠EAZ = ∠ZED (the last equality because ED is tangent to ω ); hence ZMED is a cyclic quadrilateral. It follows that ∠ZDM = ∠ZEA = ∠ZAD, which is enough to conclude that MD is tangent to the circumcircle of AZD. Remark. The statement remains valid if ∠B ≥ ∠C. 9. Set an+1 = 1−(a1 +· · ·+an ). Then an+1 > 0, and the desired inequality becomes a1 a2 · · · an+1 1 ≤ . (1 − a1)(1 − a2 ) · · · (1 − an+1 ) nn+1 To prove it, we observe that √ 1 − ai = a1 + · · · + ai−1 + ai+1 + · · · + an+1 ≥ n n a1 · · · ai−1 ai+1 · · · an+1 . Multiplying these inequalities for i = 1, 2, . . . , n +1, we get exactly the inequality we need. 10. We shall first prove the inequality for n of the form 2k , k = 0, 1, 2, . . . . The case k = 0 is clear. For k = 1, we have √ √ √ ( r1 r2 − 1)( r1 − r2 )2 1 1 2 + −√ = ≥ 0. √ r1 + 1 r2 + 1 r1 r2 + 1 (r1 + 1)(r2 + 1)( r1 r2 + 1) For the inductive step it suffices to show that the claim for k and 2 implies that for k + 1. Indeed, 2k+1 ∑ i=1 1 2k 2k ≥ √ + √ k k 2 r r ···r k + 1 2 r k ri + 1 1 2 2 2 +1 r2k +2 · · · r2k+1 + 1 2k+1 ≥ k+1 , √ 2 r1 r2 · · · r2k+1 + 1 (1) 640 4 Solutions and the induction is complete. We now show that if the statement holds for 2k , then it holds for every n < 2k as √ well. Put rn+1 = rn+2 = · · · = r2k = n r1 r2 . . . rn . Then (1) becomes 1 1 2k − n 2k + ···+ +√ ≥ . √ n r ···r + 1 n r ···r + 1 r1 + 1 rn + 1 n n 1 1 This proves the claim. Second solution. Define ri = exi , where xi > 0. The function f (x) = vex for x > 0: indeed, f ′′ (x) to f (x1 ), . . . , f (xn ), we get = 1 r1 +1 ex (ex −1) (ex +1)3 1 1+ex is con- > 0. Thus by Jensen’s inequality applied + · · · + rn 1+1 ≥ n √ n r ···rn +1 . 1 11. The given inequality is equivalent to x3 (x + 1) + y3 (y + 1) + z3 (z + 1) ≥ 34 (x + 1)(y + 1)(z + 1). By the A-G mean inequality, it will be enough to prove a stronger inequality: 1 x4 + x3 + y4 + y3 + z4 + z3 ≥ [(x + 1)3 + (y + 1)3 + (z + 1)3]. 4 (1) If we set Sk = xk + yk + zk , (1) takes the form S4 + S3 ≥ 14 S3 + 34 S2 + 34 S1 + 34 . Note that by the A-G mean inequality, S1 = x + y + z ≥ 3. Thus it suffices to prove the following: If S1 ≥ 3 and m > n are positive integers, then Sm ≥ Sn . This can be shown in many ways. For example, by Hölder’s inequality, (xm + ym + zm )n/m (1 + 1 + 1)(m−n)/m ≥ xn + yn + zn . (Another way is using the Chebyshev inequality: if x ≥ y ≥ z then xk−1 ≥ yk−1 ≥ zk−1 ; hence Sk = x · xk−1 + y · yk−1 + z · zk−1 ≥ 13 S1 Sk−1 , and the claim follows by induction.) Second solution. Assume that x ≥ y ≥ z. Then also 1 (x+1)(y+1) . 1 (y+1)(z+1) Hence Chebyshev’s inequality gives that ≥ 1 (x+1)(z+1) ≥ x3 y3 z3 + + (1 + y)(1 + z) (1 + x)(1 + z) (1 + x)(1 + y) ≥ 1 (x3 + y3 + z3 ) · (3 + x + y + z) . 3 (1 + x)(1 + y)(1 + z) Now if we put x + y + z = 3S, we have x3 + y3 + z3 ≥ 3S and (1 + x)(1 + y)(1 + z) ≤ (1 + a)3 by the A-G mean inequality. Thus the needed inequality reduces to 6S3 ≥ 34 , which is obviously true because S ≥ 1. (1+S)3 Remark. Both these solutions use only that x + y + z ≥ 3. 4.39 Shortlisted Problems 1998 641 12. The assertion is clear for n = 0. We shall prove the general case by induction on n. Suppose that c(m, i) = c(m, m − i) for all i and m ≤ n. Then by the induction hypothesis and the recurrence formula we have c(n + 1, k) = 2k c(n, k) + c(n, k − 1) and c(n + 1, n + 1 − k) = 2n+1−k c(n, n + 1 − k) + c(n, n − k) = 2n+1−k c(n, k − 1) + c(n, k). Thus it remains only to show that (2k − 1)c(n, k) = (2n+1−k − 1)c(n, k − 1). We prove this also by induction on n. By the induction hypothesis, c(n − 1, k) = and c(n − 1, k − 2) = 2n−k − 1 c(n − 1, k − 1) 2k − 1 2k−1 − 1 c(n − 1, k − 1). 2n+1−k − 1 Using these formulas and the recurrence formula we obtain (2k − 1)c(n, k) − (2n+1−k − 1)c(n, k − 1) = (22k − 2k )c(n − 1, k) − (2n − 3 · 2k−1 + 1)c(n − 1, k − 1) − (2n+1−k − 1)c(n − 1, k − 2) = (2n − 2k )c(n − 1, k − 1) − (2n − 3 · 2k−1 + 1)c(n − 1, k − 1) − (2k−1 − 1)c(n − 1, k − 1) = 0. This completes the proof. Second solution. The given recurrence formula resembles that of binomial coefficients, so it is natural to search for an explicit formula of the form c(n, k) = F(n) F(k)F(n−k) , where F(m) = f (1) f (2) · · · f (m) (with F(0) = 1) and f is a certain function from the natural numbers to the real numbers. If there is such an f , then c(n, k) = c(n, n − k) follows immediately. After substitution of the above relation, the recurrence equivalently reduces to f (n + 1) = 2k f (n − k + 1) + f (k). It is easy to see that f (m) = 2m − 1 satisfies this relation. Remark. If we introduce the polynomial Pn (x) = ∑nk=0 c(n, k)xk , the recurrence relation gives P0 (x) = 1 and Pn+1 (x) = xPn (x) + Pn (2x). As a consequence of the problem, all polynomials in this sequence are symmetric, i.e., Pn (x) = xn Pn (x−1 ). 13. Denote by F the set of functions considered. Let f ∈ F , and let f (1) = a. Putting n = 1 and m = 1 we obtain f ( f (z)) = a2 z and f (az2 ) = f (z)2 for all z ∈ N. These equations, together with the original one, imply f (x)2 f (y)2 = f (x)2 f (ay2 ) = f (x2 f ( f (ay2 ))) = f (x2 a3 y2 ) = f (a(axy)2 ) = f (axy)2 , which implies f (axy) = f (x) f (y) for all x, y ∈ N. Thus f (ax) = a f (x), and we conclude that a f (xy) = f (x) f (y) for all x, y ∈ N. (1) We now prove that f (x) is divisible by a for each x ∈ N. In fact, we inductively get that f (x)k = ak−1 f (xk ) is divisible by ak−1 for every k. If pα and pβ are the exact powers of a prime p that divide f (x) and a respectively, we deduce that kα ≥ (k − 1)β for all k, so we must have α ≥ β for any p. Therefore a | f (x). Now we consider the function on natural numbers g(x) = f (x)/a. The above relations imply 642 4 Solutions g(1) = 1, g(xy) = g(x)g(y), g(g(x)) = x for all x, y ∈ N. (2) Since g ∈ F and g(x) ≤ f (x) for all x, we may restrict attention to the functions g only. Clearly g is bijective. We observe that g maps a prime to a prime. Assume to the contrary that g(p) = uv, u, v > 1. Then g(uv) = p, so either g(u) = 1 or g(v) = 1. Thus either g(1) = u or g(1) = v, which is impossible. We return to the problem of determining the least possible value of g(1998). Since g(1998) = g(2 · 33 · 37) = g(2) · g(3)3 · g(37), and g(2), g(3), g(37) are distinct primes, g(1998) is not smaller than 23 ·3 ·5 = 120. On the other hand, the value of 120 is attained for any function g satisfying (2) and g(2) = 3, g(3) = 2, g(5) = 37, g(37) = 5. Hence the answer is 120. 14. If x2 y + x + y is divisible by xy2 + y + 7, then so is the number y(x2 y + x + y) − x(xy2 + y + 7) = y2 − 7x. If y2 − 7x ≥ 0, then since y2 − 7x < xy2 + y + 7, it follows that y2 − 7x = 0. Hence (x, y) = (7t 2 , 7t) for some t ∈ N. It is easy to check that these pairs really are solutions. If y2 − 7x < 0, then 7x − y2 > 0 is divisible by xy2 + y + 7. But then xy2 + y + 7 ≤ 7x − y2 < 7x, from which we obtain y ≤ 2. For y = 1, we are led to x + 8 | 7x − 1, and hence x + 8 | 7(x + 8) − (7x − 1) = 57. Thus the only possibilities are x = 11 and x = 49, and the obtained pairs (11, 1), (49, 1) are indeed solutions. For y = 2, we have 4x + 9 | 7x − 4, so that 7(4x + 9) − 4(7x − 4) = 79 is divisible by 4x + 9. We do not get any new solutions in this case. Therefore all required pairs (x, y) are (7t 2 , 7t) (t ∈ N), (11, 1), and (49, 1). 15. The condition is obviously satisfied if a = 0 or b = 0 or a = b or a, b are both integers. We claim that these are the only solutions. Suppose that a, b belong to none of the above categories. The quotient a/b = ⌊a⌋/⌊b⌋ is a nonzero rational number: let a/b = p/q, where p and q are coprime nonzero integers. Suppose that p 6∈ {−1, 1}. Then p divides ⌊an⌋ for all n, so in particular p divides ⌊a⌋ and thus a = kp + ε for some k ∈ N and 0 ≤ ε < 1. Note that ε 6= 0, since otherwise b = kq would also be an integer. It follows that there exists an n ∈ N such that 1 ≤ nε < 2. But then ⌊na⌋ = ⌊knp + nε ⌋ = knp + 1 is not divisible by p, a contradiction. Similarly, q 6∈ {−1, 1} is not possible. Therefore we must have p, q = ±1, and since a 6= b, the only possibility is b = −a. However, this leads to ⌊−a⌋ = −⌊a⌋, which is not valid if a is not an integer. 16. Let S be a set of integers such that for no four distinct elements a, b, c, d ∈ S, it holds that 20 | a + b − c − d. It is easily seen that there cannot exist distinct elements a, b, c, d with a ≡ b and c ≡ d (mod 20). Consequently, if the elements of S give k different residues modulo 20, then S itself has at most k + 2 elements. Next, consider these k elements of S with different residues modulo 20. They give k(k−1) different sums of two elements. For k ≥ 7 there are at least 21 such 2 sums, and two of them, say a + b and c + d, are equal modulo 20; it is easy to 4.39 Shortlisted Problems 1998 643 see that a, b, c, d are distinct. It follows that k cannot exceed 6, and consequently S has at most 8 elements. An example of a set S with 8 elements is {0, 20, 40, 1, 2, 4, 7, 12}. Hence the answer is n = 9. 17. Initially, we determine that the first few values for an are 1, 3, 4, 7, 10, 12, 13, 16, 19, 21, 22, 25. Since these are exactly the numbers of the forms 3k + 1 and 9k + 3, we conjecture that this is the general pattern. In fact, it is easy to see that the equation x + y = 3z has no solution in the set K = {3k + 1, 9k + 3 | k ∈ N}. We shall prove that the sequence {an } is actually this set ordered increasingly. Suppose an > 25 is the first member of the sequence not belonging to K. We have several cases: (i) an = 3r +2, r ∈ N. By the assumption, one of r + 1, r +2, r + 3 is of the form 3k + 1 (and smaller than an ), and therefore is a member ai of the sequence. Then 3ai equals an + 1, an + 4, or an + 7, which is a contradiction because 1, 4, 7 are in the sequence. (ii) an = 9r, r ∈ N. Then an + a2 = 3(3r + 1), although 3r + 1 is in the sequence, a contradiction. (iii) an = 9r + 6, r ∈ N. Then one of the numbers 3r + 3, 3r + 6, 3r + 9 is a member a j of the sequence, and thus 3a j is equal to an + 3, an + 12, or an +21, where 3, 12, 21 are members of the sequence, again a contradiction. Once we have revealed the structure of the sequence, it is easy to compute a1998 . We have 1998 = 4 · 499 + 2, which implies a1998 = 9 · 499 + a2 = 4494. 18. We claim that, if 2n − 1 divides m2 + 9 for some m ∈ N, then n must be a power of 2. Suppose otherwise that n has an odd divisor d > 1. Then 2d − 1 | 2n − 1 is also a divisor of m2 + 9 = m2 + 32 . However, 2d − 1 has some prime divisor p of the form 4k − 1, and by a well-known fact, p divides both m and 3. Hence p = 3 divides 2d − 1, which is impossible, because for d odd, 2d ≡ 2 (mod 3). Hence n = 2r for some r ∈ N. Now let n = 2r . We prove the existence of m by induction on r. The case r = 1 is trivial. Now for any r > 1 note that r 22 − 1 = (22 r−1 − 1)(22 r−1 + 1). r−1 The induction hypothesis claims that there exists an m1 such that 22 − 1 | r−1 r−2 m21 + 9. We also observe that 22 + 1 | m22 + 9 for simple m2 = 3 · 22 . By the Chinese remainder theorem, there is an m ∈ N that satisfies m ≡ m1 (mod r−1 r−1 22 − 1) and m ≡ m2 (mod 22 + 1). It is easy to see that this m2 + 9 will be r−1 r−1 r divisible by both 22 −1 and 22 + 1, i.e., that 22 −1 | m2 +9. This completes the induction. α α 19. For n = p1 1 p2 2 · · · pαr r , where pi are distinct primes and αi natural numbers, we have τ (n) = (α1 + 1) · · · (αr + 1) and τ (n2 ) = (2α1 + 1) . . . (2αr + 1). Putting ki = αi + 1, the problem reduces to determining all natural values of m that can be represented as 644 4 Solutions m= 2k1 − 1 2k2 − 1 2kr − 1 · ··· . k1 k2 kr (1) Since the numerator τ (n2 ) is odd, m must be odd too. We claim that every odd m has a representation of the form (1). The proof will be done by induction. This is clear for m = 1. Now for every m = 2k − 1 with k odd the result follows easily, since m = 2k−1 k · k, and k can be written as (1). We cannot do the same if k is even; however, in the case m = 4k − 1 with k odd, we can write it as 6k−1 m = 12k−3 6k−1 · 3k · k, and this works. In general, suppose that m = 2t k − 1, with k odd. Following the same pattern, we can write m as m= 2t (2t − 1)k − (2t − 1) 4(2t − 1)k − 3 2(2t − 1)k − 1 ··· · · k. t−1 − 1)k − (2 − 1) 2(2t − 1)k − 1 (2t − 1)k 2t−1 (2t The induction is finished. Hence m can be represented as odd. τ (n2 ) τ (n) if and only if it is n 20. We first consider the special case n = 3r . Then the simplest choice 10 9−1 = 11 . . . 1 (n digits) works. This can be shown by induction: it is true for r = 1, while the inductive step follows from  r−1   r r−1 r−1 103 − 1 = 103 − 1 102·3 + 103 + 1 , because the second factor is divisible by 3. In the general case, let k ≥ n/2 be a positive integer and a1 , . . . , an−k be nonzero digits. We have A = (10k − 1)a1 a2 . . . an−k = a1 a2 . . . an−k−1 a′n−k 99 . . 99} b1 b2 . . . bn−k−1 b′n−k , | .{z 2k−n where a′n−k = an−k − 1, bi = 9 − ai, and b′n−k = 9 − a′n−k . The sum of digits of A equals 9k independently of the choice of digits a1 , . . . , an−k . Thus we need only choose k ≥ n2 and digits a1 , . . . , an−k−1 6∈ {0, 9} and an−k ∈ {0, 1} in order for the conditions to be fulfilled. Let us choose  r 3 , if 3r < n ≤ 2 · 3r for some r ∈ Z, k= 2 · 3r , if 2 · 3r < n ≤ 3r+1 for some r ∈ Z; and a1 a2 . . . an−k = 22 . . . 2. The number A = 22 . . . 2} 1 |99 .{z . .99} 77 . . . 7} 8 | {z | {z n−k−1 2k−n n−k−1 thus obtained is divisible by 2 · (10k − 1), which is, as explained above, divisible by 18 · 3r . Finally, the sum of digits of A is either 9 · 3r or 18 · 3r ; thus A has the desired properties. 4.39 Shortlisted Problems 1998 645 21. Such a sequence is obviously strictly increasing. We note that it must be unique. Indeed, given a0 , a1 , . . . , an−1 , then an is the least positive integer not of the form ai + 2a j + 4ak , i, j, k < n. We easily get that the first few an ’s are 0, 1, 8, 9, 64, 65, 72, 73, . . .. Let {cn } be the increasing sequence of all positive integers that consist of zeros and ones in base 8, i.e., those of the form t0 + 23t1 + · · · + 23q tq where ti ∈ {0, 1}. We claim that an = cn . To prove this, it is enough to show that each m ∈ N can be uniquely written as ci + 2c j + 4ck . If m = t0 + 2t1 + · · · + 2rtr (ti ∈ {0, 1}), then m = ci + 2c j + 22 ck is obviously possible if and only if ci = t0 + 23t3 + 26t6 + · · · , c j = t1 + 23t4 + . . ., and ck = t2 + 23t5 + · · · . Hence for n = s0 + 2s1 + · · · + 2r sr we have an = s0 + 8s1 + · · · + 8r sr . In particular, 1998 = 2 + 22 + 23 + 26 + 27 + 28 + 29 + 210, so a1998 = 8 + 82 + 83 + 86 + 87 + 88 + 89 + 810 = 1227096648. Second solution. Define f (x) = xa0 + xa1 + · · · . Then the assumed property of {an } gives 1 f (x) f (x2 ) f (x4 ) = ∑ xai +2a j +4ak = ∑ xn = . 1 − x n i, j,k We also get as a consequence f (x2 ) f (x4 ) f (x8 ) = x) f (x8 ). Continuing this, we obtain 1 , 1−x2 which gives f (x) = (1 + 2 f (x) = (1 + x)(1 + x8)(1 + x8 ) · · · . Hence the an ’s are integers that have only 0’s and 1’s in base 8. 22. We can obviously change each x into ⌊x⌋ or ⌈x⌉ so that the column sums remain unchanged. However, this does not necessarily match the row sums as well, so let us consider the sum S of the absolute values of the changes in the row sums. It is easily seen that S is even, and we want it to be 0. A row may have a higher or lower sum than desired. Let us mark a cell by − if its entry x was changed to ⌊x⌋, and by + if it was changed to ⌈x⌉ instead. We call a row R2 accessible from a row R1 if there is a column C such that C ∩ R1 is marked + and C ∩ R2 is marked −. Note that a column containing a + must contain a − as well, because column sums are unchanged. Hence from each row with a higher sum we can access another row. Assume that the row sum in R1 is higher. If R1 , R2 , . . . , Rk is a sequence of rows such that Ri+1 is accessible from Ri via some column Ci and such that the row sum in Rk is lower, then by changing the signs in Ci ∩ Ri and Ci ∩ Ri+1 (i = 1, 2, . . . , k − 1) we decrease S by 2, leaving column sums unchanged. We claim that such a sequence of rows always exists. Let R be the union of all rows that are accessible from R1 , directly or indirectly; let R be the union of the remaining rows. We show that for any column C, the sum in R ∩C is not higher. If R ∩C contains no +’s, then this is clear. If R ∩C 646 4 Solutions contains a +, since the rows of R are not accessible, the set R ∩C contains no −’s. It follows that the sum in R ∩ C is not lower, and since column sums are unchanged, we again come to the same conclusion. Thus the total sum in R is not higher. Therefore, there is a row in R with too low a sum, justifying our claim. 23. (a) If n is even, then every odd integer is unattainable. Assume that n ≥ 9 is odd. Let a be obtained by addition from some b, and b from c by multiplication. Then a is 2c + 2, 2c + n, nc + 2, or nc + n, and is in every case congruent to 2c + 2 modulo n − 2. In particular, if a ≡ −2 (mod n − 2), then also b ≡ −4 and c ≡ −2 (mod n − 2). Now consider any a = kn(n − 2) − 2, where k is odd. If it is attainable, but not divisible by 2 or n, it must have been obtained by addition. Thus all predecessors of a are congruent to either −2 or −4 (mod n − 2), and none of them equals 1, a contradiction. (b) Call an attainable number addy if the last operation is addition, and multy if the last operation is multiplication. We prove the following claims by simultaneous induction on k: (1) n = 6k is both addy and multy; (2) n = 6k + 1 is addy for k ≥ 2; (3) n = 6k + 2 is addy for k ≥ 1; (4) n = 6k + 3 is addy; (5) n = 6k + 4 is multy for k ≥ 1; (6) n = 6k + 5 is addy. The cases k ≤ 1 are easily verified. For k ≥ 2, suppose all six statements hold up to k − 1. Since 3k is addy, 6k is multy. Next, 6k − 2 is multy, so both 6k = (6k − 2) + 2 and 6k + 1 = (6k − 2) + 3 are addy. Since 6k is multy, both 6k + 2 and 6k + 3 are addy. Number 6k + 4 = 2 · (3k + 2) is multy, because 3k + 2 is addy (being either 6l + 2 or 6l + 5). Finally, we have 6k + 5 = 3 · (2k + 1) + 2. Since 2k + 1 is 6l + 1, 6l + 3, or 6l + 5, it is addy except for 7. Hence 6k + 5 is addy except possibly for 23. But 23 = ((1 · 2 + 2) · 2 + 2) · 2 + 3 is also addy. This completes the induction. Now 1 is given and 2 = 1 · 2, 4 = 1 + 3. It is easily checked that 7 is not attainable, and hence it is the only unattainable number. 24. Let f (n) be the minimum number of moves needed to monotonize any permutation of n distinct numbers. Let us be given a permutation π of {1, 2, . . ., n}, and let k be the first element of π . In f (n − 1) moves, we can transform π to either (k, 1, 2, . . . , k − 1, k + 1, . . ., n) or (k, n, n − 1, . . .,k + 1, k − 1, . . ., 1). Now the former can be changed to (k, k − 1, . . . , 2, 1, k + 1, . . . , n), which is then monotonized in the next move. Similarly, the latter also can be monotonized in 4.39 Shortlisted Problems 1998 647 two moves. It follows that f (n) ≤ f (n − 1) + 2. Thus we shall be done if we show that f (5) ≤ 4. First we note that f (3) = 1. Consider a permutation of {1, 2, 3, 4}. If either 1 or 4 is the first or the last element, we need one move to monotonize the other three elements, and at most one more to monotonize the whole permutation. Of the remaining four permutations, (2, 1, 4, 3) and (3, 4, 1, 2) can also be monotonized in two moves. The permutations (2, 4, 1, 3) and (3, 1, 4, 2) require 3 moves, but by this we can choose whether to change them into (1, 2, 3, 4) or (4, 3, 2, 1). We now consider a permutation of {1, 2, 3, 4, 5}. If either 1 or 5 is in the first or last position, we can monotonize the rest in 3 moves, but in such a way that the whole permutation can be monotonized in the next move. If this is not the case, then either 1 or 5 is in the second or fourth position. Then we simply switch it to the outside in one move and continue as in the former case. Hence f (5) = 4, as desired. 25. We use induction on n. For n = 3, we have a single two-element subset {i, j} that is split by (i, k, j) (where k is the third element of U ). Assume that the result holds for some n ≥ 3, and consider a family F of n − 1 proper subsets of U = {1, 2, . . . , n + 1}, each with at least 2 elements. To continue the induction, we need an element a ∈ U that is contained in all nelement subsets of F , but in at most one of the two-element subsets. We claim that such an a exists. Let F contain k n-element subsets and m 2-element subsets (k + m ≤ n − 1). The intersection of the n-element subsets contains exactly n + 1 − k ≥ m + 2 elements. On the other hand, at most m elements belong to more than one 2-element subset, which justifies our claim. Now let A be the 2-element subset that contains a, if it exists; otherwise, let A be any subset from F containing a. Excluding a from all the subsets from F \ {A}, we get at most n − 2 subsets of U \ {a} with at least 2 and at most n − 1 elements. By the inductive hypothesis, we can arrange U \ {a} so that we split all the subsets of F except A. It remains to place a, and we shall make a desired arrangement if we put it anywhere away from A.  26. Put n = 2r + 1. Since each of the n2 pairs of judges agrees on at most two  candidates, the total number of agreements is at most k n2 . On the other hand, if the ith candidate is passed by xi judges and failed by n − xi judges, then the number of agreements on this candidate equals     xi n − xi x2 + (n − xi)2 − n r2 + (n − r)2 − n (n − 1)2 + = i ≥ = . 2 2 2 2 4 2 Therefore the total number of agreements is at least m(n−1) , which implies that 4   n m(n − 1)2 k n−1 k ≥ , hence ≥ . 2 4 m 2n  Remark. The obtained inequality is sharp. Indeed, if m = 2r+1 and each canr didate is passed by a different subset of r judges, we get equality. A similar 648 4 Solutions example shows that the result is not valid for even n. In that case the weaker n−2 estimate mk ≥ 2n−2 holds. 27. Since this is essentially a graph problem, we call the points and segments vertices and edges of the graph. We first prove that the task is impossible if k ≤ 4. Cases k ≤ 2 are trivial. If k = 3, then among the edges from a vertex A there are two of the same color, say AB and AC, so we don’t have all the three colors among the edges joining A, B,C. Now let k = 4, and assume that there is a desired coloring. Consider the edges incident with a vertex A. At least three of them have the same color, say blue. Suppose that four of them, AB, AC, AD, AE, are blue. There is a blue edge, say BC, among the ones joining B,C, D, E. Then four of the edges joining A, B,C, D are blue, and we cannot complete the coloring. So, exactly three edges from A are blue: AB, AC, AD. Also, of the edges connecting any three of the 6 vertices other than A, B,C, D, one is blue (because the edges joining them with A are not so). By a classical result, there is a blue triangle EFG with vertices among these six. Now one of EB, EC, ED must be blue as well, because none of BC, BD,CD is. Let it be EB. Then four of the edges joining B, E, F, G are blue, which is impossible. For k = 5 the task is possible. Label the vertices 0, 1, . . . , 9. For each color, we divide the vertices into four groups and paint in this color every edge joining two from the same group, as shown below. Then among any 5 vertices, 2 must belong to the same group, and the edge connecting them has the considered color. yellow: red: blue: green: orange: 01 12 20 23 34 42 45 56 64 67 78 86 89 90 08 36 69 93 58 81 15 70 03 37 92 25 59 14 47 71 57 79 91 13 35 48 60 82 04 26. A desired coloring can be made for k ≥ 6 as well. Paint the edge i j in the (i+ j)th color for i < j ≤ 8, and in the 2ith color if j = 9 (the addition being modulo 9). We can ignore the edges painted with the extra colors. Then the edges of one color appear as five disjoint segments, so that any complete k-graph for k ≥ 5 contains one of them. 28. Let A be the number of markers with white side up, and B the number of pairs of markers whose squares share a side. We claim that A + B does not change its parity as the game progresses. Suppose that in some move we remove a marker that has exactly k neighbors, among them r with white side up (0 ≤ r ≤ k ≤ 4). Of course, this marker has its black side up. When it is removed, the r white markers get black side up, while the k − r black ones become white. Thus A changes by k − 2r. As for B, it decreases by k. It follows that A decreases by 2r and preserves its parity, as claimed. Initially, A = mn − 1 and B = m(n − 1) + n(m − 1); hence A + B = 3mn − m − n − 1. 4.39 Shortlisted Problems 1998 649 If we succeed in removing all the markers, we end up with A + B = 0. Hence 3mn − m − n − 1 = (m − 1)(n − 1) + 2(mn − 1) must be even, or equivalently at least one of m and n is odd. On the other hand, the game can be finished successfully if m or n is odd. Assume that m is odd. As shown in the picture, we can arrive at the position (1) in m 1 m−1 moves; with m+1 2 moves we reduce it to the position (1 2 ), and with the next 2 moves to the  position (2). We continue until we empty all the columns. • ◦ ◦ •◦ • •   ◦◦◦ •◦ •◦ •   ◦ ◦ ◦ • ◦ • •   •◦ •◦ • ◦ ◦ ◦ −→ −→ −→ m   ◦◦◦ •◦ • •   ◦◦◦ •◦ •◦ •    ◦ ◦ ◦ • ◦ • •  (0) (1) (1 12 ) (2) 650 4 Solutions 4.40 Solutions to the Shortlisted Problems of IMO 1999 1. Obviously (1, p) (where p is an arbitrary prime) and (2, 2) are solutions and the only solutions to the problem for x < 3 or p < 3. Let us now assume x, p ≥ 3. Since p is odd, (p − 1)x + 1 is odd, and hence x is odd. Let q be the smallest prime divisor of x, which also must be odd. We have q | x | x p−1 | (p − 1)x + 1 ⇒ (p − 1)x ≡ −1 (mod q). Also from Fermat’s little theorem (p − 1)q−1 ≡ 1 (mod q). Since q − 1 and x are coprime, there exist integers α , β such that xα = (q − 1)β + 1. We also note that α must be odd. We now have p − 1 ≡ (p − 1)(q−1)β +1 ≡ (p − 1)xα ≡ −1 (mod q) and hence q | p ⇒ q = p. Since x is odd, p | x, and x ≤ 2p, it follows x = p for all x, p ≥ 3. Thus       p p−1 p p−1 x 2 p−2 p | (p − 1) + 1 = p · p − p + ···− +1 . 1 p−2 Since the expression in parenthesis is not divisible by p, it follows that p p−1 |p2 and hence p ≤ 3. One can easily verify that (3, 3) is a valid solution. We have shown that the only solutions are (1, p), (2, 2), and (3, 3), where p is an arbitrary prime. 2. We first prove that every rational number in the interval (1, 2) can be represented 3 +b3 2 in the form aa3 +d 3 . Taking b, d such that b 6= d and a = b + d, we get a − ab + 2 2 2 b = a − ad + d and a3 + b3 (a + b)(a2 − ab + b2) a+b = = . 3 3 2 2 a +d (a + d)(a − ad + d ) a + d For a given rational number 1 < m/n < 2 we can select a = m +n and b = 2m −n a+b such that along with d = a − b we have a+d = mn . This completes the proof of the first statement. For m/n outside q of the interval we can easily select a rational number p/q such pn p p3 m 3 that m < q < 3 2n m . In other words 1 < q3 n < 2. We now proceed to obtain a, b and d for p3 m q3 n as before, and we finally have p3 m a3 + b3 m (aq)3 + (bq)3 = 3 ⇒ = . 3 3 q n a +d n (ap)3 + (d p)3 Thus we have shown that all positive rational numbers can be expressed in the 3 +b3 form ac3 +d 3. 3. We first prove the following lemma. Lemma. For d, c ∈ N and d 2 | c2 + 1 there exists b ∈ N such that d 2 (d 2 + 1) | b2 + 1. Proof. It is enough to set b = c + d 2 c − d 3 = c + d 2 (c − d). 4.40 Shortlisted Problems 1999 651 Using the lemma it suffices to find increasing sequences dn and cn such that cn − dn is an increasing sequence and dn2 | c2n + 1. We then obtain the desired sequences an and bn from an = dn2 and bn = cn + dn2 (cn − dn ). It is easy to check that dn = 22n + 1 and cn = 2ndn satisfy the required conditions. Hence we have demonstrated the existence of increasing sequences an and bn such that an (an + 1) | b2n + 1. Remark. There are many solutions to this problem. For example, it is sufficient to prove that the Pell-type equation 5an (an + 1) = b2n + 1 has an infinity of solutions in positive integers. Alternatively, one can show that an (an + 1) can be represented as a sum of two coprime squares for infinitely many an , which implies the existence of bn . 4. (a) The fundamental period of p is the smallest integer d(p) such that p | 10d(p) − 1. Let s be an arbitrary prime and set Ns = 102s + 10s + 1. In that case Ns ≡ 3 (mod 9). Let ps 6= 37 be a prime dividing Ns /3. Clearly ps 6= 3. We claim that such a prime exists and that 3 | d(ps ). The prime ps exists, since otherwise Ns could be written in the form Ns = 3 · 37k ≡ 3 (mod 4), while on the other hand for s > 1 we have Ns ≡ 1 (mod 4). Now we prove 3 | d(ps ). We have ps | Ns | 103s − 1 and hence d(ps ) | 3s. We cannot have d(ps ) | s, for otherwise ps | 10s − 1 ⇒ ps | (102s + 10s + 1, 10s − 1) = 3; and we cannot have d(ps ) | 3, for otherwise ps | 103 − 1 = 999 = 33 ·37, both of which contradict ps 6= 3, 37. It follows that d(ps ) = 3s. Hence for every prime s there exists a prime ps such that d(ps ) = 3s. It follows that the cardinality of S is infinite. (b) Let r = r(s) be the fundamental period of p ∈ S. Then p | 103r − 1, p ∤ 10r − j−1 1 ⇒ p | 102r + 10r + 1. Let x j = 10p and y j = {x j } = 0.a j a j+1 a j+2 . . . . Then a j < 10y j , and hence f (k, p) = ak + ak+r + ak+2r < 10(yk + yk+r + yk+2r ) . 10k−1 N p We note that xk + xk+s(p) + xk+2s(p) = is an integer, from which it p follows that yk + yk+s(p) + yk+2s(p) ∈ N. Hence yk + yk+s(p) + yk+2s(p) ≤ 2. It follows that f (k, p) < 20. We note that f (2, 7) = 4 + 8 + 7 = 19. Hence 19 is the greatest possible value of f (k, p). 5. Since one can arbitrarily add zeros at the end of m, which increases divisibility by 2 and 5 to an arbitrary exponent, it suffices to assume 2, 5 ∤ n. If (n, 10) = 1, there exists an integer w ≥ 2 such that 10w ≡ 1 (mod n). We also note that 10iw ≡ 1 (mod n) and 10 jw+1 ≡ 10 (mod n) for all integers i and j. Let us assume that m is of the form m = ∑ui=1 10iw + ∑vj=1 10 jw+1 for integers u, v ≥ 0 (where if u or v is 0, the corresponding sum is 0). Obviously, the sum of the digits of m is equal to u + v, and also m ≡ u + 10v (mod n). Hence our problem reduces to finding integers u, v ≥ 0 such that u + v = k and n | u + 10v = k + 9v. Since (n, 9) = 1, it follows that there exists some v0 such that 0 ≤ v0 < n ≤ k 652 4 Solutions and 9v0 ≡ −k (mod n) ⇒ n | k + 9v0 . Taking this v0 and setting u0 = k − v0 we obtain the desired parameters for defining m. 6. Let N be the smallest integer greater than M. We take the difference of the numbers in the progression to be of the form 10m + 1, m ∈ N. Hen